You are on page 1of 666

BOOKCHAPTER

AcidBaseBalance
BjornI.EngstromMD,ThomasA.BrownMDandSonaliJ.Shah
USMLEStep1Secrets,Chapter5,117133

AcidbasebalanceisafivestartopicfortheUSMLEStep1.Itislikelythatyouwillbeaskedto
diagnosevariousacidbasedisordersbasedonmedicalhistoryandlaboratoryvalues.Payclose
attentiontoallofthevariouscausesofmetabolicandrespiratoryacidosisandalkalosis,andbe
abletorecognizethemechanismsbywhichthebodycompensatesforthesedisorders.
Insider'sGuidetoAcidBasefortheUSMLEStep1

Basicconcepts
1 Howisextracellularhydrogenionconcentrationregulated?
Thehydrogenionconcentration[H+]intheextracellularfluidisverytightlycontrolledandis
regulatedbytherespiratorysystem,thekidneys,andvariousbuffers.Theverycloserelationship
betweenthe[H+],Pco2,and[HCO
+

(H

]canbeexpressedasfollows:

) = 25 (PCO 2 /[HCO3

])

Aprocessthatraises[H+]iscalledanacidosiswhereasaprocessthatlowers[H+]iscalledan
alkalosis.IftheseprocessesleadtoanalterationinbloodpHto<7.36or>7.44,thereismetabolic
derangementthatcanbedescribedasanacidemiaoranalkalemia,respectively.However,in
everydaylanguagethesetermsareusedratherloosely,andosisisoftenusedwhenemiaisreally
whatismeant.

2 HowdoesachangeinHCO

orinPco2affectpH?

AchangeinHCO of10mEq/LupordowncausespHtoincreaseordecreaseby0.15unit,
respectively.Acutely,achangeinPco2of10mmHgupordowncausespHtodecreaseorincrease

by0.08unit,respectively.AchronicchangeinPco2levelsaltersbloodpHby0.03unitper10mm
HgofPco2(duetorenalcompensationseeCase52(st0055)fordetails).

Thefirststepininterpretationistodeterminetheprimarydisorder.Figure51(f0010)canbeusedfor
thispurpose.Normalvaluesfor[HCO

]andPco2are22to28mEq/Land35to45mmHg,

respectively.

Case51
Anolderwomanhashaddiarrheafor2days.Sheistachypneiconexamination.Herlaboratory
valuesareasfollows:
pH:7.2
Pco2:19mmHg
[HCO

]:7mEq/L

[Cl]:120mEq/L
[Na+]:140mEq/L

Figure51
Determiningtheprimaryacidbaseabnormality.
(FromPicciniJP,NilssonKR:TheOslerMedicalHandbook,2nded.Baltimore,JohnsHopkinsUniversity,2006.)

1 Whatistheprimaryacidbasedisorder?
Thelaboratoryresultsrevealacidemia(morespecifically,metabolicacidosiswithnormalaniongap),
whichisconsistentwiththehistoryofdiarrheafor2days.
Note:Ifyouhadtroublearrivingatthisdiagnosis,wewillattempttowalkyouthroughithere.The
firstthingyoumustdoislookatthepHandrealizethatitisabnormallylow.Thisnumbertellsus
ourpatienthasanacidemia.Next,determinewhethertheacidemiaistheresultofrespiratoryor
metaboliccauses.BicarbonateandPco2arebothlow.Bicarbonateisabase,anddecreasedlevelsof
bicarbonatewillresultinmetabolicacidosis.CO2,ontheotherhand,isanacid.LowlevelsofCO2
thusresultinanalkalosis.Inthissituation,thebodytriestocorrectforthedecreasedpHof
metabolicacidosisthroughhyperventilationandlossofCO2.Therefore,thispatienthasametabolic
acidosiswithrespiratorycompensation.Incontrastwithmetaboliccompensationbythekidneys,
respiratorycompensationoccursimmediately.

Aniongap(AG)canbeassessedusingtheformula:
AG

= Unmeasured anions Unmeasured cations


= Measured cations Measured anions
+

= ((Na

+ K

)) [(Cl

+ HCO3

)]

Becausepotassiumconcentrationisgenerallysmallincomparisontotheotherelectrolytesinthe
formula,itisnormallydiscountedfromtheformula.Notethattheaniongapisnormallypositive
becauseunmeasuredanions(primarilyserumproteins)faroutnumberunmeasuredcations.
Therefore,AG = (Na ) [(Cl + HCO )] .Inthispatient,AGis13mEq/L(normalrange=8
16mEq/L).Inthenormalstate,theseunmeasuredanionsconsistprimarilyofalbuminand
phosphate.
+

Youmustknowallofthecausesofaniongapandnonaniongapmetabolicacidosis.These
causesarediscussedlaterinthechapter.
Step1Secret

2 Whatarethemechanismsinvolvedinmetabolicacidosisduetodiarrhea?
Diarrheaessentiallyrepresentsthelossofbicarbonaterichfluid.Intestinalfluidisfairlyrichin
bicarbonate,butnormallymuchofthisbicarbonateisreabsorbedviaaCl /HCO exchange
processinthecolon.Diarrheashortensintestinaltransittime,limitingtheopportunityforcolonic
Cl /HCO
exchangeandtherebyincreasingtheconcentrationofbicarbonateinthestool.This

lossofbicarbonateisthuselectricallybalancedbyanincreaseinserumClconcentration.As
mentionedintheprecedingnote,theAGremainsunchanged.AsimilarprocessofClretention
occursinallnonAGacidoses.ThisiswhyanonAGmetabolicacidosiscanalternativelybedescribed
asahyperchloremicmetabolicacidosis.

3 Whatelseisconsideredinthedifferentialdiagnosisfornonaniongap
metabolicacidosis?
NonAGmetabolicacidosescanberoughlydividedintorenalandnonrenalcauses.Inthesettingof
anacidemia,theappropriateresponseofthekidneyistoexcreteexcessacidasNH .ThisNH is
+

generallypairedwithCltomaintainelectricalneutrality.Assuch,theurineexcretionofClcanbe
measuredtodeterminerenalNH excretion.Thiscalculationisspecificallydoneusingthe
equationfortheurineaniongap(UAG):
+

UAG

= unmeasured anions unmeasured cations


= measured cations measured anions
= [(UNa + UK ) UCl ]

NotethaturineHCO isexcludedfromthisequationbecause,duetotheacidityofurine,its
concentrationistypicallynegligible.AnegativeUAGsignifiesanappropriaterenalresponsesince

makesupalargeportionoftheunmeasuredcationsintheequation,and,likewise,urineCl
concentrationwillbeincreasedduetopairingwithNH .Afailuretoexcreteammoniumduringan
acidemiawillgiveapositiveUAG.
NH 4

AnacidosisresultingfromsuchafailurebythekidneytoexcreteNH iscalledarenaltubular
acidosis(RTA).SeeTable51(t0010)foradescriptionofthethreetypesofrenaltubularacidoses.
+

Table51
ClassificationofRenalTubularAcidosis(RTA)

Type

Pathophysiology

Urine Degreeof Serum[K


pH
Acidosis +]

I(distalRTA)

InabilitytosecreteH+indistaltubule

>5.3

Severe
(serum
HCO3

%HCO3
Excretion
(After
Bicarbonate
Load)

Decreased <3%

often<10
mEq/L)
II(proximal

Deficitofcarbonicanhydraseand

RTA)

reabsorptioninproximal
tubule,canbegintoreabsorb
bicarbonateafterpHdecreasesbelow
athresholdlevel
Mayleadtoimpairedproximaltubular
reabsorptionofothernutrients(e.g.,
glucose,phosphate,aminoacids)
HCO3

<5.3

IV
Hypoaldosteronismoraldosterone
(hyperkalemic resistance
RTA)

Modest

Decreased >15%

(serum

HCO3

1216
mEq/L)

<5.3

Mild
(serum
HCO3

Increased

<3%

1420
mEq/L)
AnegativeUAGinthesettingofanonAGacidosismostoftenindicatesdiarrheaasthenonrenal
cause.However,othercausesincludeureteralcolonicfistulas,exogenousacidingestionor
administration(considerparenteralnutritioninahospitalizedpatient),posthypocapnicacidosis,
earlyrenalfailure,anddilutionalacidosis.Inthecaseofureteralcolonicfistulas,urinerichinCl

entersthecolon,wheretheClisreabsorbedinexchangeforHCO ,resultinginHCO lossina


mechanismsimilartodiarrhea.Posthypocapnicacidosisisaresultofpersistingrenalcompensation
toachronicrespiratoryalkalosis(seeCase52(st0055),question2).Acidosisinearlyrenalfailure
canresultfromlossoftheabilitytogenerateammonialaterenalfailure,incontrast,generally
resultsinamixedAG/nonAGacidosis(seenextquestion).Dilutionalacidosis,whichiscommonly
seeninhospitalizedpatients,isaresultofexcessnormalsalineadministrationnormalsaline

representsalargeClload(specifically,154mEq/LofCl),whichresultsinincreasedbicarbonate
excretiontomaintainelectricalneutrality.

4 Isthereappropriatecompensationoristhereamixeddisorderinthispatient?
AnappropriatecompensationformetabolicacidosisistodecreasePco2,thatis,excretevolatile
acid,throughhyperventilation.Suchrespiratorycompensationisvirtuallyimmediate.Usingthe
equationinTable52(t0015),theexpectedchangeinPco2=1.2(257)=21.6.Theactualchange
inthiscaseis(4019)=21,reflectingappropriatecompensation.
Table52
RespiratoryCompensationforMetabolicAcidBaseDisturbances

Condition

PrimaryChange

ExpectedCompensation

Metabolicacidosis DecreasedHCO3
Metabolicalkalosis IncreasedHCO3

DecreasedPCO2
IncreasedPCO2

= 1.2 HCO3

= 0.7 HCO3

InlookingatTable52(t0015),itisapparentthatthelungs,inrespondingtometabolicacidbase
disturbances,canmorereadilyexcreteCO2thanretainit.Thisresponseisbothratherintuitive
(severehypercapniacanbequitedangerous,rapidlyleadingtomentalstatuschangesandcoma)and
usefulinrememberingthatthedegreeofrespiratorycompensationformetabolicacidosisisgreater
thanthecompensation.Note:Compensationduringanyacidbasedisturbanceisnevercomplete.

5 Whatwoulditmeanif,inthesamepatient,appropriatecompensationwasnot
presentandPco2wasinstead30mmHg?
Thisvaluewouldmeanthatthereisamixeddisorder,thatis,morethanoneprimarydisorder
occurringatthesametime.APco2of30mmHginthiscasewouldrepresentinappropriate
retentionofCO2,indicatingarespiratoryacidosisinadditiontothemetabolicacidosis.

FirstdeterminetheprimarydisturbanceusingpH,Pco2,andHCO

Thendetermineifthedegreeofcompensationisappropriateorifthereisamixed
disorder.

ItiseasierforthebodytoblowoffCO2thantoretainit,thatis,greatercompensation
formetabolicacidosesthanformetabolicalkaloses.
Forametabolicacidosis,firstdeterminetheaniongap(AG).
AG = (Na ) [(Cl + HCO
normalaniongapis~10.
+

)]

)=unmeasuredanionsunmeasuredcations

IfanonAGmetabolicacidosisexists,searchforcluesinthehistory(e.g.,diarrhea,rapid
infusionofnormalsaline)anddeterminetheurineaniongap(UAG).
UAG=(UNa+UK)UCl=unmeasuredanionsunmeasuredcations.
IftheUAGisnegative,thekidneysareworkingnormally(i.e.,areexcretingNH4Cl).
IftheUAGispositive,renaltubularacidosis(RTA)islikelylookaturinepH,serumpH,
serumpotassium,andthefractionalexcretionofHCO afterabicarbonateloadto
determinewhichtype.

SummaryBox:AcidBaseDisturbances,MetabolicAcidosis

Case52
Youarecalledtoconsultonanintensivecareunitpatientwhohasdevelopedsuddenonset
tachypneaandtachycardia.Hislaboratoryvaluesfollow:
pH:7.5
Pco2:20mmHg
[HCO3]:20mEq/L

1 Whatistheprimaryacidbasedisorder?
FollowingFigure51(f0010),weseethattheprimarydisorderisarespiratoryalkalosis.

2 Whatisthedifferentialdiagnosisinthispatient?
Intensivecareunit(ICU)patientsareatincreasedriskforpulmonaryembolism(PE)becausethey
areimmobileandmoreoftenhaveseriousdiseasessuchascongestiveheartfailure(stasis),recent
surgery(injurytoendothelium),orcancer(hypercoagulablestate).Therefore,aPEisthemostlikely
causeofhiscondition(particularlygiventhesuddenonsetofhisvitalsignschanges).Pleuriticchest
pain,mildfever,orhemoptysiswouldalsosupportadiagnosisofPE,butsignsandsymptomsofPE
inanICUpatientareoftenmuchmoresubtle.

HypoxiafromaPEleadstohyperventilationandrespiratoryalkalosis.However,hypoxiaand
hyperventilationcanoccurinvirtuallyanyformoflungdisease,includingpneumonia,pulmonary
edema(ascausedbycongestiveheartfailure[CHF]oracuterespiratorydistresssyndrome[ARDS]),
orrestrictivelungdisease.Pain,anxiety,andvariouscentralnervoussystem(CNS)disordersare
commoncausesofrespiratoryalkalosisintheICUthatoccurintheabsenceofhypoxia.Asthma
patientsareinterestingbecausealthoughtheycandevelophypoxia,thesepatientsverydramatically
hyperventilateduringasthmaattackssuchthatadegreeofrespiratoryalkalosisdevelopsthatisout
ofproportiontothehypoxia.
Itisimportanttonotethatifhyperventilationpersists,respiratorymusclefatiguecanarise,leading
toCO2accumulationandrespiratoryacidosis(aprocessreferredtoashypercapnicrespiratory
failure).
Step1lovestoaskquestionsaboutpulmonaryembolism(PE)becauseitissuchacommon
diagnosis.ManystudentsaretrickedbythefactthatPEmorecommonlyproducesrespiratory
alkalosisratherthanrespiratoryacidosis.RememberthatPEpatientsaretachypneic
hyperventilationleadstoexcessremovalofCO2fromthebloodstream.
Step1Secret

3 Isthereappropriatecompensationoristhisamixeddisorder?
Becausethefulleffectofrenalcompensationforrespiratorydisturbancesisnotimmediate,foracid
basedisturbanceswitharespiratorycause,onemustfirstdetermineifthedisturbanceisacuteor
chronic.Inthispatient,therespiratoryalkalosisisacute.Theappropriatecompensationforany
respiratoryalkalosisistodecreaseserumHCO throughincreasedrenalexcretion.Usingthe

equationinTable53(t0020),theappropriateacutechangein
HCO
= 0.2 PCO = 0.2 20 = 5 .Theactualchangeinthiscaseis2520=5therefore,

thereisappropriateacutecompensation.
ItisnotlikelythattheUSMLEwillaskyoutocalculatethedegreeofrenalorpulmonary
compensationforanacuteorchronicacidbasedisorderusingtheformulaslistedinTables52
(t0015)and53(t0020).However,theseareimportantformulastoknowforyourclinicalyears,

particularlybecausetheycanhelpyouspotmixeddisorders.
Step1Secret
Table53
RenalCompensationforRespiratoryAcidBaseDisturbances

Condition
Acuterespiratoryacidosis

PrimaryChange

ExpectedCompensation

IncreasedPco2

IncreasedHCO3

= 0.1

Chronicrespiratoryacidosis IncreasedPco2

IncreasedHCO3

= 0.35

Acuterespiratoryalkalosis

Pco2
Pco2

DecreasedPco2 DecreasedHCO3

= 0.2

Pco2

Chronicrespiratoryalkalosis DecreasedPco2 DecreasedHCO3

= 0.5

Pco2

4 Doesthedegreeofcompensationallowyoutodrawanyconclusionsastothe
durationofthecondition?
Yes.Evenifthehistoryofanacuteprocesswasnotavailableinthispatient,wecouldconcludefrom
hislaboratoryvaluesthathisrespiratoryalkalosisisacute.Ifthiswereamorechronicpresentation,
asoccursathighaltitudeorinpregnantwomen(progesteroneinducedincreaseintidalvolume),the
kidneyswouldhaverespondedbyexcretingmorebicarbonate.ThedecreaseinHCO wouldhave

been0.5Pco2=10.Becauseittakesacoupleofdaysforthekidneystofullyadjusttoan
alkalosis,weknowthatthedurationofthisconditionismuchshorter.
Ifyouareattemptingtoremembertheprecedingnumbers,itisusefultorealizethatthekidneys
moreeasilyandmorerapidlyexcreteHCO thanretainHCO whenrespondingtorespiratory

acidbasedisturbances.Thus,renalcompensationforrespiratoryalkalosesisbothmorecomplete
andmorerapidthanthecompensationforrespiratoryacidoses(seeTable53(t0020)andFig.52
(f0015)).

Arespiratoryalkalosisiseitherduetoprimaryhyperventilation(pain,anxiety,central
nervoussystemdisease)orinducedbyhypoxemia(variousformsoflungdisease),orboth
(acuteasthmaattack).
Maximalcompensationbythekidneystakesacoupleofdays.
Itiseasierforthekidneystoexcretebicarbonatethantoretainitwhenarespiratory
acidbasedisturbanceexists,resultingingreatercompensationforrespiratoryalkaloses
thanforrespiratoryacidoses.
SummaryBox:RespiratoryAlkalosis

Case53
A41yearoldwomancomesinforapreemploymentphysicalexaminationandisfoundto
havemildtomoderatehypertension,withthefollowinglaboratoryvalues:
pH:7.55
[HCO

]:35mEq/L

Pco2:19mmHg

Figure52
Timecourseofacidbasecompensatorymechanisms.Inresponsetoametabolicacidoralkalineload,individual
approachestocompletionofdistributionandextracellularbufferingmechanisms,cellularbufferingevents,and
respiratoryandrenalregulatoryprocessesarepresentedasafunctionoftime.ECF,extracellularfluid.
(FromBrennerBM:BrennerandRector'sTheKidney,7thed.Philadelphia,WBSaunders,2004.)

1 Whatistheprimaryacidbasedisorder?
Followingthealgorithmgivenearlier,weseethattheprimarydisorderisconsistentwithametabolic
alkalosis.

2 Whatisthedifferentialdiagnosisinthispatient?
Lossofgastricsecretions(i.e.,vomiting),diuretics,volumedepletion,hypokalemia,
mineralocorticoidexcess,Cushingdisease,excessivelicoriceconsumption,andsomerarediseases
includingBartter,Gitelman,andLiddlesyndromesand11hydroxysteroiddehydrogenase(11
HSD)deficiency.PatientswhoareintheICUandwhohaveachronicrespiratoryacidosiscorrected
tooquicklycanalsodevelopasimilarpicture,becausetheirkidneys,whichhavebecomeusedto
excretingalargeacidloadtocompensate,willtakeseveraldaystoreadjust.

3 Whatisthepathophysiologyofeachoftheconditionsinthedifferential
diagnosis?
Volumedepletion(contractionalkalosis):
Sodiumandbicarbonatereabsorptionaredirectlylinkedintheproximaltubule.
VolumedepletionresultsinanattempttoretainNa+andtherebyleadstoincreased
bicarbonatereabsorption.
Volumedepletionstimulatesthereninangiotensinsystem.Theincreasedaldosterone

promotesNa+reabsorptioninexchangeforK+andH+inthedistaltubule.
Hypokalemia:
K+andH+competeforsecretioninthedistaltubulehypokalemiastimulatesK+
retention,resultinginincreasedH+secretion.
HypokalemiaresultsintranscellularshiftofK+outofcells,whichiselectrically
balanced(inpart)byashiftofH+intocells,directlylowingserumH+.
ThetranscellularshiftofH+alsoresultsinintracellularacidification,which,atthe
proximaltubule(wherethenephron'spHsensorislocated),stimulatesincreased
ammoniaproduction,ultimatelyresultinginincreasedexcretionofH+intheformof
NH
.
+

Lossofgastricsecretions:
LossofH+willdirectlycauseadecreaseinHCO

andanincreaseinpH.

LossofchlorideionswillinhibitHCO secretioninthedistaltubuleandcollecting
duct(becauseofdecreasedCl /HCO exchangebyintercalatedcells).

Seemechanismsinvolvedinvolumedepletion,asdescribedearlierinlist.
Nowondervomitingandnasogastrictube(NGT)suctioningaresuchpowerful
producersofmetabolicalkalosis!
Diuretics(loopdiureticsandthiazidesonly!)Rememberthatacetazolamideand,toalesser
extent,K+sparingdiureticstendtocauseanacidosis:
Volumedepletion
Hypokalemia
Mineralocorticoidexcess:Theactionofaldosteroneinthedistaltubuleresultsinanincrease
inH+(andK+)secretioninexchangeforNa+.
Cushingdisease:Anexcessofcorticosteroidswillactivatethemineralocorticoidreceptor.
Barttersyndrome:DefectiveNa+/K+/CltransporterintheloopofHenlemimicstheeffect
ofaloopdiuretic!
Gitelmansyndrome:DefectiveNa+/Cltransportermimicstheeffectofathiazidediuretic!
Liddlesyndrome:AdefectintheNa+channelinthedistaltubulethatisnormallystimulated
byaldosteroneresultsinthechannelbeingpermanentlyactivated(mimicsmineralocorticoid

excessinthissettingspironolactonewillnotwork,soonemustusetriamtereneoramiloride
instead).
11HSDdeficiency:Defectoftheenzymethatnormallybreaksdowncortisolintocortisone
withinaldosteroneresponsivecellscausescortisoltobuildupandactivatethe
mineralocorticoidreceptor.
Licorice:Reallicorice(blacklicorice,notTwizzler'sredcandy)containsasubstancethat
inhibits11HSD.

4 Isthereappropriatecompensationoristhereamixeddisorder?
No,thereisnotappropriatecompensation.Theexpectedcompensationforametabolicalkalosis
wouldbeanincreaseinPco2of0.7 HCO

= 0.7 10 = 7

mmHg.Therefore,amixed

disorderispresent.

Case53continued:
Whenthepatientisfurtherquestionedatthefollowupappointment,shealsoadmitstohaving
polyuria,polydipsia,muscleweakness,andheadaches.Shehasthefollowinglaboratoryvalues:
[K+]:3.1mEq/L
[Na+]:149mEq/L
Aldosterone/reninratio:>20

5 Whatisthemostlikelydiagnosis?
Thepresentationandlaboratoryvaluesareclassicforprimaryhyperaldosteronism(Conn
syndrome).Connsyndromeshouldbesuspectedinanypatientwithhypertensionandhypokalemia.
Thispatient'shypokalemiaandhypernatremiaand,toalesserextent,hersignsandsymptomscan
beseeninallsyndromesofhighaldosteroneactivity,includingsecondaryhyperaldosteronism
(reninsecretingtumor,CHF,renovasculardisease)andnonaldosteronemineralocorticoidexcess
(Cushingsyndrome,Liddlesyndrome,11HSDdeficiency,licoriceingestion,exogenous
mineralocorticoids).However,itisonlyinprimaryhyperaldosteronismthatthealdosteronerenin
ratiois>20.Forsecondaryhyperaldosteronismbothreninandaldosteroneareincreased,butin
nonaldosteronemineralocorticoidexcesstheyarebothdecreased.

6 Wouldadministrationofsalinebehelpfulinthispatient?
No,itwouldnot.Thecausesofmetabolicalkalosiscanbedividedintosalineresponsiveandsaline
resistantcategories(Fig.53(f0020)).Evenwithoutknowingthediagnosis,itispossibletofigureout
ifasalineinfusionwillhelpbylookingattheurinechloridelevel(UCl).AUCl<10mEq/Lindicates

salineinfusionwouldnotbehelpful,whereasaUCl>20mEq/Lindicatessalineinfusionwouldbe
helpful.TherangeofUCl10to20mEq/Lisagrayerareabutmostoftenindicatesasalineresistant
process.

Figure53
Differentialdiagnosisofmetabolicalkalosis.
(FromPicciniJP,NilssonKR:TheOslerMedicalHandbook,2nded.Baltimore,JohnsHopkinsUniversity,2006.)

Thispatientneedsanadrenalcomputedtomography(CT)scanormagneticresonanceimaging
(MRI)todetermineiftheprimaryhyperaldosteronismisduetoadrenalhyperplasiaortoan
adenomaorcarcinoma.Spironolactoneisthetreatmentforhyperplasia,whereassurgeryisthe
treatmentforadenomaorcarcinoma.
Donotworryaboutgettingtoocaughtupinthedetailsofthisquestion.Thistopicisbeyondthe
scopeofStep1andissimplyincludedtoexpandyourknowledgeinpreparationforyourclinical
years.

Themostcommoncausesofmetabolicalkalosisarelossofgastricsecretions(vomiting
ornasogastricsuctioning),diuretics(loopsandthiazides),volumedepletion,and
mineralocorticoidexcess.
Lossofgastriccontentsisaverypowerfulstimulantofmetabolicalkalosisbecauseit
worksthroughthreeseparatebutadditivemechanisms:directlossofH+,lossofCl,
andvolumedepletion.
SummaryBox:MetabolicAlkalosis

Case54
An18yearoldmanpresentswith2daysoffatigue,abdominalpain,andvomiting.
Examinationshowstachycardiawithpulserateof120beats/min.
pH:7.15

[Na+]:140mEq/L
[Cl]:90mEq/L
[HCO

]:22mEq/L

Pco2:36mmHg
[K]:5.2mEq/L
BUN:52mg/dL

1 Whatistheprimaryacidbasedisorder?
Thislookslikeaprimarymetabolicacidosis(pHislessthan7.4andbicarbonateislessthan25).

2 Isthereappropriatecompensationoristhisamixeddisorder?
ThePco2isdecreasedby4,whichisconsistentwithappropriaterespiratorycompensation(
1.2 HCO3

= 1.2 3 = 3.6

).

3 Whatisthenextstepindiagnosisofthisdisorder?
Calculatetheaniongap!ItiscriticaltodistinguishbetweenAGacidosisandnonAGacidosisbecause
thedifferentialsforeacharedrasticallydifferent.
Remember,AG

= Na (Cl

+ HCO3

.Inthispatient,AG=140(90+22)=28mEq/L.

Youwillbeexpected(butnotnecessarilyinstructed)tocalculateaniongaponStep1ifthe
diagnosisinvolvesmetabolicacidosis.Beonthelookoutforthistypeofquestion!
Step1Secret

4 HowcantherebesuchalargeaniongapwithsuchanextremelylowpHwhen
thedisturbanceinHCO issominimal?

Thiseffectiscausedbyasuperimposedmetabolicalkalosis!Forevery1mEqofanioncontributingto
thechangingAG,[HCO ]shouldfallby1mEqfromitsnormalvalue.Therefore,thechangein

AGshouldequalthechangeinbicarbonateconcentrationiftheAGacidosisistheonlymetabolic
disturbancepresent.However,ifadiscrepancyexists,morethanonemetabolicprocessispresent:
theAGacidosisandeitherasecond,ahyperchloremic,metabolicacidosisorametabolicalkalosis.
Thisideacanbesummarizedthroughtheconceptofthedeltadelta.Thedeltadeltasimply
representsthecomparisonoftheAGandtheHCO
4(f0025)):

.Itshouldbeinterpretedasfollows(Fig.5

IfAG

= HCO3

IfAG

< HCO3

thehighAGacidosisisthesolemetabolicacidbasedisorder.

thereisahyperchloremicmetabolicacidosisinadditiontothehighAG

acidosis(sincethechangeinbicarbonateisgreaterthanexpected).
IfAG > HCO thereisametabolicalkalosisinadditiontothehighAGacidosis(since
thechangeinbicarbonateissmallerthanexpected).

Inthiscase,theAG=2812=16mEq/L.TheHCO
AG > HCO3

is2412=2mEq/L.Thus,

,indicatingahiddenmetabolicalkalosisaswesuspected.

Case54continued:
Thehistoryrevealsthatthepatienthaspolyuria,polydipsia,andanodorofacetoneonhis
breath.

Figure54
Diagnosticalgorithmformetabolicalkalosis.Thealgorithmisbasedontheurinechlorideconcentration.HTN,
hypertensionJGA,juxtaglomerularapparatus.
(FromBrennerBM:BrennerandRector'sTheKidney,7thed.Philadelphia,WBSaunders,2004.)

5 Whatisthemostlikelydiagnosisinthispatient?

Thispatientmostlikelyhasdiabeticketoacidosis(DKA)!DKAisaboardfavorite,soyouneedto
understanditwell.Thisconditionoccursmainlyintype1diabetics(althoughrarelyitcanoccurin
type2diabeticsaswell).Itisnotuncommonforan18yearoldmantopresentinDKAwithout
havingpreviouslybeendiagnosedwithtype1diabetesinfact,about20%ofpatientspresentwithno
previoushistoryofdiabetes.Theclinicalmanifestationsconsistofpolyuriaandpolydipsia(resultof
hyperglycemiainducedosmoticdiuresis)nausea,vomiting,abdominalpain,andileus(dueto
effectsofhyperglycemiaandelectrolytedisturbanceonthegastrointestinal[GI]tract)Kussmaul's
(deep)respirations(tocompensateforthemetabolicacidosis)odorofalcoholonthebreath
(acetone,avolatileketonebody,beingexpiredinthelungs)signsofvolumedepletion,suchas
tachycardiaandhypotension(resultofdiuresisandvomiting)andchangesinmentalstatus(from
acidosisandelectrolytedisturbances).
ThelaboratoryfindingsinatypicalcaseincludeAGacidosis(fromtheketoacids,suchas
hydroxybutyrate[HB],pseudohyponatremia(duetohyperglycemia),increasedbloodurea
nitrogen(volumedepletion),increasedserumglucose,serumandurineketones(measuredasserum
HBoronurinedip),andleukocytosis(whichsometimesreflectsaninfectionthattriggeredthe
episode).
Totalbodypotassiumisusuallyreduced(asaresultoftheosmoticdiuresisandvolumedepletion).
However,serumK+levelsareusuallynormalorevenelevatedatpresentationduetotranscellular
shiftofK+outofcells(causedbyboththelackofinsulinandtheacidosis).SerumK+mustbe
followedcloselybecauseitcandropprecipitouslyoncetreatment(withintravenousfluidsand
insulin)isinitiated.
Inthiscase,thesuperimposedmetabolicalkalosisisduetothevomitingandvolumecontraction.
Rememberthatvomitingridsthebodyofgastricacid,thusincreasingpH.Metabolicalkalosiscanbe
especiallydangerousbecausethebody'scompensatoryresponseinvolvesrespiratoryacidosis,which
potentiallyresultsinhypoxemia(recallthatincreasedPco2resultsinanobligatorydecreaseinPo2
).
AlthoughDKAoftenoccurswhenatype1diabeticfailstotakeinsulin,itisalsoquiteoften
precipitatedbyatriggeringillness.Infections,suchaspneumonia,urinarytractinfections(UTIs),
andskinorsofttissueinfections,areparticularlycommon.Otherseriousillnesses,suchas
myocardialinfarction(MI)orpancreatitis,canserveastriggersaswell.
ThehyperglycemiaseeninDKAisduetoincreasedgluconeogenesisandincreasedglycogenolysisby
theliverinthesettingofdecreasedconsumptionofglucosebytheperipheraltissues,allaresultof
insufficientinsulinrelativetoglucagonandtheothercounterregulatoryhormones.Theketosisalso
arisesfromtheinabilityoftheperipheraltissuestoutilizeglucose.Thelackofavailableglucose
stimulatesthereleaseoffreefattyacidsbyperipheraladiposetissuesintothebloodstream.Thefatty
acidsarethenconvertedinthelivertotheketonesHBandacetoacetatesothattheycanusedby
vitalorgans(suchastheCNSandheart),whichnormallydependonglucoseforenergy.

6 Whatisonthedifferentialdiagnosisforlargeaniongapmetabolicacidosis?
TheclassicmnemonicisMUDPILES:MethanolandMetformin,Uremia(renalfailure)Diabetic
andotherketoacidoses,PhenforminandParaldehyde(rarelyseenanymore),IsoniazidandIron
supplements,Lacticacidosis,EthyleneglycolandEthanol,Salicylates.
Ineachcase,anunmeasuredorganicacidoranionaccumulatestoproducetheAG.Methanoland
ethyleneglycolaremetabolizedtoformicandoxalicacid,respectively.TheAGseeninrenalfailure
resultsfromtheaccumulationofphosphates,sulfates,andotherorganicanionsnormallyexcretedby
healthykidneys.Salicylatesincludeaspirinandotherderivatesofsalicylicacid.Theantituberculosis
drugisoniazidisalsoaderivativeofanorganicacid(isonicotinicacid).
Itisimportanttoremembertherearemanycausesoflacticacidosis.Sepsis,seizures(withincreased
muscleactivityorimpairedbreathing),ischemiaoflimbsororgans(especiallysmallbowel
ischemia),cyanideorcarbonmonoxidepoisoning,andcirculatoryorrespiratoryfailurecanallcause
lacticacidosis.Liverfailureresultsinimpairedclearanceoflacticacid.
Inaddition,recallthatketosiscanalsoresultfromstarvationandchronicalcoholism,albeitmuch
lesscommonlyandusuallywithamuchlesssevereacidosis.Inchronicalcoholicsthisismostoften
seenafewdaysafterheavybingedrinkinginthesettingofpoorfoodintaketherearenumerous
mechanismsinvolved,includingdepletionofNAD+byhepaticoxidationofalcohol,reducednutrient
intake,anddehydration(whichdecreasesurinaryketoneexcretion).
TheMUDPILESmnemonicisamustknowforStep1.Amongthesecommoncausesof
metabolicacidosis,diabeticketoacidosisandlacticacidosis(causedbyischemia,diabetic
drugs,etc.)areUSMLEfavorites.
Step1Secret

7 Whatisthecorrecttreatment?
Insulin,aggressiverehydrationstartingwithnormalsaline,potassiumrepletion(oncetheK+drops
downtohighnormallevels),andtreatmentofpossibleprecipitantssuchasinfections.Treatmentof
theacidosiswithbicarbonateiscontroversial(asthereisnodocumentedimprovementinoutcome)
andisgenerallydoneonlyinseverecaseswithapH<7.0.

Case55
A38yearoldhomelessmansufferingfromchronicalcoholismcomesintotheemergency
departmentwithnausea,vomiting,andblurryvision.Interestingly,hedeniesanyrecent
ingestionofalcohol,sayingthatheranoutofbooze.Hisbloodalcohollevelisundetectable.
Hislaboratoryvaluesareasfollows:
pH:7.29

[Na+]:135mEq/L
[Cl]:100mEq/L
[HCO

]:14mEq/L

[Glucose]:90mg/dL

1 Whatisthemostlikelydiagnosis?
Thismanhasmethanolintoxication.TheAGis21mEq/L.VisualimpairmentinthesettingofanAG
acidosisishighlysuggestiveofmethanolpoisoning(seeMUDPILESmnemonic).Methanolisfound
inwindshieldwasherfluidandisthereforeaneasilyaccessiblealcohol.Itisalsofoundinshellacand
varnish.Methanol,likeethanol,ismetabolizedbytheenzymealcoholdehydrogenase.Theformic
acidthatresultsisamitochondrialtoxinthatactsbyinhibitingcytochromeoxidaseofoxidative
phosphorylation.Theretina,opticnerve,andbasalgangliaareespeciallyvulnerable.Acomponentof
lacticacidosisoftencontributestotheAGduetotheimpairmentofoxidativephosphorylation
(similartocyanidepoisoning)andduetotheoverconversionofnicotinamideadeninedinucleotide
(NAD)toitsreducedform,NADH,intheliver(similartoethanolpoisoning).Inthefirst6hoursof
ingestion,thesignsandsymptomsofmethanolintoxicationresemblethoseofethanolintoxication.
Followingthisperiod,visualdisturbancesanddepressedconsciousnessthenbecomeprominent.
Examinationmightrevealpapilledema.Blurryvision(inthesettingofelevatedAGacidosis)isthe
buzzwordtorecognizeontheboards.

Case55continued:
Nowassumethatthishomelessmancomesintotheemergencydepartmentmerely6weeks
laterwithapparentinebriationandsignsofpulmonaryedemaandcardiovascularcollapseon
examination.Againhedeniesalcoholconsumption!Inaddition,heisfoundtohavecalcium
oxalatestonesonurinemicroscopy.HisurinealsofluoresceswhenunderaWood'slamp.

2 Nowwhatisthemostlikelydiagnosis?
Nowthismanhasethyleneglycolintoxication.Ethyleneglycolcanbefoundinantifreezeandde
icingsolutions.AntifreezeintheUnitedStatescommonlyhasanadditivethatproducestheWood's
lampfluorescence.Alcoholdehydrogenaseisagainthemetabolizingenzyme,producingoxalicacid.
Oxalicacidcancombinewithcalciumtoproducecrystalsintheurineseenonmicroscopic
examinationifextensivecrystalprecipitationoccurswithinthekidneys,acuterenalfailurecould
result.Again,acomponentoflacticacidosiscanresultfromtheoverconversionofNAD+toNADH,
resultingfromoxidationofthealcohol.Inseverecases,renalfailureoccurs,alongwithdepressed
consciousness,coma,andcardiopulmonarycollapse.

3 Whatisthetreatmentforthemaninquestions1and2?

Fomepizoleinhibitsalcoholdehydrogenaseandisthereforeveryeffectiveinbothmethanoland
ethyleneglycolintoxicationbecauseitcanpreventthebuildupofthetoxicmetabolitesformicand
oxalicacids,respectively.Intravenousethanolhasalsobeenusedeffectivelyinthepast,ashigh
ethanollevelscancompetitivelyinhibitthemetabolismofmethanolorethyleneglycolbyalcohol
dehydrogenase,butfomepizoledoesnothavethesideeffectsofethanol(anditsuseinclinical
practiceismuchlesscontroversial).Inseverecases,suchasthoseresultinginsevereacidemia,
severeendorgandamage,orvisualdisturbances(frommethanolingestion),hemodialysisis
indicated,asiteffectivelyremovesmethanol,ethyleneglycol,andtheiracidmetabolites.
Asmentionedthroughoutthisbook,drugsideeffectsandtoxicitiesareextremelyhighyieldfor
Step1.Knowingantidotestocommondrugoverdoses(e.g.,fomepizoleformethanol,ethanol,
andethyleneglycolpoisoning)isaneasywaytoearnpointsontheUSMLE.
Step1Secret

Case56
Apatientwithahistoryofdepressionisbroughtintotheemergencydepartmentwithnausea,
vomiting,andtinnitus.Herlaboratoryvaluesareasfollows:
[Na+]:140mEq/L
[K+]:3.5mEq/L
[Cl]:104mEq/L
[HCO

]:16mEq/L

[glucose]:100mg/dL
pH:7.50
Pco2:20mmHg
Po2:125mmHg

1 Whatisthemostlikelydiagnosis?
Thispatienthasamixeddisorderconsistingofarespiratoryalkalosis,alargeAGacidosis,anda
nearnormalpH.TheAGinthiscaseis20mEq/L.RememberthatanelevatedAG,eveninthe
settingofanormalpHoranalkalemia,alwaysreflectsametabolicacidosis.Forthatreason,itis
oftenworthwhiletocalculatetheAGinacidbaseproblemsregardlessofthepH(particularlyifthe
questionmakeslittlesenseotherwise!).

Theprototypicalmixeddisorderthatappearsontheboardsisaspirinintoxication.Inthiscase,the
patient'shistoryofdepressionsuggestsariskofsuicidalingestion,whilethetinnitusinparticularis
amajorclueforsalicylatepoisoning.However,ontheboardsyoushouldalsoconsideraspirin
ingestioninapatientwiththecombinationofanAGacidosisandrespiratoryalkalosis(Note:Initial
aspirinoverdosedirectlyactivatestherespiratorycenterinthemedulla,resultinginhyperventilation
andarespiratoryalkalosis.)
Althoughaspirinisitselfanacid,mostoftheAGproducedinaspirinpoisoningoccursfromlactic
acidbuildupthatresultsfromaspirin'sabilityattoxiclevelstointerferewithcellularmetabolism.
SideeffectsofaspirincanberecalledusingthemnemonicASPIRIN:Asthma,Salicylism,Peptic
ulcers,IntestinalBleeding,Reyesyndrome,Idiosyncraticreactions,andNoise(tinnitus).

Forcausesofaniongap(AG)acidosis,rememberMUDPILES:Methanol,M
etformin,Uremia(renalfailure),Diabeticandotherketoacidoses,Paraldehyde,P
henformin,Isoniazid,Ironsupplements,Lacticacidosis,Ethyleneglycol,Ethanol,S
alicylates.
IfAG

= HCO3

IfAG

< HCO3

thehighAGacidosisisthesolemetabolicacidbasedisorder.

thereisahyperchloremicmetabolicacidosisinadditiontothe

highAGacidosis(becausethechangeinbicarbonateisgreaterthanexpected).
IfAG

> HCO3

thereisametabolicalkalosisacidosisinadditiontothehighAG

acidosis(becausethechangeinbicarbonateissmallerthanexpected).
Diabeticketoacidosis(DKA)couldbethepresentationforpreviouslyundiagnosedtype1
diabetes(knowthepresentationofDKAwell!).
AGacidosis+blurryvision:Thinkmethanolingestion.
AGacidosis+urinewithoxalatecrystalsand/orWood'slampfluorescence:Think
ethyleneglycolingestion.
AGacidosis+respiratoryalkalosistinnitus:Thinkaspirinintoxication.
Rememberallthecausesoflacticacidosis:sepsis,circulatoryorrespiratoryfailure,limb
ororgan(especiallysmallbowel)ischemia,cyanideorcarbonmonoxide,hepaticfailure.
SummaryBox:AnionGapMetabolicAcidosis

Case57
TheemergencydepartmentadmitsaCHFpatientpresentinginsevererespiratorydistress.His
laboratoryvaluesareasfollows:

pH:7.0
Pco2:60mmHg
[HCO

]:28mEq/L

1 Whatistheprimaryacidbasedisorder?
Respiratoryacidosis(Pco2>40)isthedisorder.

2 Whatisthedifferentialdiagnosisinthispatient?
ThedifferentialdiagnosisofCO2retentionisvastandincludesnotonlylungdiseasebutcentral
hypoventilationfromanycause(sedatives,CNStrauma,pickwickiansyndrome),neuromuscular
disorders(e.g.,myastheniagravis,GuillainBarrsyndrome,amyotrophiclateralsclerosis[ALS],
musculardystrophy,poliomyelitis),upperairwayobstruction(acuteairwayobstruction,
laryngospasm,obstructivesleepapnea),andthoraciccageabnormalities(pneumothorax,flailchest,
scoliosis).
Note:Lungdiseasegenerallyimpairsgasexchangethroughdeadspace(lungthatisventilatedbut
notperfused)orshunting(lungthatisperfusedbutnotventilated)or,mostcommonly,a
combinationofboth(i.e.,ventilationperfusionmismatch).Disorderswithprominentdeadspace
ventilation(suchasemphysema)tendtocauseprominentandearlyCO2retention.Disorderswith
prominentintrapulmonaryshunting(suchasasthma,pulmonaryedema,pneumonia,atelectasis,or
PE)tendtocauseprominenthypoxiabutcanresulthypercapnialateraswell.

3 Whatisthemostlikelydiagnosisinthispatient?
Fromthelimitedamountofinformationwearegiven,acutepulmonaryedemaduetoCHFisthe
mostlikelyexplanation.Anintrapulmonaryshuntiscreatedwhenalveoliarefilledwithfluid,suchas
inpneumonia,pulmonaryedema(fromARDSorCHF),oratelectasis.Intrapulmonaryshunts
generallyleadtohypoxemia,whichofteninturnstimulateshyperventilationandadecreased(or
normal)Pco2.However,whentheshuntfractionisverylargeorwhentheincreased
hyperventilationandincreasedworkofbreathingleadtomusclefatigue,respiratoryacidosisisthe
result.

4 Isthereappropriatecompensationoristhisamixeddisorder?
TheincreaseinHCO

is2825=3mEq/L.Anacuterespiratoryacidosiswouldbeexpectedto

raisebicarbonateby0.1Pco2=0.120=2mEq/L(seeTable53(t0020)).Thereis,therefore,
appropriatecompensation.Recallthatmaximalcompensationbythekidneyinthesettingofa
respiratoryacidosistakesabout3days(seeFig.52(f0015)).

Case57continued:

ThepatientwasintubatedandadmittedtotheICUhesteadilyimprovedoverthenextcouple
ofdaysandthensuddenlydevelopedarespiratoryacidosisagain.

5 Howcanyoudetermineifthisexacerbationisduetocentralhypoventilation
fromsedationorduetoaventilationperfusion( / )mismatch,suchas
worseningofhispulmonaryedemaordevelopmentofaventilatorassociated
pneumonia?
Oftenthehistorywillrevealanobviouscauseofarespiratoryacidosis.However,itcangetverytricky
whentherearemultiplepossiblecausesthatfitwiththepatient'shistory.Todetermineifthe
patient'shypercapniaisaresultofsedationorofalungprocess,youcancheckthepatient'saa
gradient(alveolararterialO2gradient).Anormal/unchangedaagradientindicatesthatthelungs
areexchanginggasesnormallybutthattheimpairmentisfromaseparateprocess,suchascentral
hypoventilationfromoversedation(orneuromusculardisease,etc.).Lungdiseaseresultingin /
mismatchwillrevealanincreasedaagradient.
Ifneuromusculardiseasewereincludedinthedifferentialdiagnosisinthispatient,itcouldbe
distinguishedfromcentralhypoventilationbymeasuringinspiratorypressures.Neuromuscular
disorderswillhaveanormalaagradientbutlowpeakinspiratorypressures,whereasboththeaa
gradientandinspiratorypressureswouldbenormalincentralhypoventilation.

Thedifferentialdiagnosisincludeslungdisease(usuallyviaventilation/perfusion
mismatch)aswellascentralhypoventilation,neuromusculardisorders,upperairway
obstructions,andthoraciccageabnormalities.
Maximalcompensation(i.e.,bicarbonateretention)bythekidneyinthesettingofa
respiratoryacidosistakesabout3days.
Deadspaceventilation(suchasinemphysema)resultsinhypercarbiaearly,butin
disordersinvolvingintrapulmonaryshunts(suchasasthma,pulmonaryedema,
pneumonia,atelectasis,orpulmonaryembolism[PE])hypercarbiadoesnotoccuruntil
thereisalargeshuntorrespiratorymusclefatiguedevelops.
Anincreasedalveolararterial(aa)gradientsuggestsa / abnormalityandalung
defect.
Anormal/unchangedaagradientsuggestscentralhypoventilationoraneuromuscular
disorderasthecause.
Neuromusculardisorderswillreveallowpeakinspiratorypressures,whichwillbe
normalincentralhypoventilation.
SummaryBox:RespiratoryAcidosis

Copyright2015Elsevier,Inc.Allrightsreserved.

BOOKCHAPTER

Anemias
AllysonM.Reid,ThomasA.BrownMDandSonaliJ.Shah
USMLEStep1Secrets,Chapter12,371403

HematologyontheUSMLEStep1isdividedintothreemajorsubjects:anemias,bleeding
disorders,andhematologicmalignancies.Wewillbecoveringeachofthesesubjectsinaseries
ofthreechaptersbecauseofthesheervolumeandimmenseimportanceofallthesetopics.
Anemiasthemselvesarecommonlytestedonboards,andasyoumayhavefiguredout,there
aremanydifferentcauses.TheUSMLEwillexpectyoutoreasonthroughthecauseofa
patient'sanemiabasedonclinicalhistoryandlaboratoryvalues.Imagesofbloodsmearswill
oftenbeprovidedtoaidyouinyourdiagnosis.Youshouldknowthedifferentcellmorphologies
associatedwithvarioustypesofanemia(e.g.,sickledcells,bitecells,spherocytes,Heinzbodies)
andwhatthesecellslooklikeonabloodsmear.Youshouldalsoknowthevariousclinicaltests
andlaboratoryparametersusedtoclassifyanddiagnosedifferentanemias.Thesearediscussed
laterinthechapter.
Itwillbeofenormousbenefittoyoutogroupthecausesofanemiaaccordingtotheirfindings
(e.g.,intravascularvs.extravascular,hemolyticvs.nonhemolytic)asyoustudy.Ifyoudevelopa
systematicapproachtotacklinganemias,youcanmakeacomplicatedsubjectmuchsimpler.
Wewillattempttodemonstratethispracticewithinthischapter.
Insider'sGuidetoAnemiasFORtheUsmleStep1

Basicconcepts
1 Whatisanemiaandhowisitdefined?
Anemiaisthelackofnormallyformed,properlyfunctioningredbloodcells(RBCs)inthecirculation,
whichimpairsthebody'sabilitytooxygenatethetissuesatoptimallevels.Anemiamaybe
quantitative(decreasedRBCcount),qualitative(disorderedcellularmorphologyorhemoglobin
structure),oracombinationofboth.Theseabnormalitiescanbeshownonacompletebloodcount
(CBC),hemoglobinelectrophoresis,peripheralbloodsmear,orrarely,abonemarrowaspirateif
necessary.

Normalreferencerangesforallimportantlaboratoryparameters(includinghematologic
parameters)willbeprovidedtoyou,sodonotwastetimememorizingallofthesenumbers.
Instead,focusonspecificcluesintheclinicalpresentation,laboratorytests,andhistologic
findingsthatarediagnosticforeachparticularcondition,aswellasthemajortreatmentsand
complicationsassociatedwitheach.
Althoughnotrequired,itmaybehelpfultofamiliarizeyourselfwiththemostcommon
laboratoryvaluestosaveyourselfprecioustimeontheexamination.BothUSMLEWorldand
KaplanQbankprovidethesevaluesinchartsthatcloselyresemblethoseyouwillfindonyour
actualexamination.
Step1Secret

2 Whatarethethreepathophysiologicmechanismsresultinginanemia?
Simplyput,anemiacanresultfromdecreasedproductionofRBCs,increasedprematuredestruction
ofRBCs,orlossofRBCsfromleakageoutofthecirculation,asinbleeding.
Impairedmarrowproductionisseeninsubstratedeficiencystates(iron/folatedeficiency),disorders
ofhemesynthesis(sideroblasticanemias),disordersofhemoglobinsynthesis(thalassemias),
impairedmarrowresponsivenesstoerythropoietin(anemiaofchronicdisease),bonemarrow
infiltrativeconditions(malignancies),andconditionsassociatedwithreducederythropoietin
production(renalfailure).
IncreasedRBCdestructioncanbeseeninavarietyofinheritedandacquiredconditions.Someof
theseareassociatedwithabnormalitiesinRBCstructureandfunction,suchashereditary
spherocytosis,sicklecellanemia,glucose6phosphatedehydrogenase(G6PD)deficiency,and
paroxysmalnocturnalhemoglobinuria(PNH).Othersareassociatedwithautoimmuneanddrug
reactions,oftenleadingtohypersplenismasthedamagedcellsareremovedfromthecirculation.
Chronicbloodloss(asinaslowintestinalbleed)typicallycausesanunderproductionanemiaby
depletingthebody'sironstores.Thislosscanalsooccurinamenstruatingwomanwithheavy
menstrualbleeding,endometriosis,oruterinefibroids.

2 Whatarereticulocytes?Whatisanormalreticulocytecount?
ReticulocytesareimmatureRBCsproducedbythebonemarrowandreleasedintotheblood.The
numberofreticulocytespresentintheperipheralbloodprovidesanindicationofhoweffectivelythe
bonemarrowisproducingRBCs,andthusrespondingtoananemia.
Anormalreticulocytecountvariesbetweenapproximately0.5%and1.5%.Alowornormal
reticulocytecountinasettingofanemiatypicallyindicatesanunderproductionanemia.However,
calculationofthereticulocyteindex(RI)canhelpdeterminewhetherornotthemarrow
compensationisappropriatefortheseverityoftheanemia.TheRIcanbecalculatedbycorrecting
thereticulocytecountforthedegreeofanemia:

RI = Reticulocyte count (Patient's hematocrit/Normal hematocrit)

Withsevereanemia(hematocrit<25%),reticulocyteprecursorsarereleasedfromthemarrowearlier
thannormalreticulocytes.ThesemustbeaccountedforwhendeterminingRI.Simplydividethe
calculatedRIbyafactorof2toyieldthepercentageofreticulocytes.
Example:Assumingasevereanemiawithahematocritof10%andareticulocytecountof2.5%,we
have
RI = (2.5% 10/40)/2 = 0.625

Here,wehadtodivideby2toaccountfortheproreticulocytesfoundincirculationduringsevere
anemia.Althoughthisisusefultoknowforyourclinicalyears,itisunlikelythatyouwillbeexpected
tomakethiscalculationonboards.Attheminimum,knowhowtocalculateRIwithoutworrying
abouttheseverityofanemia.
AnormalRIis1.Intheprecedingexample,thebonemarrow(BM)isnotappropriately
compensatingforthesevereanemiabyincreasingreticulocytereleaseintothecirculation.Wewould
expectthereticulocytecounttoexceed3%incasesofanemia.Inthissituation,wecansaythatthe
bonemarrowisunabletoproduceenoughRBCprecursorstoamelioratetheanemia.
Note:Anabsolutereticulocytecountcanbeobtainedbymultiplyingthereticulocytecountbythe
concentrationofRBCs.Forexample,inanormalpatientwithareticulocytecountof1%andaRBC
countof5106/L,theabsolutereticulocytecountis50,000RBCs/L.

3 Whatarethehemolyticanemiasandhowaretheytypicallyclassified?
HemolyticanemiasareanemiasinwhichRBCdestruction(hemolysis)isthecauseoftheanemia.
Thecausemaybeeitherhereditaryoracquired.Hereditarycausesincludehereditaryspherocytosis,
PNH,G6PDdeficiency,andsicklecellanemia.Acquiredhemolyticanemiascanbefurtherclassified
asautoimmune(e.g.,autoimmunehemolyticanemia),druginduced,ortraumatic(e.g.,mechanical
prostheticheartvalves).

4 Regardinghemolyticanemia,whatisthedifferencebetweenintravascularand
extravascularhemolysis?
IntravascularhemolysisoccurswhenRBCsaredirectlylysedwithinbloodvessels.Causativefactors
rangefromcomplementmediatedhemolysisinPNHtomechanicalfragmentationofRBCsby
mechanicalprostheticvalvesorbyfibrinclotproducts,asinmicroangiopathichemolyticanemias,
whichoftenoccurinthesettingofdisseminatedintravascularcoagulation(DIC).Extravascular
hemolysis,asthenameimplies,takesplaceoutsidethevasculature.Inextravascularhemolysis,
splenicmacrophagesorKupffercellsintheliverdestroyRBCsduetostructuralormorphologic
abnormalitiesoftheRBCs(e.g.,hereditaryspherocytosis,hypersplenism).

5 Whyisthereagreaterdegreeofhemoglobinemiaandhemoglobinuriain
intravascularhemolysisthaninextravascularhemolysis?
Inintravascularhemolysis,lysedRBCsspilltheirhemoglobindirectlyintothebloodstream
(hemoglobinemia),whichmaythenbefilteredoutintotheurine(hemoglobinuria).Inextravascular
hemolysis,thehemoglobininphagocytosedRBCsismetabolizedintracellularlytobilirubin,reducing
theamountofhemoglobinthatendsupinthebloodorurine(Table121(t0010)).
Table121
CharacteristicsofHemolysis

HematologicFeature IntravascularHemolysis

Extravascular
Hemolysis

Hemoglobinemia

Yes

Noneorslight

Hemoglobinuria

Yes

Noneorslight

Plasmahaptoglobin

Largedecrease

Normalorslight
decrease

Anemia

Yes

Yes

Jaundice

Yes

Yes

Hepatosplenomegaly No

Often

Exampledisorders

Immunemediated
(ABOmismatch),
hypersplenism

Microangiopathichemolyticanemia,disseminated
intravascularcoagulation,thromboticthrombocytopenic
purpura,paroxysmalnocturnalhemoglobinuria

Haptoglobin,aserumproteinthatsopsupfreehemeinthecirculation,willattempttoreducethe
freehemesecondarytohemolysisregardlessofetiology,causingitslevelstodecrease.Forreasons
justexplained,haptoglobinwillbemoregreatlyreducedinthesettingofintravascularhemolysis
thaninextravascularhemolysis.Youshouldkeepthisinmindforboards!

6 Howaremicrocytic,macrocytic,andnormocyticanemiasdefined?
ThisclassificationschemeusedtocategorizeanemiasisbasedonthesizeoftheRBCsfoundinthe
patient'sbloodstream.RBCsizeisgenerallydeterminedbymeancorpuscularvolume(MCV),which
iscalculatedbydividinghematocritbyRBCcountperliter.Normalreferencerangeis80to100fL.
Anemiasthatremaininthisrangeareclassifiedasnormocytic.Thosethatfallbelowthisrangeare
microcytic,andthoseabovetherangearetermedmacrocytic.Variouscausesofthesethreetypesof
anemiaswillbeexploredthroughoutthecasespresentedinthischapter.

7 Whatistheusualcauseofanemiainendstagerenalfailure?

Inthesettingofendstagerenalfailure,anemiaisprincipallyduetoreducedproductionof
erythropoietinbythediseasedkidneys.However,uremiafromrenalfailurecanalsomakethebone
marrowlessresponsivetoerythropoietin.Anemiacausedbyrenalfailuretypicallyrespondswellto
exogenouslyadministerederythropoietin.

Case121
An8yearoldKenyanimmigrantboypresentstohispediatriciancomplainingofgeneralized
fatigueandmildyellowtingetohissclerasforthepastseveralmonths.Hehasnofamily
historyofanemia.Theremainderofhisphysicalexaminationisnormal.Concernedabout
elevatedbilirubinlevels,yousuggesttoyourattendingtoobtainaCBCtocheckforanemia,as
wellasliverfunctiontestsandhepatitisserologictestingtoruleoutliverfailureasacauseof
jaundice.Pertinentlaboratoryfindingsareasfollows:CBCrevealshemoglobinof9.0g/dL
(normal1215),hematocritof27%(normal3542%),andanormalMCV.

1 Whatisthedifferentialdiagnosisfornormocyticanemia?
Aplasticanemia
Hemolyticanemias
Hereditary
Spherocytosis
PNH
Sicklecellanemia
G6PDdeficiency
Pyruvatekinasedeficiency
Acquired
Autoimmune
Druginduced
Traumatic
Myelodysplasia
Renaldisease
Anemiaofchronicdisease
Bloodloss

Case121continued:
AperipheralbloodsmearrevealssickledRBCsandHowellJollybodies(Fig.121(f0010)).

Figure121
PeripheralbloodsmearforpatientinCase121(s0055).
(FromSteinJH:InternalMedicine,4thed.St.Louis,Mosby,1998.)

2 Whatistheexpecteddiagnosisnowandwhatistheconfirmatorytest?
Sicklecellanemia,ahereditaryhemoglobinopathycausedbyapointmutationinposition6ofthe
globinchain,isexpected.Hemoglobinelectrophoresiscanprovidedefinitiveevidencetomakea
diagnosisofsicklecellanemia.Inthistest,RBCsareobtainedfromthepatient,lysedtofreeupthe
hemoglobinproteins,andseparatedbysizeandchargeonagel.Abandingpatterndevelopsthat
correspondswiththesizeofthehemoglobinproteins.Eachspecifictypeofhemoglobinhasan
identicalbandingpatternbasedonthehemoglobinvariantpresent.Therefore,apatientwithsickle
cellanemiashouldshowahemoglobinelectrophoresisbandingpatternconsistentwithhemoglobinS
(HbS)(thesicklecellvariant).
Note:Thepresenceofsickledcellsonperipheralsmearisdiagnosticofsicklecellanemiabutisnot
theconfirmatorytest.Electrophoresisisthegoldstandard.

Case121continued:
Hemoglobinelectrophoresisofthepatient'sbloodshowsthepresenceofabnormalHbS.

3 Whatarethethreemaintypesofhemoglobinfoundwithinnormaladultred
bloodcells?Howdoesthisexpressionpatterndifferinsicklecellanemia?
ThethreeprimarytypesofhemoglobininadultRBCsarehemoglobinA(22),hemoglobinA2(
22),andfetalhemoglobinF(HbF)(22).Thevastmajority(>95%)ofhemoglobininadultRBCs

isnormallyofthehemoglobinA(HbA)type.Becausesicklecellanemiaiscausedbyasubstitutionof
thehydrophobicvalineforthehydrophilicglutamicacidintheglobinprotein,thereisdeficient
productionofHbAwithincreasedexpressionofabnormalHbS.

4 Whatcausessicklingofredbloodcellsinthisdisease?

Inthedeoxygenatedform,HbSissignificantlylesssolublethanHbAandisthereforepredisposed
toprecipitatefromthecytoplasmunderconditionsthatcausehigherconcentrationsof
deoxyhemoglobin(e.g.,hypoxemia,acidosis,hyperosmolarity/dehydration).Althoughtheeffectsof
hemoglobinprecipitationareinitiallyreversible,repeatedboutsofhemoglobinprecipitationleadto
irreversibledefectsinstructureandfunctionoftheRBCmembrane,resultinginchronicallysickled
cells.

Case121continued:
Physicalexaminationrevealsmildscleralicterusandnosplenomegaly,andisotherwise
unremarkable.Liverfunctiontestsrevealatotalbilirubinof1.4mg/dLanddirectbilirubinof
0.3mg/dL.

5 Whatisthemechanismofanemiainsicklecellanemia?
Anemiaresultsfromincreasedintravascularhemolysisduetomechanicalfragmentation,although
thereisalsoasmallcomponentofimpairederythropoiesisatthelevelofthebonemarrow.

6 Ifthereishemolysisproducingtheanemia,whyisthespleennotpalpable?
Bytheageof5years,94%ofsicklecellpatientswillhaveexperiencedautoinfarctionofthespleenas
aresultofthetortuouscirculationthatthemisshapenRBCsmustpassthrough.Lackingadequate
oxygenation,thespleenbecomessmall,dense,andfibrotic.Calificationmaybeseenonabdominalx
rayfilms.
Note:Priortoautoinfarction,splenomegalymayoccur.SplenomegalyoccurswhenRBCsocclude
theefferentsplenicvessels,leadingtoaccumulationofbloodwithinthespleenandconsequent
engorgementoftheorgan.Thiscansometimesleadtoasplenicsequestrationcrisis,inwhichRBCs
andplateletsnormallyfoundwithinthegeneralcirculationpoolintheenlargedspleen.Platelet
countsandplasmalevelsofhemoglobinandhematocritmayfallbelownormal,causinghypovolemic
shockanddeath.

7 Whyareparentsofchildrenwithsicklecelldiseasetaughthowtopalpatethe
spleenwhenevertheirchildrendevelopafebrileillness?
Variousinfectionspredisposetosplenicsequestrationcrisesinwhichthespleenenlargesacutelyto
aidintheimmunologicresponseagainsttheinfection.ThisinmassiveRBCsequestrationand
hemolysisinsicklecellpatientsasthespleniccordsarecompressed,makingitevenmoredifficultfor
therigidsickledcellstopassthrough.Suchsplenicsequestrationcausesarapiddropinthe
hematocritandmaycausesymptomsofintravasculardepletionandhypovolemicshock.Ifaccessto
medicalcare(e.g.,RBCtransfusion)isnotavailable,themortalityrateforthesesequestrationcrises
canapproach15%.

Case121continued:

Onfurtherquestioning,thepatientadmitstointermittentepisodesofsharpabdominalpain.

8 Whatarevasoocclusivecrises?
ThisisatermtodescribeconditionsunderwhichincreasedRBCsicklingoccurs.Theincreased
sicklingofRBCoccludesbloodvessels,resultinginlocallacticacidosisandpainsecondarytotissue
hypoxia.

9 Whatfactorsarethoughttoincreasetheriskofexperiencingthesecrises?
Theaforementionedfactorsthatincreasedeoxyhemoglobinconcentrations,suchasdehydration,
hypoxemia,andacidosis,mayalltriggerthesecrises.

Case121continued:
Pastmedicalhistoryisremarkablefortwoprevioushospitaladmissionsforsepsiscausedby
encapsulatedbacteria.

10 Whyshouldthisboyreceivevaccinationsagainstencapsulatedbacteria?
Recurrentvasoocclusivecrisesinthespleentypicallyleadtofibroticscarringofthespleen(referred
toasautosplenectomy),whichsignificantlyincreasessusceptibilitytoinfectionbyencapsulated
bacteriasuchasStreptococcuspneumoniaeandHaemophilusinfluenzae.
Note:Justasthelymphnodesfilterbacteriafromlymphaticfluid,thespleenfiltersencapsulated
bacteriafromtheblood.

11 Thisboyhasonebrotherwhodoesnothavethedisease.Neitherofhis
parentsareaffected.Howwouldyoudescribethegeneticsofthisdisease?
Thefactthatnoothermembersofhisfamilyareaffectedsuggestsautosomalrecessiveinheritance.
Bothparentsmustbeunaffectedcarriersofsicklecelltraittopassthetraitontotheirson.
Approximately25%oftheoffspringgeneratedbytwocarrierswillbeaffectedwithsicklecelldisease.
Thisboy'sbrothermayalsobeacarrierofsicklecelltrait,havinggottenonecopyofthemutation
fromoneparentandanormalcopyfromtheother,orhemaybecompletelyunaffected,having
gottennormalcopiesofthegenefrombothparents.

12 Howdoessicklecelltraitdifferfromsicklecelldisease?
Theheterozygouscarrierofthesicklecellmutationissaidtohavethesicklecelltrait,genotypically
referredtoasHbAS.Thesepatientsarerelativelyasymptomatic,withveryminimalsymptomsof
anemia,andtypicallydonotexperienceepisodesofpainfromvasoocclusivecrises.The
predominanthemoglobininsicklecellcarriersisHbA,justasinunaffectedindividuals.

13 Whatistheevolutionarypressureforsicklecelltrait?

ThesicklecellmutationismorecommoninAfricanAmericansbecauseinitsheterozygousformit
providesprotectionfrominfectionwithPlasmodiumfalciparum.Infact,inregionsofAfricawhere
malariaiscommon,upto25%to30%ofthepopulationisheterozygousforthismutation.

Case121continued:
Twoweekslaterthispatient'smotherbringshiminbecauseofasignificantincreaseinfatigue
(evidencedbydecreaseddesiretoplay),irritability,andafacialrashwithaslappedcheek
appearance.

14 Whatinfectiondoyoususpectandwhatseriouscomplicationshouldbe
considered?
Theslappedcheekappearanceischaracteristicofparvovirusinfection.Parvovirusisknowntoinfect
erythrocyteprogenitorcellsandcauseaplasticcrisisinsicklecellanemia.Thecombinationof
increasedRBChemolysis(sickledcellshaveaseverelyreducedlifespan)andimpaired
erythropoiesisinsicklecellanemiacanprecipitateaseverestateofanemia.

Case121continued:
Fourmonthslater,thispatientpresentstohislocalemergencyroomwithchestpainand
shortnessofbreaththatdevelopedashewasplayingoutdoors.Oxygensaturationis84%on
roomair,andchestxraystudyrevealsperihilarinfiltrates.

15 Whatisthelikelydiagnosisandwhydoesitoccurinsicklecellanemia?
Acutechestsyndromeiscausedbyocclusionofthepulmonaryvasculaturebysickledcells.Thiscan
alsocausepulmonaryedemaandelevatedwhitebloodcell(WBC)countandmaybe
indistinguishablefrompneumoniaonchestxrayfilm.Treatmentincludesrespiratorysupport,
exchangetransfusion,andempiricantibioticsforpneumoniaduetofluidstasisinthelungs.
Achildexperiencingchestpainduringplayisacommonlyusedclinicalvignetteonboards.
Wheneveryouseethis,consideracutechestsyndromeinasicklecellpatient.Additional
medicalhistorytosupportyourdiagnosiswillbeprovidedtoyou.
Step1Secret

16 Whataretheindicationsforexchangetransfusioninsicklecelldisease?
Strokeortransientischemicattack(TIA),acutechestsyndrome,priapism,thirdtrimester
pregnancy,andintractablevasoocclusivecrisisareallindicationsforexchangetransfusion.

17 Whyisthisboyisatriskforpapillarynecrosisofthekidneys?

Theconditionsofhypoxemia,acidosis,andhyperosmolarityspecificallypresentintherenalmedulla
(anareathatisespeciallypronetohypoxia)increasessicklingofRBCs,resultinginvasoocclusionof
thevasarectaintherenalpapillae.Thisresulteventuallyleadstopapillarynecrosis.

Case121continued:
Ashisphysician,yourecommendprophylacticcholecystectomyonceheisstable.

18 Whyarepatientswithsicklecelldiseaseatanincreasedriskforgallstones?
About70%ofpatientswithsicklecelldiseasewillgetsymptomaticcholelithiasis.Recallthatthese
gallstoneswillbepigmentedduetohyperbilirubinemiasecondarytochronichemolysis.Havinga
prophylacticcholecystectomycanalsohelpdistinguishgallbladderpainfromcholelithiasisand
abdominalpainsecondarytovasoocclusivecrises.

19 Patientswithsicklecelldiseasecandevelopcomplicationsinmanyother
organsystemsaswell.Whattwobonediseasesarethesepatients
predisposedtoandwhy?Towhichotherconditionsarepatientswithsicklecell
diseaseprone?
Vasoocclusivephenomenainsicklecellanemiacancauseavascularnecrosisofbones,particularly
avascularnecrosisofthefemoralhead.Theseavascularareasofbonearethenmoresusceptibleto
thedevelopmentofinfection(osteomyelitis).Salmonellaosteomyelitisisseenmorefrequentlyin
sicklecellanemia,yetStaphylococcusaureusisthemostfrequentcauseofosteomyelitisinthe
generalpopulation.
Note:ThereasonforthisdiscrepancyisattributedtothefactthatSalmonellaisanencapsulated
organism.Patientswithsicklecellcrisiswhoundergoautosplenectomyareatriskforinfectionby
encapsulatedorganisms.
Otherconditionsaffectingsicklecellpatientsincludethefollowing:
Enlargedheart/congestiveheartfailure(CHF)
Ischemicretinopathy
Priapism

20 Somepatientswithsicklecelldiseasearetreatedwiththechemotherapeutic
drughydroxyurea.Whatwouldbeyourrationaleforstartingthispatienton
thistreatment?
Hydroxyureaisachemotherapeuticdrugthatincreasestheproductionoffetalhemoglobin(HbF),
whichpresumablydecreasestheamountofHbSexpressionandthereforedecreasesHbS
polymerizationandRBCsickling.However,theprecisemechanismofactionofhydroxyurearemains

poorlyunderstood.Someauthorsbelievethatitsmainmechanismofactionisthroughstabilizing
RBCmembranes.Nevertheless,studentsshouldknowthatitisanimportanttherapeuticagentfor
sicklecelldisease.

21 WhataresicklecelltraitandHemoglobinC(HbC)?
Sicklecelltraitoccurswhenapersonhasoneabnormalsicklecellallele(HbS)butisnothomozygous
forthemutation.Becausetheallelesarecodominant,heterozygouspatientsproducenormaland
abnormalhemoglobin.Approximately90%abnormalhemoglobinproductisrequiredtoproduce
sicklecellsymptoms,soindividualswithsicklecelltraitareoftenasymptomatic.
HbCistheproductofanalternativemutationinposition6oftheglobingeneinwhichglutamic
acidissubstitutedforlysine.Inthesamefashionasforthesicklecellmutation,individualswhoare
heterozygousforthemutationoftendonothaveanyanemia,andhomozygousindividualshavea
mildhemolyticanemia.IndividualswhoareHbSChaveamilderformofsicklecelldiseasethan
HbSSpatients.

Sicklecelldiseaseisinheritedinanautosomalrecessivemanner.
Carriersofsicklecelltraitareusuallyasymptomaticandareprotectedfromfalciparum
malaria.Africansaremoreoftenaffected.
Examinationmayshowscleralicterus,mildjaundice,andhepatosplenomegalyin
patientsyoungerthan5years.
Bloodtestsshownormocyticanemia,reticulocytosis,andelevatedindirectbilirubin.
Bloodsmearshowssickledcells,HowellJollybodies,andreticulocytosis.
HemoglobinelectrophoresisshowshemoglobinS(HbS).
Treatmentincludeshydroxyurea,pneumococcalandHaemophilusinfluenzaevaccines,
andanalgesiaandhydrationfortreatmentofacutecrises.
Sicklecellcrisesareprecipitatedbyinfection,dehydration,andhypoxemia.
Complicationsincludethefollowing:
Cholelithiasis
Renalpapillarynecrosis
Asepticnecrosisoffemoralhead
Infectionwithencapsulatedbacteria:H.influenzae,Streptococcuspneumoniae,
Salmonella

Parvovirusinfectioncausingaplasticanemia
Priapism
Splenicsequestrationcrisis
SummaryBox:SickleCellDisease

Case122
A6montholdboyofGreekdescentisbroughttotheofficebyhisparentsbecauseoverthe
past2weekshehasbeensleepingmuchmorethanusual.Theexaminationrevealsconjunctival
pallor,scleralicterus,andhepatosplenomegaly.Concernedaboutapossibleanemia,the
pediatricianobtainsaninitialCBCtocheckforanemiaandinfection.TheCBCreveals
hemoglobinof4.5g/dL,MCVof73fL,andincreasedreticulocytedistributionwidth(RDW).

1 Whatisthedifferentialdiagnosisforamicrocyticanemia?
Irondeficiency,anemiaofchronicdisease,thalassemia,sideroblasticanemia,andleadpoisoningare
considerations.

Case122continued:
Serumiron,ferritin,andtotalironbindingcapacity(TIBC)areallnormal.Aperipheralsmear
showsmanynucleatedRBCsandtargetcells.

2 Howdoesthischangeyourdifferentialdiagnosis?
Thislargelyrulesoutthetwomostcommoncausesofhypochromic,microcyticanemiairon
deficiencyandanemiaofchronicdisease.Bothoftheseconditionsdemonstratedecreasedserum
iron.

Case122continued:
GelelectrophoresisrevealselevatedhemoglobinA2(HbA2)andHbFandthecomplete
absenceofglobinsubunits.

3 Whatisthediagnosis?
Thalassemiamajor,alsoknownasCooley'sanemia.Iftherewaspartialexpressionofglobin,then
thediagnosiswouldbethalassemiaminor,butthisisusuallyeitherverymildorcompletely
asymptomatic.

4 Whatisthepathogenesisofthalessemiamajor?

Impairedsynthesisofthesubunitofhemoglobinduetoahomozygousmutation.Normally,HbAis
atetramermadeoftwochainsandtwochains.Impairedproductionofthechainsleadsto
polymerizationofthechainswithinRBCs.Theseaggregatesareinsolubleandprecipitateand
damagetheRBCmembrane,causingprematurehemolysiswithinthespleenandineffective
erythropoiesisinthebonemarrow.Thecombinationoftheaccelerateddestructionandtheimpaired
productionofRBCsexplainsthesevereanemia.

5 Whydiditnotmanifestuntiltheageof6months?
Forthefirst6months,thisboywasasymptomatic,becausehestillhadlargeamountsofHbF,fetal
hemoglobin,presentinhiscirculation.Remember,HbFcontainstwochainsandtwochainsand
doesnotrequireglobinsynthesisatallforproperfunctioning.Ashetransitionedtosynthesisof
adulthemoglobin(HbAcontainstwoandtwochains),thesymptomsbegantomanifestashe
becamereliantonhemoglobinAproduction.

Case122continued:
Theexaminationrevealsconjunctivalpallor,scleralicterus,andhepatosplenomegaly.

6 Whydoyouseescleralicterusandorganomegaly?
RBChemolysisreleasesheme,whichisdegradedintobilirubin.Theunconjugatedbilirubin
accumulatesanddepositsinthesclera,causingicterus.Thiseventuallyresultsingeneralized
jaundice.
Hepatosplenomegalyoccursfortworeasons.First,thereisincreasedhemolysisofabnormalRBCsby
macrophagesinthespleenandliver.Second,thereisextramedullaryerythropoiesisinresponseto
impairedbonemarrowerythropoiesis.(However,thisisnomoreeffectivethaninthebonemarrow
becauseofthegeneticdefectinglobin.)
Youmaybegivenaradiographofaskullthatdemonstratesacrewcutappearance(seeFig.
122(f0015)).Thisisasignofbonemarrowexpansionandisassociatedwiththalassemiaand

sicklecelldisease.

Figure122

Thalassemia:xrayfilmoftheskullshowingnewboneformationontheoutertable,producing
perpendicularradiationsresemblingacrewcut.
(CourtesyofDr.JackReynolds,DepartmentofRadiology,UniversityofTexasSouthwesternMedical
School,Dallas,TX.)

Step1Secret

Case122continued:
Youtelltheparentsthattheirsonwillneedfrequentbloodtransfusionsfortherestofhislife.
Attheageof4,theboyisseenintheemergencydepartmentbecauseheisunabletowalkafter
fallingoffhistricycle.Thexraystudyshowsafractureoftherighttibia.

7 Whymightthisboybemoresusceptibletofractures?
Ineffectiveerythropoiesisinthebonemarrowresultsinmarkedlyhyperplasticbonemarrowand
bonemarrowexpansion(Fig.122(f0015)).Thisbonemarrowexpansionerodesawaythecancellous
andcorticalbone,resultinginsignificantstructuralweakness.

Case122continued:
Thispatientdoeswellformanyyears,receivingfrequentbloodtransfusions.Attheageof35,
heisseenforaroutinecheckupandisfoundtohaveafastingglucoselevelof130mg/dL.His
skinisnotedtobetannedalthoughhespendslittletimeinthesun.Theelectrocardiogram
(ECG)showssomenonspecificSTTwavechanges.

8 Whatisthediagnosisandwhydiditoccurinthispatient?
Secondary(acquired)hemochromatosisisacommoncomplicationofrepeatedtransfusionsinthese
patients.ExcessironisdeliveredinthetransfusedRBCsandcandepositinorganssuchastheheart,
pancreas,andskin,leadingtorestrictivecardiomyopathyandsocalledbronzediabetes.
Keepinmindthatthalassemiaisadisorderofglobinchaindeficiency,soironstudieswillbe
completelynormal.Theboardsexamlovestoteststudentsontheirunderstandingofvarious
disorders,soyoushouldexpecttobeaskedconceptualquestionslikethisonyourexamination.
Thefollowingtypeofquestionformatcommonlyappearsonboards:
Example:Whichofthefollowinglaboratoryfindingswouldbeexpectedinapatientwith
thalassemiamajorwhoisreceivingbloodtransfusions(comparedwithvaluesforanormal
patient)?

Hemoglobin HemoglobinF SerumIron


A. /normal

Normal

B. /normal

C. /normal

D.

Normal

ThecorrectanswerchoiceisC.Hematology,cardiology,nephrology,andpulmonology
questionsareespeciallysuitableforthis//normalformat.Thus,whenyoustudythese
subjects,youshouldbethinkingalongcomparativeterms.Purchasingquestionbanksoftware
willgiveyousomeadditionalpracticewithhandlingthesetypesofquestions.
Step1Secret

9 Howcouldithavebeenprevented?
Treatmentwithironchelatingagentssuchasdeferoxaminecanreducetheincidenceof
hemochromatosis.Obviously,phlebotomytoreduceironstores(theusualtreatmentforhereditary
hemochromatosis)wouldbecounterproductiveforsomeonereceivingtransfusionsforanemia.

10 Ifthispatienthadpresentedinhisearly20swithsimilarsymptomsand
bloodtestsbutglobinispresentongelelectrophoresis,whatwouldbeyour
diagnosis?Whatisthepathogenesis?
Thalassemiaiscausedbyimpairedproductionofthesubunitofhemoglobin.Therearetwo
genesforthesubunitoneachchromosomeforatotaloffourgenes.Unlesssomeoneismissingall
fourcopiesofthechaingene,thatpersonwillstillsynthesizesomechains.Forthisreason,
thalassemiapatientstypicallypresentlaterinlifewithmildersymptomsthaninthalassemia.
Peoplemissingonecopyofthegeneareasymptomatic,asenoughchainisproducedbythe
remainingthreecopiesofthenormalgene.Missingtwocopiescancauseamildanemiathat
generallydoesnotrequiretreatment.MissingthreecopiesresultsinaconditioncalledHbHdisease.
Thegelelectrophoresiswillshownormalglobinanddecreasedglobinsubunits.HbH
precipitatesonstainingwithbrilliantcresylblue.Typically,thisoccursinpeopleofAsianorAfrican
descent.ThoseofAfricandescentaremorelikelytohavereceivedonebadcopyfromeachparent,
leadingtoan/genotype.ThoseofAsiandescentaremorelikelytohavegottentwobadcopies
fromoneparent,leadingtoan/genotype.

11 Thalassemia,initsmostsevereform,iscalledhydropsfetalisandcanbe
fatalinutero.Whatishydropsfetalisandwhyisitfatal?
Hydropsfetalisoccurswhenthefetusismissingallfourcopiesoftheglobingene.HbA,HbA2,
andHbFallrequirethesubunitstoform,soinitscompleteabsence,noneofthesecanbemade.
Thefetusinsteadproduceshemoglobinconsistingoffourchains(hemoglobinBarts).Thisformof

hemoglobinhasanextremelyhighaffinityforoxygenandhinderstheabilityofRBCstodeliverthis
oxygentotissues.Fetusesaffectedwiththisdiseasearebornwithprofoundanemiaduetotissue
asphyxiaaswellasCHFfromthisasphyxia.Thisisoftenfatalinutero,aspropergrowthand
developmentcannotoccurinthesettingofprofoundhypoxia.Thisissimilartotheconditionof
babiesafflictedwithhemolyticdiseaseofthenewborn(RhpositivebabiesborntoRhnegative
sensitizedmothers).
Paycloseattentiontoage,racial,andethniccluesprovidedinquestionstems.Boardswill
commonlypresenttextbookcasesintheirclinicalvignettes,sothesecluesmayaidyouin
prioritizingtheconsiderationsinyourdifferentialdiagnosis.Forinstance,thalassemiais
mostcommoninAfricansandAsians,butthalassemiaismostcommoninAfricanand
Mediterraneanpopulations.NoticethatthepatientinthiscaseisofMediterraneanheritage.
Step1Secret

12 Whyissicklecelldiseaselessseverethannormalinapatientwhoisa
hemoglobinS/thalassemiaheterozygote?
PatientswiththalassemiaproducehighlevelsofHbF,whichdecreasessicklingofRBCs.

Thalassemia
Causedbydecreasedglobinsynthesis
Africans:/
Asians:/
Symptomsbasedonthenumberofgenesdeleted
Asymptomatic
Mildanemia
HbHdisease:splenomegaly
HbBarts:hydropsfetalisincompatiblewithlife
Diagnosis
Smear:microcyticanemia,hypochromia,targetcells,Heinzbodies
HbHinclusionbodiesseenonbrilliantcresylbluestain
Treatment:bloodtransfusions

Thalassemia
Causedbydecreasedglobinsynthesis

Epidemiology:Mediterranean
Symptoms
Thalassemiamajor:anemia,jaundice,andsplenomegalyat6monthsdueto
switchfromfetaltoadulthemoglobin
Thalassemiaminor:nosymptoms
Diagnosis:increasedfetalhemoglobin(HbF)andhemoglobinA2(HbA2)and
decreasedhemoglobinA(HbA)ongelelectrophoresis
Treatment
Transfusions+deferoxaminetoincreaseironexcretion
Splenectomy
Possiblebonemarrowtransplant
SummaryBox:Thalassemias

Case123
A68yearoldwomanisevaluatedinaroutinephysicalexamination.Shenotesincreasing
fatigueandshortnessofbreathoverthepastfewmonths.CBCrevealsamicrocyticanemia.

1 Whatisthedifferentialdiagnosisforamicrocyticanemia?
Irondeficiencyanemia,sideroblasticanemia,thalassemia,leadpoisoning,andanemiaofchronic
diseaseareallconsidered.

Case123continued:
Youorderironstudiesbecauseirondeficiencyisthemostcommoncauseofmicrocyticanemia.
Serumironandferritinarebothdecreased.TIBCisincreased,indicatinganirondeficiency
anemia.AsmearshowshypochromicRBCsandalowrecticulocyteindex.

2 Whyistheanemiamicrocyticandhypochromic?
Ironisrequiredforhemoglobinsynthesis.Reducedcytoplasmichemoglobinresultsinsmaller
(microcytic)cellsthathavelesscolor(hypochromia).

3 Whyaretherefewerreticulocytes?

DecreasedabilitytoproducehemoglobinduetolowironresultsinlessRBCproduction.Anyanemia
resultingfromdecreasederythropoiesiswillresultinalowreticulocytecount(seequestion2from
BasicConceptsforfurtherdiscussion).

4 Whyistotalironbindingcapacityincreased?Whyisferritindecreased?
TIBCistheamountoffreeserumtransferrinthatisavailabletobindiron.Inirondeficiency,not
onlyisthelowserumironcontributingtoanincreasedproportionofunboundtransferrin,butthe
liverisalsoproducingmoretransferrin.
Ferritinisthestorageformofiron,particularlyintheliverandbonemarrow.Ferritinlevelsclosely
parallelthebodystoresofiron,suchthatinconditionsofirondeficiency,ferritinislow,whereasin
conditionsofironoverload(e.g.,hemochromatosis),ferritinishigh.

Case123continued:
Thiswomanhasnothadacolonoscopyin15years.Whenasked,sheadmitstoseeingsome
bloodinherstoolrecentlybuthadassumedithadbeenfromhemorrhoids.Youdoarectal
examinationbutfindnohemorrhoids.Fecaloccultbloodtestispositive.

5 Whatisthecauseofherirondeficiencyanemia?
Bloodlossthroughtheintestinaltractisthecause.Inanyolderpatientwithirondeficiencyanemia,
youshouldalwaysconsideranintestinalbleedwithcoloncancerasthecauseuntilprovenotherwise.

Case123continued:
Acolonoscopyisperformed,andanecroticmassisvisualizedandremoved.Pathologic
diagnosisisreportedasadenocarcinoma.

6 Supposethiswomanwere30yearsoldandworkupwasnegativeforintestinal
bleeding.Whatwouldbethemostlikelycauseofheranemia?
Inawomanofreproductiveage,irondeficiencyisusuallyduetomenorrhagia(severebleeding
duringmenstruation)orpregnancy(thoughthisisphysiologicandduetoexpansionofplasma
volumetogreaterthanthatofRBCs).

7 Ironexistsinmanyformsinthebodyandisusedfromhemesynthesisforred
bloodcells.Howisdietaryironabsorbed?
Dietaryironisprimarilyabsorbedintheproximalduodenum,wheretheacidicpHandpresenceof
ferricreductaseenzymefacilitatetheconversionofferriciron(Fe3+)toferrousiron(Fe2+),which
ismorerapidlyabsorbedbyenterocytes.Dietaryironisthentransportedintothecirculationbythe
transmembraneproteinferroportin.Withinthecirculation,ironiscomplexedwithtransferrin,

whichissecretedbytheliver.Transferrindeliversirontoallcellsofthebodywhereironisthen
storedintracellularlycomplexedtotheironstorageproteinferritin.Youmayseethisstorageformof
ironreferredtoashemosiderin.

8 Whichfactorsincreaseordecreasedietaryironabsorption?
Increasedabsorption:organiciron,ferrousiron,acids(e.g.,citrate),lowironstores,high
erythropoietin,pregnancy
Decreasedabsorption:inorganiciron,ferriciron,alkali(e.g.,phosphates),highironstores,
lowerythropoietin,infection,tannins(i.e.,excessiveteadrinking)
Ironabsorptionisregulatedbytheproteinhepcidin,whichisencodedbyageneinvolvedinthe
maintenanceofironhomeostasis.Hepcidindownregulatesferroportinexpressiononenterocytes.
Whenironstoresinthebodyarehigh,hepcidinexpressionincreasesandironabsorptiondecreases
secondarytoferroportindownregulation.Whenironstoresarelow,hepcidinproductionis
decreased.

Case123continued:
SupposethiswomanhadahistoryofHelicobacterpyloriinfectionandcametotheemergency
departmentwitharapidlybleedingpepticulcer.Sheisfoundtohaveahemoglobinof8.1g/dL
withanMCVof81fL.

9 Howwouldthetemporalcourseofanintestinalbleedaffectwhetherthe
anemiawillbenormocyticormicrocytic?
Withasignificantintestinalbleedthatrapidlychangesthehematocrit(withoutdepressingbodily
ironstores),theanemiawillinitiallybenormochromicandnormocytic,butasenoughironislost,it
willevolvetohypochromicandmicrocytic.

Case123continued:
Nowsupposeadifferentwomanwasbroughtinbyambulancefromthesceneofanauto
accident.Heartrateis120beats/minandbloodpressureis80/50mmHg.Anultrasound
showssplenicruptureandintraabdominalbleeding,andthewomanistakentotheoperating
roomforlaparotomy.

10 Wouldyouexpectherhematocrittobelow,normal,orhighandwhy?
HematocritisthepercentageofbloodvolumeoccupiedbyRBCs,andwitharapidbleed,thisremains
unchanged.Thehematocritwouldbenormalbecausewitharapidhemorrhage,sheislosingplasma
andRBCstogether.

Anemiaisduetodecreasedironstoresbecauseironisneededforhemoglobinsynthesis.
Commoncauses:bloodlossfrom(slow)intestinalbleedingormenorrhagia,
malnutrition(irondeficiencyindiet),pregnancy
Symptoms:increasedfatigueanddyspnea
Physicalexaminationfindingsinseveredeficiency:
Glossitis:redness,swelling,andlossofpapillaeonthetongue*(fn0010)
*Notethatthetriadofirondeficiency,esophagealweb,andatrophicglossitisisreferredtoas
PlummerVinsonsyndrome.

Angularcheilosis:cracksatthecornersofthemouth
Koilonychia:spoonnails
Esophagealweb*(fn0010)
Pica:eatingclayorice
Laboratorytests:lowhemoglobin,lowmeancorpuscularvolume(MCV),lowserum
iron,lowferritin,hightotalironbindingcapacity(TIBC)
Treatment:ironreplacement
SummaryBox:IronDeficiencyAnemia

Case124
A42yearoldwomanwithrheumatoidarthritiscomesintoyourofficecomplainingof
increasingfatigueandshortnessofbreathwithexertion.HerCBCshowsamicrocyticanemia.

1 Whatisthedifferentialdiagnosisformicrocyticanemia?
Irondeficiencyanemia,sideroblasticanemia,leadpoisoning,thalassemia,andanemiaofchronic
diseaseareallconsidered.

Case124continued:
IronstudiesreveallowserumFe,elevatedserumferritin,andreducedTIBC.

2 Whatisyourdiagnosisnow?
Anemiaofchronicdiseasesecondarytorheumatoidarthritistypicallypresentsasamild
hypochromicanemia,althoughitcanalsobenormochromicandnormocytic.

3 Whyisferritinelevatedinthiswomanwhileserumironislow?
Thechronicinflammatoryresponseresultsinlargeamountsofhepcidin,whichsequestersiron,
resultinginreducedlevelsofplasmairon.Bodilystoresofironarenormalbutunavailablefor
erythropoiesis.Thisisshownbythenormalferritinlevels.Ferritinisanintracellularstorageformof
iron.Ferritinlevelscloselyparallelbodystoresofiron.Therefore,ferritinlevelsarelowiniron
deficiencyanemiabutnormaltohighinanemiaofchronicdisease.Ferritinlevelsmaybeelevated
abovenormalinanemiaofchronicdiseasebecauseferritinisanacutephasereactantthatissecreted
bytheliverininflammatoryconditions.
Note:Thepostulatedreasonforironsequestrationinstatesofchronicinflammationisthatthisis
thebody'sautomaticattempttopreventmicrobesthathavepotentiallyinvadedthehostfrom
acquiringironrequiredfortheirgrowth.

Case124continued:
Transferrinsaturationisreduced.

4 Whyaretransferrinsaturationandtotalironbindingcapacityreduced?
Transferrinissecretedbytheliverandbindsironinthebloodstream.TIBCisactuallyameasureof
transferrinthatisnotboundtoiron.ThemechanismoflowTIBCinanemiaofchronicdiseaseisnot
actuallyknownbutitispostulatedthatthechronicinflammatoryresponsedecreasestransferrin
productionbytheliver.Inaddition,theelevatedlevelsofhepcidinsequestersironinthe
bloodstreamresultinginreducedtransferrinsaturationwithiron.

Case124continued:
Furtherhistoryrevealsthatshehashadrheumatoidarthritisfor10yearsthathasbeen
moderatelycontrolledonmethotrexateandnonsteroidalantiinflammatorydrugs(NSAIDs).

5 Whatdoyouhavetoruleoutasacauseforheranemiabasedonthis
additionalinformation?
NSAIDscancausegastriculcersandupperintestinalbleeding.InapatientchronicallyonNSAIDs,
youhavetoruleoutbleedingandresultantirondeficiencyasacauseforheranemia.Thiscanbe
donewithanoccultbloodtestorupperendoscopyifsuspicionishighenough.Methotrexatecan
suppressthebonemarrow,resultinginapancytopenia.ThiscanberuledoutwithaCBCbylooking
atallthreecelllines(i.e.,RBCs,WBCs,andplatelets).

6 Whydoesherlongstandinginflammatoryconditionpredisposeherto
developinganemia?

Chronicinflammatorydisorderswithsystemicinvolvementresultinelevatedplasmalevelsofmany
cytokines.Thesecytokinesincreasethephagocyticactivityofimmunecells,particularlyinthe
spleen,resultinginincreasedphagocyticdestructionofRBCsandadecreaseinRBClifespan.
Additionally,thesecytokinesinhibitthesecretionoferythropoietin,whichnormallystimulates
erythropoiesisinthebonemarrow.Anothercontributingfactoristhathepcidin,whichisreleasedby
inflammatorycells,bindsserumironandmakesitunavailableforerythropoiesis.Althoughanemia
ofchronicdiseaseistypicallymild,ifitissevere,thesepatientsrespondwelltoexogenously
administerederythropoietin.

7 Ifthiswomanhadheavymensesorhemepositivestool,youwouldbeworried
aboutirondeficiencyanemia.Howwouldyoudifferentiatebetweenanemiaof
chronicdiseaseandirondeficiencyanemiaonthebasisofserumiron,TIBC,
ferritin,transferrinsaturation,andbonemarrowironstores?
Table122(t0015)comparesthevaluesofthesetestsforbothtypesofanemia.

TheinformationlistedinTable122(t0015)isparticularlyhighyieldforboards.
Step1Secret

Typically,seenasmildanemiainpatientswithchronicinflammatoryconditions
includingrheumatologicdiseases,malignancy,andinfectionssuchastuberculosisand
endocarditis
Characterizedbyafunctionalirondeficiency:impairmentofironmobilizationdespite
sufficientironstores
Symptoms:fatigue,dyspneaonexertion,symptomsoftheunderlyingcondition
Laboratorytests:lowserumironandlowtotalironbindingcapacity(TIBC),elevated
ferritin
Treatment:severeanemia(rare)respondswelltoexogenouserythropoietintreat
underlyingcondition
SummaryBox:AnemiaofChronicDisease

Case125
A65yearoldmanwithahistoryofchronicgastritispresentswithincreasingfatigueanda
tinglingsensationinhistoesforthelastfewmonths.CBCrevealshemoglobinof9.5g/dLand
anMCVof110fL.

Table122
DifferentiationofAnemiaofChronicDiseaseandIronDeficiencyAnemia

Study

CharacteristicResult
AnemiaofChronicDisease IronDeficiencyAnemia

Serumiron

Low

Low

TIBC

Lowtonormal

High

Ferritin

Normaltohigh

Low

Percenttransferrinsaturation Low

Low

Bonemarrowironstores

Low

High

1 Whatisthedifferentialdiagnosisforamacrocyticanemia?
VitaminB12deficiency,folatedeficiency,liverdisease,alcohol,drugs,andmyelodysplastic
syndromesareconsideredinthissituation.

Case125continued:
Furtherworkuprevealsalowreticulocytecountandaperipheralsmearshowing
hypersegmentedneutrophils(Fig.123(f0020)).

Figure123
Megaloblasticchangesofmacrocytosisandhypersegmentedneutrophils.
(FromtheAmericanSocietyofHematologyImageBank,image2611.Copyright1996,American
SocietyofHematology.)

2 Whatistheprobablediagnosisandwhatistheunderlyingdisorderinthis
man'scase?
HehasvitaminB12deficiencysecondarytochronicgastritis.

Figure123(f0020)isparticularlyhighyieldforboards.Youshouldautomaticallyassociate

hypersegmentedneutrophilswithvitaminB12orfolatedeficiency.
Step1Secret

3 Whatarethesomeoftheothercausesofthisdeficiency?
StrictvegandietsaregenerallylackinginvitaminB12.Also,malabsorptionofvitaminB12maybe
duetoilealresection,bacterialovergrowth,celiacsprue,Crohn'sdisease,orDiphyllobothriumlatum
(fishtapeworm)infection.

4 Whyaretherehypersegmentedneutrophils?Lowreticulocytecount?
RememberfrombiochemistrythatvitaminB12(cyanocobalamin)isrequiredfornucleicacid
synthesisandrecyclingoftetrahydrofolate(THF).Why,then,dotheneutrophilsappeartohave
moregenetricmaterialthannormal?WithoutproperDNAsynthesis,thecellscannotprogress
throughthecellcyclefromtheG2(growth)phasetothemitosisphase.Therefore,thecells
continuallygrowwithoutdivision,leadingtomacrocytosisandhypersegmentationofthenuclear
material.Thisisparticularlyprominent(anddiagnostic!)inmegaloblasticanemia.Beware,however,
thatthisisnotspecificforonlyvitaminB12deficiency,asfolatedeficiencywillpresentwithidentical
histologicfindings!
Becausethecellscannotprogressthroughmitosis,rapidlydividingcelllines,especiallythe
erythrocytelineage,willbeunabletokeepupanormalsynthesisrate.Reticulocytosis,thenormal
physiologicresponsetoanemia,cannotoccur.Remember,vitaminB12isimportantforprecursorcell
synthesis,too!Thus,thereticulocytecountislow.
ReviewtheroleofvitaminB12andfolateinbiochemicalpathwayshowandpayparticular
attentiontodefectsinthepathwaysleadtodisease.TheUSMLElovescrossoversfrom
biochemistryintopathophysiology.Rememberthatmostquestionsonyourexamwillbe
secondandthirdorderaswellasinterdisciplinary!
Step1Secret

Case125continued:
Anupperendoscopyandgastricbiopsyshowchronicinflammationofthemucosalliningofthe
gastricfundus.

6 Whatisthepathogenesisofthisman'svitaminB12deficiency?

Chronicgastritis,eitherautoimmuneoridiopathicinorigin,resultsindestructionofgastricparietal
cellsthatarefoundprimarilyinthefundusofthestomach(i.e.,atrophicgastritis).Parietalcells
normallyproduceintrinsicfactor,whichfacilitatesvitaminB12absorptionfromtheterminalileum.
LossofparietalcellsandthusintrinsicfactorpreventsadequatevitaminB12absorption.

7 ExplainhowvitaminB12isabsorbedfromtheintestine
IntrinsicfactorproducedbyparietalcellsofthegastricfundusbindsdietaryvitaminB12inthe
intestinaltract.PancreaticenzymesfacilitatetheabsorptionofvitaminB12boundtointrinsicfactor
intheterminalileum.

8 WhatarethenormalfunctionsofvitaminB12andhowdothesefunctions
relatetotheclinicalsignsandsymptomsofvitaminB12deficiency?
VitaminB12isinvolvedintwoenzymaticreactions,onethatcatalyzestheconversionofboth
homocysteinetomethionineandofmethyltetrahydrofolatetoTHF,andanotherthatcatalyzesthe
conversionofmethylmalonicacidtosuccinylcoenzymeA.Theanemiaisbelievedtobedueto
reducedlevelsofTHF,theformoffolicacidinvolvedinDNAsynthesis.Theneuropathythat
developsinvitaminB12deficiencyisthoughttobeduetoadeficiencyofmethionine,because
methionineservesasaprecursorforSadenosylmethionine,whichisinvolvedinthesynthesisof
variousmyelinproteinsandphospholipids.Decreasedmyelinationofthedorsalandlateralcolumns
resultsindecreasedvibrationsense,decreasedproprioception,gaitapraxia,paresthesias,
incontinence,andimpotence.AnemiaalwaysprecedesneurologicsymptomsinvitaminB12
deficiency(Fig.124(f0025)).

Figure124
RoleoffolateandvitaminB12.TheonlywaytoreformtetrahydrofolateisviavitaminB12dependent
synthesisofmethionine:themethioninesalvagepathway.
(FromClarkA:CrashCourse:MetabolismandNutrition.Philadelphia,WBSaunders,2005.)

Note:Anypatient,particularlyanelderlypatient,presentingwithsymptomsofanemiaand
neurologicdisturbance(eitherperipheralneuralsymptomsoralteredmentalstatus)mustbe
evaluatedforvitaminB12deficiency.Thoughfolatedeficiencypresentswithanidenticalhistologic
picture,folatedeficiencyisnotassociatedwithneurologicdysfunctioninpatientsafterbirth.
Remember,folatedeficiencyisassociatedwithdefectsinneuraltubeclosureinuteroandmay
presentasspinabifida,myelomeningocele,orrarelyanencephaly.

Case125continued:
YoudoaSchillingtest,whichconfirmsyourdiagnosisofperniciousanemia.

9 Whatisperniciousanemia?HowdoestheSchillingtestwork?
Perniciousanemiaistheautoimmunedestructionofparietalcellsinthegastricfundus.Thisleadsto
decreasedproductionofintrinsicfactorandthereforedecreasedvitaminB12absorptioninthe
terminalileum.
TheSchillingtestisnolongerwidelyusedowingtoconcernsaboutuseofradiation.However,this
testnicelydemonstratesthepathophysiologyofvitaminB12deficiencyduetoinadequateintrinsic
factorproduction:
Step1:UnlabeledvitaminB12isgivenparenterallytosaturatethecobalaminreceptorssuch
thatwhenvitaminB12isgivenorallyasignificantportionthatisabsorbedwillbeexcretedin
theurine.
Step2:RadioactivelylabeledvitaminB12isgivenorally.
Step3:TheamountoflabeledvitaminB12ismeasuredina24hoururinesample.Iflessthan
10%oftheamountthatwasadministeredorallyisfoundintheurine,thisindicatespoor
absorptionofvitaminB12intheintestine,asthevitaminB12mustbeabsorbedintothebodyin
thesmallintestinetobelaterexcretedbythekidney.
Step4:Repeatthetestwiththeadditionofexogenousintrinsicfactor.Ifperniciousanemiais

thecauseofvitaminB12deficiency,intrinsicfactorwillcorrectvitaminB12absorption.

10 Whatisthetreatmentforperniciousanemia?
Forseriousdeficiencies,vitaminB12isgiveninweeklyintramuscularinjectionsindosagesashighas
1000g.Thisregimenmayneedtobecontinuedormaybetaperedtolessfrequentinjectionsifthe
patient'svitaminB12levelnormalizes.Oraltherapyrequiresdosagesof1000gto2000gto
ensureadequateabsorption.Remember,oraltherapymustnotbeusedforpatientswith
autoimmuneperniciousanemiabecausevitaminB12willnotbeabsorbedwithoutintrinsicfactor!
Also,becausevitaminB12iswatersoluble,overdoseisnotusuallyaconcern.

11 Supposethismanhadahistoryofchronicpancreatitisandtheadditionof
intrinsicfactordidnotincreasevitaminB12absorption.Whatwouldyoudo
nowtoconfirmadiagnosis?
Thenextstepwouldbetodeliverexogenouspancreaticenzymesandseeifthiscorrectedthevitamin
B12absorption.Finally,ifthisdidnotwork,youcouldtreatthepatientwith4weeksoftetracycline
andreadministerthetest.IfthishelpedtoincreasevitaminB12absorption,thepresumeddiagnosis
wouldbebacterialovergrowth.

12 FolatedeficiencycanpresentsimilarlytovitaminB12deficiencybutwith
somekeydifferences.WhichsymptomsofvitaminB12deficiencyarenot
foundwithfolatedeficiency?
Folatedeficiencydoesnotcauseneurologicsymptoms.BothvitaminB12andfolatedeficiencyshowa
macrocyticanemiawithlarge,hypersegmentedneutrophils,lowreticulocytecount,andincreasedl
lactatedehydrogenase(LDH).

13 Whataresomeofthecausesoffolatedeficiency?
Deficientintake(commoninalcoholicsandtheelderlywhotypicallylackgreen,leafyvegetablesin
theirdiets),pregnancy(duetoincreasedfolaterequirement),andcertaindrugs(methotrexate,
phenytoin)thatinterferewithfolatedependentbiochemicalpathwayscancausefolatedeficiency.

14 Whatisthebestbloodtesttodeterminefolatedeficiency?
ThebestindicatorofchronicfolatedeficiencyisthetotalRBCfolatelevel.Thistestindicatesoverall
storagelevelsoffolate,whereasserumfolateisjustasnapshotintime.Forexample,ifthepatient
justateabigspinachsalad(richinfolate),serumfolatecouldbenormalbecauseofthemeal,butthe
storagelevelmaystillbedepleted.

16 CovereachcolumnofTable123(t0020)andtrytofillintheremaining
informationforyourself

VitaminB12Deficiency
Megaloblasticanemia
Etiology:inadequateintake,perniciousanemia,malabsorption
Pathophysiology:deficiencyofintrinsicfactorpreventsadequatevitaminB12absorption
Symptoms:glossitis,gastricatrophy,neurologicsymptoms(decreasedvibratoryand
jointpositionsense)
Laboratorytests/diagnosis:macrocytosis,largehypersegmentedneutrophils,increasedl
lactatedehydrogenase(LDH),positiveSchillingtest
Treatment:parenteralorintramuscularvitaminB12weeklyuntillevelsarenormal

FolateDeficiency
Megaloblasticanemia
Etiology:deficiencyintake,pregnancy,drugs
Symptoms:fatigue,pallor,dyspneaonexertion
Laboratorytests/diagnosis:macrocytosiswithlowreticulocytecount,increasedLDH,
neutropenia,hypersegmentedneutrophils,thrombocytopenia
Treatment:parenteralfolicacidafterconfirmationofthediagnosis.Folicacidis
contraindicatedinvitaminB12deficiencybecauseitmaskstheanemiaalthoughthe
neurologicsymptomswillprogress.
SummaryBox:VitaminB12andFolateDeficiencies

Case126
An8yearoldwhiteboyisbroughttoyourofficebyhismotherforevaluationoffatigueand
pallor.CBCshowshemoglobinof7.2g/dL,MCVof80fL,andareticulocytecountof11%.Total
bilirubinis2.0mg/dL.

Table123
VitaminB12andFolateDeficiency

Feature

VitaminB12

Folate

Food

Animalproducts

Vegetables

Stores

Longterm,212years

Shortterm,45months

Water

Yes

Yes

Terminalileum

Duodenumandproximaljejunum

sources

soluble
Siteand

mechanism Absorbedwithintrinsicfactor
of

Deconjugationofpolyglutamate

absorption
Function

Cofactorforenzymaticreactions

1carboncarrier

importantforDNAsynthesisand
myelination
Dietary

Uncommonoccurswithvegan

Commonoccursinalcoholicsaleafygreen

deficiency

diet

saladwillincreaseserumlevelsbutnotRBC
folate,sotestingforbothisrequired

Neurologic

Yesdorsalcolumnsmedial

damage

lemniscus,leadingtodecreased

No

vibratoryandjointpositionsense

1 Whatisthedifferentialdiagnosisforanormocyticanemia?
Hemolyticanemia(autoimmune,druginduced,traumatic,hereditary),hemoglobinopathies,aplastic
anemia,myelodysplasia,hypothyroidism,andanemiaofchronicdiseasecouldcausenormocytic
anemia.

Case126continued:
Bothoftheboy'sparentsarecurrentlyhealthy,butthefatherdidhaveasplenectomyyearsago
forsometypeofanemia.

2 Howdoesthischangeyourdifferentialdiagnosis?
Withapossiblefamilyhistoryofanemia,youshouldfocusyourdiagnosisongeneticcausesof
anemia.Hereditaryspherocytosis,G6PDdeficiency,sicklecellanemia,andpyruvatekinase
deficiencyarealloptions.Tohelpnavigatethemanygeneticcausesofanemia,identifythe
inheritancepatternpresentedinthefamily.G6PDisXlinked.Remember,bydefinition,Xlinked
diseasesareneverpassedfromfathertoson!Pyruvatekinasedeficiencyisautosomalrecessive,so
eventhoughhisfatherisaffected,hismotherwouldalsohavehadtobeacarrierofthisrare
condition.Thisseemsunlikely,althoughnotimpossible.Sicklecellanemia,anotherautosomal
recessivecondition,presentssimilargeneticimplausibility.Also,theboyiswhite,makingthisa

muchmorerarepossibility.Hereditaryspherocytosis,anautosomaldominantcondition,would
typicallypresentwiththispedigreeofaffectedfatherandson.Also,thisdiseaseisquitecommon
amongindividualsofNorthernEuropeandescent.Hereditaryspherocytosisjustjumpedtothetopof
thedifferentialdiagnosislist!
Inheritancepatternsofgeneticdiseasesareveryimportant(andhighyield!)forStep1.Taking
thetimetodistinguishbetweendiseaseswithsimilarpresentationsbutdifferentgenetic
pedigreesmayoftenunlockadiagnosisforyou.Onceagain,payspecialattentiontodiseases
thataffectparticularethnic,racial,oragegroups.Althoughneveranabsolute,these
demographiccluesmayalsopointyouintherightdirectionofdiagnosis.Youareexpectedto
knowtheinheritancepatternsofdiseasescommonlytestedontheUSMLE.Thisinformationis
nicelyorganizedinFirstAid.
Step1Secret

Case126continued:
AperipheralbloodsmearshowssphericalRBCsthatlackcentralpalloraswellasHowellJolly
bodies(Fig.125(f0030)).Alaboratorytestusingtesttubesfilledwithsolutionsofincreasing
saltconcentrationrevealsanabnormallyincreasedosmoticfragilityofRBCs.

Figure125
Hereditaryspherocytosis(peripheralsmear).
(CourtesyofDr.RobertW.McKenna,DepartmentofPathology,UniversityofTexasSouthwestern
MedicalSchool,Dallas,TX.)

3 Whatisthediagnosis?
Theboyhashereditaryspherocytosis.

4 Whatistheetiologyofthisconditionandwhydotheredbloodcellsassumea
sphericalconformation?
Mostcommonly,hereditaryspherocytosisiscausedbydysfunctionoftheproteinspectrin,a
cytoskeletalproteinthatprovidesstabilityandplasticitytotheplasmamembraneofRBCs.Inorder
forspectrintofunctionappropriately,itmustinteractwithothercytoskeletalproteins,suchas

ankyrinandprotein4.1.Therefore,mutationsingenesotherthanthespectrincanalsocompromise
spectrinfunction.AnyofthesemutationscandestabilizetheRBCmembraneandcausethecellsto
assumeasphericalshape.Intheabsenceofthenormalbiconcaveshape,thedegreeofcentralpallor
createdbyhemoglobindisplacementtotheperiphery(normally~ofthediameteroftheRBC)is
markedlyreduced.Thissphericalconformationminimizesthesurfaceareatovolumeratioofthe
RBCs.Thisshapeisnotaseasilydistortedinsmallcapillariesasthebiconcaveshape,makingthese
cellspronetomechanicalhemolysiswhensqueezingthroughsmallvessels.Alsoasaresultofthe
unstablemembrane,theseRBCsswellandburstwhenplacedinincreasinglyhypotonicsaltsolutions
aswaterrushesintracellularly.Thisisthebasisforthepositiveosmoticfragilitytest,whichis
diagnosticforhereditaryspherocytosis.

Case126continued:
Physicalexaminationisnotableforpallor,mildjaundice,andsplenomegaly.Youperformboth
directandindirectCoombstestsonthispatient'sblood.Asexpected,botharenegative.

5 HowdotheseCoombstestswork?Whyaretheybothnegative?
ThedirectCoombstestlooksforantigensonthepatient'sRBCs.TheindirectCoombstestchecks
forantibodiestoRBCsinthepatient'sserum.BothCoombstestsareassaysforantibodiesinthe
plasmaorantigensontheRBCsurfacethatleadtoanintravascularhemolysisduetoantigen
antibodyinteraction.Inhereditaryspherocytosis,youwouldnotexpectanimmuneprocesstobea
primarycauseofRBCdestruction.Instead,theabnormalRBCsaresequesteredinthespleendueto
theirlackofdistensibilityandasaresultaresubjecttoextravascularhemolysis.
MoredetailedinformationaboutdirectandindirectCoombstestscanbefoundinChapter15.

6 Whyissplenomegalyseeninthiscondition?
Duringtheirnormalcoursethroughthespleen,RBCsmustundergoimpressiveconformational
changesinordertoexitthespleniccords(i.e.,cordsofBillroth)andenterthesplenicsinusoids.
SphericalRBCsaremuchlessabletoundergothisconformationalchangethanarenormalbiconcave
RBCs.Asaresult,spherocytesobstructthespleniccords,resultinginsplenomegaly.Theyare
ultimatelyphagocytosedbythesplenicmacrophagesatanabnormallyhighrate,leadingtoanemia.

7 Youreferthisboytoapediatricsurgeonforatherapeuticsplenectomytotreat
hisanemia.Whydoestakingouthisspleenhelpwithhissymptoms?
AlthoughsplenectomydoesnotfixthefundamentaldefectintheseRBCs,itdoespreventtheanemia.
Removingthespleenpreventsthehighrateofextravascularhemolysisthatcausestheanemiaseen
inthiscondition.

8 Whatinfectionsisthisboyatriskforaftersplenectomyandhowwouldyou
helptopreventthem?
Infectionsbyencapsulatedbacteria(e.g.,S.pneumoniae,H.influenzae,Neisseriameningitidis).

Infectionbyallthreeofthesebacteriacanbepreventedwithvaccinations,whichthisboyshould
receiveaftersplenectomy.

Case126continued:
About20yearslater,thissamepatientisseeninalocalemergencydepartmentwithcolicky
rightupperquadrantpainthatincreasesaftereating.HehasapositiveMurphy'ssign.

9 Whatisyourdiagnosis?Whyisthispatientatincreasedriskforthis
condition?
Cholecystitis.Patientswithhereditaryspherocytosisareatincreasedriskforgallstonesdueto
hemolysisanddevelopmentofbilirubin(pigment)stonesinthebileductsystem.Asexpected,the
incidenceofbilirubinstoneformationwillbemuchlowerifasplenectomyisperformedtodecrease
therateofhemolysis.However,giventherelativefragilityofspherocytes,postsplenectomypatients
stillhaveamoderatelyincreasedrateofhemolysis.Thus,elevatedindirectbilirubincanpredispose
tobilirubin(pigmented)gallstones.

Defectinredbloodcellmembraneprotein(spectrinorankyrin)causingabnormally
shapederythrocytes.Redbloodcells(RBCs)becomesequesteredinthespleenand
hemolyzed.
Genetics:autosomaldominant
Symptoms:hemolysis(elevatedbilirubinandhaptoglobin,jaundice),splenomegaly,
gallstones,familyhistory
Laboratorytests:increasedosmoticfragility,spherocytesonperipheralsmear,
reticulocytosis,hyperbilirubinemia
Treatment:splenectomy+vaccinationforStreptococcuspneumoniae,Haemophilus
influenzae,Neisseriameningitidis
SummaryBox:HereditarySpherocytosis

Case127
A28yearoldAfricanAmericanmanisplanningtotraveltoIndiaforwork.Heisgiven
quinidineforantimalarialprophylaxis.Severaldayslaterhebecomesfatigued,andaworkup
revealsanormocyticanemia.

1 Whatisthedifferentialdiagnosisforanormocyticanemia?

Hemolyticanemia(autoimmune,druginduced,traumatic,hereditary),hemoglobinopathies,aplastic
anemia,myelodysplasia,andhypothyroidismshouldbeconsidered.

Case127continued:
PeripheralsmearisnotableforHeinzbodies(Fig.126(f0035)).

Figure126
Heinzbodies.
(FromMcPhersonRA,PincusMR:Henry'sClinicalDiagnosisandManagementbyLaboratory
Methods,21sted.Philadelphia,WBSaunders,2006.)

2 Whatisthelikelydiagnosis?
G6PDdeficiencyiscausedbyreducedactivityofG6PD,whichcatalyzestheratelimitingstepinthe
hexosemonophosphatepathway.G6PDdeficiencyisaninherited,Xlinkedrecessivedisorder.It
mostcommonlyaffectsmenofAfrican,Asian,andMediterraneandescent.Itisthemostcommon
inheritedhemolyticanemiaanditisthoughtthat,asinsicklecellanemia,mutationsintheG6PD
genecreateaselectiveadvantagetoheterozygotesbycreatingapoorhabitatinRBCsforthemalarial
merozoite.

3 WhatareHeinzbodiesandhowaretheyformed?
Oxidationofthesulfhydrylgroupsonhemoglobinresultsinclumpingandprecipitationof
hemoglobinwithinthecytoplasmofRBCs.Thesedepositsofinsolublehemoglobinarereferredtoas
inclusionbodiesorHeinzbodies.Theseinclusionbodiesarethenremovedbysplenicmacrophages
astheseRBCspassthroughthespleniccords,producingsocalledbitecells(Fig.127(f0040)).

Figure127

Peripheralbloodsmearinglucose6phosphatedehydrogenasedeficiency.Thearrowshowsabitecellwithpart
oftheredbloodcellmembraneremoved.Theinsetshowsaperipheralbloodsmearwithasupravitalstain
visualizingpunctateinclusionsrepresentingdenaturedhemoglobin(Heinzbodies).
(FromKumarV,FaustoN,AbbasA:RobbinsandCotranPathologicBasisofDisease,7thed.Philadelphia,WB
Saunders,2004,Fig.138insetfromWickramasingheSN,McCulloughJ:BloodandBoneMarrowPathology.
London,ChurchillLivingstone,2003,Fig.88.)

4 Whatisthenormalfunctionofthehexosemonophosphateshuntinredblood
cells?
InRBCs,thehexosemonophosphateshunt(pentosephosphatepathway)isusedprimarilyto
generatethereducedformofnicotinamideadeninedinucleotidephosphate(NADPH).TheNADPH
generatedrecyclesglutathione(viareductionofoxidizedglutathione),andtheglutathioneis
involvedincombatingoxidativedamagebyreactiveoxygenspecies.Failureofthispathwayresultsin
theinabilityofRBCstohandleincreasedoxidativestressesandpredisposesRBCstointravascular
hemolysis.

Case127continued:
Onexaminationyounotescleralicterusandsplenomegaly.Totalbilirubiniselevated.
Urinalysisisnotableforgrossblood,highurinesodium,andmuddybrowngranularcasts.

5 Isthisman'shyperbilirubinemiamostlikelycausedbyconjugatedor
unconjugatedbilirubin?
Hyperbilirubinemiacanbebrokendownintoprehepatic,hepatic,orposthepaticorigin.Hemolytic
anemiaisaprehepaticcauseofelevatedbilirubinandwouldthereforebeexpectedtobecomposed
largelyofunconjugated(indirect)bilirubin.TheincreasedRBCbreakdownresultsinthereleaseof
greaterthanaverageamountsofhemoglobinbreakdownproducts,bilirubin,intotheblood.This
hemoglobinmustcirculatetothelivertobeprocessedbytheliver.Thisprocessingincludes
conjugatingthebilirubintoprotein,makingthecompoundwatersolubleandthereforeeasily
excretedbythekidneys.
Formoreinformationaboutcausesofhyperbilirubinemia,refertoChapter7.

6 Arehishaptoglobinlevelslikelytobehighorlow?
Haptoglobinisaplasmaproteinthatsequestersfreehemeinthecirculation.Therefore,levelsof
haptoglobinwouldbereducedasitgetsconsumedbythelargeamountoffreehemegeneratedvia
oxidativedamageandintravascularRBClysis.

7 Whatrenaldiagnosisdoeshisurinalysisrevealandwhyisthisoccurring?
Whatishisprognosis?

Grossblood,highurinesodium,andmuddybrowncastsareredflagsfordamagetotherenal
architecture.Specifically,muddybrowncastsonurinalysis(UA)arediagnosticforacutetubular
necrosis(ATN).InATN,theacutedropinhemoglobinduetohemolysiscausesischemiaand
necrosisoftheepithelialliningoftherenaltubules.Withouttheepitheliallining,grossbloodescapes
intotheurinaryspaceandconcentratingabilityislost,leadingtohighurinesodium.Themuddy
browncastsarecomposedofthenecrosed,sloughedofftubularepithelialcellspassedintotheurine.
Prognosisisverygoodaslongasthetubularbasementmembraneisintact.Hisrenalfunctionwould
beexpectedtorecoverwithin1to2weeksastheepithelialliningregenerates.

Case127continued:
Youobtainsomefurtherhistoryanddiscoverthatthepatientonceremembershavingbloodin
hisurinewhenhewasachildaftereatingsomefavabeans.

8 Whatistheoffendingagentinthissituation?Whataresomeotheroffenders?
ManyagentsandcompoundscantriggerahemolyticeventinanindividualwithG6PDdueto
increasedreactiveoxygenspeciesandoxidativestress.Manydrugscanprecipitatehemolysisbecause
ofoxidantstress,includingisoniazid,sulfonamides,primaquine,ciprofloxacin,NSAIDs,
nitrofurantoin,vitaminC(acids),andtrimethoprimsulfamethoxazole.Certainfoods,suchasfava
beans,andinfectionscanalsoprecipitatehemolysis.IndividualswithG6PDmustbeveryclosely
monitoredduringinfections,especiallyurinarytractinfections,asmanyofthecommonlyprescribed
medicationsforEscherichiacolicystitisareincludedintheprecedinglistofcontraindicated
medicationsduetoincreasedriskofhemolysis.
Theclassicclinicalvignetteonboardsforglucose6phosphatedehydrogenase(G6PD)
deficiencyisablackindividualwhoservesasamissionaryworkerinaremoteareaandis
treatedformalariapriortotheappearanceofhemolyticanemia.However,beonthelookout
forsomeoftheseotherpharmacologictriggersofhemolysisinG6PDdeficientpatients.
Step1Secret

9 Whatisthetreatmentforthisman'scondition?
FortheUSMLE,alwaysconsidertheleastinvasiveinterventionfirst.Inthiscase,theoxidant
stressor(quinidine)mustbeimmediatelydiscontinuedtopreventadditionalhemolysis.Hydration
withnormalsalinemaybenecessarytokeepintravascularvolumeup.ConsiderapackedRBC
transfusionifthepatientishypoxemicandsymptomatic.Acutecriseswillresolvespontaneouslyin
about1weekasnewerythrocytesareproducedwithincreasedG6PDactivity.

Case127continued:
Twodayslaterthisman'shemoglobinisstill9.7g/dL,buthisMCVisnow95fL.

10 Whymightthemeancorpuscularvolumebeslightlyelevatedinthismanand
howdoesthisreflecttheselflimitednatureofthishemolyticanemia?
Acompensatoryerythrocytosisinresponsetohemolysisproducesincreasednumbersofcirculating
reticulocytes,whichareprematureRBCsthataremuchlargerthanmatureRBCs.Thisexplainsthe
increasedMCV,whichisareflectionofRBCsize.Furthermore,itturnsoutthatG6PDactivityin
reticulocytesismuchhigherthaninmatureRBCs.AsomewhatselectivedestructionofolderRBCs
thereforeoccursinG6PDdeficientpatientswhoareexposedtooxidativestress.Thishelpstolimit
thenatureofthehemolyticcrisis,evenifmildexposuretotheoxidativestressorcontinues.

11 Supposethismanpresentedwithasimilarepisodebutalsocomplainedof
severebackandabdominalpain.Whatwouldyoubeconcernedaboutinthis
caseandwhy?
Mesentericischemiaandrenalischemiacanoftenbecomplicationsofacutehemolyticcrisesin
G6PDdeficientpatientsbecausetheirhemoglobindropstoorapidlyforcompensationtotakeplace.
Treatmentissupportiveunlesssignsofperitonitisdevelop,inwhichcasethepatientmayrequire
surgerytoremovepotentiallynecroticbowel.

Cause:enzymedefectinthehexosemonophosphatepathwayresultinginhemolysis
whenexposedtooxidantstresses
Themostcommonmetabolicdisorderofredbloodcells(RBCs)
XlinkeddisorderaffectsAsian,African,Mediterraneanmen
Precipitants:infection,acidosis,antimalarialdrugs,sulfadrugs,favabeans
Symptoms:jaundice,darkurine,acutetubularnecrosis(ATN),anemia,organomegaly
fromchronichemolysis
Complications:mesentericandrenalischemia
Laboratorytests:lowhemoglobinandincreasedreticulocytes,lowhaptoglobin,highl
lactatedehydrogenase(LDH),elevatedbilirubin(indirect),negativeCoombstests,
urinalysis(UA)showinghemoglobinuriaandmuddybrowngranularcasts
Treatment:supportiveremovaloftheoffendingagent,administrationoffluidsand
transfusionifnecessary
SummaryBox:Glucose6PhosphateDehydrogenaseDeficiency

Case128

A1dayoldnewbornhasjaundicethatstartedonhisfaceandspreadtohisbody.Physical
examinationrevealsscleralicterusandconjunctivalpallorandhepatosplenomegaly.Thereis
nocephalohematoma.Deliverywasuneventfulexceptforthefindingthattheplacentawas
moderatelyenlarged.

1 Whataresomecausesofjaundiceintheneonate?
Infection,physiologicjaundice,intestinalobstruction,inbornerrorsofmetabolism,andhemolytic
diseaseofthenewborncancausejaundice.

2 Whatisthemostseriouscomplicationofneonataljaundiceandhowdoesit
develop?
Kernicterusisthedepositionofinsolubleunconjugatedbilirubininthebrain,whichcancausebrain
damage(particularlyinthebasalgangliaandhippocampus).Earlysignsincludelethargy,poor
feeding,vomiting,andhypotonia.Latersymptomsincludeirritability,hypertonia,seizures,and
deafness.Infantswiththeirfirstcaseofjaundiceareathighestriskforkernicterus.Riskfactors
includeprematurity,sepsis,Asianancestry,hemolyticdisease,andhighaltitude.

Case128continued:
Laboratorytestsaresignificantforahemoglobinof12g/dL(lowforanewborn!)andamarked
reticulocytosisandelevatedindirectbilirubin.AdirectCoombstestispositive.Bloodtyping
revealsthatthemotherisRhnegative,thefatherisRhpositive,andthebabyisRhpositive.

3 Whatisyourdiagnosis?Whatisthepathophysiologyofthisdisease?
HemolyticdiseaseofthenewbornduetoRhincompatibility.ParentalheterozygosityallowsanRh
positiveinfanttobecarriedbyanRhnegativemother.Maternalbloodcomesintocontactwithfetal
bloodcells,andmaternalantibodiesareproducedagainsttheRhantigenpresentonthefetalblood
cellsurface.DuringasubsequentpregnancywithanRhpositivefetus,maternalIgGantibodiescan
crosstheplacentaandbindtofetalRBCs,leadingtohemolysisviaopsonizationandcomplement
mediateddestruction.DestructionoffetalRBCscausesincreasedunconjugatedbilirubininthefetal
circulation,whichismetabolizedbytheplacentawhilethefetusisstillinutero.Afterdelivery,
however,theinfantmustprocesstheunconjugatedbilirubininhisownimmaturehepatocytes.
Becausetheuridinediphosphate(UDP)glucuronyltransferaseactivityisnotyetmaximalattimeof
delivery,theinfantisfunctionallyincapableofhandlingthehighbilirubinlevelcreatedbythe
hemolyticanemiaofRhincompatibility.Severeanemialeadstoextramedullaryerythropoiesis,
whichresultsinhepatosplenomegalythatispotentiallyvisibleonprenatalultrasound.

Case128continued:
Thisbabyhasa3yearoldsisterwhoisalsoRhpositive.Shewasasymptomaticasanewborn.

4 Whywastheoldersisterunaffected?
ThesisterwasthefirstchildandprobablysensitizedthemothertotheRhantigenwhentherewas
mixingofmaternalandfetalbloodduringdelivery.Sensitizationcausedthemothertoproduceanti
Rhantibodies,whichcancrosstheplacenta.Priortoherfirstpregnancy,themotherdidnotmake
antiRhantibodies,sononecouldcrosstheplacentaandcausehemolysisinthefirstchild.

5 Whatshouldhavebeengiventothemotherpriortodeliveryofherfirstchild?
AntiRhimmuneglobulin(RhoGAM)canbegiventoprovidethemotherwithpassiveimmunity
againsttheRhantigen.ThisantibodybindstoanyfetalRhantigensthatmayenterthematernal
circulationduringdeliveryandsopreventsthemother'simmunesystemfromeverrecognizingthe
Rhantigen.WithoutsocalledsensitizationoftheimmunesystemtotheRhantigen,noantiRh
antibodiesareeversynthesized.Thus,thereisnoantiRhIgGavailabletocrosstheplacentaand
opsonizethefetalRBCs.
Note:RhoGAMshouldbegiventoanRhnegativewomaninanysituationinwhichthereis
potentialformixingofmaternalandfetalblood,includingspontaneousabortions,electiveabortions,
andabruptioplacentae(abruptionplacenta).

Case128continued:
Thebabyistreatedwithphototherapyandexchangetransfusions.Furthertestingrevealsthat
themother'sbloodtypeisO,thefather'sisA,andthebaby'sisA.

6 WhyisABOincompatibilitysounlikelytobethecauseofhemolyticdiseaseof
thenewborn?
AntiAandantiBantibodiesarepredominantlyIgMantibodies,whichcannotcrosstheplacenta.
TherearealsomultipleothercellsthatexpresstheAandBantigensinthefetus,andthesecells
mopupmostofanyantiAorantiBantibodiesthatcrosstheplacenta.

7 SupposeanRhpositivebabyborntoanRhpositivemotherdeveloped
jaundiceonthethirddayoflife.Whatisyourdiagnosisnow?
Physiologicjaundice.

8 Whatisphysiologicjaundiceandwhydoesitdevelop?
PhysiologicjaundiceresultsfromtheincreaseddestructionofRBCswithfetalhemoglobinduringthe
newbornperiod.Thesecellsareslowlybeingreplacedbycellswithadulthemoglobinoverthefirst6
monthsoflife.Normalhemoglobininthenewbornisnormally14to20g/dL,whichhelpsto
compensateforthedecreasedpartialpressureofO2availabletofetalRBCsinutero.Atbirth,with
theonsetofrespirations,erythropoiesisslowsdownastherelativehypoxemiaisreduced.Thefetal
RBCalsohasadecreasedsurvivalratecomparedwiththeadultRBC.Withinthefirst3months,
bloodvolumeincreasesmarkedly.AllofthesefactorsleadtoincreasedRBCdestruction,adecreased

hemoglobinconcentrationandclinicallyvisiblejaundicethatpeaksondays3to4oflifeandstartsto
improvebydays4to5.Itisthoughtthatinfantswhodevelopphysiologicjaundicedonotyethave
thehepaticcapacitytocleartheexcessbilirubinthatformsduringthisperiod.AstheactivityofUDP
glucuronyltransferaseincreasesoverthefirstweeksoflife,thejaundiceusuallybecomesselflimited.
Alsonotethatthoughthisisaclinicalcondition,nearlyeverynewbornwilldemonstrateamild
degreeofphysiologicjaundice.Phototherapyisindicatedforthosenewbornswhodevelopbilirubin
levels>25mg/dL,topreventkernicterus.Specificultraviolet(UV)wavelengthlightdirectedatthe
skinofthesenewbornsallowsforaconformationalchangeintheunconjugatedbilirubinthatallows
ittobemorewatersoluble,effectivelyskippingthehepaticmetabolismtoallowforrenalexcretion
ofthebilirubin.

Erythroblastosisfetalis
CausedbymaternalsensitizationtoRhantigensinanRhnegativemotherwiththe
productionofantiRhantibodies,whichcancrosstheplacentaandcausehemolysisof
fetalredbloodcells(RBCs)inanRhpositiveinfant
Sensitization:requirespreviousexposuretoRhpositivebloodfromafirstpregnancy,
abortion,placentalabruption,orprevioustransfusion
Symptoms:largeplacenta,elevatedindirectbilirubin,rapidlyprogressivejaundiceafter
birth,hepatosplenomegaly,generalizededema(ascites,scalpfluid,purpura,cyanosis,
abductionoflimbs,lossoflimbflexion)
Laboratorytests:positivedirectCoombstest,hyperbilirubinemia
Treatment:exchangetransfusionstotheinfant
Prophylaxis:testingthemother'sbloodtypeearlyinpregnancyandadministering
RhoGAMduringandimmediatelyafterdeliverytopreventalloimmunization
SummaryBox:HemolyticDiseaseoftheNewborn

Case129
A2yearoldboyisbroughttoyourofficebyhismotherforrecurrentstomachpainthatcomes
andgoesbuthasbeenworseningoverthepastfewweeks.Upontakingafullsocialhistory,you
learnthattheboyandhisfamilyrecentlymovedintotheareaandarelivinginhis
grandmother'shome.Becauseheisanewpatient,youperformafullphysicalexamination,
includingaCBCtoruleoutinfectionorinflammation.Pertinentfindingsincludeguaiac
negativestool.Hemoglobinis8.5g/dL.MCVis75fL.Aperipheralbloodsmearisshownin
Figure128(f0045).

Figure128
PeripheralbloodsmearofpatientinCase129(s0750).
(FromMcPhersonRA,PincusMR:Henry'sClinicalDiagnosisandManagementbyLaboratory
Methods,21sted.Philadelphia,WBSaunders,2006.)

1 Whatisthelikelydiagnosis?
Leadpoisoningmustberuledoutintheworkupofrecurrentabdominalpainwithanemiaina
pediatricpatient.Inthiscase,wecanassumethatthesourceoftheleadisfromleadbasedpaintin
thepatient'snewhome.Rememberthathomesbuiltbefore1978typicallycontainedleadbased
paint.Thoughtheageofthehouseisnotspecificallystated,youdonotneedthisinformationto
suspectleadpoisoning,giventhelaboratorytestsandperipheralsmearfindings.Thebloodsmear
showsbasophilicstippling,whichisseeninleadtoxicityduetoaccumulationofleadintheRBCs.
Also,rememberthegeneraldevelopmentalmilestones.Aboythisage(24months)willput
everythinginhismouth,includingchipsofpaintthatmaybepeeling.

Case129continued:
Thechildstartedwalkingat12months,butrecentlyhehasbecomeunsteadyonhisfeet.His
speechhasalsoregressed.Anxrayfilmofhislegshowsathicktransverseradiodenselinein
themetaphysis.

2 Whyisthisboyataxic?
Acuteencephalopathycausedbytheleadaccumulationinneuronscanleadtohisunsteadygaitand
lossofspeech.Leadpoisoningblocksthefunctionofporphobilinogensynthaseleadingtothe
accumulationofaminolevulinicacidinthehemesynthesispathway.Thisaminolevulinicacidis
thoughttobeneurotoxicbymimicryofthefunctionofaminobutyricacid(GABA).These
symptomsmaybereversiblewithchelationofleadfromthechild'sbodyifcaughtearly,butmay
becomeirreversibleifleftuntreated.

3 Whataretheradiographicfindings?
Leadlinesarevisibleinareasofgrowthincludingthemetaphysisofbones(Fig.129(f0050)).
Remember,leadaccumulatesinrapidlydividingcellssuchasthemetaphysesoflongbonesand
RBCs.

Figure129
Leadlinesinthedistalradius.
(FromFordMD,DelaneyK,LingL,EricksonT:ClinicalToxicology.Philadelphia,WBSaunders,2001.)

4 Howdoesleadtoxicitycauseanemia?
Leadisanonessentialmetalthatbindsirreversiblytosulfhydrylgroupsofproteins,including
hemoglobin.Leadinterfereswithenzymesinvolvedinhemeproduction(porphobilinogensynthase)
andimpairsproperironutilization.Iffunctionalhemoglobincannotbeproduced,thepatientwillbe
functionallyanemic.Treatmentincludesenvironmentalcontrolandchelationathigherleadlevels
withethylenediaminetetraaceticacid(EDTA)orsuccimer.
SignsofleadpoisoningcanberememberedusingthemnemonicABCDEFG:
Anemia
Basophilicstippling
Colickypain
Diarrhea
Encephalopathy
Footdrop
Gums(leadline)

Step1Secret

Variantofirondeficiency:interfereswithironutilizationandhemoglobinsynthesis
Etiology:leadcontainingpaint,pica
Symptoms:abdominalpain,acuteencephalopathy,leadlines
Laboratorytests:microcytichypochromicanemia,basophilicstippling,increasedserum
leadlevelandprotoporphyrin
SummaryBox:LeadPoisoning

Copyright2015Elsevier,Inc.Allrightsreserved.

BOOKCHAPTER

BacterialDiseases
ThomasA.BrownMDandSonaliJ.Shah
USMLEStep1Secrets,Chapter21,631673

Thereisnobetterwaytosayit:theUSMLElovesbacterialdiseases!Thisisoneofthehighest
yieldsubjectsontheexamination,soyoumustknowitwell!Ourbookhasdivided
microbiologyintotwochapters,butyoushouldnotethatthebreakdownoftheexaminationis
notlikelytobeevenlydistributedamongbacteria,viruses,fungi,andparasites.Bacterial
diseasesaretestedfarmorecommonlythantheotherthreetypes,butrecently,fungaldiseases
havebeenheavilyrepresentedonmanystudentsforms.Fungaldiseasesarediscussedmorein
Chapter22.
Asyoumayknow,microbiologyisnotinherentlydifficult,butitdoestaketimetolearn.The
mosteffectivewaytostudyformicrobiologyontheUSMLEistointroduceyourselftothis
materialearlyon,preferablyduringyourmicrobiologyclassinmedicalschool.Thisisone
subjectforwhichmultipleresourcesmaybequitehelpfultoyou.Forthoseofyouseekingto
combineyourmedicalschooleducationwithboardsstudying,werecommendusingClinical
MicrobiologyMadeRidiculouslySimpleandMicrocardswhenyoufirstbeginlearningthe
material.Pullthehighestyieldfactsfromthesealreadyhighyieldmaterialsandwritethem
intoFirstAid.YoucanthenstudyfromyourannotatedcopyofFirstAidandthecasesinthis
bookonceyourfocusshiftsentirelytoboards.
Howshouldyoubeexpectedtoknowwhichfactsarethemostimportanttolearnforboards?
Thatiswhyyoupurchasedthisbook!Asalways,wewillbepointingthisinformationoutalong
theway.However,youshouldkeepinmindthattheUSMLEwillexpectyoutoknowthemajor
diseasesandtoxinsassociatedwitheachandeverymedicallyimportantbacterialspecies.The
Step1placesheavyemphasisonthemechanismsofvariousbacterialtoxinsaswellasthe
associatedcharacteristicsofindividualbacterialspeciesthatcanbehelpfulinidentifyingand
differentiatingamongtheminthelaboratory.
Insider'sGuidetoBacterialDiseasesfortheUsmleStep1

BasicConcepts
1 Whatmakesanorganismgrampositiveorgramnegative?

Bothgrampositiveandgramnegativeorganismshaveaninternalcellmembraneandcellwalls
madeofpeptidoglycan.However,gramnegativebacteriahavemuchthinnercellwallsand,in
addition,haveanoutermembraneoutsidethecellwall.Grampositiveorganismshavetechoicacid
intheircellwalls,andgramnegativeorganismshavelipopolysaccharide(endotoxin)(Fig.211
(f0010)).

Figure211
Structureofthecellwallingrampositiveandgramnegativebacteria.A,Grampositivebacteriahaveathick
peptidoglycanlayerthatcontainsteichoicandlipoteichoicacids.B,Gramnegativebacteriahaveathin
peptidoglycanlayerthatisconnectedbylipoproteinstoanoutermembrane.LPS,lipopolysaccharide.
(FromRosenthalK,TanJ:RapidReviewMicrobiologyandImmunology,2nded.Philadelphia,Mosby,2007.)

2 Whyaregramnegativeinfectionsmorelikelytoproducebacterialsepsis?
Theoutermembraneofgramnegativeorganisms(seepreviousquestion)containslipidA,an
endotoxinthatispartofthelipopolysaccharideinthecellwallofgramnegativebacteria.LipidA
getsreleaseduponbacterialdeathandhaspotentproinflammatoryeffects.

3 DescribethemechanismbywhichlipidAcausestoxicity?

LipidAactivatesmacrophagestosecreteinterleukin1(IL1)andtumornecrosisfactor(TNF),both
ofwhicharereferredtoasacutephasecytokines.LipidAalsostimulatesthereleaseofnitricoxide
(NO)fromendothelialcells.LargeamountsoflipidAmayleadtoshockandintravascular
coagulationviathisstimulatoryeffect(Fig.212(f0015)).

Figure212
MechanismbywhichlipidAcausestoxicity.
(FromBrownTA,BrownD:USMLEStep1Secrets.Philadelphia,Hanley&Belfus,2004.)

4 Whatareexotoxins?
Exotoxinsareproteinsreleasedbybothgrampositiveandgramnegativebacteriaduringtheir
normallifecycle.Exotoxinsreleasedintofoodcancausepoisoning,asinBacilluscereusand
Staphylococcusaureusfoodpoisoning.PyrogenicexotoxinsreleasedbyS.aureusandStreptococcus
pyogenescancauserash,fever,andtoxicshocksyndrome.Enterotoxinsactonthegastrointestinal
system,whereasneurotoxinsactonnervesormotorendplates.Forexample,infectiousdiarrheais
causedbyenterotoxinsreleasedbyVibriocholerae,Escherichiacoli,Campylobacterjejuni,and
Shigelladysenteriae.

5 Whatisacapsuleandwhatpurposedoesitserve?
Certainspeciesofbacteriaproduceaslipperyoutermostcoveringcalledacapsule.Thiscovering
consistsofhighmolecularweightpolysaccharides,whichhelpthebacteriatoevadephagocytosisby
neutrophilsandmacrophages.NotethatBacillusanthracishasaproteinaceouscapsulewhich
consistsofdglutamicacid.Thecapsuleisnotessentialforgrowthandservesonlyinaprotective
capacity.ThemostcommonmedicallyrelevantencapsulatedorganismsareStreptococcus
pneumoniae,Klebsiellapneumoniae,Haemophilusinfluenzaetypeb,Pseudomonasaeruginosa
,Neisseriameningitidis,andCryptococcusneoformans(afungus).
RememberthatSomeKillersHavePerfectlyNastyCapsules.Thismnemonicwillhelpyourecall
theencapsulatedorganismsthatareimportanttoknowforboards.
Note:IntheQuellungreaction,whichtestsforthepresenceofencapsulatedbacteria,encapsulated
bacteriawillswellwhenexposedtospecificantibodies.ThelatexagglutinationassayandIndiaink
stainaretwoothermethodsfordetectingcapsularpresence.

AlthoughtheQuellungreactionandseveralothertechniquesinthisbookmaybeclinically
outdated,youshouldrememberthatmanyofthephysicianswhoauthorboardsquestionswill
haverelieduponthistechnologyduringthecourseoftheircareersandwillthusexpectyouto
knowthenamesandbasicprinciplesbehindthesetests.Asageneralrule,youshouldfocuson
learningthetechniqueslistedinthisbookandinFirstAid.Youarenotexpectedtoknow
complexorcuttingedgetechnologiesthatarenotmentionedinyourUSMLEstudyresources.
Step1Secret

6 Whatsortofindividualsaresusceptibletoinfectionbyencapsulatedbacteria?
Becausethespleennormallysequestersencapsulatedbacteria,patientswhohaveundergone
splenectomyareatagreaterriskforincurringinfectionbyencapsulatedbacteria.

7 IdentifytheGramstainandthemorphologyoftheorganismsinTable211
(t0010)
Table211
BacterialImages

Image

GramStainandMorphology
Grampositivecocciinpairs:
S.pneumoniae

Gramstainofasputumsample
infectedwithStreptococcuspneumoniae.
Grampositivecocci
(stainingpurpleinacolor
image)inchains:
Staphylococcusaureus

Expectoratedsputumwith
gramnegativerodsinapatientwithKlebsiellapneumoniae
pneumonia.
Gramnegativecocci
(stainingpinkinacolor
image)inpairs:Neisseria

spp.

Sputumsmear,stainedwith
Gramstain,showsmanyneutrophilsandintracellulargramnegative
diplococci,suggestiveofNeisseriameningitidisinfection(oil
immersion).
Gramnegativerods:many
possibilities

KlebsiellapneumoniaeimagefromMandellGL,BennettJE,DolinR:Mandell,Douglas,andBennett'sPrinciples
andPracticeofInfectiousDiseases,7thed.Philadelphia,ChurchillLivingstone,2009.ImagesofStreptococcus
pneumoniae,Neisseriameningitidis,andgramnegativerodsfromMcPhersonRA,PincusMR:Henry'sClinical
DiagnosisandManagementbyLaboratoryMethods,22nded.Philadelphia,WBSaunders,2011.

8 CoverthetwocolumnsontherightinTables212through219(t0015)totest
yourknowledgeofthepropertiesoftheclinicallyrelevantbacterialistedinthe
leftcolumn
Table212
GramPositiveCocci

Organism

Associated
Disease(s)

PearlstoRemember

Staphylococcus
aureus

Cellulitis
Acuteendocarditis(in
previouslynormal
valve)
Osteomyelitis
Pneumonia
Carbuncles/furuncles
Stye(hordeolum)

Toxinmediateddiseases:
Staphylococcaltoxicshock
Scaldedskinsyndrome
Staphylococcalgastroenteritis
Note:Likeallstaphylococcalspecies,S.aureusis
catalasepositivebutitalsoiscoagulasepositive.

Staphylococcus
epidermidis

Prostheticvalve
endocarditis

Normalskinflora
Novobiocinsensitive

Staphylococcus
saprophyticus

Cystitisinyoung
women
Secondmost
commoncauseof
UTI(behindE.coli)

Novobiocinresistant

Streptococcus
Neonatalpneumonia, Normalvaginalflora
agalactiae(groupB meningitis,sepsis
Hemolysis
streptococci)
Chorioamnionitis
Bacitracinresistant
Streptococcus
pneumoniae

Pneumonia
Meningitis
Sinusitis
Otitismedia

Bilesoluble
Optochinsensitive
Hemolysis
SeeCase211formoredetails

Streptococcus
pyogenes(groupA
streptococci)

Pharyngitis
Impetigo
Erysipelas

Remainslargelysensitivetopenicillin
Hemolysis
Bacitracinsensitive

Cellulitis
Necrotizingfasciitis
Rheumaticfever
Poststreptococcal
glomerulonephritis
Enterococcusspp.

Viridans
streptococci

UTI

Partofnormalbowelflorathatcausesdiseasewhen

Bacteremia/sepsis
Endocarditis

hostisimmunocompromisedorgastrointestinaltract
hasbeenbreached

Abdominalabscess

orHemolysis

Dentalcaries(S.
mutans)

Normaloralflora
Hemolysis

Subacutebacterial
endocarditis(S.

Optochinresistant

sanguis)
UTI,urinarytractinfection.
Table213
GramPositiveBacilli

Organism

AssociatedDisease(s)

PearlstoRemember

Bacillus
anthracis

Cutaneousanthrax(most
commonform)

Painlessblackescharswithcutaneousanthrax
Woolsortersatriskforpulmonaryanthrax

Pulmonaryanthrax

(woolsorter'sdisease)

Sporeforming
Corynebacterium Diphtheria
spp.
Granulomatous

Normalskinflora
Pseudomembraneoresophagealweb

lymphadenitis
Pneumonitis

Toxincausesdiseaseandisencodedby
prophage

Pharyngitis
Skininfections

Metachromaticgranules
ADPribosylationofEF2

Endocarditis
Listeria
monocytogenes

Listeriosis

Perinatal/neonatalinfections
Immunocompromisedpersonsatrisk
Rawmilkanddairyproducts

ADP,adenosinediphosphateEF2,elongationfactor.
Table214
GramNegativeCocci

Organism

AssociatedDisease(s)

PearlstoRemember

Neisseria
meningitidis

Meningitis
Septicemia

Hasacapsule
Purpuricnonblanchingrash

(meningococcus) WaterhouseFriderichsen
syndrome

Vaccineavailable
SeeCase218

Neisseria
gonorrhoeae

Infectssuperficialmucosal
surfaceslinedwithcolumnar

Novaccine
Maincauseofinfectiousarthritisinsexually

(gonococcus)

epithelium:
Urethra:urethritis

activepersons
Prepubescentvaginalepitheliumiscolumnar

(gonorrhea)
Vagina:vulvovaginitisin

becausenotyetactedonbyestrogento
becomesquamous

younggirls
Rectum:proctitis
Conjunctiva:ophthalmia
neonatorum
Table215
EntericGramNegativeRods

Organism

AssociatedDisease(s)

Campylobacter Enteritis
jejuni

PearlstoRemember
Presentinanimalfeces

Escherichia
coli

Enteritis
UTI

Normalgutflora
E.coliO157:H7aparticularlyvirulent

Meningitis
Peritonitis

pathologicstrainassociatedwithHUS
SeeCase212

Mastitis
Septicemia
Gramnegativepneumonia
HUS
Salmonella

Foodborneillness

Osteomyelitisinpatientswithsicklecell

spp.

Typhoidfever(Salmonellatyphi
)

anemia

Shigellaspp.

Shigellosis(bacterialdysentery)

Bloodydiarrhea
Fecaloralrouteoftransmission
Lowinoculumrequired
Toxinmediated

Helicobacter
pylori

Pepticulcerdisease
Gastritis

Positiveureabreathtestduetopresenceof
enzymeurease

Duodenitis
Gastriccancer

Livesinstomachbutcommoninduodenal
ulcers

Mucosaassociatedlymphoid
tissue(MALT)lymphoma

Tripletreatment:amoxicillin,clarithromycin,
andprotonpumpinhibitor

HUS,hemolyticuremicsyndromeUTI,urinarytractinfection.
Table216
OtherGramNegativeRods

Organism

Associated
Disease(s)

PearlstoRemember

Bordetella
pertussis

Pertussis
(whoopingcough)

Highlycontagiousspreadbycoughingandnasaldrops

Brucellaspp.

Brucellosis(also

Transmittedviacontaminatedorunpasteurizedmilk

calledundulant
fever)
Francisella
tularensis

Tularemia(rabbit
fever)

Reservoirinrabbitstransmittedbytick
Symptoms/signssimilartothoseofplague
Culture,drainagecontraindicatedowingtohighvirulence

Haemophilus

Meningitis(typeb) Typebencapsulatedandmorevirulent

influenzae

Bacteremia
Cellulitis

Vaccineavailablefortypebstrain

Pneumonia
Sinusitis
Pseudomonas Pneumoniain
aeruginosa
cardiacfailure

ThinkPseudomonasinfectioninburnpatientsand
intravenousdrugusers

patients
Externalotitis

Cancauseblackskinlesions
Culturesmakebluegreenpigment

Osteomyeltisin
diabetics

HasendotoxinA
Resistanttomanyantibiotics

Endocarditis
UTI
Hottubfolliculitis
Legionella

Legionnaire's

Legionnaire'sdisease:acutepneumoniawithmultisystem

pneumophila

disease
Pontiacfever

involvementfromwatersource,sonopersontoperson
spread
Pontiacfever:similartoflu

Yersinia
pestis

Bubonicplague

Transmittedbyfleasfromrodentstohumans
Blackbuboes

Yersinia
enterocolitica

Enterocolitis

Pseudoappendicitis
Seeninnurseryschools

Table217
Anaerobes

Organism

Associated

PearlstoRemember

Disease(s)
Clostridium

Anaerobiccellulitis

Crepitusisassociatedwithgasgangrene

perfringens

Gasgangrene
(myonecrosis)

Alphatoxin(lecithinase)

Foodpoisoning
Clostridium

Tetanus

tetani

Clostridium
botulinum

Exotoxinthatcausesspasticparalysisbyblockingglycine
releasefromRenshawcellsinspinalcord
Vaccineisavailable

Botulinism

Foodpoisoningthatcausesflaccidparalysis
PreformedtoxinpreventsreleaseofACh
Classicscenariofromconsumptionofdentedcannedgoods
orhoney

Clostridium

Pseudomembranous Causedbyantibioticuse,especiallyclindamycinorampicillin

difficile

colitis

Treatwithmetronidazoleororalvancomycin

ACh,acetylcholine.
Table218
Spirochetes

Organism

Associated

PearlstoRemember

Disease(s)
Borrelia

Lyme

burgdorferi

disease

Borrelia
recurrentis

Relapsing
fever

Organismswitchessurfaceproteinstoevadeimmuneresponse,
leadingtointermittentfevers

Treponema
pallidum

Syphilis

SeeCase214

Leptospira

Leptospirosis Transmittedbywaterthatiscontaminatedbyanimalurinethrough

interrogans

SeeCase215

cracksintheskin,eyes,ormucousmembranes

Table219
IntracellularOrganisms

Organism

Associated

PearlstoRemember

Disease(s)
Mycoplasma

Atypical

Nocellwall

pneumoniae

(walking)
pneumonia

Treatwithmacrolides
BloodshowsIgMcoldagglutinins
Chestradiographdemonstratesdiffuseinterstitialinfiltrates
radiographicchangesoftenmoreextensivethanexpected
frompatient'ssymptoms

Chlamydia
trachomatis

Urethritis
Pelvic
inflammatory
disease
Blindness
Lymphogranuloma
venereum
Neonatal

SeeCase217
Treatneonateswitherythromycineyedropsforconjunctivitis

conjunctivitis
Chlamydia
psittaci

Psittacosis(flulike Transmittedfrombirddroppingsviaaerosol
syndrome)

Chlamydia
pneumoniae

Atypical
pneumonia

Mycobacterium Tuberculosis

Transmittedviaaerosols

SeeCase216

tuberculosis
Mycobacterium Leprosy(Hansen's Tuberculoidform:milderwithfeworganismsinlesions
leprae

disease)

Lepromatousform:severewithmanyorganismsinlesions
Growsincooltemperatures,soaffectsdistalsites
Treatwithdapsone

Rickettsia
rickettsii

RockyMountain
spottedfever

Rashthatstartsonpalmsandsolesandmigratescentrally
(centripetalmigration)
WeilFelixtestresultspositiveforrickettsialdiseases

Basicconceptsinantibacterialpharmacology
1 Whatarethelactamantibioticsandwhatistheirmechanismofaction?
Thelactamantibioticsincludethepenicillins,cephalosporins,andcarbapenems(imipenem,
meropenem).Byvirtueoftheirlactamchemicalmoiety,theyallinhibitbacterialcellwall
synthesis.Resistancetotheseantibioticsismediatedbybacteriallysynthesizedlactamaseenzymes
thatdestroythelactamring(Fig.213(f0020)).

Figure213
Generalstructureofpenicillinsandcephalosporins.
(FromRosenthalK,TanJ:RapidReviewMicrobiologyandImmunology,2nded.Philadelphia,Mosby,2007.)

2 Whyareclavulanicacidandsulbactamaddedtosomepenicillins?
Theseagentsinhibitlactamase,therebyreducingresistanceofbacterialspeciestothepenicillins.

3 Whatistheantibacterialspectrumofthevarioussubclassesofpenicillinsand
cephalosporins(Table2110(t0055))?

About10%ofpeoplereceivingpenicillinwillhaveahypersensitivityreaction.Approximately10%to
20%ofpeoplewithapenicillinallergywillalsohaveahypersensitivityreactiontocephalosporins.
Thereisnocrossreactivitybetweenpenicillinsandaztreonam.
Table2110
Lactams

DrugClass

Examples

Coverage

Naturalpenicillins

PenicillinV(oral)

Mostlygrampositive

PenicillinG(intravenous)
Benzathinepenicillin
Extendedspectrum

Ampicillin

Grampositiveandincreasedgram

penicillins

Amoxicillin(oral)

negative

Antistaphylococcal

Dicloxacillin

Staphylococcusaureus

penicillins

Cloxacillin

Antipseudomonal

Ticaracillin

Increasinggramnegativecoverage,

penicillins

Piperacillin

includingPseudomonas

Penicillinplusbeta
lactamaseinhibitor

Ampicillinsulbactam
(Unasyn)

Lactamresistantbacteria

Amoxicillinclavulanicacid
(Augmentin)
Piperacillintazobactam
(Zosyn)
Firstgeneration
cephalosporins

Cephalexin
Cefotetan

Mostlygrampositive

Cefazolin
Secondgeneration

Cefuroxime

cephalosporins

Cefaclor

Mostlygrampositive

Cefoxitin
Thirdgeneration

Ceftazidime

Penetratesthebloodbrainbarrier

cephalosporins

Ceftriaxone

CeftazidimecoversPseudomonas

Aztreonam

Aztreonam

Gramnegativerods

Noticethatthefirstgenerationcephalosporinsaresimilarinspectrumtothenaturalpenicillins,and
thatthethirdgenerationcephalosporinsaresimilarinspectrumtotheextendedspectrum
penicillins.Knowingthisgeneralpatternhelpsinunderstandingselectionofantimicrobialtherapy.

4 Whatistheantibacterialspectrumofthefluoroquinolonesandwhatistheir
mechanismofaction?
Thisclasshasabroadspectrumofactivity,includingbothgrampositiveandgramnegative
organisms.TheyalsocoverPseudomonas,makingthemfairlysimilarinspectrumtothe
antipseudomonalpenicillins.TheseantibioticsworkbyinhibitingbacterialDNAsynthesisby
inhibitingthebacterialtopoisomerase(DNAgyrase)protein.

5 Whatisthespectrumandmechanismofactionofthemacrolides?
Theseantibioticshavegoodgrampositivecoverage,andseveralmembersofthisclassareeffective
againstintracellularorganisms.Theyworkbyinhibitingbacterialproteinsynthesis.

6 Whatisspecialaboutthetetracyclines?
Thesedrugsarethemostimportantagentsforthetreatmentofintracellularorganisms.Theyalso
workbyinhibitingbacterialproteinsynthesis.

7 Whatarethemechanismofactionandspectrumoftheaminoglycosides?
Theseareirreversibleinhibitorsofproteinsynthesisthataregenerallyonlyeffectiveagainstgram
negativerods.However,theymaybeusedincombinationwithpenicillinsforenterococcal
endocarditis(agrampositiveorganism).

8 Howdoeschloramphenicolworkandwhyisitnotusedmoreoften?
Thisantibioticalsoinhibitsproteinsynthesis,butbecauseoftheriskofaplasticanemiaitisnot
commonlyusedinindustrializednations.

9 Whyistrimethoprimcommonlygivenincombinationwithsulfamethoxazole,
asTMPSMX?
Becausetheseagentsinhibitfolicacidsynthesisatdifferentsteps,theircombinationissynergistic.

10 CoverthetwocolumnsontherightofTable2111(t0060)anddescribethe
mechanismofactionandmechanismofbacterialresistanceforthe
antimicrobialagentslistedintheleftcolumn
Table2111
MechanismsofDrugAction/Resistance

DrugClass

MechanismofAction

MechanismofResistance

Penicillin,

Inhibittranspeptidaseandstimulationof

Formationoflactamases

cephalosporin,

autolysins

thatbreakthelactamring

Inhibitscellwallsynthesisbybindingdalanine

dAlaninereplacedwithd
lactate

aztreonam
Vancomycin

Tetracyclines

Bindtothe30Ssubunitofthebacterialribosome, Decreasedtransportintothe
inhibitingproteinsynthesis
cellandincreasedtransport
outofthecell

Aminoglycosides Impairsproperassemblyoftheribosome,
causingthe30Ssubunittomisreadthegenetic

Acetylation,adenylation,or
phosphorylation

code
Clindamycin

Binds50Ssubunittopreventpeptidebond
formation

Chloramphenicol Reversiblyinhibitsproteinsynthesisbybindingto Acetylation


the50Ssubunit
Linezolid

Bindsto50Ssubunittopreventproteinsynthesis

Macrolides

Bindto50Ssubunitofribosome,inhibiting

Methylation

translocation
Fluoroquinolones InhibitDNAgyrase,preventingDNAreplication
Trimethoprim

Inhibitsfolicacidsynthesisbyinhibiting
dihydrofolatereductase

Sulfonamides

Metronidazole

Inhibitfolicacidsynthesisbybeingastructural

ModificationsofPABA

analog(competitiveinhibitor)ofPABA,a
precursoroffolicacidinbacteria

enzymeandincreased
synthesisofPABA

Convertstoatoxicmetabolitethatpreventscell
wallsynthesis

Polymyxins

Interactwithphospholipidstodisruptthebacterial
cellwall

PABA,paraaminobenzoicacid.

11 CovertherightcolumninTable2112(t0065)anddescribetheadverseeffects
foreachoftheantimicrobialagentslistedintheleftcolumn
ThelistofantibioticstoknowforStep1isquiteextensive,andstudentsoftenwonderhowin
depththeirknowledgemustbetolearnthissubjectforboards.Ourbestguessisthatyou
shouldexpectanywherefromonetothreequestionsonantibiotics.FirstAidhasagreatreview
ofthistopic,butthereisstillquiteabitofinformationinthesepages.Ifyoucanlearnitall,
great!Ifyoufindyourselfshortontime,goforthehighestyieldpoints.Foreachantibioticyou
shouldthereforelearnthisinformationinthefollowingorder:
Mechanismofactionandmodeofresistance

Uniquesideeffectsandtoxicitysymptoms:Notethatwesaidyoushouldlearnthe
uniquesideeffectsofeachdrug.Boardswillnottestyouonthefactthatcertain
antibioticscancauseoccasionalgastrointestinal(GI)upsetorheadache.Thesesymptoms
arecharacteristicoftoomanydrugstomakeforgoodtestquestions.Focusonthe
toxicitieslistedinTable2112(t0065).
Clinicaluses:Notethatyoushouldknowthegeneralusesforeachdrug(e.g.,
vancomycinisusedforgrampositiveorganismsonly,aminoglycosidesforseriousgram
negativeinfections,aztreonamforgramnegativerods,metronidazoleandclindamycin
foranaerobes)butyoudonotnecessarilyneedtolearntheindividualorganismsaffected
byeveryantibiotic.Wedonotmeantoimplythatthismaterialisnotimportantforyour
clinicalyearsorfairgameforboards,butitislesslikelytobetestedthantheprevioustwo
points.However,youshouldbesuretoknowwhichdrugscanbeusedforselectbacterial
speciesnamely,Pseudomonas,methicillinresistantStaphylococcusaureus(MRSA),
andEnterococcus.
Step1Secret

Case211
A64yearoldmanisevaluatedfora3dayhistoryofproductivecough,fever,andchills.He
describeshisphlegmasrustcoloredandnotesthathisribshurtwhenhetakesadeepbreath.
Onexamination,thepatientisfebrilewithatemperatureof101.5FandanO2saturationof
89%.Cracklesareheardintherightlowerposteriorlungfield.Laboratoryworkuprevealsa
significantleukocytosis.Chestxraystudyandsputumculturearepending.

Table2112
AdverseDrugEffects

Drug

AdverseEffects

Lactams

Hypersensitivity
Diarrhea
Cephalosporinshave1020%crossreactivityinpenicillinallergicpatients

Tetracyclines

Gastrointestinalupset
Discolorsteethinchildren
Toxicityinpatientswithrenalimpairment
Photosensitivity
Affectsbonegrowthinchildren

Aminoglycosides Nephrotoxicityandototoxicity

Macrolides

Gastrointestinaldistress
Acutecholestatichepatitis
ProlongedQTinterval

Fluoroquinolones Damagescartilageinyoungchildren
Tendonruptureinadults
Chloramphenicol Aplasticanemia
Graybabysyndrome
Trimethoprim

Mimicsfolicaciddeficiency(megaloblasticanemia,leukopenia,
granulocytopenia)

Sulfonamides

Allergicreactions
Hemolysisinglucose6phosphatedeficiency
Photosensitivity

Vancomycin

Nephrotoxicityandototoxicity
Thrombophlebitis
Redmansyndrome(preventedbyantihistamines)

Metronidazole

Disulfiramlikereactionwithconcurrentalcoholintake
Metallictaste

1 Whatisthemostlikelydiagnosis?
Thecombinationoffever,chills,pleuriticchestpain,hypoxemia,andproductivecoughisvery
suggestiveofpneumonia.Furthermore,therustcoloredsputumsuggestsstreptococcalpneumonia.
GrampositivediplococciandrustcoloredsputumarecommonbuzzwordsforStreptococcus
pneumoniae.Salmoncoloredsputum,ontheotherhand,isassociatedwithStaphylococcus
aureus.Youshouldknowthebuzzwordsassociatedwithvariousmicroorganisms.Wewill
drawattentiontothesebuzzwordsthroughoutthemicrobiologychapters.
Step1Secret

2 Whatdefensemechanismspreventpneumoniainthehealthyindividual?
Therespiratorytracthasmanydefensesinplacetopreventaccesstothelungsbypotential
pathogens.Thenasalhairs,mucosa,anddynamicsofairflowallactearlytopreventinhalationof
microorganisms.Theepiglottisandcoughreflexbothacttopreventparticulatematterfromtraveling
intothedeeperairways.Therespiratorytractislinedwithmucusuntiltheterminalbronchiolesare
reached.Thismucusispropelledupwardbytheciliatedepithelium,eliminatingforeignmaterialas
expectorant.Thelastlineofdefenseisinandaroundthealveolarcomplexandiscomposedof
macrophages,neutrophils,immunoglobulin,andcomplement.Thesecomponentswillbecome
hyperactiveduringaninfectiousprocessbecausemanyoftheirtriggersareforeignantigens.

Note:Anystatethataltersthelevelofconsciousness(anesthesia,seizure,intoxication,sedation,and
neurologicdisorderssuchascoma)predisposestoaspirationpneumoniaduetosuppressionofthe
coughreflex.Theorganismscausingthistypeofinfectionareusuallyanaerobesfromthemouthor
refluxedgastriccontents.

3 Whymightapatientintheintensivecareunitwhoisintubatedbeatincreased
riskfordevelopingpneumonia?
Mechanicalventilationbypassesthenormalhostdefenses(e.g.,mucociliaryclearance)for
preventingcontaminationofthesterilelowerrespiratorysegments.Foreachdayonmechanical
ventilation,itisestimatedthatthepatienthasa1%chanceofacquiringnosocomialpneumonia.The
expecteddurationofintubationmustthereforebeaconsiderationindecidingwhetherornottoplace
apatientonmechanicalventilation.

4 Whyisitimportanttodistinguishbetweencommunityacquiredand
nosocomialpneumonia?
Thereisgenerallyadifferentspectrumoforganismsthatcausethesetwotypesofpneumonia,so
empiricalselectionofantibioticsisdifferent.Themostcommonpathogenscausingcommunity
acquiredpneumoniaincludeS.pneumoniae,H.influenzae,Legionellapneumophila,and
Mycoplasmapneumoniae.ThemostcommonpathogencausingnosocomialpneumoniaisS.aureus
.

5 Whatisatypical(walking)pneumonia,andisthepatientinthiscasemore
likelyhaveatypicaloranatypicalpneumonia?
Atypicalorwalkingpneumoniahasamoreinsidiousonsetthanwhathasbeendescribedinthis
case.Itischaracterizedbyheadache,nonproductivecough,lowgradefever,andanonspecific
diffuseinterstitialinfiltrateonxraystudythatlooksworsethanmightbeexpectedfromthepatient's
appearance.AtypicalpneumoniaisgenerallycausedbyvirusesorintracellularbacteriasuchasL.
pneumophila,M.pneumoniae,andspeciesofChlamydiasuchasChlamydiapsittaci.M.
pneumoniaeistheclassiccausativeorganismandcanbedifferentiatedfromothercausesbased
uponahightiterofcoldagglutinins(IgM).Mostofthebacterialcausescanbetreatedwitha
macrolideortetracycline,ourfavoritedrugsforintracellularbugs.
Thispatientmostlikelyhasatypicalpneumoniabasedontherapidityofonsetandproductive
cough.
Note:ThetermcoldagglutininsreferstothefactthatIgMantibodieswillbindtoredbloodcells
(RBCs)atlowtemperaturesandcausethemtoagglutinate,orsticktogether.Thiscanbe
demonstratedatthebedsidewhenabloodsamplebecomesclumpywhenplacediniceandfluid
againwhenrewarmed.Boardscommonlyteststudentsontheassociationbetweencoldagglutinins
andM.pneumoniae.

Case211continued:

ChestxrayfilmisshowninFigure214A(f0025).SputumGramstainrevealslargenumbersof
slightlyelongated,grampositivecocciinpairsandchains(Fig.214B(f0025)).

Figure214
A,Chestfilmshowingclassicpneumococcalpneumonia(arrows).B,Gramstainedsputumfroma
patientwithpneumoccalpneumoniaat1000magnification.
(AfromBrownTA,BrownD:USMLEStep1Secrets.Philadelphia,Hanley&Belfus,2004.Bfrom
MandellGL,BennettJE,DolinR:PrinciplesandPracticeofInfectiousDiseases,6thed.Philadelphia,
ChurchillLivingstone,2005.)

6 Whatisthediagnosis?
ThesputumGramstainshowinggrampositivediplococciissuggestiveofstreptococcalpneumonia
infection.Forthesakeofcompleteness,thechestxrayfilmshowsopacification(consolidation)of
therightupperlobe,consistentwithalobarpneumonia.
Althoughinterpretationofcomplexchestxrayfilmsisbeyondthepurviewofthesecondyear
medicalschoolcurriculum,youshouldbeabletorecognizesomecommonchestxrayfindings
suchaslobarandinterstitialpneumonia,pneumothorax,pleuraleffusion,andcongestiveheart
failure(CHF)associatedpulmonarycongestion.Werecommendthatyouperuseananatomy

atlasorcredibleonlinesitestostudytheseimages.Boardsarebecomingincreasinglyclinical,
andwriterswillinsertpathologicimagesandperhapsevensomeradiographsthroughoutyour
test.Itmaynotalwaysbenecessarytousetheprovidedimagestoarriveatthecorrectanswer,
butyoushouldnottakethisgamble.YoushouldalsobeawarethattheUSMLEStep1
occasionallyinsertspathologicalandradiographicalimagesasanswerchoicestoitsquestions,
soyouwouldbewisetoprepareforthispossibility.
Step1Secret

7 Howshouldthispatientbetreatedpharmacologically?
AlthoughpenicillinGhasbeenfirstlinetherapyforcommunityacquiredpneumonia,arising
incidenceofpenicillinresistanceamongstrainsofS.pneumoniaeoftennecessitatestheuseofan
alternativeagentsuchasceftriaxone.Noticethatasathirdgenerationcephalosporin,ceftriaxone
cancoverthemorecommongrampositiveandgramnegativeorganismsthatleadtocommunity
acquiredpneumonia(e.g.,pneumococciandH.influenzae,respectively).

8 UseTable2113(t0070)toquizyourselfonthemostcommoncausesof
pneumoniaindifferentagegroups
Table2113
CausesofPneumoniabyAge

Neonates(06

Children(6Week18

Adults(1840

Adults(4065

Elderly(>65

Weeks)

Years)

Years)

Years)

Years)

GroupB

Viruses

Mycoplasma

S.pneumoniae

S.pneumoniae

streptococci

Mycoplasma

C.pneumoniae

Haemophilus
influenzae

Viruses
Anaerobes

Escherichiacoli

Chlamydia
pneumoniae

S.pneumoniae

Anaerobes

Streptococcus

Viruses

H.influenzae

pneumoniae

Mycoplasma

Grampositive
rods

9 UseTable2114(t0075)toquizyourselfontheimportantcharacteristicsofthe
organismsthatareknowntocausepneumonias
NosocomialinfectionismostcommonlycausedbyStaphylococcusaureus.
Atypicalpneumoniahasamoreinsidiousonset,withaclassicclinicalpresentationof
headache,nonproductivecough,andanonspecificdiffuseinterstitialinfiltrateonxray

filmthatlooksworsethanwouldbeexpectedfromthepatient'scondition.Common
causesincludeLegionellapneumophila,Mycoplasmapneumoniae,andChlamydia
psittaci.
RedcurrantjellysputumissuggestiveofKlebsiellapneumoniae.
Forapatientonarespirator,thereisa1%riskperdayofacquiringnosocomial
pneumonia.
SummaryBox:Pneumonia

Case212
A26yearoldwomanpresentstoyourofficecomplainingofseverediarrheaforthepastday.
Sheinformsyouthatherbowelmovementsarewaterywithsmallstoolparticles,andshe
deniesthepresenceofblood.ShejustreturnedfromaweeklongtriptoMexico,whereshe
drankonlybottledwatersupplementedwithicefromherhotelroom.Shehasnoother
complaintsorproblems.Examinationisremarkablefortachycardiaanddrymucous
membranes.

Table2114
CharacteristicsofOrganismsthatCausePneumonia

Streptococcuspneumoniae
Seenin:

Communityacquiredpneumonia

Stain

Grampositive

Morphology

Diplococci

Catalase

Negative

Hemolysis

Alpha

Optochin

Sensitive

Quellungreaction

Positive

Bilesolubility

Soluble

Sputum

Rustcolored

Staphylococcusaureus
Seenin:

Nosocomialpneumonias

Stain

Grampositive

Morphology

Cocciinclusters

Catalase

Positive

Coagulase

Positive

Hemolysis

Beta

Klebsiellaspp.
Seenin:

Alcoholics
Diabetics
Aspirations

Stain

Gramnegative

Morphology

Rods

Lactosefermentation Positive
Sputum

Redcurrantjelly

Pseudomonasaeruginosa
Seenin:

Cysticfibrosis

Stain

Gramnegative

Morphology

Rod

Lactosefermentation Negative
Oxidase

Positive

GroupBStreptococci
Seenin:

Neonates

Stain

Grampositive

Morphology

Chains

Catalase

Negative

Hemolysis

Beta

Bacitracin

Resistant

Mycoplasmaspp.
Seenin:

Atypicalpneumonias

Stain

None

Growthmedium

Eaton'sagar

Bloodtest

Coldagglutinins

Escherichiacoli
Seenin:

Neonates

Stain

Gramnegative

Morphology

Rod

Lactosefermentation Positive
Chlamydiapneumoniae
Seenin:

Atypicalpneumonia

Stain

Giemsa

1 Whatisthemostlikelydiagnosis?
Thispatientismostlikelysufferingfromtraveler'sdiarrhea.Despiteherbesteffortstodrinkonly
bottledwater,shehasmadeaverycommonmistakeamongtravelers:sheusedicemadewithlocal
water.UseTable2115(t0080)toreviewthetypesofE.coliandthesyndromestheycause.
Table2115
EscherichiaColiStrains

Strain

Syndrome

Enterotoxigenic

Traveler'sdiarrhea

Enteroadherent

Traveler'sdiarrhea

Enteropathogenic

Infantilediarrhea

Enterohemorrhagic Bloodydiarrheahemorrhagiccolitisandhemolyticuremicsyndrome
Enteroinvasive

Bloodydiarrhea(dysentery)

Enteroaggressive

PersistentdiarrheainchildrenandHIVinfectedpatients

HIV,humanimmunodeficiencyvirus.

2 WhatothertypesofdiarrheacanbecausedbyEscherichiacoli?
EnterohemorrhagicE.coli(EHEC)andenteroinvasiveE.coli(EIEC)bothcauseadysenterylike
syndromewithfeverandbloodystools.EnteropathogenicE.coli(EPEC)isacommoncauseof
diarrheaininfants,andenteroadherentE.coliisanothercauseoftraveler'sdiarrhea.

3 Whatisthedifferencebetweenosmoticandsecretorydiarrhea?Nameacause
foreachtype

foreachtype
Secretorydiarrheaiscausedbyactivesecretionoffluidsbytheintestines.Examplesofthis
typeofdiarrheaareV.cholerae(Fig.215(f0030))andenterotoxigenicE.coli(ETEC),the
causeoftraveler'sdiarrhea.

Figure215
Mechanismofcholeratoxin,anABtypetoxin.
cAMP,cyclicadenosinemonophosphate.(FromRosenthalK,TanJ:RapidReviewMicrobiologyand
Immunology,2nded.Philadelphia,Mosby,2007.)

Osmoticdiarrheaiscausedbyosmoticallyactiveagentswithinthegutlumenthatresultin
passivemovementofwaterintotheintestinallumenalongosmoticgradients.Anexamplein
whichthismayoccurisnutritionalmalabsorption(e.g.,inceliacsprueorpancreatic
insufficiency),inwhichtheosmoticallyactivenutrientspullwaterintotheintestines.

4 WhatpredisposestoClostridiumdifficilecolitisandwhatsortofdiarrhea
doesthiscause?
Theuseofantibioticssuchasampicillinandclindamycinmustbecarefullymonitoredtoavoid
inducingC.difficilecolitis,alsoknownaspseudomembranouscolitis.Pseudomembranouscolitis
occurswhenamemberofthenormalintestinalflora(C.difficile)proliferatesinexcessafter
eliminationofcompetitorspeciesfollowingantibioticuse,resultinginsuperinfection.Thisbacteriais
associatedwithtwoexotoxins,referredtoastoxinsAandB,thatresultinsecretorydiarrheaand
damagethegutmucosa.DetectionofToxinBinthestoolcanbeusedtoconfirmC.difficileinfection.

5 Howisdiarrheatreated?
Generally,supportivetherapytoreplacelostfluidsandelectrolytesisallthatisneeded.Forthemore
seriousbugssuchasthosecausingbloodydiarrhea,broadspectrumantibioticsmaybehelpful
althoughthisrunstheriskofinducingC.difficileinfection(Tables2116(t0085)and2117(t0090)).

Table2116
CausesofWateryDiarrhea

InfectiousAgent Comments

Treatment

ETEC

Causestraveler'sdiarrheaandisanimportantcauseofdiarrhea

Fluidand

inchildrenyoungerthan2yearsofageinthedevelopingworld

electrolyte

heatlabiletoxinactsonadenylatecyclase,heatstabletoxinacts replacement
onguanylatecyclase
Vibriocholerae

ActsonGproteintostimulateadenylatecyclase,leadingto

Fluidand

increasedClreleaseintolumenofgutpossiblericewater

electrolyte

diarrhea

replacement

Giardia

Transmittedbycystsinwateranddiagnosedbytrophozoitesin

Metronidazole

(protozoan)

stool

Norwalkvirus

Acalcivirus

Fluidand
electrolyte
replacement

Rotavirus

Fataldiarrheainchildren

Fluidand
electrolyte
replacement

Cryptosporidium CanbesevereinAIDS

None

(protozoan)
AIDS,acquiredimmunodeficiencysyndromeETEC,enterotoxigenicEscherichiacoli.
Table2117
CausesofBloodyDiarrhea

Infectious

Comments

Treatment

Lowinoculum(101)nonmotiletransmittedby4
Fs(fingers,food,feces,flies)doesnotinvade

TMPSMX

Agent
Shigella

beyondgutmucosa
Salmonella

Higherinoculum(105)motiletransmittedfrom

TMPSMX

animalproducts,especiallypoultryandeggscan
becomedisseminated
EHEC

Shigaliketoxinthatcancausehemolyticuremic

Fluidandelectrolyte

syndrome(HUS),especiallyO157:H7

replacement(withglucose)

EIEC

Signs/symptomssimilartothoseofshigellosis
beginsaswateryandcanproceedtobloody
diarrhea

Campylobacter Thermophilic(optimalgrowthtemperatureis42 Usuallyselflimitinggivefluid


C)characteristiccommaorSshapeoxidase

andelectrolytereplacement

andcatalasepositive
Clostridium

Causespseudomembranouscolitiscanbeseen

Metronidazoleororal

difficile

aftertheadministrationofclindamycinor

vancomycin

ampicillin
Yersinia

Transmittedviapetfeces,milk,orporkcauses

Fluidandelectrolyte

enterocolitica

daycareoutbreakswithsymptoms/signssimilar

replacement(although

tothoseofappendicitis,called
pseudoappendicitis

antibioticsareindicatedif
infectionisinvasive)

Transmittedbycystsinwater

Metronidazole

Entamoeba
histolytica
(protozoan)

EHEC,enterohemorrhagicEscherichiacoliEIEC,enteroinvasiveEscherichiacoliTMPSMX,trimethoprim
sulfamethoxazole.

Diarrheacanusuallybetreatedbyreplacinglostfluidsandelectrolytes(supportive
therapy).
AntibioticssuchasclindamycintherapyincreasetheriskofClostridiumdifficilecolitis.
StrainsofEscherichiacolithatcausewaterydiarrheaincludeenterotoxigenic,
enteroaggregative,andenteropathogenictypes.
StrainsofE.colithatcausebloodydiarrheaincludeenterohemorrhagicand
enteroinvasivetypes.
SummaryBox:Diarrhea

Case213
A65yearoldwomanwithahistoryofhypertensionandrheumaticfeverasachildisevaluated
fora2to3weekhistoryofnightsweats,fever,malaise,andmyalgias.Cardiacauscultation
revealsapreviouslyundetectedfaintdiastolicmurmur.Findingsoninspectionofthefingers
andfunduscopicexaminationareasshowninFigures216(f0035)and217(f0040).
Echocardiogramandbloodcultureresultsarepending.

Figure216
FingerinspectionofpatientinCase213.
(FromKorzeniowskiOM,KayeD:Infectiveendocarditis.InBraunwaldE[ed]:HeartDisease,4thed.
Philadelphia,WBSaunders,1992.)

Figure217
FundoscopicexaminationofpatientinCase213.
(FromKorzeniowskiOM,KayeD:Infectiveendocarditis.InBraunwaldE[ed]:HeartDisease,4thed.
Philadelphia,WBSaunders,1992.)

1 Whatisthemostlikelydiagnosis?
Thiscasedescribesthepresentationofacutebacterialendocarditis,aninfectionoftheendothelial
liningoftheheart(Fig.218(f0045)).

Figure218
A,Acuterheumaticendocarditis.Grossphotographofanaorticvalvewithsmallvegetations(verrucae)alongthe
linesofvalveclosure.B,Chronicrheumaticendocarditis.Grossphotographofamitralvalvewithmassive
fibrosisanddistortionoftheleafletsandfusionofthechordaetendineae.
(FromKingT:Elsevier'sIntegratedPathology.Philadelphia,Mosby,2007.)

BacterialendocarditisisahighyieldtopicforStep1.Studentswilloftenseeimagessimilarto
thoseinFigure218(f0045)ontheirexaminations.
Step1Secret

2 Whatarethemajorriskfactorsfordevelopingendocarditis?
Themajorriskfactorforthedevelopmentofendocarditisisastructurallyabnormalheartvalve
causingaberrantflowstreams.Commonstructuralabnormalitiesareprostheticvalvesornative
valvelesions,calcifications,rheumaticheartdisease,andcongenitalabnormalities.Amajorityof
infectionsoccurintheleftsideoftheheart(themitralvalveisthemostfrequentlyaffectedvalvein
bacterialendocarditis,buttheaorticvalvemayalsobeinvolved),butwithintravenous(IV)druguse,
rightsidedtricuspidvalvelesionsmayoccurasaresultofintroductionofthepathogensintothe
venoussystem.BacterialspeciesassociatedwithIVdrugabuseincludeS.aureusandP.aeruginosa.
Candidaalbicansisafungalcauseofrightsidedendocarditis.

3 Whataretheclinicalsignsofbacterialendocarditis?
Bacterialendocarditiscommonlypresentswithlowgradetohighfever,newonsetheartmurmur,
chills,nightsweats,weightloss,fatigue,andmildanemiaofchronicdisease.Thetimelineofthis
presentationdependsonwhethertheendocarditisisacuteorsubacute.Bacterialendocarditisalso
presentswithRoth'sspots(whitedotsontheretinasurroundedbyareasofhemorrhageseeFig.217
(f0040)),Osler'snodes(painful,elevatedlesionsonthepadsofthefingersandtoes),Janewaylesions

(painless,flatdiscolorationsonthepalmsandsoles),andsplinterhemorrhages(seeFig.216(f0035)
).Roth'sspots,Osler'snodes,Janewaylesions,andsplinterhemorrhagesareallmanifestationsof
smallbacterialemboli(Table2118(t0095)).
Table2118
SymptomsandSignsofBacterialEndocarditis

Symptom/Sign Description
Fever

Canbespiking

Rothspots

Retinalhemorrhageswithpale,whitecenterscomposedoffibrin

Osler'snodes

Tender,raisedlesionsoffingerortoepads

Murmur

Neworchangingduetovalvulardamage

Janeway

Nontendererythematousmaculesonpalmsorsoles

lesions
Anemia

Anemiaofchronicdisease

Nailbed

Oftencalledsplinterhemorrhagesandcanbeseenunderthenailbeddueto

hemorrhages

microemboliblockingsmallervessels

Emboli

Canleadtostrokeorgangreneofdistalextremities

4 Whataretheclinicalsignsofrheumaticfever?
RheumaticfeverisasequelaofS.pyogenes(groupA,hemolytic)pharyngitis.Acuterheumatic
fevermostcommonlyoccursinchildrenbuthasbeenseeninadults.Thesymptomsofacute
rheumaticfeverusuallyoccur2to3weeksfollowingpharyngitis,makingprompttreatmentofS.
pyogenespharyngitisanimportantpartofrheumaticfeverprevention.Table2119(t0100)liststhe
clinicalmanifestationsofacuterheumaticfever,althoughnotallneedtobepresenttomakethis
diagnosis.
Table2119
AcuteRheumaticFever

Symptom/Sign Description
Migratory

Multiplejointinvolvement,buteachonlyforashortperiodoftimearthritisusually

arthritis

theinitialmanifestation

Carditis

Neworchangingmurmursmayappearpericardium,epicardium,myocardium,
andendocardiumallaffectedmayseecardiomegalyonradiologicstudies

Chorea

St.Vitusdancesudden,nonrhythmic,purposelessmovementalsocansee

Chorea

St.Vitusdancesudden,nonrhythmic,purposelessmovementalsocansee
muscleweaknessandemotionaloutbursts

Subcutaneous Mostcommonlyseenoverbonyprominencesnonpainfulandnoninflammatory
nodules
Erythema

ArashsimilartothatofLymedisease,inwhichtheerythematousregionextends

marginatum

outwardasthecenterbecomespale,formingaringmostoftenseenontrunkand
notthefaceoccursearlyinthediseaseandpersiststhroughoutitscourse

Note:Aschoffbodiesarethepathognomonichistologicfindinginrheumaticheartdisease.Theyare
foundinthemyocardiumandconsistofregionsoffibrinoidnecrosiswithmononuclearand
multinucleatedgiantcellinfiltrates(Fig.219(f0050)).Also,theantistreptolysinO(ASO)titerisused
todetectarecentS.pyogenesinfectionandshouldbeelevatedincasesofacuterheumaticfever.

Figure219
MicroscopicappearanceofanAschoffbodyinapatientwithacuterheumaticcarditisthereiscentralnecrosis
withacircumscribedcollectionofmononuclearinflammatorycells,someofwhichareactivatedmacrophages
(Anitschkowcells)withprominentnucleoli(arrowheads).
(FromKumarV,CotranR,RobbinsS:RobbinsBasicPathology,8thed.Philadelphia,WBSaunders,2008.)

RheumaticfeverisahighyielddiagnosisforStep1.YoushouldbeabletoidentifyFigure219
(f0050)asanAschoffbody,apathognomonicfindinginthisdisease.

Step1Secret

5 Whichbacteriaaremostcommonlyassociatedwithbacterialendocarditis?
Endocarditiscanbeclassifiedintoacuteorsubacutetypesdependingonthetimecourse.Acute
infectionsoccurwithindaystoweeksandpatientsareextremelysickduringthistimetheyaremost
oftenduetoStreptococcusorStaphylococcus.Subacuteinfectionspresentwithmildersymptoms
andarecharacterizedbyaconsistentlylowgradeillnessfor3to4weekstheyarefrequentlycaused
byStreptococcusviridansandgroupDstreptococcisuchasStreptococcusbovis(Table2120(t0105)
).
Table2120
AcuteVersusSubacuteEndocarditis

Characteristic

Acute

Subacute

Organisms

Staphylococcus

Streptococcusviridans(S.sanguis)afterdental

aureus

procedures

Rapid(days

Insidious(34weeks)

Onset

weeks)
Clinical

Severesickness

Mildsickness

Vegetationsize

Large

Smaller

Typesofvalves

Previouslynormal

Damagedorcongenitallyabnormalvalves

affected

valves

manifestations

Note:Bacterialendocarditisthatoccurssoonafterprostheticvalvularsurgeryiscommonlydueto
Staphylococcusepidermidisandisbelievedtoresultfromintraoperativecontamination.

6 Whatdrugscouldbeusedtotreatthispatient?
BecauseacuteendocarditiscanbecausedbyStreptococcusspeciesaswellasS.aureus,thedrug
chosenwillneedtocoverbothoftheseorganisms.IfthereisnosuspicionofmethicillinresistantS.
aureus(MRSA),theneitheroraldicloxacillin(PO)orIVnafcillinwouldworkonboth,butifthereis
suspicionofMRSA,thenIVvancomycinwouldbethedrugofchoice.

7 HowdoesbacterialendocarditisdifferfromLibmanSacksendocarditis?
LibmanSacks(LS)endocarditis,whichisseeninsystemiclupuserythematosus(SLE),isanaseptic
inflammationoftheheartvalves.Thevegetationstypicallyinvolvebothsidesofthevalve,whereas
thevegetationsoccurprimarilyonthedownstreamsideofthevalveinbacterialendocarditis.
Finally,thevegetationsinLSendocarditiswillnotembolize.

Anystructuralabnormalitycanpredisposetoendocarditis.
Thevalvesoftheleftsideoftheheartaremostcommonlyaffected,exceptin
intravenous(IV)drugusers,inwhomtherightsideoftheheartiscommonlyinvolved.
Staphylococcusaureusisthemostcommoncauseofacutebacterialendocarditis,and
Streptococcusviridansisanimportantcauseofsubacuteendocarditis.
LibmanSacksendocarditisisseeninsystemiclupuserythematosus(SLE),doesnot
containbacteria,andwillnotembolize.
SummaryBox:Endocarditis

Case214
A20yearoldmanisevaluatedforanewgenitallesion.Thepatientreturnedfromspringbreak
lastweekandnoticedapainlessulceronhisscrotum.Heisquiteconcernedandadmitsto
severalinstancesofunprotectedintercourse.Onexamination,thereisawelldemarcated,2cm
painlesslesionwitharaisedborderontheshaftofthepenis(Fig.2110(f0055)).Theremainder
oftheexaminationisunremarkable.

Figure2110
PhysicalexaminationofpatientinCase214.
(FromHabifTP:ClinicalDermatology,4thed.Edinburgh,Mosby,2004.)

1 Whatisthelikelydiagnosis?
SyphilisiscausedbytheorganismTreponemapallidum.Theorganismentersthebodythrough
brokenepitheliumordirectmucosalcontact.Theclassicsyphiliticchancrehasaclean,nonpurulent
basewithasharplydefinedborder,asshowninFigure2110(f0055).

2 Basedonthisman'spresentation,inwhichstageofsyphiliticinfectionishe
mostlikelytobe?
Syphilisprogressesthroughthreestages:primary,secondary,andtertiary.Thispatientdisplaysthe
classicpainlessgenitalchancreofprimarysyphilis,whichappears3to6weeksaftercontact.This
lesionishighlyinfectiousandcontinuouslyshedsmotilespirochetes.Theprimarystagewilllast4to
6weeksandthenresolve,oftenfoolingpatientsthattheyarecured.

3 Whatstageofsyphiliswouldyoususpectinapatientwithadiffuse
maculopapularrash?
Thispresentationisclassicforsecondarysyphilis.Thesecondarystageofsyphiliswillbegin
approximately6weeksaftertheprimarychancrehashealed.Thisphaseischaracterizedbya
generalizedmaculopapularrashwithorwithoutthefleshy,painlessgenitalwartstermed

condylomatalata.Thesecondarystageofsyphilisresolvesin6weeksandentersthelatentphase.If
theinfectionisnottreated,itwillprogresstotertiarysyphilisinapproximatelyonethirdofthese
patients.
Youshouldknowwhichbacteriaandvirusescausegenitallesionsandwhethertheselesionsare
painfulorpainless.Rememberthatthetwobugsassociatedwithpainfulgenitallesionsare
HSV2(genitalherpes)andHaemophilusducreyi.Aneasywaytokeepthisinmindisto
rememberthatthosewithgenitalherpesdocry(ducreyi)inpain.Bycontrast,infections
associatedwithsyphilis,gonorrhea,chlamydia,lymphogranulomavenereum,human
papillomavirus(HPV),trichomoniasis,andbacterialvaginosisarepainless.
Step1Secret

Case214continued:
Thispatientdoesnotseektreatmentandpresentstoyouroffice10yearslaterwitha
regurgitantmurmurheardbestovertherightsecondintercostalspaceandanataxicgait.

4 Whatisthelikelydiagnosis?
Thispatientispresentingwithsymptomsoftertiarysyphilis.Thisstagecandevelopanywherefrom5
to35yearsaftertheinitialinfection.Tertiarysyphilisisasystemicdiseasewiththreemajor
components:granulomatouschange(gummas),cardiovascularsyphilis,andneurosyphilis.
Inflammatorydestructionisthepathophysiologicmechanismthatisinherenttoallthree
components.Knowthatcardiovascularsyphilismayresultinaorticvalveinsufficiencyandaortic
aneurysm,andneurosyphiliscancausetabesdorsalis,aconditionthatcausesdorsalcolumndisease
ofthespinalcordandsubsequentataxia.

5 UseTable2121(t0110)toquizyourselfonthethreestagesofsyphilitic
infection
Table2121
StagesofSyphilis

Parameter

Primary

Timing

Secondary

Tertiary

Congenital

3weeksof Weekstomonthsafter

130yearsafterprimary

Transmitted

incubation

infection(becauseof

tofetus

emergenceofpapule

followedby
emergence

latentperiodbetween
secondaryandtertiary)

ofpapule
Characteristic

Painless

Disseminateddiseasewith

Gummas(granulomas),

Stillbirth

symptoms/signs papuleon
genitals

constitutionalsymptoms

aortitis,tabesdorsalis

saber

possiblerashthatcan

(neurosyphilisofdorsal

shins,

involvepalmsandsoles

columns),Argyll

saddle

condylomatalataarewhite

Robertsonpupil

nose

lesionsongenitalsmost

(constrictionto

deformity,

infectiousstage

accommodationbutnot

deafness

tolight)
Treatment

PenicillinG PenicillinG

None

Symptom
dependent

6 Whatdiagnostictestscouldbedonetodefinitivelydiagnosesyphilisinthis
man?
Directvisualizationbydarkfieldmicroscopycanbedoneduringtheactivephasesofstage1andstage
2syphilis.Thisisconductedbyobtainingasamplefromthelesionandobservingthemotile
spirochetes.Serologictestswerealsodevelopedtosatisfytheneedforasyphilisscreen.TheVenereal
DiseaseResearchLaboratory(VDRL)andtherapidplasmareagin(RPR)testsweredevelopedto
detectantibodiespresentagainstcertaincomponentsreleasedaftercelldeath.Thesetestsare
nonspecifictreponemaltestsand,ifpositive,requireamorespecificmeasure,thefluorescent
treponemalantibodyabsorption(FTAABS)test.ThekeypointisthattheVDRLandtheRPRtests
areeffectiveforscreeninghighriskpatients.TheVDRLtestiseasierandlessexpensive,soitis
usuallydonefirst.However,itcanhavefalsepositiveresultsbecauseitcrossreactsinthepresence
ofvariousdrugs,viruses,andrheumatologicdiseases(e.g.,SLE).TheVDRLtestwillbecomepositive
inlateprimarysyphilisandbecomesnegativeagaininlatesecondarysyphilis.Inadditiontobeing
morespecific,theFTAABStestalsobecomespositiveearlierandstayspositivelonger.Therefore,
theFTAABStestcanbeusedtodiagnosetertiarysyphilisandtoconfirmapositivescreeningVDRL
test(Table2122(t0115)).
Table2122
SyphilisTests

Test

Use

Darkfield

Testofchoicewhenachancreispresentandabiopsyofthelesioncanbetakenfor

microscopy

directobservation

VDRL

Firsttestusedwhensecondarysyphilisissuspectedmayneedtobeconfirmedby
FTAABStestingduetohighnumberoffalsepositives

FTAABS

TestofchoicefortertiarysyphilisusedtoconfirmapositiveresultonVDRLtest

FTAABS,fluorescenttreponemalantibodyabsorptionVDRL,VenerealDiseaseResearchLaboratory.

7 Howwouldyoutreatthispatient?

Fortunately,syphilisisoneoftheeasiestdiseasestotreat.AdministerpenicillinG,andifthe
patientispenicillinallergic,offertetracyclineordoxycycline.Itisimportanttorememberthatonly
primaryandsecondarysyphiliscanbecuredwithmedication.Antibioticsdonothingfortertiary
syphilis.

8 Laterthatnight,thepatientcallsyouathomewithseriousconcernsabouta
reactiontopenicillin.Hestatesthatseveralhoursafterbeingtreatedhe
developedanewrash,alongwithfever,headache,andmuscleaches.Whatare
youconcernedaboutinthispatient?
Thispatienthaslikelysufferedfromacommonreactiontothepenicillintreatmentofsyphilisknown
astheJarischHerxheimerreaction.Thissideeffectoftreatmentisduetotheimmunesystem's
reactiontothelysisoftreponemes.Whenexposedtothetremendousloadofforeignantigens,the
bodyreleasesIL1andTNF,causingfeverandpossiblyshock.Thisentityshouldnotbeconfused
withanallergytopenicillinandrequiresonlytreatmentofsymptomsandclosemonitoring.

Thechancreseeninprimarysyphiliswillappear3to6weeksfollowingexposure.
Thesecondstageofsyphilisischaracterizedbyarashonthepalmsofthehandsandthe
solesofthefeetandtheemergenceofcondylomatalata.
Thethirdstageofsyphiliscanoccuryearsaftertheprimaryinfectionandcancause
aortitis,tabesdorsalis,andArgyllRobertsonpupil.
Syphilisistreatedwithpenicillin.
VenerealDiseaseResearchLaboratory(VDRL)assayisusedasascreeningtestand,if
positive,diagnosisisconfirmedwithfluorescenttreponemalantibodyabsorption(FTA
ABS)test
SummaryBox:Syphilis

Case215
Afranticmotherhasbroughther8yearoldsoninforanemergentvisit.Sheisconcerned
aboutanenlargingrashlocatedonthechild'sback.Sheaddsthathehasbeencomplainingofa
flulikeillnesssincethefamily'sreturnfromahikingtripinNewEngland,duringwhichhewas
bittenbyatick.Onexaminationyouappreciatealarge,welldemarcated20cmerythematous
rashwithcentralclearing(Fig.2111(f0060))andsomeregionaladenopathy.

Figure2111
LesionfrompatientinCase215.Notethevariationincolorandtargetlikeappearanceofthelesion.
Thebitesiteisvisibleinthecenter.
(CourtesyofDr.StevenLuger,OldLyme,Connecticut.)

1 Whatisthemostlikelydiagnosis?
Lymedisease,causedbythespirocheteBorreliaburgdorferi,ismostlikely.Thisbugistransmitted
fromthebiteofanIxodestick,endemictothewoodlandsofNewEngland.TheimageinFigure2111
(f0060)showsanexpandingerythematouslesionknownaserythemachronicummigrans.

Note:TheIxodestickisthevectorforB.burgdorferi(Lymedisease),Babesia(babesiosis),and
Anaplasmaphagocytophilum(granulocyticehrlichiosis).TreponemaldiseasesincludeLyme
disease,syphilis,andyaws.
TheUSMLEcommonlyasksstudentsaboutvectorsforvariousbacterialandparasitic
infections.CoinfectionwithLymediseaseandbabesiosisisaparticularfavoritebecauseboth
bugssharethesamevector.
Step1Secret

2 WhatstageofLymediseasewouldyoususpectinthischild?
Ourpatienthasmanifestationsconsistentwithstage1orearlylocalizedLymedisease.Lyme
diseaseissimilartosyphilisinthatbothillnessesarecausedbythedisseminationofaninfectious
spirocheteandprogressthroughthreestages:earlylocalizedstage,anearlydisseminatedstage,and
alatestage(stages1,2and3,respectively).Thispatientisinstage1,whichconsistsoftheexpanding
erythematouslesionknownaserythemachronicummigrans.Aflulikesyndromeandregional
adenopathyoftenaccompanytherashofstage1Lymedisease.

3 Howwouldyourdiagnosischangeifthispatientpresentedwithasimilar
historybuthadcomplaintsofvariouspainfulswollenjointsandadiffuse
macularrashalloverhisbody?
Hewouldthenmostlylikelybesufferingfromstage2orearlydisseminatedLymedisease.Thisstage
ischaracterizedbythespreadofB.burgdorferitofourcomponentsofthebody:joints,heart,
nervoustissue,andskin.Migratorymusculoskeletalpainsoccurandusuallyaffectthelargejoints
suchastheknee.Thesejointsbecomeswollenandtender.Cardiaccomplicationscanvary,ranging

fromconductionblocktomyocarditis,andneuralissuesrangefromviralmeningitistonervepalsies,
mostclassicallyabilateralBell'spalsy.Theskinlesionsofstage2Lymediseasearesimilartostage1
rashesbutaresmallerandmorewidelydistributedoverthebodysurface.

4 Ifthispatientdoesnotreceiveappropriatetreatment,whatisthelikelihood
thattheinfectionwillprogresstostage3Lymedisease?
ThelatestageofLymedisease(stage3)occursinonly10%ofuntreatedpatientsandischaracterized
bythedevelopmentofachronicarthritis,whichinvolvesmultiplelargejointsandaprogressive
centralnervoussystem(CNS)disease(Table2123(t0120)).
Table2123
StagesofLymeDisease

Stage

CharacteristicSymptoms

Earlylocal

Erythemachronicummigransflulikesymptomsoccurswithin1monthoftickbite

(stage1)
Early
Monoarticularoroligoarticulararthritis,Bell'spalsyorothercranialnervepalsy,
systemic(stage andatrioventricularconductionblockscanoccurdaystomonthsaftertickbite
2)
Late(stage3)

Migratorypolyarthritisandneurologicsymptomsoccursmonthstoyearsafter
initialinfection

5 WhatisthetreatmentforLymedisease?Nameapreventivemeasurethatcan
betaken
Lymediseaseiseffectivelytreatedwithdoxycycline.LaterstagesofLymediseaseshouldbetreated
withceftriaxone.Recently,aneffectivevaccinehasbeendeveloped.
Note:Lymediseaseismostcommonlytransmittedduringthesummer,sothevaccineshouldbe
giveninthespring.

6 DescribetheIxodeslifecycle
Remember,theIxodestickisonlythevectorfortheinfectiousspirocheteB.burgdorferi.TheIxodes
lifecycleextendsover2years.Eggsarelaidinthespringandwilldevelopintolarvaethatfeedinthe
summer,preferablyonmice.ThemiceactasthereservoirforB.burgdorferi,anditisherethat
Ixodesacquiresthespirochetethatitcanlatertransmit.Ixodesisdormantinthefallandwinterand
willbecomeanymphinthefollowingspring.Itwillfeedonamouseorahuman(butnotethatthe
humanisnotnecessaryforthelifecycleofthetick)andBorreliacanbetransmittedatthistime.
Afterfeeding,thetickbecomesanadultandwillmate,oftenonadeer(Fig.2112(f0065)).

Figure2112
LifecycleofIxodesscapularis(alsoknownasIxodesdammini).
(AdaptedfromanillustrationbyNancyLouMakrisinRahnDW,MalawistaSE:Lymedisease.WestJMed
154(6):708,1991.)

Lymediseaseiscausedbyaspirochete,Borreliaburgdorferi,andiscarriedbythetick
Ixodes.
TreatmentofLymediseaseiswithdoxycycline.
EarlylocalLymediseaseischaracterizedbyabull'seyetargetlikerashandflulike
symptomsthatoccurwithin1monthofthetickbite.
Earlysystemicdiseasecanmanifestasarticulardisease,Bell'spalsy,andheartblock.
LaterstagesofLymediseasemaymanifestasmigratoryarthritisandneurologic
symptoms.
SummaryBox:LymeDisease

Case216
WhileyouareinPakistanonamedicalmission,apatientpresentswithan8weekhistoryof
fever,nightsweats,andaproductivecough,attimestingedwithblood(hemoptysis).Hehas
lost20lbduringthistimeandhasbeengenerallyfatiguedandweak.Achestxrayfilmreveals

apulmonaryinfiltrate,andapurifiedproteinderivative(PPD)skintestispositive.Thepatient
reportsthatthesametestwasnegativeayearago.Asputumstainforacidfastbacilliis
positive.

1 Whatisthepresumptivediagnosis?
Hehastuberculosis(TB),causedbyMycobacteriumtuberculosis.Notethatthisisapresumptive
ratherthanadefinitivediagnosisbecauseseveralotheroccasionallypathogenicmycobacteriasuchas
Mycobacteriumaviumintracellulare(MAC)canproduceasimilarclinicalpresentationandpositive
acidfaststainresult.

2 Howisthisdiseaseprimarilytransmitted?
Aerosolizationofcontaminatedrespiratorysecretions(e.g.,coughing)spreadsthedisease.

3 Whyistheacidfaststainrequiredtovisualizethisbacterium?
MycobacteriumdoesnotstainwellwiththeGramstainbecauseitcontainsmycolicacidsinitscell
wallratherthanpeptidoglycan.However,itdoesstainwellwiththeacidfaststain,whichiswhyitis
referredtoasanacidfastbacterium.
Note:AcidfastbacteriaarevisualizedwithZiehlNeelsenstain,andM.tuberculosisisgrownon
LowensteinJensenagar.

4 Doesthispatientmostlikelyhaveprimarytuberculosis,latenttuberculosis,or
recrudescent(secondary)tuberculosis?
BecausehispreviousPPDtestwasnegative,hemostlikelyhasprimaryTB,whichresultsfrominitial
infectionwiththeorganism.Morespecifically,heprobablyhasaprogressiveprimaryinfection,in
whichsymptomsmanifest.Thislatterdistinctionismadebecausemostpatientswhobecome
infectedwiththemycobacteriumdonotdevelopsymptoms.LatentTBdevelopsaftersymptomshave
resolvedfromprimaryTB(iftherewereanysymptoms)andisduetotuberclebacilliresidingin
macrophages.RecrudescentTBdevelopsaftersomeformofimmunologiccompromisethatallows
thelatenttuberclebacillitobeginproliferatingagain(Fig.2113(f0070)).

Figure2113
Pathogenesisandclinicalcourseoftuberculosis(TB)causedbyMycobacteriumtuberculosis.
CNS,centralnervoussystemGI,gastrointestinal.(FromRosenthalK,TanJ:RapidReviewMicrobiologyand
Immunology,2nded.Philadelphia,Mosby,2007.)

Note:About10%ofpatientsinfectedwithTBintheUnitedStateswilleventuallyhavea
recrudescence.MiliaryTBoccurswhenthebacilliaretransmittedandcausefociofinfection
throughoutthebody.
AGhoncomplexreferstoaregionofthelungandassociatedperihilarlymphnodesthathavebeen
exposedtoTBandhavebecomegranulomatous.AGhoncomplexindicatesthattherehaseitherbeen
anexposuretoTBthatthebodywasabletoresolveimmunologicallyorthatthereisacurrent
primaryinfection.

5 Whatarethefirstlinedrugsfortreatingtuberculosisandwhyaretheyalways
usedincombination?
Thesedrugsincludeisoniazid(alsousedforprophylaxis),rifampin,ethambutol,streptomycin,and
pyrazinamide.Theyareusedincombinationbecausethereisahighincidenceofresistancetothese
drugs.IntheUnitedStates,infact,about10%to15%ofisolateshaveresistancetoonethesedrugs
evenbeforetreatmentisstarted.

6 Ifthispatientistreatedwithisoniazidaspartofhisregimen,whyshouldhe
alsoreceivesupplementalpyridoxine(vitaminB6)?
Oneofthemainsideeffectsofisoniazidisaperipheralneuropathy,whichiscausedbythedrug
stimulatingpyridoxineexcretionandcreatingarelativepyridoxinedeficiency.Rememberthatoneof
thefeaturesofpyridoxinedeficiencyisperipheralneuropathy.
Note:Isoniazidiswellknownforitshepatotoxicity.Itcanevencauseafullblownhepatitiswith
nausea,vomiting,jaundice,andrightupperquadrantpain.Isoniazidisalsoknowntocausealupus
likesyndromeandcanleadtohemolysisinglucose6phosphatedehydrogenasedeficiency.Itisan
inhibitoroftheP450system.

7 Ifthispatientistreatedwithrifampinaspartofhisregimen,whymayheneed
largerdosesofopioidanalgesicsforpaincontrolinotherillnesses/injuries?
RifampininduceshepaticP450enzymes,includingthosethatmetabolizeopioids.

8 Threeweeksafterstartingatherapeuticregimenwithrifampinandisoniazid
thepatientcomplainsoforangeurine.Whatisprobablycausingthis?

thepatientcomplainsoforangeurine.Whatisprobablycausingthis?
Thisisawellknownandcommonsideeffectofrifampin.Rifampinalsooftenturnssweat,tears,and
contactlensesanorangecolor.

9 Ifthispatientbeginscomplainingofvisionproblems,whatwouldyoususpect
isthecause?
Ethambutolhasasideeffectofopticneuropathy(decreasedvisualacuityandcolorblindness).

10 Whyisthestandardtreatmentregimenthatthispatientwillbeputonso
prolonged?
Severalcharacteristicsofthetuberclebacillusmakesitdifficulttocontrolquickly.Oneproblemisits
intracellularlocation,wheredrugsdonotpenetratewell.Inaddition,thebacillusisoftenfoundin
largecavitieswithavascularcenters,intowhichdrugsdonotpenetratewelleither.Finally,the
tuberclebacillushasaveryslowgenerationtime.

Relatedquestions
11 Iscellmediatedimmunityorhumoralimmunitymoreimportantforfighting
tuberculosis?Why?
Becausethetuberclebacillusresidesintracellularlyinmacrophages,cellmediatedimmunityismore
importantbecauseittargetsintracellularpathogens.

12 Howdoesthepurifiedproteinderivativeskin(Mantoux)testwork?
PPDismadefromthebacterialcellwallofM.tuberculosis.Wheninjectedintoanindividualwhose
immunesystemhasbeenexposedtothetuberclebacilli,thePPDelicitsatypeIVhypersensitivity
response,whichmanifestsasaninduratedareaatthesiteofinjectionwithinabout48hours.

13 Whyisreactivationtuberculosismorelikelytooccurintheapicallungs
ratherthaninthelowerlobes?
Becausemycobacteriaareobligateaerobes,thehigheroxygentensionintheapexofthelung
facilitatestheirgrowththere.However,primaryinfectionsaremorelikelytooccurinthelower
segmentswherethebacteriaareinitiallydeposited.

14 Whattypeofnecrosisisassociatedwithgranulomatouscelldeathin
tuberculosis?
Caseousnecrosis,whichhasacheesywhiteappearance.Forboards,othertypesofnecrosisinclude
liquefactive(e.g.,stroke),coagulative(e.g.,MI),fat(e.g.,pancreatitis),andgangrenousnecrosis(e.g.,
bacterialinfection).
Note:TBistheonlygranulomatousdiseaseassociatedwithcaseousnecrosis.Othergranulomatous
diseases(e.g.,syphilis,catscratchfever,leprosy,Crohn'sdisease,CGD,Wegener'sgranulomatosis,
berylliosis,sarcoidosis,systemicfungalinfections,Listeriainfection,andforeignbodies)are
noncaseating.

15 Whattypeofsecondaryinfectioncanbeseeninpulmonarycavitationsuchas
thatassociatedwithtuberculosis?
Aspergilluscancolonizeinpreviouslyformedlungcavities.Thesecoloniesareoftencalled
aspergillusballsorfungusballs.

16 Howcantuberculosiscauseaurinalysistoshowmicroscopicpyuriaand
hematuria(withredbloodcellcasts)inthefaceofasterileculture?
HematogenousspreadofTBtothekidneyscancausepyelonephritis.TBisnotoriouslydifficultto
culture,andtheurineisnotculturedroutinelyunlessspecificallyrequested.

17 WhymightPott'sdiseasebesuspectedinapatientwithtuberculosiswhohas
newonsetbackpainbutdeniesanytraumathatmightexplainthepain?
HematogenousspreadofTBtothespinecanleadtovertebralosteomyelitis,referredtoasPott's
disease.

Mostcasesoftuberculosis(TB)intheUnitedStatesareduetorecurrenceoflatent
infections.
TBisalwaystreatedwithdrugcombinationsbecauseresistancewilldevelopifonlya
singledrugisgivenatatime.
CellmediatedimmunityisthemostimportantdefenseagainstTB.
SummaryBox:Tuberculosis

Case217
A21yearoldwomanpresentstoyourclinicbecauseofabdominaldiscomfort,whichshe
describesasbecomingincreasinglysevereoverthepastweek.Shehasalsonoticedayellow,
malodorousvaginaldischargeaswellasoccasionalvaginalbleedingfollowingsex.Itis
becomingmoreuncomfortableforhertourinate,buttherehasbeennochangeinurgencyor
frequency.Shehashadthreesexualpartnersoverthelast3monthsandusesanintrauterine
device(IUD)ascontraception,whichwasmostrecentlychanged2weeksago.

1 Whatisthemostlikelydiagnosis?
Thesesymptomsmostlikelypointtopelvicinflammatorydisease(PID).

2 Whatisuniqueaboutthechlamydialcellwall?
Thepeptidoglycanlacksmuramicacid.ThisrenderslactamantibioticsuselessagainstChlamydia.

3 WhenasampleofinfectedtissueisstainedwithGiemsa,wherewillthe
chlamydialbacteriabeseen?
Chlamydiaeareobligateintracellularorganismsbecausetheycannotmaketheirownadenosine
triphosphate(ATP),andtherefore,theywillbeseeninthecytoplasmoftheinfectedcell.Rickettsiais
anotherexampleofanobligateintracellularorganism.

4 Howispelvicinflammatorydiseasetransmittedandwhycanitleadtopelvic
discomfort,vaginaldischarge,andvaginalbleeding?
PIDistheresultofacervicalorvaginalinfectionthatascendsthefemalereproductivetracttocause
endometritisorsalpingitis.Theinflammationoftheuterineliningorthefallopiantubesleadstothe
pelvicdiscomfort.Theoriginalinfectionofthelowerreproductivetractandtheresulting
inflammatoryresponsecanresultindischarge.Theinfectedepitheliumismorelikelytobleedwith
evenmildcontact.

5 Whatarethetwoorganismsthatcouldmostlikelycausehersymptoms?
PIDismostlikelycausedbyChlamydiatrachomatisorNeisseriagonorrhoeae.

6 Whattestwouldyoudotodifferentiatebetweenthesetwoorganismsand
why?
AGramstainwouldbethebesttesttodifferentiatebetweenthesetwobecauseC.trachomatisisan
obligateintracellularorganismwhereasN.gonorrhoeaeisagramnegativediplococcus.

7 HowareChlamydiatrachomatisandNeisseriagonorrhoeaetransmitted?
Botharetransmittedbycontactwithinfectedgenitals,mostcommonlyviasexualcontactoratbirth.

Case217continued:
Thelaboratoryresultsreportthepresenceofcytoplasmicinclusionsbutnogramnegative
diplococci,leadingtothediagnosisofC.trachomatisinfection.

8 Whatshouldbeprescribedasatreatmentforyourpatient?
Chlamydialinfectionsrespondbesttotetracyclines.AzithromycinismostoftenusedforC.
trachomatis.BecausepatientswithChlamydiaareatriskforsimultaneousgonorrhealinfection,
youshouldalsotreatthemwithceftriaxone.

9 Ifthepatient'scurrentandpastpartnersdonothaveanysymptoms,should
theyalsobeconsideredfortreatment?
Yes.Anyonewhohashadsexualcontactwiththepatientinthe60daysleadinguptohersymptoms
shouldalsobetreated.C.trachomatisgenitalinfectionsareoftenasymptomaticandarean
importantreservoirforcontinuingtheinfectiouscycle.Despitethefactthattheycanbe

asymptomatic,chlamydialinfectionscanstillleadtosterilityinwomen,mostoftenduetothe
inflammatoryeffectsonthefallopiantubes(salpingitis).

10 Whyisthefactthatthepatientwasusinganintrauterinedevicesignificantin
thiscase?
AnIUDmayhelptheinfectiontoascendfromthelowerreproductivetractintotheendometriumof
theuteruswhentheIUDisinserted.

11 Whatareotherriskfactorsforthedevelopmentofpelvicinflammatory
disease?
Anyactthatmayhelpthepassageofaninfectionfromthelowergenitaltractintotheuppergenital
tract,includingdouching,abortingapregnancy,andparturition,isariskfactor.

12 Ifthispatientwasnotusinganybirthcontrolandhadbeentryingtobecome
pregnant,whatotherconcernswouldyouneedtotakeintoaccount?
PIDincreasestheriskofectopicpregnancyandcanalsoleadtoinfertilityduetoscarringofthe
fallopiantubesasasequelaoftheinflammatoryresponse.

13 WhatisReiter'ssyndrome?
ThisautoimmunediseaseiscausedwhentheantibodyiesformedagainstC.trachomatisreact
againstantigensontheurethra,joints,anduvealtract.Thisresultsintheclassictriadofurethritis,
arthritis,anduveitis.RememberthatReiter'ssyndromeisassociatedwithchlamydialinfection.

14 Describetheuniquelifecycleofachlamydialinfection
Infectionbeginswhenanelementarybodyattachestoandentersanepithelialcell.Theelementary
bodywillthentransformintoareticulatebody,whichwilldividemanytimesbybinaryfission.The
manyreticulatebodieswillthenbeorganizedintoelementarybodies,anditisatthispointthatthe
cytoplasmicinclusionbodiesmaybeseenmicroscopically.Theelementarybodieswillbereleased
fromthecell,andeachisthencapableofinfectinganotherepithelialcell(Fig.2114(f0075)).

Figure2114
LifecycleofChlamydiaspp.
(FromCohen,PowderlyWG,BerkleySF,etal:InfectiousDiseases,2nded.Edinburgh,Mosby,2004.)

15 WhataretheserotypesofChlamydiatrachomatisthatcancausepelvic
inflammatorydisease?
PIDiscausedbyserotypesDthroughK.SeeTable2124(t0125)fordiseasescausedbyotherC.
trachomatisserotypes.
Table2124
SerotypesofChlamydiaTrachomatis

Serotypes Disease
A,B,C

BlindnessinAfricaduetochronicinfections

DK

Pelvicinflammatorydisease,neonatalpneumonia,neonatalconjunctivitis

L1,L2,L3 Lymphogranulomavenereum

16 WhataretheotherspeciesofChlamydiaandwhatdiseasesdotheycause?
Table2125(t0130)presentsthediseases,modesoftransmission,andrecommendedtreatmentfor

Chlamydiaspecies.
Table2125
ChlamydialSpeciesandAssociatedDiseases

Species

Disease

Transmission

Treatment

Chlamydia

Reactivearthritis,

Sexualor

Tetracyclineor

trachomatis

nongonococcalurethritis(NGU),

passage

erythromycin(eyedrops

conjunctivitis,blindness,

throughbirth

forneonatalconjunctivitis)

lymphogranulomavenereum

canal

Atypicalpneumonia

Aerosol

Chlamydia
pneumoniae
Chlamydia

Tetracyclineor
erythromycin

Atypicalpneumoniawithavianreservoir

psittaci

Aerosol

Tetracyclineor
erythromycin

Chlamydiaisanobligateintracellulardisease.
Therearetwophasesofthechlamydiallifecycle.Theelementalbodywillinfectnew

cellsandthereticulatebodywilldividewithinthecell.
SerotypesL13causelymphogranulomavenerum(LGV),serotypesACcauseblindness,
andserotypesDKcausepelvicinflammatorydisease(PID).
SummaryBox:Chlamydia

Case218
A20yearolduniversitymusicmajorisbroughttoyourclinicbyoneofhisroommates,who
reportsthatthepatientwascomplainingofaheadachelastnightandwasconfusedwhenhe
wasawakenedthismorning.Uponquestioning,thepatientknewhisnamebutthoughtthathe
wasinadifferentcityandthattheyearwas2008.Hereportshavingasevereheadacheand
asksforthelightstobeturneddownintheoffice.Hisroommatesaysthathehasnohistoryof
migrainesandthattheyhaveknowneachotherforthelast3years.Histemperatureistaken
andshowntobeelevatedat38.7C.Onexamination,hehaspositiveBrudzinskiandKernig
signs.

1 Whatisthemostlikelydiagnosis?
Thispatientmostlikelyhasmeningitis.

2 Whatistheclassictriadofsymptomsassociatedwithmeningitis?
Theclassictriadofmeningitisisfever,nuchalrigidity,andalteredmentalstatus(confusion).
However,onlyaboutonethirdofpatientswithmeningitiswillpresentwithallthreeofthese
symptoms.Photophobiaandheadachecancommonlybeseeninmeningitisbutarenotconsidered
partofthetriad.

3 WhataretheBrudzinskiandKernigsigns?
TheBrudzinskisignisperformedbypassivelyflexingtheneckwhilethepatientissupinethetestis
consideredtobepositiveifthepatientspontaneouslyflexesthehips.TheKernigsignisperformedby
flexingthehipwiththekneeflexedandthenhavingthepatientextendatthekneewhilekeepingthe
hipflexed.Ifthepatientisreluctanttofullyextendbecauseofnuchaldiscomfort,thetestispositive.
Bothtestsassessfornuchalrigiditysecondarytomeningealinflammation.

4 Whatarethepossiblecausesformeningitis?Whattestscanbedonetomake
thediagnosisofmeningitisandidentifythecausativeagent?
Meningitiscanbecausedbybacteria,viruses,orfungi.Alumbarpunctureshouldbeperformedto
findthecause.AGramstain,aswellasotherlaboratorytests,isorderedtoassessthecerebrospinal
fluid(CSF).UseTable2126(t0135)toquizyourselfonthedifferentCSFfindingsforeachcauseof
meningitis.
Table2126

CerebrospinalFluidFindingsinMeningitis

InfectiveAgent WBCDifferential CellType

Protein Glucose OpeningPressure

Bacterial

PMNs

Viral

Normal

Lymphocytes Normal Normal

Normal/

Fungal

Normal/

Lymphocytes

PMNs,polymorphonuclearneutrophils(leukocytes)WBC,whitebloodcell.

5 Whatarethemostcommoncausesofmeningitisbyagegroup?UseTable21
27(t0140)toquizyourself
Table2127
CausesofMeningitisbyAge

02Years

218Years

1860Years

60+Years

Escherichiacoli

Neisseriameningitidis

N.meningitidis

S.pneumoniae

GroupBstreptococci

Streptococcuspneumoniae S.pneumoniae

Listeriamonocytogenes Haemophilusinfluenzae

H.influenzae

L.monocytogenes
N.meningitidis

L.monocytogenes GroupBstreptococci
H.influenzae

6 Haemophilusinfluenzaeusedtobethemostcommoncauseofmeningitisin
newbornsbutisnowonlyrarelyseeninthisagegroup.Whyhasthischanged?
NewbornsarenowgivenavaccinetoprotectthemagainstH.influenzaetypeb,butthevaccineis
effectiveonlyforaboutthefirst2yearsoflife.

7 Whenwouldbeanappropriatetimetoinitiateantibiotictherapyinthis
patientandwhatantimicrobialagentcouldbeused?
Antibiotictherapymustbeinitiatedimmediatelywhenbacterialmeningitisissuspected.Basedon
theageofthepatientandthemorphologyontheGramstainanappropriateantimicrobialcanbe
chosen.OftenacombinationofIVvancomycinandceftriaxoneisusedbecauseoftheircentral
nervoussystem(CNS)penetrationandbroadcoverage.

8 AGramstainofthecerebrospinalfluidshowsgramnegativecocciinpairs.
Whatisthemostlikelycauseofthemeningitis?Shouldtheroommateand
contactsofthepatientbenotified?
N.meningitidisisthemostlikelycausativeagentbaseduponthisGramstainandthepatient'sage.
Contactsofapatientwithbacterialmeningitisshouldbetreatedprophylacticallywithrifampin.

9 Inapatientwithhumanimmunodeficiencyvirus(HIV),whatinfectiveagents
maybemorelikelytocausemeningitisthaninapatientwhohasafully
competentimmunesystem?
InapatientwithHIV,opportunisticinfectionssuchastoxoplasmosis,Cryptococcus,andJCvirus
mustbeconsideredinthedifferentialdiagnosis.

Theclassictriadinmeningitisisfever,nuchalrigidity,andalteredmentalstatus.
Thecharacteristicsofspinalfluidcanbeusedtoidentifythecauseofmeningitis.
Vancomycinandceftriaxoneareoftenusedtotreatbacterialmeningitisbecauseoftheir
goodcentralnervoussystem(CNS)penetration.
GroupBstreptococciandEscherichiacoliareimportantcausesofneonatalmeningitis.
SummaryBox:Meningitis

Copyright2015Elsevier,Inc.Allrightsreserved.

BOOKCHAPTER

BehavioralSciences
ThomasA.BrownMDandSonaliJ.Shah
USMLEStep1Secrets,Chapter24,722735

BehavioralsciencesoftenareoverlookedbymedicalstudentstakingtheUSMLEStep1,butin
ouropinion,thisisahugemistake.Moststudentssaythattheywishtheyhadstudiedmorefor
thissection,becauseitisoftenahugescoreboosterforthosecomfortablewiththematerial.
Expecttoseemultiplequestionsthatwillpresentethicaldilemmasandthenaskyouwhatyou
woulddointhosesituations.Theidealwaytoprepareforsuchquestionsistopracticereading
throughasmanyethicalscenariosaspossible!Inotherwords,yourbestresourceswillbethe
casesinthischapterandthosepresentedtoyouinquestionbanksoftwareprograms.Other
highyieldbehavioralsciencestopicsincludedevelopmentalmilestonesandthephysiologyand
pathophysiologyofsleep.Youshouldalsoknowaboutinformedconsent,advanceddirectives,
andcareforminors.Besuretopayspecialattentiontoexceptionsforanyrulesthatapplyto
ethicalsituations.
Insider'sGuidetoBehavioralSciencesfortheUSMLEStep1

Case241
A37yearoldmanpresentstoapsychiatristforevaluationofsymptomshebelievesmightbe
depression.Hereportstroublesleepingforthelast6months,statingheneverfeelsrestedafter7to8
hoursofsleep.Healsonoticesdecreasedconcentrationatwork.

1 Whataretheconsiderationsinthedifferentialdiagnosis?
Depression,sleepapnea,sleepdisorders(dyssomnias),adjustmentdisorder,hypothyroidism,
chronicrenalfailure,anemia,dementia,substanceabuseorwithdrawal,andanxietyshouldbe
considered.
Hehashadrelationshiptroubleswithhiswifeandisrecentlydivorced.Hereportsthatherprimary
reasonforleavinghimwasthathenolongerseemedtocareabouther,asheneverwantedtogoout
ordothethingstheyusedtodo.Sheevenwentsofarastoaccusehimofhavinganaffair.They
stoppedsleepinginthesameroom2yearsagobecausehisexcessivesnoring,withintermittent
burstsofawakeningshortofbreath,keptherupatnight.Hesayshejustdoesn'thaveenergytodo
thingsanymore.Healsorelateshavingbeenrecentlyreprimandedatworkforfallingasleep.On

examination,heisamoderatelyobese,otherwisehealthyappearingmiddleagedman.Hismental
statusexaminationisunremarkable.Hedeniesanythoughtsofsuicide,appetitedisturbances,or
feelingsofguiltorhopelessness.Hedoesfeellikehehashaddepressedmoodsincehiswifehasleft.

2 Inadditiontoadiagnosisofadjustmentdisorderwithdepressed
mood,whatsleeprelateddisorderlikelyexplainsmostofthispatient's
problem?
Thispatientissufferingfromsleepapneaandwouldappropriatelybediagnosedwithabreathing
relatedsleepdisorder.Thesepatientsareoftenobese,andacollarsizegreaterthan17inchesshould
bearedflagpresumably,theweightofthefataroundtheneckcollapsestheairway.Sleepisoften
interruptedatnightbecauseoftheoccludedairway,leadingtoexcessivedaytimesleepinessand
fatigue.Chronicpoorsleepcanleadtoirritability,poorconcentration,andtheneedtonapduring
theday.
Associateexcessivedaytimesleepinesswithnarcolepsyandobstructivesleepapnea.Both
diseasesarefavoritesontheUSMLE.
Step1secret

3 Whattreatmentcanbeemployedtoallowthismantosleepatnight?
Thetherapyusedtoallowthesepatientstosleepinvolvespressurizingtheairwaytokeepitpatent.
Thepatientwearsamaskthatprovidespositiveairwaypressuretokeeptheairwayfrombeing
obstructed.Positiveairwaypressureisonlyonetreatmentoptionforthosepatientswhosufferfrom
obstructivesleepapnea,asdoesthisman.Asalways,lifestylemodificationsareimportantaswell.
Thispatientshouldbeencouragedtoloseweight,whichshouldreducethecompressiveforcesonthe
airwayandtherebydecreasetheairwayobstruction.Uvuloplastyornasalsurgerymayalsobe
indicated.
Note:Sleepstudieswillshowapneicepisodeswithincreasingbreathingeffortagainstanobstructed
airway,frequentarousals,anddecreasedrapideyemovement(REM)sleep.

Obstructivesleepapneamaybeassociatedwithloudsnoring,difficultyconcentrating,
poormemory,andwakingupfeelingunrestedaftersleep.
Sleepstudieswillshowapneicepisodes,frequentarousals,anddecreasedrapideye
movement(REM)sleep.
Dependingonthecase,treatmentmayconsistoflifestylemodificationsandnasal
continuouspositiveairwaypressure(CPAP).
SummaryBox:SleepApnea

Case242
A29yearoldwomanpresentsfollowinganautomobileaccidentinwhichshefellasleepatthe
wheel.Shenotesthatshefrequentlyfallsasleepduringthedayandfeelsrestedafterthese
episodes.

1 Whataretheconsiderationsinthedifferentialdiagnosis?
Sleepdeprivation,primaryhypersomnia,narcolepsy,sleepapnea,substanceabuseorwithdrawal,
hypothyroidism,andanemiaareconsiderations.
Inaddition,shestatesthatsometimessheawakensbutisutterlyunabletomoveamuscle.She
statesshehasalwaysbeenabletofallasleepquickly.Shedeniesanyuseofdrugsormedications.You
excuseyourselftoanswerapageandfindherasleepwhenyoureturntoyouroffice.Onawakening
sheisstartledatfirstbutthenseemstoregainherorientationandasks,Whatiswrongwithme?

2 Whatisthelikelydiagnosisandwhatwouldbetheexpected
electroencephalographicfindings?
Thispatienthasnarcolepsy.Theelectroencephalogram(EEG)inasleepstudywouldlikelyshowa
decreasedREMlatency,meaningsherapidlyprogressesintoREMsleep.Thisaccountsforthe
restfulnessthesepatientsfeeluponfallingasleep.
Note:Patientswithprimaryhypersomniahaveacompletelynormalsleeparchitecture.

3 Whichtreatmentisavailableforpatientswithnarcolepsy?
Aregimenofaregularscheduleofforcednapsduringthedaycanbeasuccessfultreatmentforsome
patients.Inseverecasesofnarcolepsy,amphetaminessuchasmethylphenidate(Ritalin)arealso
usedinthetreatmentofnarcolepsy.Theseagentscausethereleaseofnorepinephrine,dopamine,
andserotonin,butallhavesomeabusepotential.Aneweragent,modafinil,hasbeenaddedthathas
lowerabuseliability.Modafinilappearstoselectivelydecreasesomnolenceinnarcolepticpatients
however,themechanismofactionisunknown.

4 Whatarethestagesofsleepandwhathappensphysiologicallyinthese
stages?
SleepisdividedintononREM(NREM)andREMsleep.NREMsleepisdividedintofourstages,each
beingadeepersleep.Thestagesarefurtherdescribedasfastwaveorslowwavesleep.Theearliest
twostagesarefastwavesleepandstages3and4aretermedslowwavesleepbasedontheEEG
appearanceofbrainwaves.REMreferstorapidconjugateeyemovement.Asapersonfallsasleep,he
passesthroughstages1to4andthenentersREMsleepthefirsttime,normallyafterapproximately
90minutes.ThefirstREMepisodelaststypicallylessthan10minutes,andthenthepersoncycles
throughthestagesagain,withfurtherREMepisodesofabout15to40minuteseach.

Physiologically,duringNREMsleep,aperson'spulse,respirationrate,andbloodpressureare
decreasedandshowlessminutetominutevariation.Restingmuscletoneisrelaxedsomewhat,and
thereareepisodicbodymovementsduringNREMsleep.Malesdonotexperienceerection,andblood
flow,includingcerebralcirculation,issomewhatlower.Bycontrast,REMsleepischaracterizedby
higherpulserate,respiratoryrate,andbloodpressureEEGpatternsaresimilartothoseofonewho
isawake.REMsleepisalsotermedparadoxicalsleepbecauseofitssimilaritiesonEEGtoaperson
whoisawake.Menwillexperiencepartialorfullerection.Additionally,apersoninREMsleep
experiencesneartotalskeletalmuscleparalysis,andmovementisquiterare.Abstractandsurreal
dreamsoccurduringthisphaseofsleep.MostREMsleepoccursinthelastonethirdofthenight.

5 InTable241(t0010),coverthecolumnstotheright,andforeachstage
ofsleeplistedintheleftcolumn,nametheEEGappearanceand
describethefrequencyandvoltageofthewavesseen:
Table241
Electroencephalographic(EEG)CharacteristicsofSleepStages

State

EEGAppearance Frequency

Voltage

Awake

waves

Randomfastwaves

Low

Eyesclosed waves

812cycles/sec

Low

Stage1

waves

37cycles/sec

Low

Stage2

Sleepspindles
Kcomplexes

1214cycles/sec
Low
Slow,triphasicwaves High

Stage3

waves

0.52.5cycles/sec

High

waves

Randomfastwaves

Low

Stage4
REMsleep

REM,rapideyemovement.

6 Howdonightmaresdifferfromnightterrors?
NightmaresoccuralmostexclusivelyinREMsleep.Patientswhoexperiencenightmaresareableto
recalltheeventsofthesefrighteningevents,whichusuallyinvolvethreattolife,security,orself
esteem.Uponawakening,thepersonrapidlybecomesoriented.NightterrorsoccurindeepNREM
sleep(stages3and4).Often,thepersonwakeswithapanickyscream.Thesepatientsareoften
unresponsiveuponawakening,haveamnesiafortheepisode,andshowsignsofautonomicarousal,
suchastachycardia,tachypnea,anddiaphoresis.Nightterrorscanbetreatedwithbenzodiazepines.

7 An82yearoldwomancomplainsthathersleeppatternshavechanged
asshehasaged.Whatchangesinsleeparetypicalaspeopleage?

Thoughthisissomewhatcontroversial,forthepurposeofboardsyoushouldassumethataspeople
age,theyexperienceadecreaseintheamountoftimeinslowwavesleep(stages3and4)andREM
sleep.Thistypicallyresultsinareducedneedfortimespentsleeping.Insomniaiscommoninthe
elderlypopulation.

8 Thiswomanhadbeengivenabenzodiazepinetoassisthersleep,which
improvedforawhile,butnowshecomplainsofpoorsleeponcemore.
Whyhavehersleepproblemsreturned?
Sheisexperiencingtolerancetotheeffectsofhermedication.Benzodiazepinesmaybeusedfor
shorttermmanagementofinsomnia,especiallywhenthereisanidentifiableprecipitant,butnotfor
longtermmanagement,becausetoleranceanddependencemayresult.Reevaluationshouldfollowa
7to10daytrialofabenzodiazepine,andotheragentsshouldbeconsidered.

9 Howdobenzodiazepinesmanifesttheirpharmacologiceffect?
Benzodiazepinesareagonistsofaminobutyricacid(GABA)receptors,whichareboundtochloride
channels.GABAistheprimaryinhibitoryneurotransmitterinthecentralnervoussystem(CNS).
ThisCNSinhibitionleadstodecreasedalertness,drowsiness,andlessagitation.

10 Whywouldthisbeanotherreasonbenzodiazepinesshouldbeavoided
intheelderlypopulation?
Theagedpopulationhasamarkedlyincreased(about25%)incidenceoffallswhengiventhesetypes
ofmedications,duetodrowsinessandimpairedbalance.Thiseffectwouldbeespeciallyconcerning
inthiselderlypostmenopausalwoman,whomayhaveunderlyingosteopeniaorfrankosteoporosis.
Additionally,thegeriatricpopulationismoresensitivetothiseffectandshouldbestartedonalower
doseinitially.

11 Therearenowanumberofdrugsotherthanbenzodiazopinesthat
alsoactontheaminobutyricacidbenzodiazepinereceptorandthat
reachhypnoticeffectswithlesstoleranceandlessdaytimesedation.
Whataresomeexamplesofthese?
Zaleplon,zolpidem,andeszopicloneareexamplesofthesedrugs.
Note:Sedatingantidepressantssuchastrazodoneandnefazodone(rememberthezzzzzzzzgroup)
mayalsobeused.

12 Whenevaluatingapersonforsleepproblems,perhapsthefirstand
mostimportantstepistomakesurethatthepatienthasgoodsleep
hygiene.Whatdoesgoodsleephygieneentail?
Noalcohol
Nocaffeineornicotine

Regularexercise(butnottoolateintheday)
Relaxingactivitybeforebed(e.g.,bath,reading)
Onlysleepandsexinthebedroom(noTV)
Noclockwatching
Nodaytimenaps
Nolatemeals

Narcolepsy:daytimesleepiness,sleepattacks,decreasedrapideyemovement(REM)
latency,restedaftersleep
Primaryhypersomnia:excessivesomnolence(>1month),normalsleeparchitecture
Treatmentfornarcolepsyandprimaryhypersomnia:methylphenidateormodafinil
Insomnia:difficultyinitiatingormaintainingsleep
Nonbenzodiazopinesactingatthebenzodiazepineaminobutyricacid(GABA)receptor
maybethebesttreatmentforinsomniabecausetheyallowforhypnoticeffectswithless
toleranceandlessdaytimesedation.
Summarybox:sleepdisturbance

Case243
An80yearoldmanwithseverepulmonarydiseaserequiresalungtransplant.Soonafterthe
surgery,thepatientdevelopsrespiratoryfailure,whichrequireshimtoreceivemechanical
ventilation.Whenthepatient'sfamilyasksthedoctorhowlongthepatientwillrequire
ventilation,thephysicianrespondsthatheisunsure,butthatitislikelytobemaintainedforan
extendedperiodoftime.Thepatient'swifeexpressestoyouthatherhusbandhastoldher
manytimesthathewouldnotwishtobekeptaliveonmechanicalventilation.Atthismoment
thepatient'soldestson,whocurrentlysupportshisparents,demandsthathisfatherbekepton
mechanicalventilationuntilanalternativesolutioncanbefound.

1 Whatshouldthephysiciando?
Thephysicianshouldterminatemechanicalventilation,accordingtothepatient'sownwishes.

2 Whatareadvancedirectives?

Advancedirectivesareinstructionsprovidedbyapatientinanticipationoftheneedforadecisionto
bemaderegardinghisownmedicalcare.Theycanbeoral,written(e.g.,livingwill),orintheformof
adurablepowerofattorney.Adurablepowerofattorneyisresponsibilityassignedtoapersonbythe
patienttomakemedicaldecisionsonhisorherbehalfintheeventthathe/shelosesthecapacityto
doso.Statementsmadetoothersbythepatientcanqualifyasoraladvancedirectives.Theygain
morevalidityiftheywererepeated,heardbymultiplepersons,andrecent.Althoughoraladvance
directivesprovidemoreflexibilitythanwrittendirectives,problemsmayarisefrominaccurate
communicationofthepatient'swishesordeviationsininterpretation.

3 Howiscompetence(decisionmakingcapacity)defined?
Thepatientmustbeinformed(providedwithadequateinsightregardingalloptions),abletomake
andcommunicateastablechoice,andfreefromtheinfluenceofothers.Thedecisioncannotresult
fromdelusionsorhallucinations.

4 Whatissubstitutedjudgment?
Ifamedicaldecisionmustbemadeonbehalfofanincompetentpatientwhodoesnothaveany
advancedirectivesinplace,theruleofsubstitutedjudgmentcanbeused.Thephysicianandthe
patient'sfamilymemberscanmakeadecisionforthepatientbasedonwhattheywouldexpectthat
thepatientwouldhavewanted.Thepersonalwishesofthephysicianorfamilymembersshouldnot
affectthisdecision.

Apatientisconsideredtobecompetentifhe/sheisinformed,abletomakeand
communicateastabledecision,andisfreefromtheinfluenceofdelusions,hallucinations,
orotherindividuals.
Advanceddirectivescanbewritten,oral,orintheformofadurablepowerofattorney.
Substitutedjudgmentcanbeappliedwheneveranincompetentpatientdoesnothave
anyadvanceddirectivesinplace.Inthiscircumstance,thephysicianandthepatient's
familycanmakeadecisiononbehalfofthepatientaccordingtowhattheyexpectthe
patientwouldhavewanted.
Summarybox:advanceddirectives

Case244
A15yearoldgirlcomesintoyourofficeaskingforbirthcontrol.Sheadmitsthatherparentsdo
notknowthatsheissexuallyactive,andsheimploresyounottellthem.

1 Whatshouldyoudo?

Writetheprescriptionandagreenottotellherparentsbutdiscusstherisksandbenefitsofusing
oralcontraceptiveswiththepatient.Youshouldalsoencouragethepatienttocommunicatewithher
parents.

2 Whataretherulesregardingparentalconsentforminors?
Parentalconsentisrequiredforminorsundertheageof18,unlesstheminorisemancipated
(married,selfsupporting,orinthemilitary).Thereare,however,severalsituationsinwhich
parentalconsentisnotrequired.Thesesituationsincludeemergencies,prescriptionoforal
contraceptives,pregnancyrelatedmedicalcare,andtreatmentofsexuallytransmitteddiseases
(STDs)ordrugproblems.

Parentalconsentmustbeobtainedunlesstheminorisemancipated.
Exceptionstothisruleincludeemergencysituations,drugabuse,pregnancyrelated
medicalcare,prescriptionoforalcontraceptives,ortreatmentofsexuallytransmitted
diseases(STDs).
Summarybox:consentforminors

Case245
Apatientcomesintoyourofficewithdepressivesymptoms.Youworkwithheroverthenext
fewmonthstotreatherforherdepression.Duringafollowupvisit,sheexpresseshergratitude
foryourdevotionandassistanceandsaysshewouldliketomakeituptoyoubytakingyouout
todinner.Shewinks,andyouunderstandthatsheintendsittobeadate.Althoughyoudonot
admitittoher,youfindthatyouareindeedattractedtoheraswell.

1 Whatdoyoudo?
Itisneveracceptableforyoutohavearomanticrelationshipwithyourpatients.Youshouldpolitely
declineherinvitationandcontinuetoseeherasyourpatient.Itisnotnecessarytoreferherto
anotherphysicianifyoucancontinuetobeprofessional,butitwouldbeagoodideatoinvitea
chaperoneintotheoffice.

2 Isitagoodideaforyoutobehonestandtellherthatyoucannothavea
relationshipwithherwhilesheisyourpatient?
No.Thiswouldsendthemessagethatifyourprofessionalrelationshipwereterminated,youwould
bewillingtopursueapersonalrelationshipwithher.

WhenevertheUSMLEasksyouwhattodoinasituationsimilartotheoneinCase245(b0060)
,theywilloftentrytoenticeyouwithananswerchoicethatsuggestsyoureferthepatientto
anotherphysician.Forthepurposeofboards,thiswillalmostneverbecorrect.Thecorrect
choicewillrequireyoutobeanactiveparticipantinthesolution.
Step1secret

Thephysicianpatientrelationshipshouldneverextendbeyondprofessionalboundaries.
Undernocircumstancesisitacceptabletopursuearomanticrelationshipwithapatient.
Intheinstancethatapatientbreachesthisboundary,yourbestcourseofactionisto
continuetoseethepatientbuttoclarifytheprofessionalnatureofyourrelationship.Itis
notnecessarytoreferthepatienttoanotherphysician,butyoumaywanttobringa
chaperoneintotheofficeduringfutureappointmentswiththispatient.
Summarybox:thephysicianpatientrelationship

Case246
Apatientconfidestoyouthathehasbeencheatingonhiswifeandnowsuspectsthathemaybe
infectedwithhumanimmunodeficiencyvirus(HIV).Youperformtheappropriatetests,which
allturnouttobepositive.YoutellthepatientthatyouwilltreathimforHIV,butthatitishis
responsibilitytotellhispartner.Heimmediatelybreaksdownandtellsyouthathecannottell
hiswifeandallothersexualpartnersbecausehiswifewillleavehimonceshefindsoutthathe
acquiredHIVwhilecheatingonher.

1 Whatdoyoudo?
PatientswhoareHIVpositivehaveadutytoprotecttheirsexualpartnersfromacquiringthe
infection.Ifthepatientfailstodoso,thephysicianislegallyallowedtoinformthepatient'spartner.

2 Underwhatotherconditionsisitacceptabletoviolatepatient
confidentiality?
Patientconfidentialityshouldbemaintainedunlessthepatientisatsignificantriskforsuicideor
posesarisktoanotherindividual.Thephysiciancanalsointerveneintheinstanceofchildorelder
abuse.
Note:TheTarasoffdecisionprovidesphysicianswiththelegalabilitytowarnatargetedvictimand
notifytheappropriateofficialsifapatientposessignificantrisktoanotherindividual.

Patientconfidentialityshouldbemaintainedunlessapatientisatriskforsuicideor
harminganotherindividual.
Itistheresponsibilityofthepatienttowarnanysexualpartnersifhe/sheacquiresalife
threateningsexuallytransmittedinfectionsuchashumanimmunodeficiencyvirus(HIV).
Ifthepatientfailstoprotecthis/hersexualpartnersfromacquiringtheinfection,the
physicianisentitledtoinformthemdirectly.
Summarybox:patientconfidentiality

Case247
A70yearoldobesemanwithahistoryofcongestiveheartfailureandnewlydiagnosed
depressioncomesintoyourofficebecausehecannolongersustainanerection.Heseems
upset,becausethisisgreatlyaffectinghissexlife.Headmitsthatheistooembarrassedto
discussthisproblemwithhiswife.

1 Whatisthedifferentialdiagnosisforthispatient'ssexualdysfunction?
Drugeffects(betablockers,selectiveserotoninreuptakeinhibitors[SSRIs],ethanol),diseases
(atherosclerosis,depression,diabetes,decreasesintestosteronelevels),andpsychologicaleffects
(e.g.,performanceanxiety)canleadtosexualdysfunction.
Giventhisman'shistoryofcongestiveheartfailure,itislikelythathehasbeentakingbetablockers
forsometime.HewasalsonewlydiagnosedwithdepressionandmayhavebeengivenanSSRI.Side
effectsofbothofthesedrugsincludesexualdysfunction.Thisman'sageandobesityputhimatrisk
foratherosclerosisanddiabetes,whichcanalsocontributetosexualdysfunction.Performance
anxietymustbeincludedinthedifferentialdiagnosis,particularlyifhecansustainerectionsat
certaintimesoftheday(e.g.,intheabsenceofhispartner).Asthephysician,youshouldincludethis
questioninyourmedicalhistorytaking.

2 Whatchangesoccurintheelderlywithregardtosexualhealth?
Menareslowertoachieveerectionsandejaculationandhavelongerrefractoryperiods.After
menopause,womenexperiencevaginaldrynessandirritation.Unlesspatientsareonparticular
medications,libidodoesnotdecrease.Neverassumethatyourelderlypatientsarenotinterestedin
sex.Ifyoudonotincludesexualhealthinyourhistoryandphysicalexamination,theymaybetoo
timidtobringuptheirconcernsontheirown!

Elderlymenmaybeslowertoachieveerections/ejaculationandmayexperience
increasedrefractorytime.

Postmenopausalwomenmayexperiencevaginaldrynessandirritation.
Forthepurposeofboards,sexualinterestdoesnotdecreaseintheelderly.
Sexualdysfunctionmaybeattributedtodrugeffects,disease,orpsychologicaleffects.
Summarybox:sexualhealthintheelderly

Case248
A24yearoldpatientcomesintoyourofficewithflulikesymptoms.Yoususpectaviral
infectionandtellthepatienttorestandtakeplentyoffluids.Hebecomesirritatedwiththis
adviceanddemandsthatyouprescribehimantibioticssothathecangetoverhissickness
beforehisvacationthefollowingweek.Youhesitatebecauseyouknowthatantibioticswouldbe
ofnobenefittothecourseofthispatient'sillness.

1 Whatshouldyoudo?
Askthepatientwhyhefeelsheneedstheantibiotics,andpolitelyexplainwhyyoufeelthatitis
unnecessarytoprescribethem.Thepatientmaybecomeargumentative,andyoushoulddoyourbest
toavoidconflictbutalwayskeepinmindthatitisyourdecisionwhetherornottoprescribe
medicationtoapatient.Avoidwritingunnecessaryprescriptions.

Avoidwritingaprescriptionforapatientifyouasthephysiciandonotconsiderthe
medicationtobeanappropriatetreatment.
Summarybox:patientrequestedprescriptions

Case249
Youareworkingalongsideasecondyearresidentduringyourinpatientmedicinerotation.
Overthepast2weeks,youhavenoticedabruptchangesintheresident'sdressandbehavior.
Heoftenarrivestoworklateandungroomed.Youhavealsonoticedthathisbreathfrequently
smellslikealcohol.Yoususpectthathehasbeendrinkingheavilybeforeandafterworktotake
theedgeoffhisday.

1 Whatdoyoudo?
Itisyourresponsibilitytoprotectpatientsfromreceivinginadequateornegligentcarefroman
impairedorincompetentmedicalprofessional.Youshouldinformtheattendingphysicianincharge
oftheresidentofyoursuspicions.Donotattempttoconfronttheresidentyourself.Inthissituation
itisbettertoleavetheattendinginchargeofgettingtheresidentthehelpthathemayneed.

2 WhatistheCAGEquestionnaire?
TheCAGEquestionnaireisawidelyusedmethodforscreeningforalcoholabuse.Youareexpectedto
knowthisacronymforboards.Ifapatientrespondswithyestomorethanoneofthefollowing
questions,thepatientshouldbeexaminedfurtherforalcoholism.
HaveyoueverfeltlikeyoushouldCutdownonyourdrinking?
AreyoueverAnnoyedbypeoplecriticizingyoufordrinking?
HaveyoueverfeltGuiltybecauseofyourdrinking?
Haveyoueverneededadrinkfirstthinginthemorning(Eyeopener)togetoutofbedor
startyourday?

Itisyourresponsibilitytoprotectpatientsfromreceivingcarefromanymedical
professionalwhoisundertheinfluenceofalcoholordrugs.
Ifyoususpectthatacolleaguehasbeenabusingdrugsoralcohol,informthatperson's
immediatesupervisor(residentinthecaseofamedicalstudent,attendingphysicianin
thecaseofaresident).
TheCAGEquestionnaireisoftenusedasascreeningtoolforalcoholism.Youshould
knowthecomponentsofthisacronym(seetext).
Summarybox:alcoholabuse

Case2410
Amotherbringsher2yearoldchildtothepediatrician'sofficeforawellchildvisit.Sheis
concernedthatherchildstilldoesnotspeakinfullsentences.Shealsosaysthatdespite
numerousattempts,shehasbeenunabletotoilettrainherchildeventhoughherneighbor's2
yearoldchildhashadsuccess.

1 Isthischilddevelopingnormally?
Yes.YoushouldknowthedevelopmentalmilestoneslistedinTable242(t0015).
Table242
DevelopmentalMilestones

Age

GrossMotor

FineMotor

Language

Other

Birth3

Rollsover(3

Rootingreflex

Orientstovoice

months

months)

36

Sitsup(6

Putshandstogether

Stringssyllables

Socialsmile

months

months)

(3months)

together

Mororeflexdisappears

Passesitems(6
months)
69
months

Crawls

Feedsself
Strangeranxiety

12

Walks

Stacks3blocks

Speaks13

Drinksfromacup

months

words

15

Runs

months

Walks

Speaks6words

Babinskireflexdisappears
Separationanxiety

backward
18

Climbsstairs Stacks4blocks

months

Kicksball

2years

Jumps

Stacks6blocks

(upward)
3years

Jumps

Combineswords

Brushesteethwithhelp

Uses2word

Washeshands

sentences
Stacks9blocks

(forward)
Drawscirclesand
Ridestricycle dashes

Completely

Brushesteeth

understandable

Playsboardgames
Toilettraining
Developsgenderidentity

4years

Hopsonone

Copiesstickfigure

foot

Dressesself
Playscooperativelyandwith
imaginaryfriends

5years

Drawssquaresand

triangles
Tiesshoes

Identifiescolors
Countsto5

2 Whatshouldyoutellthisconcernedparent?
Themothershouldbetoldthateverychilddevelopsdifferently,butthatherchildisontrackfor
normaldevelopment.Donotautomaticallydismissthemother'sconcernsbesurethatshefeels
comfortablecomingtoyouifshenoticesanythingelsethatsheconsidersunusual.

YoushouldknowtheinformationlistedinTable242(t0015).Thisisahighyieldtopic
forboards.
Summarybox:developmentalmilestones

Case2411
A24yearoldpatientwithtype1diabetesisadmittedtothehospitalafteraninsulinoverdose
thatresultedinhypoglycemicseizures.Yougointoseethepatientoncesheisstabilized.You
askherwhethersheusesherinsulinregularly,andshetellsyouthatshegivesherselfinjections
twiceadayaccordingtothedoctor'sinstructions.Whenyouaskherhowmuchinsulinshe
injects,sheshrugsandtellsyouthatitvaries,dependingonthefoodsheeats.Youaskherto
clarify,andshetellsyou,IgivemyselflessifIskipmealsandmorewheneverIeatjunkfood.

1 Howshouldyouhandlethissituation?
Thisisaclearexampleofanoncompliantpatientwhoisnotproperlyfollowingtheinstructionsof
hertreatmentplan.Notonlyisthispatientadministeringherinsulinincorrectly,butsheisnot
adheringtoaproperdiabeticdiet.Themostimportantthingtorememberwhendealingwitha
noncompliantpatientisthatscoldingwillbeineffectiveinpreventingfuturemishaps(andwillnever
bethecorrectansweronboards!).Instead,youmusthaveadiscussionwiththepatienttofigureout
thereasonforthenoncomplianceandworktogethertofixtheproblem.
Note:Inseverecases,patientsmaybedismissedbyaphysicianfornoncompliance.Forthepurpose
ofboards,thisisnotlikelytobethecorrectanswer.

2 Howcancompliancebeincreasedinthefuture?
Asmentionedpreviously,itiscrucialtodeterminethereasonforthepatient'snoncompliance.
Therefore,itisimportanttofigureoutwhetherthispatientisneglectingthephysician'sinstructions
because(a)shedoesnotunderstandthem,(b)itisdifficultforhertoadheretothem,or(c)shedoes
notknowtheimportanceoffollowingthem.Ifyougetthefeelingthatapatientdoesnotunderstand
thedirections,doyourbestnottoembarrassthepatient.Instead,tellthepatientthatthiscould
happentoanyoneandsimplifyyourinstructions.Havethepatientrepeattheinstructionsbackto
youwhenyouaredonesothatyouknowshehasunderstoodcorrectly.Writetheinstructionsdown
wheneverpossible.Thisisespeciallyimportanttoconsiderwheneverthepatientisnotanative
Englishspeaker.
Sometimes,itisdifficultforapatienttoadheretothetreatmentplan.Insulin,forexample,mustbe
refrigerated.Considerascenarioinwhichadiabetictravelsalotforworkanddoesnotalwayshave
accesstoarefrigerator.Heorshemightskipinsulindosagesfrequently.Onceagain,simplifythe
treatmentregimenwheneverpossible.
Itisalsoagoodideatomakesurethatthepatientunderstandswhyitisimportanttofollowa
specifiedtreatmentplan.Perhapsthispatientdoesnotunderstandwhyjunkfoodisespecially
harmfultoadiabetic,orwhyskippingmealscanleadtohypoglycemia.Perhapsshedoesnot
understandthereasonbehindregulatedinsulindoses.Educatingthepatientwillmostlikely
motivatehertofollowthetreatmentplancorrectly.

Donotattempttoscarethepatientintocomplyingwithatreatmentplan.(Forexample,itis
unethicaltoshowthepatientgraphicpicturesofgangreneandsay,Thiswillhappentoyouifyou
don'tshapeup!)

Nonadherenceisacommonhurdlefacedbyallphysicians.
Patientsshouldnotbescoldedfortheirnoncompliance.Itismoreimportantto
determinethereasonforthenoncomplianceandattempttofixtheproblem.
Neverusescaretacticsinanattempttoimproveapatient'scompliance.
Summarybox:thenoncompliantpatient

Case2412
A68yearoldmanisbroughttoyourofficebyhiswifebecauseofabdominalpain,jaundice,
andunintentionalweightloss.Acomputedtomography(CT)scanoftheabdomenreveals
adenocarcinomaoftheheadofthepancreas.Whenyouwalkintotheofficetobreakthenews
tothepatient,hiswifeaskstospeaktoyoualoneoutside.Thetwoofyoustepoutoftheoffice
andsheconfessesthatshehasafeelingyouarereturningwithbadnews.Pleasetellmefirst,
shebegs.Ifit'sreallybadIknowmyhusbandwon'tbeabletohandleit.IfIknowwhatitis,I
canhelpbreakthenewstohimintime.

1 Howshouldyouhandlethissituationwiththepatient'swife?
Itisunlawfultodiscloseapatient'smedicalinformationtofamilyorfriendswithoutthepermission
ofthepatient.Therefore,youshouldavoidrevealinganyinformationtothepatient'swifeatthis
time.Youshould,however,findoutwhythepatient'swifeissoconcernedaboutherhusband's
abilitytohandlethenews.Herconcernswillperhapsguideyourapproachtohandlingthispatient.

2 Whatshouldyousaytothepatientwhenyouwalkintotheroom?
Youshouldtellthepatientthatyouhavesomenewstodiscusswithhimandpolitelydismisshiswife
forthetimebeing.Atthispoint,youcanaskthepatientwhetherhewouldlikehiswifetobepresent.
Ifheagrees,youcaninviteherbackintotheroom.Askingthepatient'spermissionforhiswifeto
remainintheroominherpresencemightpressurehisdecision.

Itisunlawfultodiscloseanypatientinformationtofamilyorfriendswithoutexplicit
permissionfromthepatient.
Alwaysasktospeaktoapatientprivatelybeforediscussingconfidentialmedical
informationinfrontofothers.

Summarybox:disclosureofpatientinformation

Case2413
A45yearoldpatientcomesintoyourofficewithhiswifeandcomplainsthathehasbeen
experiencingafrequentsensationofhislegsfallingasleep.Hisdiscomfortcausesanurgeto
constantlymovehislimbsbecausehefeelsmuchbetterwhenheisactive.Hiswifetestifiesthat
herhusbandcontinuallyjerkshislegsinhissleep.Thisactivitydisruptsbothhisandhersleep
patterns,andbothprofessfeelingtiredthroughouttheday.

1 Whatisthemostlikelydiagnosis?
Restlesslegssyndrome(RLS)isadisorderofunknownetiologythatcausesaconstanturgetomove
inattempttorelieveunpleasantsensationsinthelowerlimbs.Ithasbeenlinkedtoseveral
conditions,includingParkinson'sdisease,rheumatoidarthritis,diabetes,kidneyfailure,andiron
deficiencyanemia.UseofcertainmedicationsmayalsotriggerRLS.However,RLScanbeidiopathic
innature.

2 Whatarethemostcommonsymptomsofrestlesslegssyndrome?
Symptomsincludeanunpleasantsensationinthelegs,urgetoconstantlymove,reliefupon
movement,andworseningofsymptomswheninactive.TypicallegmovementsassociatedwithRLS
arejiggling,pacing,tossing,rubbing,andstretching.Limbmovementsoftenoccurduringsleep.

3 Howisrestlesslegssyndrometreated?
AlthoughthereisnodirectcureforRLS,thetreatmentplaninvolvescorrectingtheunderlyingcause
oftheconditionwheneverpossible.Treatmentalsofocusesonsymptomrelief,andincludessleep
improvement,alcoholavoidance(alcoholmaytriggerRLSsymptoms),walking,andheat/coldpacks
ontheaffectedlimbs.

Restlesslegssyndrome(RLS)isassociatedwithaconstanturgetomoveinresponseto
anunpleasantsensationinthelegs.
ThereisnodirecttreatmentforRLS.Focusontreatingtheunderlyingcauseofthe
diseaseandprovidingpatientswithsymptomrelief.
Summarybox:restlesslegssyndrome

Copyright2015Elsevier,Inc.Allrightsreserved.

BOOKCHAPTER

Biostatistics
ThomasA.BrownMDandSonaliJ.Shah
USMLEStep1Secrets,Chapter25,736756

Likebehavioralsciences,biostatisticsisasubjectthatmostmedicalstudentsdonotspend
nearlyenoughtimestudyingbecauseitappearstobecommonsense.Unfortunately,students
oftenmissalotofstraightforwardbiostatisticsquestionsbecauseoflackofpractice.
Biostatisticsisoneofthehighestyieldboardssubjectsandaneasywaytoearnpointsonyour
examifyoutakethetimetounderstandtheconcepts.Thischapterintroducesyoutothetypes
ofquestionsyouarelikelytoseeonyourexamandpreparesyoutosolvetheminthemost
efficientmannerpossible.
Notethatyouwillbegivenawhiteboardtouseduringyourexampriortothestartofyourtest.
Youmaytakeupto5minutesbeforeyoubeginyourexamtowriteanythingyouwouldlikeon
yourwhiteboard.Throughoutthechapter,theformulasthatstudentsfindmosthelpfultoadd
totheirwhiteboardarehighlighted.Itwouldbeagoodideatoreviewtheseformulasandthe
informationyouplantoincludeonyourwhiteboardthedaybeforeyourexam.
Insider'sGuidetoBiostatisticsfortheUSMLEstep1

BasicConcepts
TestCharacteristics
1 Whatdoesthesensitivityofadiagnostictestmeasure?
Sensitivityisameasureofhoweffectivelyadiagnostictestcandetectthediseaseinapatientwho
trulyhasthedisease(truepositive).Inotherwords,sensitivitymeasurestheproportionof
individualswithadiseasewhotestpositiveforit.
Sensitivitycanbecalculatedbydividingthenumberoftruepositivesbythetotalnumberofpeople
testedwiththedisease:truepositives/(truepositives+falsenegatives),ora/(a+c)inthe22table
(Table251(t0010)).Youcanalsocalculatesensitivitybysubtractingthefalsenegativeratefrom1
(i.e.,sensitivity=1falsenegativerate)however,youwillseldombegiventhefalsenegativerateon
boards.

Practicesettingupthesetableswheneveryouencounterabiostatisticsproblemthatinvolves
sensitivityorspecificity.TheywillhelpyouimmenselyontheUSMLE.Itmightalsobehelpful
tocopythistableontoyourwhiteboardbeforethestartofyourexam.
Step1Secret
Table251
Sample22table

PresenceofDisease
+
TestResult + Truepositives(a)

Falsepositives(b)

Falsenegatives(c) Truenegatives(d)

2 Whatdoesthespecificityofadiagnostictestmeasure?
Thespecificityofadiagnostictestisameasureofhoweffectivelythetestcandetecttheabsenceof
diseaseinapatientwithoutthedisease(truenegative).Itisanindicationofhowspecificapositive
testresultistothediseaseitisdesignedtodetect.Thegreaterthenumberofdifferentconditions
thatcauseapositivetestresultotherthanthediseasethetestisdesignedtodetect,thelessspecific
thetest.
Specificitycanbecalculatedbydividingthenumberoftruenegativesbythetotalnumberofpeople
testedwhodonothavethedisease:truenegatives/(truenegatives+falsepositives),ord/(b+d)in
the22table).Specificityisalsoequalto1falsepositiverate.

3 Quickterminologyreview:CovertherightcolumninTable252(t0015)anddefine
eachofthetermsintheleftcolumn
Table252
BasicTerminology

Term

Definition

Truepositive

Apositivetestresultinsomeonewhotrulyhasthedisease

Falsepositive

Apositivetestresultinsomeonewhotrulydoesnothavethedisease

Truenegative

Anegativetestresultinsomeonewhotrulydoesnothavethedisease

Falsenegative Anegativetestresultinsomeonewhotrulydoeshavethedisease

4 Howdoesthesensitivityofatestrelatetoitsspecificity?

Sensitivityandspecificitymoveinoppositedirectionsastestparameterschange.Inotherwords,as
sensitivityincreases,specificitydecreases,andviceversa.Thisoccursbecause,inordertoimprove
thesensitivityofatest(i.e.,detectmorepeoplewithadiseaseofinterest),thelimitsonwhatresults
areconsideredtobepositivemustbemadelessstringent.Indetectingmorepeoplewiththedisease,
thetestwillthereforealsoyieldpositiveresultsinmorepeoplewithoutthedisease.
Forexample,therheumatoidfactor(RF)isoftenusedtoaidinthediagnosisofrheumatoidarthritis
(RA).RFispositivein70%ofpatientswithRA.IfyouwanttocatchmorecasesofpatientswithRA,
youcanusetheerythrocytesedimentationrate(ESR).TheESRispositivein90%ofpatientswith
RA.However,withthisincreasedsensitivitycomesdecreasedspecificity.ESRisverynonspecificand
canbepositiveinanyinflammatoryprocess,frompneumoniatotemporalarteritis.Givenitshigh
sensitivity,anegativeESRishelpfulinrulingoutinflammatorydisease(seelaterdiscussion).
SPINandSNOUTareusefulmnemonics.SPINtellsusthatspecifictestsruleindisease.Thatis,the
morespecificatest,themorelikelyitisthatapositiveresultindicatesrealdisease.SNOUTtellsus
thatsensitivetestsruleoutdisease.Thatis,themoresensitiveatest,themorelikelythatanegative
resultrulesoutdisease.Inseriousdiseasesthatcanbetreatedeffectivelyifdetected,agreater
sensitivityisdesired(oftenattheexpenseofspecificity).
SPIN:SPecifictestsruleIN
SNOUT:SeNsitivetestsruleOUT

5 Whatinformationisgivenbytherelativerisk?
Therelativerisk(RR)isaratiothatcompareseventratesinonegroupversusanother.Itcanbe
calculatedbydividingtheprobabilityofoccurrenceofdisease(incidence)intheexposedgroupby
theprobabilityofoccurrenceofdiseaseintheunexposedgroup.
Forexample,inthe22table(Table253(t0020)),theprobabilityoflungcancerinsmokersis
90/100or.90.Theprobabilityoflungcancerinnonsmokersis10/100,or.10.Therefore,theRRfor
smokingis.90/.10,or9.AnRRof9impliesthatsmokersare9timesmorelikelytogetlungcancer
thannonsmokers.
Table253
SmokingExposureandLungCancer

LungCancer
+

SmokingExposure + 90

10

10

90

AnRRof>1meansthattheeventismorelikelytooccurintheexposedgroup.AnRRof<1means
theeventislesslikelytooccurintheunexposedgroup.AnRRof1meansthatthereisnodifference
betweenthegroups.

6 Whatinformationisgivenbytheoddsratio?
Theoddsratio(OR)alsocompareseventratesbetweentwogroupsbutiscalculatedbycomparing
oddsratherthanprobabilities.
Fortheprecedingexample,theORcanbecalculatedbydividingtheoddsofsmokersdeveloping
lungcancer(90:10)bytheoddsofnonsmokersdevelopinglungcancer(10:90),asfollows:
90/10

OR =

10/90

9090
1010

9
1

AswithRR,anORof1meanstheeventwasequallylikelyinbothgroups,anORof>1meansthe
eventwasmorelikelytooccurintheexposedgroup,andanORof<1meanstheeventwasmore
likelytooccurintheunexposedgroup.
YoumustknowthatRRandORsareusedfordifferenttypesoftests.RRisusedforcohortstudies
andrandomizedcontrolledtrials(studytypesthatmeasureriskofoutcomebasedonexposure
status)becausethosestudiesallowforcalculationofthepercentagesofparticipantsthatareaffected.
ORsareusuallyusedincasecontrolstudies,whichapproximateoddsofexposurebasedonratesof
outcomeoccurrenceinwhichpercentagescannotbecalculated.Youwillreadmoreabouttypesof
testslaterinthischapter.
TheRRandORsforonestudymaybedrasticallydifferent(asintheprecedingexample)forcommon
eventsbutapproximateeachotherforrareevents.

7 Whatisthedifferencebetweenprobabilityandoddsandhowaretheymeasured?
Probabilityismeasuredalongacontinuumfrom0to1,where0meanstheeventiscertainnotto
happenand1meanstheeventiscertaintohappen.
Oddsaremeasuredalongacontinuumfrom0toinfinity.
Probabilitiescanbeeasilyconvertedtoodds,andviceversa.
Odds = Probability/1 Probability

and
Probability = Odds/1 + Odds

Ifthepretestprobabilityofacertaindiagnosisis90%,thentheoddsare.9/1.9=9(sometimes
statedas9to1).

8 Whatisthepositivepredictivevalue?negativepredictivevalue?

Thepositivepredictivevalue(PPV)istheprobabilitythat,givenapositivetestresult,thediseasein
questionisactuallypresent.Thenegativepredictivevalue(NPV)istheprobabilitythatdiseaseis
absentifthereisanegativetestresult.Ifa22tableisprovided,thePPVcanbecalculatedby
dividingthenumberoftruepositivesbythetotalnumberofpositivetestresults(a/a+b)andthe
NPVcanbecalculatedbydividingthenumberoftruenegativesbythetotalnumberofnegativetest
results(d/c+d)(Table254(t0025)).
Table254
CalculatingPredictiveValuesUsingthe22Table

PresenceofDisease
+

TestResult + Truepositives(a)

Falsepositives(b)

Falsenegatives(c) Truenegatives(d)
Becausea22tableisnotnormallyprovided,thePPVcanalsobecalculatedifthesensitivityand
specificityofthetestandtheprevalenceofthediseasebeingtestedareknown,asdiscussedlater.
Whathappenstothepositiveandnegativepredictivevaluesasdiseaseincidencechanges?Thisisthe
typeofquestionyoumightseeonboardsisthereasonwhysettingupthe22tablecanbeso
helpful.Ifdiseaseincidenceincreases,thenumbersintheleftcolumn(truepositives[TP]andfalse
negatives[FN])willbothincrease.BecausePPV=TP/(TP+FP),PPVwillincreaseifthenumberof
TPincrease.NPV,ontheotherhand,willdecreaseasincidenceincreasesbecauseFNincreasein
value.RecallthatNPV=TN/(TN+FN),whereTN=truenegatives.

9 Whatisthepositivelikelihoodratio?
Thepositivelikelihoodratio(PLR)reflectshowmuchapositivetestresultincreasestheprobability
ofdiseasebeingpresent.Thehighertheratio,themorelikelyitisthatdiseaseispresent.Thisratiois
calculatedasthesensitivitydividedby1specificity.Forexample,ifthesensitivityis85%andthe
specificityis90%,thePLRis0.85/(10.9)=8.5.Consequently,themoresensitiveandspecificthe
testis,thehigherthePLR.ThePLRisnowwidelyusedinevidencebasedmedicinebecauseofthe
easewithwhichitcanbeusedtocalculatethePPVofatest.

10 Howisthepositivelikelihoodratiousedtocalculatethepositivepredictivevalue?
Thisisdonebyconvertingtheprevalenceofadiseasetoanoddsratio(e.g.,20%to1:4)andthen
multiplyingthefirstpartoftheratiobythePLR(8.51:4=8.5:4).Thentheratioisconvertedback
toapercentage:8.5/(8.5+4)=0.68or68%.
Posttest probability = Pretest probability PLR

Let'sassumethepretestprobabilityofcoronaryarterydiseaseina40yearoldmanwithdiabetes
whosmokesis20%andthatthepresenceofapositiveexercisestresstesthasaPLRof10.
Expressedinodds,20%is1:4,sothePPVoftheexercisestresstestiscalculatedasfollows:
10 1 : 4 = 10 : 4, or5 : 2

5/(5 + 2) = 5/7 =. 71, or71%

Sotheprobabilitythatthismanhascoronaryarterydiseaseifhehasapositivestresstestis71%,
whichissubstantiallyupfromhispretestprobabilityof20%.

11 Whatismeantbythereliabilityofatest?
Reliability(precision)istheconsistencyorrepeatabilityofatest.Atestisreliableifitproducesthe
sameresultseachtimewhentheconditionsarethesame.Forexample,theSATexamwouldbe
consideredreliableifastudentcouldtakeitmorethanonceandgetclosetothesamescoreeach
time.Itwouldnotbeconsideredareliabletestifthesamestudentreceiveddramaticallydifferent
scoresontwodifferentdays.Precisionisimprovedbyreductioninrandomerror.Randomerroris
unpredictableandaffectsallmeasurements.Itisusedtodescribeanydeviationsfromthetruevalue
ofameasurementthatresultfromfluctuationsinthereadingsprovidedbythemeasurementtool.

12 Whatismeantbythevalidityofatest?
Validity(accuracy)isameasureofhowwellatestresultcorrespondstowhatitclaimstomeasure.In
otherwords,atestresultisconsideredvalidifitcloselymatchestheactualvalueofwhateverisbeing
measured.Notethatthisactualvalueisusuallyestimatedusingthegoldstandardtestmethod.
Validityisgenerallyreducedthroughsystematicerror.Forexample,anIQtestthatdependsinpart
onreadingcomprehensionisinvalidbecauseIQshouldnotdependonliteracy.Thisisasystematic
errorindesigningorconductingthetest.Systematicerrorreferstodeviationsfromthetruevalueof
ameasurementthatresultfrompoorinstrumentcalibrationorflawedobservationmethods.

13 Instatisticalanalysesofdifferencesbetweengroups,aPvalueisoftenincludedto
reflecthowsignificantthedifferenceis.WhatisthemeaningofthisPvalue?
Atitsmostbasiclevel,thePvaluereflectstheprobabilitythatthedifferenceobservedbetween
groupscouldoccurbychancealone.Forexample,ifthePvalueis0.05,thereisa5%chancethatthe
differenceobservedcouldhavebeenentirelyduetorandomchance,andnotduetowhatever
interventionwasusedintheexperiment.Nevertheless,mostofmodernscienceconsidersa
differencesignificantifthereisa<5%(P<.05)chancethatthedifferencecouldhaveoccurredby
randomchance.Obviously,thesmallerthePvalue,thesmallertheprobabilitythatanydifference
wasduetochance,andwiththat,themoreconvincingitisthattheinterventionevaluatedwas
responsibleforthedifference.

PvaluesarenotashighyieldforStep1assomeotherbiostatisticsrelatedtopicsbutwillbe
veryimportantforyoutounderstandwheninterpretingscientificliterature.Ontheotherhand,
typeIandtypeIIerrors(seequestion14)arecommonlytestedboardstopics.
Step1Secret

14 WhatarethedifferencesbetweentypeIandtypeIIerror?Howispowerrelatedto
typeIIerror?
TypeI()errorreferstofalsepositiveerror.Inotherwords,=theprobabilityofclaimingthata
truedifferenceexistsbetweentwomeanswheninrealitynoneexists.Thenullhypothesis(H0)
claimsthatthereisnodifferencebetweentwomeans.Ifaresearcherclaimsastatisticaldifference
betweentwogroupswhennoneexistsandfalselyrejectsthenullhypothesis,theresearcherhas
committedatypeIerror.
Bycontrast,typeII()errorisfalsenegativeerror.Itreferstotheprobabilityofstatingthatno
differenceexistsbetweenthemeansoftwovalueswheninfacttheretrulyisadifference.TypeII
errorisanacceptanceofthenullhypothesiswhenitshouldindeedberejected.
Let's,forinstance,saythatwewishtodeterminethefactorsthatinfluencethedifferenceinthemean
USMLEscoresobservedbetweentwogroupsofmedicalstudents.Wehypothesizethattimespent
studyingfortheUSMLEcouldbesignificantlydifferentbetweenthetwogroupsandmaythusexhibit
acausalrelationshipwiththescoresobtainedbythestudents.Wedeviseamethodtomeasurethis
variableanddonotfindthatthissignificantlydiffersbetweenthetwogroups.Ifadifferencetruly
exists,wehavemadeatypeIIerrorinthiscase.
Thetermstatisticalpowerisareflectionofastudy'sabilitytocorrectlydetectadifferencebetween
meanswhenonetrulyexists.Powerisequalto1.Thismakesgoodsense,right?Sinceisthe
probabilityofcommittingatypeIIerror(notdetectingadifferencethattrulyexists),powershould
inverselycorrelatewiththisvalue.

15 Whataresomedeterminantsthatcanbeusedtoevaluatetheexistenceofacausal
relationshipbetweentwovariables?
Consistency(e.g.,themorestudiesthatsupportthehypothesis,thebetter)
Strengthofcorrelation(e.g.,thehighertheRR,thebetter)
Biologicplausibility(e.g.,coloncancerisunlikelytobecausedbyultravioletlightexposure)
Temporality(causemustprecedediseaseoutcome)
Supportiveexperimentalstudies(e.g.,animalstudies)
Doseresponserelationship(e.g.,higherexposureleadstomoreseverephenotype)

16 Whatisthedifferencebetweenprevalenceandincidence?

16 Whatisthedifferencebetweenprevalenceandincidence?
Prevalenceisthepercentageofthepopulationthatcurrentlyhasthedisease.Forexample,25%of
Americansareobese,sotheprevalenceofobesityis25%.Incidencereferstohowmanypeople
developadiseasewithinagiventimeframe(usuallyannually).Thus,if300,000peoplearenewly
diagnosedwithdiabeteseachyear,theannualincidenceofdiabetesis300,000.

17 Howdotheincidenceanddurationofadiseaseaffectitsprevalence?
Thehighertheincidenceandthelongerthedurationofthedisease,thegreatertheprevalence.
Chronicdiseases(e.g.,arthritis)areunlikelytorapidlyresultindeath,sotheirprevalenceishigh.
Diseasesthathaveashortduration(e.g.,meningitis),eitherbecausetheyrapidlyresultinrapid
deathorbecausetheyresolvequickly,willhavealowprevalenceinthepopulation.Incidenceand
prevalenceareroughlyequalindiseasesthathaveashortduration.

MeasuresofSpread
18 Thefollowingsampledistributionpatternliststheagesof11patientsseenby
aphysicianonagivenday:
1, 2, 3, 3, 3, 4, 5, 5, 7, 8, 80

A.Whatarethemean,median,andmodefortheagesofthepatientsseenbythe
physicianonthisday?
Themeanissimplytheaverageofthesample,whichiscalculatedbyaddingtogetherallofthe
resultsandthendividingbythenumberofresults.Inthisexample,themeanwouldbe11(121/11=
11).Themedianisthenumberinthemiddleofthedatasetwhenorderedsequentially.Halfofyour
datashouldlieabovethemedianandhalfwillliebelow.Inthisexample,thenumber4hasequal
numbersofsubjectsoneitherside.Incasesinwhichthereisanevennumberofresults,themedian
iscalculatedbytakingtheaverageofthemiddletwonumbers.Themodeisthenumberpresentwith
thehighestfrequency.Inthiscase,themodeis3.
B.Isthemeanorthemedianmorerepresentativeofcentraltendency?
Themedianislessaffectedbyoutliers(e.g.,80)thanthemeanandisthereforeabetter
representationofcentraltendencythanthemean,particularlyforsmallsamplesizescontaining
multipleoutliers.However,keepinmindthatbecausethemeanencompassesallindividualsina
study,ithasmuchmorestatisticalpowerthaneitherthemedianorthemode.
C.Isthissampleskewedatallandifso,inwhichdirection?
Yes,thissamplepopulationispositivelyskewedtotherightbecauseoftheoutliervalue80.InFigure
251(f0010),graphArepresentsapositiveskewandgraphBanegativeskew.Positiveskewsoccur
whenthereareoutliersthathaveahighervaluethanthenumbersclosertothemean.Negativeskews
occurwhentheoutliershavealowervaluethenthenumbersclosetothemean.

YoumaybeaskedtointerpretdiagramslikethosedepictedinFigure251(f0010)onboards.The
easiestwaytorememberthedifferencebetweenpositivelyandnegativelyskeweddataisto
makeupyourowndatasettomatchthefiguresshowninFigure251A(f0010).Wewillreferto
midlineasthenumberalongthexaxisofthecurvethatfallsinthedirectcenterofthedata
range.Thecurveisasymmetrical,withmostofthedatafallingtotheleftofmidlineandthetail
ontheright.Ifthenumbersthatthisdatasetrepresentsrangefrom0to100(midlineis50),
thisdatasetcouldplausiblyconsistof20,20,and80.Themeanofthisdatasetis40andthe
medianis20.Thus,apositiveskewhasmean>median.Theoppositewillbetruefornegatively
skeweddata(Fig.251B(f0010)).Here,mostofthedatafallstotherightofmidline.Thisdata
setcouldconsistof20,80,and80.Here,meanis60andmedianis80.Negativelyskeweddata
willthusshowmedian>mean.Recallthatanormal(gaussian)statisticaldistributionwillhave
mean=median=mode.
Step1Secret

Figure251
Skewness.

19 Whatdoesthestandarddeviationofapopulationrepresent?

Thestandarddeviationisameasureofhowspreadoutatestpopulationis.Ifmostofthevaluesare
closetothemean,thestandarddeviationissmall.However,ifmanyofthevaluesarefarfromthe
mean,thestandarddeviationislarger.Youwillnotneedtocalculatethestandarddeviation,but
understandthatitiscalculatedbyaddingtogetherthedifferencesordeviationsofeachvaluefrom
themeanandthentakingtheaverageofthesedeviations.
Bydefinition,ifonetakesallthemembersofapopulationwithinonestandarddeviationofthemean
(bothaboveandbelow),thesememberswillconstitute68%ofthetotalpopulation.Iftwostandard
deviationsfromthemeanaretaken,thesememberswillconstitute95%ofthetotalpopulation.
Threestandarddeviationswillcontain99.7%ofthedataset.However,thisistrueonlyifthe
populationfallsintoanormalorbellcurve.Figure252(f0015)showsthedistributionofabell
curve.
YouareexpectedtoknowthebellcurvedistributionforStep1.Recallthatthedataare
distributedevenlytobothsides!Lotsofstudentsgetconfusedbythisconceptonboards.If,for
example,youareaskedtocalculatehowmuchofthedatafalloutoftherangeoftwostandard
deviationsinagaussiandistribution,theansweris5%because95%ofthedatawillfallwithin
therangeoftwostandarddeviations.Ifyouareaskedtocalculatehowmuchofthedatafall
abovetwostandarddeviations,theansweris2.5%(halfofthe5%willfallabovetwostandard
deviationsandhalfwillfallbelow).Asobviousasthismaysoundtoyou,paycloseattentionto
thequestionbeingasked!Itiseasytofallintothesetypesoftrapswhenyouareunderpressure
ontestday.
Step1Secret

Figure252
Distributionofabellcurve.

StudyDesigns
20 Whatismeantbythetermbiasandwhichstudydesignbesteliminates
bias?

Biasissystematicerrorthataffectsonestudygroupmorethantheother.Thisdiffersfromrandom
error,whichtypicallyaffectsbothgroupsequallyandshouldnotadverselyaffectthestudyifthere
areenoughparticipants.Randomizedclinicaltrialscontrolmosteffectivelyforbias,whereasacase
controlstudycontrolsleasteffectivelyforbias.Othertypesofstudydesigns(e.g.,cohort,cross
sectional)fallsomewherebetweenthesetwoextremesintheirabilitytoeliminatebias(Table255
(t0030)).

Case251
Inthemid1800s,Londonwasplaguedbyrecurrentoutbreaksofcholera,whichextracteda
highdeathtoll.Althoughthecauseoftheseoutbreakswasunknown,theprevailinghypothesis
wasthatcholerawasspreadbymiasmus,apoisonousodoremittedfromdecayingorganic
materialfoundinopengraves,sewers,andswamps.ThenowfamousepidemiologistJohn
Snowdisagreedwiththemiasmustheoryandpostulatedinsteadthatcholerawasspreadby
contaminatedwater.Hebelievedthisinpartbecausetheinitialsymptomsofcholerawere
intestinalinnatureandhereasonedthataninhaledpoisonousodorwouldnotmanifest
symptomsinthisway.
Tostudythishypothesis,herevieweddeathcertificatesandplottedaddressesforeachperson
inwhomthedeathcertificateimplieddeathfromcholerainfection.OnamapofLondon,he
thenmappedoutwherethesepeoplehadlivedpriortotheirdeathandcomparedtheirlocation
tothosewhodiedofcausesunrelatedtocholerainfection.Whathefoundwasthatthe
incidenceofcholerawasmuchhigherinLondonresidencesthatobtainedtheirwatersupply
fromthewatercompany.

Table255
StudyDesigns

Study
Design

Setup

Strengths

Limitations

Cohort

Acohortofsubjectsis

Relativelyeasyto Confoundingvariables:The

study

classifiedaccordingto

setupcompared

exposurebeingstudiedmay

exposureandthenfollowed withrandomized
todeterminetheeffectof
studies

correlatewiththedisease
outcomebutmaynotbethecause

exposureondisease

.Theremaybeother

outcome.

Allowsforthe
studyofexposures
thatareknownor
suspectedtobe
harmful
Establishesa

confoundingvariablesthatare
morecausative.

causalrelationship
betweenexposure
andoutcome
variables
Case
control

Subjectsareclassified
accordingtothepresence

study

orabsenceofdisease,and
correlationsaremade
betweenpastexposures
andthepresenceof
disease.

Easiesttosetup
Allowsforthe
studyofexposures
thatareknownor
suspectedtobe
harmful

Recallbias:Participantsmay
remembertheirexposures
differentlydependingonthe
presenceorabsenceofdisease
Interviewerbias:Interviewers
mayassumethatsomeonewitha
diseasehasbeenexposed
Doesnotallowforcalculationof
therelativeriskorthepercentages
ofthoseintheexposedversus
unexposedgroupswhogoonto
developdisease
Confoundingvariables

Randomized Participantsarerandomly

Provides

study

evidencefor

assignedtoexposure

Costlyandtimeconsuming

Cannotbeusedtostudy
groupsandfollowedforthe cause,andnotjust
exposuresthatareknownor
developmentofdisease.
correlation,
suspectedtobeharmful
becauseonlyone
exposureis
manipulatedata
time

1 Whatsortofstudydesignwasthis?
Thiswasacasecontrolstudybecauseparticipantswereselectedonthebasisofeitherhavingornot
havingthediseaseofinterest(cholera).

2 Howdoesaretrospectivecasecontrolstudydifferindesignfroma
retrospectivecohortstudy?
Thesestudiesdifferlargelywithrespecttohowsubjectsareclassifiedandselected.Inacasecontrol
study,subjectsareclassifiedaccordingtothepresenceorabsenceofdisease.Bycontrast,ina
retrospectivecohortstudy,subjectsareclassifiedbasedonthepresenceorabsenceofexposure.Only
thenisdiseasestatusdetermined.

3 Whatarethestrengthsofacasecontrolstudy?
Casecontrolstudiesarerelativelyeasytosetupbecausetheresearchersimplyhastolocatepeople
whohavebeenaffectedbyadisease.Anotherstrengthisthatcasecontrolstudiescanbeusedtolook
attheeffectofexposuresthatareknownorsuspectedtobeharmful.Forexample,arandomized
studycouldnotlookattheeffectofchildabuseonthechancesthatsomeonewillabusehisorher
ownchildren,becauseitwouldbeunethicalandillegaltorandomizeparticipantstobeabused.
However,withacasecontrolstudy,aresearchercouldlookatchildabusersaswellasnonabusers
andcomparetheincidenceofabuseintheirchildhoods.

4 Whatarethelimitationsofacasecontrolstudy?
Althoughcasecontrolstudiesareeasytodesign,theyhaveanumberofflaws.Asmentioned,case
controlstudiessimplyuncoveranassociationbetweentwovariablesbutdonotestablishacausal
relationship.Forexample,ifyoudidacasecontrolstudyandfoundthatthosewithlungcancerhave
higherratesofalcoholism,youmightconcludethatalcoholismleadstolungcancer.However,
alcoholicsmightbemorelikelythannonalcoholicstosmokecigarettes,andsmokingcouldbethe
actualcauseoftheirlungcancer.
Anotherflawincasecontrolstudiesisrecallbias,whichisthetendencyofthosewithadiseaseto
exaggeratetheirexposuresandthosewithoutadiseasetominimizetheirexposures.Forexample,a
womanwithachildwhohasbeenbornwithabirthdefectmightrecallmanymorechestxray
studiesduringherpregnancythanmightawomanwithhealthychildren.Interviewerbiasisalso
anissue.Thisoccurswhenaninterviewerassumesapersonwiththediseasehasbeenexposedtorisk
factorswhereasthehealthypersonhasnot.Forexample,theinterviewermightaskapersonwith
lungcancerthequestionHowmanypacksperdaydidyousmoke?whereasahealthypersonmight
beaskedthequestionYouneversmoked,didyou?Eventhoughbothquestionsaskforsimilar
information,thesenseofjudgmentimposedbythelattermaydetertheintervieweefromproviding
accurateresponses(Table256(t0035)).
Biasoccurswhenoneoutcomeisfavoredoveranotherduetosystematicerror.Thevarious
typesofstatisticalbiasarecommonlytestedonStep1.Youshouldknowthetypesofbias
definedinTable256(t0035).
Step1Secret
Table256
StatisticalBias

BiasType

Definition

Example

Selection

Generaltermfor

SeeBerkson'sbias.

bias

nonrandomassignmentof
participantstovarious

groupswithinastudy
Berkson's

Selectionbiasperformed

Instudyingdurationofadiseaseinhospitalizedpatients,

bias

onhospitalizedpatients

thesubjectsmayhavemoreseveresymptomsorbetter
accesstocare,whichcanalterdiseasecourse.

Recallbias

Datacanbealteredbya

Administeringasurveyimmediatelyafteraspecific

participant'smemoryof

incidentor10yearslatermayaltertheaccuracyof

thetestedvariable

responsesacquiredfromparticipantsbasedon
recollectionoftheevent.

Confounding Theeffectofan
bias

Studyinggeneticlinkageofcancerincidencebasedon

independentvariableona familyhistorywithoutcontrollingforage,race,gender,or
dependentvariableis
othervariablesthatmayindependentlyaltercancerrisk
distortedbyathird,

mayaffectfindings.

unmeasuredvariable
Hawthorne

Patientschangetheir

Inusingdietaryrecallstoexaminenormalfruitand

effect

normalbehavioronce

vegetableintakeinapopulation,studyparticipantsmay

theyknowtheyarebeing
studied

changetheirnormalintakeofthesefoodsiftheyknow
thisisbeingmeasured.

Leadtime

Earlydetectionofa

Diagnosisofbreastcancerwithanoveltechnology

bias

diseaseismistakenfor

increasespatientsurvivalby6monthsitispossiblethat

increasedsurvival

thecancerissimplydiagnosedatanearlierstagewith
thenewtechnologyandthatdiseasecourseis
completelyunaltered.

Pygmalion
effect

Studyconclusionsare
influencedbythe

Instudyingtheeffectivenessofusingnewsurgical
technologybasedonsurgeonreporteddata,ifthe

researcher'sownbeliefin surgeonbelievesintheefficacyofthetechnology,his
aspecifichypothesis

reportmaybebiasedbyhisownbelief.

Observer

Observerknows

Researcherreportsareducedincidenceofdepressionin

bias

exposurestatus(not
doubleblinded)

agroupthatheknowsiscurrentlyplacedonanew
SSRI.

Procedure
bias

Subjectsindifferentstudy Inaweightlossstudyinwhichonegroupisplacedona
groupsarenottreated
dietpillandthesecondisplacedonaplacebo,the
equally

formergroupistrainedmoreintenselythanthelatter
differencesinexerciseroutinesmayaccountforsomeof
theobservedweightloss.

Sampling

Selectingparticipants

Instudyingfactorsthatcontributetoheartattackriskin

bias

whodonotrepresentthe
makeupoftheoverall

anuppermiddleclasssuburbantown,subjectsmay
havefinancialandstressrelatedfactorsthatdonot

population,sothat

matchthosepresentinthegeneralpopulationbut

findingswillnotbe

neverthelessinfluencetheirriskforheartattack.

generalizable
Latelook

Acquisitionofdataatan

Inasurveyofpatientstostudytheimpactoflupusina

bias

incorrectorinappropriate
time

community,patientswhohavealreadydiedfromlupus
areunabletorespondtothesurvey.

5 Whatratiocanbeusedtocompareeventratesinacasecontrolstudy?
OR(knowthis!).CasecontrolstudiescannotbeusedtocalculatetheRRofanexposureonthe
diseaseoutcomeortodefinethepercentageofpeoplewithacertainexposurethatwillgoonto
developadisease.ThisisbecausethecalculationofRRrequirestheincidencerate,whichcannotbe
calculatedfromacasecontrolstudybecausethereisnofollowuptime.ORsareusedforcase
controlstudies,andRRisusedforcohortstudies

6 Howistheoddsratiocalculated?
TheORiscalculatedbydividingtheoddsofdevelopingthediseaseforthoseintheexposedgroup
versustheoddsofdevelopingthediseaseforthoseintheunexposedgroup.Forexample,ifJohn
Snowchartedthewatersupplyof100peoplewhodiedofcholeraand100peoplewhodiedofother
causesandfoundthat80ofthe100peoplewhodiedofcholeralivedinhomessuppliedbythewater
company,butonly10ofthe100peoplewhodiedofothercauseslivedinhomessuppliedbythe
watercompany,thentheORwouldbecalculatedasfollows:
80/20

OR =

10/90

8090
2010

7200
200

= 36

AnOR>1impliesthatthediseaseismorelikelytooccurintheexposedgroup,socholeraoccurred
moreofteninthosewhoreceivedtheirwatersupplyfromthewatercompany.Thisdoesnotprove
thatthewaterwasthesourceoftheinfection,becausetheremayhavebeenconfoundingvariables
thatweretheactualcause.Forexample,itmayhavebeenthatthosewholivedinhomessuppliedby
thewatercompanywerepoorerthanthosewhohadtheirownwells,andthereforetheyhadjobs
wheretheywereexposedtolesssanitaryconditions.Theseunsanitaryworkingconditions,rather
thanthewaterintheirhomes,mayhavethenexposedthemtocholera.

Inacasecontrolstudy,subjectsareclassifiedaccordingtothepresenceorabsenceof
disease.
Strengthsofcasecontrolstudiesarethattheyareeasytosetupandcanbeusedto
studyexposuresthatareknownorsuspectedtobeharmful.
Limitationsofcasecontrolstudiesarethattheyleaveroomforrecallbias,interviewer
bias,andconfoundingvariables.

Casecontrolstudiescannotbeusedtodefinerelativerisk(RR)butcanbeusedto
calculateanoddsratio(OR).
SummaryBox:CaseControlStudies

Case252
Bloodpressureismonitoredregularlyinagroupof500adultmen.Themeanbloodpressure
forthegroupisreportedas13010mmHg,andthecoefficientofvariation(CV)isnotedtobe
small.Thebloodpressuremeasurementswithinthegrouparedescribedashavinganormal
distribution.

1 Whatdoestheexpression13010mmHgmeanwithrespecttothe
distributionofbloodpressureinthissample?
The130mmHgreferstothemeanbloodpressureofthesample,whereasthe10mmHgrefersto
thestandarddeviation.Astandarddeviationof10mmHgimpliesthat68%ofthepopulationhad
bloodpressureswithin10mmHgofthemeanof130(i.e.,between120mmHgand140mmHg).For
95%ofthepopulationofthisstudy,thebloodpressureiswithintwostandarddeviationsofthe
mean,orwithin110mmHgand150mmHg.Thetop2.5%ofthepopulationhasabloodpressure
abovetwostandarddeviationsofthemean(>150mmHg)andthebottom2.5%ofthepopulation
hasabloodpressurebelowtwostandarddeviationsofthemean(below110mmHg).

2 Whatdoesitmeanwhenthebloodpressureinthispopulationissaidtobe
normallydistributed?
Tobenormallydistributedmeansthatifaplotofthemagnitudeofthevariablebeinganalyzed(in
thiscase,bloodpressure)againstthefrequencyofeachmagnitudeismade,thecurvetakesona
bellshapedformthatiswelldescribedbyaspecificmathematicalequation,whichcanbeusedto
accuratelycalculatethestandarddeviation.Anormalcurve(seeFig.252(f0015))representsa
populationinwhichamajorityofpeoplehavemeasurementsclosetothemean,andthefartherfrom
themean,thefewerpeoplehavethatmeasurement.Curvesareoftenassumedtobenormalforthe
sakeofeasycalculations,butsomecurvesdifferlargelyfromanormalcurve.Forexample,ifyou
askedagroupofpeoplewhattemperaturetheyliketheircoffee,mostwouldsayeitherveryhotor
verycold.Almostnoonewouldsaythattheylikelukewarmcoffee.Thisstudywouldproducea
bimodaldistribution(Fig.253(f0020)),withtwohumps,whichwouldnotfitthenormalcurve.

Figure253
Bimodaldistribution.

3 Whatdoesasmallcoefficientofvariationforthesampleinthebloodpressure
studyimply?
TheCVisusedtoexpressthestandarddeviationasapercentageofthesamplemean.Thiscanbe
veryinformative.Standarddeviationtypicallydecreaseswithincreasingsamplesize.Forexample,
thisstudyexpressesbloodpressureas13010mmHg.Inthisstudy,theCV=10/130100=7.6%.
SmallersamplesizesareassociatedwithlargerCVs.Forexample,ifbloodpressuresweremeasured
in10menasopposedto500,onemightgetavaluesuchas13060mmHg.Inthiscase,theCV
wouldequal60/130100=46%.AgoodstudywillusuallyhaveaCV10%.

4 Quickreview:Whatpercentageofthemeninthisstudyhadabloodpressure
inthefollowingranges?
A.120and140mmHg
68%
B.110and150mmHg
95%
C.Above150mmHg
2.5%
D.Below110mmHg
2.5%

Standarddeviationcanbeusedtoestimatethepercentageofapopulationthatfallsinto
acertainrange,aslongasthepopulationfitsanormaldistribution.
Somepopulationsdonotfitthenormaldistribution.
Coefficientofvariationusesthestandarddeviationandthesamplemeantocreatean
evenmoreinformativewaytolookatthedistributionofapopulation.
SummaryBox:NormalDistribution,StandardDeviation,andCoefficientofVariation

case253

A27yearoldmancomplainsoffatigueandgeneralmalaisebeginningseveralmonthsearlier.
Althoughhispastmedicalhistoryisunremarkable,hismorerecenthistoryissignificantforthe
useofintravenousdrugsandforunprotectedsexwithprostitutes.Withthepatient'sconsent,
youscreenhimforhumanimmunodeficiencyvirus(HIV)infectionusingatestwithareported
sensitivityof95%andspecificityof75%.

1 Whydoesitmakesensetouseascreeningtestwithahighsensitivity,evenat
thecostofspecificity,forthispatient?
Screeningtestsingeneral,andparticularlyforlifethreateningdiseasessuchasHIVinfection,should
haveahighsensitivitysothattheywilllikelydetectthediseaseifitispresent.Becausescreening
testsingeneralmustbeinexpensive,thishighsensitivitymaycomeatthecostofasuboptimal
specificity.However,becauseitismuchmoreimportanttonotmissalifethreateningdisease(i.e.,
fewfalsenegatives)thanitistoinconvenience(oreventraumatize!)someonewithafalsepositive
result,thisisconsideredacceptable.
Notallscreeningtestshavehighsensitivity.Whenusedasasingledatapoint,mammogramsand
Papanicolaou(Pap)smears,forexample,havelowsensitivity.However,whenusedonaregularbasis
(e.g.,annually)theybecomemuchmoreeffectivescreeningtoolsbecauseofahighcumulative
sensitivity.

2 Ifthispatienttestspositive,isitreasonabletotellhimthatyouare95%
confidentthatheisinfectedwithhumanimmunodeficiencyvirus?
No.Sensitivityandspecificityvaluessimplyrepresenthowgoodatestisatrulinginorrulingouta
disease,andperhapswhetherthetestisidealforscreeninglargepopulationsforagivendisease.
Althoughapositivetestresultwillundoubtedlybeconcerningtotheclinicianandthepatient,with
theinformationprovidedthereisnowayofdeterminingifthisisatruepositiveorafalsepositive
result.WhatisneededtocalculatethePPVforthistest?Thatdependsonadditionalinformation
(prevalenceofthediseaseinthespecificpopulationwithinwhichthepatientfalls),asdiscussedin
thenextcase.

3 Whatifthetestcomesbacknegative?Canyoutellthispatientthatyouare
75%confidentthathedoesnothavehumanimmunodeficiencyvirus
infection?
Again,suchastatementcannotbemadeunlessyouknowtheNPVofthetest,whichwasnot
provided.

4 Nowlet'sassumethat90yearoldgrandmaandouryoungdrugabusing
modelcitizeninthisvignettebothtestpositiveforhumanimmunodeficiency
virususingthistest.Aretheybothequallylikelytohavethedisease?

No,andthisquestionaddressestheimportantconceptofutilizingscreeningtestsappropriately.The
goalofcliniciansistoselectivelyscreenonlythoseindividualsathigherriskfordevelopingagiven
disease.ThisisbecausethePPVofatestdependsontheprevalenceofthediseaseinthegiven
populationbeingtestedaswellasonthespecificityandsensitivityofthetest.TheprevalenceofHIV
in90yearoldwomenismuchlowerthaninyoungintravenousdrugusers.Therefore,ifthe
grandmothertestspositiveforHIV,sheismuchmorelikelytohaveafalsepositivethanourother
patientis,becauseshehadasmallerchanceofhavingHIVbeforeweranthetest.
Considerthehavocthatwouldbecreatedifphysiciansscreenedallwomenstartingattheageof20
forbreastcancerbyperformingannualmammograms.Giventhattheprevalenceofbreastcancerin
youngwomanislow,suchtestingwouldyieldnumerousfalsepositives,necessitatingunnecessary
referralsandexpensiveworkupsbyspecialists,nottomentionalotofunneededanxiety!Usingthis
sametesttoscreenonlywomenover40makesabitmoresense,asthenumberoftruepositiveswill
increaseandthenumberoffalsepositiveswilldecreaseowingtotheincreasedprevalenceofbreast
cancerwithaging.

Ingeneral,screeningtestsshouldhavehighsensitivitysoasnottomissanyonewiththe
disease.
Thesensitivityorspecificityofatestcannotbeusedtodeterminethelikelihoodthata
personhasadiseaseifhetestspositiveornegativeforthatdisease.Inordertodetermine
thelikelihoodofthediseaseaftertesting,thepositivepredictivevalueornegative
predictivevalueofthetestmustbecalculated.
Thepositiveandnegativepredictivevaluesofatestresultdependontheprevalenceof
diseaseinthepopulationbeingtested,soapositivetestresultinapersonbelongingtoa
lowriskpopulationmaynotbeasgreatapredictorofdiseaseasitwouldbeforaperson
withinamorehighriskpopulation.
SummaryBox:LikelihoodRatios,PredictiveValues,andPrinciplesofScreening

Case254
Inatownof1000individuals,theprevalenceofcoronaryarterydiseaseacrossallagegroupsis
20%(asdeterminedbyangiography,thegoldstandard).Youhavecreatedawonderfully
inexpensivescreeningtestthatyoubelieveisbothhighlysensitiveandspecificfordetecting
coronaryarterydisease.

1 Giventhedatapresentedinthe22tableinTable257(t0040),whatisthe
sensitivityofthisnewtest?

Sensitivitycanbecalculatedbydividingthenumberoftruepositivesbythetotalnumberofpersons
testedwiththedisease,ora/(a+c)inthe22table.Therewere180truepositivesofthe200
patientstestedwhohaddisease,yieldingasensitivityof180/200,or90%.Thismeansthatthistest
detects(senses)thediseasein90%ofpeoplewhohavethedisease.
Table257
TestingforCoronaryArteryDisease

CoronaryArteryDisease
+
NewTest + 180(a)
20(c)

80(b)
720(d)

2 Whatisthespecificityofthisnewtest?
Specificitycanbecalculatedbydividingthenumberoftruenegativesbythetotalnumberofpeople
testedwhodonothavethedisease(truenegativesplusfalsepositives),ord/(b+d)inthe22
table.Therewere720truenegativesand80falsepositives,sothespecificityofthistestis720/800,
or90%.

3 Whatinformationcanbeobtainedfromcalculatingthepositivelikelihood
ratio?
AsdescribedintheBasicConceptssection,thePLRreflectshowmuchapositivetestresultincreases
theprobabilityofthepresenceofdisease(i.e.,indicatesposttestprobabilityofdisease).Thehigher
theratio,themorelikelyitisthatdiseaseispresent.
Thisratioiscalculatedasthesensitivitydividedby1specificity
PLR = sensitivity/1 specificity

Forthisexample,giventhatsensitivityandspecificityareboth90%,thePLRcanbecalculatedas
PLR

=. 90/1. 90
=. 9/. 1
= 9

Soapositivetestmeansthatthispatient'sposttestprobabilityis9timesgreaterforhavingthe
diseasethanwashispretestprobability.

4 Howisthepositivelikelihoodratiousedtocalculatethepositivepredictive
value?

Thiscalculationcanbedoneifthediseaseprevalenceisknown.First,thediseaseprevalenceneedsto
beconvertedtoaratio(e.g.,20%to1:4).ThenwemultiplythefirstpartoftheratiobythePLR(9.0
1:4=9:4).Thentheratioisconvertedbacktoapercentage9/(9+4)=.69,or69%.Soifinthis
casetheprevalenceofcoronaryarterydiseasewas20%andthePLRwas9,thePPVofthetestwould
be69%,indicatingthatsomeonetestingpositivewouldhavea69%chanceofactuallyhaving
coronaryarterydisease.

5 Ifthepositivelikelihoodratioisnotknown,whatisanotherwaytocalculate
thepositivepredictivevalue?
Itcanbecalculated(byusingthe22table)asthenumberoftruepositivesdividedbythetotal
numberofpositives(trueandfalse).Thiscalculationrequiresknowledgeofsensitivityandspecificity
ofthetest,aswellastheprevalenceofthedisease,togeneratethenumberoftruepositivesandfalse
positives.
Fortheprecedingexample,giventhefactthatthediseaseprevalenceis20%andthesensitivityand
specificityofthescreeningtestareboth90%,thePPVofapositivetestresultwouldequal69%(see
followingcalculation).Soyouwouldtellyourpatientthatheisonly69%likelytohavecoronary
arterydiseasebasedonhispositivetestresult.
Positive predictive value

= TP/(TP + FP)
=

180/180 + 80
= 69%

6 Howisthenegativepredictivevaluecalculated?
TheNPVistheprobabilitythatthediseaseisabsentifthetestisnegative.Itiscalculatedastrue
negativesdividedbybothtruenegativesandfalsenegatives:TN/(TN+FN).

7 Usingthesameprecedingexampleforthecalculationofthepositivepredictive
value,calculatethenegativepredictivevalue
Negative predictive value

= TN/(TN + FN)
=

720/(720 + 20)
= 0.97or97%

So,97%ofthepeopleinthissamplewhohadanegativetestresultwouldnothavethedisease.

Sensitivityandspecificitycanbecalculatedfroma22table.
Positivelikelihoodratioreflectshowmuchapositivetestresultincreasestheriskof
diseaseandcanbecalculatedbedividingsensitivityby1specificity.
Positivepredictivevaluecanbecalculatedfromthepositivelikelihoodratioorfroma2
2table.

Negativepredictivevaluecanalsobecalculatedfroma22table.
SummaryBox:Sensitivity,Specificity,LikelihoodRatios,andPredictiveValues

Case255
Inthe1940s,astudywasperformedonemployeesatanuclearpowerplanttodetermineifan
associationexistsbetweenradiationexposureandcancerrates.Inthisstudy500employees
withhighlevelradiationexposureand500employeeswithverylimitedexposurewere
followedfor10years,andtheincidenceratesforcancerwerecomparedinthetwogroups
throughoutthistime.Theresultsaredepictedinthe22tableshowninTable258(t0045).
Table258
RadiationExposureandCancer

Cancer
+

Exposure + 50(a) 450(b)


5(c)

495(d)

1 Whattypeofstudydesignisthis?
Thisisa(prospective)cohortstudybecauseindividualsareclassifiedonthebasisofexposure,not
disease(aswithacasecontrolstudy).Furthermore,thiswasanongoingstudyinwhichthe
complicationsassociatedwithradiationexposurewereanalyzedastheyoccurred.

2 Whatisthedifferencebetweenaprospectivecohortstudyandaretrospective
cohortstudy?
Inaprospectivecohortstudy,individualswithagivenexposurearefollowedovertimetoseeifthere
isanincreasedordecreasedfrequencyofdiseasedevelopment.Inaretrospectivecohortstudy,a
groupofindividualswhowereexposedsometimeinthepastareevaluatedtoseeiftheyhavea
higherfrequencyofthedisease.Inbothcases,thestudypopulationisgroupedaccordingto
exposure.

3 Whatisthemajorlimitationofcohortstudies?
Althoughthegroupsmaybedistinctfromeachotheraccordingtothefactorbeingstudied,thereare
manyotherfactorsthatmaybedifferentbetweenthegroupsandwhichcouldbeinfluencing
outcome(confoundingvariables).Forexample,acohortstudyfoundthatpeoplewhoeatmore
carotenehadalowerincidenceoflungcancer.However,thisdidnottakeintoaccountthatpeople

whoatemorecarotenemayeatsubstantiallymorefruitsandvegetablesingeneral,whichitself
maybeprotectivefromcancer.Infact,whenarandomizedtrialwasdone,carotene
supplementationactuallyincreasedtheriskoflungcancer.

4 OnthebasisofdatapresentedinTable258(t0045),whatistherelativerisk
forcancerintheexposedgroup?
TheRRisdeterminedbycomparingincidenceratesinexposedindividuals(IE)toincidenceratesin
nonexposedindividuals(INE),asshownbelow.Thus,theRRfortheemployeesexposedtoradiation
is10timesgreaterthanforthenonexposedemployees.
RR =

IE
I NE

a/a+b

c/c+d

50/500

5/500

= 10

5 Whatismeantbyattributableriskandattributableriskpercent?Calculate
bothfortheprecedingexample
Attributablerisk(AR),alsoreferredtoastheabsoluterisk,istheincidenceofdiseaseintheexposed
groupcausedsolelybyexposure.Itcanbecalculatedbythedifferenceinincidenceratesbetween
exposedandnonexposedgroups,asshowninthefollowingequationfortheprecedingexample.
AR

= IE INE
=

50/500 5/500
=

45/500
= 0.09

ThisARof0.09impliesthat9%ofpeopleexposedtoradiationdevelopedcancerasaresultofthat
exposure(i.e.,whichcouldbeattributedtothatexposure).
Theattributableriskpercentisameasureofthepercentageofpeoplewhowereexposedand
developedthedisease,andinwhomthedevelopmentofdiseasewasduetotheexposure.Itcanbe
calculatedbydividingtheARbytheincidenceofdiseaseintheexposedgroup:
AR%

= AR/IE 100
= 0.09/0.10 100
= 90%

ThisARpercentof90%impliesthat90%ofpeoplewhowereexposedtoradiationanddeveloped
cancerdevelopedtheircancerasaresultoftheradiation.
AbsoluteriskreductionpercentiscalculatedinthesamewayasforARbutisusedinreferenceto
exposuresthatreduceone'schancesofacquiringthediseaseoutcome.Forexample,ifyoufounda
reductionincholesterollevelsinagrouptakingstatinscomparedwithagroupnotcurrentlyon
cholesterolloweringmedication,youwouldusethetermabsoluteriskreductiontodescribethe
differenceinriskofdevelopinghighcholesterolbetweenexposedandunexposedgroups.

Numberneededtotreatisrelatedtoabsoluteriskreduction.Numberneededtotreat=1/absolute
riskreduction.Inotherwords,howmanyparticipantsmustgainexposurestatustopreventonefrom
developingthediseaseoutcome?Numberneededtoharmiscalculatedusingthesameconcept,
exceptthatitequals1/AR.
Youshouldknowhowtocalculateoddsratio,relativerisk,absoluteriskreduction,attributable
risk,andnumberneededtotreat/harm.Thesearehelpfulformulastoaddtoyourwhiteboard
beforethestartofyourexam!
Step1Secret

6 Whatexperimentaldesignovercomestheshortcomingsofthecohortstudy?
Thebestexperimentaldesign,whichistheoneleastsusceptibletoconfoundingandbias,isthe
randomizedcontrolledtrial.Inthistrialdesign,individualsarerandomlyallocatedtotreatmentor
controlgroups,therebyreducingconsiderablytheeffectsofanyconfoundingfactors.
Thebestkindofrandomizedstudyisadoubleblindplacebocontrolledtrial.Doubleblindrefers
tothefactthatneithertheinvestigatornorthestudysubjectsknowwhoisreceivingthetreatment.
Placebocontrolledmeansthatthosewhodonotreceivethetreatmentbeingtestedreceivea
placeboinstead,whichshouldbesimilarenoughtothetreatmentthattheparticipantscannottell
whethertheyarereceivingtheplaceboorthetreatment.

Inacohortstudy,participantsareclassifiedaccordingtoexposure.
Themajorlimitationofcohortstudiesistheexistenceofconfoundingvariables.
Cohortstudiesallowforcalculationofrelativerisk.
Absoluteorattributableriskistheriskthatcanbeattributedsolelytoexposure.
Thebestkindofexperimentaldesignisarandomizeddoubleblindplacebocontrolled
trial.
SummaryBox:CohortStudies

Copyright2015Elsevier,Inc.Allrightsreserved.

BOOKCHAPTER

BleedingDisorders
AllysonM.Reid,ThomasA.BrownMDandSonaliJ.Shah
USMLEStep1Secrets,Chapter13,404417

BleedingdisordersontheUSMLEStep1arerelativelystraightforwardifyouunderstandthe
pathophysiologybehindthediseasesandcorrelatethemwithclinicalfindings.Thebestwayto
studyforthissectionistoreviewthedisordersmentionedinFirstAidandbesurethatyoucan
reasonthroughallthelistedclinicalfindingsandlaboratoryparameters.Youshouldthen
attempttogothroughthesecasesandcreateyourowndifferentialdiagnosisbeforereviewing
theanswers.Thiswillbegreatpracticeforthetypesofquestionsyouwillseeandthethought
processesthatwillbeexpectedofyouonboards.
Inaddition,pharmacologyisespeciallyimportantinthissection.Manyofthedrugsthatyou
areaccountableforonboardsandtheirassociatedmechanismsarelistedforyouinthis
chapter.Besuretopayattentiontopharmacologicantidoteswhenevertheyarementioned.
Insider'sGuidetoBleedingDisordersfortheUSMLEStep1

Basicconcepts
1 Differentiatebetweentheprocessesofprimaryandsecondaryhemostasis
Primaryhemostasisinvolvestheformationofatemporaryplateletplugfollowingbindingof
plateletstoexposedcollagen,plateletsecretionofprocoagulantsubstances(e.g.,adenosine
diphosphate[ADP],Ca2+),andplateletaggregation.Thisplateletplugisreferredtoastemporary
becauseitisreadilyreversibleatthisstage.Secondaryhemostasisinvolvesthecovalentcrosslinking
offibrinbetweenplatelets,resultingintheformationofanirreversibleplateletplug.Thisprocessis
dependentonactivationofthecoagulationcascade(Table131(t0010)).
Table131
PrimaryandSecondaryHemostasis

Feature

PrimaryHemostasis

SecondaryHemostasis

Definition

Temporaryplateletplug

Permanentplateletplug

Exampledisease(s)

VonWillebranddisease

HemophiliaAandB

Laboratorytesting

Bleedingtime,plateletaggregationstudies

Prothrombintime(PT),

Laboratorytesting

Bleedingtime,plateletaggregationstudies

Prothrombintime(PT),
partialthromboplastintime
(PTT)

Clinical
manifestationsof
disturbedmechanism

Relativelymild(e.g.,excessivebleedingafter
dentalworkorsurgery),mucocutaneous
petechiae

Relativelysevere(e.g.,
hemarthrosis)

Note:Disordersaffectingplateletfunctionwillimpairprimaryhemostasis,whereasdisorders
affectingthecoagulationcascadewillimpairsecondaryhemostasis.

2 Whatmoleculeisresponsibleforthebindingofplateletstocollagen?
VonWillebrandfactor(vWF),whichissynthesizedbyvascularendotheliumandattachestoboth
collagenandplateletreceptors(specificallyglycoproteinIb[GPIb]),isresponsibleforthebindingof
plateletstocollagen.

3 Whatconstitutestheextrinsic,intrinsic,andcommonpathwaysinthe
coagulationcascadethatformsthefibrinclot(secondaryhemostasis)?
Theextrinsicandintrinsicpathwaysareseparatebiochemicalpathwaysthatactivatethecoagulation
cascade.Boththeextrinsicandintrinsicpathwaysleadintothecommonpathwaytocauseformation
ofthefibrinclot.
Theextrinsicpathwayisreferredtoasextrinsicbecausethefactorthatactivatesit(tissuefactor)is
normallyextrinsictothevascularspaceandisexposedtothevascularspaceonlywhenthereis
damagetothevascularendotheliumthatallowsextracellularfluidandcellstoenter.Theextrinsic
pathwayincludestissuefactor(factorIII)andfactorVII.
Theintrinsicpathwayhasallofitsconstituentsinthebloodhence,itisintrinsic.Itisactivatedby
exposuretonegativelychargedforeignsubstances.TheintrinsicpathwayincludesfactorsXII,XI,IX,
andVIII.
Asmentioned,thecommonpathwayisactivatedbyeithertheextrinsicpathwayorintrinsicpathway,
anditeventuatesintheformationofafibrinclot.ThispathwayincludesfactorsX,V,II,andI(Fig.
131(f0010)).

Figure131
Simplifiedcoagulationcascade.HMWK,highmolecularweightkininogenPK,prekallikreinPL,plasminogenPT,
prothrombintimePTT,partialthromboplastintimeTF,tissuefactor.
(FromFerriF:Ferri'sBestTest:APracticalGuidetoClinicalLaboratoryMedicineandDiagnosticImaging.St.
Louis,Mosby,2004.)

4 Whatinformationcanbeprovidedbymeasuringtheprothrombintimeand
activatedpartialthromboplastintime?
Theprothrombintime(PT)reflectsthetimerequiredforformationofafibrinclotinaparticular
assaysystem(essentially,tissuefactorisaddedtoasampleofthepatient'splasma).Becausethe
extrinsicpathwaymustactivatethecommonpathwaytoformafibrinclot,thePTreallyreflectsthe
timeittakesfortheextrinsicpathwayandthecommonpathwaytoformtheclot.ThePTcanbe
prolongedbyclottingfactordeficienciesineitherpathway.
Theactivatedpartialthromboplastintime(aPTT)alsoreflectsthetimerequiredforformationofa
fibrinclotinaparticularassaysystem(inthiscaseanactivatoroftheintrinsicpathwayisaddedtoa
sampleofthepatient'splasma).Inotherwords,thisvaluemeasuresthetimeittakestheintrinsic
pathwayandcommonpathwaytoformaclot.TheaPTTcanbeprolongedbyclottingfactor
deficienciesineithertheintrinsicorcommonpathway.

5 Whatisthebleedingtime?Whatisitsclinicalsignificance?
Thebleedingtimereflectsthetimerequiredtoformtheplateletplug(primaryhemostasis)andso
reflectsplateletfunctionratherthanfunctioningofthecoagulationcascade.Morecommonly,
however,invitroplateletfunctiontestsareperformedinplaceofmeasuringthebleedingtime.

6 Whatisthemechanismofactionofthefollowingdrugs?Howdotheyaffect
thetimesjustdiscussed?
A.Aspirin

Aspirininhibitsplateletfunctionbyirreversiblyinhibitingtheenzymecyclooxygenase(COX),which
normallyfunctionstosynthesizethromboxaneA2.ThromboxaneA2normallyfunctionsto
stimulateplateletaggregationandconstrictionofbloodvessels,bothofwhichacttolimitbleeding
followingvesseltrauma.Consequently,inhibitionofthromboxaneA2byaspirinresultsina
prolongedbleedingtime.
Note:BecauseaspirinisanirreversibleinhibitorofplateletCOX,itseffectonplateletfunctionlasts
aslongastheaffectedplateletsremaininthecirculation,~7days.
B.Heparin
HeparinstimulatestheactivityofantithrombinIII(ATIII),whichisapotentinhibitorofthrombin
(factorII),aswellasseveralotherfactorsintheintrinsicpathway.Becauseofthispreferential
inhibitionoftheintrinsicpathway,heparinactsrapidlytoprolongtheaPTTbutwilldosoonlyat
higherdoses.Ithasnoeffectonthebleedingtimebecauseitdoesnotaffectplateletfunction(except
incasesofheparininducedthrombocytopenia[HIT]).
Note:Excessivebleedingfromheparintoxicitycanberapidlyreversedbyadministeringprotamine
sulfate.
C.Warfarin
WarfarininhibitstheproductionofvitaminKdependentclottingfactorsintheliver(factorsII,VII,
IX,andX)byantagonizingtheactionofvitaminK.ItpreferentiallyprolongsthePTmorethanthe
aPTT.Becausewarfarininhibitsthesynthesisoftheseclottingfactors,andnottheiractivityinthe
blood,thereisadelaybetweenitsadministrationanditsonsetofaction(typicallyafewdays).For
thisreason,patientswhorequirelongtermanticoagulationareusuallystartedonheparinand
warfarinsimultaneously,andoncethepatientissufficientlywellanticoagulatedwithwarfarin,the
heparincanbediscontinuedandtheprothrombintimemaintainedwithinatherapeuticrange.
Note:TheeffectsofwarfarincanbereversedbyadministeringvitaminK,andthistakesaboutaday
ortwo.Foremergentsurgicalprocedures,freshfrozenplasmacanbegiventoimmediatelyreplace
thedeficientclottingfactors.WarfarinalsoinhibitstheproductionofproteinCandproteinS,both
vitaminKdependentproteinssecretedbytheliverthatexertanticoagulanteffects.Consequently,
inhibitionofproteinCandproteinSsynthesisbywarfarincaninitiallyresultinahypercoagulable
state(inwhichthepatientispronetoclotting).Forthisreason,loadingdosesofwarfarinarenot
recommended.Instead,warfarinshouldbeadministeredwithheparinduringthefirstweekof
therapy.

7 Whatisthemechanismofactionoftissueplasminogenactivator?
Asitsnamesuggests,tissueplasminogenactivator(tPA)isanenzymethatactivatestheplasma
enzymeplasminogenbyconvertingitintoitsactiveform,plasmin.Plasminisanenzymethat
proteolyticallycleavesfibrinstrands,therebydegradingfibrinclotsthatmayobstructvessels.In

additiontotPA,streptokinaseandurokinasearesometimesreferredtoasclotbusters(Table132
(t0015)).

Primaryhemostasisistheprocesswherebyplateletsadheretotheunderlyingcollagenof
damagedvascularendotheliumandformatemporaryplateletplug.
Secondaryhemostasisistheprocesswherebytheseplateletsarecovalentlycrosslinked
toformanirreversibleplateletplug,aprocessthatrequiresclottingfactors.
Disordersaffectingplateletfunctionwillimpairprimaryhemostasis.Plateletfunction
canbetestedbycheckingbleedingtime(rarelydone)orinvitroplateletfunctiontests
(morecommonlydone).
Boththeextrinsicandintrinsicpathwaysleadintothecommonpathwaytocause
formationofthefibrinclot.
Theprothrombintime(PT)reflectsthetimeittakesfortheextrinsicandcommon
pathwaystoformafibrinclot.ThePTcanbeprolongedbyclottingfactordeficienciesin
eitherpathway.
Theactivatedpartialthromboplastintime(aPTT)reflectsthetimeittakesforthe
intrinsicandcommonpathwaystoformafibrinclot.TheaPTTcanbeprolongedby
clottingfactordeficienciesineitherpathway.
Aspirinirreversiblyinhibitsplateletsbyinhibitingcyclooxygenase(COX),preventing
theformationofthromboxaneA2,whichnormallypromotesplateletaggregationand
vasoconstrictioninresponsetoendothelialdamage.Aspirintherapyresultsina
prolongedbleedingtime.
HeparinstimulatesantithrombinIIIactivity,therebyinhibitingtheintrinsicpathway
andprolongingtheaPTT.Heparinhasnoeffectonbleedingtimebecauseitdoesnot
(typically)affectplateletfunction.Heparintoxicitycanbereversedwithprotamine
sulfate.
Warfarin(Coumadin)inhibitsthesynthesisofvitaminKdependentclottingfactors(II,
VII,IX,andX)byantagonizingtheactionofvitaminK.ItpreferentiallyprolongsthePT.
Thereistypicallyadelayofseveraldaysbeforepatientsachieveatherapeutic
internationalnormalizedratio(INR)onwarfarinbecausewarfarininhibitsthesynthesis
offurtherclottingfactorsbutdoesnotinhibittheactivityofexistingplasmaclotting
factors.
WarfarintoxicitycanbereversedbyadministeringvitaminK(takes12days)orfresh
frozenplasma(rapid).

Tissueplasminogenactivator(tPA),streptokinase,andurokinaseareclotbusting
drugsthatcanbeusedinparticularclinicalscenariossuchasSTsegmentelevation
myocardialinfarctionoracuteischemiccerebrovascularaccident.
SummaryBox:Hemostasis,CoagulationCascade,Pharmacotherapy

Case131
A7yearoldboyisbroughttoyourofficebyhismotherbecauseofaswollenrightknee.When
questioned,shestatesthatthishashappenedintheabsenceofsignificanttraumaseveraltimes
overthepastfewyears.Hedoesnotappearfebrile,andacompletebloodcount(CBC)is
normal.Bothparentsarehealthy,althoughthemothersaysthatherfather(theboy'smaternal
grandfather)hadsometypeofbleedingdisorder.

Table132
PharmacotherapyforHemostasis

Agent

MechanismofAction

Effecton

Antidote

Clotting
Profile
Aspirin

Irreversiblyinhibitsplateletfunctionbyinhibitingthromboxane
A2synthesis(mediatedbyCOX)
bleeding
time

Heparin StimulatesATIII,whichinhibitstheintrinsicpathway

aPTT

Protamine
sulfate

Warfarin AntagonizesvitaminK,therebyinterferingwithproductionof
clottingfactorsII,VII,IX,andXaswellastheanticoagulant

tPA

PT

VitaminK,
freshfrozen

proteinsCandS

plasma

Stimulatesproductionofplasmin,whichdegradesfibrinclot

Aminocaproic
acid

aPTT,activatedpartialthromboplastintimeATIII,antithrombinIIICOX,cyclooxygenasePT,prothrombintime
tPA,tissueplasminogenactivator.

1 Whatisthemostlikelydiagnosis?
Hemophiliatypicallypresentswithspontaneousbleedingintojoints(hemarthrosis)andsofttissues
orprolongedbleedingfollowingdentalproceduresorminorsurgery.BecausehemophiliaAismore
commonthanhemophiliaB,thisboymostlikelyhashemophiliaA.

2 Whatisthecauseofthisdisorderandhowisitinherited?
HemophiliaAiscausedbyahereditarydeficiencyoffactorVIIIandisinheritedinanXlinked
recessivemanner.Femalesareveryrarelyaffectedbecausetheapproximate50%factorlevelsin
mostcarriersaresufficienttopreventexcessivebleeding.Inrarecircumstances,femalesmaybe
affectedduetounequalinactivation(lyonization)offactorVIIIorfactorIXalleles(seeChapter11,
GeneticandMetabolicDisease,formoreinformation).

Inthiscasestudy,inwhichthematernalgrandfatherwasaffected,theboy'smotherisan
asymptomaticfemalecarrierwhotransmittedthebadXchromosome(fromherfather)tothe
child.
Note:HemophiliaBisanXlinkedrecessivediseaseinwhichfactorIXofthecoagulationcascadeis
deficient.

3 WhichmeasureofcoagulationwillbeabnormalinhemophiliaA?
AdeficiencyoffactorVIII(hemophiliaA)orfactorIX(hemophiliaB)impairsthrombinproduction
bythefactorIXa/factorVIIIacomplex,resultingindysfunctionoftheintrinsicpathwayand
prolongedbleedingfromincreasedaPTT.

4 Whatisthemainstayofmedicaltreatmentforthisdisease?
Treatmentconsistsofinfusionofthemissingfactor(factorVIIIforhemophiliaAorfactorIXfor
hemophiliaB).
Note:Historically,isolationofclottingfactorsfrompooledcollectionsofbloodresultedinhuman
immunodeficiencyvirus(HIV)contaminatedfractions.Thisproblemwasmostpronouncedbefore
thebeginningofHIVscreeningofthebloodsupply.Currently,factorsVIIIandIXcanalsobe
producedbyrecombinantDNAmethods,whichobviatesthisproblem.

HemophiliaAandBarehereditarycoagulopathiescausedbydeficienciesinfactorsVIII
andfactorIX,respectively.TheyareinheritedinanXlinkedrecessivemanner,somale
offspringaremuchmorecommonlyaffected.
BecausefactorsVIIIandIXarecomponentsoftheintrinsicpathway,theactivated
partialthromboplastintime(aPTT)isprolongedinhemophilia.
HemophiliaAandBaretypicallydiagnosedfollowingspontaneousbleedingintojoints
(hemarthrosis)andsofttissuesorprolongedbleedingwithtraumaorminorsurgery.
Treatmentofhemophiliainvolvesreplacementofthemissingclottingfactor.
SummaryBox:Hemophilia

Case132
A35yearoldwomanwhoishavingamissedabortion(fetusdiesbutconceptusisretainedin
uteroforafewmonths)developspetechiaeonherskinandbuccalmucosa,beginscoughingup
someblood(hemoptysis),andobservesbloodinherstool.Whileitisbeingarrangedforher
uterustobeevacuated,aCBCisdoneandrevealsanemiaandthrombocytopenia.Aperipheral
smearrevealsschistocytes,thePTandPTTareelevated(althoughsheisnottakingany
anticoagulants),andtheDdimerlevelisincreased.Freshfrozenplasmaandplateletsare
ordered.

1 Whatisthemostlikelydiagnosis?
Sheappearstobegoingintodisseminatedintravascularcoagulation(DIC),whichischaracterizedby
thrombocytopenia,elevatedPTandPTT,andanelevatedDdimerlevel.

2 Whatisthepathogenesisofdisseminatedintravascularcoagulation?
Procoagulant(thromboplastic)substancessuchastissuefactorandfibrinarereleasedthroughout
thecirculationandactivatetheclottingmechanism.Thisdepletesclottingfactorsandplatelets
(consumptivecoagulopathy),whichthencausesexcessivebleeding.DICisthereforecharacterized
byexcessiveclottingandbleedingoccurringsimultaneouslythroughoutthebody.Laboratoryvalues
willshowdecreasedplateletcountandincreasedbleedingtimeinadditiontothefindingsmentioned
inquestion1.ThemaindiseasestatesinwhichDICoccursareobstetriccomplications(missed
abortion,abruptioplacentae),gramnegativesepsis,andmalignancies.

3 Whatareschistocytesandwhydotheyformindisseminatedintravascular
coagulation?
Schistocytesarefragmentedredbloodcells(RBCs)thathavebeenclotheslinedbyfibrinstrands
thatstreakacrossbloodvessels,tearingoffpartoftheRBCmembraneastheRBCspassby(Fig.132
(f0015)).ThewidespreadfibrindepositionthatoccursinDICcanleadtothiscondition.

Figure132

Schistocyte.Notethattheschistocytepatternshowsredbloodcellswithrefractileringsindicativeofwater
artifact.
(FromYoungNS,GersonSL,HighKA:ClinicalHematology.Philadelphia,Mosby,2005.)

Note:Schistocytesareahallmarkofmicroangiopathichemolyticanemia(hemolyticanemiadueto
intravascularfragmentationofRBCs).

4 Howdoesthepathophysiologyofthromboticthrombocytopenicpurpura
differfromthatofdisseminatedintravascularcoagulation?
Thromboticthrombocytopenicpurpura(TTP)ismarkedbydeficiencyofADAMTS13,a
metalloproteasethatisresponsibleforthedegradationofvWFmultimers.AbsenceofADAMTS13
preventsvWFbreakdownandpromotesahypercoagulablestate.
TTPinvolvesthrombosis,thrombocytopenia,andabnormalbleedingasevidencedbypurpuraon
examination.Itisprecipitatedbywidespreaddamagetotheendothelium.Becauseplateletsadhere
totheexposedsubendothelialcollagenofdamagedvessels,massiveactivationofplateletbindingand
aggregationcancausethrombocytopenia,resultinginbleeding(purpura)andprolongedbleeding
time.However,theclottingfactorsarenotconsumedastheyareinDIC.Additionally,thecauseof
mostcasesofTTPisunknown.BothTTPandDICshowevidenceofamicroangiopathichemolytic
anemia.

5 Whatisthepentadofthromboticthrombocytopenicpurpuraandhowdoes
thisdisorderdifferfromhemolyticuremicsyndrome?
ThepentadofTTPconsistsofmicroangiopathichemolyticanemia,thrombocytopenia,neurologic
symptoms,fever,andrenaldysfunction.Mostofthemanifestationsareexplainedonthebasisofclot
formation,withfibrinstrandscausingthemicroangiopathichemolyticanemia.Theneurologic
symptomsandrenaldysfunctionareduetoclotsandocclusionofthecerebralcirculationand
glomerularcapillaries,respectively.Hemolyticuremicsyndrome(HUS)isverysimilartoTTPand
involvesmostofthesamesymptomsandlaboratoryfindings.Thedistinctiontomakeforboardsis
thatHUSdoesnotinvolveneurologicmanifestations.
Note:EnterohemorrhagicEscherichiacoliO157andShigellaarebothwellestablishedcausative
agentsofHUS.

6 Quickreview:CoverthetwocolumnsattherightinTable133(t0020)andtry
todifferentiatethromboticthrombocytopenicpurpuraanddisseminated
intravascularcoagulationonthebasisofpathogenesis,changesinblood
elements,bleedingtime,andchangesinprothrombintime,activatedpartial
thromboplastintime,andDdimerlevels
AsmentionedinChapter12,hematologyisagreatsubjectfor//normalquestions.We
recommendthatyouthoroughlyunderstandlaboratoryparametersforbleedingdisorders.
TheyareafavoriteUSMLEtesttopicbecause,asyoumayhaveguessed,theyfitinverynicely

withthispopular,multiplechoicequestionformat.
Step1Secret

Disseminatedintravascularcoagulation(DIC)isassociatedwithobstetriccomplications
(missedabortion,abruptioplacentae),gramnegativesepsis,andmalignancy.
DICischaracterizedbythrombocytopenia,prolongedprothrombintime(PT)and
activatedpartialthromboplastintime(aPTT),elevatedDdimerlevel,andthepresenceof
schistocytes.
Schistocytesarefragmentedredbloodcell(RBCs)thathavebeenclotheslinedby
fibrinstrandsthatstreakacrossbloodvessels.Thewidespreadfibrindepositionthat
occursinDICcanleadtothiscondition.Schistocytesareindicativeofamicroangiopathic
hemolyticanemia.
Thromboticthrombocytopenicpurpura(TTP)isprecipitatedbywidespreaddamageto
theendothelium,althoughtheprecisecauseinmostcasesofTTPisunknown.Itis
characterizedbythrombosis,thrombocytopenia,andabnormalbleedingasevidencedby
purpuraonexamination.
TTPandhemolyticuremicsyndrome(HUS)areessentiallythesameentity.
ThepentadofTTPconsistsofmicroangiopathichemolyticanemia,thrombocytopenia,
neurologicsymptoms,fever,andrenaldysfunction.HUSconsistsofallofthese
symptomsexceptneurologicdamage.
SummaryBox:DisseminatedIntravascularCoagulation

Case133
A27yearoldmanisevaluatedforpersistentbleedingfromthegumssinceadentalcleaning
thepriorafternoon.Onexamination,hisgumsappeartobebleedingprofusely,andhismouth
requirespackingwithgauzepadstolimitthebleeding.Hedeniesanyhistoryofabnormal
bleedingoranyfamilyhistoryofbleedingdisorders.Heisnottakingaspirinorother
nonsteroidalantiinflammatorydrugs(NSAIDs).Reviewofsystemsissignificantonlyfor
intermittentpainfulswellingofhisrightkneeoverthepast10years.Suspectingadiagnosisof
hemophilia,theclinicianordersPTandaPTT,bothofwhicharecheckedandreturnnormal.
However,bleedingtimeisprolongedat15minutes.

Table133
ThromboticThrombocytopenicPurpura/HemolyticUremicSyndrome(TTP/HUS)andDisseminatedIntravascular

Coagulation(DIC)

Feature

TTP/HUS

DIC

Pathogenesis

Endothelialdamageleadstoplatelet
consumption

Releaseofprocoagulantsor
endothelialdamage

Changesinformed Thrombocytopenia,schistocytes

Thrombocytopenia,schistocytes,

bloodelements

microangiopathichemolyticanemia

microangiopathichemolyticanemia

Bleedingtime

Increased

Increased

PT

Normal

Increased

PTT

Normal

Increased

Ddimer

Normal

Increased

PT,prothrombintimePTT,partialthromboplastintime.

1 Whatisthemostlikelydiagnosis?
Theprolongedbleedingtimeindicatesplateletdysfunction,andthenormalPTandaPTTargue
againsthemophilia.Givenhisadditionalhistoryofhemarthrosisofthekneeforthepast10years,
thismanlikelyhasvonWillebranddisease,whichisthemostcommonlyinheriteddisorderof
plateletdysfunction.

2 WhatisthenormalfunctionofvonWillebrandfactorandwhatisthe
pathogenesisofthisman'sdisease?
ThehighmolecularweightproteinvWFlinksplateletstoexposedsubendothelialcollagen,oneof
thefirststepsrequiredforclottingtooccur.ThevWFissecretedbyvascularendothelialcells.A
quantitativeorqualitativedeficiencyinvWFcausesvonWillebranddisease.

3 WhymaysomeonewithvonWillebranddiseasebemistakenlydiagnosedwith
hemophiliaA?
HemophiliaAiscausedbylowlevelsoffactorVIII.BecausevWFactsasacarrierproteinforfactor
VIII,patientswithvonWillebranddiseasecommonlyhavelowlevelsoffactorVIIIaswell.Although
thispatient'sclinicalpresentationwasconsistentwithhemophilia,thenormalPTandaPTTlargely
ruleoutthispossibility.
HemophiliaandvonWillebrandfactor(vWF)deficiencyappearwithsimilarclinicalfindings.
Bothmaypresentwithelevatedactivatedpartialthromboplastintime(aPTT)andprolonged
bleedingtime,buthemophiliaisoftenassociatedwithhemarthroses(vWFdeficiencydoesnot
dothis).Thisisagoodexampleofwhyitisimportanttoreadquestionstemscarefully.The
USMLEStep1examoftenprovidesthesegiveawaydetails!

Step1Secret

4 Whatisthemechanismofactionwherebyadministrationofdesmopressin
acetatemighthelpthisman'ssymptoms?
Desmopressinacetate(DDAVP)isusefulinmanyofthebleedingdisorders,asitinducesthehepatic
productionofplasmaclottingfactors.ItiseffectiveinvonWillebranddiseasebecauseitstimulates
thereleaseofvWFfromendothelialcells.However,DDAVPmaynotbeeffectiveinvariantsofvon
WillebranddiseasecausedbyqualitativedefectsinvWFfunction.

Differentialdiagnosis
5 IfthismanhadnormallevelsoffunctionalvonWillebrandfactor,andplatelet
functionstudiesrevealedadefectinplateletadherencetocollagen,whatrare
disorderofplateletfunctionmightyoususpect?
YoumightsuspectBernardSouliersyndrome,whichiscausedbyalackoforabnormalfunctionof
theplateletGPIbIXreceptor.TheGPIbIXreceptorfunctionsinplateletadherenceto
subendothelialcollagenbybindingtovWF(whichisboundtosubendothelialcollagen).

6 Ifplateletfunctionstudiesdemonstratedplateletscapableofadheringto
collagenbutunabletoaggregatewithotherplatelets,whatotherraredisorder
ofplateletfunctionmightyoususpect?
Glanzmann'sthrombastheniaisapossibility.ThisrarediseaseiscausedbyalackoftheGPIIbIIIa
receptoronplatelets,whichmediatesplateletaggregationviaafibrinogenbridge.
Note:ItmaybedifficulttodifferentiateclinicallybetweenvonWillebranddisease,BernardSoulier
disease,andGlanzmann'sthrombasthenia,sotheristocetinassayisoftenused.Ristocetinisa
moleculethatallowsvWFtobindtoGPIb,resultinginagglutination.Intheabsenceofeitherofthese
components(vonWillebranddiseaseandBernardSouliersyndrome),ristocetinassaywillbe
abnormal.InthecaseofGlanzmann'sthrombasthenia,normalaggregationwilloccurafterristocetin
isadded.Boardswillexpectyoutounderstandtheprinciplesbehindtheristocetinassayandhowit
canbeusedtodifferentiatebetweentheaforementioneddiseases.Youshouldalsokeepinmindthat
vonWillebranddiseasewillaffectPTTbecausevWFaidsincarryingfactorVIIIintheblood,whereas
BernardSoulierdiseaseandGlanzmann'sthrombastheniawillnot.

7 GiventhepreviouslymentionedfunctionoftheGPIIbIIIareceptor,whyare
drugssuchasabciximab(Integrilin)giventopatientswithcardiovascular
disease?
ThesedrugspreventplateletaggregationbyantagonizingtheGPIIbIIIareceptorsonplatelets
therefore,theylowertheriskforthromboemboliceventsinhighriskpatients.

8 Whymightyoususpectanabnormalbleedingtimeinthismanifhesuffered
fromdiabeticnephropathyandosteoarthritisforwhichheroutinelytakes
aspirin?

UremiaandNSAIDsarecommoncausesofacquiredplateletdysfunction.

9 Quickreview:CoverthetwocolumnsontherightsideofTable134(t0025)
andtrytodescribethemechanismsofactionforthelistedantiplateletdrugs
ThemostcommonlyinheriteddisorderofplateletdysfunctionisvonWillebrand
diseasevonWillebrandfactor(vWF)functionstolinkplatelets(viatheglycoproteinIb
[GPIbIX]receptor)toexposedsubendothelialcollagen.
PatientswithvonWillebranddiseasemaybemistakenlydiagnosedashaving
hemophiliaAbecausevWFisacarrierproteinforfactorVIII.Anormalactivatedpartial
thromboplastintime(aPTT),however,shouldargueagainstadiagnosisofhemophiliaA.
Desmopressinacetate(DDAVP)maybeusefulintreatingvonWillebranddiseasecaused
byaquantitativedeficiencyinvWFbecauseitstimulatesthereleaseofvWFfrom
endothelialcells.
BernardSouliersyndromeandGlanzmann'sthrombastheniaareraredisordersof
plateletfunction.
BernardSouliersyndromeiscausedbylack/dysfunctionoftheGPIbIXplatelet
receptor,whichmediatesbindingofplateletstocollagenviavWF.
Glanzmann'sthrombastheniaiscausedbylack/dysfunctionoftheGPIIbIIIaplatelet
receptor,whichmediatesplateletaggregationviaafibrinogenbridge.
Drugssuchasabciximab(Integrilin)preventplateletaggregationbyantagonizingthe
GPIIbIIIaplateletreceptortheythereforedecreasetheriskforthromboembolic
phenomenainhighriskcardiovascularpatients.
SummaryBox:VonWillebrandDisease

Case134
Apreviouslyhealthy35yearoldwomancomplainsofeasybruisingandoccasionalnosebleeds
overthelastfewmonths.Shedoesnottakeanymedications.Physicalexaminationreveals
diffusepetechiaeandecchymoses.Laboratorytestsshowaplateletcountof5000/L,white
bloodcell(WBC)countof7200/L,RBCcountof4.6106/L,andanormalPTandaPTT.A
peripheralbloodsmearisunremarkable.Specializedtestingrevealsthepresenceofantiplatelet
antibodies.

Table134
AntiplateletDrugs

Agent

MechanismofAction

Comments

Aspirin

IrreversibleinhibitionofCOX1andCOX2

Mostcommoncauseof

(inhibitsthromboxaneA2synthesis)

plateletdysfunction

Clopidogrel(Plavix)

InhibitsplateletADPreceptoractivation

Abciximab(ReoPro)

PlateletGPIIbIIIainhibitors

Eptifibatide(Integrilin)

MimicsGlanzmann's
thrombasthenia

Tirofiban(Aggrastat)
ADP,adenosinediphosphateCOX1,2,cyclooxygenase1,2.

1 Whatisthemostlikelydiagnosisinthiswoman?
Thissoundslikeimmunethrombocytopenicpurpura(ITP).However,itshouldberealizedthatITPis
adiagnosisofexclusion.Therefore,themyriadofothercausesofthrombocytopenia(e.g.,marrow
infiltrativeprocessessuchascancer,infectionssuchasrubella,folateorvitaminB12deficiency,DIC,
TTP)mustbeconsidered.

2 Whatistheetiologyofimmunethrombocytopenicpurpura?
ITPiscausedbytheproductionofimmunoglobulinG(IgG)autoantibodiestoplatelets,resultingin
destructionofantibodycoatedplateletsinthespleen.Inchildren,ITPisoftenprecededbyanupper
respiratorytractinfection.
Note:ThespleenisalsotheprimarysiteforsynthesisofantiplateletantibodiesinITP.Forthis
reason,asplenectomycanoftenbetherapeuticinrefractorycases.

3 Wouldabonemarrowbiopsyinthiswomanlikelyrevealincreasedor
decreasednumbersofmegakaryocytesandwhy?
Normaltoincreasedmegakaryocytes,theprecursorstoplatelets,wouldberevealed.Thisfinding
indicatesthatthethrombocytopeniaisduetoincreasedplateletdestruction,notreducedproduction.

4 Whatanticoagulantiswellknownforcausingthrombocytopenia?
Heparincancausethrombocytopenia.Asmanyas1%to3%ofpatientsreceivingheparinmay
developheparininducedthrombocytopenia(HIT),which,oddlyenough,canpresentwitheither
excessivebleedingorexcessivethrombosis.ThrombocytopeniainHITisthoughttobecausedbythe
productionofanIgGantibodyagainstheparin.Theseantiheparinantibodiesbindtobothheparin
andaproteincalledplateletfactor4(PF4).Uponinitiationofthisbinding,theFcregionofthe
antibodybindstotheplateletsurface,resultinginwidespreadplateletactivationandsubsequent
plateletdepletion(i.e.,thrombocytopenia).

5 Whyistheabsenceofsplenomegalyimportantandthereforehelpfulin
determiningthecauseofthrombocytopenia?

Splenomegaly/hypersplenismcanresultinincreasedplateletsequestrationandthrombocytopenia.
Splenomegalymostcommonlyresultsfromcongestionduetoportalhypertension,butmayalso
resultfromleukemia,lymphoma,andseveralotherdiseaseprocesses.Splenomegalyisnottypically
seeninITP,althoughitisoccasionallyseeninchildrenwithITPfollowingaviralinfection.

6 Canpregnantwomenwiththisdiseaseaffecttheplateletcountoftheir
fetuses?
Yes,ITPcancauseneonatalthrombocytopeniabecausetheautoantibodies,predominantlyIgG,are
abletocrosstheplacentaandattackfetalplatelets.

7 Whatisthetreatmentstrategyforthisdisease?
Corticosteroidssuchasprednisonewillsuppresstheimmunologicallymediateddestructionof
platelets.Ifsteroidsfail,intravenousimmunoglobulin(IVIG)maybehelpful.Otheragentswith
someefficacyincludecyclophosphamide,azathioprine,anddanazol.Ifnecessary,asplenectomyis
oftenhelpful.

Immunethrombocytopenicpurpura(ITP)isanautoimmunethrombocytopeniacaused
bytheproductionofantibodiestoplateletsresultinginsplenicdestructionofcirculating
platelets.
Inchildren,ITPisoftenprecededbyanupperrespiratorytractinfection.
ITPisadiagnosisofexclusion.Othercausesofthrombocytopeniasuchasmarrow
infiltrativeprocesses,infections,drugs,andfolateandvitaminB12deficiencyneedtobe
considered.
BonemarrowbiopsyinITPwillshownormaltoincreasedlargemegakaryocytes.
Heparininducedthrombocytopeniaisaconcerninhospitalizedpatientsreceiving
heparin.ItoccursasaresultofthesynthesisofanIgGantibodyagainstheparinandthis
antibodyalsoreactswithplatelets.
SplenomegalyisnotnormallypresentinITPinanadultwithsplenomegaly,one
thereforehastoquestionthediagnosisofITP.
Treatmentoptionsincludecorticosteroids,intravenousimmunoglobulin,danazol,
immunosuppressantssuchascyclophosphamideandazathioprine,andsplenectomy.
SummaryBox:ImmuneThrombocytopenicPurpura

Case135

A58yearoldwomancomplainsofaswollenandpainfulrightlegsincereturningfromatripto
Europe2daysago.Pastmedicalhistoryissignificantformetastaticbreastcancer,forwhich
sheiscurrentlybetweenchemotherapyregimens.Onexamination,herrightcalfiswarmand
tendertopalpationandmeasures20cmindiameter,comparedwithherleftcalf,whichis
nontendertopalpationandmeasures16cmindiameter.Compressionultrasonographyreveals
aclotintherightfemoralvein.

1 Whatisthediagnosis?
Shehasdeepvenousthrombosis(DVT).RiskfactorsforDVTincludeprolongedimmobilization(e.g.,
recentsurgery,transcontinentalflight),malignancy,estrogenandoralcontraceptives,obesity,heart
failure,andahereditarypredisposition.
Note:Forcompleteness,otherriskfactorsincludethefactorVLeidenmutation(whichresultsin
activatedproteinCresistance),prothrombinmutant,homocysteinemia,antiphospholipidsyndrome,
andHIT.

2 Fromwhatsitedodeepvenousthromboses,whichgiverisetopulmonary
embolisms,typicallyarise?
Theyarisefromthedeepveinsofthecalfmusclesoriginally,butcalfDVTsneedtopropagateintothe
deepveinsoftheproximalleg(popliteal,femoral,oriliac)tobecomelargeenoughtocauseclinically
significantpulmonaryemboli.

3 WhatisVirchow'striadandhowdoesthisrelatetothispatient?
Virchow'striadincludesthethreegeneralcausesofincreasedclotting,whichareabnormalitiesof
thevesselwall(asinvasculitisandatherosclerosis),abnormalitiesofbloodflow(e.g.,stasis),and
abnormalitiesofbloodcoagulability(e.g.,deficienciesofanticoagulants,presenceofprocoagulants).
Ofthese,thispatienthadarecenthistoryofimmobilizationfromherlongflight,whichcancause
hemostasis.Additionally,shelikelyhasahypercoagulablestatesecondarytohermalignancy.

4 HowdodeficienciesofproteinsCandSandantithrombinIIIpredisposeto
deepvenousthrombosis?
ProteinsC,proteinSandATIIIareallinhibitorsofvariousclottingfactors.Deficienciestherefore
resultsinahypercoagulablestate.
Note:ProteinCnormallyfunctionstodegradefactorV(amongotheractions).Patientswiththe
factorVLeidenmutationsynthesizeafactorVthatisresistanttodegradationbyactivatedproteinC,
resultinginahypercoagulablestate.

5 Whatisantiphospholipidsyndrome?

Antiphospholipidsyndromeisanautoimmunecoagulationdisorderthatresultsfromantibody
productionagainstphospholipids,whichareoftenboundtoplasmaproteins.Ithasahigh
associationwithotherautoimmuneconditions(particularlysystemiclupuserythematosusandHIV)
andpredisposespatientstoarteriorandvenousthrombosissyndromes.Youshouldconsiderthis
diagnosisinawomanwithanautoimmuneconditionwhoexperiencesstroke,DVT,orhepaticvein
thrombosis.Inaddition,thewomanmayhaveexperiencedaspontaneousabortionduetoplacental
thrombosis.

Riskfactorsfordeepvenousthrombosis(DVT)includeprolongedimmobilization(e.g.,
recentsurgery,transcontinentalflight),malignancy,estrogenandoralcontraceptives,
obesity,heartfailure,andahereditarypredisposition.
StillotherriskfactorsincludethefactorVLeidenmutation,prothrombinmutant,
homocysteinemia,antiphospholipidsyndrome,andheparininducedthrombocytopenia.
DVTs,whichgiverisetopulmonaryembolisms,mostcommonlyarisefromthedeep
veinsoftheproximalleg.DVTsinthecalfaretypicallytoosmalltocauseclinically
significantpulmonaryemboli.
Virchow'striadreferstothethreegeneralconditionsthatpredisposetoclotting:
vasculardamage,venousstasis,andclottingfactordeficiencies/abnormalities.
DeficienciesinproteinC,proteinS,andantithrombinIIIaswellasthefactorVLeiden
mutationmayresultinahypercoagulablestate.
SummaryBox:DeepVenousThrombosis

Copyright2015Elsevier,Inc.Allrightsreserved.

BOOKCHAPTER

Cardiology
RajGaneshan,ThomasA.BrownMDandSonaliJ.Shah
USMLEStep1Secrets,Chapter1,133

CardiologyisawidelytestedsubjectontheUSMLEStep1,soitisimportanttoachieveagood
understandingofboththephysiologyandpathologyoftheheart.Becomewellversedin
pressurevolumeloops,theWiggersdiagram,murmurs,andheartsounds(youwilllikelygeta
fewaudioquestionsontheUSMLEthatwillrequireyoutoidentifyvalvulardefectsbasedon
thequalitiesofthemurmursdetectedthroughamovablevirtualstethoscope),actionpotentials
ofatrialandventricularmyocytesversuspacemakercells,andcommonpathologicconditions
oftheheart(e.g.,rheumaticfever,congestiveheartfailure,cardiomyopathies,endocarditis).
Cardiacpharmacologyisalsoahighyieldsubject.Themajorityofcardiologyquestionsonthe
USMLEwillrequireyoutoapplyconceptsratherthanfacts,soworkingthroughthecasesin
thischapterwillbeoftremendousvalueinyourpreparationforthissubject.
Insider'sGuidetoCardiologyfortheUSMLEStep1

Basicconceptshemodynamics
1 Whatarethemathematicaldeterminantsofthearterialbloodpressure?
Themeanarterialpressure(MAP)isdeterminedbyhowmuchbloodtheheartpumpsintothe
arterialsysteminagiventime(thecardiacoutput[CO])andhowmuchresistancethearterieshave
tothisinput(totalperipheralresistance[TPR]).Mathematically,thisisexpressedasMAP=CO
TPR.Consequently,alldrugsthatlowerbloodpressureworkbyaffectingeithertheCOorTPR(or
both).
Note:Theprimarydeterminantofsystolicbloodpressure(SBP)isCO,whereastheprimary
determinantofdiastolicbloodpressure(DBP)isTPR.Becauseapproximatelyonethirdofthe
cardiaccycleisspentinsystoleandtwothirdsindiastole,theMAPcanbecalculatedasMAP=1/3
SBP+2/3DBP.

2 Whataretheprimarydeterminantsofcardiacoutput?
TheCOistheamountofbloodpumpedbytheventriclesperunittime.Itisdeterminedbythe
volumeofbloodejectedduringeachventricularcontraction(strokevolume[SV])andhowfrequently
theheartbeats(heartrate[HR]),expressedasCO=HRSV.TheHRcanbeaffectedbyavarietyof

factorsbutisprincipallyunderthecontroloftheautonomicnervoussystem.Betablockerscan
reduceCObydecreasingHRandcontractility.
Note:Inadditiontotheirnegativeinotropiceffect,themorecardioselective(nondihydropyridine)
calciumchannelblockers(verapamil,diltiazem)canalsoreduceHRbyslowingimpulsetransmission
throughtheatrioventricular(AV)node.Theyachievepartoftheirantihypertensiveeffectthrough
thismechanism.

3 Whatarethethreemainfactorsthataffectstrokevolume?
ThedeterminantsofSVarepreload,contractility,andafterload.

4 Whatispreload,andhowdoesitaffectstrokevolume?
Preloadisthedegreeoftension(load)ontheventricularmusclewhenitbeginstocontract.The
primarydeterminantofpreloadisenddiastolicvolume.
ThemostwidelyacceptedtheoryexplainingtherelationshipofpreloadandSVistheFrankStarling
mechanism,whichdescribeshowanincreasedpreloadresultsinanincreasedSV.Itstatesthat
stretchingofventricularmusclefibersoccurswithincreasingenddiastolicvolumes,causinggreater
overlapbetweenactinandmyosinwithinsarcomeres.Thisresultsinagreaterextentandvelocityof
myocyteshorteningduringcontraction,whichallowsforastrongerventricularcontractionand
largerSV.Thismechanismallowsthehearttomaintainitsejectionfractioninthefaceofincreased
preload.Bydecreasingintravascularvolume,diureticsreducepreloadandcanbeusedtolowerblood
pressure.Venodilatorsalsoreducepreloadandcanthereforebeusedforsimilarpurposes.
Inaddition,avarietyofpositionsormaneuverscanbetriedtomanipulatevenousreturn(preload)to
theheart.Forexample,boththeValsalvamaneuver(expirationagainstaclosedglottis)andstanding
willdecreasepreload,andbothsquattingandpassivelegraisingwillincreasepreload.Havingthe
patientperformtheseactionscanbeusefulwhendistinguishingvariousmurmursfromoneanother(
Fig.11(f0010)).

Figure11
Increasedventricularoutputasafunctionofenddiastolicvolume(reflectedbyatrialpressure).
(FromGuytonAC,HallJE:TextbookofMedicalPhysiology,11thed.Philadelphia,WBSaunders,2006,p112.)

Note:AnothertheorytoexplaintheFrankStarlingrelationshipproposesthatcardiactroponin
becomesincreasinglysensitivetocytosoliccalciumatgreatersarcomerelengths,therebyresultingin
increasedcalciumbindingandincreasedforceofmusclecontraction.Notethatbeyondacertain
point,increasingpreloadswillresultinlessefficientventricularcontractionandasmallerSV.This
situationoccursinheartfailure.

5 Whatiscontractilityandhowdoesitaffectstrokevolume?
Contractilityisameasureofhowforcefullytheventriclecontractsatagivenpreload.Naturally,a
moreforcefulcontractionwillejectagreaterfractionofbloodfromtheventricle,therebyincreasing
theSV.Contractilityisprincipallyinfluencedbytheactivitiesofthesympatheticnervoussystem(1
adrenergicreceptors)andparasympatheticnervoussystem(muscarinic[M2]cholinergicreceptors)
onventricularmyocytes.Byantagonizingthissympatheticinputtothemyocardium,betablockers
exertpartoftheirantihypertensiveeffectsbyreducingcontractility,whichreducesSV,CO,and
oxygendemand.Contractilityisalsoincreasedbyincreasedconcentrationsofintracellularcalcium
(whichisindirectlyachievedbydigitalisanddecreasedconcentrationsofextracellularsodium).This
mechanismwillbeexplainedinfurtherdetailinthediscussionregardingdigitalis.Inadditiontobeta
blockade,contractilityisdecreasedbysystolicdysfunction,hypoxia,hypercapnia,calciumchannel
blockade,andacidosis(K+lossfromcellssecondarytoH+/K+exchangeresultsinamorenegative
transmembranepotentialthatdecreasesmyocyteexcitability).

6 Whatisafterloadandhowdoesitaffectstrokevolume?
Afterloadisthepressureorresistanceagainstwhichtheventriclesmustpumpblood,theprimary
determinantofwhichissystemicarterialpressure.Foragivenpreloadandcontractility,increasing
theafterloadwilldecreasetheSV.Asimplifiedwayofunderstandingthisistothinkofthetime
availableforelectricalandmechanicalsystoleasfinite.Withincreasedafterload,moretimeistaken
upbyisovolumiccontractiontobuilduptoapressurethatexceedstheaorticpressureandallowthe
aorticvalvetoopen.Thisstepleaveslesstimeforbloodtoentertheaortafromtheventricle(SV)
duringtherapidandslowejectionphases.
Note:Inaorticstenosis,thestenoticaorticvalveincreasestheafterload,whichintheabsenceof
compensatorychangessuchasventricularhypertrophytendstoreduceSVandCO.Systemic
hypertensionalsoincreasesafterloadbyincreasingthepressureagainstwhichtheleftventriclemust
pump.

7 Whataretheprimarydeterminantsofperipheralresistance?
Totalperipheralresistance(TPR)tobloodflowisprincipallymediatedbyarteriolardiameter,which
ismodifiedbyarteriolarvasoconstrictionanddilation,respectively.Recallthatresistancetoblood
flowthroughavesselisinverselyproportionaltothefourthpoweroftheradius.Hence,relatively
smallchangesinarteriolardiameter(andthusradius)canhaveprofoundeffectsonbloodflow.

Thesympatheticnervoussystempromotesarteriolarvasoconstrictionbystimulating1adrenergic
receptors,whichincreasescalciuminflux(viacalciumchannels)intoarteriolarsmoothmuscleand
stimulatestheircontraction.Consequently,1adrenergicreceptorsandarteriolarcalciumchannels
aretwoselectivetargetsforantihypertensivedrugs.
Inallorgansexceptforthelungs,arteriolarvasodilationispromotedbytissuehypoxiaand
accumulationofmetabolicwastes,suchasadenosine,thataccumulatewhenoxygendemand
increases(e.g.,duringexercise).Thisvasodilationallowssupplytomeetdemand.
Note:Ingeneral,thereisnodirectparasympatheticinnervationofthevasculature.However,
vasodilationofarteriolescanbecausedbyexogenouscholinomimeticadministration.Thesedrugs
actonuninnervatedmuscarinicreceptors(M3receptors)onendothelialcellsandstimulaterelease
ofnitricoxide.Nitricoxidediffusestotheadjacentsmoothmuscle,resultinginvasodilationand
decreasedperipheralresistance.

8 Whatisthemechanismbywhichthesympatheticnervoussystemrespondsto
areductioninbloodpressure?
Whenbloodpressuredrops,arterialbaroreceptorslocatedwithinthecarotidsinus(afferentlimb
mediatedbytheglossopharyngealnerve)sensedecreasedvesselstretchandfirelessfrequently.This
responseincreasesefferentsympatheticoutflowandinhibitsparasympatheticoutflow,whichhelps
restorethebloodpressurebyincreasingheartrateandstimulatingperipheralvasoconstriction.
Conversely,ifthebloodpressureincreases,baroreceptorsinthecarotidsinusoraorticarch(afferent
limbmediatedbythevagusnerverespondsonlytoincreasesinbloodpressure)firemorefrequently
becausetheyarebeingstretchedmore,whichcausesgreaterinhibitionofthesympatheticoutflow(
Fig.12(f0015)).

Figure12
Controlofbloodpressurebythebaroreceptorreflex.
(FromBrownTA:RapidReviewPhysiology.Philadelphia,Mosby,2007,p144.)

Note:Theaorticarchandcarotidsinusesalsohavechemoreceptors,whichshouldnotbeconfused
withthebaroreceptors.ChemoreceptorsworktomaintainPo2,Pco2,andpH.

9 Howdothe1receptorantagonistswork?

The1receptorantagonistsincludethezosins(prazosin,terazosin,doxazosin)andantagonize
peripheralvasoconstrictionstimulatedbythesympatheticnervoussystem(whichismediatedby1
receptors).1ReceptorsarelocatedonvascularsmoothmuscleandcoupledtoGqproteins.
Antagonistscausedecreasedreleaseofinositoltriphosphate(IP3)andsubsequentlypreventthe
releaseofcalciumfromintracellularstores,resultinginsmoothmusclerelaxationandarteriolar
vasodilation.
1Receptorsarealsoresponsibleforcontractionofthepupillarydilatormuscleand
intestinal/bladdersphincters.Thus,1receptorantagonistscanleadtomiosisandbladder/bowel
movement.

10 Howdothe1receptorantagonistscauseorthostatichypotension?
Uponstandingfromasupineorsittingposition,transienthypotensionandlightheadedness(from
cerebralhypoperfusion)mightoccurasaresultofvenouspoolinginthelowerextremities,which
decreasesvenousreturnandMAP.Thisresponseisordinarilycompensatedforbythebaroreflex,
whichpromotesperipheralvenoconstrictionandtachycardia.However,ifthe1receptorsare
blockedintheperipheralvenules,thisreflexwillbelesseffectiveatrestoringthebloodpressure.
Nevertheless,areflextachycardia,whichismediatedbyreceptors,willbemaintained.This
increaseinpulseratecanbeusedindiagnosingorthostatichypotension.
Note:ReflextachycardiaoccurstomaintainCO.RecallthatCO=HRSV.Thus,ifSVisreduced
becauseofdecreasedvenousreturntotheheart,HRmustincreasetomaintainCO.

11 Whathemodynamicchangesoccurduringexercise?
Exerciserequiresmoreoxygentobedeliveredtoskeletalmuscletomeetitsincreasedmetabolic
demand.ThisdeliveryisaccomplishedmainlybyanincreaseinCOsecondarytoincreasesinboth
SVandHR.Contractionofthelowerlimbmusclespushesbloodtowardtherightatriumand
increasesvenousreturn.MAPisonlymodestlyincreasedduringexercisedespitethelargeincreasein
CO,becauseskeletalmusclevasodilationismediatedmostlybylocalcellularmetabolites,which
significantlydecreasetheSVRandallowskeletalmuscletoreceiveupto85%oftheincreasedCO.

12 WhichclinicalscenarioswouldshifttheCOandvenousreturncurvestothe
pointslabeled1to4onFigure13(f0020)?
1.Exercise:Lowerlimbmusclespushbloodtowardtherightatriumandincreasevenous
return.SympatheticactivityincreasesCObyincreasingHR,SV,andcontractility.
2.Arteriovenousfistulas:Increasedvenousreturnfromanarteriovenousfistulawillshiftthe
venousreturncurvetotheright.COdoesincreasebutonlybecauseoftheincreasedpreload
(FrankStarlingmechanism)thisincreaseisthereforenotduetoachangeincontractility
(inotropy).Ifthesearteriovenousanastomosesweremuchlarger,theoperatingpointofthe

heartwouldbeshiftedto(1)becauseitwouldcausealargedecreaseinSVRandstimulatethe
activityofthesympatheticnervoussystem,increasinginotropytheselargeanastomosesare
sometimesreferredtoasAVshunts.
3.Compensatedheartfailure:Patientswiththisconditionhaveelevatedrightatrialpressures
duetoanincreasedvolumestatuscausedbytheactivityofthereninangiotensinaldosterone
system(RAAS).Theircardiacfunctionisdecreased(decreasedinotropy),buttheycanmaintain
anormalCOatrestwiththeincreasedvolume(FrankStarlingmechanism).
4.Ventricularfibrillation:Ventricularfibrillationcausesequalizationofallpressures.Right
atrialpressureincreasestobecomeequaltothemeansystolicfillingpressure.COinventricular
fibrillationsimplybecomesequaltozero.

Figure13
Cardiacoutput(CO)andvenousreturncurves.LVEDV,leftventricularenddiastolicvolume.

Basicconceptsexcitationcontractioncoupling
1 Whatisthesourceofcytosoliccalciumduringventricularsystole?
Duringtheplateauphase(phase2)oftheventricularmyocyteactionpotential,voltagegatedcalcium
channelsallowcalciuminfluxfromtheextracellularfluidintothecytosol,stimulatingcalcium
releasefromthesarcoplasmicreticulum,aphenomenonreferredtoascalciuminducedcalcium
release.Infact,themajorityofthecytosoliccalciumcomesfromthesarcoplasmicreticulum,notthe
extracellularfluid.Thismechanismofcalciumreleaseisincontrasttoreleasefromskeletalmuscle,
inwhichdepolarizationofthecellmembranetriggerssarcoplasmiccalciumreleasewithoutentryof
extracellularcalciumintothecytosol.

2 Whatisthefunctionofcalciumincardiacmusclecontraction?
CytosoliccalciumbindstotroponinC,resultinginaconformationalchangethatremoves
tropomyosinfrommyosinbindingsitesonactintoallowfortheslidingfilamentmechanismof
contraction.TheforceofcontractionisproportionaltotheintracellularCa2+level.Notethatunlike
skeletalmuscle,cardiacmuscleisdependentonextracellularcalciuminfluxforcontractiontooccur.
Thecardioselectivecalciumchannelblockers(verapamil,diltiazem)reducecontractilityby
antagonizingextracellularcalciumentryandthesubsequentcalciuminducedcalciumreleasethat
occursinheartmuscle.Inadditiontodecreasingheartrate,thisactionisanothermechanismby
whichcalciumchannelblockersworktolowerbloodpressure.

3 Whatisthemechanismbywhichadrenergicstimulationincreasescardiac
contractility?
Adrenergicstimulationresultsinanincreaseincyclicadenosinemonophosphate(cAMP),which
promotescAMPdependentphosphorylationofanumberofproteinsviaproteinkinaseA(PKA).
PhosphorylationofLtypecalciumchannelsresultsinincreasedcalciumentryintothemyocyte.In
addition,adrenergicstimulationresultsinphosphorylationandinhibitionofaproteincalled
phospholamban,whichnormallyservesasaninhibitorofthesarco/endoplasmicreticulumcalcium
adenosinetriphosphatase,orATPase(SERCA).Thus,inhibitionofphospholambanallowsfor
increasedcalciumentryintothesarcoplasmicreticulumandsubsequentincreaseincalciumrelease
duringthenextactionpotential,whichaugmentsmyocytecontractility.

4 Whatisthecontributionofthesympatheticnervoussystemtoventricular
relaxation?
Inadditiontostimulatingcalciuminflux,theadrenergicpathwayalsostimulatescalciumuptake
bytheventricularsarcoplasmicreticulumduetophosphorylationofSERCAbyPKA.Thisremovalof
cytosoliccalciumintothesarcoplasmicreticulumisrequiredforventricularrelaxationsothemore
rapidlyitisremoved,themorerapidlytheventriclesrelax.Suchrapidventricularrelaxationat
elevatedHRsisimportanttoensureadequateventricularfillingduringthedecreasedperiodof
diastole.RecallthatHRisincreasedwithsympatheticstimulationviadirectbindingofcAMPto
specialchannelsinthepacemakercellsthatconducttheIfcurrent(funnycurrentthiscurrentis
carriedbysodiumionsandallowsthemembranepotentialtobecomeprogressivelylessnegative
duringtherepolarizationphaseofthepacemakercell),whichincreasestheprobabilityoftheiropen
time,therebypromotingsodiuminfluxandincreasingtheslopeofphase4depolarization.
Note:Calciumuptakeintothesarcoplasmicreticulumisanenergyrequiringprocess,andin
ischemicheartdiseasethereducedoxygendeliverymakescalciumuptakelessefficient,thereby
impairingventricularrelaxationandcausingdiastolicdysfunction.

Basicconceptsarrhythmias
1 Whatistherelationshipbetweenthevariousphasesoftheventricularmyocyte
actionpotentialandthedifferentionfluxesacrossthecellmembrane?

Inphase0oftheactionpotential,thesharpriseinmembranevoltageisduetosodiuminflux.Phase
1involvesabriefrepolarizationthatisduetothetransientoutwardflowofpotassiumthatfollows
sodiumchannelinactivation.Inphase2,theactionpotentialplateausareduetoabalancebetween
calciuminfluxandpotassiumefflux.Duringphase3,thereisrapidrepolarizationduetounopposed
potassiumefflux.Phase4istherestingpotential,whichismaintainedpredominantlythroughthe
openingofpotassiumchannels.IntracellularconcentrationsofK+aremaintainedathighlevelsin
cardiacmyocytesbecauseoftheactionofmembraneboundNa+K+ATPase.Openingofpotassium
channelsduringphase4leadstopotassiumefflux(downitsconcentrationgradient).Sincethecellis
permeableonlytopotassiumatthistime,negativelychargedcounterionsforK+areunableto
diffuseoutwardwithpotassium.Aspotassiumleavesthecell,anionsleftbehindcausethecellto
becomeincreasinglynegativeincharge.Thereforetheeffluxedpotassiumionsareattractedback
towardtheinteriorofthecelltomaintainrestingpotential.Becausephase4isdominatedby
potassiumpermeability,itthereforehasavalueclosetothepotassiumreversalpotential(85mV)(
Fig.14(f0025)).

Figure14
Phasesoftheventricularmyocyteactionpotential.
(FromBrownTA,BrownD:USMLEStep1Secrets.Philadelphia,Hanley&Belfus,2004,p77.)

Note:Theantiarrhythmicagentsallworkbyaffectingoneormorecomponentsoftheaction
potential.ClassIantiarrhythmicsblocksodiumchannelsandantagonizephase0.ClassIII
antiarrhythmicsworkbyblockingpotassiumchannels,whichprolongsphase3depolarization.Some
classIAandallclassIIIantiarrhythmicsincreaseactionpotentialdurationaswellastheQTinterval.
Toxicityoftheseagentscanleadtotorsadesdepointes,whichisassociatedwithlongQTsyndrome.

2 Whatisresponsibleforthedriftingoftherestingmembranepotentialinnodal
cells?
Thesecellsaremorepermeabletosodium,sosodiuminfluxduringtherestingmembranepotential
causesthemembranetograduallydepolarize.Becausenodalcellslackthefastvoltagegatedsodium
channels(INa)foundintherestofthemyocardium,thisisaccomplishedbytheIfsodiumcurrent,a
uniqueleakysodiumchannelthatpromotesthegradualdepolarizationofthesecellsthrough
sodiuminflux.Eventually,whenthemembranedepolarizestoitsthreshold,thisstatewillactivate
slowcalciumchannelsthatengenderanactionpotential(Fig.15(f0030)).

Figure15
Rhythmicdischargeofasinusnodalfiber.Thesinusnodalactionpotentialisalsocomparedwiththatof
ventricularmusclefiber.
(FromGuytonAC,HallJE:TextbookofMedicalPhysiology,11thed.Philadelphia,WBSaunders,2006,p117.)

Note:Thecalciumchannelblockersverapamilanddiltiazemaffectheartratebyantagonizingthese
slowcalciumchannelsontheSAnode.ThesedrugsareconsideredclassIVantiarrhythmics.

3 Throughwhatmechanismdoessympatheticstimulationincreaseheartrate?
Thereleaseofnorepinephrinefromsympatheticneuronscausesactivationof1adrenergic
receptorsinnodaltissue.ThesereceptorsstimulateproductionofcAMP,resultinginanincreaseinI
fandapositivechronotropiceffectontheheart.Inessence,sympatheticstimulationincreasesthe
cellularinfluxofsodiumionsanddecreasestheeffluxofpotassiumions,thusincreasingtheslopeof
therestingpotentialinthenodalcells.Betablockersreduceheartratebyantagonizingthiseffect.
Note:ThebetablockersareconsideredclassIIantiarrhythmics.

4 Whataretheclassesofantiarrhythmicsandhowdotheirmechanismsof
actionandpotentialsideeffectsvary?
SeeTable11(t0010)forthisinformation.
Table11
AntiarrhythmicDrugs

Drug MechanismofAction(tf0010)
Class

Prototype
Agent(s)

IA

Quinidine,
Lupuslikesyndrome
procainamide (procainamide),torsadesde

InhibitsNa+andK+channels,prolongs
QRScomplexandQTinterval,prolongs

pointes

effectiverefractoryperiod(ERP)
IB

InhibitNa+channels,shortens
repolarization,QTinterval

PotentialSideEffects

Lidocaine

IC

InhibitNa+channels,prolongsQRS

Flecainide

complex
II

PRinterval,automaticity(slopeof
phase4depolarizationinnodalcells)

Propranolol

III

InhibitsK+channels

Amiodarone

Pulmonaryfibrosis,corneal
deposits,graymansyndrome,
hepatotoxicity,thyroid
dysfunction

IV

Inhibitscalciumchannels,PRinterval,

Verapamil,

automaticity

diltiazem

Flushing

*Thereisconsiderableoverlapregardingthemechanismsofactionoftheseantiarrhythmics.Forthe
sakeofsimplicity,onlytheprimarymechanismofactionisconsideredinthisclassification.

Case11
A60yearoldmanpresentsforhisthirdvisitin2monthswithabloodpressureof
approximately155/95mmHgoneachoccasion.Physicalexaminationisunremarkable.A3
monthtrialofdietandexercisemodificationsfailstoreducehisbloodpressure.

1 Whatarethetypesofhypertensionandwhichdoesthispatientmostlikely
have?
Thetwotypesofhypertensionareessential(primary,idiopathic)hypertensionandsecondary
hypertension.Essentialhypertensionisthoughttoaccountforapproximately90%ofcasesof
hypertensionandismostlikelytobeduetoaninabilityofthekidneytoproperlyexcretesodiumata
givenfilteredloadthishasbeendescribedthroughthepressurenatriuresistheory.When
approachingapatientwithhypertension,itisimportanttofirstruleoutsecondaryhypertension,
whichindicatesadditionalpathologicchanges.Treatmentofsecondaryhypertensionisaimedat
addressingtheunderlyingcauseofthecondition.Potentialsourcesofsecondaryhypertension
includerenalarterystenosis,primaryhyperaldosteronism,pheochromocytoma,coarctationofthe
aorta,chronicrenaldisease,excessivealcoholuse,pregnancy,increasedintracranialpressure,and
variousmedications,suchasmonoamineoxidaseinhibitors,oraldecongestants,nonsteroidalanti
inflammatorydrugs,andoralcontraceptives.Ifcausesofsecondaryhypertensionareruledout,then
essentialhypertensionisdiagnosedbyexclusion.
Note:Themechanismsinvolvedinessentialhypertensionarepoorlyunderstood.Asecondtheory
proposesthatpeoplewithessentialhypertensionhaveincreasedvascularresistance.Thismaybedue
toincreasedcirculatingvasoconstrictors,increasedsensitivitytothesesubstances,oradeficiencyof
thenitricoxidevasodilationpathway.

2 Howishypertensiondefinedandwhatarethepotentialcomplications?
TheJointNationalCommitteeonDetection,Evaluation,andTreatmentofHighBloodPressure
guidelines(JNC7)fordefininghypertensionarepresentedinTable12(t0015).
Table12
JointNationalCommitteeonDetection,Evaluation,andTreatmentofHighBloodPressureGuidelinesfor
DefiningHypertension

Category

BloodPressureRange(mmHg)
Systolic

Diastolic

Normal

<120

<80

Prehypertension

120139

8089

Hypertension:stage1 140159

9099

Hypertension:stage2 160

100

Evidencesuggeststhattheriskforcomplicationsinhypertensivediseaseisacontinuum,increasing
asbloodpressurerises.Itisalsolargelyinfluencedbycomorbidconditions.Majorcomplicationsof
hypertensionincludeacceleratedatherosclerosis,prematurecardiovasculardisease,diastolic(andto
alesserextent,systolic)heartfailure,stroke,intracerebralhemorrhage,chronicrenalinsufficiency,
endstagerenaldisease,retinopathy,andacutehypertensivecrisis.

3 Whatarefourclassesofdrugsthatcouldbeusefulintreatingthisman's
hypertension?
Diuretics(e.g.,thiazides,loop,potassiumsparing)
Inhibitorsofthereninangiotensinaldosteronesystem(e.g.,angiotensinconvertingenzyme
[ACE]inhibitors,angiotensinreceptorblockers)
Vasodilators(e.g.,directacting,calciumchannelblockers)
Sympatholytics(e.g.,clonidine)

Case11continued:
Becausethispatienthasreceivedthreeelevatedbloodpressurereadingsinthepast2months
andisunresponsivetolifestylemodifications,heisstartedonmetoprolol.

4 Whatisthemechanismbywhichmetoprololwillreducebloodpressureinthis
man?

Metoprololisarelativelyselective1adrenergicreceptorblocker.Antagonizingthesereceptors
decreasesheartrateandcontractility,bothofwhichreduceCO,therebyloweringbloodpressure
(recallthatMAP=COTPR).Betablockersalsoreducebloodpressurebyblockingtherenal
secretionofrenin.Both1and2adrenergicblockersappeartoinhibitthereninangiotensin
aldosteronesystem(RAAS)tosomeextent.Forthisreason,betablockersmightbemoreeffectivein
hypertensivepatientswithelevatedplasmareninlevelsasopposedtonormalorlowplasmarenin
levels.

5 Whatpharmacologicpropertymakescertainbetablockersmore
cardioselectivethanothers?
Somebetablockers(atenolol,esmolol,nebivolol,andmetoprolol)aremoreselectiveforthe1
receptorsontheheart,withlesseffectonthe2receptorsthatleadtosmoothmusclerelaxationin
thebronchiolesandskeletalmusclearterioles.Thus,specific1antagonistshavelessaffinityto
inhibitcatecholaminemediatedbronchodilationandperipheralmusclevasodilation.

6 Cautionshouldbeusedinprescribingbetablockersforpatientswithwhich
comorbidconditionsandwhy?
Asthma:Riskofworseningbronchospasmbypreventing2mediatedbronchodilation.(Even
1selectiveantagonistshavesomeeffect,especiallyathighdoses.)Forthepurposeofthe
USMLE,rememberthat1selectiveblockersshouldbeusedinpatientswithpulmonary
disease.
COPD(chronicobstructivepulmonarydisease)patientsforsimilarreasons.
Peripheralarterialdisease:Exacerbationofclaudication.(Vasodilationinskeletalmuscle
arteriolesismediatedby2adrenergicreceptorspresentonvascularsmoothmusclecells.)
Firstdegreeatrioventricular(AV)block:BetablockersdecreaseAVconductionandthus
furtherlengthenthealreadyprolongedPRintervalontheelectrocardiogram(ECG).Thiscan
resultinexcessivecardiacdepression.(Betablockersshouldnotbegivenatalltopatientswith
secondorthirddegreeAVblockbecausetheycoulddelayAVconductionevenfurther.Certain
calciumchannelblockers,suchasverapamilanddiltiazem,mightalsoreduceAVconduction
and,whenusedincombinationwithbetablockers,canproduceaseriousAVblock.)
Diabetes:Betablockersreducethenormalsymptomsofhypoglycemia(e.g.,headache,
confusion,slurredspeech,anxiety,tremors,palpitations)thatprovidewarningtodiabetic
patientspriortoreachingdangerouslylowbloodsugarlevels.
Becauseoftheirabilitytoreducemyocardialoxygendemand,betablockerusehasbeen
showntohaveasurvivalbenefitinpatientswithcongestiveheartfailure.However,caution
shouldbeusedwhenplacingpatientsonbetablockers,astheymayinitiallyexacerbateCHF

symptoms(decompensatedheartfailure).
Erectiledysfunction:betablockersmayinhibitb2adrenergicreceptormediatedvasodilation.
Depression:Mechanismofactionisunclear,butpresumablyinvolvesalterationsin
monoamineneurotransmittersignaling.

Case11continued:
Afteranumberofmonths,thepatientreturnstotheofficecomplainingofanimpairedability
tokeepupwithhiswifeontheirdailyhikes.Hefeelsthathegetstiredmorequickly,attributes
thistothemetoprolol,andplanstoquittakinghismedication.

7 Whatmightbeoccurringinthispatientandhowwouldyourecommendhe
discontinuehismedication?
BetablockersarelikelyimpairinghisexercisetolerancebypreventingthenecessaryincreaseinHR
tomeethisoxygendemand.Tryinghimonanotherclassofantihypertensiveswouldbereasonable,
buthismetoprololshouldfirstbetaperedbecausechronicbetablockerusesubstantiallyincreases
thesensitivityofthehearttocatecholaminesbyupregulatingbetareceptorsoncardiomyocytes.
Consequently,suddenwithdrawalofthesedrugscanleadtotachycardia,arrhythmias,andacute
myocardialinfarction.Forthisreason,patientsshouldalwaysbegraduallyweanedoffbetablockers.

Essentialhypertensionisthemostcommontypeofhypertension.
Themostcommoncausesofsecondaryhypertensionarerenalarterystenosisand
primaryhyperaldosteronism.
Betablockerslowerbloodpressureinpartbyreducingreninsecretionfromthekidneys
(primarilya1receptormediatedeffect).
Betablockersdecreaseatrioventricular(AV)conductionandthereforeshouldbeused
cautiouslyinpatientswithfirstdegreeheartblock.
SummaryBox:Hypertension

Case12
A48yearoldmanisnewlydiagnosedwithbothtype2diabetesmellitusandhypertension.To
helpcontrolhisbloodsugar,heisstartedonmetformin.

1 Howwouldthefactthatthispatientalsohasdiabetesinfluencetreatmentfor
hishypertension?Considerdrugchoiceandaggressivenessoftreatment

Angiotensinconvertingenzyme(ACE)inhibitorsarethepreferredfirstlineantihypertensivesin
diabetics.ACEinhibitorshavebeenshowntoreducetheprogressionofproteinuria(andthe
subsequentnephropathy)indiabeticpatients.Althoughthemechanismbehindthenephroprotective
actionsofACEinhibitorsremainsunclear,itcouldberelatedtotheirabilitytopreferentiallydilate
theefferentarterioleandtherebyreduceglomerularfiltrationpressures.
Diabetesisacardiovascularriskfactorandpredisposespatientstobothmicrovascularand
macrovasculardamage.Tightbloodpressurecontroliscriticalindiabeticstoreducethemortality
riskfrommacrovasculardisease(CAD,CVA,PVD).Additionally,itreducestheriskofmicrovascular
complicationssuchasretinopathy,nephropathy,andneuropathy.

2 WhatarethemechanismsbywhichACEinhibitorslowerbloodpressure?
ThesedrugspreventtheconversionofangiotensinItoangiotensinII.AngiotensinIIcontributesto
peripheralvasoconstriction,stimulationofthirst,renalsodiumreabsorptionintheproximaltubule,
andaldosteroneproduction,allofwhichareantagonizedbyACEinhibitors(Fig.16(f0035)).

Figure16
Enzymaticcascadeinthereninangiotensinaldosteronesystem.Notediagrammaticrepresentationof
physiologicactionsofangiotensinII.ACE,angiotensinconvertingenzyme.
(FromBrownTA:RapidReviewPhysiology.Philadelphia,Mosby,2007,p148.)

3 Whymighttheuseofbetablockerstocontrolthisman'shypertensionnotbe
anidealchoice(atleastforthepurposeofboards)?
Thesedrugscouldmaskimportantsignsofhypoglycemia,suchastremorandpalpitations,thatare
mediatedthroughthesympatheticnervoussystem.Betablockersalsoantagonizeepinephrine
stimulatedhepaticgluconeogenesisandglycogenolysis,therebycontributingtohypoglycemia.
However,hypoglycemiaismuchmoreofaconcernwithdiabeticsonsecondgeneration
sulfonylureas,dipeptidylpeptidase(DPP4)inhibitors,orinsulin.

Note:Althoughbetablockerscouldmaskmanyofthesymptomsassociatedwithhypoglycemia,they
willnotpreventthediaphoresiscommonlyseeninseverehypoglycemia,becausesweatglandsare
innervatedbysympatheticcholinergicnervesratherthansympatheticadrenergicnerves.

4 Whateffectmightstartingathiazidediuretichaveonglycemiccontrolinthis
patient?
Diureticscommonlyworsenhyperglycemiaindiabetics.Althoughthemechanism(s)remain
uncertain,thehyperglycemiaislikelyrelatedtoacombinationofhypokalemiaandintravascular
volumedepletionthatoccurswithinitiationofdiuretictherapy.Hypokalemiacancontributeto
impairedglucosetolerancebecauseitinhibitsinsulinsecretion,stimulatesinsulinresistance,and
impairscellularglucoseuptake(potassiumisanecessarycotransporterforglucoseuptake).
TheintravascularvolumedepletionresultingfromdiuresisreducesCO,whichstimulatesthe
sympatheticnervoussystem,therebypromotinginsulinresistanceandreducedglucoseuptakeby
theliverandskeletalmuscle.
Whereasthisisseldomseenclinically,forthepurposeofboardsyoushouldnotethatthiazide
diureticsshouldbeusedcautiouslyindiabeticsbecausetheymaytheoreticallyincreaseriskof
hyperlipidemiaandhyperuricemia.

Case12continued:
Thepatientisstartedonlisinopril,whichheinitiallytolerateswell.Afteraboutamonthhe
developsanannoyingdrycoughandwondersifitissomehowrelatedtohisnewmedication.

5 WhatisthemechanismbywhichACEinhibitors,suchaslisinopril,cancause
acough?
Angiotensinconvertingenzyme(ACE)isalsoknownaskininaseitcandegradebradykinininthe
blood.AccumulationofbradykininwithACEinhibitorsisbelievedtobetheprimarycauseofthe
cough.Itisthoughtthattheaccumulatedbradykininsomehowstimulatesnociceptorsintheairways,
therebyinitiatingthecoughreflex.Thisundesirablesideeffectcanbeavoidedbyusingangiotensin
receptorblockers(ARBs)suchaslosartan.

6 CoverthecolumnsontherightsideofTable13(t0020),andforeach
antihypertensivedrugintheleftcolumn,trytonametheclassofdrug,
mechanismofaction,andprimarysideeffectsassociatedwithitsuse
ACEinhibitorsarefirstlinetherapyforhypertensionindiabeticsbecauseoftheir
nephroprotectiveactions.TheprimarysideeffectofACEinhibitorsiscough,whichis
likelyduetobradykininaccumulation.
Betablockersarerelativelycontraindicatedfortreatinghypertensionindiabetics

becauseoftheconcern(primarilytheoretical)forhypoglycemicunawareness.
Thiazidediureticscanexacerbateglycemiccontrolindiabetics,potentiallyrelatedto
theirpropensitytocausehypokalemiaandstimulatethesympatheticnervoussystemvia
intravascularvolumedepletion.
SummaryBox:PharmacotherapyforHypertension

Case13
A65yearoldmanpresentstoyourofficecomplainingofa6monthhistoryofchestpainwith
exertion,whichresolveswithrestand,ifneedbe,sublingualnitroglycerin.Thepainisa
substernalpressuresensationthatremainslocalizedanddoesnotradiatetothearms,
shoulder,orjaw.Hedeniesanyassociatednausea,vomiting,ordiaphoresisduringthese
episodesofchestpain.Theepisodesofchestpressureseemtoappearonlywithphysical
exertionandhavenorelationtomealsorswallowing.Physicalexaminationisunrevealing.

Table13
AntihypertensiveDrugs

Drug

DrugClass

Hydralazine

Arterialvasodilator Unknown

Lupuslikesyndrome
Reflextachycardia

Captopril
Lisinopril

ACEinhibitors

ATIIproduction
Peripheralresistance

Drycough,
Hyperkalemia,

Aldosteronesecretion

Angioedema(

Accupril

PrimaryMechanismofAction

Benazepril

PrimarySideEffects

bradykinin)
Acutekidneydisease
(GFR)

Losartan

ARBs

InhibitATIIactionintissues

Valsartan

Goodsideeffect
profile

Methyldopa

2Receptor

Clonidine

agonists

CNSsympatheticoutflow

Sedation/depression
Rebound
hypertension(on
withdrawalof
clonidine)

Atenolol

Selective1

Negativechronotropicand

Bradycardia/Heart

Esmolol

receptorblockers

inotropiceffects(reducecardiac

block

output)

Sexualdysfunction

Metoprolol

Nebivolol

Bronchospasm(at
highdoses)
Depression(athigh
doses)

Propranolol

Nonselectivebeta

Negativechronotropicand

Bronchospasm

blocker

inotropiceffects(reducecardiac

Bradycardia/Heart

output)

block
CHF
Exerciseintolerance

Prazosin

1Receptor

Terazosin

antagonists

Peripheralresistance

Posturalhypotension

Doxazosin
Labetolol

Combinedand

Combinedeffectof1blockers

Carvedilol

receptor
blockers

andbetablockers

Diltiazem

Calciumchannel

Cardiacoutput

Bradycardia/Heart

Verapamil

blockersnon

Peripheralsresistance(Note:

block

dihydropyridine

thiseffectisspecifictodiltiazem
seeTable233)

Amlodipine

Calciumchannel

Peripheralresistance(more

Nifedipine

blockers
dihydropyridine

specificforvascularsmooth
musclethanheart)

Hydrochlorothiazide Thiazidediuretic

Inhibitssodiumandwater

Reflextachycardia

Hypokalemia

reabsorptionindistalconvoluted Hyperuricemia
tubule

Hyperglycemia
Hyperlipidemia
Hypercalcemia

Furosemide
Bumetamide

Loopdiuretic

thickascendinglimbofloopof

Hypokalemia
Hyperuricemia

Henle

Hyperglycemia

InhibitsNa+Cl2K+pumpin

Hypocalcemia
Ototoxicityin
combinationwith
aminoglycosides
Spironolactone

Potassiumsparing Antagonizesactionof
diuretic

Hyperkalemia

aldosterone

ACE,angiotensinconvertingenzymeATII,angiotensinIIAV,atrioventricularCNS,centralnervoussystem

GFR,glomerularfiltrationrate.

1 Whatisthelikelydiagnosis?
Anginapectoris,chestpaincausedbyimpairedbloodflowtothemyocardium,isusuallydueto
stenosisofthecoronaryarteriesbyatheromatousplaques.Theresultingmyocardialischemia
promotesanaerobicrespirationandthereleaseofsubstancessuchaslacticacidthatcausecardiac
pain.
Note:Recallthatvisceralcardiacpainissensedbysympatheticfibersthattravelparalleltothe
coronaryvesselsandenterthespinalcordbetweenC8andT4.Bealertforsilentcardiacischemia
inpatientswithautonomicneuropathy(e.g.,diabetics)orpatientswhohavereceivedaheart
transplant(inwhichtheautonomicfibershavebeensevered).

Case13continued:
Thepatientstatesthathisepisodesofchestpressuregenerallylastaround5minutesbutnever
longerthan15minutes.Theyoftenoccurwhenheusesthepushmowertomowthelawnon
thesteeppartoftheyardorwhenherideshisbicycleupahill,butnotwhenheiswalking,
sitting,orsleeping.Hethinkstheseverityandpatternofchestpainhavestayedprettymuch
thesamesincehefirstnoticedit6monthsago.

2 Doesthispatienthavestableorunstableangina?Howaretheydifferent,and
whichismoreserious?
Hehasstableangina.Instableangina,aconstantandpredictablelevelofphysicalexertionelicitsthe
chestpainorsubsternaldiscomfort.Stableanginausuallylastsbetween2and5minutes,sometimes
over10minutes,butrarelyover15minutesorlessthan1minute.Itisalwayspromptlyrelievedby
rest.Incontrast,unstableanginaischaracterizedbythedevelopmentofchestpainatrestorafter
onlymildexertionandcanlastlongerthan15minutes.Unstableanginaismoreseriousbecauseit
indicatesadisruptionwithintheatheromatousplaque(s)inthecoronaryarteriesthatiscausing
ischemia.Itmayalsoinvolvetheformationofanonocclusivethrombus.Whenunstableangina
develops,thepatientshouldbeemergentlyevaluatedtoruleoutanimpendingmyocardialinfarction.

3 Howcananginaoccurintheabsenceofcoronaryatherosclerosis?
ThisanginaoccursinPrinzmetal's(variantorvasospastic)angina,inwhichtransmuralcardiac
ischemiaoccursduetovasospasmofthecoronaryarteries.YoushouldsuspectPrinzmetal'sangina
inapatientwhohasanginalsymptomsatrestorinthemorning.Prinzmetal'sanginaisthoughttobe
chemicallymediatedbyanincreasedplateletproductionofthromboxaneA2(TXA2)oranincreased
productionofendothelin1indamagedendothelialcells.Severeanemiacouldalsotheoretically
produceanginabecauseofreduceddeliveryofoxygentotheheartandanincreaseddemandonthe
hearttopumpmorebloodbecausetherearefewerredbloodcellsincirculation.Onrareoccasions,
patientswithpheochromocytomawithoutcoronaryarterydiseasecanexperienceseverecoronary

vasospasmresultingincardiacischemiaand,attimes,evenmyocardialinfarction.Lastly,ventricular
hypertrophymaycausesubendocardialischemiaandanginabecausetheincreaseinmyocardial
perfusionisusuallynotasgreatproportionallyasthedegreeofmyocardialhypertrophy.

4 Whataretheprincipalphysiologicdeterminantsofmyocardialoxygensupply?
Myocardialoxygensupplyisdependentoncoronaryarteryperfusionandthearterialoxygen
carryingcapacity.
Myocardialperfusionisprimarilydependentondiastolictime,diastolicperfusionpressure,
thicknessofthemyocardium,andthevascularresistanceofthecoronaryarteries.
Arterialoxygencarryingcapacityislargelydependentonthehemoglobinconcentrationand
theefficiencyofgasexchangeatthelungs(oxygensaturation).
Althoughsevereanemiaorhypoxemiacandecreasemyocardialoxygensupplysubstantially,supply
ismoreoftenlimitedasaresultofinadequatemyocardialperfusion.Thiscanresultfromtachycardia
(decreasedtimespentindiastole,theperiodwhenthemajorityofleftventricularbloodflowoccurs),
inadequatediastolicperfusionpressureresultingfromhypotension,dehydration,valvular
abnormalitiessuchasaorticregurgitation,oranincreasedleftventricularenddiastolicpressure
(LVEDP),whichoccursinconcentricandeccentrichypertrophy.

5 Whataretheprincipalphysiologicdeterminantsofmyocardialoxygen
demand?
Anyfactorthataffectsthehemodynamicsofthecardiovascularsystemcanaffectmyocardialoxygen
demand.Thus,oxygendemandisincreasedasHR,contractility,afterload,andsympatheticactivity
increase.Tocompensateforelevatedoxygendemand,theheartincreasescoronarybloodflow,which
inturnincreasesoxygendelivery.Recallthatoxygenextractionfromthecoronaryarteriesisvery
efficient(assumedtobe100%),sotheonlywaytoincreaseoxygendeliverytotheheartistoalter
coronarybloodflow.

6 Whichfactorscontributetoamyocardialoxygendemandthatexceedssupply
inthisman?
DemandisincreasedbythesympatheticmediatedincreaseinHRaswellascontractilityinresponse
toexercise.Atthesametime,supplyisdecreasedbecauseanelevatedHRgiveshisscleroticcoronary
arterieslesstimetoincreasebloodflowtothemyocardiumbyshorteningtheamountoftime
availablefordiastole.

7 Whyisnitroglycerineffectiveineliminatinganginalpain?
Nitroglycerinisconvertedwithinendothelialcellstonitricoxide,whichisalsoreferredtoas
endotheliumderivedrelaxationfactor(EDRF)becauseofitsvasodilatoryeffects.Perhapsthemost
importantreasonnitroglycerineliminatesanginalpainisitseffectonreducingmyocardialoxygen
demandsecondarytovenousdilation(decreasingpreload)andarteriolardilation(reducing
afterload).ForStep1,however,itisimportanttoknowthattheeffectofnitroglycerinismuchgreater

onreducingpreloadthanafterloadbecauseitdilatesveinsmorethanarteries.Inanycase,
decreasingpreloadandafterloadreducescardiaccontractilityandmyocardialwalltension,which
additionallyallowsgreatermyocardialperfusionduringsystole.Nitroglycerinandotherorganic
nitratesalsoexerteffectsdirectlyonthecoronaryvasculature,includingvasodilationofthecoronary
arteriesandreliefofcoronaryarteryspasm.Theprecisemechanismsbywhichnitratesreduce
symptomsofanginalpainthereforewilldependonwhichpathologicmechanismisresponsiblefor
theanginainagivenpatient(e.g.,atheroscleroticocclusion,vasospasm).
Note:Highdosesofnitratescanproducereflextachycardia,whichoccursinresponseto
hypotension,andthiscanfurtherexacerbateanginalpainbyincreasingmyocardialoxygendemand.
Becausenitratesrelaxbothvascularandnonvascularsmoothmuscle,theycanrelievethepainof
bothanginaandesophagealspasm,makingitdifficulttodistinguishbetweenthesetwoconditions
basedsolelyontheirresponsetonitrates.Thecompensatorytachycardiathatdevelopsbecauseof
nitroglycerinmediatedvasodilationcanbepreventedwithbetablockers.

8 Whatarethemechanismsbywhichbetablockersdecreasemyocardialoxygen
demandand,therefore,thesymptomsofangina?
Betablockersexertnegativechronotropicandinotropiceffectsontheheart,producingareductionin
afterload,whichreduceswallstressandmyocardialO2demand,andincreasedmyocardialperfusion
asaresultofincreasedtimespentindiastole,whenthemyocardiumislargelyperfused.Forthese
reasonsbetablockersarehelpfulinpreventinganginaandincertaincircumstancescanalsobeused
totreatangina.

Case13continued:
Anexercisestresstestisperformedtoevaluatethepossibilityofcoronaryarterydiseaseandto
assessthepatient'slevelofcardiopulmonaryfunction.Whileheisonthetreadmill,theECG
revealscardiacischemia(shownbySTsegmentdepression)whenhisheartrateandblood
pressurehavebothincreased~50%abovebaseline.

9 Howcouldasimilarevaluationbemadeinapatientinwhomanexercise
stressiscontraindicated(e.g.,orthopedicconditionorchroniclungdisease)?
Pharmacologicagentscanbeusedtosimulatethestressofexercise.Dobutamine(asynthetic
catecholamine)haspositiveinotropicandchronotropiceffectsontheheart,soitsadministrationis
physiologicallysimilartophysicalexertionbecausebothcauseanincreaseinmyocardialoxygen
demand.However,adenosine(apotentcoronaryvasodilatorwithashorthalflife)ismore
commonlyused,alongwithpositronemissiontomography(PET)scanningtoacquireanaccurate
imageofcoronaryperfusion.

Thethreemainformsofanginaarestable,unstable,andvariant.Distinguishingamong
thesepatternshasimportantimplicationsforprognosisandtreatment.

Animbalancebetweenmyocardialoxygensupplyanddemandunderliesthe
pathophysiologyofangina.
Nitroglycerin'sprimarymechanismofactionisdilationofperipheralveins,which
reducespreloadandthereforemyocardialoxygendemand.Italsohasdirecteffectsonthe
coronaryarteries,soitcanbeusefulforallthreeformsofangina.
SummaryBox:Angina

Case14
A52yearoldmanpresentstotheemergencydepartmentforevaluationofacrushing
substernalpressuresensationforthepasthour.Heisobeseanddiaphoretic.Thepainradiates
tohisleftarmandhisjaw.Heisconcernedbecauseheusuallygetschestpainonlyafter
exercising,whichistypicallyrelievedbyrestbutthispainbeganatrest.Additionally,hefeels
nauseated,whichisnotsomethinghenoticedduringpastepisodes.

1 Whatdisorderisontopofthedifferentialdiagnosislistatthispointandwhy?
Myocardialinfarction(MI),likelyduetounderlyingcoronaryarterydisease,istheprobable
diagnosis.Thetypicalpresentationinvolvescrushingsubsternalpainforlongerthan30minutesthat
isnotrelievedbynitroglycerin.Thisdiagnosisshouldbeconsideredimmediatelybecauseatimely
interventionisessentialtothepatient'soutcome.AcuteMIisaleadingcauseofdeathinadultsinthe
UnitedStatesandhasahighincidencerateinmenbetweentheagesof40and65.

2 Whataresomecommonriskfactorsformyocardialinfarctionand
cardiovasculardiseaseingeneral?
Malegenderunderage65,familyhistoryofprematureCAD,hypertension,diabetesmellitus,tobacco
use,dyslipidemia(highlowdensitylipoprotein[LDL],lowhighdensitylipoprotein[HDL],high
triglycerides),andincreasedCreactiveprotein(CRP)arejustafewoftheriskfactors.

Case14continued:
AnECGisperformedimmediatelyandrevealsSTsegmentelevationintwoconsecutiveleads.
Achestxraystudydoesnotshowmediastinalwideningorotherabnormalities.Thepatientis
administeredmorphine,oxygen,nitroglycerin,metoprolol,andaspirin.Theconsulting
cardiologistdecidesthatthepatientisasuitablecandidateforemergentangioplasty,andheis
takentothecardiaccatheterizationlaboratory.

3 Whatkindofmyocardialinfarctionisthispatientexperiencing?

HeisexperiencinganSTsegmentelevationmyocardialinfarction(STEMI),thepathophysiologyof
whichclassicallyinvolvescompleteocclusionofacoronaryarteryresultingintransmuraltissue
infarction.Studieshaveshownthatthesepatientsarebesttreatedemergentlybyangioplastyifthisis
available.Ontheotherhand,subendocardialMIs(affectonlyaportionoftheventricularwall
thickness,)lackSTsegmentelevation,andarethusreferredtoasnonSTEMI(NSTEMI).The
subendocardiumisparticularlyvulnerabletoischemiaowingtoexposuretohighpressuresanda
relativelackofcollateralcirculation.NSTEMIsaretypicallycausedbyaneartotalocclusionofa
coronaryartery.PatientswithNSTEMIsalsohaveelevationsincardiacenzymesduetomyocardial
damage,andthesepatientsmayalsobenefitfromcardiaccatheterization.

4 Inadditiontoanalysisoftheelectrocardiogram,whatserumtestscouldbe
orderedtoconfirmorruleoutamyocardialinfarction?
WhenapatientpresentstotheemergencydepartmentwithanMI,anECGisthegoldstandardto
determinewhethertosendthepatienttothecatheterizationlaboratoryoradministerthrombolytics
whenacatheterizationlaboratoryisnotreadilyaccessible.Levelsofcardiacenzymes,whichhave
leakedoutofthenecroticmyocardialtissue,maybemoresensitiveindicatorsofMIbutmaynot
elevatetodiagnosticlevelsuntilseveralhoursaftertheonsetofchestpain.Theseenzymesinclude
troponinI(TnI),creatinekinaseMBfraction(CKMB),aspartatetransaminase(AST),and
myoglobin,witheachhavinguniquespecificitiesanddifferenttimesforpeakelevation.Myoglobin
risessoonafteronsetofcardiacpain.Althoughitisaveryearlymarker,itisnonspecificforMI.
TroponinIrisesafter4hoursandisthemostspecificamongthemarkersitalsoremainselevated
forthelongestperiodoftime(710days).CKMBhasgoodspecificityandbeginstorisewith
troponin.Itdoesnotremainelevatednearlyaslongastroponinsdo,butthispatterncanbeclinically
usefulindiagnosingarecurrentMIifCKMBbecomeselevatedagainafter72hours.ASThaspoor
specificityandpeaksthedayafteranMI.
Becausetheseenzymesremainelevatedforsometime,theymaybemoreusefulthananECG(which
providesonlyasnapshotoftheheartforamomentintime)insomeonewhodescribessymptomsof
anMIthatoccurredinthepastfewdays(Fig.17(f0040)).

Case14continued:
Inthecardiaccatheterizationlaboratory,thepatient'sangiogramshowsa95%narrowingofthe
leftanteriordescendingcoronaryartery.Angioplastyisperformed,andabaremetalstentis
successfullyplaced,restoringpatencytothevessel.After6daysinthehospital,thepatientis
dischargedwithprescriptionsforlisinopril,lovastatin,clopidogrel,andmetoprololinaddition
tothedailyaspirinandasneedednitroglycerinhewasalreadytaking.

Figure17
Myoglobin,creatinekinaseMBfraction(CKMB),andcardiactroponinincreaseafteramyocardialinfarction(MI).
CKMBpeaksafterabout16to24hoursandremainselevatedforapproximately72hours.Troponinpeaks
around24hoursafteranMIandremainselevatedfor7to10days.
(FromHenryJB:ClinicalDiagnosisandManagementbyLaboratoryMethods,20thed.Philadelphia,WB
Saunders,2001,p297.)

5 Whatothertreatmentoptionisavailablebesidesangioplastytorestore
coronarybloodflow?Whatarethemajorcontraindicationstoitsuse?
Thrombolytictherapywithtissueplasminogenactivator(tPA)orstreptokinase,whichmustbe
performedwithinacertaintimeframetobeeffective.Thrombolytictherapyiscontraindicatedin
patientsathighriskforhemorrhage(e.g.,recentmajorsurgery,bleedingdisorder,anticoagulantuse,
severehypertension,recenthemorrhagiccerebrovascularaccident[CVA]).
Note:Bleedingcausedbythrombolytictherapycanbetreatedwithaminocaproicacid,which
inhibitstheactivationofplasminogen.
KnowingtheantidotesforcommondrugoverdoseswillbeusefulforStep1.
Step1Secret

6 Whatisthephysiologicrationaleforgivingbetablockerstopatientswhohave
hadheartattacks?
Asinangina,thesedrugslowermyocardialoxygendemand,makingtheheartlesssusceptibleto
infarction.Additionally,becauseinfarctioncreatesanareaoffibrosisthatcanproducean
arrhythmia,betablockersarealsousefulfortheirantiarrhythmicproperties.However,ifthe
infarctioninvolvesthetissuesoftheconductionpathwayandresultsinheartblock,betablockeruse
maybecontraindicated.Overall,betablockershavebeenshowntoreducemortalitywhengivenafter
anMI.

7 Howmightamyocardialinfarctionresultinthefollowingshortandlong
termabnormalities?
A.Pulmonaryedema?
Pulmonaryedemacanresultintwoways:(1)Theweakenedleftventricle(LV)mightnolongerbe
abletoeffectivelypumpbloodintotheaorta,resultinginareducedejectionfraction.Thisistermed
systolicdysfunction.(2)Theleftventriclecouldstillpumpeffectivelybutmightbecomestiff

(noncompliant)asaresultoftheinfarct(andongoingischemia),thusnecessitatingincreased
ventricularfillingpressurestoachieveanadequateenddiastolicvolume.Thisistermeddiastolic
dysfunction.Regardlessoftheprecisecause(LVpumpfailureornoncompliance),theincreasedleft
ventricularfillingpressuresresultinincreasedleftatrialandpulmonaryhydrostaticcapillary
pressures,ultimatelyleadingtotransudationoffluidintothepulmonaryinterstitiumandalveolar
space.
Initially,thisinterstitialfluidiscompletelyremovedbythepulmonarylymphatics.However,when
thepulmonarycapillaryhydrostaticpressureincreasestoomuch(typically>30mmHg),theability
ofthelymphaticstoremoveexcessfluidisovercome,andinterstitialfluidandintraalveolarfluid
accumulate,resultinginpulmonaryedema.Pulmonaryedemareducesoxygendiffusionacrossthe
pulmonarymembrane,causinghypoxemia,whichfurtherexacerbatesthefailingheart.Thisisone
reasonwhyoxygentherapymaybebeneficialafterasevereMI.
B.Arrhythmias?
Leakageofelectrolytesfromthenecroticmyocardialcellsresultsinelectrolyteimbalancesthatcan
instigatearrhythmias.ThesearrhythmiasaremostcommonwithinafewdaysaftertheMI.
C.Murmur?
IftheMIcausesruptureoftheventricularseptumorapapillarymuscle,thentherespective
murmursofaventricularseptaldefect(VSD)orvalvularregurgitationwouldbeheard(bothofwhich
areholosystolicmurmurs).
D.Ventricularrupture?
Theinflammatoryreactionintheinfarctedsiteweakensthemyocardialwall.Ventricularrupture
typicallyoccursbetween3and7daysaftertheinfarctandcanresultinhemopericardium.
E.Pericarditis?
Theinflammatoryreactiontonecroticmyocardiumcanalsocausepericarditis(usuallywithinthe
firstweekaftertheMI).Suspectthisifthepatienthasapericardialfrictionrub.Dressler'ssyndrome
isatypeofpericarditisthatdevelops1to10weeksafteranMIandispresumedtoresultfroman
autoimmuneprocessthatoccurssecondarytoleakageofintracellularproteinsfromnecrotic
myocardialcells.Considerthisdiagnosiswheneveryouseeapatientwhopresentswithpleuritic
chestpainandsymptomsofpericardialeffusionseveralweeksafterMI.Patientswiththis
autoimmunepericarditisalsotypicallyexperiencefever.

8 Whatissuddencardiacdeath,andwhyarepatientswhohavehadmyocardial
infarctionspredisposedtoit?

Suddencardiacdeathisdefinedasdeathwithin1hourofonsetofsymptoms(usuallyduetoalethal
arrhythmia).Thisismostcommonlyassociatedwithnonocclusiveclots,whichusuallycause
subendocardialinfarcts.SubendocardialinfarctsareoftenreferredtoasnonQwaveMIsor
NSTEMIs.

Majormodifiablecoronaryriskfactorsincludehypertension,smoking,and
dyslipidemia,andmajornonmodifiableriskfactorsincludeage,malegenderifunderage
65,andfamilyhistoryofprematureheartdisease.ElevatedCreactiveprotein(CRP)and
homocysteinuriaareemergingasotherimportantriskfactors.
CardiacmuscleenzymessuchascreatinekinaseMBfraction(CKMB)andcardiac
specifictroponinsaresensitiveindicatorsofmyocardialinfarction(MI)dependingon
whentheirbloodconcentrationsaremeasured.
PostMIpatientsareatincreasedriskforanotherMIaswellasanumberofother
complications,includingpulmonaryedema,ischemicmyocardialrupture,pericarditis,
arrhythmias,andsuddencardiacdeath.
SummaryBox:CoronaryArteryDisease

Case15
A72yearoldwomanwithalonghistoryofpoorlycontrolledhypertensionanddiabetes
presentswitha1monthhistoryofworseningfatigueandshortnessofbreath(dyspnea).
Initially,sheexperienceddifficultybreathingonlywithexertion,butrecentlyitoccursevenat
rest.Sheadmitstosupportingherselfwithtwopillowsatnighttohelpwithbreathing(two
pilloworthopnea).

1 Whatdisorderisatthetopofyourdifferentialdiagnosislist?
Thesesymptomsareaclassicpresentationofheartfailure,butcausessuchasmyocardialischemia,
lungdisease,anemia,andatrialfibrillationshouldbeconsidered.Diagnosisofheartfailureislargely
basedonacarefulhistoryandphysicalexaminationandsupportedbyteststhatassesscardiac
function.

Case15continued:
Onexaminationshehasdistendedneckveins,bibasilarpulmonarycrackles,andbilaterallower
extremityedema.Herapicalimpulseisdisplacedlaterallypastthemidclavicularline,andanS
3gallopisappreciatedonauscultation.Onchestxray,thecardiacsilhouetteappearsslightly

enlarged,andanechocardiogramrevealsanejectionfractionof38%andnovalvular
abnormalities.Plasmalevelsofbrainnatriureticpeptide(BNP)aresubstantiallyelevated.

2 Inpathophysiologicterms,whatisheartfailure?
Heartfailureresultsfromeither(1)pathologicallydepressedCOor(2)normalCOthatcanonlybe
maintainedatelevatedventricularfillingpressures,whichpathologicallyincreasesvenous
hydrostaticpressures.Itthereforefollowsthattherearetwomajorcategoriesofsymptomsseenin
heartfailure:thoseduetodepressedCOandthoseduetofluidaccumulationcausedbyincreased
fillingpressures.Heartfailureisalsocategorizedaccordingtothesideoftheheartthathasfailedto
function.Leftsidedheartfailureresultsinadecreaseintheinotropicabilityofthehearttopumpthe
necessaryamountofbloodtotherestofthebody.Itcanbecompensatedforbytherenin
angiotensinaldosteronesystem(RAAS),whichwillincreaseintravascularvolume,venousreturn,
andcontractilitytomaintainCO,oritcanbedecompensatedwithworseningheartfailureandresult
inadecreasedCO.Symptomssuchasfatigue,lethargy,andweaknessareduetoinadequateCOand
areworseonexertion.
Whethertheheartfailureiscompensatedordecompensated,ventricularpressuresbecomeelevated,
andvaryingdegreesofpulmonaryedemaensue,leadingtodyspnea.Ifleftsidedheartfailureisnot
treated,thepulmonarypressureswillremainelevatedandcancausetherightsideofthehearttofail
thisiscalledbiventricularfailure.Thisleadstoasimilarbackupofbloodinthevenouscirculation,
resultinginjugularvenousdistention,hepatomegaly,andpittingedema.
Heartfailurethatmanifestswiththesymptomslistedpreviouslyiscalledcongestiveheartfailure
(CHF).Noticethatthiswomanhassignsofbothleftsidedandrightsidedheartfailure.
Note:Themostcommoncauseofrightsidedheartfailureisleftsidedheartfailure.Otherless
commoncausesofrightsidedheartfailureincludepulmonaryhypertension,tricuspidregurgitation,
pulmonarystenosis,andseptaldefects.

3 Whatarethedifferencesbetweensystolicanddiastolicheartfailure?Which
doesthiswomanmostlikelyhave?
Heartfailurecanbebroadlyclassifiedassystolic(pump)failureordiastolic(filling)failure.Systolic
heartfailureischaracterizedbyinsufficientcontractilityoftheventricles,withanejectionfraction
below40%.Diastolicheartfailureischaracterizedbypoorventricularcompliance,resultingin
insufficientfillingofventriclesduringdiastolehencediastolicfailureisalsoknownasheartfailure
withapreservedejectionfraction.Itisestimatedthatapproximatelytwothirdsofpatientswith
heartfailurehavesystolicfailureandtheremainingonethirdhavediastolicfailure.However,
becausemostpatientswithsystolicdysfunctionhavecomponentsofdiastolicdysfunctionaswell,
thisclassificationschemeischaracterizedbysubstantialoverlap.
ThispatientlikelyhassystolicheartfailurebecauseofthereducedejectionfractionandS3heart
sound.Althoughhypertensionisaprimarycauseofdiastolicheartfailure,itisalsoamajorrisk
factorforcoronaryarterydisease,whichmaybetheunderlyingcauseofhersystolicfailure.

Note:Theejectionfractionisdefinedasstrokevolumedividedbyenddiastolicvolume(SV/EDV)
andisnormally55%to75%.

4 Whatistheetiologyofheartfailure?
Systolicheartfailureisassociatedwithmyocardialdamageorischemiaaswellasvolumeoverloaded
statessuchasvalvularregurgitationandkidneydisease.Diastolicheartfailureisusuallyassociated
withpressureoverload(hypertension)andmyocardialischemia,butmayalsobecausedby
infiltrativediseasessuchasamyloidosis.
Note:Theendresultofcoronaryheartdisease(ifnotproperlymanaged)isanischemic
cardiomyopathywithcomponentsofbothsystolicanddiastolicfailure,becausebothsystoleand
diastoleareenergyrequiringprocesses.

5 Thebody'sresponsetoheartfailureisinitiallyhelpfulbutbecomes
maladaptivewithtime.Foreachofthefollowingphysiologicresponses
observedinheartfailure,describeboththeadaptiveandpathologicresults
A.Increasedsympatheticactivity
Adaptive:Increasedsympatheticoutflowresultsintachycardiaandincreasedcontractilityofthe
heart,bothofwhichincreaseCO.Interestingly,patientsinheartfailuresecretethreetofourtimes
morenorepinephrineadaythannormal,healthyindividuals.Recallthatthedeterminantsofcardiac
outputaregivenbytheequation
CO = HR SV

Additionally,inthesettingofreducedCO(suchassystolicheartfailure),vasoconstrictioncausedby
elevatedsympatheticoutflowhelpsmaintainsufficientarterialpressuretoprovideadequate
perfusiontocriticalorgans.Remember,thedeterminantsofarterialpressureareCOandTPR:
MAP = CO TPR

Pathologic:Thesympathoadrenalactivationseeninthecontextofafailingheartresultsinareduced
amountoftimespentindiastole.Thisdecreasesthetimeavailableforcoronaryperfusion(supply),
inasettingwheretheloadontheheart(demand)isalreadybeingincreasedbysympatheticactivity.
Aftertime,theincreasedloadresultsincardiacremodelingandworseningcardiacfunctionandis
exacerbatedbyadecreasedbloodsupply.Additionally,thesympatheticallymediatedchronic
vasoconstrictioninskeletalmusclesthatoccursduringheartfailureandthedecreasingCOare
largelyresponsibleforthemusclefatigueobservedintheseindividuals.
B.Fluidretention
Adaptive:ThekidneyssensereducedCOthroughdecreasesinrenalperfusionandglomerular
filtrationrate(GFR).Theyrespondbyactivatingthereninangiotensinaldosteronesystemtoretain
fluidandexpandtheplasmavolume.Thiselevationinintravascularvolumeincreasesvenousreturn

totheheartandsubsequentlyincreasespreload.Thisresponsehasapositiveinotropiceffectonthe
heartviatheFrankStarlingrelationshipandwillincreaseCO.
Pathologic:Theincreasedpreloadfromfluidretentionplacesanincreasedworkloadontheheart.
Thiscanprecipitatesymptomsofanginasecondarytoinsufficientcoronaryperfusion.Excessive
preloadsstretchthemyocardiumtoapointofsuboptimaloverlapofactinandmyosinfilamentsin
thesarcomeres,reducingcontractility.Finally,fluidretentioncanalsocausecomplications
associatedwithexcessivevolumeexpansion,suchaspulmonaryedema.
Note:Becausenitratesanddiureticsbothdecreasepreload(aswellasafterloadtosomedegree),
theyhelpalleviatethesymptomsofCHFassociatedwithexcessivevolumeexpansion.
C.Myocardialhypertrophy
Adaptive:Thevalueofthisprocessdependsonthetypeofoverloadthatoccursinheartfailure.Ina
pressureoverloadedheart(fromhypertensionoraorticstenosis),thereisconcentrichypertrophy
(circularthickeningofthemyocardium)thatstrengthensventricularcontractionsinthesettingofa
significantafterload.Inpressureoverloadedventricles,theincreasedsystolicwallstresscauses
additionofsarcomeresinparallel,whichreducesthestressoneachsarcomereaccordingtotheLaw
ofLaplace(wallstress=pressureXradius/thickness).Inavolumeoverloadedheart,theincreasein
diastolicwallstressfromincreasedenddiastolicvolumecausespooralignmentofsarcomerefibrils
(pasttheadaptivepointoftheFrankStarlingrelationship),sosarcomeresareaddedinseriesto
expandthechambervolumeandoptimizefiberalignment.
Pathologic:Oxygendemandofthehypertrophiedheartisincreased,whichmightexacerbatean
existingischemiccondition.Infact,thevascularsupplytotheheartoftendoesnotincrease
proportionatelytothemuscularhypertrophy.Additionally,thethickenedmyocardiumrequiresa
largerdistanceforoxygentodiffuse,whichisalreadyexacerbatedbytheelevatedventricular
diastolicpressuresofheartfailure.Thisreducesthegradientforoxygendiffusionfromthecoronary
arteriesthroughthemyocardium(seeCase13(st0125),question4).Hypertrophyalsoreduces
ventricularcompliance,whichcancauseorworsendiastolicdysfunction.Thesympatheticnervous
systemandangiotensinIIareinvolvedinmediatingtheventricularremodelingfoundinhypertrophy
aswellasinventriculardilation.
Note:IfanadequateCOisrestoredbythesecompensatorymechanisms,theheartfailureissaidto
becompensated.IfthesephysiologicreflexesalonecannotrestoreadequateCO,theheartfailureis
saidtobedecompensated.

6 Giventhepathophysiologicadaptationsinheartfailurejustnoted,whymight
betablockersbebeneficialinheartfailure?
Betablockershavemanyeffectsthatadditivelydecreasetheoverallcardiacworkloadandimprove
functionofanailingheart:
1.Inhibitsympatheticactivity

Decreasepreloadbypreventingsympatheticmediatedvenoconstriction
Decreasecontractilityandheartrate
Decreaseafterload
2.Decreasereninsecretion
Decreasefluidretentionandafterload
3.Decreasecardiacremodeling

7 Howisdigitalis,whichisusedinheartfailure,believedtoincreasecardiac
contractility?
Similartoskeletalmusclefibers,cardiacmusclefiberscontractwhentheintracellularcalciumlevels
rise.Digitalisincreasestheintracellularcalciumbyanindirectmechanisminvolvingionexchanges.
Byinhibitingthesodium/potassiumpump,digitalisincreasesintracellularsodium.However,itisthe
extracellular/intracellularsodiumgradientthatdrivesthesodium/calciumexchanger.Consequently,
inthepresenceofhighintracellularsodiumlesscalciumispumpedoutofthecell,increasing
intracellularcalciumandcontractility.
Note:IncontrasttodrugssuchasACEinhibitorsandbetablockers,whichhavebeenshownto
extendlife,digitalishasbeenshowntoimprovecardiacperformanceandqualityoflifewithoutan
improvementinmortalityrisk.Itisnotafirstlineagentinthetreatmentofheartfailure.
DigitalisisahighyielddrugforStep1.Inadditiontoitsmechanismofaction,itisimportantto
knowcommonsideeffectsofdigitalis.Theseeffectsincludecholinergiceffects(e.g.,diarrhea,
vomiting,increasedPRinterval),arrhythmias,andblurryyellowvision.Digitalis/digoxin
toxicityistreatedbystoppingthemedicationandadministeringpotassium,magnesium,and
antidigoxinFabfragments.Lidocaineisgivenfordigoxininducedarrhythmias.
Step1Secret

8 InTable14(t0025),covertherighthandcolumnandattempttogivethe
mechanismofactionforeachofthelisteddrugsusedinCHF
Table14
DrugsUsedforCongestiveHeartFailure

Drug

DrugClass

MechanismofAction

Digitalis

Cardiacglycoside

inotropiceffect,chronotropiceffect,ejectionfraction

Metoprolol Betablocker

chronotropiceffect,inotropiceffect,myocardialdemand

Captopril

aldosterone,plasmavolume,actionsofATII

ACEinhibitor

Losartan

ATIIreceptorantagonist InhibitsactionsofATII

ACE,angiotensinconvertingenzymeATII,angiotensinII.

OtherRelatedQuestions
9 Whatishighoutputheartfailure?
Highoutputheartfailurereferstotheinabilityofthehearttomaintainanelevatedcardiacoutput
inpathologicsituationsthatdemandit,forexample,inhyperthyroidism,arteriovenous
malformations,anemia,andsepsis.
Note:Inthecaseofarteriovenousmalformations,thedrasticdropinTPRwhengoingfromahigh
pressurearterioletoalowpressurevenuledemandsanincreaseinCOtomaintainMAP.Thehigh
COcausesanincreasedloadontheheart,whichwhenmaintainedforalongtimecancausethe
functionofthehearttoeventuallydeteriorate.

Heartfailureisacomplexclinicalsyndromethatrepresentsafinalcommonpathwayfor
avarietyofpathologicprocessesthatimpaircardiacfunction.
Heartfailureisclassifiedbytwogeneraltypes:systolic(pump)anddiastolic(filling)
dysfunction,withconsiderableoverlap.
Theinitiallyadaptivephysiologicresponses(increasedsympatheticactivity,fluid
retention,andmyocardialhypertrophy)becomemaladaptivewhenprolonged,leadingto
progressivedeteriorationofcardiacfunctionandeventualdeath.
Whenthinkingaboutheartfailure,categorizethefindingsaccordingtowhetherthey
suggestleftsidedversusrightsidedheartfailure,preservedversusreducedejection
fraction,andcompensatedversusdecompensatedcardiacoutput.
SummaryBox:HeartFailure

Case16
A50yearoldmanpresentscomplainingofchestpainthatoccursatgraduallydiminishing
levelsofphysicalexertion,aswellastworecentepisodesofsyncopewhilegolfing.
Cardiovascularexaminationrevealsabloodpressureof120/90mmHg,aloudcrescendo
decrescendosystolicmurmurbestappreciatedattheupperrightsternalborder(withradiation
tobothcarotidarteries),andaweakanddelayedcarotidupstroke.AnECGrevealsleft
ventricularhypertrophy,andanechocardiogramrevealsabicuspidaorticvalvewithreduced
valvularorifice(<1cm2).

1 Whatisthediagnosis?
Aorticstenosis,acommonvalvulardisorderinwhichexcessivenarrowingoftheaorticvalve
increasesafterload,isthelikelydiagnosis.Ifleftuntreated,aorticstenosismayresultinangina,
exertionalsyncope,dyspneafromheartfailure,andincreasedcardiovascularmortality.Symptoms
areoften,butnotalways,seenwhentheareaoftheaorticorificeislessthan1cm2(normalareais3
cm2).

2 Whatlikelypredisposedthispatienttodevelopingaorticstenosis?
Stenosisgenerallyoccursonlyinelderlypatientssecondarytocalcification,whichisreferredtoas
senilecalcificaorticstenosis.However,congenitallybicuspidorevenunicuspidvalves(asopposedto
thenormaltricuspidaorticvalve)calcifyandnarrowatanearlierage,usuallyinthelate40sorinthe
50s,ashappenedwiththisman.

3 Whatcausesheartmurmursandwhydoesthispatienthaveone?
Murmursarecausedbyturbulentflow,whichoccursatelevatedflowvelocities.Inthiscase,the
stenoticaorticvalveforcesthehearttocontractmoreforcefully,whichgeneratesasignificant
pressuregradientbetweentheleftventricleandtheaorta,creatinghighflowvelocitiesacrossthe
aorticvalve.Themurmurofaorticstenosisdecreasesinintensitywhenpreloadisdecreased,suchas
whatoccurswhenstandingstill.Themurmurincreasesinintensitywhenpreloadisincreased,which
occurswithinspirationandValsalvamaneuver,forexample.
Note:Carotidbruitsareduetothesamemechanismasmurmurs,withthestenoticlumencausing
increasedflowvelocitiesandaresultingturbulentbloodflowthatcanbeauscultated.

4 Whatcompensatoryleftventricularchangesoccurasaresultofaortic
stenosis?
Cardiomyocytesrespondtopressureandvolumeoverloadstressorsdifferently.Inresponseto
volumeoverload(e.g.,aorticregurgitation),sarcomereswithinmyocytesareaddedinseries,which
hastheeffectofincreasingventricularlumenvolumeanadaptiveresponsetermedeccentric
hypertrophy.Aorticstenosis,however,ischaracterizedbyapathologicallyelevatedafterload,and
thusleadstopathophysiologicchangesthatresemblethoseobservedwithsystemichypertension.
Themyocardiumrespondstoanincreasedafterloadbyaddingsarcomeresinparallel,resultingina
hypertrophiedmyocardiumthatisbetterabletoejectbloodagainstincreasedresistance.This
adaptiveresponseistermedconcentrichypertrophy.Oneofthedrawbacksofbothformsof
myocardialhypertrophyisthatthethickenedmyocardiumistypicallylesscompliant,requiring
increasedfillingpressuresandpredisposingtodyspneaandpulmonaryedema.

5 Whydoesthispatienthaveaweak,delayedcarotidupstrokewithanarrowed
pulsepressureonphysicalexamination?Thinkabouthowtheleftventricular
pressurevolumeloopchangesinaorticstenosis

Inaorticstenosis,asignificantproportionofcardiacworkisdevotedtogeneratingsufficientforceto
overcomethevalvularresistance.Consequently,asmallerproportionofcardiacworkisusedtoeject
blood,resultingindecreasedstrokevolume(SV)andpulsepressure.Thisisalsoaffectedbythe
amountoftimeavailableforejectionbecausemoretime,ofthefiniteamountavailableforsystole,is
consumedbyisovolumiccontraction.Additionally,ventricularhypertrophycanimpairdiastolic
filling,reducingpreload.RecallthatelevatedpreloadnormallycompensatesfordecreasedSVvia
theFrankStarlingmechanism.
ALVpressurevolumeloopcomparingchangesinLVpressureandvolumethroughoutthecardiac
cycleinanormalheartandonewithaorticstenosisisshowninFigure18(f0045).Notethatinaortic
stenosis,higherpressuresmustbegeneratedduringisovolumiccontraction(phaseII)becauseofthe
increasedafterload.Thisleavescorrespondinglylessenergyandtimeavailablefortheejectionphase
(phaseIII),andthereforetheSVinaorticstenosis(uncompensated)isreduced.ThisreducedSV
explainstheweakanddelayedcarotidupstroke(pulsusparvusettardus)andthedecreasedpulse
pressureevidentonphysicalexamination.
Youshouldpracticeinterpretingdiagramsdepictingpressurevolumeloopsinrelationtothe
eventsinthecardiaccycle.
Step1Secret

Figure18
Pressurevolumechangesinaorticstenosis.SV,strokevolume.
(FromBrownTA:RapidReviewPhysiology.Philadelphia,Mosby,2007,p128.)

6 Asaresultofaorticstenosis,doventricularmyocytesspendmoretimein
isotonicorisometric(isovolumic)contraction?
Myocytesspendmoretimeinisometric(isovolumic)contractiontoovercometheincreasedafterload
causedbythestenoticaorticvalve.Becausethetimeavailableforelectricalandmechanical
ventricularsystoleisfinite,thisresultsinashortenedisotonicejectionphase.

7 Explainthecauseofthispatient'spresentingcomplaints.Whatismostlikely
causinghisepisodesofsyncope?Hischestpain?

Patientswithstenoticaorticvalveshaveleftventricularoutflowobstructionthatlimitstheirabilityto
augmentcardiacoutput.Thisparticularlyoccursinthesettingofexercise,whenwidespread
peripheralvasodilationnecessitatesincreasedcardiacoutput,resultinginrelativecerebral
hypoperfusionandsymptomsofsyncope.Thus,thesyncopeexperiencedbythispatientwhilegolfing
resultsfromhisinabilitytomeettherequiredincreaseincardiacoutput.Hischestpainisduetothe
accumulationofbyproductsofanaerobicrespirationincardiomyocytes.Anaerobicmetabolism
occursandisduetothemyocardialischemiaresultingfromtheincreasedmyocardialdemand
causedbytheworkrequiredtopumpagainstastenoticvalveaswellasadecreasedcoronarysupply
secondarytoareducedSV.

8 Whataresomecausesofincreasedmyocardialoxygendemandinaortic
stenosis?
Theincreasedleftventricularmassrequiresmoreoxygenfornormalcontractilefunction.In
addition,theincreasedleftventricularpressuresthatdeveloptoovercometheoutflowobstruction
increasetheworkloadontheleftventricle,furtherincreasingthemyocardialvolumeofoxygen(MV
o2)demand.Additionaltimespentinsystoletoejectthebloodalsoincreasesthemyocardialoxygen
demand.Finally,inadequateCOinaorticstenosisactivatesthesympatheticnervoussystemand
increasescontractility,whichfurtherelevatesmyocardialoxygendemand.

9 Whataresomecausesofdecreasedmyocardialoxygensupplyinaortic
stenosis?
AsheartrateincreasestocompensateforadepressedCO,lesstimeisspentindiastole(whichis
whenthemajorityofcoronaryperfusionoccurs).Additionally,owingtodecreasedcompliance,left
ventriculardiastolicpressureincreases,whichfurtherreducesperfusion.Finally,aorticpressureis
reducedbecauseofthedecreasedSV,whichdecreasescoronarybloodflow(Fig.19(f0050)).

Figure19
Causesofincreasedmyocardialoxygendemandanddecreasedmyocardialoxygensupply.Ao,aorticLV,left

ventricularLVET,LVejectiontime.
(FromBoudoulasH,GravanisMB:CardiovascularDisorders:PathogenesisandPathophysiology.St.Louis,
Mosby,1993,p64.)

10 Whyisatrialfibrillationaparticularlydangerouscomplicationinaortic
stenosis(asidefromtheriskofembolicstroke)?
Aorticstenosisoftenresultsinelevatedleftatrialpressuresandsubsequentdilationoftheleft
atrium.Dilationoftheleftatriumcompressestheconductingfibers,therebypredisposingpatientsto
atrialfibrillation.
Normallytheatrialkick,theadditionalventricularfillingthatatrialcontractionnormallyprovides,
doesnotprovideasignificantpercentageofthecardiacoutput(CO).However,insituationsinwhich
theCOisimpaired,suchasaorticstenosis,thisextrafillingisessentialtosustainanadequateCO.
BecausehisCOisalreadycompromised,atrialfibrillationwouldremovetheatrialkick'scontribution
toCOandcouldprecipitatesevereheartfailure.

11 Describethemurmurofaorticstenosis
Aorticstenosisischaracterizedbyasystoliccrescendodecrescendoejectionmurmurbest
appreciatedattheupperrightsternalborder.Itmayradiatetothecarotidarteries.
Themurmurofaorticstenosisisenhancedbyincreasedpreload(squatting,passivelegraising)and
expiration.Asageneralruleofthumb,leftsidedmurmursincreaseonexhalationandrightsided
murmursincreaseoninspiration.
Ifaorticstenosisresultsintheformationofanoncompliantventricle,anS4heartsoundmaybe
heard.AtrialfibrillationwillpreventtheS4heartsoundfromdeveloping.
MurmurcharacteristicsareafavoritetopicforStep1.Besurethatyoucandistinguish
murmursaccordingtolocation,characteristicsymptoms,andmaneuversusedtoaltertheir
sounds.Remembertopracticelisteningtoaudiofilesofthemosttestablemurmurs.These
murmursincludethoseaccompanyingaortic/pulmonarystenosis,aorticregurgitation,
mitral/tricuspidregurgitation,mitral/tricuspidregurgitation,mitralvalveprolapse,
hypertrophiccardiomyopathy,ventricularseptaldefects,andpatentductusarteriosus.
Step1Secret

Aorticstenosismostcommonlypresentsasacrescendodecrescendosystolicmurmur
heardloudestattheupperrightsternalborderandradiatingtobothcarotidarteries.A
delayedcarotidupstrokeandnarrowedpulsepressureareassociatedfindings.
Decreasedcardiacoutput(indecompensatedstates)andincreasedmyocardialoxygen

demandareimportantconsequencesofaorticstenosis.Itfollowsthatthenaturalhistory
ofthisconditionleadstoangina,syncope,heartfailure,andprematuredeath.
SummaryBox:AorticStenosis

Case17
A56yearoldmoderatelyobese,postmenopausalwomanwithtype2diabetesisevaluated
duringherannualexamination.Examinationissignificantforabdominalobesityaswellasa
curvilinearpatchofdarklypigmentedskinaroundherneck.Bloodworkrevealsanelevated
lowdensitylipoprotein(LDL)cholesterol,lowhighdensitylipoprotein(HDL)cholesterol,
markedlyelevatedtriglycerides,andafastingplasmaglucoselevelof145mg/dL.

1 Thispatient'sconstellationofsymptomsisconsistentwithwhatsyndrome,
andwhatisitssignificance?
Thesesymptomsareconsistentwiththemetabolicsyndrome(insulinresistancesyndrome,
syndromeX).ThissyndromeishighlyprevalentandincreasingintheUnitedStatesandissignificant
becauseitidentifiespatientsatincreasedriskofprematuredeathfromcardiaccauses.Metabolic
syndromeisparticularlycommoninpostmenopausalwomenandmayaffectuptoaquarterofthe
peoplecurrentlylivingintheUnitedStates.Asdefinedbythe2001NationalCholesterolEducation
Program(ATPIII),themetabolicsyndromeisdiagnosedbythepresenceofatleastthreeofthe
followingfivefeatures:
Centralobesity(waistcircumference>35inchesinwomen,>40inchesinmen)
Triglyceridelevels150mg/dL
HDLcholesterol<50mg/dLinwomen,<40mg/dLinmen)
Fastingbloodglucose110mg/dL
Bloodpressure130/85mmHg
Itisclearthatpatientswhofulfillcriteriaforthemetabolicsyndromeareatsignificantriskfor
increasedmorbidityandmortalityassociatedwithcardiovasculardiseaseanddiabetes.Hencethe
diagnosisofmetabolicsyndromecanhelpidentifypatientswhoshouldbetreatedwithstringent
cardiovascularriskfactormodificationstrategies,despitethefactthatthesefactorsmaynot
individuallyindicatetodoso.Managementofthemetabolicsyndromeconsistsofcontrollingeachof
thecomponentfactorsindividually,aimingforweightcontrol,improvedlipidprofiles,increased
insulinsensitivity,anddecreasedbloodpressure.

2 Partofthiswoman'spresentationisdyslipidemia,adisorderoflipoprotein
metabolism.Whatisthestructureofalipoprotein,wherearelipoproteins
synthesized,andwhatarethemajortypesoflipoproteins?

Lipoproteinsaremacromolecularstructurescomposedofaninnercoreofcholesterolestersand
triglycerides(thelattermayalsobereferredtoastriacylglycerols)andanoutercoreof
apolipoproteins,phospholipids,andunesterifiedfreecholesterol.Themajortypesoflipoproteins
includechylomicrons,verylowdensitylipoproteins(VLDLs),intermediatedensitylipoproteins
(IDLs),LDL,andHDL.Chylomicronsaresynthesizedwithinintestinalenterocytes,whereasVLDLis
synthesizedwithintheliver.IDLandLDLarebothformedwithinthecirculationviaVLDL
catabolism.HDLissynthesizedbytheliverandintestine.

3 Whatarethefunctionsofthevariousformsoflipoproteins?Howarethey
removedfromthecirculation?
Chylomicrons:deliveryofdietarytriglyceridestoadiposetissue,skeletalmuscle,andcardiac
musclecholesterolrichchylomicronremnantsarethentakenupbytheliver.
VLDL:deliveryofliversynthesizedtriglyceridestoadiposetissue,skeletalmuscle,andcardiac
muscleremovedbyintravascularconversiontoIDLandultimatelyLDL.
LDL:deliveryofcholesteroltocellsthroughoutthebodyremovedbyinternalizationviaLDL
receptor(principallyintheliver).Note:DefectsinLDLreceptororinternalizationofLDL
receptorcausefamilialhypercholesterolemia.
HDL:returnofexcesscholesterolfromcellstotheliverforbiliaryexcretioncanremove
cholesterolfromatheromatousplaques.

Case17continued:
Thepatientdecidestotrylifestylemodifications,focusingonalowfat,lowcholesteroldietand
walkingfor45minutes4daysaweek.After3months,herfollowupvisitrevealsthather
weightandlipidprofilearelargelyunchanged.Asaresult,sheisprescribedsimvastatin.Before
shetakesherfirstdose,herliverenzymesarechecked,andsheistoldthatthistestwillneedto
berepeatedperiodically.

4 Whatisthemechanismofactionofsimvastatin(andotherstatins)?
Statinsinhibitthehepaticsynthesisofcholesterolbyinhibiting3hydroxy3methylglutaryl(HMG)
CoAreductase,whichistheenzymethatcatalyzestheratelimitingstepincholesterolsynthesis.Not
onlydothesedrugsreducecholesterolsynthesis,buttheystimulatecellstorespondtodecreased
intracellularcholesterollevelsthroughupregulationofLDLreceptorsontheirsurfaces,which
promotescholesterolintakeandfurtherlowerscholesterollevelsintheblood.Statinsarethemost
potentpharmacologicagentsforloweringcholesterolcontainedinLDL.Theyalsocauseamodest
reductionoftriglyceridesbuthaveonlyasmalleffectonincreasinghighdensitylipoprotein
cholesterol(HDLC).

5 Inadditiontolivertests,whichotherenzymesmightbemonitoredwhenusing
astatin?Withthesetoxicitiesinmind,addingwhichotherclassofcholesterol
loweringdrugstoherregimenshouldonlybedonewithgreatcaution?
Becausestatinsandfibratescanindependentlycausemuscledamage(myositis)andliverdamage
(hepatotoxicity),theserisksareincreasedwhenthetwoclassesareusedtogether.Justas
hepatotoxicityisevaluatedbyperiodicallymonitoringhepaticenzymes,suchasalanine
transaminase(ALT)andaspartatetransaminase(AST),myopathyandrhabdomyolysiscanbe
evaluatedbymonitoringcreatinephosphokinase(CPK)levels.

6 Whatarethecauseandclinicalsignificanceofthiswoman's
hypertriglyceridemia?
Triglyceridescomefromtwoplaces:thoseconsumedinthediet(dietary)andthosesynthesizedby
theliver(nondietary).Dietarytriglyceridesarebrokendowninthegutandthenreformedin
intestinalenterocytes,wheretheyarepackagedinchylomicrons.Chylomicronsenterthelymphatics
andeventuallydrainintothevenouscirculationviathethoracicduct.Nondietarytriglyceridesare
primarilysynthesizedintheliverandenterthebloodstreampackagedinVLDLs.
Recentmetaanalysessuggestthatelevatedtriglyceridesareanindependentriskfactorforcoronary
vasculardisease.Onetheoreticalmechanismforthisfindingmaybeattributedtothetransferof
triglyceridestoHDLbycholesterolestertransferprotein(CETP).TriglyceriderichHDLisrapidly
catabolizedbylipoproteinlipase,whichresultsinadecreaseinHDLlevelswhenevertriglycerides
areelevated.

7 Whatenzymaticmechanismclearstriglyceridesfromthecirculation?
Lipoproteinlipase(presentontheluminalsurfaceofcapillaryendothelialcellsinadiposetissue,
skeletalmuscle,andcardiacmuscle)releasesfattyacidsfromtriglyceridespresentinchylomicrons
andVLDL.Thereleasedfattyacidsthendiffuseintothecells.
Note:Insulinstimulateslipoproteinlipaseproteinsynthesis.Ifthisenzymeconcentrationislowas
intype1diabetes,thentriglyceridesaccumulateinthecirculationandhastenthedevelopmentof
atherosclerosis.

8 WhatistheclinicalsignificanceofelevationsinLDLlevels?
Suchabnormalitieshavebeenshowntobeassociatedwithanumberofpathophysiologicconditions,
includingatherosclerosis,chylomicronemia,obesity,Alzheimer'sdisease,xanthomas,and
dyslipidemiaassociatedwithdiabetes,insulinresistance,andinfection.

9 Whatistheclinicalsignificanceofthispatient'slowHDLC?
LowHDLCisanotherriskfactorforcoronaryheartdisease.HDLCisinvolvedinreverse
cholesteroltransport,inwhichexcesscholesterolistransportedfromperipheraltissuesand
atheromatousplaquestotheliverforconversionintobilesaltsorunalteredexcretioninthebile.
Recallthatbiliaryexcretionistheonlymajormechanismforcholesterolremovalfromthebody.

Additionally,thereisgrowingevidencetosupportaroleforHDLCinpreventingLDLoxidation.
ThiscouldbeanimportantfindingbecauseoxidizedLDListheatherogenicform(e.g.,macrophages
donotphagocytosenormalLDLbutonlyoxidizedLDL,andarethentransformedintofoamcells).
Note:UnlikenormalLDL,oxidizedLDLcanbetakenupbyscavengerreceptors,thuspromoting
pathologiccholesterolaccumulationandatherosclerosis.
PlasmaHDLlevelscanincreasewithexercise,moderatealcoholconsumption,andpharmacologic
agentssuchasniacin(mostnotably),fibricacids,andstatins(generallymodesteffect).
Note:AratiooftotalcholesteroltoHDLcholesterolcanbeusedtopredictcoronaryvasculardisease
(CVD)risk.Anoptimalratiois3.5.Apersonwithtotalcholesterolof180mg/dLbutanHDLof
only30mg/dLwouldhavearatioof6,whichwouldplacethatpersonathighriskfordeveloping
CVD.

10 Whatisthemechanismofactionofthelipidloweringfibrates/fibricacid
derivatives(e.g.,gemfibrozil,fenofibrate)?
TheseagentsarePPARalpha(peroxisomeproliferatoractivatedreceptoralpha)agoniststhatlead
toincreasedsynthesisoflipoproteinlipase,causingasignificantreductionintriglyceridelevel.They
alsoincreasetheexpressionofenzymesinvolvedinfattyacidoxidation,whichresultsinadecrease
intheavailabilityoftriglyceridesforVLDLsynthesis.Lastly,theyincreaselevelsofapoAIandapo
AII,bothofwhichpromoteincreasedHDLlevels.FibrateshavelittleeffectonLDLdirectly.
Sideeffectsoffibratesincludehepatotoxicity,gallstones,andmyositis.

11 Howdoesniacininfluencethelipidprofile,andwhatistheunique
pharmacologicfeatureofniacininmanaginglipidlevels?
Niacin(nicotinicacid)isawatersolubleBvitaminthathasmultiplebeneficialeffectsontheoverall
lipidprofile.ItincreasesHDLCby25%to35%,reducesLDLby15%to25%,andsignificantly
reducestriglycerides(TGs).MechanismsofactionincludepromotionofLPLactivitytoenhanceTG
clearancefromcirculatingVLDL(thusreducingLDL)anddecreasedhepaticuptakeofapoAI
(whichdecreasesclearanceofHDLfromcirculation).Niacinisuniqueamongthehypolipidemic
agentsinthatitisthemostpotentagentforincreasingHDLlevels.
Note:Amajorsideeffectofniacinisfacialandupperbodyflushing(niacinrush).Takingaspirinor
anothertypeofnonsteroidalantiinflammatorydrug(NSAID)priortotheniacintoreducethe
synthesisofvasodilatoryprostaglandinscanminimizethisflushing.Donotconfusethisreaction
withtheflushingobservedwithvancomycin,whichcausesredmansyndromethatcanberelieved
withantihistamineusepriortoadministration.

12 Howdothebilesequesteringresins(e.g.,cholestyramine,colestipol)lower
LDLcholesterollevels?

Thebilesequesteringresinsbindbileacidsintheintestineandpreventtheirreuptakeinthedistal
ileum.Thisstimulatesincreasedhepaticsynthesisofbileacids.Becausetheseacidsareformedfrom
cholesterol,increasingtheirsynthesiswillincreasecholesterolcatabolism,upregulateLDLreceptors
onhepatocytes,anddecreaseserumcholesterollevels.
Note:Thesedrugsimpairthebilemediatedemulsification,digestion,andabsorptionoffats,fat
solublevitamins(vitaminsA,D,E,andK),andcertaindrugs.Consequently,adverseeffectsofbile
sequesteringresinscommonlyincludebloating,flatulence,abdominalpain,steatorrhea,deficiencies
offatsolublevitamins,andinadequateoralbioavailabilityofsomedrugsincludingwarfarin,
thiazides,andselectstatins.Decreasedbileacidreabsorptionalsopromotestheformationof
cholesterolgallstones,whichisenhancedbycoadministrationwithfibrates.

OtherRelatedQuestion
13 Whatisthecauseofthedisorderoflipidmetabolismknownas
abetalipoproteinemia?
Abetalipoproteinemiaisanautosomalrecessivegeneticdisordercharacterizedbytheabsenceof
apolipoproteinB,resultinginadeficiencyofchylomicrons(apoB48),VLDL(apoB100),andLDL
(apoB100).SincechylomicronscannotbeproducedandsecretedwithoutapoB48,intestinal
biopsywillreveallarge,lipidvacuolatedenterocytes,andbloodtestswillrevealmarkedlydecreased
(orevenabsent)plasmachylomicrons.Symptomsincludesteatorrhea,weightloss,potentialanemia,
andmalabsorptionoffatsolublevitamins.AdeficiencyinvitaminEcanspecificallypresentwith
neurologicsymptomsduetospinocerebellarandcorticospinaldegeneration.

Themetabolicsyndromeischaracterizedbyabdominalobesity,dyslipidemia,insulin
resistance,andhypertension.Itisamarkerforincreasedcardiovascularrisk.
Treatmentofthemetabolicsyndromeconsistsoftreatingeachofitscomponentswith
lifestylemodificationsandpharmacologicagentstocontrolweight,improvelipidprofiles,
heighteninsulinsensitivity,andlowerbloodpressure.
Cholesterolloweringmedicationsincludestatins(mostpotentforloweringLDLC),
niacin(mostpotentforincreasingHDLC),fibrates,andbilesequesteringresins.
SummaryBox:TheMetabolicSyndromeandDyslipidemias

Copyright2015Elsevier,Inc.Allrightsreserved.

BOOKCHAPTER

ClinicalAnatomy
StephenB.Marko,ThomasA.BrownMDandSonaliJ.Shah
USMLEStep1Secrets,Chapter26,757804

Studentsoftenwonderhowtostudyanatomyforboards.Youmayhavenoticedthatthe
anatomysectionsofFirstAidarerathersparseincomparisonwiththedepthatwhichyoumay
havestudiedthissubjectinyourmedicalschoolcurriculum.Donotinterpretthistomeanthat
anatomywillnotbepresentonyourexamination.Althoughsomestudentshavereported
havingrelativelyfewanatomyquestions,othershavefound5to6questionsperblock.We
sharethisinformationnottofrightenyoubut,rather,togiveyoutheideathatboardsconsiders
anatomytobeimportant.
Unfortunately,itismuchmoredifficulttoprepareforanatomythanitisforsomeoftheother
subjectsonboards.Thereissimplytoomuchmaterialforyoutobeabletolearneverythingat
thispoint.Inconsiderationofthisreality,howshouldyougoabouttacklingthissubject?
Simple:Berealisticanddonotexpecttoknowitall.Focusonhighyieldtopics.Inotherwords,
thisisnotthetimetorelearneachbranchofeverynerveandalloftheoriginsandinsertionsof
everymuscleinthebody.Youarewelcometodothisifyouwouldlike,butyouwouldbe
compromisingtimethatyoucouldbespendingonmoretestworthytopics.
Themostimportantthingtokeepinmindisthatboardsanatomyquestionsareclinically
based.Youwillbegivenclinicalvignettesforwhichyouwillbeaskedtorelatepatients
symptomstoanatomiclesionsanddeformities.Expecttoseexrayfilms,magneticresonance
imaging(MRI)studies,computedtomography(CT)scans,andangiograms.Youmaybegivena
questioninwhichyoumustfirstdeterminethesiteofthelesionbasedonthepatient's
symptomsandthenlocatethedeformedstructureonaradiograph.Spendsometimeperusing
ananatomyatlasandsomecredibleonlinesitesforthesetypesofimages.
Nomatterhowmuchyouprepareforboards,youwillencounterquestionsonmaterialthatyou
failedtostudy.Donotletthisfrustrateyou.Makeyourbestguessandmoveon.Youcanstill
earnaterrificscoreifyoumisstheserandomquestions.Centerthemajorityofyourstudytime
onthetopicswiththebestoddsofappearingonyourexamination.Thischapterwillhelpguide
youtohighyieldanatomytopicsforboards.
NeuroanatomyspecifictipsarediscussedinChapter17.

Insider'sGuidetoClinicalAnatomyfortheUsmleStep1

Case261
Stanley,a68yearoldretiredsoldier,presentstoyourfamilymedicineclinicwitha2yearhistoryof
painandcrampingofthelowerextremities(LEs)withwalking.Thishasnotbeenmuchofa
problem,butforthepastmonth,hehasnoticedpaininhisrightfootthatawakenshimfromsleep.
Hehasa50packyearsmokinghistory,a10yearhistoryoftype2diabetesmellitus,anderectile
dysfunction.

1 Whatisthedifferentialdiagnosisforhisfootandlowerextremitypain?
Peripheralvasculardisease(PVD),neurogeniccauses(e.g.,diskherniationresultingin
radiculopathy),arthritis,trauma,diabeticneuropathicpain,andcoarctationoftheaortaare
considerations.

Case261continued:
Uponfurtherquestioning,youfindthatthepainandcrampingareabsentatrestandbegin
afterabout50metersofwalking.Stoppingforabout5minutesrelievesthepain.Therightfoot
painoccursonlyatnightandisrelievedbyhangingthefootoverthesideofthebed.On
physicalexamination,lighttouch,pinprick,vibration,andtemperaturesenseareintactonthe
LEsbilaterally.Heisnotedtohaveintactfemoralpulses,weakpoplitealpulses,weakposterior
tibialpulses,aweakleftdorsalispedispulse,andanabsentrightdorsalispedispulse.

2 Whatisnowthemostlikelydiagnosis?
PVDischaracterizedbyclaudicationsymptoms(painandcrampingoftheLEswithwalking)that
appearafteraspecificwalkingdistanceandresolveafteraspecificdurationofrest.Theperipheral
pulsefindingsonphysicalexaminationalsoarestronglysuggestiveofPVD.

3 Whichhistoricalfeaturesinthispatientincreasethelikelihoodofa
peripheralvasculardiseasediagnosis?
SmokinganddiabetesmellitusarestrongPVDriskfactors.PatientswithPVDoftenhaveevidenceof
atherosclerosiselsewhere,asdemonstratedbythispatient'spastMIanderectiledysfunction.Other
PVDriskfactorsincludehypertension,hypercholesterolemia(mostnotablyincreasedlowdensity
lipoprotein[LDL]),obesity,sedentarylifestyle,andafamilyhistoryofatheroscleroticdisease.

4 Whydoeshisnocturnalrightfootpainresolvewhenhehangsthe
affectedfootoverthebedside?
RestpaincommonlyoccursinthefeetatnightinPVD.Whenapatientissupine,thereisno
gravitationalassistanceinfootbloodflow.Reducedbloodflowresultsinischemiaandpain.Hanging
thefootoverthesideofthebedplacesthefootbelowtheleveloftheheartthengravityincreasesthe

flowofbloodtotheischemicareas,reducingthepain.

5 Describethepathofarterialbloodfromthehearttothefemoralsheath
Bloodleavestheheartthroughtheaorticvalvetoentertheascendingthoracicaorta,archofthe
aorta,anddescendingthoracicaorta.Thethoracicaortapassesthroughtheaortichiatusofthe
diaphragmatthelevelofT12tobecometheabdominalaorta.Theabdominalaortabifurcatesinto
therightandleftcommoniliacarteriesatthelevelofL4.Eachcommoniliacarterybifurcatesinto
theinternalandexternaliliacarteriesjustanteriortothesacroiliacjoint.Theinternaliliacprimarily
suppliespelvicstructures,whiletheexternaliliacrunsdeeptotheinguinalligamenttobecomethe
commonfemoralartery.

6 Outlinethebordersofthefemoraltriangle
Thefemoraltriangleisborderedbytheinguinalligament(superiorly),thesartorius(laterally),and
theadductorlongus(medially).Fromlateraltomedial,itcontainsthefemoralNerve,common
femoralArtery,femoralVein,femoralcanal(Emptyspacecontaininglymphnodesthatisthesite
offemoralhernias),andtheLacunarligament.TheclassicmnemonicisNAVEL(Fig.261(f0010)).

Figure261
Anatomyofthefemoralcanal.
(FromKhatriVP,AsensioJA:OperativeSurgeryManual.Philadelphia,WBSaunders,2003.)

7 Describethepathofarterialbloodfromthefemoralsheathtothefeet
Thecommonfemoralarteryquicklybifurcatesatthelevelofthefemoraltrochantersintothe
profundafemoris(supplyingprimarilythethigh)andthesuperficialfemoralartery,whichruns
throughtheadductorcanaltothepoplitealfossa(viatheadductorhiatus)tobecomethepopliteal

artery.Intheposteriorcompartmentoftheleg,thepoplitealarterybifurcatesintothetibiofibular
trunkandtheanteriortibialartery,whichperforatesthesuperiormostportionoftheinterosseous
membraneanddescendsintheanteriorcompartmentuntilitcrossestheanklejointtobecomethe
dorsalispedisartery,whichcanbepalpatedonthedorsumofthefootlateraltothetendonofthe
extensorhallucuslongus.Thetibiofibulartrunkbifurcatestobecomethefibularartery,whichruns
downwardinthedeepposteriorcompartmentoftheleg,andtheposteriortibialartery,whichcanbe
palpatedbetweenthemedialmalleolusandthecalcaneusbeforebifurcatingtoformthelateraland
medialplantararteries,whichsupplytheplantaraspectofthefoot(Fig.262(f0015)).

Figure262
Remnantsoftheaxialarteryandarteriesthatdevelopwithlaterdifferentiationareindicated.
(FromLevienLJ,BennC:Adventitialcysticdisease:Aunifyinghypothesis.JVascSurg28(2):193205,1998.)

8 Atwhichsitesisarterialplaqueformationmostlikely?
Themostlikelysitesofplaqueformationarearterialbranchpoints,suchasthebifurcationofthe
tibiofibulartrunkandanteriortibialartery,andtetheredarteries,suchasthesuperficialfemoral
arteryintheadductorcanal.Pathophysiologically,theturbulentbloodflowoccurringatbranch
pointsandtetheringsitescausesshearforcesontheendothelium,increasingthelikelihoodof
endothelialdamageandpotentiallyleadingtoatherosclerosis.Suchchangesareevidentinthis

patient,whoappearstoshowsignificantbilateralsuperficialfemoralatheroscleroticnarrowing
(givenintactpulsesatthefemoraltriangleandweakpulsesatthepoplitealfossa)andmarkedright
anteriortibialnarrowing(supportedbytheabsenceofthedorsalispedispulse).

9 InTable261(t0010),coverthetwocolumnsontherightandattemptto
listthedrugclassandmechanismofactionforeachofthedrugs
commonlyusedintreatmentofperipheralvasculardisease
Peripheralvasculardisease(PVD)iscausedbyatheroscleroticnarrowingofperipheral
arteries,usuallyofthelowerextremities(LEs).
Atheroscleroticlesionsformpreferentiallyatbranchpointsandsitesoftethering,asa
resultofturbulentbloodflowandendothelialshearstress.
BloodflowtotheLE:aortacommoniliacexternaliliaccommonfemoral
profundafemoris(endsinthigh)andsuperficialfemoralpoplitealanteriortibial
(suppliesanteriorcompartmentandendsasdorsalispedisinthedorsumofthefoot)and
tibiofibulartrunkfibular(supplieslateralcompartment)andposteriortibial(supplies
posteriorcompartment)lateralandmedialplantararteries.
Fromlateraltomedial,thefemoraltrianglecontainsthefemoralNerve,Artery,Vein,
canal(Emptyspace),andtheLacunarligament.RemembertheacronymNAVEL.
SummaryBox:PeripheralVascularDisease

Case262,PartA
Oscar,a22yearoldcollegestudent,presentstotheemergencydepartment(ED)afterrear
endingacarwhiledrivinghisnewmotorcycle.Heclaimsthatthemajorsiteofimpactwashis
leftshoulder,buthisheadandneckwerewrenchedtotherightaswell.Heisclearlyintoxicated
andhasmanagedtositup,thoughhisleftupperextremity(UE)hangsbyhissideinmedial
rotation.Hisforearmisextendedandpronatedandhiswristisfrozeninflexion.Theinternon
callclaimstobeabletodiagnoseOscar'sinjuryfromacrosstheroom.

Table261
SelectedDrugsUsedtoTreatPeripheralVascularDisease

Drug

Class

MechanismofAction

Aspirin

Nonselective
cyclooxygenase
inhibitor

IrreversiblyinhibitsCOX1,decreasingplatelet
productionofthromboxaneA2,a
vasoconstrictorandpromoterofplatelet

aggregation
Clopidogrel

P2Y12antagonist

IrreversiblyinhibitsP2Y12,aplateletADP
receptornecessaryforactivationofthe
glycoproteinIIb/IIIapathwayofplatelet
aggregation

Statins(e.g.,atorvastatin,
rosuvastatin,fluvastatin,
lovastatin,pravastatin,
simvastatin)

HMGCoA
reductase
inhibitors

Cilostazol

Phosphodiesterase InhibitscAMPphosphodiesteraseIIItoreduce
IIIinhibitor
cAMPdegradation,vasodilatingperipheral
arteriesandinhibitingplateletaggregation

Pentoxifylline

Phosphodiesterase InhibitscAMPphosphodiesteraseincreases
inhibitor

InhibitHMGCoAreductase,theratelimiting
stepinendogenousproductionofcholesterol,
toreducethebuildupofcholesterolin
atheroscleroticplaques

plateletflexibilityanddecreasesblood
viscosity

ADP,adenosinediphosphatecAMP,cyclicadenosinemonophosphateCoA,coenzymeACOX1,
cyclooxygenase1HMG,hydroxymethylglutarate.

1 WhatstructurehasOscarinjured,andhowhasitledtohisupper
extremityposition?
OscarhastornnerverootsC5andC6(ortheuppertrunk)ofthebrachialplexus,resultingina
conditionknownasErbDuchennepalsy.HisleftUEhangsbyhissideinmedialrotationbecausethe
C5componentoftheaxillarynerveisnecessaryforshoulderflexionandabduction(viathedeltoid)
andlateralrotation(viatheteresminor).HisforearmisextendedandpronatedbecauseC5andC6
arethemaincomponentsofthemusculocutaneousnerve,whichsuppliesthetwomajorforearm
flexors(brachialisandbiceps)andthemajorforearmsupinator(biceps).Hiswristisflexedbecause
theC6componentoftheradialnerveisnecessaryforwristextension(viatheextensormusclesofthe
posteriorcompartmentoftheforearm).

2 IfOscarhadforcedhisupperextremityabovehisheadbygrabbingthe
handlebarsofthemotorcycletopreventhisfall,hemayhavepresented
withalossofsensationandimpairedflexionindigits4and5,impaired
wristflexion,hyperextensionofthemetacarpophalangealjoints,andan
inabilitytoabductandadductdigits2to5.Whatwouldbethediagnosis
inthissituation?
ThispatternofinjuryischaracteristicofatearofnerverootsC8andT1(oralowertrunktear)ofthe
brachialplexus,aconditionknownasKlumpke'sparalysis.Theulnarnerveisexclusivelysuppliedby
C8andT1andisresponsibleforsensoryinnervationtothefifthdigit,themedialhalfofthefourth

digit,andthecorrespondingpalmarsurfaceofthehand.Itcontrolsthemajorityofmedialdigit
flexion(viathemedialheadsoftheflexordigitorumprofundusandtheflexordigitiminimimuscles)
aswellasabduction(viathedorsalinterossei)andadduction(viathepalmarinterossei)ofdigits2to
5.Itispartiallyresponsibleformetacarpophalangealjointflexion(viathelumbricalsofdigits4and
5)andwristflexion(viatheflexorcarpiulnaris)(Fig.263(f0020)).

Case262,PartAcontinued:
Oneyearlater,Oscar'swildwayshavecontinued,andheadmitstotwistinghisankleduringan
episodeofextremeintoxicationlastweekend.Hedidnotgotothedoctorthen,ashewasabit
embarrassed,andhisanklefeelsmuchbetter.Hepresentstodaywithaninabilitytoextendhis
rightelbowandrightwristdropsincehestartedusingcrutchesafriendlethimborrow.

Figure263
Roots,trunks,divisions,cords,andbranchesofthebrachialplexus.
(FromMillerRD:Miller'sAnesthesia,6thed.Philadelphia,ChurchillLivingstone,2005.)

3 WhatstructurehasOscarinjuredthistimeandhowhasitledtohis
upperextremityposition?
Acompressioninjurytotheposteriorcordofthebrachialplexuscanoccurifunderarmcrutchesare
usedincorrectly.Theinabilitytoextendhiswristisduetoalackofradialnerve(abranchofthe
posteriorcord)inputtotheposteriorcompartmentoftheforearm.Theradialnervealsoinnervates
thetricepsbrachiiandanconeusmuscles,whicharenecessaryforelbowextension.Pureradialnerve
palsies(whichoftenresultfrominjurytothenerveatthespiralgrooveofthehumerus)generallydo
notaffectthetriceps,becausethebranchofradialnervetothetricepsisveryclosetotheoriginatthe
posteriorcord,proximaltothespiralgroove.

Case262,PartB

Onemonthlater,Oscar'sfather,George(whohappenstobeawriter),presentstoyourclinic
withparesthesiasandpaininvolvinghislateralpalm,digits1to3,andthelateralhalfofhis
fourthdigit.Hetellsyouthathehasbeentypinglonghoursoverthepastfewmonthsandnow
hasalmostfinishedhislatestmasterpiece.Onphysicalexamination,younoticemildatrophyof
thethenareminence,apositiveTinelsign(tappingthemiddleofthewristcreaseelicits
paresthesiasofthelateralhand),andapositivePhalentest(palmarflexionofthewristfor
longerthan1minuteresultsinparesthesiasofthelateralhand).

4 Whatstructurehasbeeninjuredandhowhasthishappened?
Repetitiveuseofthehandstyping,inGeorge'scasecanleadtoinflammation,swelling,and
subsequentcompressionofthestructureswithinthecarpaltunnel.Thisconditionisknownascarpal
tunnelsyndrome,andthepain,paresthesias,andmusclewastingareduetomediannerveinjury
withinthetunnel.George'spainandparesthesiashaveariseninthedistributionofthesensory
componentofthemediannerve,thelateralpalm,andthelateral3digits.Becausethemedian
nervealsoinnervatestheintrinsicmusclesofthethumb,earlythenarwastinghasoccurred.

5 Outlinethecontentsofthecarpaltunnel
Thecarpaltunnelisformedbytheeightwrist(carpal)bonesandthetransversecarpalligament
(flexorretinaculum),whichspansfromthetuberclesofthescaphoidandtrapeziumtothepisiform
andthehookofthehamate.Thecontentsincludethefourflexordigitorumprofundustendons,the
fourflexordigitorumsuperficialistendons,theflexorpollicislongustendon,andthemediannerve(
Fig.264(f0025)).

Figure264
Crosssectionalanatomyofwrist.Tendonsandmediannervemaybecompressedbyinflammationorinfection
becausetheyareencompassedbysynovialsheathandflexorretinaculum.
(FromNobleJ:TextbookofPrimaryCareMedicine,3rded.St.Louis,Mosby,2001.)

6 Namethemusclesofthethenarandhypothenareminencesand
describetheirfunction
Thenareminence(locatedproximaltothethumb):opponenspollicis,abductorpollicisbrevis,
andflexorpollicisbrevis(allinnervatedbythemediannerveafteritpassesthroughthecarpal
tunnel).
Hypothenareminence(locatedproximaltothefifthdigit):opponensdigitiminimi,abductor
digitiminimi,andflexordigitiminimi(allinnervatedbytheulnarnerve).
Function:Thesetwogroupshavesymmetricalfunction.Astheirnamessuggest,eachgroup
hasamusclethatperformsopposition,abduction,andflexion.

7 Describethepatternofsensoryinnervationofthehand
Mediannerve:lateralpalm,thumb,digits2and3,andlateralhalfofdigit4.
Ulnarnerve:medialpalm,medialdorsalhand,digit5,andmedialhalfofdigit4.
Radialnerve:lateraldorsalhand,extendingdistallytotheproximalinterphalangeal(PIP)
jointofdigits1to3andthelateralhalfofdigit4(Fig.265(f0030)).

Figure265
Nervesofthehand.A,Volaraspect.B,Dorsalaspect.antebr.cut.,antebrachialcutaneouscut.br.,
cutaneousbranchdig.br.,digitalbranch.
(FromNobleJ:TextbookofPrimaryCareMedicine,3rded.St.Louis,Mosby,2001.)

Case262,PartBcontinued:
AsGeorgewalksoutoftheoffice,heslipsonicystepsandfallsforward,breakingthefallwith
hisrighthand.Heimmediatelyfeelsseverepaininhiswrist,andashisdistalforearmisvery
obviouslydeformed,hecarefullywalksbackintotheclinic.Youorderanxrayfilm,andthe
radiologistremarksthatsheseestheclassicdinnerforkdeformityofacertainfracture
commonlyseeninpatientsovertheageof50.

8 WhatfracturehasGeorgesuffered?
Collesfractureisafractureofthedistalradiusinwhichthedistalfragmentisdisplaced
posteriorly/dorsally.Radiographically,theangleoftheradiusandthefragmentincombinationwith
theangleofthefragmentandthehandresemblesthecurvatureofafork.Thisfractureiscommon
aftertheageof50andmostoftenoccurswhenonebreaksafallwithanoutstretchedhand.

Case262,PartC
Twomonthslater,youareworkingintheEDagain,andyoucomeacrossanothermemberof
thefamily,20yearoldJake.Heisaformerhighschoolstartingpitcherwhowasgettinga
lessonfromOscaronhowtorideamotorcycle.Unfortunately,hetoohastakenanastyfall.The
attendingattheEDsayssheisworriedaboutahumerusfracture.

9 Whatarethethreemostcommonsitesofhumerusfracture,andwhich
nerveandarteryareatriskateachofthesesites?
SeeTable262(t0015).
Table262
ThreeMostCommonSitesofHumerusFracture

HumerusFracture
Site

Nerve

Artery

Surgicalneck

Axillary Anteriorandposteriorcircumflexhumeral(branchesoftheaxillary
artery)

Midshaft

Radial

Profundabrachii(branchofthebrachialartery)

Supracondylar

Median Brachial

10 OnreviewingJake'spastmedicalhistory,younotethathisbaseball
careerwasmarredbyapartiallytornrotatorcuff.Describewhythe
rotatorcuffmakestheglenohumeraljointdifferentfromotherjoints,and
nameitsfourcomponents
Mostjointsarestabilizedprimarilybyaligamentouscapsule,buttheglenohumeraljointisstabilized
primarilybytherotatorcuff,whichconsistsofthetendonsoffourmusclessupraspinatus,
infraspinatus,teresminor,andsubscapularis(Fig.266(f0035)).Thisdesignallowsthe
glenohumeraljointtohavethewidestrangeofmotionofalljointsinthebody,attheexpenseof
stabilityandresistancetoinjury.Therotatorcuffstabilizestheglenohumeraljointbypullingthe
headofthehumerustowardtheglenoidfossaofthescapulaasothermusclesflex,extend,abduct,or
adductthearm.Rapid,forceful,orrepetitivemovements(suchasrepeatedlythrowingabaseball)
cantearthetendonsoftherotatorcuff,leadingtoalackofjointstability,restrictedmovement,and
pain.
Upperextremityinjuries,particularlythosethatinvolvethebrachialplexus,arepopular
subjectsforStep1anatomyquestions.Youshouldlearnallofthebrachialplexuscomponents
andthemusclesthattheyinnervate.Remember,boardsquestionsusuallyhaveaclinicalfocus!
Whenyoustudythebrachialplexus,spendmostofyourtimereasoningthroughthevarious
injuriesthatresultfromlesionstothedifferentnerves,branches,trunks,divisions,andsoon.
Youareexpectedtoknowthemostcommonwaysinwhichtheseinjuriescanoccur,becauseit
islikelythatyouwillhavetodeducethisfromthestemofthequestion(e.g.,associatefrequent
computerusewithmediannervedamage).Lowerextremitynerveinjuriesarealsotestedon
boardsbutnotquiteascommonlyasupperextremityinjuries.
Step1Secret

Fromproximaltodistal,thebrachialplexusconsistsof:
Fiveroots:C5throughT1
Threetrunks:superior,middle,andinferior
Sixdivisions:threeanteriorandthreeposterior
Threecords:lateral,posterior,andmedial
Fivemajorbranches:musculocutaneous,axillary,radial,median,andulnar
Themusculocutaneousnerveinnervatestheflexorsoftheelbow.
Theaxillarynerveinnervatesthedeltoidandteresminormusclesaswellasthelong

headofthetricepsbrachii.
Theradialnerveinnervatestheextensorsinthearmandforearm.
Theulnarnerveinnervatesthemedialheadsoftheflexordigitorumprofundus,the
flexorcarpiulnaris,mediallumbricals,interossei,andhypothenarmuscles.
Themediannerveinnervatestheanteriorforearmmusclesnotinnervatedbytheulnar
nerve,thelaterallumbricals,andthethenarmuscles.
Thebloodsupplytotheupperextremity(UE)isfromthebrachialartery,the
continuationoftheaxillaryarteryafteritcrossestheteresmajor.
Thecarpaltunnelconsistsoftheeightcarpalbonesandthetransversecarpalligament.
Itcontainsthemediannerveandthetendonsofthelongflexorsofthedigits.
Therotatorcuffconsistsofthetendonsoffourmuscles:supraspinatus,infraspinatus,
teresminor,andsubscapularis.
SummaryBox:UpperExtremityInjuries

Case263,PartA
Youareworkinginanoutpatientpediatricsclinic,andyournextpatientisa14dayoldinfant
whowasborn2weeksprematurely.Themotherhasnocomplaints,andyouproceedto
examinethechild.Theinfantismildlydiaphoretic,andyounoticeacontinuous(bothsystolic
anddiastolic)machinerymurmur,auscultatedbestatthesecondleftintercostalspace.

Figure266
Crosssectionalviewoftheshoulderwithhumeralheadstabilizedinshallowscapularglenoidbyrotatorcuffand
capsule.
(FromNobleJ:TextbookofPrimaryCareMedicine,3rded.St.Louis,Mosby,2001.)

1 Whatisthemostlikelydiagnosis?
Patentductusarteriosus(PDA)islikely.

2 Whatistheutilityoftheductusarteriosus?

Inutero,thisconnectionbetweentheaortaandthepulmonaryarteryactsasarighttoleftshunt.
Itallowsthemajorityofoxygenatedblood(whichhasenteredtherightsideoftheheartthroughthe
routeofplacentaumbilicalveinductusvenosusinferiorvenacava)tobypassthedeveloping
(nonfunctioning)lungsandenterthesystemiccirculation.Justafterbirth,theductusarteriosus
normallyclosesandundergoesfibroticdegenerationtobecometheligamentumarteriosum(Fig.26
7(f0040)).

Figure267
Courseoffetalcirculationinlategestation.Notetheselectivebloodflowpatternsacrosstheforamenovaleand
theductusarteriosus.
(FromMillerRD:Miller'sAnesthesia,6thed.Philadelphia,ChurchillLivingstone,2005.)

3 Whatcausestheductusarteriosustocloseafterbirth?

Duringlifeasafetus,circulatingprostaglandinsandalowbloodPo2keeptheductusarteriosus
open.BloodPo2riseswhenbreathingisinitiated,signalingthenewborn'sabilitytoobtain
oxygenatedbloodfromthelungsratherthanfromtheumbilicalvein.AlongwithrisingPo2,falling
levelsofprostaglandinsacttoclosetheductusarteriosus.Intheeventthattheductusarteriosus
remainspatentafterbirth,itcanoftenbeclosedbyadministeringadrugthatblockstheproduction
ofprostaglandinE2,suchastheCOXinhibitorindomethacin.Alternatively,intheeventthatababy
isbornwithacongenitaldefectsuchastranspositionofthegreatvessels,itisnecessarytokeepthe
ductusarteriosusopenuntilthetranspositioncanbesurgicallyfixed.Thiscanbeachievedby
administeringalprostadil,aprostaglandinE1analog.

Case263,PartB
ItisaslowmorningonyourcardiologyrotationwhenRick,a32yearoldmanwithahistoryof
factorVLeidenanddeepvenousthrombosis(DVT),isbroughtbyambulancetotheED
apparentlysufferingfromastroke.Hehasnohistoryofatrialfibrillation,valvulardisease,or
MI.Oncardiacauscultation,youhearamildsystolicejectionmurmurandwide,fixedsplitting
ofS2.NeurologyconfirmsthatRickhashadanischemicstroke,andatransesophageal
echocardiogram(TEE)revealsinteratrialbloodflow.

4 Whatisthemostlikelydiagnosis?
Atrialseptaldefect(ASD)ismostlikely.Athromboembolicstrokeisrelativelyrareinpatients
withoutatrialfibrillation,valvulardisease,orapastMI.AlthoughfactorVLeidenproducesa
hypercoagulablestate,itismuchmorelikelytocauseDVTthanaleftsidedheartorarterial
thrombus.ThephysicalexaminationandTEEstudyfindingsstronglysupportASD,soRick'sstroke
wasmostlikelycausedbyaparadoxicalembolus(inwhichanembolusofvenousoriginthattraveled
throughtheASDandthentothecerebralvasculature).

5 Isatrialseptaldefectthemostcommoncongenitalheartdefect?
No.Ventricularseptaldefect(VSD)isthemostcommon.ASDisthesecondmostcommoncongenital
heartdefect,andPDAisthethird.

6 Whatarethethreemostcommontypesofatrialseptaldefect?
1.Ostiumsecundumdefect
OstiumsecundumdefectisthemostcommontypeofASD.Theatrialseptumisformedbythe
septumprimumandtheseptumsecundum.Inostiumsecundumdefect,thereisusuallyexcessive
absorptionoftheseptumprimum,inadequategrowthoftheseptumsecundum,orenlargementof

theforamenovale(theopeningattheinferiormarginoftheseptumsecundum).Asubtypeofostium
secundumdefectispatentforamenovale,inwhichtheseptumprimumandseptumsecundumfailto
fuse.Thiscommondefectmayallowinteratrialbloodflow(notethatthisisphysiologicinfetallife).
2.Ostiumprimumdefect
Inostiumprimumdefect,theseptumprimumfailstofusewiththeendocardial(atrioventricular
[AV])cushion.Thisisoftenduetoanendocardialcushiondefect,commonlyassociatedwithDown
syndrome.Notethattheendocardialcushionisthepointoffusionfortheatrialseptum,ventricular
septum,mitralvalve,andtricuspidvalve,andthemagnitudeofthedefectdeterminesthepathology.
Forinstance,partialostiumprimumdefectcausesaninteratrialconnection,butcompleteostium
primumdefectcausesanAVconnection.
3.Sinusvenosusdefect
Normally,theatrialseptumdevelopscompletelytotheleftofthesinusvenosus,thestructurethatis
tobecomethesuperiorandinferiorvenaecavaeandpartoftherightatrium.Inthisraredefect,the
septumdevelopsanteriortothesinusvenosus,allowinginteratrialflowviathesinusvenosus(Fig.
268(f0045)).

Figure268
Developmentofatrialseptum.A,Theseptumprimum(SP)growsdownfromtheroofoftheprimitivecommon
atriumtomeettheatrioventricularcushions(AVCs)anddividestheprimitivecommonatriumintorightatrium
(RA)andleftatrium(LA).ThedefectbelowthegrowingfreeloweredgeofSPiscalledostiumprimum(OP)
shownhereasadottedoval.TheseptumprimumhasreachedtheAVCs,whichhavedevelopedintotricuspid
(TV)andmitral(MV)valves.ThedrawingshowsthattheupperpartoftheSPhas(normally)degeneratedto
leavethelargeostiumsecundum(OS)orfossaovaledefect.B,Asecondinteratrialseptumseptumsecundum
(SS)growstotherightoftheSP.Theupperportionsofthetwoseptafuseandaportionoftheupperpart
degeneratestoformtheOS.TheloweredgeoftheSSgrowsdownwardtopartiallycovertheOSbutdoesnot
reachtheAVCs.Avalvelikeopeningforamenovale(FO)istherebyestablished,permittingashuntfromRA
toLAbutnotinthereversedirection.TheFOpersistsduringfetallife(duringwhichittransmitsanessential
righttoleftshunt),butafterbirthitusuallysealsoffbyfusionofthelowerpartofSPwithSS.Notethatthe
interventricularseptumseparatesrightventricle(RV)fromleftventricle(LV).Theinterventricularseptummeets
AVCstotherightoftheatrialseptum,sothatoneportionoftheAVCseparatesRAfromLV.Thisportionlater
formstheupperpartoftheinterventricularseptum,anditisthroughthisportionthattheGerbodedefectoccurs.
(FromGraingerRG,AllisonD,AdamsA:Grainger&Allison'sDiagnosticRadiology:ATextbookofMedical

Imaging,4thed.Philadelphia,ChurchillLivingstone,2001.)

7 Howmightatrialseptaldefectleadtorightsidedheartfailure?
Leftsidedheartpressuresarehigherthanrightsidedheartpressures,soanASDallowsforleftto
rightshuntingofblood.Thisincreasesflowvolumesthroughtherightsideoftheheart,leadingto
rightventricular(andatrial)dilation,increasedstrokework,decreasedpumpingability,and
eventuallyrightsidedheartfailure.

8 Whatisthedreadedlatecomplicationofatrialseptaldefect?
Eisenmenger'scomplexreferstoasituationinwhichinappropriatelyincreasedrightsidedheart
flowandpressuredamagethepulmonaryvasculature,causingsmallvesselfibrosistodevelop.The
fibroticvasculaturefurtherincreasespulmonaryhypertensionanddoesnotcontributetogas
exchange,causingtherightsideofthehearttoincreaseitsoutput.Eventually,rightsidedheart
pressuresbecomehighenoughtoreversetheshuntofASDtorighttoleftshunt,resultingin
cyanosisandheartfailure.

Case263,PartC
Youareinthenewbornnurseryonthefirstdayofyourinpatientpediatricsrotation,andthe
neonatologyfellowinvitesyouintoseeaninfant.Theinfantisobviouslycyanoticbutseemsto
behavingnorespiratorydifficultyotherthanmildtachypnea.

9 Whatarethefivecardiogeniccausesofcyanosisinanewborn?
ThesefivecausesareknownasthefiveTs:TetralogyofFallot,Transpositionofthegreatvessels,T
runcusarteriosus,Tricuspidatresia,andTotalanomalouspulmonaryvenousreturn.

Case263,PartD
Oncardiacauscultation,youheara3/6systoliccrescendodecrescendomurmuratthesecond
leftintercostalspaceandaloudS2atthefourthleftintercostalspace.Thefellowshowsyouthe
infant'schestradiograph,whichshowsabootshapedheart,signifyingrightventricular
hypertrophy.

10 Whatisthemostlikelydiagnosis?
TetralogyofFallotisthecongenitalheartdefectmostlikelytocausecyanosisininfants.The
murmursoftetrologyofFallotvarydependingonthedegreeofpulmonarystenosisandtheextentof
theVSD.Inthiscase,thepulmonarystenosismurmurovershadowsanyVSDmurmurthatmightbe
appreciable.TheloudS2isduetotheclosureoftheaorticvalve.Recallthefourcomponentsofthis
disease:VSD,overridingaorta,pulmonarystenosis,andrightventricularhypertrophy.

11 HowdoestetralogyofFallotcausecyanosis?
Pulmonarystenosiscausesahighresistancetoflowthroughthepulmonarytrunk.Coupledwith
concentricrightventricularhypertrophy,thisleadstohighrightsidedheartpressures,causingright
toleftflowthroughtheVSD(Fig.269(f0050)).Whenacriticalproportionofdeoxygenatedbloodis
shuntedthroughtheVSDandmixedwithoxygenatedblood,cyanosisappears.Itisclassicallyseen
firstinthefingersandlips.

Figure269
Schematicdiagramofthemostimportantrighttoleftshunts(cyanoticcongenitalheartdisease).Tetralogyof
Fallot.Diagrammaticrepresentationofanatomicvariants,indicatingthatthedirectionofshuntingacrossthe
ventricularseptaldefect(VSD)dependsontheseverityofthesubpulmonarystenosis.Arrowindicatesthe
directionofthebloodflow.AO,aortaLA,leftatriumLV,leftventricleRA,rightatriumRV,rightventricle.
(CourtesyofWilliamD.Edwards,M.D.,MayoClinic,Rochester,MN.)

12 WhatisthecauseoftetralogyofFallot?
Unequalpartitioningoftheprimitivetruncusarteriosusbythetruncoconalridgesmisplacesthe
infundibularseptum,thestructurethatdividesthetwoventricularoutflowtracts.The
anterosuperiordisplacementoftheinfundibularseptumcausespulmonarystenosisinadditionto
causingtheaortatooverridetheVSDthatresultsfromfailureofthetruncoconalridgestofusewith
themuscularinterventricularseptum.Stenosisofthepulmonarytrunkincreasesafterload,leading
toconcentricrightventricularhypertrophy,seenonaradiographasabootshapedheart.

Theductusarteriosusconnectsthepulmonaryarterytotheaortaandallowsbloodto
bypassthedevelopinglungsduringfetallife.
ThepatencyoftheductusarteriosusismaintainedbylowPo2andprostaglandinE2.It
canbepharmacologicallyclosedwithindomethacinorkeptopenwithalprostadil.

Duringfetallife,bloodalsobypassesthedevelopinglungsviatheforamenovale,which
isappropriatelypatentatthistime.
Atrialseptaldefect(ASD)canberecognizedoncardiacauscultationbyamildsystolic
ejectionmurmurandwide,fixedsplittingofS2.
Ventricularseptaldefect(VSD)isthemostcommoncongenitalheartdefect.
Eisenmenger'scomplexreferstoarighttoleftshunt(resultingincyanosis)that
occurredbecausealefttorightshunt(VSD,ASD,orpatentductusarteriosus[PDA])
causedrightsidedheartoverloadandpulmonaryhypertension,leadingtohighright
sidedheartpressuresandshuntreversal.
TetralogyofFallotischaracterizedbyVSD,overridingaorta,pulmonarystenosis,and
rightventricularhypertrophy.Itisthemostcommoncauseofrighttoleftcyanoticshunt
ininfants.
SummaryBox:CongenitalHeartDefects

Case264,PartA
John,a19yearoldcollegefreshman,presentstotheEDwithsevereneckstiffnessanda
headache.Histemperatureis38.8Candhehasbeenexperiencingchills.Accordingtofriends,
hehasbecomeincreasinglyconfusedinthepast24hours.

1 WhatisthedifferentialdiagnosisforJohn'ssymptoms?
Meningitis,encephalitis,masslesionofbrain(abscessortumor),andsubarachnoidhemorrhageare
possible.

Case264,PartAcontinued:
Uponfurtherquestioningofhisfriends,itisdiscoveredthatJohnwassufferingfromanupper
respiratorytractinfectionduringthe3dayspriortohiscurrentillness.Onexamination,
Brudzinski'ssign(flexionoftheneckwiththepatientsupineelicitspainandinvoluntaryhip
andkneeflexion)andKernig'ssign(whenthehipisflexed,attemptedextensionoftheknee
elicitspainandisdifficultduetohamstringstiffness)arepositive.

2 Whatisthemostlikelydiagnosisandwhatisthenextsteptoconfirm
thissuspicion?
Hemostlikelyhasmeningitis,whichcanbeconfirmedbylumbarpuncture(LP).

3 Atwhatspinallevelshouldalumbarpuncturebeperformed?Why?
Inadults,theneedleshouldbeinsertedbetweenthespinousprocessesofL4andL5.Thespace
betweenL3andL4isalsoanacceptablechoice,becausetheconusmedullaris(theterminalportion
ofthespinalcord)usuallyendsnearthesuperiorborderofL2.Recallthatthecaudaequina
continuesbelowthislevel,butthefreefloatingnatureofthenervebundlesinthecerebrospinalfluid
(CSF)makesthemlesslikelytosustainpuncturedamage.Itshouldbenotedthatinchildren,anLP
shouldbeperformedonlybetweenL4andL5,becausethespinalcordinchildrencanextendtoL3.

4 Throughwhatmajorstructuresandspaces,fromsuperficialtodeep,
shouldtheneedlepassinalumbarpuncture?
Skinsubcutaneoustissuespinalligaments(supraspinousligament,interspinousligament,and
ligamentumflavum)epiduralspaceduramaterarachnoidmatersubarachnoidspace
(fromwhichCSFcanbedrawn).

5 Describethethreelayersofthemeninges
1.Duramater
Thisoutermostlayerisfusedtotheinsideoftheskullviatheperiostealdurallayer.Thedurais
doublelayeredintheskull,allowingcranialcompartmentalizationandinvestmentofthevenous
sinuses.Incontrast,itissinglelayeredwithinthevertebralcanal,whereitisseparatedfromthe
sidesbytheepiduralspace.TheduralsacextendstoS2andisattachedtothecoccyxviathefilum
terminaleexternum.
2.Arachnoidmater
Thislayerisfusedtotheinnersurfaceoftheduraandsendstrabeculaetotheoutersurfaceofthe
pia.Betweenthearachnoidandthepia(thesubarachnoidspace)liestheCSF.
3.Piamater
Thisisessentiallytheoutermostlayerofthebrain,spinalcord,andnerveroots,asitcannotbe
separatedfromthem.Itinveststhebloodvesselsofthebrainandspinalcord.Thepiacontinuesafter
theconusmedullarisasthefilumterminaleinternum(whichisnotpartofthecaudaequina,because
itdoesnotcontainanyaxons)untiltheendoftheduralsacatS2,whereitisinvestedwithdurato
becomethefilumterminaleexternum,terminatingatthecoccyx(Fig.2610(f0055)).

Figure2610
Spinalcordanatomy.Noticetheterminationofthespinalcord(i.e.,conusmedullaris)atL1L2.
(FromMillerRD:Miller'sAnesthesia,6thed.Philadelphia,ChurchillLivingstone,2005.)

6 Whatcerebrospinalfluidfindingswouldyouexpecttofindwith
differentcausesofmeningitis?
SeeTable263(t0020).
Table263
CerebrospinalFluidFindingsinMeningitis

PathogenicCategory CellType

Pressure Protein

Sugar

Bacterial

PMN

Tuberculosis

Lymphocyte

Fungal

Lymphocyte

Viral

Lymphocyte Normal/ Normal/ Normal

PMN,polymorphonuclearneutrophil(leukocyte).

7 Whatarethemostcommoncausesofmeningitisbyagegroup?
SeeTable264(t0025).
TheinformationinTables263(t0020)and264(t0025)ishighyieldforStep1,andthis
knowledgewillearneasypointsforyouifyoutakethetimetolearnitwell.Thecausesof
meningitisinTable264(t0025)arenotlistedinanyparticularorder,althoughyoushouldnote

thatHaemophilusinfluenzaeisbecominganincreasinglyrarecauseofmeningitissecondaryto
vaccination.ThemnemonicforrecallingthecommoncausesofmeningitisbyagegroupisGEL
MESH,MESHGeLS,whereeachwordrepresentsaseparateagegroup(seeTable264
(t0025)).
Step1Secret

Case264,PartB
Ava,a1montholdgirl,presentsforawellchildcheckup.Sheisdevelopmentallynormalfor
herage.Onexamination,atuftofhairoverlyinga1cmdarklypigmentedpatchisfoundatthe
levelofL5.Thereisnoevidenceofanyneurologicdeficits.

Table264
CommonCausesofMeningitisbyAgeGroup

06Months

6Months6Years

GroupBstreptococci Neisseriameningitidis
Escherichiacoli
Enteroviruses

6Years60Years

60+Years

N.meningitidis
Enteroviruses

Gramnegative
rods

Listeria

Streptococcuspneumoniae S.pneumoniae

S.pneumoniae

monocytogenes

Haemophilusinfluenzae

Herpessimplexvirus

L.monocytogenes

typeb

(HSV)

Table265
EmbryologyoftheFaceandNeck

Structure

Derivative(s)

Firstpouch

Auditorytubeandmiddleear

Secondpouch

Palatinetonsil

Thirdpouch

Inferiorparathyroidsandthymus

Fourthpouch

Superiorparathyroidsandultimobranchialbody(formsthyroidparafollicularC
cells)

Firstarch

Malleus,incus,mandible,maxilla,zygomaticandsquamousportionofthe
temporalbonesmusclesofmasticationanteriorbellyofdigastric,mylohyoid,
tensortympani,andtensorvelipalatinimusclesinnervatedbycranialnerves
V2andV3

Secondarch

Stapes,styloid,mostofhyoidbonemusclesoffacialexpressionstapedius,

stylohyoid,andposteriorbellyofdigastricmuscleinnervatedbycranialnerve
VII
Thirdarch

Greatercornuofhyoidbone,stylopharyngeusmuscleinnervatedbycranial
nerveIX

Fourthandsixth
arches

Laryngealanduppertrachealcartilagemusclesofthesoftpalate,pharynx,
andlarynxstriatedmuscleofesophagusinnervatedbycranialnerveX

Firstcleft

Externalacousticmeatus

Second/third/fourth Cervicalsinus(eventuallybecomesobliterated)
cleft

8 Whatdisorderofneurologicdevelopmentcanbecharacterizedby
thesefindings?
Spinabifidaoccultaisadisorderinwhichtheposteriorneuraltubefailstoclose,resultingina
failureofmidlinevertebralarchclosurewithintactdura.Therearenoassociatedneurologicdeficits.

9 Whataretheothersignificantdisordersrelatedtoafailureofposterior
neuraltubeclosure?
A.Spinabifidacysticameningocele
Failureofposteriormidlineclosureofboththevertebralarchandtheduramater.Thearachnoid
materherniatesthroughthedefect,creatingacyst.Neurologicdeficitsmayormaynotoccur.
B.Spinabifidacysticameningomyelocele
Failureofposteriormidlineclosureofboththevertebralarchandtheduramater,butthedefectis
wideenoughtoallowspinalcordherniationwiththearachnoid.Neurologicdeficitsarelevel
dependentbutusuallyincludeparalysisofsomedegree.
C.Rachischisis
Becauseofanunderlyinginabilityoftheneuralfoldstofuse,thespinalcordfailstorollintoaround
structure,andtheposteriorneuraltubeisunabletoclose.Thisresultsinexposureoftheflattened
spinalcord.Thisdefectmayberestrictedtoasmallareaoritmaybeextensive.Aswiththethree
typesofspinabifida,rachischisisisthoughttodevelopasaresultofmultiplegeneticlesionscoupled
withenvironmentalfactors,withmaternalfolatedeficiencyduringfetalorganogenesisbeingthe
mostprominentexample.

Case264,PartC

Jeremy,a64yearoldcarpenter,presentswithseverelowerbackpain.Mostofthepainis
localizedtoasinglespotinhislowerback,buthehasilldefinedpainandtinglingthatbegins
intheleftglutealregionandcoursesdownthelateralsideofhisleftLEtohisfoot.Healsohas
experiencedleftLEweakness.Hereportsthatallofthesesymptomsstarted3daysago,coming
onsuddenlywhilehewasworkinginhisyard.Onphysicalexamination,passiverightstraight
legraisecausesmoderatepaininthedistribution[ontheleft]asdescribedhere.

10 Whatisthemostlikelydiagnosis?
Jeremyhassufferedanintervertebraldiskherniation.Thisissupportedbyhissevere,acuteonset
lumbarbackpain(upwardof90%ofdiskherniationsinvolvetheL4/L5orL5/S1disks),unilateral
motordeficit,andpainandtinglingthatfollowaspinalnervedistribution.

11 Describeintervertebraldiskanatomyandhowherniationusually
occurs
Thenucleuspulposusisthecentralelasticcartilaginousportionofthedisk.Itissurroundedbythe
anulusfibrosus,whichconsistsofconcentricringsoffibrocartilage.Withage,thenucleipulposi
becomethinandlosetheirelasticityandtheannulifibrosidegenerate.Thismakesthediskmore
likelytoherniate,anactioncharacterizedbythenucleuspulposusbreakingthroughalocalized
weaknessintheanulusfibrosus.Herniationsusuallyoccurintheposterolateraldirection,because
thisisasiteofrelativeanulusfibrosusthinnessandthereisnosupportfromtheanteriororposterior
longitudinalligamentsofthevertebralcolumn.Posterolateraldiskherniationsoccurproximaltothe
intervertebralforaminathroughwhichthespinalnervespass,potentiallycompressingthespinal
nervesandleadingtoradiculopathy(Fig.2611(f0060)).

Figure2611
Lumbosacraldiskherniation.Themostcommonposterolateralherniationcompressesthenerveroottraveling
downwardtoemergeonelevelbelowtheleveloftheexitingroot.Hence,L5S1herniationmostcommonly
compressesthedescendingS1root(horizontalhatching).Morelateralherniationmaycompresstheroot
exitingatthelevelofherniation(diagonalhatching).Alargecentralherniationmaycompressmultiplebilateral
descendingrootsofthecaudaequina(verticalhatching).
(FromGoetzCG:TextbookofClinicalNeurology,2nded.Philadelphia,WBSaunders,2003.)

12 Describethepatternofnervecompressionseeninintervertebraldisk
herniations
Recalltheschemefornumberingspinalnerves:ThecervicalspinalnervesC1toC7exitthespinal
canalsuperiortothevertebrawiththesamenumber.ThenamingschemechangesatC8,whichexits
inferiortoC7.Thereafter,thespinalnerverootsexitbelowthevertebrawiththesamenumber(the
L1spinalnerveexitsbelowtheL1vertebra).Despitethechangewithinthisnumberingscheme,disk
herniationstendtocompressthenerverootwiththesamenumberasthevertebrabelowthe
intervertebraldisk.Forexample,C4/C5diskherniationsresultincompressionoftheC5nerveroot,
andL4/L5diskherniationsresultincompressionoftheL5nerveroot.Thisrelationshipis
maintainedbecauseoftheincreasinglyacuteanglesatwhichthespinalnervescomeoffthespinal
cordasitdescends(Fig.2612(f0065)).However,thischangeinangleandthepresenceofthecauda
equinaallowformultiplenervecompressionstooccurwitharelativelymedialherniationinthe
lowerlumbarregion(e.g.,anL5/S1herniationcancompressboththeL5andS1nerves).

Thebestsiteforalumbarpuncture(LP)istheL4/L5interspace,wellbelowtheendof
thespinalcordatL2.
InperforminganLP,theneedleshouldpassthroughthethreespinalligaments,
epiduralspace,dura,andarachnoidtoreachthesubarachnoidspace,whichcontainsthe
cerebrospinalfluid(CSF).
Thethreelayersofthemeninges(superficialtodeep)areduramater,arachnoidmater,
andpiamater.
Spinabifidaisadisorderinwhichtheposteriorneuraltubefailstoclose.
Spinabifidaoccultaistheleastseveresubtype,followedbymeningocele,andthen
bymeningomyelocele.
Intervertebraldiskherniation:
Occurswhenthecentralnucleuspulposusherniatesthroughtheanulusfibrosus.

Usuallyoccursintheposterolateraldirection,wheresupportfromtheanterior
andposteriorlongitudinalligamentsislacking.
Usuallyoccursinthelowerlumbarregion.
Oftenresultsincompressionofthenerverootnamedforthevertebrabelowthe
intervertebraldisk.
SummaryBox:SpinalCordandVertebralColumn

Case265
Alexander,an18yearoldcollegestudent,presentswitha1monthhistoryofanintermittent
bulgeintherightsideofhisscrotum.Hehasrecentlystartedbodybuildingandstatesthatthe
bulgeismorelikelytoappearduringworkoutsandlesslikelytoappearwhileheislyingdown.
Thereisnoassociatedpainorscrotalerythemaoredema.Onphysicalexamination,the
scrotumdoesnottransilluminate,andasoftstructurecanbereducedthroughthesuperficial
inguinalring.

Figure2612
Comparisonofpointsatwhichnerverootsemergefromcervicalandlumbarspine.
(FromKikuchiS,MacnabI,MoreauP:Localisationofthelevelofsymptomaticcervicaldiscdegeneration.J
BoneJointSurg63B:272,1981.)

1 Describethecharacteristicsofthemostlikelydiagnosis
Indirectinguinalherniaischaracterizedbyaprotrusionofparietalperitoneumandviscerathrough
apartoftheabdominalwalllateraltotheinferiorepigastricvessels.Thevisceraexittheabdominal
cavityviathedeepinguinalringandenterthescrotumviathesuperficialinguinalring,passing
throughtheentiretyoftheinguinalcanal.Recallthatparietalperitoneumenvelopsthetesticles

duringtheirdescentoutoftheabdomenintothescrotum,eventuallyformingthetunicavaginalis.
Duringthedescent,thecavityofthetunicavaginalisisconnectedtotheperitonealcavitybythe
processusvaginalis,whichisnormallyobliteratedintheperinatalperiod.Incertaincases,a
persistentprocessusvaginalisremains,formingapotentialspacewithinthespermaticcordthrough
whichindirectinguinalherniascanprotrude.Notethatthisetiologymakesindirectinguinalhernias
congenital(Fig.2613(f0070)).

Figure2613
Indirectinguinalhernia.
(FromRobertsJR:ClinicalProceduresinEmergencyMedicine,4thed.Philadelphia,WBSaunders,2004.)

2 Whatdifferentiatesadirectfromanindirectinguinalhernia?
Adirectinguinalherniaischaracterizedbyprotrusionofparietalperitoneumandviscerathrough
Hesselbach'striangle,borderedlaterallybytheinferiorepigastricartery,mediallybythelateral
marginoftherectusabdominis,andinferiorlybytheinguinalligament.Theherniasacisusually
composedoftransversalisfascia,andalthoughitmaypassthroughaportionoftheinguinalcanal,it
rarelyentersthescrotumandisnotwithinthespermaticcord(Fig.2614(f0075)).Mostdirect
inguinalherniasareacquired.
DifferentiatingbetweenthevariousclassesofherniasiscommonlytestedonStep1.Youshould
understandtherelationshipofthedifferentherniatypestotheirrespectiveanatomicborders.
Step1Secret

Figure2614
Directinguinalhernia.
(FromRobertsJR:ClinicalProceduresinEmergencyMedicine,4thed.Philadelphia,WBSaunders,2004.)

3 Describethestructureoftheinguinalcanal
Theinguinalcanalisaninferomediallydirectedpassagewaythatconnectstwoopenings,thedeep
andthesuperficialinguinalrings.Thedeepinguinalringisinthetransversalisfascia,justlateralto
theinferiorepigastricvessels.Thesuperficialinguinalringisintheexternalobliqueaponeurosisand
liesjustsuperolateraltothepubictubercle.Twowalls,aroof,andafloordelineatethecanalformed
betweenthesetworings.Itsmajorconstituentsareasfollows:Thetransversalisfasciaformsthe
posteriorwall,theexternalobliqueaponeurosisformstheanteriorwall,theinternalobliqueand
transversusabdominismusclesformtheroof,andtheinguinalligamentformsthefloor.Itshouldbe
notedthatthelayersofthespermaticcordareappliedviapassageofthetesticlesthroughthe
inguinalcanal.Consequently,theinternalspermaticfasciaiscontinuouswiththetransversalis
fascia,thecremastericfasciaiscontinuouswiththeinternaloblique,andtheexternalspermatic
fasciaiscontinuouswiththeexternalobliqueaponeurosis.

4 Discussthemajorcontentsofthespermaticcord
Externalspermaticfascia:Continuationoftheexternalobliqueaponeurosis.
Cremastericmuscleandfascia:Continuationoftheinternalobliquemuscleandfascia
drawstestessuperiorly,ofteninresponsetocoldtemperatures.
Internalspermaticfascia:Continuationofthetransversalisfascia.
Vasdeferens:Transportsspermfromtheepididymistotheejaculatoryduct.
Testicularartery:Suppliestestesandepididymis(testiculartorsionisamedicalemergency
becausetwistingofthespermaticcordleadstoocclusionofthisartery).
Pampiniformplexus:Avenousnetworkthatdrainsintotherightandlefttesticularveins.
Thevenousbloodofthepampiniformplexusiscoolerthantheadjacentbloodfromthe
testicularartery.Thiscountercurrentflowcoolstheblooddestinedforthetestes,maintaining
anintratesticulartemperaturejustbelowcorebodytemperature.
Genitalbranchofthegenitofemoralnerve:Suppliessensoryinnervationtotheanterior
aspectofthescrotumandsuppliesmotorinnervationtothecremastermuscle.
Ilioinguinalnerve:Thisnervepiercestheinternalobliquemuscletoentertheinguinalcanal,
thereaftertravelingonthesurfaceofthespermaticcord,ratherthanwithinit,tosupplysome
sensoryinnervationtothesuperioraspectofthescrotumandrootofthepenis.

Other:Autonomicnervefibersandlymphaticvesselsthatdraintotheparaaortic(lumbar)
andpreaorticlymphnodesarealsopresent.

5 Whichlymphnodesarethemostlikelysiteoffirstmetastasisin
testicularcancer?Whyisthisthecase?
Testicularlymphaticfluiddrainsdirectlytothepreaorticandparaaortic(lumbar)lymphnodes.
Recallthatduringembryogenesis,eachdevelopinggonadarisesfromacombinationofmesoderm
andmesotheliumcalledthegonadalridge,whichliesjustmedialtothemesonephros(itself,lying
medialtothemetanephros,whichdevelopsintothekidney).Thus,thetestes(andovaries)develop
markedlysuperiortotheirpositioninadultlife.Consequently,thebloodsupplyandlymphatic
drainageofthetestesarelocatedclosertothekidneysthantoanystructuresofthepelvis.For
example,thetwotesticulararteriesbranchdirectlyfromtheaortajustinferiortotheoriginofthe
renalarteries.Itshouldbenotedthat,incontrastwithtesticularcancer,cancerofthescrotum
initiallymetastasizestothesuperficialinguinallymphnodes.
Youshouldexpecttogetaquestionregardingthesitesoflocalmetastasisforvarioustypesof
cancers.TheUSMLEisespeciallyfondofthefactthattesticularandovariancancers
metastasizetotheparaaorticlymphnodes.
Step1Secret

6 Afteravasectomy,bywhatmeansdoesamaleproduceanejaculate
thatdoesnotincludesperm?Includeasummaryofthepathofsperm
fromspermatogenesistoexitfromtheurethra
Normally,spermpassfromtheirpointoforiginintheseminiferoustubulestotheepididymisand
onwardintothevasdeferens.Thetwovasadeferentiamergewiththeoutletsofthetwoseminal
vesiclestoformtheejaculatoryduct.Theejaculatoryductfeedsintotheprostaticurethra,wherethe
prostateglanddepositsitssecretions.Theprostaticurethraleadstothepenileurethra,wherethe
bulbourethralglandsdeposittheirsecretions.Fromthepenileurethra,theejaculateexitsthebody(
Fig.2615(f0080)).Inavasectomy,thevasadeferentiaareligatedbilaterally,sospermcannotpass

intotheejaculatoryduct,andmostofthemdegenerateintheproximalvasdeferensandepididymis.
Thesecretionsoftheseminalvesicles,prostate,andbulbourethralglandsenterthesystemdistalto
theligationpointsatthevasdeferens,andthesesecretionsareejaculatedwithoutsperm,which
nominallycontributesverylittletothevolumeofnormalejaculate.Thus,thevolumeofejaculateis
notnoticeablychangedbyvasectomy.

Figure2615
Testisandepididymis.Onetothreeseminiferoustubulesfilleachcompartmentanddrainintheretetestisinthe
mediastinum.Twelveto20efferentductulesbecomeconvolutedintheheadoftheepididymisanddrainintoa
singlecoiledductoftheepididymis.Thevasisconvolutedinitsfirstportion.
(FromRetikA,VaughanEJr,WeinA,WalshP:Campbell'sUrology,8thed.PhiladelphiaWBSaunders,2002.)

7 Describetheneurologicbasisforerection,emission,andejaculation
Intheunarousedstate,arteriovenousanastomosesallowmostofthebloodfromthedeeparteryof
thepenistobypassthehelicinearterieswithinthecorporacavernosa.Uponsexualstimulation,
parasympatheticinputtothehelicinearteriescausesvasodilationandvesselstraightening,greatly
increasingbloodflowtothecorporacavernosa,whichbecomeengorged.Asthecorporacavernosa
increaseinvolume,theycompresstheobliquelyexitingveinsagainstthetunicaalbuginea,blocking
outflowofblood.Blooddrainageisalsorestrictedbycontractionoftheischiocavernosusand
bulbospongiosusmuscles,causingacompleteerectiontooccur.Emissionoccursviasympathetic
input,whichcausescontractionofthesmoothmuscleoftheepididymis,vasdeferens,seminal
vesicles,andprostate(effectivelydeliveringspermandsecretionstotheprostaticurethra).
Ejaculationisamixedautonomicandsomaticresponse.Asthesympatheticsystemclosesthe
internalurethralsphinctertoguardagainstbackflow,theparasympatheticsystemcausesperistalsis
oftheurethralmusclewhilethepudendalnervecausescontractionofthebulbospongiosusmuscleto
propelthesemenforward.

8 Namethemostcommondrugsusedfortreatmentoferectile
dysfunctionandoutlinetheirmechanismofaction
Sildenafil(Viagra),vardenafil,andtadalafilarethedrugsmostcommonlyusedtotreaterectile
dysfunction.
Sexualstimulationnormallyresultsinparasympatheticmediatedendothelialcellnitricoxide(NO)
releasewithinthehelicenearteriesofthecorporacavernosa.NOdiffusestotheadjacentvascular
smoothmuscle,whereitcausesvasodilationthroughamultisteppathway.NOdirectlyactivates
guanylylcyclasetoproducecyclicguanosinemonophosphate(cGMP).Thisactivatesproteinkinase
G(PKG),whichthenactivatesmyosinlightchainphosphatase(MLCP),whichdephosphorylates
myosinlightchains,leadingtoarterialsmoothmusclerelaxationandincreasedbloodflowtothe
corporacavernosa.

Sildenafil,vardenafil,andtadalafilinhibitcGMPspecificphosphodiesterase5(PDE5),which
breaksdowncGMP.Notethatthesedrugsdonotactintheabsenceofsexualstimulation,whichis
theinitialeventthatcausesheliceneNOtobeproduced.Giventheirmechanismofaction,itshould
benotedthatthesedrugsshouldnotbeadministeredwithnitrates,ashypotensionmayresult.Note
thatthecommercialwarningsofpriapismrepresentanexceedinglyraresideeffect.Infact,themost
commoncauseofdruginducedpriapismistrazodone,aselectiveserotoninreuptakeinhibitorused
totreatdepression.

Indirectinguinalhernia:Protrusionbeginslateraltotheepigastricvessels,runsthrough
deepinguinalringintoinguinalcanal,andoftenentersscrotum.Itiscongenital.
Directinguinalhernia:Protrusionbeginsmedialtotheepigastricvessels,bypassesdeep
inguinalringintoinguinalcanal,andrarelyentersscrotum.Itisacquired.
Inguinalcanal:beginswiththedeepinguinalringandendswiththesuperficialring.
Anteriorwall:externalobliqueaponeurosis
Posteriorwall:transversalisfascia
Roof:internalobliqueandtransversusabdominismuscles
Floor:inguinalligament
Spermaticcord:investedinlayerscontinuouswiththeabdominalmusclesasitpasses
throughtheinguinalcanalcontentsincludeexternalandinternalspermaticfascia,
cremastermuscleandfascia,vasdeferens,testicularartery,pampiniformplexus,nerves,
andlymphatics.
Testicularlymphaticdrainageistothepreaorticandparaaortic(lumbar)lymph
nodes.
Pathofsperm:seminiferoustubulesepididymisvasdeferensejaculatory
ducturethraurethralmeatus.
Erectionisachievedbytheparasympatheticnervoussystem,emissionbythe
sympatheticnervoussystem,andejaculationbymixedautonomicandsomatic
input.
Erectiledysfunctiondrugsworkbyinhibitingphosphodiesterase5(PDE5).
SummaryBox:HerniasandMaleReproductiveFunction

Case266

Carlos,a24yearoldfishingguide,presentswithintermittentscrotalenlargement,whichhe
firstnoticedafewmonthsago.Herelatesthattheleftscrotumislargerthantherightandthat
theswellingdecreasessignificantlywhenheliesdown.Occasionally,heexperiencesanaching
scrotalpainandheaviness.Onreviewofsystems,youdiscoverthatCarlosandhiswifehave
beenseeninthefertilityclinic,astheyhavefailedtoconceiveafter15monthsoftrying.

1 WhatisthedifferentialdiagnosisforCarlos'ssymptoms?
Indirectinguinalhernia,varicocele,hydrocele,hematocele,testicularcancer,andinfection
(epididymitisorinfectionofthescrotalskin)areallconsiderations.Testiculartorsionandtrauma
shouldberuledoutbutaremuchlesslikely,giventhattheonsetisnotacute.

Case266continued:
Carlosisfoundtobeafebrileandinnoacutedistress.Heisnotcurrentlyhavingtesticularpain.
Uponscrotalpalpation,theleftsidefeelslikethereisabundleofwormssuperiortothetesticle.
ThescrotumbecomeslesstenselyswollenwhenCarlosmovesfromtheuprighttothesupine
position.Thescrotumdoesnottransilluminate.Youareunabletoappreciateaherniasacor
anyfocaltesticularmasses.

2 Whichofthepossibilitiesisnowthemostlikelydiagnosis?
Varicoceleismostlikely.Theinfertility,achingscrotalpainandheaviness,andbagofwormson
testicularpalpationsuggestthisdiagnosis.

3 Outlinevaricocelepathophysiology.Besuretoexplainwhyvaricocele
ismorelikelytooccurontheleftthanontherightandhowthis
conditionmayhaveledtoCarlos'sinabilitytohavechildren
Avaricocelereferstoavaricosity(dilatedandtortuousveins)ofthepampiniformplexus.Theexact
causeisstilldebated,butthereareatleastthreeimportantfactors.Recallthatthelefttesticularvein
drainsintotheleftrenalvein,whichthencrossesbetweenthesuperiormesentericarteryandaorta
todrainintotheinferiorvenacava(IVC).Theangleofthetesticularrenalveinjunctionislarge
enoughtodisturbflow,whichmayresultinbackpressuredownintothepampiniformplexus.
Likewise,becauseitrunsbetweentwoarteries,theleftrenalveinissubjecttocompression
(nutcrackersyndrome),whichresultsinbackpressureintothedistalrenalvein,lefttesticularartery,
andleftpampiniformplexus(Fig.2616(f0085)).However,thoughmorerare,varicocelecanoccur
ontherightsideaswell(therighttesticularveindrainsdirectlyintotheIVCatanacuteangleand
hasnomajorcompressionpoints),soitislikelythatdefectivevalvesinthetesticularveinsplayarole
inthedevelopmentofvaricoceleonbothsides.Thisconditionresultsinsomedegreeofvenousstasis
inthepampiniformplexus,thusdecreasingitsabilitytocoolthearterialbloodenroutetothetestes.
Thehighintratesticulartemperaturesdecreasespermproductionandquality.

Figure2616
Theretroperitonealspacehasbeenexposed.Theduodenumhasbeenkocherizeditssecond,third,andfourth
portionshavebeenreflectedsuperiorly,alongwiththepancreasandthesuperiormesentericartery(SMA).The
entirerightcolonhasbeenmobilizedandexteriorized.IMA,inferiormesentericarteryIMV,inferiormesenteric
veinIVC,inferiorvenacavaLRV,leftrenalvein.
(FromWeinAJ,KavoussiLR,NovickAC,etal:CampbellWalshUrology,9thed.Philadelphia,WBSaunders,
2007.)

4 Describethedifferencebetweenhydroceleandhematocele
Hydroceleisacollectionofexcessnonsanguineousfluidwithinthetunicavaginalis.Itcanbecaused
byapersistentprocessusvaginalisthatcommunicatesbetweenthecavityofthetunicavaginalisand
peritonealcavity,orchitis,epididymitis,corditis,oritcanbeidiopathic.Scrotaltransilluminationis
oftenseenonphysicalexamination.
Hematoceleoccurswheninjurytothespermaticvesselsleadstohemorrhageintothecavityofthe
tunicavaginalis.

Case266continued:
Carlosundergoessurgicalcorrectionofhisvaricocele.Hegoesontohavethreechildrenandis
sopleasedwithhowyoutreatedhimthathisentirefamilyhastransferredtoyourcare.His
father,Miguel,a65yearoldbartender,firstpresentstoyourofficewithapparentcirrhosisand
portalhypertension.Hehasbeenfeelingextremelyfatiguedforthepast3days.Intheoffice,he
isjaundicedandtachypneicandhasa2/6systolicflowmurmur.Youquicklysendhimtothe
ED,whereheisadmittedformelenaandanemia.Upperendoscopyrevealsbleeding
esophagealvarices.

5 Whatareesophagealvarices?

Dilatedesophagealveins.Theesophagealvenoussystemisoneofthesitesofportacaval(portal
systemic)anastomosis.Esophagealveindilationoccursbecauseofhighportalpressuresthatforce
venousflowintothesystemiccircuitinhighervolumesthannormal.

6 Howdotheesophagealveinsconnecttheportalandsystemicvenous
systems?
Toreachportalcirculation,theesophagealveinsdrainintotheleftgastricvein.Theleftgastricvein
feedsdirectlyintotheportalvein.Toreachsystemiccirculation,theesophagealveinsdrainintothe
veinsoftheazygoussystem.Otherportacavalanastomosesinclude:
Superiorrectalveins(portal)withinferiorandmiddlerectalveins(systemic)(dilationcan
leadtohemorrhoids)
Paraumbilicalveins(portal)withsuperficialepigastricveins(systemic)(dilationcanleadto
caputmedusae)
Variousbranchesofthecolicveins(portal)withtheretroperitonealveinsofRetzius
(systemic)
Branchesofthesplenicvein(portal)withtheleftrenalvein(systemic)

7 Outlinetheflowofbloodtothesuperiorvenacavathroughtheveinsof
theazygoussystem
Theazygoussystemprimarilydrainstheposteriorwallsofthethorax(viaintercostalandvertebral
veins)andtheabdomen(viaascendinglumbarandvertebralveins).Italsoreceivesthemediastinal,
bronchial,andesophagealveins.Theazygoussystemisinfamousforvariability,butingeneral,the
primaryveinistheazygosvein,whichrunsverticallyalongtherightanterolateralaspectofthe
vertebralcolumnwithinthethorax.Notethattherightascendinglumbarveinbecomestheazygos
veinasitcrossesthediaphragm.Theazygosveinendsbyarchingoverthehilumoftherightlungto
drainintothesuperiorvenacava.Theleftascendinglumbarveindrainsposteriorabdominal
structuresbeforeascendingthroughthediaphragmtobecomethehemiazygosvein.Itdrains
posteriorthoracicstructuresasitcontinuesuptoT9alongtheleftanterolateralaspectofthe
vertebralcolumn,whichitthencrossestodrainintotheazygosvein.Theaccessoryhemiazygosvein
drainsfromavariablelevelbetweenT2andT4downtoT8,whereitcrossesthevertebralcolumn
anddrainsintotheazygosvein(Fig.2617(f0090)).

Avaricoceleischaracterizedbyvaricoseveinsofthepampiniformplexus.
Hydroceleandhematoceledifferinthattheseresultfromfluidandblood,respectively,
inthecavityofthetunicavaginalis.
Therightgonadal(testicularorovarian)veindrainsdirectlyintotheinferiorvenacava

(IVC),andtheleftgonadalveindrainsintotheleftrenalvein.
Theleftrenalveinrunsbetweenthesuperiormesentericarteryandtheabdominal
aorta.
Theesophagealveinsareoneofthefivemajoranastomosesthatconnectthesystemic
(viatheazygoussystem)andportal(viatheleftgastricvein)systems.
Theazygoussystem,headlinedbytheazygosveinontherightandthehemiazygosand
theaccessoryhemiazygosveinsontheleft,isthemainvenousdrainageoftheposterior
wallsofthethoraxandabdomen.
SummaryBox:AsymmetriesoftheVenaCava

Case267,PartA
Youarevolunteeringasateamdoctorforalocalhighschoolfootballteam.Youwatchinhorror
asaplayerfromtheopposingteamputsavicioushitonyourteam'sstarrunningback,Nikolai.
Theprimarypointofimpactisthelateralaspectoftherightknee,whichwasthelegNikolai
hadplantedinanattempttochangedirection.Nikolaiisunwillingtoputanyweightonhis
rightlegbecauseoftheextremepain.Onexaminationoftheknee,younotethatanabnormal
degreeofpassivetibialvalgusdeviationisachievable,andthereisapositiveMcMurraytest,as
wellasapositiveanteriordrawersign.

Figure2617
Anatomicstructuresatthecarinallevelasseenfromthesurgeon'sperspectivestandingatthepatient'shead.
AO,aortaAZYGV,azygosveinE,esophagusLMB,leftmainbronchusLSCA,leftsubclavianarteryRA,right
atriumRPA,rightpulmonaryarterySVC,superiorvenacava.
(FromSellkeFW,delNidoPJ,SwansonSJ:Sabiston&SpencerSurgeryoftheChest,7thed.Philadelphia,WB
Saunders,2005.)

1 ListthestructuresthatNikolaihasinjured
Nikolaiissufferingfromtheunhappytriadofkneeinjuries:hehastornhistibial(medial)
collateralligament(MCL),allowingtibialvalgusdeviationlateralmeniscuswithapositiveresulton
aMcMurraytestandanteriorcruciateligament(ACL),associatedwithananteriordrawersign.Note
thattheMcMurraytestcanbeusedtocheckforbothmedialandlateralmeniscustears,depending
onwhetherthemedialorthelateralmeniscusisstabilizedbytheexaminer.

2 Howdoestheposteriorcruciateligamentdifferfromtheanterior
cruciateligament?
TheACLrunsfromtheposteromedialaspectofthelateralcondyleofthefemurtotheanterior
intercondylarareaofthetibia.Itisweakerthantheposteriorcruciateligament(PCL)anditprevents
anteriordisplacementofthetibia.Thus,atornACLyieldsapositiveanteriordrawersign:flexingthe
kneeto90degreesandpullingthetibiaanteriorlyunderafixedfemurresultsinthetibia'sbeing
pulledoutashortdistancelikeadrawer.
ThePCLrunsfromthemedialcondyleofthefemurtotheposteriorintercondylarareaofthetibia,
crossingposteriortotheACL.ThePCLpreventsposteriordisplacementofthetibia.Themost
commonwaytosufferatornPCLisanimpacttothesuperiortibiawithaflexedknee.Consequently,
atornPCLallowsthetibiatobedisplacedposteriorlyunderafixedfemur,amaneuverknownasthe
posteriordrawersign(Fig.2618(f0095)).

Figure2618
Commonmechanismsofkneeinjury.ACL,anteriorcruciateligamentMCL,medialcollateralligamentPCL,
posteriorcruciateligament.
(FromBrownerBD,JupiterJB,LevineAM,etal:SkeletalTrauma:BasicScience,Management,and
Reconstruction,3rded.Philadelphia,WBSaunders,2003.)

3 ExplainwhyNikolai'smedialcollateralligamentandanteriorcruciate
ligamenttearsledtothetearinhislateralmeniscus
InNikolai'scase,hewashitinthekneefromthelateralaspectwithhisfootplanted.Theforceofthe
hitabductedhiskneejoint,rupturingtheMCL,andpropelledhistibiaforwardinrelationtothe
femur,rupturinghisACL.Becauseofthekneeabduction,themedialcondyleofhisfemurwas
essentiallyliftedoffofthemedialmeniscus,puttingtheburdenoftheimpactontothelateral
meniscus,whichtoreinresponsetoshearforceswithinthejointallowedbyhisrupturedACL.It
shouldbenotedthattheMCLfibersintertwinewiththoseofthemedialmeniscus,makingitpossible
forchronicdamagetotheMCLtoextendtothemedialmeniscus.However,inacuteinjuries,suchas
Nikolai's,medialmeniscusinjuriesoccuronlyincombinationwithlateralmeniscusinjuries.

Case267,PartAcontinued:
Nikolaireturnstoyouayearandahalflaterwithissuesresultingfromanotherfootballinjury.
Thispastseason,hesufferedarightfibularneckfracture,andthelegwasimmobilizedfor
severalweeks.Sincehiscastwasremoved,hehasnoticedthathisrightfoothangsandthat
hemuststephigherthanhedidbeforeinordertopreventhistoesfromdraggingonthe
ground.Onphysicalexamination,testinghisfootdorsiflexionreveals5/5strengthontheleft
and2/5strengthontheright.Inaddition,hehasreducedsensationoverthedorsumofhis
rightfoot.

4 Nikolai'sclinicalpicturesuggestsinjurytowhatstructure?
Thecommonfibular(orperoneal)nerveisinjured.Thisnerverunslateraltothefibularneck,
comingfromjustposteriortothefibularheadandcoursinganteriortothefibularneck,whereit
dividesintothedeepandsuperficialfibularnerves.Owingtoitscloseproximitytothefibularneck,
fracturesofthisstructureofteninjurethecommonfibularnerve.

5 Howdoesinjurytothecommonfibularnerveresultinfootdrop,as
seeninNikolai?
Footdropischaracterizedbydifficultywithoraninabilitytoperformdorsiflexionandeversionofthe
foot,leadingtopassiveplantarflexionandinversionofthefoot,especiallywhenwalking.Injuryto
thecommonfibularnerveisresponsibleforthisdysfunction,becausethedeepfibularnerve
innervatestheanteriorcompartmentmuscles(tibialisanterior,extensorhallucislongus,extensor
digitorumlongus,andfibularistertius)thatdorsiflexthefootandthesuperficialfibularnerve
innervatesthelateralcompartmentmuscles(fibularislongusandbrevis)thatevertthefoot.Note
thatthesuperficialfibularnervehassensorybranchesdistributeduponthedorsumofthefootand
thedistalthirdoftheanteriorleg.Alsonotethatthefibularistertiusactstoevertthefootandthat
theactionsofallofthesemuscleshavebeensimplifiedforthisdiscussion(Fig.2619(f0100)).
Damagetothecommonperonealnerveisafavoriteonboards.Anotherfavoriteisdamageto
thetibialnerve,whichinnervatesmusclesthatinvertandplantarflexthefoot.Thetibialnerve
issensorytothesoleofthefoot.
Step1Secret

Figure2619
Commonperonealnerve,majorbranches,rightleg,anterolateralview.

(FromStewartJD:FocalPeripheralNeuropathies,4thed.WestVancouver,BritishColumbia,Canada,JBJ
Publishing,2010,p473.)

6 Listthemusclesoftheposteriorcompartmentoftheleganddescribe
theirinnervation
Thesuperficialmusclegroupconsistsofthegastrocnemius,soleus,andplantaris.Thisisseparated
fromthedeepmusclegroupbythefibroustransverseintermuscularseptum.Thedeepmusclegroup
includesthepopliteus,flexorhallucislongus,flexordigitorumlongus,andtibialisposterior.The
primaryfunctionoftheposteriorcompartmentmusclesisplantarflexion.Theyareallinnervatedby
thetibialnerve,whicharisesjustsuperiortothelateralfemoralcondylefromthedivergenceofthe
twonervesthatcomposethesciaticnerve(theotherbeingthecommonfibularnerve).

Case267,Partacontinued:
Nikolai'sastonishinglybadluckcontinues.Oneyearlater,heisinvolvedinanautomobile
accidentandsuffersadistalleftfemurfracture,whichshearshispoplitealartery.Heundergoes
emergentvascularsurgeryandhispoplitealarteryissuccessfullyrepaired.However,4hours
aftersurgery,Nikolaibeginscomplainingofseverecalfpain.Onexamination,hislegappears
paleandhiscalfisfirm.Thelegispainfulwithpassivemovementandhasmarkedlydecreased
sensation.

7 WhatdreadedvascularsurgerycomplicationhasNikolaisuffered?
Compartmentsyndrome.

8 Whatdividesthecompartmentsoftheleg?
Thecruralfasciaenvelopsallofthemusclesandbonesoftheleg.Thefourcompartments(anterior,
lateral,deepposterior,andsuperficialposterior)areseparatedbytheanterior,posterior,and
transverseintermuscularsepta,aswellastheinterosseousmembrane(whichconnectsthetibiaand
fibula)(Fig.2620(f0105)).

Figure2620
Fourcompartmentsoftheleg:transversesectionthroughmiddleportionofleftleg.
(RedrawnfromMubarakSJ,OwenCA:Doubleincisionfasciotomyoftheleg.JBoneJointSurg59A:184,1977.)

9 Describethemajorpathophysiologiccharacteristicsofcompartment
syndrome
Thefourcompartmentsofthelegareinvestedwithfasciathatishighlyresistanttostretching.Thus,
relativelysmallvolumeincreases,aswouldbeseeninswelling,resultinrapidincreasesinpressure.
Becauseacompartmentrepresentsaclosedsystem,anincreaseinintracompartmentalpressureis
directlytransmittedtothevasculature,leadingfirsttocompressionofsmall,thinwalledvessels.
Higherpressuresresultincompressionofprogressivelylarger,thickerwalledvessels.Ifprolonged,
thisleadstoischemiaandnecrosis.InNikolai'scase,hisintracompartmentalpressurerosebecause
ofinflammationandswellingresultingfromreperfusioninjury.

Case267,PartB
Beatricesustainsagunshotwoundtotherightpelvis.Sheundergoesemergencysurgeryto
removethebullet.Thesurgeryissuccessful,buttheattendingsurgeonadmitsthatsomenerve
damagewasunavoidableduringtheprocedure.Twomonthslater,Beatricepresentstoyour
clinicclaimingthatthesurgerymadeherrightlegshorterthanherleftleg.Toprovethistoyou,
shedemonstratesthattokeepthefootofherlongerlegoffthegroundwhilewalking,she
mustconsciouslyliftithigher,orshemustleantotherightwhilewalking,ineffect,usinga
waddlinggait.Younotethatwhileherleftfootisintheair,herleftpelvissags.

10 Whatstructurehasbeeninjuredandhowhasthisledtoherawkward
gait?
Injurytotherightsuperiorglutealnervehasledtoparalysisofthegluteusmediusandgluteus
minimusmuscles.Thesemusclesabductthethigh,andwhentheyarenotfunctional,thepelvis
cannotbestabilizedwhilestepping.Whilethepatientisstandingononefoot,thecontralateralpelvis
sags,acharacteristicknownastheTrendelenburgsign(Fig.2621(f0110)).Thisisdefinitelya
clinicaltesttoknowforboards.

Notethelowerextremityterminology:thigh=hipjointtoknee,leg=kneetoankle,foot
=ankletodigits.
Theunhappytriadconsistsofinjurytothemedialcollateralligament(MCL),anterior
cruciateligament(ACL),andlateralmeniscus.
TheACLpreventsanteriordisplacementofthetibia,whiletheposteriorcruciate
ligament(PCL)preventsposteriordisplacementofthetibia.
Thecommonfibularnerveisofteninjuredinkneeinjuriesorfibularneckfractures.

Thecommonfibularnerveinnervatesthemusclesoffootdorsiflexionand
eversioninjurytothisnerveresultsinfootdrop.
Thelegisdividedintofourcompartmentsbyunyieldingfasciathus,excessiveswelling
leadstovascularcompressionandischemia(compartmentsyndrome).
Afunctionalgluteusmediusandgluteusminimus(innervatedbythesuperiorgluteal
nerve)arenecessaryformaintainingpelvicstabilitywhilewalking.
SummaryBox:LowerExtremityInjuries

Case268,PartA
Maureen,a42yearoldlibrarian,suffersfromGravesdiseaseandrequiresathyroidectomy.
Shepresentstoyouroffice1monthafterhersurgerywithachiefcomplaintofhoarseness,
whichshefirstnoticedafterhersurgery.

Figure2621
Trendelenburgtest.A,Positionofthehipswhenstandingonthenormalleftleg.Notethatthehipelevatesasa
resultofcontractionofthelefthipmusculature.B,Positionofthehipswhenstandingontheabnormalrightleg.
Notethatthelefthipfallsasaresultoflackofadequatecontractionoftherighthipmuscles.

(FromSwartzMH:TextbookofPhysicalDiagnosis,4thed.Philadelphia,WBSaunders,2001.)

1 WhatisthedifferentialdiagnosisforMaureen'shoarseness?
Vocalfoldparalysis,laryngitis(infectiousorasaresultofgastroesophagealrefluxdisease[GERD]),
carcinomaofthevocalfolds,noduleofthevocalfolds,laryngealmusclespasm,andidiopathicorigin
areconsiderations.

2 Giventhesurgicalhistory,whichofthesediagnosesismostlikelyand
why?
Vocalfoldparalysisresultsfrominjurytotherecurrentlaryngealnerve.Thetworecurrentlaryngeal
nervesrunjustposteriortothethyroidgland,andarepronetoinjuryduringthyroidectomy.

3 Howdoesinjurytotherecurrentlaryngealnerveresultinhoarseness?
Therecurrentlaryngealnervegivesrisetotheinferiorlaryngealnerve,whichinnervatesallofthe
intrinsiclaryngealmusclesexceptforone,thecricothyroidmuscle.Aninjurytoeitherofthesenerves
resultsinnearlycompletevocalcordparalysisonthesideoftheaffectednerve,causinghoarseness.
Notethatdysfunctionoftheposteriorcricoarytenoidmuscleiskeyinthedevelopmentofhoarseness,
asthisistheonlymusclethatcanabductthevocalfolds.Thus,bilateralinjurytotherecurrent
laryngealnervecanresultindyspneaandstridor,causedbyaninabilitytoabducteithervocalfold,
whichobstructstheairwayatthelarynx.
Beonthelookoutfordamagetotherecurrentlaryngealnervethatresultsfromthyroidsurgery.
Thisisafavoritescenarioonboards.
Step1Secret

4 Describethepathoftherecurrentlaryngealnerve,notingany
asymmetries
Inthedevelopingembryo,therightandleftrecurrentlaryngealnervesbranchoffoftherightandleft
vagusnervesandlooparoundthefourthaorticarchesontheirwaybackintotheneck,eventually
endingastheinferiorlaryngealnerves.Therightfourthaorticarchbecomestherightsubclavian
artery,sothisisthearterythattherightrecurrentlaryngealnerveloopsaroundwhendevelopmentis
complete.Incontrast,theleftfourthaorticarchbecomesthearchoftheaorta,sothisstructureis
whattheleftrecurrentlaryngealnerveloopsaroundwhendevelopmentiscomplete(Fig.2622
(f0115)).

Figure2622
Diagramofthevagusnerve(cranialnerveX),specifically,itsbranch,therecurrentlaryngealnerve,andits
relationshiptothelargevesselsoftheneck.Therightandleftnervesarenotidentical,andtherecurrent
laryngealnervebranchesatthebaseoftheneckontherightandinthethoraxontheleft.
(FromGoetzCG:TextbookofClinicalNeurology,2nded.Philadelphia,WBSaunders,2003.)

5 Describetheinnervationoftheloneintrinsiclaryngealmusclenot
innervatedbytherecurrentlaryngealnerve:thecricothyroidmuscle
Thecricothyroidmuscleisatensorofthevocalcordsthatallowshighpitchedphonation.Itis
innervatedbytheexternallaryngealnerve.Sometimesthisnervecanbeinjuredduringthyroid
surgeryaswell.Thisisoneoftwobranchesofthesuperiorlaryngealnerve.Theotherbranchisthe
internallaryngealnerve,whichsuppliessensoryinnervationtothemucousmembranessuperiorto
thevocalfolds.Thesuperiorlaryngealnerveisadirectbranchfromthevagusnerve.

Case268,PartB
Youareatafancyrestauranthavingdinnerwithyourdate.Unfortunately,themanatthetable
nexttoyoubeginschokingonapieceofsteak.YouattempttheHeimlichmaneuverwithno
success.Youtheorizethatthepieceofsteakhasenteredthelarynxandsenttheintrinsic
laryngealmusclesintospasm,thustensingthevocalfoldsandobstructingtheairway.Asthe
manlosesconsciousness,yourequestasharpknifefromthenearestwaitress.

6 Tosavethisman'slife,whichstructuremustyouincise?Why?
Thecricothyroidmembranemustbecut.Thisfibrousmembraneliesinferiortothethyroidcartilage
andsuperiortothecricoidcartilage,connectingthetwostructures.Notethatthethyroidglanddoes
notoverliethethyroidcartilage,asthebulkoftheglandismuchmorecaudal,lyinginferiortothe
cricoidcartilage.Creatinganopeninginthecricothyroidmembraneallowsthepassageoflifesaving
airtobypasstheobstruction,becausetheincisionsiteisinferiortothevocalfolds,whicharedeepto
thethyroidcartilage.

7 Describethesurfaceanatomyoftheneckthatallowsonetofindthe
cricothyroidmembrane

Thelaryngealprominence(Adam'sapple)isthemedianprotrusionofthethyroidcartilage.Thislies
inferiortothehyoidboneataboutthelevelofC5.Byrunningthefingersinferiortothelaryngeal
prominence,downtoaboutthelevelofC6,thearchofthecricoidcartilagecanbepalpated.The
cricothyroidmembraneliesjustsuperiortothecricoidcartilage.Theincisionshouldbemadehere,
withcarenottomovetoofarsuperiorly,becausethethyroidcartilageliesjustabove.Disruptionof
thethyroidcartilagecoulddamagetheintrinsiclaryngealmuscles(Fig.2623(f0120)).

Case268,PartC
ItisSaturdaynightonyourpediatricsrotationandyouarecoveringtheobstetricsfloor.You
entertheroomofthenextnewbornyouhavepreparedtoseeandfindamotherveryupset
aboutthefactthatherchildhastwofissuresrunningtohismouth,onefromeachnostril.On
physicalexamination,younotethatthefissures,justlateraltoeachsideofthephiltrum,extend
intothemouthandmeetattheincisiveforamen,inessencecreatingaUshapefromthetwo
nostrilstotheincisiveforamen.

Figure2623
Anatomyoftheneck.A,Surfaceanatomyoftheneck,showingimportantexternallandmarks.B,Anteriorviewof
theneck,showingvariousinternalstructures(overlyingsuperficialskinandstructuresremovedtoshow
cricothyroidmembrane).C,Lateralviewoftheneck,showingvariousstructures.
(FromRobertsJR:ClinicalProceduresinEmergencyMedicine,4thed.Philadelphia,WBSaunders,2004.)

8 Withwhatdefect(s)hasthischildbeenborn?
Thechildhasbilateralcleftlipandcleftprimarypalate.

9 Describetheembryologicbasisforcleftlip
Cleftlip,oneofthemorecommondevelopmentaldefects,occurswhenoneorbothofthetwo
maxillaryprocessesfailtocompletelyfusewiththecorrespondingmedialnasalprocess.Notethat
thetwomedialnasalprocessestogethermakeuptheintermaxillarysegment,thesuperiorportionof
whichbecomesthephiltrum.

10 Describetheembryologicbasisforcleftpalate
Cleftsoftheprimarypalateoccurwhenthepalatalshelvesofthemaxillaryprocessesfailtofuse
withtheprimarypalate,itselfformedbythefusionofthemaxillaryandmediannasalprocesses.Ifit
isbilateral,itresultsinaUshapedfissure,withtheapexoftheUattheincisiveforamen(thislies
nearthethreewayfusionpointoftheprimarypalateandthetwopalatalshelves).
Cleftsofthesecondarypalateoccurwhenthetwopalatalshelvesfailtofuseatthemidline.The
nasalseptumcanbevisualizedinthemiddleofthecleftsecondarypalate(Fig.2624(f0125)).

Figure2624
A,Schematicdrawingoftheintermaxillarysegmentandmaxillaryprocesses.B,Theintermaxillarysegment
givesrisetothephiltrumoftheupperlip,themedianpartofthemaxillaryboneanditsfourincisorteeth,andthe
triangularprimarypalate.
(FromSadlerTW:Headandneckembryology.InSadlerTW,LangmanJ[eds]:Langman'sMedicalEmbryology,
6thed.Baltimore,Williams&Wilkins,1990.)

Otherimportantconceptsinembryologyofthefaceandneck
11 Discussthedifferencebetweenpharyngeal(branchial)pouches,arches,and
clefts

Thesixpharyngeal(branchial)archesconsistofacombinationofneuralcrestcellsandmesoderm,
andplayaroleintheformationofmanystructuresofthefaceandneck.
Thefourpharyngealpoucheslieinternally,betweenthepharyngealarches,andarelinedwith
foregutendoderm.
Thefourpharyngealcleftslieexternally,betweenthepharyngealarches,andarelinedwith
ectoderm.

12 InTable265,covertherightcolumnandnamethederivativesofeach
structurelistedintheleftcolumn
Embryologyinitselfisarelativelylowyieldsubjectonboards,butTable265(t0030)presents
extremelyhighyieldcontent.Youshouldexpectatleastonequestiononthismaterial.
Step1Secret

13 Fromwheredothepartsofthethyroidgland,otherthantheparafollicularC
cells,originate?
Thethyroidglanddevelopsfromaproliferationofforegutendodermatthebaseofthetongue.From
here,thethyroidglanddescendsthroughthethyroglossalducttojustinferiortothecricoidcartilage.
Itremainsconnectedtotheforamencecumviathethyroglossalductduringdevelopment.The
thyroglossalductnormallydegeneratesbeforebirth,buttheforamencecumpersists,markingthe
locationoftheoriginalepithelialproliferationthatformedthethyroid.

Therecurrentlaryngealnervesliejustdeeptothethyroidandaresusceptibletoinjury
duringthyroidsurgery.
Thesenervesinnervatetheintrinsiclaryngealmusclesexceptthecricothyroids.
Therightrecurrentlaryngealascendsfrombeneaththerightsubclavianartery.
Theleftrecurrentlaryngealascendsfrombeneaththeaorticarch.
Theexternallaryngealnerves,directbranchesofftherightandleftvagusnerves,
innervatethecricothyroidmuscles.
Thecricothyroidmembraneliesinferiortothevocalcords(whicharedeeptothe
thyroidcartilage)anairwayformedinthismembranecanbypassalaryngeal
obstruction.
Cleftlip:Maxillaryprocessfailstofusewithmedialnasalprocess.
Cleftprimarypalate:Palatalshelvesfailtofusewiththeprimarypalate.

Cleftsecondarypalate:Palatalshelvesfailtofuseatthemidline.
SummaryBox:NeckAnatomyandEmbryology

Case269
Janice,a20yearoldwaitress,presentstotheurgentcareclinicwithsevere,sharp,rightupper
quadrant(RUQ)painof6hoursduration.Thepainradiatestoherrightshoulder,and
breathingismoderatelypainful.ShestatesthatshehadamildRUQacheforafewdays,butit
didn'tbotherherandshesawnoreasontoseeadoctor.Aurinepregnancytestisnegative.

1 WhatisthedifferentialdiagnosisforJanice'srightupperquadrantpain?
Cholecystitis,choledocholithiasis,cholangitis,pepticulcerdisease,hepatitis,perihepatitis,hepatic
abscessortumor,pyelonephritis,nephrolithiasis,appendicitis,rightlowerlobepneumonia,ovarian
cystsortumors,andacuteenteritisarepossibilities.

Case269continued:
Janice'spastmedicalhistoryisunremarkable.However,hersexualhistoryisnotablefor
unprotectedsexualintercoursewithabout40differentpartnersoverthepast2years.On
physicalexamination,younotethatonpalpation,sheisexperiencingrightlowerquadrant
(RLQ)painofslightlylessseveritythanherRUQpain.Onpelvicexamination,shehas
exquisitecervicalmotiontendernessandbilateraladnexaltenderness.

2 Whatisthemostlikelydiagnosis?
Pelvicinflammatorydisease(PID)ismostlikely.

3 Namethetwomostcommonorganismsimplicatedinpelvicinflammatory
disease
ChlamydiatrachomatisandNeisseriagonorrhoeaearemostcommonlyimplicatedinPID.Note
thatrarelyPIDcanbecausedbynormalvaginalbacterialfloraaswellasviruses,fungi,and
parasites.

4 Howcanpelvicinflammatorydiseaseleadtorightupperquadrantpain?
PIDisclassicallycharacterizedbyascentofbacteriathathaveinfectedthevaginaandcervix,leading
toendometritis,salpingitis,andperitonitis.Peritonitisispossiblebecausetheinfundibulumofthe
uterinetubesopensdirectlyintotheperitonealcavity.Thismeansthatthereisadirectroutefrom
thevaginatotheperitonealcavity(viatheuterusanduterinetubes)bywhichbacteriacanascend.In
rarecases,thisperitonitiscanleadtoRUQpainwhentheoffendingbacteriareachthelivercapsule
andcauseperihepatitisandinflammationoftherighthemidiaphragm.Thisconditionisknownas
FitzHughCurtissyndrome.

5 Describehowtheuterusandovariesaresupported
Thecervixissupportedanteriorlybythepubocervicalligaments,laterallybythetransversecervical
ligaments,andposteriorlybytheuterosacralligaments.Thebodyofananteverteduterusgainsmuch
ofitssupportbyrestingonthebladder(notethatthisisnotthecaseinpatientswitharetroverted
uterus).Thebroadligament,adoublelayerofperitoneum,extendslaterallyfromtheuterusand
functionstosupporttheuterusandallassociatedstructures(notethatthemesosalpinxportionsof
thebroadligamentsupporttheuterinetubes),aswellastocarrytheuterinevasculature.Theround
ligament,analogoustothespermaticcord,supportstheuterinefundus.Eachovaryisattachedtothe
uterusviatheovarianligament,isenvelopedbythemesovariumportionofthebroadligament,and
issupportedlaterallybythesuspensoryligamentoftheovary,whichattachestothelateralpelvic
wallandcarriestheovarianvasculature(Fig.2625(f0130)).

Figure2625
Theorgansofthefemalepelvis.Theuterusissurroundedbythebladderanteriorly,therectumposteriorly,and
thefoldsofthebroadligamentslaterally.
(RedrawnfromClementeCD:Anatomy:ARegionalAtlasoftheHumanBody.BaltimoreMunich,Urban&
Schwarzenberg,1987.)

6 CanFitzHughCurtissyndromebeseeninmales?
AlthoughPIDandFitzHughCurtissyndromeclassicallydevelopinfemalesthroughtheopen
connectionbetweenthevaginauterusuterinetubesperitonealcavity,infectioncanoccur
throughlymphatic,hematogenous,ordirectspreadfromintraperitonealinfections.Inmales,
infectionscanspreadthroughtheseroutestotheperitoneumandlivercapsule,sotheyalsocan
developFitzHughCurtissyndrome.Notethattheluminaoftheurethra,ejaculatoryduct,vas
deferens,epididymis,andseminiferoustubulesarenotcontinuouswiththeperitonealcavityatany
time.

7 Outlinethecommondrugsusedinantimicrobialpharmacotherapyforpelvic
inflammatorydisease

C.trachomatisandN.gonorrhoeaearethemostcommonorganisms,butthenormalfloraofthe
vagina(e.g.,Gardnerellavaginalis,Streptococcusagalactiae)orgastrointestinal(GI)tract(e.g.,
Bacteroidesfragilis,Peptostreptococcus,Escherichiacoli)canplayaroleintheinfection.Infection
withmultipleorganismsiscommon,sobroadspectrumantibioticsarerecommended.Formildto
moderatelysevereinfections,therecommendedregimenisasingleintramuscular(IM)doseofa
thirdgenerationcephalosporin(suchasceftriaxoneorcefoxitin)plusa14daycourseoforal
doxycyclineandmetronidazole.AnalternativeandequivalenttreatmentisasingleIMdoseof
ceftriaxoneplushighdoseoralazithromycinweeklyfor2weeks.Severeinfections(forinstance,
thoseassociatedwithtuboovarianabscesses)requireinpatienttreatmentwithparenteral
antibiotics.Thetwopreferredregimensarecefotetanorcefoxitinplusoralorintravenous
doxycyclineandclindamycinplusgentamicin.

8 InTable266(t0035),coverthecolumnontherightandnametheabdominal
organsineachlocation
Inmostcasesofpelvicinflammatorydisease(PID),bacteria(usuallyChlamydia
trachomatisorNeisseriagonorrhoeae)fromthevaginapassthroughthecervixintothe
uterusanduterinetubes,causinginflammation.
Peritonitisandinflammationinthelivercapsule(FitzHughCurtissyndrome)
canresultbecausetheuterinetubesopendirectlyintotheperitonealcavity.
Inmales,thereisnodirectconnectionbetweenthelumenofthegenitourinary
tractandtheperitonealcavity,soFitzHughCurtissyndromeismuchmorerare.
AntibiotictherapyinPIDmustincludecoverageforC.trachomatis,N.
gonorrhoeae,andnormalperinealflora(withemphasisonanaerobes).
Thecervixissupportedbythepubocervical,transversecervical,anduterosacral
ligaments.
Theuterusissupportedbythebroadligament(composedofperitoneum),theround
ligaments,andbyrestinganteriorlyonthebladder.
Theovaryissupportedbytheovarianligament,themesovarium,andthesuspensory
ligamentoftheovary.
Althoughtheovariesareexposedtotheperitonealcavity,novisceraactuallyliewithin
it.
SummaryBox:StructureoftheFemaleReproductiveSystemandthePeritoneum

Case2610,PartA

David,a64yearoldnurse,sufferedanMI1monthago.HenowpresentstotheEDwithnew
onsetchestpainthatisintermittentandnotrelatedtoexertion.Thepainissevereandis
locatedintheleftprecordialandretrosternalregions,anditradiatestotheneckandback.

Table266
AbdominalOrgans

Location

AbdominalOrgans

Withintheperitonealcavity

None
note:Theovariesareexposedtotheperitonealcavity.

Intraperitoneal

Stomachandfirstpartofduodenum
Liverandgallbladder
Spleen
Tailofpancreas
Jejunumandileum
Cecumandappendix
Transverseandsigmoidcolon

Secondarilyretroperitoneal*(tf0010) Duodenum:first,second,andthirdparts
Ascendinganddescendingcolon
Rectum
Pancreas:head,neck,andbody
Retroperitoneal

Kidneys(plusuretersandadrenalglands)
Abdominalaorta
Inferiorvenacava

*Notethatsecondarilyretroperitonealorgansdevelopintraperitoneally(coveredbyvisceral
peritoneum)butlatermovetowardtheposteriorbodywallandtheretroperitonealspace,leaving
onlytheiranterioraspectcoveredbyperitoneum.

1 WhatisthedifferentialdiagnosisforDavid'schestpain?
Unstableangina,variant(Prinzmetal's)angina,MI,pulmonaryembolus,aorticdissection,
pericarditis,pleuritis,pneumothorax,pneumonia,costochondritis,ribfracture,anxiety/panicattack,
GERD,diffuseesophagealspasm,andpepticulcerdiseasearepossibilities.

2 Amyocardialinfarctioninwhatdistributionwouldbemostconcerningfor
damagetothesinoatrialandatrioventricularnodes?

Occlusionoftherightcoronaryartery(RCA).TheRCAsuppliestheAVnodeinnearly100%ofthe
populationviatheAVnodalbranch,whichoriginatesneartheoriginoftheposteriorinterventricular
artery.However,itshouldbenotedthattheposteriorinterventriculararteryisabranchoftheRCA
in80%ofthepopulationandabranchofthecircumflexarteryin15%ofthepopulation.The
remaining5%haveothervariations.Thesinoatrial(SA)nodeissuppliedbytheRCAin60%ofthe
populationviatheSAnodalbranch,whichliesneartheoriginoftheRCA.TheSAnodalbranch
originatesfromthecircumflexarteryintheremaining40%.

3 Whichcoronaryarteriessupplytheleftventricle?
Anteriorinterventricular:Shortlyafteroriginatingfromtheascendingaorta,theleft
coronaryarterybifurcates.Onebranch,theanteriorinterventricularartery,alsoknownasthe
leftanteriordescending(LAD)artery,descendsintheanteriorinterventriculargroovetothe
apex.Thisarterysuppliesnearlytheentireinterventricularseptumandmuchoftheright
ventricleaswell.
Circumflex:Thesecondbranchofthebifurcationoftheleftcoronaryartery,thisarteryruns
intheAVgroovetotheposteriorsideoftheheart.Italsosuppliestheleftatrium.
Leftmarginal:Thisbranchofthecircumflexarterydescendsalongtheleftheartborder.
Posteriorinterventricular:ThisarterybranchesfromtheRCA,circumflexartery,orboth.It
descendsintheposteriorinterventriculargroovetotheapex.Italsosuppliesasmallportionof
theinterventricularseptumandpartoftherightventricle(Fig.2626(f0135)).

Figure2626
Coronarycirculation.
(FromMcCanceK,HuetherS:Pathophysiology,5thed.St.Louis,Mosby,2006.)

4 IfDavid'schestpainwerecausedbypleuritis(alsoknownaspleurisy),from
whichpleurallayerwouldhebesensingpain?

Hewouldfeelpainintheparietalpleuraonly.Inpleuritis,theinflammationinvolvesbothpleural
layers.Althoughthevisceralpleuralackssensoryinnervationandthereforecannottransmitpain
sensation,theparietalpleuraisexquisitelysensitivetopainbecauseitisabundantlysuppliedby
somaticbranchesoftheintercostalnerves(intheareasborderingthebodywall)andthephrenic
nerves(intheareasborderingthemediastinumanddiaphragm).Becauseofitsdifferential
innervation,referredpainislocalizedtothebodywallforpleuritisinthedistributionofthe
intercostalnervesandtotheshoulderandneckforpleuritisinthedistributionofthephrenicnerves.

Case2610,PartAcontinued:
Davidgoesontosaythatthischestpaindoesnotfeellikethepainthatwasassociatedwithhis
heartattack.Hehasnoticedthatthepainwaxesandwaneswithbreathinganditisposition
dependent:henotesthatthepainisleastwhilehesitsupandleansforward.Oncardiac
auscultation,africtionrubisheard.

5 Whatisthemostlikelydiagnosis?
Acutepericarditisismostlikely.WeekstomonthsafteraMI,fibrinouspericarditiscanoccur.This
phenomenoniscalledDressler'ssyndromeandisthoughttobeanautoimmunereactiontonovel
antigensresultingfromcardiacdamage,mostofteninthesettingofMI.

6 Ifoneweretopassaneedlefromoutsidethepericardiumtothelumenofthe
leftventricle,throughwhichlayerswoulditpass,insequence?
A.Fibrouspericardium(unyielding,protectsheartfromacutevolumeoverloadfusedwiththe
diaphragm,tunicaadventitiaofthegreatvessels,andtheposteriorsternalsurface)
B.Parietallayerofserouspericardium(fusedtothefibrouspericardium)
C.Pericardialfluid(normallyathinlubricatingfilmallowsthehearttomovewithinthe
pericardialsac)
D.Viscerallayerofserouspericardium(synonymouswiththeepicardiumcontinuouswiththe
parietallayeratthebaseoftheheart)
E.Myocardium(composedofcardiacmuscle)
F.Endocardium(anendothelialliningthePurkinjefibersrunbetweenthislayerandthe
myocardium)

7 Enlargementofwhichchamberoftheheartismostlikelytocausedysphagia?
Theleftatrium,locatedatthebaseoftheheart,isdirectlyanteriortotheesophagus.Marked
enlargementofthischambercanleadtocompressionoftheesophagusaroundthelevelofT6
throughT9.

8 Enlargementofwhichchamberoftheheartismostlikelytocausea
parasternallift?
Aparasternalliftoccurswhentherightventricle,whichcomposestheanteriorsternocostalsurface
oftheheart,isenlarged.Theelevationisusuallyseenorfeltjusttotheleftofthesternum(i.e.,
parasternally).

Case2610,PartB
Attheendofalongweek,youleavetheclinicandheadtoalocalmovietheaterwithfriends.
SoonaftersettlingintoyourseattowatchSurfSummer3:FrostedTips,agroupofteenagers
clamberintotheseatsdirectlyinfrontofyou.Toentertainhisbuddiesbeforetheshow,oneof
thegroupstartstossinghischewyfruitcandieshighintotheair,catchingtheminhismouth.
Beforeyoucanpointouttheinevitable,oneofthecandieslandsintheboy'smouthjustasheis
takingabreath,andthegummygiraffeisrapidlyoutofsight.Astheboycoughs,yourecallthe
restaurantfiascoandpreparetostarttheHeimlichmaneuver.However,heimmediatelybegins
tobreathe,albeitwithquiteabitofwheezinganddyspnea.

9 Ifthecandypassedintothebronchialtree,onwhichsidewoulditmostlikely
befound?
Therightmainbronchusiswiderandorientedmoreverticallyascomparedwiththeleftmain
bronchus,soitisthemorelikelysiteofaspiration.

10 Describetheothermajorasymmetryofthebronchialtree
Therightmainbronchusdividesintothreelobarbronchi,oneforeachlobe,whereastheleftmain
bronchusdividesintoonlytwo,againoneforeachlobe.Therightlunghassuperior,middle,and
inferiorlobes.Theoblique(major)fissureseparatestheinferiorlobefromtheothertwolobesand
thehorizontal(minor)fissureseparatesthesuperiorlobefromthemiddlelobe.Theleftlunghas
onlysuperiorandinferiorlobes,separatedbytheobliquefissure.Theinferiorportionofthesuperior
lobeoftheleftlungiscalledthelingula.Itliesadjacenttotheheartandisthecounterpartofthe
rightmiddlelobe(Fig.2627(f0140)).

Figure2627
Segmentsofthepulmonarylobes.
(ModifiedfromJacksonCL,HuberJF:Correlatedappliedanatomyofthebronchialtreeandlungswithasystem
ofnomenclature.DisChest9:319,1943.)

11 Listthefourstagesinlungdevelopmentandnotewhethereachiscompatible
withlife
1.Pseudoglandularperiod(weeks518):Formationofthetracheathroughterminal
bronchioles.Thesestructurescannotpartakeingasexchange,sobirthatthistimeis
incompatiblewithlife.
2.Canalicularperiod(weeks1626):Respiratoryvasculatureformsandterminalbronchioles
giverisetorespiratorybronchioles.Survivalisrareandcanoccuronlyifbirthisverylatein
thisperiod,whenthereisenoughdevelopmenttoallowforadequategasexchange.Dueto
neonatalrespiratorydistresssyndrome,intensivecarewithintubationisnecessary,and
surfactantmustbeprovidedforanextendedperiod.
3.Terminalsacperiod(weeks2436):Formationofterminalsacs(primitivealveoli)andtype
IIalveolarcells(producesurfactant).Chanceofsurvivalisgood,butiftheinfantisbornearly
intheperiod,neonatalrespiratorydistresssyndromeoftenoccurs.Surfactantmustbeprovided
untilthenewbornisabletoproduceanadequateamountonhisown.
4.Alveolarperiod(week36toearlychildhood):Terminalsacscontinuetoformanddevelop
intomaturealveoli.Chanceofsurvivalisexcellent.

Therightcoronaryarterysuppliestheatrioventricular(AV)node.
Therightcoronaryarterysuppliesthesinoatrial(SA)nodein60%ofthepopulation,

withthecircumflexarterysupplyingtheSAnodeintheother40%.
Theanteriorinterventricular,circumflex,leftmarginal,andposteriorinterventricular
arteriessupplytheleftventricle.
Theleftcoronaryarterybifurcatesintotheanteriorinterventricularand
circumflexarteries.
Theleftmarginalarteryisabranchofthecircumflexartery.
Theposteriorinterventriculararteryisabranchoftherightcoronaryin80%of
thepopulationandthecircumflexin15%ofthepopulation.
Inpleuritis,althoughboththevisceralandparietalpleuraebecomeinflamed,painis
transmittedonlyfromtheparietalpleurabecausethevisceralpleuralackssensory
innervation.
Theoutermostfibrouspericardiumisfusedtotheparietallayeroftheserous
pericardium.
Theparietalandviscerallayersofthepericardiumarecontinuousnearthebaseofthe
heartandcontainthepericardialfluidbetweenthem.
Theleftatriumisthemostposteriorportionoftheheart,lyingjustanteriortothe
esophagus.
Therightventricleistheanteriormostportionoftheheart,andenlargementcanleadto
aparasternallift.
Therightmainbronchusiswiderandmoreverticalthantheleftmainbronchus,soan
aspiratedforeignbodyismorelikelytoenterit.
Thethreelobedrightlungpossessesamiddlelobethatcorrespondstothelingulaofthe
superiorlobeoftheleftlung,whichhasonlytwolobes.
Becauseofinadequaterespiratorydevelopment,approximately50%ofbirthsbefore24
weeksofgestationarenonviable.
SummaryBox:CardiothoracicAnatomy

Copyright2015Elsevier,Inc.Allrightsreserved.

BOOKCHAPTER

ColorPlate
ThomasA.BrownMDandSonaliJ.Shah
USMLEStep1Secrets,118

Figure271
(FromKumarV,FaustoN,AbbasA:RobbinsandCotranPathologicBasisofDisease,7thed.Philadelphia,WB
Saunders,2004.)

Figure272
(FromKumarV,FaustoN,AbbasA:RobbinsandCotranPathologicBasisofDisease,7thed.Philadelphia,WB
Saunders,2004.)

Figure273
(FromKumarV,FaustoN,AbbasA:RobbinsandCotranPathologicBasisofDisease,7thed.Philadelphia,WB
Saunders,2004.)

Figure274
(FromKumarV,FaustoN,AbbasA:RobbinsandCotranPathologicBasisofDisease,7thed.Philadelphia,WB
Saunders,2004.)

Figure275
(FromKumarV,FaustoN,AbbasA:RobbinsandCotranPathologicBasisofDisease,7thed.Philadelphia,WB
Saunders,2004.)

Figure276
(FromKumarV,FaustoN,AbbasA:RobbinsandCotranPathologicBasisofDisease,7thed.Philadelphia,WB
Saunders,2004.)

Figure277
(FromKumarV,FaustoN,AbbasA:RobbinsandCotranPathologicBasisofDisease,7thed.Philadelphia,WB
Saunders,2004.)

Figure278
(FromKumarV,FaustoN,AbbasA:RobbinsandCotranPathologicBasisofDisease,7thed.Philadelphia,WB
Saunders,2004.)

Figure279
(FromKumarV,FaustoN,AbbasA:RobbinsandCotranPathologicBasisofDisease,7thed.Philadelphia,WB
Saunders,2004.)

Figure2710
(FromKumarV,FaustoN,AbbasA:RobbinsandCotranPathologicBasisofDisease,7thed.Philadelphia,WB
Saunders,2004.)

Figure2711
(FromKumarV,FaustoN,AbbasA:RobbinsandCotranPathologicBasisofDisease,7thed.Philadelphia,WB
Saunders,2004.)

Figure2712
(FromKumarV,FaustoN,AbbasA:RobbinsandCotranPathologicBasisofDisease,7thed.Philadelphia,WB
Saunders,2004.)

Figure2713
(FromKumarV,FaustoN,AbbasA:RobbinsandCotranPathologicBasisofDisease,7thed.Philadelphia,WB
Saunders,2004.)

Figure2714
(FromKumarV,FaustoN,AbbasA:RobbinsandCotranPathologicBasisofDisease,7thed.Philadelphia,WB
Saunders,2004.)

Figure2715
(FromKumarV,FaustoN,AbbasA:RobbinsandCotranPathologicBasisofDisease,7thed.Philadelphia,WB
Saunders,2004.)

Figure2716
(FromKumarV,FaustoN,AbbasA:RobbinsandCotranPathologicBasisofDisease,7thed.Philadelphia,WB
Saunders,2004.)

Figure2717
(FromKumarV,FaustoN,AbbasA:RobbinsandCotranPathologicBasisofDisease,7thed.Philadelphia,WB
Saunders,2004.)

Figure2718
(FromKumarV,FaustoN,AbbasA:RobbinsandCotranPathologicBasisofDisease,7thed.Philadelphia,WB
Saunders,2004.)

Figure2719
(FromKumarV,FaustoN,AbbasA:RobbinsandCotranPathologicBasisofDisease,7thed.Philadelphia,WB
Saunders,2004.)

Figure2720
(FromKumarV,FaustoN,AbbasA:RobbinsandCotranPathologicBasisofDisease,7thed.Philadelphia,WB
Saunders,2004.)

Figure2721
(FromKumarV,FaustoN,AbbasA:RobbinsandCotranPathologicBasisofDisease,7thed.Philadelphia,WB
Saunders,2004.)

Figure2722
(FromKumarV,FaustoN,AbbasA:RobbinsandCotranPathologicBasisofDisease,7thed.Philadelphia,WB
Saunders,2004.)

Figure2723
(FromKumarV,FaustoN,AbbasA:RobbinsandCotranPathologicBasisofDisease,7thed.Philadelphia,WB
Saunders,2004.)

Figure2724
(FromKumarV,FaustoN,AbbasA:RobbinsandCotranPathologicBasisofDisease,7thed.Philadelphia,WB
Saunders,2004.)

Figure2725
(FromKumarV,FaustoN,AbbasA:RobbinsandCotranPathologicBasisofDisease,7thed.Philadelphia,WB
Saunders,2004.)

Figure2726
(FromKumarV,FaustoN,AbbasA:RobbinsandCotranPathologicBasisofDisease,7thed.Philadelphia,WB
Saunders,2004.)

Figure2727
(FromKumarV,FaustoN,AbbasA:RobbinsandCotranPathologicBasisofDisease,7thed.Philadelphia,WB
Saunders,2004.)

Figure2728
(FromKumarV,FaustoN,AbbasA:RobbinsandCotranPathologicBasisofDisease,7thed.Philadelphia,WB
Saunders,2004.)

Figure2729
(FromKumarV,FaustoN,AbbasA:RobbinsandCotranPathologicBasisofDisease,7thed.Philadelphia,WB
Saunders,2004.)

Figure2730
(FromKumarV,FaustoN,AbbasA:RobbinsandCotranPathologicBasisofDisease,7thed.Philadelphia,WB

Saunders,2004.)

Figure2731
(FromKumarV,FaustoN,AbbasA:RobbinsandCotranPathologicBasisofDisease,7thed.Philadelphia,WB
Saunders,2004.)

Figure2732
(FromKumarV,FaustoN,AbbasA:RobbinsandCotranPathologicBasisofDisease,7thed.Philadelphia,WB
Saunders,2004.)

Figure2733
(FromKumarV,FaustoN,AbbasA:RobbinsandCotranPathologicBasisofDisease,7thed.Philadelphia,WB
Saunders,2004.)

Figure2734
(FromKumarV,FaustoN,AbbasA:RobbinsandCotranPathologicBasisofDisease,7thed.Philadelphia,WB
Saunders,2004.)

Figure2735
(FromKumarV,FaustoN,AbbasA:RobbinsandCotranPathologicBasisofDisease,7thed.Philadelphia,WB
Saunders,2004.)

Figure2736
(FromKumarV,FaustoN,AbbasA:RobbinsandCotranPathologicBasisofDisease,7thed.Philadelphia,WB
Saunders,2004.)

Figure2737
(FromKumarV,FaustoN,AbbasA:RobbinsandCotranPathologicBasisofDisease,7thed.Philadelphia,WB
Saunders,2004.)

Figure2738
(FromKumarV,FaustoN,AbbasA:RobbinsandCotranPathologicBasisofDisease,7thed.Philadelphia,WB
Saunders,2004.)

Figure2739
(FromKumarV,FaustoN,AbbasA:RobbinsandCotranPathologicBasisofDisease,7thed.Philadelphia,WB
Saunders,2004.)

Figure2740
(FromKumarV,FaustoN,AbbasA:RobbinsandCotranPathologicBasisofDisease,7thed.Philadelphia,WB
Saunders,2004.)

Figure2741
(FromKumarV,FaustoN,AbbasA:RobbinsandCotranPathologicBasisofDisease,7thed.Philadelphia,WB
Saunders,2004.)

Figure2742
(FromKumarV,FaustoN,AbbasA:RobbinsandCotranPathologicBasisofDisease,7thed.Philadelphia,WB
Saunders,2004.)

Figure2743
(FromKumarV,FaustoN,AbbasA:RobbinsandCotranPathologicBasisofDisease,7thed.Philadelphia,WB
Saunders,2004.)

Figure2744
(FromKumarV,FaustoN,AbbasA:RobbinsandCotranPathologicBasisofDisease,7thed.Philadelphia,WB
Saunders,2004.)

Figure2745
(FromKumarV,FaustoN,AbbasA:RobbinsandCotranPathologicBasisofDisease,7thed.Philadelphia,WB
Saunders,2004.)

Figure2746
(FromKumarV,FaustoN,AbbasA:RobbinsandCotranPathologicBasisofDisease,7thed.Philadelphia,WB
Saunders,2004.)

Figure2747
(FromKumarV,FaustoN,AbbasA:RobbinsandCotranPathologicBasisofDisease,7thed.Philadelphia,WB
Saunders,2004.)

Figure2748

(FromKumarV,FaustoN,AbbasA:RobbinsandCotranPathologicBasisofDisease,7thed.Philadelphia,WB
Saunders,2004.)

Figure2749
(FromKumarV,FaustoN,AbbasA:RobbinsandCotranPathologicBasisofDisease,7thed.Philadelphia,WB
Saunders,2004.)

Figure2750
(FromKumarV,FaustoN,AbbasA:RobbinsandCotranPathologicBasisofDisease,7thed.Philadelphia,WB
Saunders,2004.)

Figure2751
(FromKumarV,FaustoN,AbbasA:RobbinsandCotranPathologicBasisofDisease,7thed.Philadelphia,WB
Saunders,2004.)

Copyright2015Elsevier,Inc.Allrightsreserved.

BOOKCHAPTER

ECG
BrandonOlivieriMD,ThomasA.BrownMDandSonaliJ.Shah
USMLEStep1Secrets,Chapter28,821828

SuccessfullylearningtointerpretECGsrequiresthreethings:practice,practice,andpractice!
Fortunately,ECGsontheUSMLEarenotahighyieldtestsubject(and,whentested,are
generallybasic).Readingthroughthisshortchaptershouldbemorethansufficientfortackling
ECGquestionsontheUSMLE.ThemosthighlytestedECGtopicsaredenotedbytheStep1
Secretsembeddedwithinthechapter.ComplexECGinterpretationwillbecomemuchmore
importantduringyourclinicalyears.
Insider'sGuidetoElectrocardiograms(ECGs)fortheUSMLEStep1

1 AnECGrecordedina30yearoldhealthywomanisshowninFigure
281(f0010).HowwouldyouinterpretthisECG?
Figure281(f0010)showsrespiratorysinusarrhythmia,anormalfinding.Typicallytheheartrate

increasesslightlywithinspirationand,duetoincreasedvagaltone,decreasesslightlywith
expiration.Variationofheartratewiththerespiratorycycleisbelievedtoincreasetheefficiencyof
gasexchangebythelungs.

Figure281
Electrocardiogramfrom30yearoldhealthywoman.
(FromGoldbergerAL:ClinicalElectrocardiography:ASimplifiedApproach,7thed.Philadelphia,Mosby,2006.)

2 AnECGrecordedinafebrilesepticpatientisshowninFigure282
(f0015).HowwouldyouinterpretthisECG?
Figure282(f0015)showssinustachycardia,asmightbeexpectedinafebrilesepticpatient.EachQRS

complexisprecededbyaPwave,sotherhythmissinus.Theheartrateisalsoquiteelevatedatclose
to150beats/min,sothisissinustachycardia.

Figure282
Electrocardiogramfromfebrilesepticpatient.
(FromGoldbergerAL:ClinicalElectrocardiography:ASimplifiedApproach,7thed.Philadelphia,Mosby,2006.)

3 AnECGrecordedinahealthymiddleagedmanafterhebecamedizzy
anddiaphoreticwhilehavingblooddrawnisshowninFigure283
(f0020).HowwouldyouinterpretthisECG?
Figure283(f0020)showsaslowheartrate(bradycardia).EachQRScomplexisprecededbyaPwave,

sotherhythmissinus.Bradycardiaischaracterizedbyaheartrateoflessthan50beats/min.A
commoncauseisvagalhyperactivity,asmayoccurwithpainorvomiting,aswellaswithnumerous
medications,suchasbetablockers.

Figure283
Electrocardiogramfrommiddleagedmanafterexperiencingabruptonsetdizziness.
(FromGoldbergerAL:ClinicalElectrocardiography:ASimplifiedApproach,7thed.Philadelphia,Mosby,2006.)

4 ThepatientwhoseECGisshowninFigure284(f0025)willalmost
certainlyremainasymptomaticthroughoutlifebutisatmarginally
increasedriskforsuddencardiacdeath.Howwouldyouinterpretthis
ECG(hint:thearrowisagiveaway)?
WolffParkinsonWhite(WPW)syndromeischaracterizedbyashortenedPRintervalandawidened
QRScomplexwithaslurredupstroke(deltawave).PatientswithWPWsyndromehaveanaccessory
pathwaythroughwhichactionpotentialsmaytravelbetweentheatriaandventriclesthisis
representedbythedeltawaveonFigure284(f0025)(shownbyarrow).Unliketheatrioventricular
(AV)node,thisaccessorypathwayisunabletoslowtransmissionofactionpotentialsfromatriato
ventricles.Asaresult,tachyarrhythmiassuchasatrialfibrillationcancausemarkedincreaseinthe
rateofventricularcontraction.SuchdysrhythmiasinWPWpatientsaredangerousbecausethe
resultingventriculartachycardiacandegenerateintopotentiallyfatalventricularfibrillation.
WolffParkinsonWhite(WPW)syndromeisaUSMLEfavorite.NotallpatientswithWPW
syndromearesymptomatic,buttheconditionmayleadtosupraventriculartachycardia(SVT).
ThedrugofchoicefordiagnosingandtreatingSVTisadenosine.Youshouldknowthe

symptomsofadenosinetoxicity,whichincludechestpain,flushing,hypotension,andgeneral
patientdiscomfort.
Step1Secret

Figure284
Electrocardiograminasymptomaticadult.
(FromFerriF:PracticalGuidetotheCareoftheMedicalPatient,8thed.Philadelphia,Mosby,2011.)

5 TheECGshowninFigure285(f0030)comesfroma52yearoldman
withchronicobstructivepulmonarydisease(COPD)following
admissiontothehospitalwithpneumonia.Howwouldyouinterpretthis
ECG(hint:lookatthepwaves)?
Inmultifocalatrialtachycardia(MAT),numerousectopicfocistimulateirregularatrialcontractionat
arateof100/min,resultinginmorphologicallydistinctPwavesontheECG.MATistypicallyseen
inpatientswithhypoxemiaorchroniclungdisease.

Figure285
ElectrocardiogramfromCOPDpatientadmittedtohospitalwithpneumonia.
(FromGoldbergerAL:ClinicalElectrocardiography:ASimplifiedApproach,7thed.Philadelphia,Mosby,2006.)

6 AnECGrecordedinapatientwithsuddenonsetofpalpitationsand
rapidheartrateisshowninFigure286(f0035).Howwouldyou
interpretthisECG(hint:lookcloselyattherhythm)?
Figure286(f0035)showsatrialfibrillation,characterizedbyarapid,irregularventricularresponseas

wellastheabsenceofPwaves.Atrialfibrillationisoftenseeninpatientswithadilatedleftatrium(as
inmitralstenosisormitralregurgitation).Somepatientsinatrialfibrillationareasymptomatic,

whereasothershaveseveresymptoms,suchaspalpitationsandevensyncope.Patientswithatrial
fibrillationareatincreasedriskforstroke.Thisriskcanbereducedbypatientstakingan
anticoagulantsuchaswarfarin(Coumadin).
Atrialfibrillationisthemostcommoncardiacarrhythmiaandisthusthemosthighlytested
arrhythmiaontheUSMLE.Notonlyshouldyoubeabletorecognizetheappearanceofatrial
fibrillationonECG,butyoushouldknowwhichconditionspredisposetoatrialfibrillation(e.g.,
mitralvalvestenosis,mitralvalveprolapse)andassociatedsymptoms(e.g.,palpitations,chest
pain,congestivesymptoms,exerciseintolerance,potentialstroke).Inaddition,recognizethe
absenceofawavesonvenouspulseandtreatmentsforatrialfibrillation(rateandrhythm
controlplusanticoagulationtopreventstroke).
Step1Secret

Figure286
Electrocardiograminpatientwithsuddenonsetofpalpitationsandrapidheartrate.
(FromGoldbergerAL:ClinicalElectrocardiography:ASimplifiedApproach,7thed.Philadelphia,Mosby,2006.)

7 AnECGina38yearoldwomanexperiencingseverenauseaisshown
inFigure287(f0040).HowwouldyouinterpretthisECG?
NotetheslowrateandprolongedPRinterval(normalPRinterval0.2second).Thisrepresents
firstdegreeAVblock,whichcanoccurwithincreasedvagalactivityinhealthyindividualsaswellas
inelderlypatientswithdysfunctionofthesinoatrialnode.

Figure287
Electrocardiogramfromhealthy38yearoldwomanexperiencingnausea.
(FromGoldbergerAL:ClinicalElectrocardiography:ASimplifiedApproach,7thed.Philadelphia,Mosby,2006.)

8 AnECGrecordedinanasymptomaticmiddleagedmanisshownin
Figure288(f0040).HowwouldyouinterpretthisECG?Doeshe
requireapacemaker?

Figure288(f0045)showsprogressivelengtheningofthePRintervaluntilaQRScomplexis

dropped.ThisECGpatternisreferredtoasMobitztypeI(Wenckebach).PatientswithMobitztype
Itypicallydonotrequireapacemaker.

Figure288
Electrocardiogramfrommiddleagedasymptomaticman.
(FromGoldbergerAL:ClinicalElectrocardiography:ASimplifiedApproach,7thed.Philadelphia,Mosby,2006.)

9 AnECGrecordedinanelderlywomanisshowninFigure289(f0050).
HowwouldyouinterpretthisECG?Doessherequireapacemaker?
Figure289(f0050)showsMobitztypeIIseconddegreeheartblock.UnlikeinMobitztypeI,thePR

intervalsdonotprogressivelyincrease,butthePwavesarenotallconducted,resultinginaregular
patternofdroppedQRScomplexes.PacemakersaretypicallyplacedinpatientswithMobitztypeII.

Figure289
Electrocardiogramfromanelderlywoman.
(FromLimEKS,LokeYK,ThompsonAM:MedicineandSurgery:AnIntegratedTextbook.Philadelphia,Churchill
Livingstone,2007.)

10 AnECGrecordedina74yearoldmanwithrecurrentsyncopeis
showninFigure2810(f0055).HowwouldyouinterpretthisECG?
Wouldhebenefitfromapacemaker?
Figure2810(f0055)showsthirddegree(complete)AVblock.NotehowthePwavesandQRS

complexesareindependentofeachothersothatthePRintervalsarevariableandsomePwavesare
lostbecausetheyfallontheQRScomplexorTwave.Thispatientshouldreceiveapacemaker.
Youshouldbeabletorecognizefirst,second,andthirddegreeatrioventriculat(AV)blockfor
theboards.ItisimportanttoknowthatfirstdegreeAVblockandMobitztypeIdonot
generallyrequiretreatment,butthirddegreeAVblockandMobitztypeII(whichcanprogress
tothirddegreeAVblock)requireimmediatepacemakerplacement.FirstdegreeAVblockis
associatedwithincreasedvagaltoneandcanthusbeaconsequenceofdigitalistoxicity.Third
degreeAVblockmaybeaconsequenceofLymedisease.

Step1Secret

Figure2810
Electrocardiogramfromanelderlymanwithrecurrentsyncope.
(FromGoldbergerAL:ClinicalElectrocardiography:ASimplifiedApproach,7thed.Philadelphia,Mosby,2006.)

11 Figure2811(f0060)showsanECGrecordedina55yearoldmanwith
ahistoryofamassivemyocardialinfarction5yearsearlierwho
experienceschestpainjustbeforecollapsing.Howwouldyouinterpret
thisECG?
Figure2811(f0060)initiallyshowsventriculartachycardia(notethewideQRScomplexes)thatinthe

laterpartoftherhythmstripdegeneratesintoventricularfibrillation.Withoutpromptelectrical
cardioversion,thisisarapidlyfatalrhythm.

Figure2811
Electrocardiogramfromamiddleagedmanwithchestpainwhohasjustcollapsed.
(FromGoldbergerAL:ClinicalElectrocardiography:ASimplifiedApproach,7thed.Philadelphia,Mosby,2006.)

12 AnECGfromamiddleagedmanstartedonprocainamideforatrial
fibrillationisshowninFigure2812(f0065).Howwouldyouinterpret
thisECG?
Figure2812(f0065)showsatypeofventriculartachycardiareferredtoastorsadesdepointes.Note

thetwistingoftheQRScomplexaroundtheisoelectricline.Thedangeroftorsadesisthatitcan
degenerateintoventricularfibrillation.Thispotentiallyfataldysrhythmiacanbetriggeredby
antiarrhythmicssuchasprocainamide,aswiththispatient.
TheUSMLElovestorsadesdepointesbecauseofitsassociationwithclassIaandclassIII
antiarrhythmictoxicity(duetoprolongedQTinterval).Torsadescanalsobetheresultof
variouscongenitalionchannelmutations.JervellandLangeNielsensyndromeisacongenital
longQTsyndromeassociatedwithprofoundhearingloss.
Step1Secret

Figure2812
ElectrocardiogramfromamiddleagedmanstartedontheclassIaantiarrhythmicagentprocainamideforatrial
fibrillation.
(FromGoldbergerAL:ClinicalElectrocardiography:ASimplifiedApproach,7thed.Philadelphia,Mosby,2006.)

13 Figure2813(f0070)showsanECGrecordedina32yearoldotherwise
healthyfemalesmokerwhotakesbirthcontrolpillsandis
experiencingsuddenonsetofdyspnea.Howwouldyouinterpretthis
ECG?
NotetheSIQIIITIIIpattern,inwhichanSwaveisseeninleadI,aQwaveinleadIII,andinvertedT
wavesinleadIII.Thispatternisrelativelyspecificforpulmonaryembolismbutisrarelyseen
becauseofitsextremelylowsensitivity.Nonetheless,it'sapatternyoushouldknowforyourclinical
years.

Figure2813
SIQIIITIIIECGpattern.
(FromGoldbergerAL:ClinicalElectrocardiography:ASimplifiedApproach,7thed.Philadelphia,Mosby,2006.)

14 Figure2814(f0075)showsanECGrecordedina52yearolddiabetic
smokerwhoisexperiencingsuddenonsetofchestpainassociated
withnausea,diaphoresis,andlightheadedness.Afterhewasgiven
somethingbymouth,hisECGisasshowninFigure2814B(f0075).What
washegiven?
Figure2814A(f0075)showsanECGwithmarkedSTsegmentdepressioninleadV4,representing

severesubendocardialischemia.Thepatientwasgivensublingualnitroglycerin,andhissymptoms
resolved.Figure2814B(f0075)showsnormalizationoftheSTsegments.

Figure2814
Electrocardiogramfromamiddleageddiabeticsmokeractivelyexperiencingchestpain.
(FromGoldbergerAL:ClinicalElectrocardiography:ASimplifiedApproach,7thed.Philadelphia,Mosby,2006.)

15 AnECGrecordedina75yearoldmanwithahistoryofcoronary
arterydiseasewhoisexperiencingseveresubsternalchestpressureis
showninFigure2815(f0080).HowwouldyouinterpretthisECG?
Figure2815(f0080)showsmarkedSTsegmentelevationintheanterior(V1V3)andlateral(V4V5

)leads,indicatinganacuteanterolateralmyocardialinfarction.

Figure2815
Electrocardiogramfromanelderlymanwithahistoryofcoronaryarterydiseasewhoisactivelyexperiencing
chestpain.
(FromGoldmanL,AusielloDA:CecilTextbookofMedicine,23rded.Philadelphia,WBSaunders,2007.)

16 Figure2816(f0085)showsanECGfroma42yearoldobesemanwho
isexperiencingseverechestpain.HowwouldyouinterpretthisECG?
Forbonuspoints,whicharteryisaffected?
Figure2816(f0085)showsmarkedSTsegmentelevationintheinferiorleads(II,III,aVF),indicating

aninferiorwalltransmuralmyocardialinfarction.Theinferiorwallistypicallysuppliedbytheright
coronaryartery(RCA).
BeabletodeterminewhichcoronaryarteryisoccludedbasedonSTsegmentelevationsin
variousECGleads.Youmayalsobeaskedtoidentifythiscoronaryarteryonanangiogram,so
practicelookingattheseimagesinananatomyatlas.
Step1Secret

Figure2816
Electrocardiogramfromanobesemiddleagedmanactivelyexperiencingchestpain.
(FromRakelRE:TextbookofFamilyMedicine,7thed.Philadelphia,WBSaunders,2007.)

8 AnECGrecordedinanasymptomaticmiddleagedmanisshownin
Figure288(f0040).HowwouldyouinterpretthisECG?Doeshe
requireapacemaker?
Figure288(f0045)showsprogressivelengtheningofthePRintervaluntilaQRScomplexis

dropped.ThisECGpatternisreferredtoasMobitztypeI(Wenckebach).PatientswithMobitztype
Itypicallydonotrequireapacemaker.

Figure288
Electrocardiogramfrommiddleagedasymptomaticman.
(FromGoldbergerAL:ClinicalElectrocardiography:ASimplifiedApproach,7thed.Philadelphia,Mosby,2006.)

9 AnECGrecordedinanelderlywomanisshowninFigure289(f0050).
HowwouldyouinterpretthisECG?Doessherequireapacemaker?
Figure289(f0050)showsMobitztypeIIseconddegreeheartblock.UnlikeinMobitztypeI,thePR

intervalsdonotprogressivelyincrease,butthePwavesarenotallconducted,resultinginaregular
patternofdroppedQRScomplexes.PacemakersaretypicallyplacedinpatientswithMobitztypeII.

Figure289
Electrocardiogramfromanelderlywoman.
(FromLimEKS,LokeYK,ThompsonAM:MedicineandSurgery:AnIntegratedTextbook.Philadelphia,Churchill
Livingstone,2007.)

10 AnECGrecordedina74yearoldmanwithrecurrentsyncopeis
showninFigure2810(f0055).HowwouldyouinterpretthisECG?
Wouldhebenefitfromapacemaker?
Figure2810(f0055)showsthirddegree(complete)AVblock.NotehowthePwavesandQRS

complexesareindependentofeachothersothatthePRintervalsarevariableandsomePwavesare
lostbecausetheyfallontheQRScomplexorTwave.Thispatientshouldreceiveapacemaker.
Youshouldbeabletorecognizefirst,second,andthirddegreeatrioventriculat(AV)blockfor
theboards.ItisimportanttoknowthatfirstdegreeAVblockandMobitztypeIdonot
generallyrequiretreatment,butthirddegreeAVblockandMobitztypeII(whichcanprogress
tothirddegreeAVblock)requireimmediatepacemakerplacement.FirstdegreeAVblockis
associatedwithincreasedvagaltoneandcanthusbeaconsequenceofdigitalistoxicity.Third
degreeAVblockmaybeaconsequenceofLymedisease.
Step1Secret

Figure2810
Electrocardiogramfromanelderlymanwithrecurrentsyncope.
(FromGoldbergerAL:ClinicalElectrocardiography:ASimplifiedApproach,7thed.Philadelphia,Mosby,2006.)

11 Figure2811(f0060)showsanECGrecordedina55yearoldmanwith
ahistoryofamassivemyocardialinfarction5yearsearlierwho
experienceschestpainjustbeforecollapsing.Howwouldyouinterpret
thisECG?
Figure2811(f0060)initiallyshowsventriculartachycardia(notethewideQRScomplexes)thatinthe

laterpartoftherhythmstripdegeneratesintoventricularfibrillation.Withoutpromptelectrical
cardioversion,thisisarapidlyfatalrhythm.

Figure2811
Electrocardiogramfromamiddleagedmanwithchestpainwhohasjustcollapsed.
(FromGoldbergerAL:ClinicalElectrocardiography:ASimplifiedApproach,7thed.Philadelphia,Mosby,2006.)

12 AnECGfromamiddleagedmanstartedonprocainamideforatrial
fibrillationisshowninFigure2812(f0065).Howwouldyouinterpret
thisECG?

Figure2812(f0065)showsatypeofventriculartachycardiareferredtoastorsadesdepointes.Note

thetwistingoftheQRScomplexaroundtheisoelectricline.Thedangeroftorsadesisthatitcan
degenerateintoventricularfibrillation.Thispotentiallyfataldysrhythmiacanbetriggeredby
antiarrhythmicssuchasprocainamide,aswiththispatient.
TheUSMLElovestorsadesdepointesbecauseofitsassociationwithclassIaandclassIII
antiarrhythmictoxicity(duetoprolongedQTinterval).Torsadescanalsobetheresultof
variouscongenitalionchannelmutations.JervellandLangeNielsensyndromeisacongenital
longQTsyndromeassociatedwithprofoundhearingloss.
Step1Secret

Figure2812
ElectrocardiogramfromamiddleagedmanstartedontheclassIaantiarrhythmicagentprocainamideforatrial
fibrillation.
(FromGoldbergerAL:ClinicalElectrocardiography:ASimplifiedApproach,7thed.Philadelphia,Mosby,2006.)

13 Figure2813(f0070)showsanECGrecordedina32yearoldotherwise
healthyfemalesmokerwhotakesbirthcontrolpillsandis
experiencingsuddenonsetofdyspnea.Howwouldyouinterpretthis
ECG?
NotetheSIQIIITIIIpattern,inwhichanSwaveisseeninleadI,aQwaveinleadIII,andinvertedT
wavesinleadIII.Thispatternisrelativelyspecificforpulmonaryembolismbutisrarelyseen
becauseofitsextremelylowsensitivity.Nonetheless,it'sapatternyoushouldknowforyourclinical
years.

Figure2813

SIQIIITIIIECGpattern.
(FromGoldbergerAL:ClinicalElectrocardiography:ASimplifiedApproach,7thed.Philadelphia,Mosby,2006.)

14 Figure2814(f0075)showsanECGrecordedina52yearolddiabetic
smokerwhoisexperiencingsuddenonsetofchestpainassociated
withnausea,diaphoresis,andlightheadedness.Afterhewasgiven
somethingbymouth,hisECGisasshowninFigure2814B(f0075).What
washegiven?
Figure2814A(f0075)showsanECGwithmarkedSTsegmentdepressioninleadV4,representing

severesubendocardialischemia.Thepatientwasgivensublingualnitroglycerin,andhissymptoms
resolved.Figure2814B(f0075)showsnormalizationoftheSTsegments.

Figure2814
Electrocardiogramfromamiddleageddiabeticsmokeractivelyexperiencingchestpain.
(FromGoldbergerAL:ClinicalElectrocardiography:ASimplifiedApproach,7thed.Philadelphia,Mosby,2006.)

15 AnECGrecordedina75yearoldmanwithahistoryofcoronary
arterydiseasewhoisexperiencingseveresubsternalchestpressureis
showninFigure2815(f0080).HowwouldyouinterpretthisECG?
Figure2815(f0080)showsmarkedSTsegmentelevationintheanterior(V1V3)andlateral(V4V5

)leads,indicatinganacuteanterolateralmyocardialinfarction.

Figure2815
Electrocardiogramfromanelderlymanwithahistoryofcoronaryarterydiseasewhoisactivelyexperiencing
chestpain.
(FromGoldmanL,AusielloDA:CecilTextbookofMedicine,23rded.Philadelphia,WBSaunders,2007.)

16 Figure2816(f0085)showsanECGfroma42yearoldobesemanwho
isexperiencingseverechestpain.HowwouldyouinterpretthisECG?
Forbonuspoints,whicharteryisaffected?
Figure2816(f0085)showsmarkedSTsegmentelevationintheinferiorleads(II,III,aVF),indicating

aninferiorwalltransmuralmyocardialinfarction.Theinferiorwallistypicallysuppliedbytheright
coronaryartery(RCA).
BeabletodeterminewhichcoronaryarteryisoccludedbasedonSTsegmentelevationsin
variousECGleads.Youmayalsobeaskedtoidentifythiscoronaryarteryonanangiogram,so
practicelookingattheseimagesinananatomyatlas.
Step1Secret

Figure2816
Electrocardiogramfromanobesemiddleagedmanactivelyexperiencingchestpain.
(FromRakelRE:TextbookofFamilyMedicine,7thed.Philadelphia,WBSaunders,2007.)

Copyright2015Elsevier,Inc.Allrightsreserved.

BOOKCHAPTER

Endocrinology
SonaliJ.Shah,ThomasA.BrownMD,AnnaRadwanandHenryL.Nguyen
USMLEStep1Secrets,Chapter8,196251

EndocrinologyontheUSMLEtestsyourabilitytoreasonthroughcomplexappearingproblems
andisthereforeafavoritesubjectamongexaminers.Luckily,theseproblemsbecomequite
straightforwardwithabitofpractice.Thischapterisaimedatgivingyouthispractice.The
mostimportantsecrettodoingwellonStep1endocrinologyquestionsistounderstand
hormonalpathwaysandtheconceptofnegativefeedback.Thisinformationalonewillhelpyou
answeralargenumberofendocrinologyquestionsonboardexaminations.Studythecasesin
thischapterandtestyourselfbydrawingouthormonalfeedbackloopswheneverpossible.
Insider'sGuidetoEndocrinologyfortheUSMLEStep1

Basicconcepts
1 Whatisthecellularmechanismofactionofthesteroidhormones?
Steroidhormonesarelipophilic.Therefore,theydiffuseacrosstheplasmamembraneandform
complexeswithcytosolicornuclearreceptorstheboundcomplexesthenactivatetranscriptionof
variousgenes.Becausesteroidhormonesrelyontheintermediaryprocessofgeneexpressionand
proteintranslation,itcantakehourstodaysfortheireffectstomanifest.Examplesofsteroid
hormonesaretestosterone,estrogen,progesterone,cortisol,andaldosterone.Cholesterolisthe
precursortoallsteroidhormones.Althoughthyroidhormoneisnotasteroidhormone,itusesthe
samecellularmechanismasthesteroids(Fig.81(f0010)).

Figure81
Mechanismsbywhichpeptideandsteroidhormonessignal.Ca2+,calciumcAMP,cyclicadenosine
monophosphateDAG,diacylglycerolmRNA,messengerRNATyrP,phosphorylatedtyrosineresidue.
(FromGoldmanL,AusielloD:CecilTextbookofMedicine,22nded.Philadelphia,WBSaunders,2004.)

2 Whatisthecellularmechanismofactionofthepeptidehormonesandthe
catecholamines?
Thepeptidehormonesandcatecholaminesarenothighlylipiddiffusibleandthuscannotcrossthe
plasmamembrane.Theybindtocellsurfacereceptors(seeFig.81(f0010)),whichinitiateavarietyof
biochemicalevents,includingactivationorinhibitionofenzymes,alterationofmembraneproteins,
andmediationofcellulartrafficking.Theseprocessescanoccurwithinsecondstominutes.
Nevertheless,thepeptidehormonescanstimulategeneexpressionaswell,andthiseffectisdelayed
asitiswiththesteroidhormones.Examplesofpeptidehormonesareinsulin,parathyroidhormone
(PTH),vasopressin(antidiuretichormone),andoxytocin.Table81(t0010)showsacomparisonof
polypeptideandsteroidhormones.
Table81
ComparisonoftheDifferentTypesofHormones

Feature

Polypeptides

ModifiedAminoAcids

Steroids

Size

Mediumlarge

Verysmall

Small

Abilitytocrosscell
membrane

No

Yes

Yes

Receptortype

Cellsurface

Cellsurfaceorintracellular

Intracellular

Solubility

Water

Water

Fat

Action

Protein
activation

Proteinactivationorsynthesis

Proteinsynthesis

Transportinthe
blood

Dissolvedinthe
plasma

Dissolvedintheplasmaorboundto
plasmaproteins

Boundtoplasma
proteins

FromMeszarosJG,OlsonER,NaugleJE,etal:CrashCourse:EndocrineandReproductiveSystems.
Philadelphia,Mosby,2006.

3 Whyisthetotalserumhormonelevelnotanaccuratereflectionofhormone
activity?
Manyofthehormonesintheserumareinactivebecausetheyareattachedtoserumbindingproteins.
Itisonlythefreehormonethatisbiologicallyactive.Anotherimportantfactorthataffectshormone
activityistheconcentrationofcellularhormonereceptorsavailableforbindingaspecifichormone
andmediatingitsaction.
Note:Freehormoneisinequilibriumwithboundhormone:
(Free hormone) + (Binding protein) (Hormone-binding protein complex)

4 Howdoesahormone'sbindingtothesametypeofreceptorhavedifferent
effectsindifferentcelltypes?
Differenttissuesaredifferentbecausetheyexpressdifferentgenes(i.e.,differentialtranscription).
Therefore,althoughdifferenttargettissuesmayexpressthesamehormonereceptor,theymayalso
expressentirelydifferentdownstreamproteintargets.Forexample,bybindingthesame2
adrenergicreceptors,epinephrineisabletoelicitahostofdifferentphysiologicresponsesdepending
onthetissuelocationofthatreceptor(e.g.,glycogenolysisintheliver,lipolysisinadiposetissue).

5 Whatarethefourprimaryclassesofmembranespanningreceptorstowhich
peptidehormonesbind?
Thefourprimaryclassesofmembranespanningreceptorstowhichpeptidehormonesbindare(1)
tyrosineandserinekinasereceptors,(2)receptorlinkedkinases,(3)Gproteincoupledreceptors,
and(4)ligandgatedionchannels(Fig.82(f0015)andTable82(t0015)).Asagrosssimplification,
theprototypicalagonistsforthesereceptortypescanbeconsideredtobegrowthfactors,growth
hormones(GHs),peptidehormones,andneurotransmitters,respectively.
ClassesofreceptorsusedbyvarioushormonesarecommonlytestedonStep1.Youshould
memorizetheinformationlistedinTable82(t0015).
Step1Secret

Figure82
Thefourmajorclassesofmembranereceptorsforhormonesandneurotransmitters.cAMP,cyclicadenosine
monophosphateE,effectorenzymeG,GproteinIP3,inositoltriphosphateR,receptor.
(FromBrownTA,BrownD:USMLEStep1Secrets.Philadelphia,Hanley&Belfus,2004.)
Table82
ClassesofReceptorsUsedbyVariousHormones

Receptor
Class

HormonesandRelatedSubstances

cAMP

LH,FSH,ACTH,TSH,PTH,hCG,CRH,glucagon

cGMP

NO,ANP

IP3

GnRH,GHRH,oxytocin,TRH

Steroid
receptor

Estrogen,testosterone,glucocorticoids,vitaminD,aldosterone,progesterone,T3

Tyrosine
kinase

Insulin,growthfactors(e.g.,IGF,PDGF),GH,prolactin

/T4

ACTH,adrenocorticotropichormoneANP,atrialnatriureticpeptidecAMP,cyclicadenosinemonophosphate
cGMP,cyclicguanosinemonophosphateCRH,corticotropinreleasinghormoneFSH,folliclestimulating
hormoneGH,growthhormoneGHRH,growthhormonereleasinghormoneGnRH,gonadotropinreleasing
hormonehCG,humanchorionicgonadotropinIGF,insulinlikegrowthfactorIP3,inositoltriphosphateNO,
nitricoxidePDGF,plateletderivedgrowthfactorPTH,parathyroidhormoneT3,triiodothyronineT4,
thyroxineTRH,thyrotropinreleasinghormoneTSH,thyroidstimulatinghormone.

6 Howdothetyrosinekinasereceptorstransducetheirmessages?
AsdepictedinFigure83(f0020),bindingofpeptidehormonetotheextracellulardomainofthe
receptorinitiatesasignaltransductioncascadebypromotingautophosphorylationofthekinase
receptorandsubsequentphosphorylationofdownstreamtargetproteins,therebyactivatingor
inhibitingtheseproteins.

Figure83
Mechanismofactivationofatyrosinekinasereceptor.
(FromMeszarosJG,OlsonER,NaugleJE,etal:CrashCourse:EndocrineandReproductiveSystems.
Philadelphia,Mosby,2006.)

7 Howdotheligandgatedionchannelswork?
Activationofligandgatedionchannelsresultsinaninflux(orefflux)ofionsinto(oroutof)thecell.
Thenicotinicreceptoronskeletalmuscleisanexampleofsuchareceptor.Bindingofacetylcholine
tothisreceptorresultsinaninfluxofprincipallysodiumionsintothecell.

8 HowdotheGproteinstransducetheirsignals?
Bindingofhormone/agonisttoGproteincoupledreceptorscausesansubunitcomplexto
exchangeguanosinediphosphate(GDP)forguanosinetriphosphate(GTP).OnceGTPisboundtothe
subunitcomplex,itdissociatesintothesubunitandaseparatesubunit.Thesedissociated
subunitsthenactivateorinhibitenzymes(adenylatecyclase,phospholipase)andionchannels(Ca2+
channels)(Fig.84(f0025)).

Figure84
SignaltransductionbyGproteins.ARK,adrenergicreceptorkinaseGAP,GTPaseactivatingproteinGDP,
guanosinediphosphateGEF,guaninenucleotideexchangefactorGTP,guanosinetriphosphatePi,inorganic
phosphate,PI3K,phosphatidylinositol3kinasePKC,proteinkinaseCPLC,phospholipaseCRaf1,effector
proteinRalGDS,RasrelatedGTPaseguaninenucleotidedissociationstimulatorRTK,receptortyrosinekinase.
(FromMannDL:HeartFailure:ACompaniontoBraunwald'sHeartDisease.Philadelphia,WBSaunders,2004.)

Note:Adenylatecyclasesynthesizescyclicadenosinemonophosphate(cAMP)fromadenosine
triphosphate(ATP),andthecAMPactivatesvarioustargetproteins.Gsreceptorsstimulateadenylate
cyclase,whereasGireceptorsinhibitadenylatecyclase.
Gproteinsareafivestartopiconboards.StudythepathwaysforGs,Gi,andGqsignaling
depictedinFigure84(f0025).Youshouldknowthedetailsofthesepathways,includingthe
predominantcellularchangesthatoccurwithactivationofeachprotein(e.g.,cyclicadenosine
monophosphate[cAMP]increasewithGsactivation,intracellular[Ca2+]increasewithGq
activation).
Step1Secret

9 Howdoendocrine,paracrine,andautocrinemechanismsofcell
communicationdiffer?
Endocrinesecretions(i.e.,hormones)affecttheirtargetorgansatconsiderabledistancefromtheir
siteofsecretion,sotheymustbecarriedbythebloodstream.Paracrinesecretionsactlocallyon
adjacentcellsandtissues.Paracrinecommunicationseemsparticularlyimportantinendocrine
tissues,suchasthepancreaticislets,whereconstantcommunicationbetweenadjacentcells(e.g.,
andcells)iscriticalforoptimalfunctioning.Autocrinesecretionsaresecretionsfromacellthat
bindtoreceptorsonthatsamecellandexertregulatoryactionsonthatcell.Neuroendocrine

secretionsinvolvethesecretionofpeptidesintothebloodfromspecializedneurons(hencetheterm
neuroendocrine).Hypothalamicpeptidesreleasedintothebloodfromtheterminalboutonsofaxons
locatedintheposteriorpituitaryareoneexampleofthismechanismofregulation(Fig.85(f0030)).

Figure85
Theroutesbywhichchemicalsignalsaredeliveredtocells.
(FromMeszarosJG,OlsonER,NaugleJE,etal:CrashCourse:EndocrineandReproductiveSystems.
Philadelphia,Mosby,2006.)

10 Describetheconceptofnegativefeedback.Whatisafeedbackloop?
Hormonesynthesisandreleasearegovernedatmultiplelevels.Hormonesynthesis/releasefroman
organofinteresttypicallyinvolvesregulationbyapituitaryhormone,whichitselfisregulatedbya
hypothalamichormone.Thisgeneralpathwaystructureiscommonlyreferredtoasahypothalamic
pituitary(organ)axis(e.g.,HPOaxisreferstotheovary,HPAaxisreferstotheadrenalgland).
Negativefeedbackoccurswhenaproductdownstreamofanaxisinhibitsproductionofareactantby
whichitisregulatedforexample,thyroidhormoneinhibitionofthyroidstimulatinghormone
(TSH).Theserelationshipsareoftendepictedusingfeedbackloops.Anexampleofthethyroid
hormonefeedbackloopisdepictedinFigure86(f0035).

Case81
A38yearoldwomandevelopedamassivepostpartumhemorrhage,forwhichshewas
eventuallystabilizedwithmultiplebloodtransfusions.Afewweekslatershecomplainsthatshe
hasnotbeenabletolactatesincedeliveringherbaby.Shealsofeelslethargicandweak,and
oftengetsdizzyuponstanding.Physicalexaminationisunremarkableexceptforsparseaxillary
andpubichair,andherpulserateincreasesby20beats/minuponstandingfromasupine

position.Injectionofcorticotropinreleasinghormone(CRH)causesonlyabluntedelevationof
serumadrenocorticotropichormone(ACTH)level.Similarly,injectionofagonadotropin
releasinghormone(GnRH)analogcausesonlyabluntedelevationoffolliclestimulating
hormone(FSH)andluteinizinghormone(LH).Serumprolactinconcentrationisabnormally
low.

Figure86
Thyroidhormonephysiology.Undernormalconditions,thyrotropinreleasinghormone(TRH)isreleasedfrom
thehypothalamus,whichcausesthereleaseofthyroidstimulatinghormone(TSH)fromtheanteriorpituitary.
TSHthencausesthereleaseoftriiodothyronine(T3)andthyroxine(T4)bythethyroidgland,whichacton
variousperipheralorgans.T3andT4alsoregulateTRHandTSHsecretionthroughnegativefeedback
mechanisms.Inprimaryhypothyroidism(duetothyroiddysfunction),T3andT4arelowdespitehighTSHlevels.
Insecondaryhypothyroidism(duetopituitaryorhypothalamicdysfunction),thelevelsofTSH,T3,andT4are
alllow.
(FromGoldmanL,AusielloD:CecilTextbookofMedicine,22nded.Philadelphia,WBSaunders,2004.)

1 Whatisthediagnosis?
ShehasSheehan'ssyndrome(orpostpartumnecrosis),whichisaninfarctionoftheanterior
pituitary.

2 Whyisthepituitarymoresusceptibletoinfarctioninpostpartumhemorrhage
thaninhemorrhagicshockunrelatedtopregnancy?

Duringpregnancy,thereishyperplasiaofthelactotrophs(prolactinsecretingcells)intheanterior
pituitary(adenohypophysis)withoutproportionalincreaseinbloodsupply,whichincreasesthis
tissue'sminimalperfusionneeds.Duringpostpartumhemorrhage,bloodsupplytotheanterior
pituitarycanbecomesufficientlyinadequatetomeetthisincreasedneed,causinginfarction.

3 WhyistheposteriorpituitarytypicallysparedinSheehan'ssyndrome?
Theposteriorpituitary(neurohypophysis)differsinembryologicoriginfromtheanteriorpituitary
andthereforehasadifferentbloodsupply.Remember,theembryologicoriginoftheanterior
pituitaryisRathke'spouch(anendodermalevaginationfromtheroofofthemouth),whereasthe
posteriorpituitaryisderivedfromaventraloutgrowthfromtheprimitivehypothalamus.Table83
(t0020)showsareviewofthefunctionsoftheposteriorpituitaryhormones.
Table83
HormonesSecretedbythePosteriorPituitaryandtheirEffects

Hormone

Synthesizedby

Antidiuretic Supraoptic
hormone
(ADH)

Stimulatedby

Inhibited
by

Target
Organ

Effect

Raisedosmolarity

Lower

Kidney

Increases

vasopressinergic lowbloodvolume
neurons

osmolarity

permeabilityofthe
collectingductto
reabsorbwater

Oxytocin

Paraventricular

Stretchreceptorsin

oxytocinergic
neurons

thenippleand
cervix,estrogen

Stress

Uterus

Smoothmuscle

and
contractionleadingto
mammary birthormilkejection
glands

FromMeszarosJG,OlsonER,NaugleJE,etal:CrashCourse:EndocrineandReproductiveSystems.
Philadelphia,Mosby,2006.

4 Secretionofwhichpituitaryhormonesmaybeaffectedinthiswoman?
ThehormonessecretedbytheanteriorpituitaryincludeFSH,LH,ACTH,TSH,prolactin,andGH.
FLATP(i)Gisausefulmnemonictorememberthesehormones.
Dependingontheextentoftheinfarction,alltheanteriorpituitaryhormonesmaybeaffected(Fig.8
7(f0040)).Thispatientisunabletolactate,whichisconsistentwithdecreasedprolactinsecretion.

(Wheneveryouseeapatientwhoisunabletolactateshortlyafterdelivery,considerSheehan's
syndrome!)Notethatheraxillaryandpubichairissparse,whichisconsistentwithdecreased
gonadotropin(FSH,LH)secretion.Thepatientisalsoweakandlethargic,whichisconsistentwith
hypocortisolismduetodecreasedACTHsecretion.

Figure87
Hormonesoftheanteriorpituitaryglandandtheirrespectivetargetorgans.ACTH,adrenocorticotropichormone
FSH,folliclestimulatinghormoneGH,growthhormoneLH,luteinizinghormoneTSH,thyroidstimulating
hormone.
(FromMeszarosJG,OlsonER,NaugleJE,etal:CrashCourse:EndocrineandReproductiveSystems.
Philadelphia,Mosby,2006.)

5 Whymayhypothalamicreleasinghormonesecretionincreasebecauseofan
infarctionoftheanteriorpituitary?
Thelossofpituitaryhormonesecretionwilldecreasenegativefeedbackonthehypothalamic
hormonesbothfromlowpituitaryhormonelevelsandfromlowtargetorganhormoneproduction.
Table84(t0025)reviewsthehypothalamicreleasinghormones.
Table84
HormonesSynthesizedandSecretedbytheAnteriorPituitaryandtheirEffects

Hormone

GH

TSH

Synthesized

Stimulated Inhibitedby

by

by

Somatotrophs GHRH

Thyrotrophs

TRH

Targetorgan

Effect

GHIHandIGF Liver

StimulatesIGF1

productionand
opposesinsulin

T3

Thyroidgland

Stimulatesthyroxine

release
ACTH

Corticotrophs

CRH

Glucocorticoids Adrenalcortex

Stimulates
glucocorticoidand
androgenrelease

LH+FSH Gonadotrophs GnRH,

Prolactin

Lactotrophs

Prolactin,sex

Reproductive

Releaseofsex

sex
steroids

steroids

organs

steroids

PRHand

Dopamine

Mammaryglands Promotesgrowthof

TRH

andreproductive theseorgansand
organs
initiateslactation

MSH

Corticotrophs

Melanocytesin
skin

Stimulatesmelanin
synthesis

Beta

Corticotrophs

Unknown

Maybeinvolvedin

endorphin

paincontrol

ACTH,adrenocorticotropichormoneCRH,corticotropinreleasinghormoneFSH,folliclestimulatinghormone
GH,growthhormoneGHIH,growthhormoneinhibitinghormoneGHRH,growthhormonereleasinghormone
IGF,insulinlikegrowthfactorLH,luteinizinghormoneMSH,melanocytestimulatinghormonePRH,prolactin
releasinghormoneT3,triiodothyronineTRH,thyrotropinreleasinghormoneTSH,thyroidstimulating
hormone.
FromMeszarosJG,OlsonER,NaugleJE,etal:CrashCourse:EndocrineandReproductiveSystems.
Philadelphia,Mosby,2006.

6 Whywouldn'thypothalamicdopaminesecretionbeelevatedfromananterior
pituitaryinfarction?
Incontrastwiththeotherhypothalamichormones,dopamineisinhibitoryinnatureandreducesthe
secretionofprolactinbythelactotrophs.Becauseserumprolactinnormallystimulatesdopamine
secretionfromthehypothalamus,inasettingofhypoprolactinemiasecondarytopituitary
infarction,wewouldexpectdecreasedsecretionofhypothalamicdopamine.

PostpartumnecrosisoftheanteriorpituitaryistermedSheehan'ssyndrome.
Anteriorpituitaryhormonesincludefolliclestimulatinghormone(FSH),luteinizing
hormone(LH),adrenocorticotropichormone(ACTH),thyroidstimulatinghormone
(TSH),growthhormone(GH),andprolactin.(RememberFLATP(i)Ganddisregardthe
i.)
Theposteriorpituitarysecretesvasopressin(antidiuretichormone[ADH])andoxytocin.

TheembryologicoriginoftheanteriorpituitaryisRathke'spouch.
Theposteriorpituitaryisderivedfromaventraloutgrowthoftheprimitive
hypothalamus.
SummaryBox:Sheehan'sSyndrome

Case82
A32yearoldwomancomplainsofrecentvisualproblemsandslightbreastdischarge
(galactorrhea).Shehasnothadherperiodforthepast6months(secondaryamenorrhea)and
isupsetthatshehasbeenunabletobecomepregnant,despitetryingforthepastyearwithher
husband.Shedeniesanyhistoryofschizophreniaorofbeingtreatedwithneuroleptics
(antipsychotics).Laboratoryworkuprevealsanegativepregnancytestresult,normalTSHlevel,
andsignificantlyelevatedlevelsofprolactin.Magneticresonanceimaging(MRI)ofthehead
showsenlargementofthestructurelocatedinthesellaturcica.

1 Whatisthediagnosis?
Prolactinomaisapituitaryadenomacausedbyabnormalproliferationoflactotrophsitisthemost
commontypeofhypersecretingpituitaryadenoma.Notethatthepituitaryissituatedinthesella
turcica.Table85(t0030)reviewspituitaryadenomas.
Table85
DisordersCausedbytheDeficiencyorExcessofAnteriorPituitaryHormones

Hormone(s) Deficiency

Excess

GH

Gigantisminchildren,acromegalyinadults

Dwarfisminchildrenoradults
GHdeficiencysyndrome

LHand
FSH

Gonadalinsufficiency(decreasedsex Extremelyrarebutcausesinfertility
steroids)

ACTH

Adrenocorticalinsufficiency
(decreasedcortisolandadrenal

Cushingdisease(increasedcortisoland
adrenalandrogens)

androgens)
TSH

Prolactin

Hypothyroidism(decreasedthyroid

Extremelyrarebutcauseshyperthyroidism

hormones)

(increasedthyroidhormones)

Hypoprolactinemia(failurein

Hyperprolactinemia(impotenceinmales,

postpartumlactation)

amenorrheainfemales,anddecreasedlibido)

ACTH,adrenocorticotropichormoneFSH,folliclestimulatinghormoneGH,growthhormoneLH,luteinizing

hormoneTSH,thyroidstimulatinghormone.
FromMeszarosJG,OlsonER,NaugleJE,etal:CrashCourse:EndocrineandReproductiveSystems.
Philadelphia,Mosby,2006.

2 Whatarethenormalphysiologicfunctionsofprolactinpreceding,during,and
followingpregnancy?
Precedingpregnancy:Prolactinlevelsarenormalduetotonichypothalamicinhibitionvia
dopamineandtotheabsenceofstimulatoryfactorssuchassucklingorhighestrogen.Ithas
numerousphysiologicfunctionsincountlessorgansystemsinthenonpregnantwoman,none
ofwhicharehighyieldforboards.
Duringpregnancy:Prolactinlevelsarehighsecondarytohighestrogenlevels(secretedby
theplacenta),whichstimulatebreastmaturationandlactogenesis.However,actuallactationis
preventedbyhighestrogenandprogesterone(whichantagonizeactionsofprolactinonthe
breast).
Followingpregnancy:Estrogenlevelsdrop,andprolactinlevelsalsowilldropunless
stimulationbysucklingoccurslevelsincreaseandlactationoccurswithsucklingstimulation.It
isimportantforyoutoknowthatprolactinwillalsoinhibitGnRHsecretion,often(butnot
always!)resultinginanovulatoryinfertilitywhilenursing.
Figure88(f0045)outlinestheotherphysiologicfunctionsofprolactin.

Figure88
Physiologicactionsofprolactin.TRH,thyrotropinreleasinghormone.
(FromBrownTA:RapidReviewPhysiology.Philadelphia,Mosby,2007.)

3 Whydoesthispatienthavegalactorrhea,whereaspregnantwomenwith
similarlevelsofserumprolactingenerallydonothavethisproblem?

Althoughprolactinstimulatesmilkproduction,thehighconcentrationsofestrogenandprogesterone
thatarepresentduringpregnancyinhibitlactation,andtherefore,galactorrhea.Incontrast,this
patienthashyperprolactinemiaintheabsenceofelevatedlevelsofestrogenandprogesterone,which
iscausinghergalactorrhea.
Note:Milkletdownoccursafterchildbirthbecause,duringpregnancy,theplacentamakesmostof
theestrogenandprogesterone.Levelsofbothhormonesdecreaseafterthisstructureisexpelledin
delivery.Additionally,oxytocinissecretedinresponsetosuckling,andthishormonestimulates
contractionofmyoepithelialcellsaroundtheglandulartissueofthebreast,causingmilkejection.

4 Hyperprolactinemiacanalsooccurinmen.Whatsymptomsmightbe
expectedinmen?
Inmen,inhibitionofGnRHsecretionbyprolactindecreasesgonadotropinmediatedtestosterone,
productionandlowtestosteronemayleadtoerectiledysfunction(impotence)andthelossofsex
drive(libido).Galactorrheacanalsorarelyoccurinmeninresponsetocertainstimulisuchassevere
stressandprolongedstarvation.Ofnote,besuspiciousofhyperprolactinemiainmenpresentingwith
malaiseanddepression.Thiscorrelationisaboardfavorite.

5 Howdoeselevatedprolactinpreventpregnancy(i.e.,whatisthemechanismof
infertilityandamenorrheainthispatient)?
ProlactininhibitsthehypothalamicreleaseofGnRH,whichisastimulusforFSHandLHsecretion.
TheconsequentreductionofFSHandLHeliminatestheovulatorycycle,resultingininfertilityand
amenorrhea.Notethathyperprolactinemiainmencancauseimpotenceandinfertilitythrougha
similarmechanism,exceptinthiscasetestosteroneisloweredasaresultofthedecreasedLH.

6 Whyisaskingaboutahistoryofschizophreniaanduseofantipsychotic
medicationsarelevantquestioninthediagnosticworkupofthispatient?
Severalantipsychotics(particularlythetypicalantipsychotics)cancausehyperprolactinemia.Thisis
notafactyouhavetomemorizeifyousimplyrecallthatantipsychoticsaredopamineantagonists,
andthathypothalamicdopamineisthemajorinhibitorofpituitaryprolactinsecretion.TheUSMLE
lovesthesetypesofcorrelations!

7 Whatisthemechanisticbasisforusingbromocriptine(usedtotreat
Parkinson'sdisease)inthetreatmentofaprolactinoma?
Bromocriptineisadopamineagonist(recallthatParkinson'sdiseaseiscausedbyalackof
dopamine)thatinhibitsprolactinsecretionbytheanteriorpituitary.

8 Howcanheadtraumawithaseveredpituitarystalkcauseasimilarincreasein
prolactin(assumingtheanteriorpituitaryitselfwasnotdamaged)?
Thisincreaseisduetodisruptionofthetuberoinfundibulartract,whichrunsfromthehypothalamus
throughthepituitarystalkandisthesourceofdopamine,whichinhibitsprolactinrelease.

Note:Plasmalevelsofallotheranteriorpituitaryhormones(e.g.,TSH,ACTH)willdecreasewitha
severedpituitarystalk.
Besuretounderstandtheconceptoftuberoinfundibilartractdisruption.Itisespecially
importantforyoutoknowthatallpituitaryhormonesdecreaseexceptforprolactin,which
increasesowingtodecreasedlevelsofdopamine.
Step1Secret

9 Whydoeshypothyroidismneedtobeconsideredintheevaluationof
hyperprolactinemia?
Hypothalamicthyrotropinreleasinghormone(TRH)stimulatesthesecretionofbothTSHand
prolactinbytheanteriorpituitary.BecausehypothyroidismcauseselevatedlevelsofTRH,itshould
beruledoutasapotentialcauseofhyperprolactinemia.

Thepituitaryislocatedinthesellaturcica.
Prolactinomaisthemostcommontypeofhypersecretingpituitaryadenoma.
Hyperprolactinemiacanbecausedbyaprolactinoma,antipsychotics(viainhibitionof
hypothalamicdopaminesecretion),hypothyroidism(viaincreasedthyrotropinreleasing
hormone[TRH]),andbreastfeedingorexcessivenipplestimulation.
Theincreaseinprolactinsecretionthatoccurswithsucklingisimportantinallowingfor
lactation.Itcanalsoinhibitgonadotropinreleasinghormone(GnRH)secretionand
causeananovulatoryinfertility,explainingwhynursingwomenmayhavedifficulty
becomingpregnant.
Inwomen,inadditiontocausinginfertility,hyperprolactinemiacancausegalactorrhea.
Itcanalsocausemalaiseanddepression.
Inmen,hyperprolactinemiacancauseimpotenceandlackoflibidobutonlyrarelywill
causegalactorrhea.
Treatmentforprolactinomaincludestheuseofdopamineagonists(forsmalladenomas)
or,lesscommonly,surgicalresection(forlargeradenomas).
SummaryBox:ProlactinomaandHyperprolactinemia

Case83

A38yearoldmanpresentscomplainingofgraduallyenlarginghandsandfeetoverthepast
severalyears.Incomparisonwithaphotofrom15yearsago,hisfacialfeatureshavebecome
obviouslycoarsened.Laboratoryevaluationshowsmildlyelevatedplasmaglucose,andMRIof
thebrainrevealsanenlargedmassinthesellaturcica.Giventhesuspecteddiagnosis,
specializedtestingisperformedinwhichGHlevelsaremeasuredfollowingadministrationof
anoralglucoseloadnomeasurabledecreaseisseen.

1 Whatisthediagnosis?
Acromegaly(fromtheGreekrootsakros[extremities]andmegalos[large]:largeextremities)is
causedbyaGHsecretingtumoroftheanteriorpituitary.Itmostcommonlyaffectsmiddleaged
adults.
Note:Onegoodwaytodiagnosethisdisorderistolookatanoldpictureofthepatientandcompare
itwiththepatient'scurrentappearance.Becausethephysicalchangestakeplaceoverdecades,family
membersandfriendsoftendonotrecognizethem.

2 Whyishyperglycemiacommonlyassociatedwiththisdisease?
GHincreasesserumglucoselevels,anditsreleaseinresponsetodecreasingglucoseconcentrationis
oneofthebody'smechanismsforpreventingserioushypoglycemia.Theelevationofbloodglucose
canbesignificantenoughinacromegalythatmanyofthesepatientswillhavefrankdiabetes.In
normalindividuals,aglucoseloadwillcausealmostcompletesuppressionofGHsecretion,butitwill
notsuppressGHsecretionasmuchoratallbyanindependentlyfunctioningpituitaryadenomathat
issecretingGH.

3 Whatarethenormalphysiologicfunctionsofgrowthhormoneandhowisits
secretionregulated?
GHsecretionoccursprimarilyatnightandinresponsetovariousstressorssuchasstarvationand
hypoglycemia.Whenreleasedduringagoodnightofsleep,itsanabolicactionsonmuscleandbone
areofprimaryimportance.Whenreleasedinresponsetophysiologicstressorssuchasstarvationand
hypoglycemia,itsmetabolicactionstoconservecarbohydratefuels(forusebythecentralnervous
system[CNS]andotherglucosedependenttissues)andmaintainproteinstores(topreservemuscle
strengthneededformobility)takecenterstage.
GHsecretionisinhibitedbyelevatedsomatostatin,glucoselevels,emotionalstress,illness,
malnutrition,obesity,glucocorticoids,anddecreasedthyroidhormone.Triiodothyronine(T3)is
requiredfornormalfunctionofGH.

4 Giventhenormalphysiologyofgrowthhormone,howcanweexplainthis
patient'spresentation?

Theenlargedmassinthesellaturcicaisexplainedbyapituitaryadenomacomposedofproliferating
somatotrophs.TheenlargedhandsandfeetresultfromtheanaboliceffectsofGHandinsulinlike
growthfactor1(IGF1)onthebones.Asdiscussedpreviously,failureofGHsuppressionbyaglucose
loadistypicallyseeninacromegaly.Wewerenottoldthatthispatienthadhyperglycemia,butone
wouldexpectitinacromegalybecauseoftheinsulinantagonizing(diabetogenic)actionsofGHon
theliverandskeletalmuscle,aswellasstimulationoflipolysisinadiposetissue.
Note:MostofthemetaboliceffectsofGHaremediatedthroughIGF1,whichissecretedbythe
liver.IGF1actsonbonetostimulatelinearandlateralbonegrowth.IGF1alsopromotesthegrowth
ofcartilageandothersofttissues(Fig.89(f0050)).

Figure89
Physiologicactionsofgrowthhormone.GH,growthhormoneGHRH,growthhormonereleasinghormone
IGFs,insulinlikegrowthfactors.
(FromBrownTA:RapidReviewPhysiology.Philadelphia,Mosby,2007.)

5 Whyisoctreotide,asomatostatinanalog,usefulinthetreatmentof
acromegaly?
Somatostatinisapeptidehormonesecretedbythehypothalamus.ItinhibitsGHsecretionbythe
anteriorpituitary.ItsanalogoctreotideisalsoeffectiveininhibitingGHsecretion.

Note:Somatostatinisalsosynthesizedbypancreaticisletsandgastricmucosaandinhibitsintestinal
activityandgastrointestinalmotility.

6 Ifthispatientdevelopedagrowthhormonesecretingtumorinhisearly
teens,howmighttheclinicalmanifestationsdiffer?
GHstimulatesbonegrowth,andiftheepiphysealplateshavenotyetclosed,excessGHcancause
patientstoattainextremesofbodyheight.Whenthisoccurs,thediseaseiscalledgigantism.Andr
theGiant,aprofessionalwrestlerandactor,sufferedfromthisdisease.

7 Whatgrowthabnormalityresultsfromdeficientsecretionofgrowthhormone
duringthegrowingyears?
Pituitarydwarfismisthemostcommonresultofthisdeficiency.
Note:ThedwarfismcausedbydeficientGHsecretionisproportional(i.e.,thelimbsandtrunkareof
normalrelativeproportions),asopposedtothecomparativelyshorterlimbscharacteristicof
dwarfismcausedbyachondroplasia.Ofnote,achondroplasiaiscausedbymutationsofthefibroblast
growthfactorreceptorgene3(FGFR3),whichinhibitbonegrowth.

Agrowthhormone(GH)hypersecretingpituitaryadenomacanresultingigantismand
acromegalydependingontheageatonset(i.e.,whethertheepiphysealplateshavefused).
GHpromotesanabolicactionsonskeletalmuscleandbone,stimulateslipolysisin
adiposetissue,andstimulateshepaticgluconeogenesis.Itgenerallyantagonizesthe
actionsofinsulin.
ThemetabolicactionsofGHarelargelymediatedbyinsulinlikegrowthfactor1(IGF
1).
GHsecretionisstimulatedbyhypoglycemia,stress,andsleep.Itisinhibitedby
somatostatin,glucose,andIGF1.GlucoseadministrationdoesnotinhibitGHsecretion
inacromegaly.
Patientswithacromegalymaypresentwithprominentjaw(macrognathia),coarsening
offacialfeatures,hyperglycemiaorfrankdiabetes,andorganomegaly.
SummaryBox:AcromegalyandPhysiologyofGrowthHormone

Case84
A35yearoldwomanpresentsforevaluationofa6monthhistoryoffatigue,weakness,and
hippain.Shehasalsonothadherperiodforthepast6monthsandnotesthathervoiceseems
deeperthanusual.Thepatientappearsmoderatelyobese(withprimarilyacentraldistribution)
andhasajovialroundedface.Nofacialhaircanbeappreciated,butshedoesadmittoshaving

onaregularbasis.Physicalexaminationissignificantforamyriadofadditionalfindings,
includingapronounceddorsocervicalfatpad,acanthosisnigricans,purpleabdominalstriae,
andproximalmuscleweakness.Laboratorytestsshowhyperglycemiaandanelevatedrandom
cortisol.

1 Whatarethegeneralcausesofthehormonalabnormalitymostlikelypresent
inthiswoman?
Thiswomanhashypercortisolism(Cushingsyndrome),asindicatedbyhersymptoms(muscle
weakness,amenorrhea,deepeningvoice),classicexaminationfindings(centralobesity,abdominal
striae,dorsocervicalfatpadorbuffalohump,acanthosisnigricans),andlaboratoryfindings
(hyperglycemia,elevatedrandomcortisol).
PerhapsthemostcommoncauseofCushingsyndromeistheiatrogenicprescriptionof
glucocorticoidsforinflammatoryconditions.Othercausesincludeacortisolhypersecretingadrenal
adenoma(common)orcarcinoma(rare),ACTHsecretingpituitaryadenoma(Cushingdisease),and
ectopic(paraneoplastic)secretion(thinksmallcellcarcinomaofthelungforboards).
ThehirsutismanddeepeningvoiceinthispatientaresuggestiveofanACTHdependentcauseof
Cushingsyndrome,inwhichshuntingofglucocorticoidprecursorsintotheandrogenicpathway
occurs,butfurtherworkupisclearlyneeded.

Case84continued:
Furtherworkupshowselevated11pmsalivarycortisolandplasmaACTHlevels,bothofwhich
suppressmoderatelyinresponsetotheadministrationofhighdosedexamethasone.

2 Whatisthecauseofthehypercortisolisminthispatient?
CushingdiseaseisduetotheelevatedcortisolandACTH.OnewouldexpectsuppressedACTHlevels
iftheadrenalglandswerehypersecretingcortisol.ParaneoplasticACTHsecretion(e.g.,smallcell
carcinoma)issuggestedbytheincreasedACTHlevel,butectopicACTHsecretionoccurs
independentlyoftheHPA(hypothalamicpituitaryadrenal)axisanddoesnotnormallysuppressin
responsetoglucocorticoidssuchasdexamethasone.Incontrast,pituitaryadenomas,whicharewell
differentiated,typicallyretainsomefeedbackresponsivenesstoglucocorticoidsthus,ACTH
secretionmaynotsuppresswithlowdosedexamethasonebuttypicallywillwithhighdose
dexamethasone(Table86(t0035)).
Table86
FeaturesofCushingSyndrome

CauseofCushing
Syndrome

ACTH Cortisol ResultsofDexamethasoneSuppressionTeston


PlasmaCortisol

Cushingdisease

High

High

Highdoselowerscortisol

EctopicACTH
production

High

High

Noeffect

Adrenal
adenoma/carcinoma

Low

High

Noeffect

Iatrogenic

Low

High

Noeffect

ACTH,adrenocorticotropichormone.

3 Whyaretheresultsofadexamethasonesuppressiontestreadataspecific
timeofday?
Cortisolsecretionhasawidecircadianrhythm,withplasmalevelsvaryingseveralfoldthroughouta
24hourperiod(normalrange520g/dL)andpeakinginthemorninghours.Themorningupsurge
isnecessarilyprecededbyanupsurgeinACTH,soACTHlevelsshouldalsobemeasuredataspecific
time(usually5am).

4 Whyhashirsutismdevelopedinthiswoman?
Hirsutismisthepresenceofexcessfacialandbodyhairinwomen,especiallyinamalepattern,and
typicallyreflectselevatedandrogens.Elevatedandrogensinthiswomanareduetoincreasedlevelsof
ACTH,whichinadditiontostimulatingcortisolsynthesisbytheadrenalsalsopromotesshunting
oftheglucocorticoidprecursorstotheandrogenpathway,.

5 Whyisn'thyperaldosteronismtypicallyseeninCushingdisease?
Althoughaldosteroneissecretedbytheadrenalcortex,itssynthesisandsecretionareinfluenced
onlyminimallybyACTHlevels.Rather,theprincipalregulatorsofaldosteronesecretionare
angiotensinIIandserumpotassium.

6 Whatmorphologicfeatureoftheadrenalglandswouldyouexpecttoseein
thiswoman?
Bilateraladrenalhyperplasia(withwideningofthezonafasciculataandreticularis,specifically)is
typicallyseenbecauseACTHistrophicfortheadrenalglands.Thisfeatureisalsopresentinectopic
productionofACTH.

7 Howishypercortisolismcontributingtohyperglycemiainthispatient?
Cortisolpromoteshyperglycemiabystimulatinghepaticgluconeogenesisandinhibitingthe
peripheralutilizationofglucose(similartoGH).Theincreaseingluconeogenesisisdueto
stimulationofthesynthesisofgluconeogenicenzymesbycortisolandalsotogreatermobilizationof
aminoacidsfromskeletalmuscletoparticipateingluconeogenesis(hencethemusclewasting)(Fig.
810(f0055)).

Figure810
Metabolicactionsofcortisol.
(FromBrownTA:RapidReviewPhysiology.Philadelphia,Mosby,2007.)

8 Howishypercortisolismcontributingtohypertensionandhypokalemiainthis
patient?
Athigherplasmalevels,cortisolexertsmineralocorticoideffectssimilartothoseofaldosterone.This
causessodiumretentionandanensuingplasmavolumeexpansionthatcontributestohypertension.
Thesodiumisretainedinexchangeforsecretingmorepotassium,whichexplainsthispatient's
hypokalemia.Anothercontributingfactortothispatient'shypertensionisthatcortisolstimulatesthe
expressionofadrenergicreceptorsinvascularsmoothmuscle.

9 Whydoesn'tcortisolhavemineralocorticoidactionsinthenormalphysiologic
setting?
Cortisolcanbindmineralocorticoidreceptorswithanaffinitysimilartothatofaldosterone,butcells
inmineralocorticoidsensitivetissues(kidneys,colon,salivaryglands)produce11hydroxysteroid
dehydrogenase(11HSD),whichbreaksdowncortisolintocortisone.Cortisonecannotbindtothe
aldosteronereceptor.Whentheplasmacortisolissignificantlyelevated,11HSDbecomessaturated,
therebyallowingintracellularcortisoltoexertitsmineralocorticoideffectsinthesetissues.This
explainstheplasmavolumeexpansionandhypertensiondiscussedpreviously.
Note:Licoricecandy(ifitcontainslicoricerootfromthelicoriceplant)alsoincreasesplasma
cortisollevelsbydecreasingtheactivityof11HSD,therebypotentiallycausinghypertension.

10 Whatwouldanxraystudyofherboneslikelyreveal?
Anxraystudyofherboneswouldlikelyshowalossofbonedensityduetoosteoporosis.Cortisol
inhibitsosteoblastsandstimulatesosteoclasts,adoublewhammy.Asaresult,thispromotesbone
resorption,leadingtoosteoporosis.Osteoporosisisanespeciallysignificantsideeffectinpatients
takingsteroidschronically.

11 Whatarethethreelayersoftheadrenalcortex,andwhichoneisresponsible
fortheexcessproductionofcortisolinthispatient?

Justthinkofglomerularfiltrationrate(GFR)forthethreelayerszonaglomerulosa,fasciculata,
andreticulariswhichsecretemineralocorticoids(e.g.,aldosterone),glucocorticoids(e.g.,cortisol),
andandrogens(e.g.,dehydroepiandrosterone[DHEA]),respectively(Fig.811(f0060)).

Figure811
Maindeterminantsofhypothalamicpituitaryadrenalaxis.ACTH,adrenocorticotropichormoneCRH,
corticotropinreleasinghormoneDHEA,dehydroepiandrosterone.
(FromBrownTA:RapidReviewPhysiology.Philadelphia,Mosby,2007.)

12 OnetreatmentforCushingdiseaseistoremovebothadrenalglands(bilateral
adrenalectomy).Whatmighthappentothepituitaryglandfollowingsucha
surgery?
Theremovalofbothadrenalglandswillstoptheproductionofcortisol.Lackingthenegative
feedbackofcortisol,thepituitarywillsynthesizeACTHandmelanocytestimulatinghormone(MSH)
unchecked,leadingtoenlargementofthepreexistingpituitaryadenomawithresultingheadacheand
diffusehyperpigmentation.ThisisknownasNelson'ssyndrome.
Note:MSHproductioniselevatedwhenACTHproductionisincreasedbecausebothareproduced
fromthesameprecursorpolypeptide(proopiomelanocortin[POMC]).

HypercortisolismregardlessofcauseistermedCushingsyndrome.Themostcommon
causeofCushingsyndromeistheiatrogenicadministrationofsteroids.
HypercortisolismfromahypersecretingpituitaryadenomaistermedCushingdisease.
ClassicsignsandsymptomsofCushingsyndromeinclude:
Centralobesity:Cortisolstimulatesproteinbreakdownintheextremities,butthe
hyperinsulinemiafromthecortisolinducedhyperglycemiapromotesfatdeposition.

Purpleabdominalstriaearetheresultofweightgainandcapillaryfragilityor
rupturefromtheeffectsofhypercortisolism.
Hypertensionisduetomineralocorticoidactionsofcortisolatthekidney.
Hirsutismisduetoadrenocorticotropichormone(ACTH)inducedshuntingof
glucocorticoidprecursorstotheandrogenicpathway.
Osteoporosisisduetocortisolinducedbonebreakdown.
BilateraladrenalectomyforCushingdisease(rarelydone!)cancauseNelson's
syndrome,whichischaracterizedbydiffusehyperpigmentationasaresultofincreased
melanocytestimulatinghormone(MSH)production.
SummaryBox:Hypercortisolism

Case85
An18yearoldmanpresentsforevaluationofa6monthhistoryoffatigueandweakness.He
alsocomplainsofanorexia,nausea,andanunintentional10lbweightloss.Onphysical
examination,hisbloodpressureis90/55mmHg,andhisskinappearstandespiteitsbeing
midwinter.Laboratoryworkupissignificantforlowsodium,highpotassium,andlowglucose
levels.

1 Whatdoyoususpectatthispoint?
Vaguesymptoms(fatigue,weakness),hypotension,andthemetabolicdisturbancesinthiscaseare
allconcerningforadrenalinsufficiency.
Recallthataldosterone(andcortisolathighlevels)stimulatessodiumretentionandpotassium
excretionfromthekidneys.Recallalsothatcortisolandepinephrineantagonizetheactionsof
insulin.Giventhesephysiologicfunctions,onecanseehowalackofmineralocorticoid,
glucocorticoid,andcatecholaminescanresultinhypotension,hyponatremia(seefollowingnote),
andhyperkalemia.
Therearemanycausesofadrenalinsufficiencybutthehyperpigmentationobservedinthispatient
suggestselevatedACTHlevels(recallthatMSHisabyproductofACTHsynthesis).Wecandeduce
fromthisthattheproblemexistsattheleveloftheadrenals(lowcortisollevelsdecreasenegative
feedbackonthepituitary,leadingtoelevatedACTHlevels).

2 Whataresomegeneralcausesofprimaryadrenalinsufficiency(Addison's
disease)?

AutoimmunedestructionisperhapsthemostcommoncauseofAddison'sdisease.Tubercular
invasionoftheadrenalsinmiliary(disseminated)tuberculosisisacommoncauseofAddison's
diseaseindevelopingcountries.Metastaticinvasionoftheadrenals,whichoccursfrequentlywith
lungcancer,isanothercause.Otherrarercausesincludedisseminatedfungalinfections(e.g.,
histoplasmosis)anddrugssuchasketoconazoleandmetyrapone,whichselectivelyinhibitthe
steroidogenesispathway.

Case85continued:
FurtherworkuprevealselevatedamplasmaACTHandlowplasmacortisol.InjectionofACTH
(cosyntropin)elicitsonlyabluntedincreaseinplasmacortisol.Heisstartedonroutine
hydrocortisolandfludrocortisone.

3 Whatwasthecauseofadrenalinsufficiencyinthispatient?
Allwecansaywiththegiveninformationisthatthisyoungmanhasprimaryadrenalinsufficiency
duetotheelevatedACTHandlackofresponsetocosyntropin.Becausetherearenocluestosuggest
malignancyorinfection,andwearenottoldthatheistakinganymedications,thelikelycauseis
autoimmunedestructionoftheadrenals.

Relatedquestions
4 Whatadrenaldiseaseshouldbesuspectedinayoungpatientwithbacterial
meningitisduetoNeisseriameningitidiswhoalsobecomesacutely
hypotensive?
WaterhouseFriderichsensyndrometypicallycausesbilateraladrenalhemorrhage,whichcanbe
rapidlyfatal.TheresponsiblebacteriumisNeisseriameningitidis.Thisisaboardfavorite.
WaterhouseFriderichsensyndromeisahighyielddiagnosisforclinicalvignettesonboards.
Beonthelookoutforsymptomsofsepticemia,disseminatedintravascularcoagulation(DIC),
adrenalhemorrhage,andpetechialrash.
Step1Secret

5 Howwouldweexpectplasmaaldosteronelevelstobeaffectedinapatient
withsecondary(pituitary)adrenalinsufficiency?
AldosteronesecretionisprimarilystimulatedbyangiotensinIIandserumpotassiumlevels.ACTH
haslittleinfluenceonaldosteronesecretion,sothishormonewillcontinuetobesecretedatnormal
levels.

Primaryadrenalinsufficiency(Addison'sdisease)canpresentwithvaguesymptoms
suchasweaknessandmalaiseaswellaswithspecificmetabolicabnormalitiessuchas

hyponatremia,hyperkalemia,andhypoglycemia.Hypotensionduetovascularcollapseis
alsocommon.
CausesofAddison'sdiseaseincludeautoimmunedestruction,tubercularandmetastatic
invasion,fungalinfectionssuchasdisseminatedhistoplasmosis,anddrugssuchas
ketoconazoleandmetyrapone,whichselectivelyinhibittheglucocorticoidpathway.
Themostcommoncauseofadrenalinsufficiencyistheiatrogenicadministrationof
steroidsforinflammatoryconditions.
Anelevatedadrenocorticotropichormone(ACTH)levelsuggestsprimaryadrenal
insufficiency,whereasalowACTHlevelsuggestspituitary,orrarely,ahypothalamic
etiology.
Anotheradrenaldiseasethatshouldbeconsideredintheappropriateclinicalscenariois
WaterhouseFriderichsensyndrome.ThisdiseaseiscausedbyNeisseriameningitidisand
resultsinbilateraladrenalhemorrhage.Itcanberapidlyfatal.
SummaryBox:AdrenalInsufficiency

Case86
A38yearoldmanwithahistoryofgeneralizedanxietydisordercomplainsofintermittent
episodesofheadache,palpitations,profusesweating,andfearofimpendingdeath.Hewas
recentlystartedonabetablockerformildhypertension,butsurprisingly,hishypertensionhas
worsened.Hedeniesanyhistoryofcocaineoramphetamineabuse.

1 Whatdoyoususpectatthispoint?
Intermittentepisodesofheadache,palpitations,profusesweating,andfearofimpendingdeathare
classicforpheochromocytoma.Theworseningofhishypertensionwithinitiationofbetablockers
alsopointstowardpheochromocytoma.However,otherdiagnosestoconsiderincludepanicattack
(disorder),hypoglycemia,mastocytosis,andcarcinoidsyndrome.

2 Whydoeshypertensionoftenworsenafterstartingabetablockerinpatients
withpheochromocytoma?
Plasmaepinephrine,whichiselevatedinpheochromocytoma,hasvasodilatoryandvasoconstrictor
effectsdependingonwhichadrenergicreceptor(or)itbinds.Inthepresenceofabetablocker,
epinephrinewillprimarilybindadrenergicreceptors,resultinginunopposedreceptormediated
vasoconstriction,therebyraisingthebloodpressure.

Case86continued:

A24hoururinecollectionrevealselevatedlevelsofcatecholaminemetabolites,andapositron
emissiontomography(PET)scanshowswhatappearstobeahighlyvascularmassabovethe
leftkidney.Theadministrationofclonidinedoesnotsuppressplasmacatecholaminelevels.

3 Whatisthelikelydiagnosis?
Pheochromocytoma,atumorthatepisodicallyreleaseslargeamountsofcatecholaminesandresults
insymptomaticepisodesofhypertension,tachycardia,palpitations,sweating,andheadache,ismost
likely.Pheochromocytomasgenerallyarisefromneuralcrestderivedchromaffincellsoftheadrenal
medulla.Tumorsarisinginparagangliaaretermedparagangliomasorextraadrenal
pheochromocytomas.
Youshouldknowallofthederivativesofneuralcrestcells.Believeitornot,thisisacommonly
testedboardstopic.Theyincludechromaffincells,parafollicularcellsofthethyroid,Schwann
cells,autonomicnervoussystem(ANS),dorsalrootandceliacganglia,melanocytes,cranial
nerves,piaandarachnoid,odontoblasts,skullbones,andtheaorticopulmonaryseptum.
Knowingtheembryologicderivativesofothertissuetypesisfairgameforboardsbutnotnearly
ashighyieldasneuralcrestderivatives.
Step1Secret

4 Howcantheadministrationofclonidinebeusedtodifferentiate
pheochromocytomafromahighstressstate?
Clonidineisacentrallyacting2agonistthatinhibitssympatheticoutflowfromtheCNS.Its
administrationshouldreducecatecholamineproductionbytheCNSandadrenalmedullabutdoes
notreducecatecholamineproductionbyanautonomouslyfunctioningpheochromocytoma.

5 Whatistheruleof10sforpheochromocytomas?
10%arefamilial(seefollowingnote)
10%areextraadrenalinlocation
10%aremalignant
10%occurinchildren
10%arecalcified
10%affecttheadrenalsbilaterally

Note:Mostpheochromocytomasarisesporadically,butapproximately10%areassociatedwith
hereditarydisorderssuchasmultipleendocrineneoplasia(MEN)IIAorMENIIB,vonHippel
Lindau(VHL)disease,orneurofibromatosis.

6 Whatmalignanttumorthatmostoftenoccursinchildrenunder5yearsofage
alsoshowsincreasedurinarylevelsofcatecholaminemetabolites?
Neuroblastomaisaneuroendocrinetumorarisingfromneuralcrestcells.Childrenwith
neuroblastomamaymanifestwithhypertensionandapalpableabdominalmass.Approximately50%
ofcasesarealsoassociatedwithopsoclonusmyoclonussyndrome,inwhichpatientsexhibitchaotic
eyemovements.Neuroblastomacommonlymetastasizestoskin,bone,andtheposterior
mediastinum.ThetumoritselfisassociatedwithNmyconcogeneoverexpressionandHomer
Wrightrosettes(neuroblastssurroundingspacesfilledwitheosinophilicneuropil).Youshouldbe
awareoftheseassociationsforboards.

Case86continued:
Thepatientisprescribedantihypertensivetherapythatincludesprazosinandpropranololand
isinformedthatsurgicalcorrectioncanofferacure.

7 Whattypeofreceptorsdonorepinephrineandepinephrinebindtoonthe
hearttoincreasetherateandforceofcardiaccontraction?
Theybindto1adrenergicreceptorsonnodalcells(positivechronotropiceffect)andcardiac
myocytes(positiveinotropiceffect).Thisiswhyabetablocker,likepropranolol,isusedaspartofthe
managementofhypertensioninpatientswithpheochromocytomaaslongasanblockerisstarted
first.

8 Whywasthispatientgivenpropranolol,anonselectivebetablocker,ifitcan
causeahypertensivecrisis?
You'llnoticehewasalsogivenprazosin,whichisan1receptorantagonistandcanpreventthis
problem.Infact,itisimportanttoachieveblockadefirstwithadruglikeprazosinbeforebeta
blockersareevenstartedinordertoavoidsuchahypertensivecrisis.
Theosins,whichincludedoxazosin,terazosin,andprazosin,are1receptorantagonists.These
receptorsarelocatedpredominantlyonvascularsmoothmuscle.

9 Onarelatednote,whatclassofantihypertensiveagent,ifgivenpriorto
epinephrine,wouldmakeitsothatepinephrineactuallyloweredtheblood
pressure?
Inthiscase,theinitialadministrationof1blockers(e.g.,prazosin)willblockallofthe1
receptors,leavingonly2adrenergicreceptorsavailableforbindingtosubsequentlyadministered
epinephrine.Stimulationof2adrenergicreceptorsresultsinvasodilation,whichlowersperipheral

vascularresistanceandcausesadropinbloodpressure.
Questionsliketheseareboardfavoritesbecausetheystressabroadconceptualunderstanding
ofphysiologyandpharmacology.Automaticnervoussystem(ANS)drugsareafivestarboards
topic.
Step1Secret

Case86continued:
Priortosurgery,bloodpressurewasnormalizedwithphenoxybenzamine.Surgicalresectionof
theaffectedadrenalglandisperformedwithoutcomplications.

10 Whywasphenoxybenzaminegivenpriortosurgery?
Phenoxybenzamineisanirreversiblenoncompetitiveantagonistof1adrenergicreceptors.Because
ofitsirreversiblebindingof1adrenergicreceptors,itispreferredoverother1blockersin
preoperativepreparation.Itminimizesthehypertensiveeffectsofcatecholaminesreleasedduring
thesurgery.

Intermittentepisodesofheadache,palpitations,profusesweating,andfearof
impendingdeathareclassicforpheochromocytoma.Worseningofhypertensionwith
initiationofbetablockersisalsosuggestiveofpheochromocytoma.
Remembertheruleof10sforpheochromocytoma:10%arefamilial,10%areextra
adrenalinlocation,10%aremalignant,10%occurinchildren,10%arecalcified,and10%
affecttheadrenalglandsbilaterally.
Clonidineisusedtodifferentiatepheochromocytomafromincreasedsympathoadrenal
outflowduetostressorpain.
Inpheochromocytoma,anblockershouldbegivenbeforebetablockadetoprevent
unopposedblockermediatedvasoconstriction.Phenoxybenzamineisanirreversible
1blockerthatispreferredoverother1blockersinpreoperativepreparationbecauseit

minimizesthehypertensiveeffectsofcatecholaminesreleasedduringthesurgery.
Neuroblastomaisamalignanttumorthattypicallymanifestsinyoungchildrenandalso
showsincreasedurinarylevelsofcatecholaminemetabolites.
SummaryBox:Pheochromocytoma

Case87

A40yearoldwomancomplainsofeasyfatigability,heatintolerance,excessivesweating,and
palpitations.Shealsonotesanunintentional20lbweightlossdespiteanexcellentappetite,as
wellasoccasionaldiarrhea.

1 Whatdoyoususpectatthispoint?
Thereareonlyafewconditionsassociatedwithunintentionalweightlossdespitenormalfoodintake.
Thesepossibilitiesincludediabetesmellitus,malabsorptionsyndromessuchassprue,cancer,and
hyperthyroidism.Bothhyperthyroidismandmalabsorptioncancausediarrhea,butgiventhis
patient'sheatintoleranceandpalpitations,hyperthyroidismseemslikely.

Case87continued:
Physicalexaminationissignificantforabloodpressureof165/75mmHg,asystolicejection
murmur,andtachycardia,aswellasdiffusenontenderenlargementofthethyroid,slight
restingtremor,finehair,separationofthefingernailplatefromthenailbed(onycholysis),and
briskreflexes.Laboratorytestsrevealelevatedfreeplasmathyroxine(T4)andreducedTSH.

2 Whatisthediagnosis?
Hyperthyroidism,themostcommoncauseofwhichisGravesdisease.Othercausesof
hyperthyroidismarelistedinTable87(t0040).
Table87
FeaturesofHyperthyroidism

Cause

Pathophysiology

Patternof
ClassicPresentation
Radioiodine
Uptake

PermanentCauses
Graves

ActivatingantibodiestoTSH

Diffuse

Goiter,ophthalmopathy,

disease

receptor

uptake

dermopathy

(diffusetoxic

throughout

goiter)

gland

Toxic
multinodular

Multiplehyperactivenodules,
mayhavemutationsingenes

Uptakein
oneora

Historyofnontoxicmultinodular
goiterinolderadult

goiter

encodingTSHreceptororG

fewoverly

Mayincludecardiaccomplications

proteins

activehot

suchasatrialfibrillationorheart

nodules

failure

Uptakein

remainder
ofthyroidis
suppressed
Toxic

Hyperactiveadenoma(s)may

Uptakein

Historyofaslowlygrowinglump

adenoma

havemutationsingenes

oneora

intheneckinayoungeradult

(Plummer's

encodingTSHreceptororG

fewhot

disease)

proteins

nodules
Uptakein
remainder
ofthyroidis
suppressed

Pituitary

HypersecretionofTSH

adenoma

Diffuse

Mayincludeadditionalsymptoms

uptake

(e.g.,headaches,bitemporal

throughout

hemianopia,nauseaandvomiting)

thyroid
TransientCauses
Autoimmune

Autoimmunedestructionof

Suppressed Hyperthyroidisminitially,followed

thyroiditis

thyroid

uptake

(e.g.,
Hashimoto's

byhypothyroidism

throughout
thyroid

disease)
Subacute

Probablysecondarytoviral

Suppressed Thyroidexquisitelypainfulto

thyroiditis(de

infectionofthyroid

uptake

Quervain's
thyroiditis)

Followsupperrespiratorytract
infection

throughout
thyroid

Iodine

Iodineoverloadmaystimulate

Suppressed Thyrotoxicosisinapatientwithtoxic

induced(Jod

autonomousnodules,which

uptake

multinodulargoiterafter

Basedow

functionindependentlyofTSH

throughout

administrationofiodinerich

effect)

stimulation,tohypersecrete

thyroid

radiographiccontrastmediaand

thyroidhormone

palpation

iodinateddrugssuchas
amiodarone

Thyrotoxicosis Inadvertentorintentional

Suppressed Ingestionofthyroidhormonetolose

factitia

ingestionoflargeamountsof

uptake

weight,typicallybymedical

thyroidhormone

throughout

personnel

thyroid
Strumaovarii

Thyroidtissueformspartof

Suppressed Hyperthyroidisminfemale

ovariangermcelltumor

uptake

(teratoma)andsecretes

throughout

excessivethyroidhormone

thyroid

Trophoblastic

Malignanttrophoblastictissue

Diffuse

Hydatidiformmoles,

tumors

secreteshumanchorionic

uptake

choriocarcinoma,metastatic

gonadotropin(hCG),which

throughout

embryonalcarcinomaofthetestis

stimulatestheTSHreceptor

thyroid

TSH,thyroidstimulatinghormone.

3 Whyhasthiswomanexperiencedweightloss?
Thethyroidhormonesincreasethebasalmetabolicrate(BMR),principallythroughincreasingthe
productionandinsertionoftheNa+/K+ATPasepumpsinvariouscelltypes.Thyroidhormones
alsoincreasetheBMRviaproductionofglycolyticenzymes.Theincreasedmetabolicrateresultsin
weightlossandcontributestoheatintolerance(Table88(t0045)).
Table88
IntracellularandPhysiologicActionsofTriiodothyronine(T3)

Siteof

IntracellularEffects

PhysiologicResults

StimulatestheNa+/K+
ATPasepump

Increaseddemandformetabolites,e.g.,glucose

Action
Cell
membrane

Mitochondria Stimulatesgrowth,replication,
andactivitybasalmetabolic

Increasedheatproduction,oxygendemand,heart
rate,andstrokevolume

rateisraised
Nucleus

Neonatal
cells

Increasesexpressionof

Lipolysis,glycolysis,andgluconeogenesis

enzymesnecessaryforenergy

increasedtoraisebloodmetabolitelevelsand

production

cellularmetaboliteuse

Essentialforcelldivisionand
maturation

Essentialfornormaldevelopmentofcentral
nervoussystemandskeleton

FromMeszarosJG,OlsonER,NaugleJE,etal:CrashCourse:EndocrineandReproductiveSystems.
Philadelphia,Mosby,2006.

4 Whatarethetwothyroidhormonesandwhichismorepotent?
T4andT3arethehormones.T3ismuchmorepotentthanT4(~5timesmoreso)andisprimarily
producedbytheperipheralconversionofT4toT3(whichiscatalyzedbytheintracellularenzyme5
deiodinase),althoughasmuchas20%ofT3canbesecretedfromthethyroidgland.Someauthors

prefertocallT4aprohormone,becauseitsactivityislargelydependentonconversiontothemore
activeT3(Table89(t0050)).
Table89
ComparisonofT3andT4

Feature

T3

T4

Proportionofsecretedthyroidhormone 10% 90%


Percentagefreeinplasma

1%

0.1%

Relativeactivity

10

Halflife(days)

T3,triiodothyronineT4,thyroxine.(FromMeszarosJG,OlsonER,NaugleJE,etal:CrashCourse:Endocrine
andReproductiveSystems.Philadelphia,Mosby,2006.)

5 Howarethethyroidhormonessynthesized?
Ultimately,fouriodineresidueshavetobeattachedtotwotyrosineresiduestoformT4,orthree
iodineresiduesareattachedtoformT3.Thefirststepinthesynthesisistheuptakeofiodideion(I
)fromplasmaintofollicularcellsandeventuallythefollicularlumenviatheiodidepump.Within
thelumen,theenzymethyroidperoxidasethencatalyzesthenexttwosteps,inwhichiodideis
oxidizedtoiodine(I2)andiodinemoleculesareattachedtotyrosineresiduesonthyroglobulin(
organificationstep).IodinatedtyrosineresiduesarethencoupledtogethertoformeitherT4orT3.
Endocytosisofthismodifiedthyroglobulinproteinintothefollicularcellsanditssubsequent
hydrolysisyieldT4andT3,whichdiffuseacrosstheplasmamembraneintothecirculation(Fig.812
(f0065)).

Figure812
Thyroidhormonesynthesisinthethyroidfollicle.DIT,diiodotyrosineI,iodideionI2,iodineMIT,
monoiodotyrosineT3,triiodothyronineT4,thyroxineTG,thyroglobulin.
(FromBrownTA:RapidReviewPhysiology.Philadelphia,Mosby,2007.)

6 Assumingtheplasmalevelsofcatecholaminesarenormalinthispatient,what
explainsthetachycardia,tremors,palpitations,andincreasedpulsepressure?
Thyroidhormonesincreasetheexpressionofadrenergicreceptorsintargettissues,resultingin
increasedsensitivitytocirculatingcatecholamines.Thyroidhormonealsoincreasesadrenergic
receptorsynthesisintheheartandhasadirectstimulatingeffect(bothinotropicandchronotropic)
ontheheart,independentofthesympatheticnervoussystem.
Asstrokevolumeincreasesandthediastolicpressureisreducedduetowidespreadvasodilationthat
iscausedbythethyroidhormonedependenttissuemetabolismneeds,theresultingeffectisan
increaseinpulsepressure.Figure813(f0070)showsotherpotentialpathologicmanifestationsof
Gravesdisease.

Figure813
Symptomsandsignsofthyrotoxicosis(hyperthyroidism).ThefeaturesinitalicsarefoundonlyinGraves
disease.
(FromMeszarosJG,OlsonER,NaugleJE,etal:CrashCourse:EndocrineandReproductiveSystems.
Philadelphia,Mosby,2006.)

Note:Betablockersareoftengiventoalleviatethesympathomimeticeffects(palpitations,tremor)
ofhyperthyroidism.Ofnote,untreatedhyperthyroidismcanpredisposetoosteoporosisandatrial
fibrillation.

7 Whatisthedifferencebetweenprimary,secondary,andtertiary
hyperthyroidismandwhichdoesthispatienthave?
Primaryhyperthyroidismresultsfromexcessiveproductionofthyroidhormonesbythethyroid
gland,whichinturnsuppressespituitaryTSHproduction.Incontrast,secondaryhyperthyroidism
resultsfromexcessivepituitarysecretionofTSH.Tertiaryhyperthyroidismiscausedbyincreased
hypothalamicsecretionofTRH.ThispatientmostlikelyhasGravesdiseaseduetoelevatedthyroid
hormonelevelsinthefaceofdecreasedTSHlevels(Table810(t0055)).
Table810
LaboratoryValuesAssociatedwithHyperthyroidism

Type

Example

TRH TSH T4

Primaryhyperthyroidism

Gravesdisease

Hypothalamictumor

Secondaryhyperthyroidism Pituitaryadenoma
Tertiaryhyperthyroidism

T4,thyroxineTRH,thyrotropinreleasinghormoneTSH,thyroidstimulatinghormone.

8 Basedonyoursuspecteddiagnosis,whatadditionallaboratoryandphysical
findingsmightyouexpect?
Gravesdiseaseisthemostcommoncauseofhyperthyroidismandisadditionallycharacterizedby
exophthalmos,pretibialmyxedema,andantibodies(IgG)totheTSHreceptorinthethyroid.The
antibodiespresumablystimulatethethyroidinthesamewayasTSHdoes.Noticethatthiswoman
hadadiffuselyenlargedthyroid,whichisconsistentwithTSHreceptorstimulationcausingdiffuse
thyroidenlargement.

Case87continued:
Onfurtherexamination,younotetheappearanceoftheskinofthelowerextremities,asshown
inFigure814(f0075).

Figure814
Appearanceoflowerextremitiesinpatientincase87.(st0370)
(FromNobleJ:TextbookofPrimaryCareMedicine,3rded.St.Louis,Mosby,2001.)

9 WhatisthepathophysiologyofthiscomplicationofGravesdisease?
Pretibialmyxedemaisanonpittingedemacausedbyaccumulationofinterstitialglycosaminoglycans
(GAGs)withinthedermis.Paradoxically,pretibialmyxedemacanalsobeseeninsevere
hypothyroidism.

10 Whatwouldathyroidiodide131uptakescanlikelyrevealinthispatient?
Becauseiodideisusedtosynthesizethyroidhormone,Gravesdiseasewouldshowanincreased
diffuseuptake,correspondingtoanincreasedsynthesisofT4.Whentheglandisinactiveowingto
exogenoushormonetherapyorbecauseoftheinflammationofthegland(e.g.,thyroiditis),uptake
wouldbelow.SeeFigures815(f0080)and816(f0085)forcomparison.

Figure815
Gravesdisease,showingdiffuselyincreasedradiolabelediodineuptake.
(FromMettlerFAJr:EssentialsofRadiology,2nded.Philadelphia,WBSaunders,2005.)

Figure816
Thyroiditis,showingnearabsenceofradiolabelediodineuptake.
(FromRakelRE:Conn'sCurrentTherapy2007,59thed.Philadelphia,WBSaunders,2007.)

11 Whattherapeuticoptionsareavailabletothispatient?
Shehastheoptionsofmedications(thionamides,betablockers),surgery,radioactive131Iablation,
andwatchfulwaiting.

12 Whymightaphysicianprescribepropylthiouracilormethimazoleforthis
woman?Howdothesedrugswork?
Bothpropylthiouracil(PTU)andmethimazolearelargelyconcentratedinthethyroidandinhibit
thyroidhormonesynthesis.Toasmallextent,PTUalsoactsbypreventingtheperipheral
deiodinationofT4toT3.
Note:PTUandmethimazolebothfreelycrosstheplacentabutcanbeusedinpregnancy(though
therearesomerisks,particularlywithmethimazole),whereasradioactiveiodideisclearly
contraindicatedinpregnancybecauseitpermanentlydestroysthyroidfunction.

13 Whatisthyroidstorm?Whyispropylthiouracilusedforthiscondition
insteadofmethimazole?
Thyroidstormresultsfromexcessivelevelsofthyroidhormone,causingasubstantialelevationinthe
BMRandextremefeverinwhichpatientscanseemtoburnuprightinfrontofyou.Itispotentially
fatal.BothPTUandmethimazoleinhibitthyroidhormonesynthesis,butbecausethethyroidhasan
abundantstoreofthyroidhormoneitmaytakeweeksforthiseffecttomanifest.However,becauseat
higherdosesPTUalsoinhibitstheconversionofT4toT3intheperipheraltissues,andT3isthe
moreactiveformofthyroidhormone,PTUcanhaveafairlyrapideffect.
Note:Bothofthesedrugscancauseafatalagranulocytosis,oftenprecededbyasorethroat,so
periodicmonitoringisthereforerequired.

14 Whyisiodidetherapygenerallyinitiated2weekspriortothyroidectomyin
hyperthyroidpatients?
Byanunknownmechanism,administrationofalargeamountofiodidedecreasesthevascularityof
thethyroidgland,whichreducesbleedingcomplicationsduringsurgery.Excessiodidealsoappears
toinhibitthesynthesisofthyroidhormonesandtheirreleasefromthethyroidgland(thesocalled
WolffChaikoffeffect),whichmaybeanintrinsicmechanismtoprotectagainsthyperthyroidismina
settingofiodineexcess.

Somedifferentialdiagnosisandphysiologyconcepts
15 HowwouldapatientwithhyperthyroidismsecondarytodeQuervain's
thyroiditis(subacutegranulomatousthyroiditis)typicallypresentclinically?
Patientswiththisdiseasehaveanexquisitelytenderthyroidglandandgenerallyhavesignsof
infection(e.g.,fever).

16 Howcanateratomaproducehyperthyroidism?
Arareformoffemaleteratomacalledstrumaovariiismadeupexclusivelyoffunctionalthyroid
tissue,whichcanproduceenoughthyroidhormonetocauseclinicalhyperthyroidism.

Hyperthyroidismclassicallypresentswithsomecombinationofthefollowing:heat
intolerance,weightlossdespiteanexcellentappetite,tremor,palpitations,anxiety,
diarrhea,osteoporosis,hypercalcemia,atrialfibrillation,andhighoutputheartfailure.
Gravesdiseaseisthemostcommoncauseofhyperthyroidismandmaybeadditionally
characterizedbyexophthalmos,pretibialmyxedema,andantibodies(IgG)tothethyroid
stimulatinghormone(TSH)receptorinthethyroid.Toxicmultinodulargoiteristhe
secondmostcommoncauseofhyperthyroidismdisease.
RadiolabelediodideuptakeisdiffuselyincreasedinGravesdiseasebutdecreasedin
hyperthyroidismcausedbyexogenousthyroidhormoneadministrationorintransient
casesofhyperthyroidismduetothyroiditis.
Thyroidhormonesincreasethebasalmetabolicrate(BMR)principallythrough
increasingtheproductionandinsertionoftheNa+/K+ATPasepumpsinvariouscell
types,aswellasbystimulatingtheproductionofglycolyticenzymes.
Thyroxine(T4)isaprohormonewhoseactivityislargelydependentonconversionto
themoreactivetriiodothyronine(T3).
Thyroidhormonesynthesis:(1)uptakeofiodideion(I)fromplasmaintofollicular
cellsandthefollicularlumen(2)oxidationofiodideion(I)toiodine(I2)and

attachmentofiodinetotyrosineonthyroglobulin,bothcatalyzedbythyroidperoxidase
(3)couplingofiodinatedtyrosinemoleculestoformT4orT3and(4)endocytosisof
modifiedthyroglobulinintofollicularcellsandhydrolysistoT4andT3,whichthen
diffuseacrosstheplasmamembraneandenterthecirculation.
Thyroidhormonesincreasethesensitivitytocirculatingcatecholamines,contributingto
tachycardia,tremor,andpalpitations.
Pretibialmyxedemaisanonpittingedemacausedbyaccumulationofinterstitial
glycosaminoglycans(GAGs)withinthedermis.
TreatmentoptionsforGravesdiseaseincludethionamides,surgery,radioactiveiodine
131ablation,betablockers,andwatchfulwaiting.
Thethionamidespropylthiouracil(PTU)andmethimazolearelargelyconcentratedin
thethyroidandinhibitthyroidhormonesynthesis.
Iodidetherapybeforesurgeryreducesthevascularityofthethyroidandthereforethe
bleedingcomplicationsofthesurgery.ViatheWolffChaikoffeffect,excessiodidealso
inhibitsthyroidhormonesynthesis,whichmaybeanintrinsicmechanismtoprotect
againsthyperthyroidisminsettingsofiodineexcess.
SummaryBox:HyperthyroidismandGravesDisease

Case88
A42yearoldwomancomplainsofrecentweightgain,fatigue,andheavyperiods
(menorrhagia).Onphysicalexaminationsheappearspale,speaksslowly,andhasadiffusely
enlargednontenderthyroidglandandayellowishtingetoherskin.

1 Whatdoyoususpectatthispoint?
Weightgain,fatigue,andmenorrhagia,aswellasconstipationandcoldintolerance(whichthis
patientdoesnothave),areallclassicsymptomsofhypothyroidism.Thepallorandpsychomotor
retardationalsopointtowardhypothyroidism.However,otherconditionssuchasdepressionand
anemianeedtobeconsidered.

Case88continued:
Shedeniesanyhistoryofbipolardisorderortreatmentwithlithium.

2 Whywasaskingaboutlithiumuserelevantinthediagnosticworkupofthis
patient?

Lithiuminhibitstheuptakeandorganificationofiodinebythethyroidglandandalsoinhibitsthe
peripheralconversionofT4toT3by5monodeiodinase,therebycausinghypothyroidism.
Amiodarone,anantiarrhythmicagent,isalsoknowntocausehypothyroidism.

Case88continued:
WorkuprevealselevatedTSHandreducedT4.

3 Whatisthediagnosis?
Shehashypothyroidism,themostcommoncauseofwhichisHashimoto's(autoimmune)thyroiditis.
Othercausesofhypothyroidismincludesubacutegranulomatousthyroiditis(deQuervain's
thyroiditis),Reidel'sthyroiditis,iatrogeniccauses(e.g.,thyroidectomy,thyroidradiotherapy),
cretinism,endemicgoiter,andmedicationssuchaslithiumandamiodarone.
Forboards,youshouldrecognizethefollowingassociationswithregardtohypothyroidism:
Hashimoto'sthyroiditisiscausedbyautoimmunedestructionofthethyroidgland.
DeQuervain'sthyroiditistypicallydevelopsafteraviralupperrespiratorytractinfection.
Reidel'sthyroiditisiscausedbyfibrosisofthethyroidglandsuchthatthethyroidglandmay
haveawoodyconsistencyonexamination.
Cretinismresultsinshortstature,protrudingumbilicus,potbelly,andcoarsefacialfeatures,
includingaprotuberanttongue.Mentalretardationoccurssecondarytoiodinedeficiencyat
earlydevelopmentalstages,andendemicgoiterresultsfromdietaryiodideinsufficiencyin
adulthood.

4 Doesthispatienthaveprimary,secondary,ortertiaryhypothyroidism?
Thiswomanhasprimaryhypothyroidism.Reducedhormoneproductionbythethyroidgland
disinhibitsthehypothalamicpituitaryaxis,resultinginincreasedTRHandTSH.Secondaryand
tertiaryhypothyroidismiscausedbypituitaryandhypothalamicdysfunction,respectively(Table8
11(t0060)).TSHlevelswouldnotbeelevatedineitherofthesecases.
Table811
FormsofHypothyroidism

ClinicalForm

T4/T3 TSH TRH

Primaryhypothyroidism

Low

High High

Secondaryhypothyroidism Low

Low High

Tertiaryhypothyroidism

Low Low

Low

Subclinicalhypothyroidism Normal High Normal


T3,triiodothyronineT4,thyroxineTRH,thyrotropinreleasinghormoneTSH,thyroidstimulatinghormone.

5 Whatdiagnosisshouldyoususpectinahospitalizedpatientwithabnormal
thyroidhormonelevels?
Sickeuthyroidsyndromereferstoabnormalitiesinthyroidfunctionthatoccurinillpatientswithout
underlyingthyroidorpituitarydisease.Sickeuthyroidsyndromemayinpartbecausedbyreduced
peripheralconversionofT4toT3,andcommonlyoccursfollowingillness,nutritionaldeficiencies
andglucocorticoidadministration.Theexactcauseofsickeuthyroidsyndromeisnotknown,butitis
thoughttobethebody'sattempttoconservecaloriesduringstatesofcaloricdeficitorincreased
caloricneed.Becauseoftheprevalenceofsickeuthyroidsyndromeandthedifficultyininterpreting
thyroidstudiesinsickpatients,mostendocrinologistsdonotrecommendtestingthyroidfunctionin
hospitalizedpatients.

Case88continued:
Furtherworkuprevealsthepresenceofplasmaantimicrosomal(antiperoxidase)antibodies,
andathyroidbiopsyshowsadiffuselymphocyticinfiltrate.

6 Whatisthediagnosis?WhymightyoualsowanttocheckserumvitaminB12
levelsinthispatient?
Plasmaantimicrosomal(antiperoxidase)antibodiesandbiopsyshowinglymphocyticinfiltrationof
thethyroidglandistypicalofHashimoto'sthyroiditis.Thisdiseaseisfrequentlyassociatedwith
autoimmuneconditionssuchasperniciousanemia,whichischaracterizedbyimpairedabsorptionof
vitaminB12(Table812(t0065)).Therefore,checkingavitaminB12levelwouldnotbeunreasonable.
Autoimmuneconditionsaremostwidelyseeninyoungtomiddleagedfemales.Youare
expectedtoknowtheantibodiesassociatedwithcommonautoimmuneconditions,andare
likelytobeaskedatleastonequestionfromTable812(t0065).
Step1Secret
Table812
AutoimmuneConditions

Disease

Autoantibody

Hashimoto'sthyroiditis

Antimicrosomal,antithyroglobulin

SLE

Antinuclearantibody(mostsensitive,nonspecific)

AntidsDNA(presentwithlupusassociatedrenaldisease
indicatespoorprognosis)
AntiSmith(veryspecific)
Druginducedlupus

Antihistone

Scleroderma

Anticentromere(CREST)
AntiScl70/antitopoisomerase(diffuse)

Gravesdisease

AntiTSH

Myastheniagravis

AntiAChR

Perniciousanemia

Antiintrinsicfactor,antiparietalcell

Rheumatoidarthritis

Rheumatoidfactor(antiIgG)

Primarybiliarycirrhosis

Antimitochondrial

Sjgren'ssyndrome

AntiRo,antiLa

Celiacdisease

Antigliadin,antiendomysial

Autoimmunehepatitis

Antismoothmuscle

Goodpasturedisease

Antibasementmembrane

Wegener'sgranulomatosis

cANCA(cytoplasmic)

Microscopicpolyangiitis

pANCA(peripheral)

Pauciimmunecrescentic

MPOANCA

glomerulonephritis
Polymyositis

AntiJo1

Dermatomyositis
Diabetestype1

Antiglutamicaciddecarboxylase

AChR,acetylcholinereceptorantibodyANCA,antineutrophilcytoplasmicantibodiesCREST,calcinosis,R
aynaudsyndrome,esophagealdysmotility,sclerodactyly,telangiectasiadsDNA,doublestrandedDNAIgG,
immunoglobulinGMPO,myeloperoxidaseSLE,systemiclupuserythematosusTSH,thyroidstimulating
hormone.

7 Whydoesthispatienthavetheweightgainandyellowskin?
Thispatient'sweightgainisduetoherlowmetabolicstate(i.e.,lowBMR)alongwithretentionof
saltandwater.Theretentionofsaltandwaterwillputheratriskforcongestivecardiomyopathy.The
yellowingofherskinisduetotheimpairedconversionofcarotenesintoretinoicacid,whichis
normallydrivenbythyroidhormone.

8 Givenherhistoryofmenorrhagia,whathematologicdisordershouldwebe
worriedabout?
Irondeficiencyanemiaisaconcern.Becausesheishavingheavyperiods,sheislosingbloodand
iron.Thecombinationofpallorandfatiguealsopointstothiscondition.

Case88continued:
Thepatientreturns10yearslaterforevaluationofprofoundfatigue.Examinationissignificant
forperiorbitaledema,blunteddeeptendonreflexes,andsinusbradycardia.Laboratoryworkup
showssodiumlevelof126mEq/dL(normalrangeis135145mEq/dL).

9 Whatisthediagnosis?
Myxedemacoma,whichcanmanifestasprofoundlethargyorcoma,weakness,hypothermia,
hypoventilation,hypoglycemia,andhyponatremia,isthediagnosis.

Somedifferentialdiagnosisconcepts
10 Ifthiswoman'shistorywassignificantforarecentupperrespiratory
infectionandherthyroidwastendertopalpation,whatwouldbethe
probablediagnosis?
Thosefindingswouldsuggestsubacutethyroiditis(deQuervain'sthyroiditis).Thisdiseasestartsout
ashyperthyroidism,duetoinflammationcausingreleaseofstoredthyroidhormones,butthen
progressestohypothyroidism.Itisthoughttoinvolveviralinfectionofthethyroidglandand
classicallyoccursfollowinganupperrespiratoryinfection.Itusuallyresolvesonitsown.

11 Howcanathyroidectomycausemusclecrampsandparesthesias?
Accidentalremovaloftheparathyroidglandsmayoccurwiththyroidectomy.Thiscanleadto
hypocalcemia,whichmanifestswiththesesymptoms.

12 Whatisthetreatmentoptionforhypothyroidism?
Thyroidhormonereplacement(levothyroxine)isneeded,withagoalofnormalizingTSHand
relievingsymptomsofhypothyroidism.

Relatedquestions
13 Howisitpossibleforathyroidglandtodevelopatthebackofthetongue?
Thethyroidbeginsitsdevelopmentatthebackofthetongueandthenmigratestoitspositionbelow
thethyroidcartilageintheneck.Failuretomigratealongthethyroglossalductmaythereforeresult
inathyroidglandatthebackofthetongue.Thisisanimportantembryologiccorrelationtomakefor
Step1.

Note:Persistenceofthethyroglossalductthatfacilitatesthemigrationofthethyroidcancausea
thyroglossalductcyst.

14 Whyarethyroidhormonelevelsroutinelyevaluatedinnewborns?
Hypothyroidismisoneofthepreventablecausesofmentalretardation.

15 CoverthetwocolumnsontherightsideofTable813(t0070)andlistthe
manifestationsofhypothyroidismandhyperthyroidismforeachfeaturein
column1
Hypothyroidismclassicallypresentswithsomecombinationofthefollowing:weight
gain,fatigue,coldintolerance,constipation,menorrhagia,anddepression.
ThemostcommoncauseofhypothyroidismisHashimoto'sthyroiditis,whichiscaused
byautoimmunedestructionofthethyroidgland.Hashimoto'sthyroiditisisoften
associatedwithotherautoimmuneconditionssuchasperniciousanemia.
Othercausesofhypothyroidismincludesubacutegranulomatousthyroiditis(de
Quervain'sthyroiditis),Reidel'sthyroiditis,iatrogenicthyroiditis(e.g.,causedby
thyroidectomyorthyroidradiotherapy),cretinism,endemicgoiter,andmedicationssuch
aslithiumandamiodarone.
Sickeuthyroidsyndromereferstoabnormalitiesinthyroidfunctionthatoccurinill
patientswithoutobviousthyroidorpituitarydisease.Itisverycommoninhospitalized
patientsandmayberelatedtoreducedperipheralconversionofthyroxine(T4)to
triiodothyronine(T3).
Treatmentforhypothyroidismisreplacementtherapywiththyroidhormone,withagoal
ofnormalizingthyroidstimulatinghormone(TSH)andrelievingsymptoms.
Typicallyinthepresenceofastressorsuchasinfection,untreatedhypothyroidismcan
occasionallyprogresstomyxedemacoma,whichcanmanifestasprofoundlethargyor
coma,weakness,hypothermia,hypoventilation,hypoglycemia,andhyponatremia.
Hypothyroidismisoneofthepreventablecausesofmentalretardation.
SummaryBox:Hypothyroidism

Case89
Apreviouslyhealthy12yearoldboypresentscomplainingoffatigueandexcessivethirst
(polydipsia).Hismothermentionsthatheusesthebathroomquitefrequently(polyuria),and
hisfriendsatschoolalwaysteasehimaboutthis.Hismotherisalsoconcernedbecausehehas

lost10lbdespiteeatingeverythinginsight.Arandom(nonfasting)plasmaglucoselevelis
220mg/dL.

Table813
ManifestationsofHypothyroidismandHyperthyroidism

Feature

Hypothyroidism

Hyperthyroidism

Metabolicrate

Decreased

Increased

Bodyweight

Gain

Loss

Intestinalactivity

Constipation

Diarrhea

Mentalstatus

Memoryloss/dementia Psychosis,agitation

Bodytemperature

Coldintolerance

Heatintolerance

Deeptendonreflexes

Hypoactive

Hyperactive

Mostseverecomplication Myxedemacoma

Thyroidstorm

1 Whatisthelikelydiagnosis?
Hemostlikelyhasdiabetesmellitus,probablytype1givenhisyoungage,butitcouldjustaseasilybe
earlyonsettype2associatedwithsedentarylifestyleandobesity.

2 Differentiateamongtype1,type2,andmaturityonsetdiabetesofyouth
Type1diabetesiscausedbyautoimmunedestructionofthebetacells.Itclassicallyoccursinchildren
andadolescents.Patientsaretypicallythin,andtheirinitialpresentationmaybeoneofdiabetic
ketoacidosis(DKA).
Type2diabetesiscausedbyacombinationofinsulinresistanceandbetacelldysfunction.It
classicallyoccursinsedentaryandoverweightadults.Type2diabeticsarepronetohyperosmolar
nonketoticsyndromeandmayoccasionallyexperienceDKA.
Maturityonsetdiabetesofyouth(MODY)isagroupofdominantlyinheriteddisorderscausedby
impairedinsulinsecretion.MODYmimicstype1diabetesbecauseoftheimpairedinsulinsecretion.
TherearecurrentlysixknowntypesofMODY,withMODY2andMODY3beingthemostcommon.
PatientswithMODYtypicallydonotexperienceDKAbecausetheyproduceenoughinsulinforfatty
aciduptake(Fig.817(f0090)).

Case89continued:

Anoralglucosetolerancetest(OGTT)showsplasmaglucoselevelof225mg/dL2hoursafter
theadministrationofa75gglucoseload.Furtherworkuprevealslowlevelsofplasmainsulin
andCpeptide.

Figure817
Chroniccomplicationsofdiabetesmellitus.MI,myocardialinfarction.

(FromMeszarosJG,OlsonER,NaugleJE,etal:CrashCourse:EndocrineandReproductiveSystems.
Philadelphia,Mosby,2006.)

3 Whichtypeofdiabetesdoesthisboyhave?
Type1diabetesmellitusisassociatedwithlowlevelsofinsulinandCpeptideduetoautoimmune
destructionofthebetacells,asdiscussedpreviously.RecallthatCpeptideiscosecreted(in
equimolaramounts)withinsulin.

4 HowcanmeasurementsofplasmaCpeptidebeusedtodifferentiatebetween
factitioushypoglycemiaandaninsulinoma?
BecauseplasmaCpeptideiscosecretedwithendogenousinsulin,theywillbothbehighinan
insulinomabutverylowinfactitioushypoglycemia,inwhichonlyrecombinantinsulinisinjected.

5 Explainthemechanismbywhichthemajormetabolicpathwaysbehaveas
thoughthebodyisinthefastingstateduringinsulindeficiency(i.e.,whyhas
thispatientlostweight)?
Afterconsumptionofameal,insulinsecretionisstimulated,whichinturnactivatesglycolysis,
glycogenesis,fattyacidsynthesis,andproteinsynthesis.Inthesettingofinsulindeficiency,allthese
pathwaysbecomelessactiveandtheopposingpathways(gluconeogenesis,glycogenolysis,andfatty
acidcatabolism)arestimulatedinessence,insulindeficiencycausesahypercatabolicstate.

6 Whatisthebiochemicalmechanismbywhichdiabeticketoacidosisdevelops
inthesettingofinsulindeficiency?
Ordinarily,insulinstimulatesfattyaciduptakebyadipocytesviastimulatinglipoproteinlipase.In
theabsenceofinsulin(orsignificantinsulindeficiency)fewerfattyacidsaretakenupbythe
adipocytes,andthesefattyacidsarethendeliveredtotheliver,wheretheyaremetabolized,and
ketoacidsareabyproduct.Furtherexacerbatingthisproblem,becauseinsulinnormallyinhibitsfatty
acidcatabolismbytheliver,intheabsenceofinsulinthispathwayisevenmoreactiveandmore
ketonebodiesareproduced.Finally,theacidosisisexacerbatedbecausethecorresponding
hyperglycemiacausesdehydration,makingitmoredifficultforthekidneystoexcreteacid.

7 Whatistheshorttermvalueofcontrollingthebloodsugarinthisboy?
Thiswillpreventthesymptomsofhyperglycemia,suchaspolyuria,polydipsia,andpolyphagia,as
wellaspreventingweightloss.Additionally,intype1diabetes,thecorrectionofinsulindeficiency
willpreventDKA.

8 Whatisthelongtermvalueofglycemiccontrolinthisboy?
Tightglycemiccontrolhasbeenprovedtoreducetheincidenceofmicrovascularcomplications,
includingretinopathy,neuropathy,andnephropathy.However,ithasnotyetbeenshowntoreduce
macrovasculareventssuchasheartattackandstroke,althoughfuturelongtermstudiesmayshow
suchaneffect.

9 WhatisthevalueofmeasuringthehemoglobinA1clevelroutinelyinthis
patient?
ThehemoglobinA1c(HbA1c)representsglycosylatedhemoglobin,andthelevelsofHbA1care
directlyassociatedwithlevelsofplasmaglucose.Itisaneffectivemeasureoflongtermdiabetes
controlbecausethelifespanofhemoglobinladenredbloodcells(RBCs)isapproximately120days
inthecirculation.Moreover,reducedHbA1clevelshavebeenshowntocorrelatewithbetterclinical
outcomes.

10 Whyshouldbloodpressurebecloselyscrutinizedinthisboyasheagesand
highbloodpressurebetreatedaggressivelywithangiotensinconverting
enzymeinhibitorsifitdevelops?
Bloodpressurereductionsubstantiallyreducestheincidenceofnephropathyandmyocardial
infarction.Theangiotensinconvertingenzyme(ACE)inhibitorsaremosteffectiveatpreventing
nephropathy.

11 Forboards,whyarebetablockersrelativelycontraindicatedindiabetics?
Betablockersmaymaskthewarningsignsofhypoglycemia,suchastremors,shakes,and
tachycardia.Additionally,byantagonizinghepaticreceptors,thesedrugsmakeitmoredifficultfor
thelivertorespondtoepinephrine,acounterregulatoryhormonethatelevatesthebloodglucoseby
stimulatingglycogenolysisandgluconeogenesis.Intype1diabetes,hypoglycemiaispredominantly
duetoinsulinoverdosing.

Case89continued:
Thepatientisdiagnosedwithtype1diabetesmellitusandstartedoninsulintherapy.Onenight
heinadvertentlytakeshisnighttimeinsulindosetwice.Henoticesnoilleffectsthatnight,but
thefollowingmorninghisprebreakfastglucosevalueissubstantiallyhigherthannormal.

12 Whathashappened?
Insulininducedhypoglycemiaduringthenighttriggersreleaseofstresshormones(e.g.,cortisol,
GH,glucagon,catecholamines),whichcauseacompensatoryincreaseinplasmaglucose.Ifthe
patientisnoteducatedaboutthis,hemayinappropriatelyincreasethenighttimedoseofinsulinto
reducethemorningbloodsugarlevelsandpotentiallyprecipitatehypoglycemiccomaorevendeath
duringthenight.

Type1diabetesusuallyoccursinchildrenandadolescents.Newlydiagnosedpatientsare
oftenthinandmaypresentinitiallyindiabeticketoacidosis(DKA).
Thepathophysiologyoftype1diabetesisrelatedtoinadequateinsulinsecretion.The
bodythereforebehavesasifitisinthefastingstate,andtheprocessesofgluconeogenesis,

glycogenolysis,andfattyacidcatabolismareallstimulated.Treatmentfortype1diabetes
istheadministrationofinsulin.
DKAoccursbecauseintheabsenceofinsulin,ratherthanbeingtakenupbyadipocytes,
fattyacidsaredeliveredtotheliverandmetabolizedtoketonebodies.Thehyperglycemia
alsocausesanosmoticdiuresis,whichcausesdehydrationandexacerbatesthemetabolic
derangements.
Maturityonsetdiabetesoftheyoung(MODY)isanautosomaldominantinheritance
disorderinwhichpeoplepresentwithmildhyperglycemiaduetoimpairedglucose
inducedreleaseofinsulin.
Longtermtightglycemiccontrolwillretardthedevelopmentofthemicrovascular
(retinopathy,neuropathy,nephropathy)andmacrovascular(myocardialinfarction,
peripheralvasculardisease)complicationsofdiabetes.
ThehemoglobinA1c(HbA1c)reflectsthepercentglycosylatedhemoglobinatanytime.
Itisaneffectivemeasureoflongtermdiabetescontrolbecausethelifespanof
hemoglobinladenredbloodcells(RBCs)isapproximately120days.ReducedHbA1c
levelshavebeenshowntocorrelatewithbetterclinicaloutcomes.
Betablockersmaymaskthewarningsignsofhypoglycemia,suchastremors,shakes,
andtachycardia.
Takingtoomuchnighttimeinsulinmayresultinanelevatedmorningplasmaglucose
levelduetothepronouncedsympatheticresponsetohypoglycemia.
SummaryBox:Type1DiabetesMellitus

Case810
Amiddleagedobesemanevaluatedforapreemploymentphysicalexaminationcomplainsof
increasedthirstandfrequenturination.Hisnonfasting(random)plasmaglucoselevelis275
mg/dL.Bothofhisparentswereoverweight,hadsugarproblems,anddiedofcardiovascular
complications.

1 Doesthispatientmorelikelyhavetype1ortype2diabetesmellitus?
Type2diabetesmellitusismostcommonlyseeninoverweightsedentarymiddleagedadultswitha
strongfamilyhistoryofdiabetes.Manyofthesepatientsarediagnosedaftercomplainingof
increasedthirst(polydipsia)andincreasedurinaryfrequency(polyuria),althoughmorecasesare
nowdiagnosedfromroutinescreeninginasymptomaticpatients.

2 Whatisthepathogenesisoftype2diabetesmellitus?

Theearlystageischaracterizedbyinsulinresistanceandhyperinsulinemiabutarelativeinsulin
deficiency(duetoinsulinresistance).Thelaterstagesarecharacterizedbybetacelldysfunction
(burnout),whichmayresultininsulindeficiency.
Whetherduetoinsulinresistanceorinsulindeficiency,hyperglycemiauniformlyoccursintype2
diabetes.Hyperglycemiaresultsfromincreasedlipolysisinadiposetissues(withtheglycerolacting
asagluconeogenicsubstrateintheliver)aswellasreducedglucoseuptakebyskeletalmuscleand
adiposetissue.
Note:Theincreasedatherogenesisthatoccursindiabetesmaybeexplainedinpartbytheincreased
plasmalevelsoffreefattyacids.

3 Howdoesbindingofinsulintotheinsulinreceptorresultinglucoseuptake
intocells?
ItcausesGLUT4(insulindependentglucosetransporter)tobeincorporatedintotheplasma
membraneofcellsinskeletalmuscleandadiposetissue.Glucoseisthencotransportedwith
potassiumintothecell.

4 Whatistheprimarymetabolicfuelinthefasting(betweenmeals)state?
Fattyacidsaretheprimaryfuel.AnotableexceptiontothisistheCNS,whichreliesexclusivelyon
serumglucoseinboththefedandfastingstates,exceptinperiodsofprolongedstarvation,inwhich
casetheCNSwillmetabolizeketonebodiesaswell.

Case810continued:
Youtaketimetoeducatehimaboutdiabetesandplanafollowupvisitfor2weeks,when
fastinglaboratorytestsshowplasmaglucoseof180mg/dL,totalcholesterolof250mg/dL,
lowdensitylipoproteincholesterolof175mg/dL,andhighdensitylipoproteincholesterolof
30mg/dL,andaurinalysisrevealsmicroalbuminuria.Heistoldhehasdiabetesandis
educatedabouttheimportanceofdietandexercise.Pharmacotherapyisstartedwith
metformin.Hereturnstotheclinic3monthslaterandhisHbA1chasdecreasedfrom8.5%to
7.5%.

5 Whymighttherapywithabiguanidesuchasmetforminmakesenseinthis
patient?
Thispatienthastype2diabetesmellitus,isobese,andhasdyslipidemia,allofwhichcanbe
improvedbymetformintherapy.
Metformin(Glucophage)isawonderdrugforthemanagementoftype2diabetesmellitus.It
normalizesplasmaglucoseprimarilybyinhibitinghepaticglucoseproductionandbystimulating
peripheraluptakeofglucosebyadiposeandskeletalmuscle.Furthermore,ithasabeneficialeffect

onthelipidprofile.Inaddition,itisverycheap.Finally,andperhapsmostimportanttosome
patients,ithasananorexiceffectandmayresultinmodestweightloss.
Note:Metformincanveryrarelycausealifethreateninglacticacidosis.Forthisreason,itshouldbe
avoidedinpatientswithcongestiveheartfailure(CHF),liverdisease,orrenaldisease.Intermsof
cardiacriskstratification,diabetesisconsideredacoronaryheartdisease(CHD)riskequivalent,and
lowdensitylipoprotein(LDL)targetgoalsaretherefore<100mg/dLindiabetics.Mostdiabetics
shouldbeonastatin,ACEinhibitor,anddailyaspirin.

6 Whatotherclassesoforalhypoglycemicagentsareavailabletotreattype2
diabetesmellitus?
Sulfonylureassuchastolbutamide,glucosidaseinhibitorssuchasacarbose,andperoxisome
proliferatoractivatedreceptor(PPAR)agonists(glitazones)suchaspioglitazone.

7 Whydotheglucosidaseinhibitorscausefrequentgastrointestinalsymptoms
andannoyingflatulence?
Thisclassofdrugs,whichincludesacarboseandmiglitol,workbyinhibitingintestinal
glucosidases(e.g.,sucrase,maltase,isomaltase)thatbreakdowndisaccharidesintomonosaccharides
thatcanbeabsorbedbytheintestines.Theundigestedsugarsaremetabolizedbycolonicbacteriato
generatelargevolumesofgas.
Note:Thesedrugsdonotcausehypoglycemia,butintheeventthathypoglycemiaoccursfroma
differentoralhypoglycemicagentwhilethepatientistakingoneofthesedrugs,oralglucoseshould
begivenbecauseitsintestinalabsorptionwillnotbeimpeded.Clearly,intravenousglucosewouldbe
giveninahospitalsetting.

8 Whatisthemechanismofactionofthesulfonylureas?
Sulfonylureassuchastolbutamide(firstgenerationagent)andglyburide(secondgenerationagent)
actbystimulatinginsulinsecretion.TheydothisbyclosingmembranespanningK+channelson
pancreaticbetacells.Thisresultsindepolarizationofthecell,whichtriggersopeningofvoltage
gatedcalciumchannelsontheplasmamembrane.Theresultantinfluxofextracellularcalcium
stimulatesinsulinsecretion,whichlowersplasmaglucose.

9 Whataresomesideeffectsofsulfonylureas?
Improvedglycemiccontrolviaincreasedinsulinsecretionmayresultinweightgain,becauseinsulin
stimulatesfatsynthesis.However,themoreserioussideeffectofsulfonylureasistheirpropensityto
causehypoglycemiabycausingexcessiveinsulinsecretion,particularlyifamealisskipped.
Firstgenerationsulfonylureascancauseadisulfiramlikereactionandarenowrarelyused.

10 Whyshouldthispatientbeeducatedabouttheimportanceofexamininghis
feetperiodically?

Longstandinghyperglycemiaindiabetesisassociatedwithmicrovasculardisease,aswellasdiabetic
neuropathy.Themicrovasculardiseasemaycausepoorperfusionofthefeet(macrovasculardisease
candothis,too),suchthatfootulcersdonothealwell.Inaddition,diabeticneuropathyallowsulcers
tofesterwithoutcausinganynoticeablepain.
Note:Diabeticneuropathyistypicallyinastockingglovedistribution,withthedistalfeetaffected
beforethemoredistalhands.Thisstockingglovedistributionisseeninothermetabolic
neuropathiesaswell,asthelongeraxonsaremoresusceptibletoametabolicabnormality.

Case810continued:
Thepatient'screatininegraduallyincreasedfrom0.7to3.2mg/dLover2years.Arenalbiopsy
isasshowninFigure818(f0095).

11 Whatrenalpathologyshouldyoususpect?
DiabeticsaresusceptibletonodularglomerulosclerosisandthesocalledKimmelsteinWilsonlesion.
TheimageinFigure818(f0095)showsnodularglomerulosclerosiswithexpansionofthemesangium
byintenselyPAS(periodicacidSchiff)positivematerialbutwithoutappreciablethickeningofthe
glomerularcapillarywalls.

Figure818
Lightchaindepositiondisease.
(FromBrennerBM:BrennerandRector'sTheKidney,7thed.Philadelphia,WBSaunders,2004.)

12 Howcanpoorglycemiccontrolcausethismantogointocoma?
Ifheoverdosesonsulfonylureas,hecangointoahypoglycemiccoma.Ontheotherhand,ifhisblood
sugarrunstoohigh,hecandevelophyperosmolarnonketoticcoma.Finally,althoughtype2
diabeticsrarelygointoDKAbecauseofthepresenceofsomeinsulin,ifhebecomesill,DKAis
anotherpossibility.

Type2diabetesmellitusaccountsforapproximately90%ofcasesofdiabetes.Most

peoplewithtype2diabetesaresedentaryandoverweight.
Theinitialstageoftype2diabetesiscausedbyinsulinresistanceandhyperinsulinemia.
Thelaterstageiscausedbybetacelldysfunctionandimpairedinsulinsecretion.
Clinicalfindingsintype2diabetesincludepolyuria,polydipsia,recurrentblurryvision,
andrecurrentinfections.
Treatmentfortype2diabetesisexercise,weightloss,andoralhypoglycemicagents.
Sometype2diabeticsrequireinsulin.
SummaryBox:Type2DiabetesMellitus

Case811
A33yearoldpregnantwomanwithanunremarkablemedicalhistoryisadmittedtothe
hospitalforher24weekgestationalscreening.Onehouraftertheadministrationofa50g
glucoseload,herplasmaglucoseis166mg/dL.Shehasnohistoryofdiabetes,andnoneofher
familymembershavediabetes.Sheisaskedtoreturn1weeklater,anda3hourglucose
tolerancetest(100gglucosedrink)isadministered.HerresultsareshowninTable814(t0075)
.
Table814
ResultsofGlucoseToleranceTestinCase811(st0725)

Time

SerumGlucoseLevel NormalRange

Fasting

95mg/dL

95mg/dLorbelow

At1hour

200mg/dL

180mg/dLorbelow

At2hours 181mg/dL

155mg/dLorbelow

At3hours 160mg/dL

140mg/dLorbelow

1 Whatisthediagnosis?
Thiswomanhasgestationaldiabetes.Gestationaldiabetesisdefinedasdiabetesdiagnosedforthe
firsttimeduringpregnancyandresolving6weeksormoreafterthepregnancyends.Itportendsan
increasedriskfordevelopingtype2diabeteslaterinlife.
Allpregnantwomenshouldbescreenedwhentheyarebetween24and28weeksgestation.Ina50g
glucosechallengetest(GCT),a1hourpostglucosechallengeplasmaglucoselevelof>140mg/dLisa
positivefinding.Confirmationofgestationaldiabetesisdoneusinga3houroralglucosetolerance
testwithapositiveresulthavingtwoormorevaluesabovethethresholdvalue.

2 Whatisthecauseoftherelativematernalinsulinresistancethatdevelops
duringpregnancyandhowisthisvaluabletothefetus?
Maternalinsulinresistanceduringpregnancyisthoughttoresultfromtheplacentalsecretionof
humanplacentallactogen(hPL),aglycoproteinthatantagonizestheactionsofmaternalinsulin.
Becauseglucosemovesacrosstheplacentabypassivediffusion,thisphysiologicalterationmay
facilitatedeliveryofglucosetothefetus.

3 Whatisthemostcommonundesiredeffectofmaternaldiabetesonfetalsize
andwhydoesthishappen?
Fetalmacrosomia(largefetus)isacomplicationofmaternaldiabetesandisaproblembecauseofthe
increasedriskofbirthinjurywhenanoversizedfetuspassesthroughthebirthcanal.Fetalsizeis
increasedbecausethematernalhyperglycemiastimulatesincreasedfetalinsulinsecretion,whichin
turnstimulatesfetalgrowth.

4 Shouldhypoglycemiaorhyperglycemiabeexpectedinababyborntoapoorly
controlleddiabeticmotherimmediatelyfollowingdelivery?Explain
Hypoglycemiaisexpected.Duringinuterolife,elevatedfetalglucoselevels(secondarytohigh
maternalglucoselevels)causechronicfetalhyperinsulinemia.Justafterdelivery,whenthefetusis
nolongerexposedtotheelevatedmaternalglucose,theresidualhyperinsulinemiacancause
hypoglycemia.Thetreatmentforthisistogivethebabyglucoseafterbirth.

5 Whatrespiratorysyndromeisthefetusatriskforatbirthifthegestational
diabetesisnotcorrectedinthemother?
RespiratorydistresssyndromeisduetothelackofsurfactantsynthesizedbytypeIIpneumocytes.
SurfactantsynthesisisdecreasedbyinsulinandincreasedbycortisolandT4.Becausemotherswith
gestationaldiabeteshavehigherthannormallevelsofinsulin,thisrisktothenewbornshouldbe
noted.

Gestationaldiabetesisdefinedasdiabetesdetectedforthefirsttimeduringpregnancy
andresolving6weeksormoreafterthepregnancyends.Itisduetotheinsulinresistance
associatedwithpregnancy.
Allpregnantwomenshouldbescreenedwithaglucosechallengetestbetween24and28
weeksgestation.A1hourafterthechallengetestaglucoselevelof>140mg/dLisa
positivescreen.Thisthenneedstobeconfirmedwitha3hourglucosetolerancetest.
Gestationaldiabetesportendsanincreasedriskfordiabeteslaterinlife.Italso
predisposesnewbornstomacrosomia,hypoglycemia,andrespiratorydistresssyndrome.
SummaryBox:GestationalDiabetes

Case812
A42yearoldregisterednursecomplainsofepisodesoftremor,diaphoresis,andpalpitations
severalhoursaftereating.Shealsonotessimilarsymptomswhenfirstwakinginthemorning.
Symptomsarealleviatedbyeating.Herplasmaglucoseislowduringsymptomaticepisodes,
rangingfrom30to60mg/dL(shehasaccesstoaglucometerbecauseherhusbandisdiabetic).
Reviewofsystemsissignificantonlyfora15lbweightgaininrecentmonths.Physical
examinationisunrevealing.

1 Whatisthedifferentialdiagnosisforhypoglycemiainthiswoman?
Hypoglycemiacanbereactiveorcanbeduetoinsulinexcess.Reactiveorpostprandial
hypoglycemiacanoccurinvarioussituations(moreonthislater).Excessinsulincanbeduetoeither
anendogenoussource(insulinoma,nesidioblastosis)oranexogeneoussource(factitious
hypoglycemia).
Forboards,realizethatapatientwithasignificantpsychiatrichistoryorwithaccesstoprescription
drugs(e.g.,nurses,doctors)issuspectforexogenousinsulinadministration(factitious
hypoglycemia).

2 Whatisreactive(postprandial)hypoglycemia?
Reactivehypoglycemiacanbecategorizedasfunctionaloralimentaryorrepresentingearly(occult)
diabetes.
Functionalhypoglycemiaisthemostcommontype.Itoccursfollowingmealsandisassociatedwith
highenergy,typeApersonalities.Themechanismofthehypoglycemiaisunclearbutisthoughtnot
toberelatedtoexcessiveinsulinsecretion.
Alimentaryhypoglycemiaoccursinresponsetorapidglucoseabsorption.Thisistypicallyseen
followinggastricresections,inwhichgastriccontentsaredeliveredtothesmallbowelatarapidrate,
resultinginasurgeofinsulinsecretionandhypoglycemia.Symptomswilloftenrespondtoreduced
carbohydrateintakeaswellassmaller,morefrequentmeals.
Earlyoroccultdiabetesmellituscanalsocausehypoglycemia.Themechanismisfelttoberelatedto
adelayinearlyinsulinreleasefrombetacells,resultinginhyperglycemia.Anexaggeratedlatephase
insulinsecretionthenoccursinresponsetothehyperglycemia.

3 HowcantheCpeptidelevelhelpdifferentiatefactitiousfromtrue
hypoglycemia?
Cpeptideiscosecretedwithinsulininequivalentamounts.Exogenousinsulinpreparationsdonot
containCpeptide.Therefore,patientsselfadministeringinsulinshouldbeexpectedtohavelow
levelsofCpeptide,whereaspatientswithendogenoushyperinsulinism(e.g.,insulinoma)should
havehighlevelsofCpeptide.Itisimportanttorecognizethatsulfonylureasareanexceptiontothis

rule.SulfonylureasincreaseinsulinandCpeptidesecretionbybetacellssuchthatapatientingesting
sulfonylureaswillhaveelevatedlevelsofinsulinandCpeptide.Sulfonylureaingestioncanbevery
difficulttodifferentiatefrominsulinomatherefore,obtainingaplasmasulfonylurealevelcanbe
helpful.

Case812continued:
Thepatientreturnsthenextdaytotheclinic,andbloodworkisperformed.Fastingplasma
glucoseislowat35mg/dL,andsimultaneousplasmainsulinismarkedlyelevated.Thepatient
ismoderatelysymptomaticfromherhypoglycemia.Sheisgivensomecrackersandsodaand
hersymptomsresolve.

4 HasWhipple'striadbeensatisfiedbythispatient?
Yes.Theconstellationofdocumentedhypoglycemia,symptomsthatcanbereasonablyattributedto
hypoglycemia(e.g.,confusion),andtheresolutionofthesesymptomswitheating(orthe
administrationofglucose)isknownasWhipple'striad.Whipple'striadissuggestiveof,butnot
specificfor,aninsulinoma.

Case812continued:
Thepatientisadmittedforanobserved48hourfast.Aplasmasulfonylureascreenisnegative.
After28hoursofobservedfasting,thepatientbecomesconfusedanddiaphoretic,and
simultaneousmeasurementsofglucoseandinsulinareagainlowandhigh,respectively.An
MRIoftheabdomenshowsasmallmassinthetailofthepancreas.

5 Whatisthediagnosis?
Shortofadefinitivepathologicdiagnosis,wecanbefairlycertainthatthispatienthasaninsulinoma.
Herinsulinlevelsareinappropriatelyhighinthepresenceofhypoglycemia.Furthermore,shedoes
notappeartobeabusingsulfonylureas,andtheabdominalMRIissuggestiveofapancreatic
insulinoma.Thenextstepforthispatientwouldbesurgery.

Hypoglycemiacanmanifestclinicallyaslethargy,tremor,andpalpitations,whichare
duetostimulationofthesympatheticandparasympathetic(sweating)armsofthe
autonomicnervoussystem.
Hypoglycemiacanbefactitiousinorigin(asinahealthcareworkerwithaccessto
insulin)reactivefollowingmeals,orsecondarytoendogenoushyperinsulinism,the
mostcommoncauseofwhichisaninsulinoma.
ThediagnosisofaninsulinomarequiresdemonstrationofWhipple'striad
hypoglycemia,hypoglycemicsymptoms,andresolutionofsymptomswithglucose.

Proinsulin,whencleaved,producestwoproducts:Cpeptideandinsulin.
Anothercauseofhypoglycemiacanbenesidioblastosis,orbetaisletcellhyperplasia.
Thisisararehyperplasiadisorderofthebetaisletcellsthatresultsinexcessinsulin
secretion,leadingtoahypoglycemicstate.
Forboards,treatmentofaninsulinomaissurgicalresection.
SummaryBox:HypoglycemiaandInsulinoma

Case813
A49yearoldmanwithahistoryofrecurrentcalciumoxalatekidneystonesandpepticulcer
disease,whichhasbeenrefractorytotherapywithprotonpumpinhibitors,isevaluatedfor
newonsetvisualdeficits.MRIoftheheadrevealsanenlargedstructurelocatedinthesella
turcica.

1 Whatdoyoususpect?
Recurrentcalciumoxalatekidneystonesaresuggestiveofhyperparathyroidism,refractorypeptic
ulcerdiseaseissuggestiveoftheZollingerEllisonsyndrome,andvisualdeficitsandtheMRIfinding
aresuggestiveofapituitaryadenoma.Theconstellationofapituitaryadenoma,pancreatic
neuroendocrinetumor,andhyperparathyroidismshouldmakeonethinkofmultipleendocrine
neoplasiatypeI(MENI).However,moreinformationisneeded.

Case813continued:
Laboratoryworkuprevealselevatedlevelsofgastrin,prolactin,andPTH.

2 Whatisthediagnosis?
Hyperparathyroidism,hyperprolactinemia,presumablyfromapituitaryadenoma,and
hypergastrinemiaconfirmthediagnosisofMENI.Hyperparathyroidismisthemostcommon
abnormalityinMENI,presentinmorethan90%ofpatients.Itoccurs10to20yearsearlierthanthe
sporadicformofhyperparathyroidismandismuchmoreaggressive.Ittypicallyinvolvesallfour
parathyroidglands,andsubtotalparathyroidectomyisthereforerarelycurative.MENIiscausedby
mutationsinthemeningene,apresumptivetumorsuppressorgene.

3 Whatisthedangerofmissingadiagnosisofmultipleendocrineneoplasiatype
IIA?
MENIIAisassociatedwithmedullarythyroidcarcinoma,pheochromocytoma,andprimary
hyperparathyroidism.Cutaneouslichenamyloidosishasalsobeenrecentlyadded.Medullarythyroid
carcinoma,whichcanbelifethreatening,hasafrequencyofgreaterthan90%inMENIIApatients.

Therefore,makingthediagnosisandinitiatingearlygeneticscreeningarecritical.MENIIAiscaused
bymutationsintheRETprotooncogene.

4 WhatismultipleendocrineneoplasiatypeIIB?
MENIIBisveryrareandisassociatedwithmedullarythyroidcarcinoma,pheochromocytoma,
mucosalneuromas,intestinalganglioneuromas,andoccasionallyamarfanoidhabitus.AswithMEN
IIA,medullarythyroidcarcinomaisthemostcommoncomponent.However,unlikeinMENIand
MENIIA,primaryhyperparathyroidismisnotpresent.MENIIBisalsocausedbymutationsinthe
RETprotooncogene.

5 WhatisZollingerEllisonsyndromeandhowisitrelatedtomultipleendocrine
neoplasiatypeI?
ZollingerEllisonsyndromeisadisorderinwhichthehormonegastrinisproducedinexcess,directly
causingthestomachtoproduceexcessivehydrochloricacid.Thistumorcanariseintheduodenum
orpancreas.Inthispatient,atumorofthepancreaticisletcell(gastrinoma)iscausingthe
continuousexcretionofgastrin.Thirtypercentofpatientswithgastrinomaofthepancreaticisletcell
alsohavetumorsoftheparathyroidglandsandthepituitary.Thesecollectivetumorsareknownas
MENI.
Note:OthercausesofhypergastrinemiaincludeGcellhyperplasia,perniciousanemia,gastricoutlet
obstruction,renalfailure,andprotonpumpinhibitors.
Multipleendocrineneoplasia(MEN)syndromesareaboardsfavorite!Youshouldknowthe
tumorsassociatedwithallthreeMENsyndromesandtheconstellationofsymptomsthatresult
fromthem.BoardsquestionsonMENsyndromesareoftenstraightforwardbutwillmostlikely
requireyoutorecognizethediagnosis,whichcanbetrickyforsomestudents.Thebestwayto
getgoodatthisistodoUSMLEpracticequestions.
Step1Secret

Theconstellationofapituitaryadenoma,pancreaticneuroendocrinetumor,and
hyperparathyroidismshouldmakeonethinkofmultipleendocrineneoplasia(MEN)type
I.HyperparathyroidismisthemostcommonabnormalityinMENI,presentinmorethan
90%ofpatients.Itoccurs10to20yearsearlierthanthesporadicformof
hyperparathyroidismandismuchmoreaggressive.Ittypicallyinvolvesallfour
parathyroidglands,andsubtotalparathyroidectomyisthereforerarelycurative.MENIis
causedbymutationsinthemeningene,apresumptivetumorsuppressorgene.
MENIIAisassociatedwithmedullarythyroidcarcinoma,pheochromocytoma,and
primaryhyperparathyroidism.Cutaneouslichenamyloidosishasalsobeenrecently
added.

Medullarythyroidcarcinoma,whichcanbelifethreatening,hasafrequencyofgreater
than90%inMENIIApatients.Therefore,makingthediagnosisandinitiatingearly
geneticscreeningarecritical.MENIIAiscausedbymutationsintheRETproto
oncogene.
MENIIBisrareandisassociatedwithmedullarythyroidcarcinoma,
pheochromocytoma,mucosalneuromas,intestinalganglioneuromas,andoccasionallya
marfanoidhabitus.
AswithMENIIA,medullarythyroidcarcinomaisthemostcommoncomponent.
However,unlikeinMENIandMENIIA,primaryhyperparathyroidismisnotpresent.
MENIIBisalsocausedbymutationsintheRETprotooncogene.
ZollingerEllisonsyndromeiscausedbyatumorintheduodenumorpancreasthat
hypersecretesthehormonegastrin.Approximately30%ofpatientswithagastrinomaof
thepancreaticisletcellshaveMENI.
SummaryBox:MultipleEndocrineNeoplasia

Case814
A41yearoldwomanwithanunremarkablepastmedicalhistoryisnotedtohave
asymptomatichypercalcemiaonroutinescreening.Shetakesnomedications.Physical
examinationisunremarkable.

1 Whataresomecausesofhypercalcemia?
Inhealthyoutpatients,primaryhyperparathyroidismisthemostcommoncauseofhypercalcemia.
Inhospitalizedpatients,hypercalcemiaofmalignancyisthemostcommoncauseofhypercalcemia.
Togetherthesetwoareresponsibleforapproximately90%ofcasesofhypercalcemia.However,there
aremanyothercausesofhypercalcemiatobeconsidered.Primaryhyperparathyroidismassociated
withotherendocrinedisordersorwithafamilyhistoryofendocrinedisordersevokesthepossibility
oftheMENsyndromes.Frequentuseofcalciumcontainingantacidsevokesmilkalkalisyndrome.
Othermedicationssuchasthiazidediureticsandlithiumcanalsocausehypercalcemia.
EndocrinopathiesresultinginexcessivebonebreakdownsuchashyperthyroidismandCushing
syndromecancausehypercalcemia.Paraneoplasticsyndromesassociatedwithproductionof
parathyroidhormonerelatedpeptide(PTHrP)cancausehypercalcemia.Ageneticdisorderofthe
calciumsensingreceptorinfamilialhypocalciurichypercalcemia(FHH)isanothercauseof
hypercalcemia.Finally,excessivevitaminD(whichmayoccurwithtoxicingestion,lymphomas,or
granulomatousdiseasessuchassarcoidosis)cancausehypercalcemia.

case814continued:

Areviewofhermedicalrecordsshowsthatshehashadmildhypercalcemiasincetheageof21.
Uponquestioning,itisdiscoveredthathermotherandanauntbothhavemildhypercalcemia.

2 Howdoesthisinformationsuggestthelikelycauseofherhypercalcemia?
Giventhestrongfamilyhistory,shelikelyhasprimaryhyperparathyroidismassociatedwithaMEN
syndromeorshehasFHH.Itwouldbenicetoknowwhethershehaseverhadkidneystonesbefore.

Case814continued:
Shedeniesapersonalorfamilyhistoryofkidneystones.A24hoururinecollectionrevealslow
amountsofurinarycalciumexcretion.Polymerasechainreaction(PCR)testingshowsa
mutationofhercalciumsensingreceptor(CASR)gene.

3 Whatisthediagnosis?
ShehasFHH.Asitsnameimplies,FHHisassociatedwithhypercalcemiaresultinginpartfrom
deficientrenalcalciumexcretion.Itisinheritedinanautosomaldominantmanner.Incontrastwith
primaryhyperparathyroidism,whichisoftenassociatedwithmarkedlyelevatedPTH,hypercalciuria
causingcalciumoxalatekidneystones,andosteoporosis,FHHpatientstypicallyhaveamodest
elevationinPTHandrarelygetkidneystones.Nonetheless,FHHisoftenconfusedwithmildcasesof
primaryhyperparathyroidism.ThedangerinmisdiagnosingFHHforprimaryhyperparathyroidism
isthatthesepatientsmayunnecessarilyundergoaparathyroidectomy,withallitsattendantrisks.

4 Howdoesaninactivatedcalciumsensingreceptorgeneinfamilial
hypocalciurichypercalcemiacausehypercalcemia?
TheCASRgeneintheparathyroidglandsmediatesfeedbackinhibitionofPTHsecretioninresponse
torisingserumcalcium.AninactivatedCASRthereforedisinhibitsPTHsecretioninresponseto
hypercalcemia.Theresultisthatahigherthannormalserumcalciumlevelisrequiredtoinhibit
PTHsecretion,resultinginamildhypercalcemiaandmodestelevationinPTH.Thedegreeof
hypercalcemiadependsonhowseverelytheCASRgeneisinactivated.

5 Whatisthetreatmentforfamilialhypocalciurichypercalcemia?
FHHusuallydoesnotrequiretreatment,andmostaffectedpersonsareasymptomatic.However,for
thosewhoaresymptomatic,thiazidediureticscanbegiven.

Relatedquestions
6 Whatarethethreehormonesthatregulatecalciumlevelsinthebloodand
tissuesandtheiroriginofsecretion?
1.PTHissecretedfromthechiefcellsintheparathyroidglands.PTHisreleasedinresponseto
lowserumcalcium,anditspurposeistoraisetheserumcalciumlevel.

2.VitaminDisproducedfromthedietaswellasfromsynthesisthroughcholesterolwiththe
helpofultraviolet(UV)light.Itspurposeistoraisetheserumcalciumlevel.
3.Calcitonincomesfromparafollicularcellsinthethyroidgland.Calcitoninissecretedin
responsetohighserumcalciumandwilllowertheserumcalciumlevel.

7 HowdoesinactivevitaminDgetconvertedtotheactiveform?
SeeFigure819(f0100).

Familialhypocalciurichypercalcemiaiscausedbyautosomaldominantlossoffunction
mutationsinthecalciumsensingreceptor(CASR)gene.Familialhypocalciuric
hypercalcemia(FHH)patientsaretypicallyasymptomaticandpresentwith
hypercalcemia,hypocalciuria,andnormalormodestlyelevatedparathyroidhormone
(PTH)levels.
FHHcanbediagnosedwithgenetictestingformutationinCASR,checkingfamily
historyforhypercalcemia,andmeasuring24hoururinecalciumexcretion.
TreatmentforasymptomaticFHHpatientsissimplyobservation.Ifthepatientis
symptomaticfromthehypercalcemia,athiazidediureticisoftenenough.
SummaryBox:FamilialHypocalciuricHypercalcemia

Case815
Anewbornbabypresentswithambiguousexternalgenitalia.Thechildseemstohavean
enlargedclitorisratherthanapenis,butthereisascrotumlikestructurethatappearstobethe
resultoflabialfusion.Physicalexaminationalsorevealstachycardia,hypotension,irritability,
andhyperpigmentationseenmostreadilyintheareolaeandgenitalia.Laboratorytestsshow
hypoglycemia,hyponatremia,andhyperkalemia.Anultrasoundrevealsnormallydeveloped
ovaries.Upontesting,thechild'skaryotypeisfoundtobe46,XX.

Figure819
ActivationofvitaminD.
PTH,parathyroidhormone.(FromMeszarosJG,OlsonER,NaugleJE,etal:CrashCourse:Endocrineand
ReproductiveSystems.Philadelphia,Mosby,2006.)

1 Whatisthemostlikelydiagnosis?
Thebabygirlmostlikelyhascongenitaladrenalhyperplasia(CAH),whichismostcommonlydueto
21hydroxylasedeficiency.AllformsofCAHarecharacterizedbydeficientcortisolproduction,but
dependingonthespecificenzymaticdefect,levelsofotheradrenalsteroidscanbeincreasedor
decreased.Deficiencyof21hydroxylaseaccountsforabout90%ofCAHcases,while11
hydroxylasedeficiencyaccountsformostoftheremaining10%.

2 Howdoes21hydroxylasedeficiencycausevirilizationoffemales?
Theadrenalsteroidbiosyntheticpathwaysproducethreemajorhormones:mineralocorticoids(such
asaldosterone),glucocorticoids(suchascortisol),andsexhormones(androgensorestrogens).In
CAH,21hydroxylaseisusuallypartiallydeficientsuchthattheproductionofaldosteroneand
cortisoldecreaseowingtothemetabolicblock(Fig.820(f0105)).Thisleadstoanoverproductionof
steroidprecursors(suchas17hydroxyprogesterone).Theseprecursorsareshuntedintothepathway
ofsexhormonebiosynthesis,leadingtoexcessiveaccumulationofadrenalandrogens,whichcauses
inuteromasculinizationoftheexternalgenitaliaindevelopingfemales.

Figure820
Pathophysiologyofcongenitaladrenalhyperplasia
(FromBrownTA,BrownD:USMLEStep1Secrets.Philadelphia,Hanley&Belfus,2004.)

Malepseudohermaphroditismoccurswhenanindividualisageneticandgonadalmalewitha46,XY
karyotypeand(undescended)testesbuthasfemalegenitalia.Themostcommondisorderofmale
pseudohermaphroditismistesticularfeminizationsyndrome(alsoknownasandrogeninsensitivity)
causedbyadefectiveandrogenreceptor.Femalepseudohermaphroditesaregeneticandgonadal
femaleswitha46,XXkaryotypeandovariesbuthavemaleexternalgenitaliaandsecondarysex
characteristics.FemalepseudohermaphroditesareusuallytheresultofCAH.Unliketrue
hermaphrodites,pseudohermaphroditeshavegonadaltissueofonlyonesex.

3 Whydidthispatientexhibithyperkalemia,hyponatremia,tachycardia,
hypotension,andhypoglycemia?

Aldosteroneisnormallyresponsibleforthemaintenanceofplasmavolumeandpotassium
concentrationbypromotingthereabsorptionofsodiuminexchangeforpotassiumintheearlydistal
tubuleofthenephron.Aldosteronedeficiencyleadstohypovolemia,hyperkalemia,andinthecaseof
severedeficiency,hyponatremiafromsaltwasting.Recallthatsodiumlossusuallydoesnotcause
hyponatremia,butseverehypovolemiafromsodiumwastingwilleventuallystimulateADHrelease,
whichinturncauseshyponatremiainanattempttomaintainvolume.
Hypoglycemiaisduetodecreasedactivityofthecounterregulatory(i.e.,insulinopposing)hormone
cortisol.
Note:Cortisoldeficiencyisalsoamajorcontributor(ifnottheprimarycontributor)tohypotension
inthesettingofadrenalinsufficiency.Cortisolplaysamajorroleinthemaintenanceof
cardiovasculartoneandbloodpressurethat,althoughrelativelypoorlyunderstood,isclinically
veryimportant.Acutecortisoldeficiencycanleadtoseverehypotensionthatwillberefractoryto
fluidresuscitationintheabsenceofglucocorticoidreplacement.Thisisacommonphenomenonin
adultintensivecareunit(ICU)patients,whooftenfailtoadequatelyincreasetheircortisolsecretion
inresponsetothestressofcriticalillness(socalledrelativeadrenalinsufficiency).
ItistheseelectrolyteandhemodynamicconsequencesofcortisoldeficiencythatmakeCAHorother
formsofacuteadrenalinsufficiencyrapidlyfatalifleftuntreated.Oncetheconditionisdiagnosed,
treatmentisfairlystraightforwardwithreplacementofcortisol.Oftenasyntheticmineralocorticoid
(i.e.,fludrocortisone)isgivenaswell.(Becausecortisolhassomemineralocorticoidactivity,some
patientsdowellwithcortisolalone.)

4 Whyaretherebilateraladrenalhyperplasiaandhyperpigmentation?
ThelowcortisollevelreducesnegativefeedbackonpituitaryACTHproductionsuchthatACTHlevels
dramaticallyincreaseandstimulateglandularhyperplasia.RecallthatACTHalsostimulatesadrenal
androgensynthesis,whichfurthercontributestothevirilizationoffemaleswithCAH.
ThehyperpigmentationisanindirectconsequenceofincreasedACTHproduction.ACTHis
synthesizedfromaPOMCthatiscleavedintoACTH,endorphins(i.e.,endogenousopioids),and
MSH.Thus,anyincreaseinACTHreleaseisobligatorilyaccompaniedbyincreasesinMSHand
endorphinrelease.Asitsnamesuggests,ariseinMSHlevelsresultsinstimulationofmelanin
productionbymelanocytes,producingthehyperpigmentation.
Note:Exceptforthevirilization,manyofthediseasemanifestationsofCAHareidenticaltothose
seeninadultswithdisordersofprimaryadrenalinsufficiencysuchasautoimmuneadrenalitis(i.e.,
Addison'sdisease).Thesefeaturesincludehypotension,saltwastingwithhypovolemiaand
hyponatremia,hyperkalemia,hypoglycemia,andhyperpigmentation.

Relatedquestions

5 Whydoes11hydroxylasedeficiencyexhibitvirilizingmanifestationssimilar
tothosein21hydroxylasedeficiencybutnotthesamesaltwasting
manifestations?
When11hydroxylaseisdeficient,aldosteroneandcortisolsynthesisaredecreasedasin21
hydroxylasedeficiency.Again,steroidprecursorsaccumulateandareshuntedintoandrogen
production,andlowcortisolresultsinincreasedACTHlevels,whichfurtherstimulateandrogen
production.
However,incontrastwithclassicalCAH,11hydroxylasedeficiencyresultsintheaccumulationof
11deoxycorticosterone(11DOC),whichisaweakmineralocorticoid.Thelevelsof11DOCarehigh
enoughtocausefluidretentionandhypertensiondespitethedeficiencyofaldosterone.

6 Whyarepatientswith17hydroxylasedeficiencyphenotypicallyfemale?
Withregardtoexternalgenitalia,allfetusesarefemalebydefaultitisonlytheproductionof
androgensthatmasculinizestheexternalgenitaliainnormalmales.However,withlowlevelsof17
hydroxylase,insufficientlevelsoftheseandrogensareproducedinfetusesthataregeneticallymale,
resultinginmalepseudohermaphrodism(geneticandgonadalmaleswith46,XYkaryotypesand
undescendedtestes,butfemaleexternalgenitalia).
Note:Becauseestrogensareinvariablysynthesizedfromandrogens(viaaromatizationof
androgens)thereisalsolowestrogenproductioninpatientswith17hydroxylasedeficiency.Thus,
femaleswiththisrareformofCAHwillusuallypresentforinvestigationoffailuretoundergo
pubertaldevelopmentormenarche.

7 Whyarepatientswith17hydroxylasedeficiencyoftenhypertensive?
Theblockinthepathwaytomakeandrogensorcortisolincreasesfluxofsubstratethroughthe
aldosteronesynthesispathway.Theresultingincreasedaldosteronelevelscauseincreasedsaltand
waterretention,resultinginhypervolemiaandhypertension.

8 Quickreview:CoverthecolumnsontherightsideofTable815(t0080)and
listthesignsandsymptomsforthemajorcongenitalmetabolicdisorders
withineachmetabolicpathwayandtheirclinicalmanifestations
Congenitaladrenalhyperplasia(CAH)isduetodefectsinglucocorticoid(i.e.,cortisol)
synthesis,withresultinghypoadrenalismand,dependingonthespecificenzyme
involved,eitherincreasedordecreasedlevelsofmineralocorticoidsorsexsteroids.
The21hydroxylasedeficiencyaccountsfor90%ofCAH.Itresultsindeficiencyof
cortisolandaldosteronebutexcessadrenalandrogens,resultinginambiguousexternal
genitaliainfemalessaltwastingwithhypotension,tachycardia,hyponatremia,and
hyperkalemiahypoglycemiaandhyperpigmentation(duetoincreased
adrenocorticotropichormone[ACTH]andmelanocytestimulatinghormone[MSH]

synthesis).
OtherformsofCAH,suchas11hydroxylasedeficiency,canleadtovirilizationand
fluidretentionandhypertensionduetoexcessmineralocorticoidproduction.Others,
suchastherare17hydroxylasedeficiency,canleadtoandrogenandestrogen
deficiency.
SummaryBox:CongenitalAdrenalHyperplasia
Table815
MajorCongenitalMetabolicDisorders

MetabolicPathway

GeneticDiseases

PrimaryCauseofClinical

MajorSigns/Symptoms

Symptoms
Glycogenolysis

Glucose6

Hypoglycemia

phosphatase
deficiency(von

Hypoglycemicepisodes,
massivehepatomegaly

Gierkedisease)
Muscleglycogen

Inabilityofmuscletoutilize

Musclepain,exercise

phosphorylase

glucosestoredasglycogen

intolerance

deficiency
(McArdledisease)
Hexose

Glucose6

RBCsusceptibilitytooxidative Hemolyticanemia

monophosphate

phosphate

stress(e.g.,sulfaor

shunt(i.e.,pentose dehydrogenase
phosphate

antimalarialdrugs,infection)

(G6PD)deficiency

pathway)
Fattyacidoxidation Mediumchain

Hypoglycemia

Hypoketotic

fattyacid

hypoglycemia,

decarboxylase

hyperammonemia

(MCAD)
deficiency
Ureacycle

Variousenzymes

Hyperammonemia

Encephalopathy

deficiencies
Aminoacid

Phenylketonuria

Toxicaccumulationof

Mentalretardation,light

metabolism

(PKU)

phenylalanineand
phenylketonederivativeslack

pigmentation,eczema,
mousyodor

oftyrosine

Maplesyrupurine Toxicaccumulationof
disease
branchedchainaminoacids

Hemesynthesis

Poorfeeding,
psychomotorretardation,

(especiallyleucine)andtheir

maplesyrupodorto

ketoacidderivatives

urine

Acuteintermittent

Accumulationofporphyrin

Neurovisceralsymptoms

porphyria(AIP)

intermediatestoxictoneurons

(neuropathy,episodic

andotheracute

abdominalpain)

porphyrias
Porphyriacutanea Accumulationofphotoreactive Photosensitivechronic
tardaandother

porphyrinswithinskinand/or

cutaneous

RBCs

blistering

porphyrias
Hemolytic

Hemolyticanemia

porphyrias
Adrenal

Congenital

Deficiencyofcortisoland

Allforms:adrenal

corticosteroid

adrenal

deficiencyand/orexcessof

insufficiencywith

synthesis

hyperplasia

mineralocorticoidsandsex

glandularhyperplasia

steroids

andhyperpigmentation

21Hydroxylase

Lowcortisoland

Virilizationinfemales

deficiency

mineralocorticoidsexcess
androgens

saltwastingwith
hypotension,
hyponatremia,
hyperkalemia

11Hydroxylase

Lowcortisol,excess

Virilizationinfemales

deficiency

mineralocorticoids(11DOC)

hypertensionandfluid

andandrogens

overload

11DOC,11deoxycorticosteroneRBCs,redbloodcells.

Copyright2015Elsevier,Inc.Allrightsreserved.

BOOKCHAPTER

FluidandElectrolytes
ThomasA.BrownMDandSonaliJ.Shah
USMLEStep1Secrets,Chapter4,97116

ManyoftheimportantconceptsthatweretoucheduponinChapter3arefurtherexploredin
Chapters4and5,onfluidandelectrolytesandacidbasebalance,respectively.Concepts
relatingtofluidandelectrolytesmakeupalargeportionoftherenalphysiologyand
pharmacologymaterialtestedonStep1.Youwillsoonseethatthisisnotasectionthatrequires
memorizinglotsofsmalldetails.Instead,itdemandsathoroughunderstandingofthe
mechanismsofactionofvarioushormonesanddrugsinvolvedinregulatingfluidand
electrolytebalance.Knowwhichsegmentsofthenephronareaffectedbytheseindividual
hormonesanddrugs.Ifapplicable,itisagoodideatocategorizehoweachhormoneordrug
affectsglomerularfiltrationrate(GFR),renalplasmaflow(RPF),filtrationfraction,etc.For
thoseofyouwhodonothaveastrongbackgroundinrenalphysiology,youmayneedtoread
throughthediscussioninthischapterafewtimestofullygraspallofthematerial.
Insider'sGuidetoFluidandElectrolytesforTheUSMLEStep1

Basicconceptsrenalfiltrationandtransportprocesses
1 Whatforcesgoverntheglomerularfiltrationrateatthelevelofthe
glomerulus?
Theseforcesarethesameforcesthataffectfluidmovementinthesystemiccapillaries.Forcesthat
drivefluidacrosstheglomerularmembraneincludethehydrostaticpressureintheglomerular
capillariesandtheoncoticpressureinBowman'sspace(Fig.41(f0010)).Becausethereisusually
verylittleproteininBowman'sspace,thecontributionfromthefiltrate'soncoticpressureistypically
negligible.Forcesthatopposefluidmovementacrosstheglomerularmembranearethehydrostatic
pressureinBowman'sspaceandtheplasmaoncoticpressure.

Figure41
Summaryofforcescausingfiltrationbytheglomerularcapillaries.Thevaluesshownareestimatesforhealthy
humans.
(FromGuytonAC,HallJE:TextbookofMedicalPhysiology,11thed.Philadelphia,WBSaunders,2007.)

Glomerularhydrostaticpressureand,inturn,glomerularfiltrationrate(GFR),isalteredby
constrictionordilationoftheafferentandefferentarterioles.Becausetheglomerulusislocated
betweentheafferentandefferentarterioles,changesinthecaliberofthearteriolestendtohave
oppositeeffectsontheglomerulus.Eitherdilationoftheafferentarterioleorconstrictionofthe
efferentarteriolewillincreaseglomerularpressureandfiltration.Likewise,constrictionofthe
afferentarterioleordilationoftheefferentarteriolewilldecreaseglomerularpressureandGFR.
Note:Mostcirculatingvasoconstrictingandvasodilatingagentsactontheafferentarteriole.An
importantexception,however,isangiotensinII,whichactspreferentiallytovasoconstrictthe
efferentarteriole.Thus,angiotensinIIworkstopreserveGFRinasettingofdecreasedrenal
perfusion.Angiotensinconvertingenzyme(ACE)inhibitorsdecreaseGFRbyinhibitingthe
formationofangiotensinIIontheefferentarteriole(seequestion2).
Inadditiontotheoncoticandhydrostaticpressures,thesurfaceareaandintegrityoftheglomerular
membranesarealsoimportantdeterminantsofGFR.Mathematically,thesefactorsarerepresented
throughafiltrationconstant.Thesefactorsaremostrelevantindiseasestatesinwhichtheglomeruli
aredamaged.TheformulaforcalculatingGFRisgiveninChapter3.

2 Howdoangiotensinconvertingenzymeinhibitorsandangiotensinreceptor
blockersaffectglomerularfiltrationrate?
ACEinhibitorsandangiotensinreceptorblockers(ARBs)tendtodecreaseGFRacutely.Blockingthe
actionofangiotensinIIresultsinvasodilationoftheefferentarteriole,whichdecreases
intraglomerularpressureand,inturn,decreasesfiltration.
AlthoughACEinhibitorsandARBsusuallyproduceadecreaseinGFR(manifestingasanincreasein
plasmacreatinine)intheshortterm,theyhaveanimportantbeneficialeffectonpreservingrenal
functioninindividualswithdiabetesandotherchronickidneydiseases.Thisbenefitlikelyresults

fromthefactthatadecreaseinglomerularpressure,despiteacutelydecreasingGFR,mayoverthe
longtermdecreasethewearontheglomeruli.Becauseofthisbenefit,acertaindegreeofcreatinine
elevationistoleratedwhenstartinganACEinhibitororARB.

3 Whatismeantbythetermfiltrationfractionandhowwillincreasingthe
glomerularcapillaryoncoticpressure(withoutchanginganythingelse)affect
thefiltrationfraction?
Thefiltrationfractionisthepercentofplasmapassingthroughtheglomerularcapillariesthatis
actuallyfilteredbytheglomerulus.Itcanbecalculatedasshownhere:
Filtration fraction (FF) =

Glomerular filtration rate


Renal plasma flow

Normally,thisfractionisabout20%.Becausetheglomerularcapillaryoncoticpressureopposes
filtration,increasingitwilldecreasethenetfiltrationpressureanddecreasethefiltrationfraction.

4 Whatarethethreelayersoftheglomerularfilterandhowdotheycontribute
totheprocessofrenalfiltrationattheglomerulus?
Thethreecomponentsoftheglomerularfilterincludetheendothelialcellsoftheglomerular
capillaries,theunderlyingbasementmembrane,andtheglomerularepithelialcells.These
componentsallcontributetorenalfiltrationindistinctways.
Theendotheliumoftheglomerularcapillariesisfenestrated.Alongwiththehighhydrostatic
pressurepresentintheglomerularcapillaries,thefenestrationofthesecapillariesallowsforthe
filtrationoflargevolumesofplasmaacrossthecapillarybed.
Theunderlyingbasementmembraneisnegativelycharged,whichhelpspreventfiltration(and
subsequentlossintheurine)oflargenegativelychargedplasmaproteins.Importantly,these
negativelychargedproteinsincludealbumin,whichiswhylossofalbuminandhypoalbuminemia
canoccurinvarioustypesofglomerulardisease(particularlyinnephroticsyndromes).
Theglomerularepithelialcellsorpodocytescomposethefinallayeroftheglomerularfilter.These
specializedcellshavecytoplasmicextensionscalledfootprocesses,withinterveningslitpores,that
togetherenveloptheglomerularcapillariesandformafinalbarrierforfilterablemoleculesto
traversepriortoenteringthecapsularspaceoftheglomerulus.
Note:Theglomerulusalsocontainsmacrophagelikemesangialcellsandmesangialmatrix
interspersedbetweentheselayers(Fig.42(f0015)).Thefunctionofthemesangiumisnotverywell
understood(thoughitmayservebothastructuralroleandahousekeepingrole).Regardless,the
mesangiumcanbeanimportantsiteofglomerulardisease.

Figure42
A,Lowpowerelectronmicrographofrenalglomerulus.CL,capillarylumenEND,endotheliumEP,visceral
epithelialcellswithfootprocessesMES,mesangium.B,Schematicrepresentationofaglomerularlobe.
(CourtesyofDr.VickiKelley,BrighamandWomen'sHospital,Boston,MA.)

5 Whatisthesignificanceofthecreatinineclearanceandhowisitmeasured?
Theclearanceofanysubstanceisdefinedasthevolumeofplasmathatisclearedofthatsubstance
perunitoftime.Forexample,acreatinineclearancerateof125mL/minimpliesthat,everyminute,
creatinineisbeingcompletelyremovedandexcreted(bythekidneys)from125mLofplasma.As
shownbythefollowingequation,theclearanceofasubstancecanbecalculatedbydividingtherate
ofurinaryexcretionofasubstancebythesubstance'splasmaconcentration.Therateofurinary
excretionofasubstancecanbedeterminedfromitsconcentrationinurineandtheurineflowrate.
Urinary flow rate(mL/min)Urinary creatinine concentration

Plasma creatinine concentration

VUcr
Pcr

Youcaneasilyderivethisformulabyrecallingthatallcreatininethatappearsintheurineisaresult
oftheremovalofcreatininefromplasma:
Rate of creatinine removal from plasma = Rate of creatinine excretion in urine
Pcr C = V Ucr
C =

VUcr
Pcr

Becausecreatinineis(forthemostpart)neitherreabsorbednorsecreted,itsclearanceratevery
closelyapproximatestheactualGFR,anditisthereforeanimportantindicatorofrenalfunction.
Clinically,asingleplasmacreatininelevelisfrequentlyused,alongwithapatient'sweight,age,and
gender(andinsomeequations,raceandserumalbuminasameasureofnutritionstatus),to
estimatecreatinineclearanceandGFR.Theseadditionalfactorsareincludedtoestimatetherateof
creatinineproduction,whichdependsonone'smusclemass(creatinineisabyproductofthemuscle
energystoragemoleculecreatine).Notethattheseequations(suchastheCockroftGaultequation
andthemoresophisticatedmodificationofdietinrenaldisease[MDRD]equation)workwellonly
whenthepatient'srenalfunctionisatasteadystate.Theyworkpoorlywhenitisrapidlychanging,as
inacuterenalfailure.

Thedeterminantsofglomerularfiltrationrate(GFR)includethehydrostaticandoncotic
pressureswithintheglomerularcapillaries,thehydrostaticandoncoticpressuresof
Bowman'sspace,andthecollectiveglomerularsurfaceareaandintegrity(i.e.,filtration
constant).
Thehydrostaticpressurewithintheglomerularcapillariesisregulatedby
vasoconstrictionandvasodilationoftheefferentandafferentarterioles.
Mostcirculatingvasoactivesubstancesacttoconstrictordilatetheafferentarteriole.
AngiotensinIIisuniqueinthatitactsprimarilytoconstricttheefferentarteriole.
Angiotensinconvertingenzyme(ACE)inhibitorsandangiotensinreceptorblockers
(ARBs)mayacutelydecreaseGFR,buttheireffectonloweringglomerularpressurehelps
preserverenalfunctionoverthelongterminpatientswithchronickidneydisease.
Thethreecomponentsoftheglomerularfilterare(1)thecapillaryendothelialcells,(2)
thebasementmembrane,and(3)theepithelialcells,orpodocytes,andtheintervening
mesangium.
CreatinineclearanceisanimportantindicatorofGFR.Clinically,itisusuallyestimated
fromasinglemeasurementofserumcreatinineconcentration.However,recallthat
numerousvariables(e.g.age,gender,musclemass)canaffectthecreatinine
concentration.

EstimatesofcreatinineclearanceareinaccurateinthesettingofarapidlychangingGFR
(i.e.,acuterenalfailure).
SummaryBox:GeneralConceptsinRenalFiltrationandTransportProcesses

Basicconceptsrenalcontrolofacidbasebalance
1 Whyisnetrenalacidexcretionnecessarytomaintainacidbasehomeostasis?
Dailymetabolismgeneratesalargequantityofnonvolatileacids(e.g.,lactate,sulfate,phosphate)
thatcannotbeexcretedbythelungs.Thesenonvolatileacidsmustbeexcretedbythekidneysto
preventthedevelopmentofametabolicacidosis.
Nonvolatileacidsarederivedprimarilyfromthemetabolismofdietaryproteinsinmeatandother
foods.Incontrast,metabolicbreakdownofcarbohydratesandfatsyieldslargelycarbondioxide,
whichiseasilyexcretedbythelungs.Vegetarians,forexample,typicallygeneratelessdietaryacidfor
theirkidneystoexcreteand,asaresult,onaveragetendtohavelessacidicurine.

2 Whatmechanismsdoesthekidneyusetomaintainacidbasebalancedespite
thisacidload?
Ingeneral,thekidneyactstopreventacidosisfromdevelopingbysecretingacidintotheurinewhile
reabsorbingbase.
Morespecifically,thekidneys(1)efficientlyreabsorbfilteredbicarbonateintheproximaltubule
(throughamechanismthatiscoupledwithhydrogenionsecretionintothetubularfluid),(2)
synthesizedenovobothbicarbonate(HCO )toberetainedandtheacidammonium(NH )to
besecreted,(3)secretetitratablebufferssuchasammoniaandphosphate(whichbindhydrogenions

andincreasetheacidexcretorycapacityoftheurinewithoutcausingaprecipitousdropinurinary
pH),and(4)activelypumpacid(intheformofhydrogenions)intothetubularfluidatthedistal
tubule.

3 Howdothekidneysreabsorbfilteredbicarbonate?
Hydrogenionsthataresecretedintothelumenoftheproximaltubule(primarilyvia
countertransportwithNa+)reactwithbicarbonateinthefiltratetoformcarbonicacid,which
rapidlydissociatesintocarbondioxideandwater,bothofwhichcanthendiffusebackintothecell(
Fig.43(f0020)).Inthecell,thereversereactiontakesplace.Thus,carbondioxidereactswithwater

togeneratebicarbonateandaproton(hydrogenion).Thebicarbonateisultimatelyreturnedtothe
venouscirculation,whereasthehydrogenionissecretedbackintothetubularlumen.

Figure43
Proximaltubule.
(FromGoldmanL,AusielloD:CecilTextbookofMedicine,22nded.Philadelphia,WBSaunders,2004.)

4 Whateffectdoesthediureticacetazolamidehaveontheacidbasebalanceof
thebody?Inwhatclinicalconditionsmightitbeused?
Acetazolamideisaninhibitorofcarbonicanhydrase,anenzymepresentintheproximaltubuleofthe
nephron(theenzymeisactuallyanchoredtotheluminalsurfaceoftheplasmamembraneoftubular
epithelialcells).Thisenzymenormallycatalyzestherapiddissociationofcarbonicacidinthetubular
fluidintoCO2andH2O,anessentialstepinthereabsorptionofbicarbonate(asdescribed
previously).Theneteffect,therefore,ofinhibitingcarbonicanhydraseisincreasedurinaryexcretion
ofbicarbonate.Becausethisnegativelychargedbicarbonatemustbeexcretedwithsome
accompanyingsodium,acetazolamideisalsoadiuretic(albeitaweakone).
Thispharmacotherapeuticmechanismofactionessentiallymimicsthepathophysiologyseenin
proximal(typeII)renaltubularacidoses,inwhichbicarbonatereabsorptionbythetubular
epitheliumisimpaired.Theresultofdecreasedbicarbonatereabsorptionisamildmetabolic
acidosis.Thisabilityofacetazolamidetocreateametabolicacidosiscanbeutilizedtherapeuticallyin
thetreatmentorpreventionofrespiratoryalkalosesthatdevelopatelevatedaltitudes(i.e.,mountain
sickness).Itisalsousefultotreatconditionssuchascystinuria(alkalinizesurinetopreventstone
formation)andglaucoma(decreasesaqueoushumorsecretion).

5 Howarebicarbonateandammoniumgenerateddenovobythekidney?
Thedeaminationofglutamineintheproximaltubulegeneratestwoammonium(NH )molecules
andtwobicarbonate(HCO )molecules.Theammoniummolecules(whichessentiallyconsistof
+

acidicprotonsbeingcarriedbyammonia)aresecretedintothetubularlumen,whereasthebasic
bicarbonatemoleculesarereabsorbedintothesystemiccirculation(Fig.44(f0025)).

Figure44
Productionandsecretionofammoniumion(NH4 )byproximaltubularcells.Glutamineismetabolizedinthe
+

cellyieldingNH4 + andbicarbonate.TheNH4 + isactivelysecretedintothelumenbymeansofasodium


NH4

HCO3

pump.Foreachglutaminemoleculemetabolized,twoNH4 areproducedandsecretedandtwo
+

arereturnedtotheblood.

(FromGuytonAC,HallJE:TextbookofMedicalPhysiology,11thed.Philadelphia,WBSaunders,2007.)

GlutaminedeaminationisstimulatedbyincreasedlevelsofH+ionsorCO2withinthecellsofthe
proximaltubule.Thus,thismechanismappropriatelyincreasestherenalsynthesisofbicarbonate(to
beretained)andammonium(tobesecreted)inacidoticconditions.
Metabolismgeneratesalargequantityofnonvolatileacidsthatmustbeexcretedbythe
kidneys.Thisisaccomplishedbyseveralmechanisms:
Thereabsorptionoffilteredbicarbonateintheproximaltubuleviaamechanism
requiringtheactivityofcarbonicanhydrase
Thedenovosynthesisofbicarbonate(toberetained)andammonium(tobesecreted)
Thesecretionoftitratablebuffers(e.g.,ammonia,phosphatetoincreasetheacid
carryingcapacityofurine)
Thesecretionofacidviaaprotonpumpinthedistaltubule
Formoreinformationontheregulationofacidbasebalance,seeChapter5.
SummaryBox:RenalControlofAcidBaseBalance

Basicconceptsrenalcontrolofextracellularfluidbalance
1 Whataretheextracellularfluidcompartmentsofthebodyandhowdotheir
relativesizescomparetotheintracellularfluidcompartment?
Theextracellularfluid(ECF)consistsoftheinterstitialfluidandplasma.

Overall,totalbodywaterinahealthyyoungmanisroughly60%ofbodyweight(whereasinafemale
subjectitisabout50%).Foranaverage70kgman,totalbodywateris42L(recallthat1Lofwater
weighs1kg).Abouttwothirdsofthetotalbodywaterisfoundwithincellstheotheronethirdmakes
uptheECFvolume.Thus,a70kgmanhasabout14LofECF:(700.6)1/3.Plasmavolume
accountsforaboutonethirdtoonefourthoftheECF(about4Lorsoina70kgindividual),whereas
therestisinterstitialfluid.

2 Howdothekidneysregulateextracellularfluidvolume?
ThekidneysregulateECFvolumebyadjustingtherateofexcretionofsodium.Incontrast,the
kidneysregulatebodyfluidosmolarityandsodiumconcentrationbyalteringtheexcretionoffree
water(waterwithoutsodium).Thisisoneofthemostimportantconceptsinrenalphysiology:Inthe
normalstate,volumeisregulatedthroughsodiumbalance,whereasosmolarityandsodium
concentrationareregulatedthroughwaterbalance.
Tobemoreprecise,itiseffectivecirculatingvolume(orECV)thatisregulatedbythebody,notthe
ECFvolume.ThisisbecausethebodyhasnowaytodirectlyfollowECFvolumelevels.Instead,
variouspressureandvolumedetectorslocatedthroughoutthecirculatorysystem(intheatria,the
aorticarch,thecarotidsinus,andtheafferentarteriolesofthekidney)monitortheECVand,through
variousmechanisms,stimulateorinhibitNa+excretion.ECVisnotameasurablevolume.Itrefersto
thevolumeofarterialbloodeffectivelyperfusingtissue.ECVisgenerallyproportionaltoECF,but
notableexceptionsoccurduringcongestiveheartfailure(CHF),cirrhosis,andnephroticsyndrome.
Thereninangiotensinaldosteronesystem(RAAS)ispossiblythemostimportantofthese
mechanisms.

3 Whatisthenormalroleofthereninangiotensinaldosteronesystem?
TheRAASactstomaintainanappropriateplasmavolumeandbloodpressureinordertoensure
adequateorganperfusion.
RAASisactivatedinresponsetodecreasedECV.Specifically,reducedrenalbloodflowissensedbya
groupofspecializedsmoothmusclecellslocatedinthewalloftheafferentarterioles(partofthe
juxtaglomerularapparatus).Thissensingmechanismallowsfordecreasedarteriolarperfusionto
stimulatereninsecretion.
ReninenzymaticallycleavestheserumprecursorproteinangiotensinogenintoangiotensinI,which,
inturn,iscleavedinthelungsandelsewherebytheendothelialenzymeACEintothephysiologically
activepeptideangiotensinII.AngiotensinIIisapotentvasoconstrictorthatactstoincreaseblood
pressureinfact,itisthemostpotentphysiologicvasoconstrictorknown.AngiotensinIIalsohasa
directeffecttopromoteNa+reabsorptionintheproximaltubule.AthirdactionofangiotensinIIis
stimulationofaldosteronereleasefromtheadrenalcortex.Aldosteronestimulatesthereabsorption
ofNa+(coupledtothesecretionofK+orH+)inthedistaltubuleofthenephron.ThisretainedNa+
(morespecificallythefluidvolumethataccompaniesit)helpsrestoreECVtowardnormal.

4 Whatistheroleofthesympatheticnervoussysteminmaintainingeffective
circulatingvolume?
Inadditiontohelpingmaintainbloodpressureviasystemicvasoconstriction,thesympathetic
nervoussystem(whichisalsoactivatedbylowECV)stimulatesNa+retentioninseveralways.First,
sympatheticallymediatedafferentarteriolarvasoconstrictiondecreasesGFR,whichindirectly
promotessodiumretentionthroughRAASactivation.Thesympatheticfiberstotheafferentarteriole
alsodirectlystimulatereninrelease.Finally,thesympatheticnervoussystem(likeangiotensinII)
promotesNa+reabsorptionintheproximaltubule.

5 Howdoestheantidiuretichormoneregulateextracellularfluidvolume?
Undernormalconditions,antidiuretichormone(ADH)doesnotworktoregulateECFvolume.
Instead,ADHnormallyfunctionstoregulatethereabsorptionoffreewaterinthecollectingductin
responsetochangesinbodyfluidosmolarity.
However,whenECVisseverelycompromised(decreasedby510%ofnormal),thesecretionofADH
bytheposteriorpituitaryisstimulated.Thus,withsignificanthypovolemia,thefunctionofADH
changestohelppreservevolumeratherthanosmolarity.
ThisabilityofADHtosacrificeosmolaritytohelpmaintainECVisanexceptiontotheprecedingrule
statingthatwaterbalanceisregulatedtomaintainosmolarityandsodiumbalanceisregulatedto
maintainvolume.Whenvolumeislowenough,thebodyabandonsthisusualdivisionoflaborand
retainssodiumandwaterregardlessofosmolarity.Thischangeisillustratedbydiseasestatessuch
asCHF,nephroticsyndrome,andcirrhosis.Becausethesethreediseasesarecharacterizedby
decreasedECV,hyponatremiacommonlyoccursinallofthemasaresultofchronicallyhighADH
levels.

Case41
A78yearoldwomanisevaluatedforrefractoryhypertension.Hercurrentantihypertensive
regimenincludesathiazidediuretic,anACEinhibitor,abetablocker,andan1receptor
antagonist.Examinationissignificantforabloodpressureof184/105mmHgandan
abdominalbruit.Arenalangiogramrevealsa95%occlusionoftheleftrenalartery.Renal
angioplastyisperformedtorelievetheocclusion,andherbloodpressuresubsequently
normalizes.

1 Whatdiseasedidthispatienthaveandwhatisitsunderlyingcause?
Thiselderlyladyhadrenovascularhypertension(orrenalarterystenosis),whichismostcommonly
duetoatherosclerosisoftherenalarteries.Fibromusculardysplasiaoftherenalarteries,aless
commondiseasethatprimarilyaffectsmiddleagedwomen,alsoproducesrenovascularhypertension
byoccludingthelumenoftherenalarteries.

2 Howdoesthereninangiotensinaldosteronesystemcontributeto
renovascularhypertension?
Ifonekidneyissignificantlyhypoperfusedbecauseofrenalarterynarrowing,itwillrelease
abnormallyhighlevelsofrenin.ThiswillincreasesystemicangiotensinIIandaldosteronelevels,
whichwillcausewidespreadvasoconstrictionandsignificantsaltandwaterretentionbyboththe
normalandtheabnormalkidneys.Bothoftheseactionsresultinincreasedbloodpressure.In
essence,renovascularhypertensionresultswhenthekidneysaremisinformedaboutthepressure
andvolumestatusofthebody.

3 Howdothekidneysregulatebloodpressureindependentlyoftherenin
angiotensinaldosteronesystem?
Alongwithrenin,angiotensinII,aldosterone,ADH,andthesympatheticnervoussystemas
describedearlier,additionalmechanismsforregulatingECVandbloodpressureexist.
Athigherarterialpressures,thekidneysarebetterperfused,whichdirectlyresultsinincreasedGFR.
IncreasedGFRaloneincreasesthevolumeofurinethatisproduced,therebyreducingtheECF
volumeandbloodpressure.Thisphenomenonisreferredtobysomeasapressurenatriuresis.At
lowerarterialpressuresthereducedperfusionreducesGFRandincreasestubularreabsorptionof
saltandwater(glomerulotubularbalance),helpingtoexpandECFvolumeandrestoretheblood
pressure.
Atrialnatriureticpeptide(ANP)isyetanotherhormonalagentthathelpsregulateECVandblood
pressure.HighECVstimulatesthecardiacatriatoreleaseANP,which,asitsnamesuggests,
promotesnatriuresis(i.e.,sodiumexcretion)bythekidneysandtherebydecreasesECV.Inthe
settingoflowECV,ANPreleaseisinhibited.

4 IfangiotensinIIpromotesvasoconstriction,whydoesn'ttheangiotensinII
releasedduringhypovolemicstatesreduceglomerularfiltrationrate?
ThepreferentialactionofangiotensinIIontheefferent(ratherthantheafferent)arterioleelegantly
allowsittomaintainGFRdespitecausingwidespreadvasoconstrictionandreducedrenalperfusion.
Althoughvasoconstrictionofefferentarteriolereducesrenalbloodflow,italsoincreasesthe
hydrostaticpressureoftheglomerulus,whichincreasesthenetfiltrationpressureandmaintainsthe
GFR.
Ifithasnotbecomeapparentyet,itiscriticaltoknowthatangiotensinIImaintainsglomerular
filtrationrate(GFR)bypreferentiallyconstrictingtheefferentarteriole.Recallthatangiotensin
IIwillalsoincreasefiltrationfraction(FF),becauseGFRisincreasedandrenalplasmaflow
(RPF)isreduced:
FF = GFR/RPF

ItisunlikelythatGFRisincreasedsimplybecauseangiotensinIIisproducedonlyinasetting
ofrelativerenalhypoperfusion,butit'sfairtosaythatangiotensinIIactstopreserveGFR.

Step1Secret

5 Whyshouldangiotensinconvertingenzymeinhibitors(orangiotensin
receptorblockers)beavoidedinpatientswithbilateralrenalarterystenosis?
ThenormalroleofangiotensinIIinmaintainingGFRinthefaceofhypovolemiaisevenmore
pronouncedinthesettingofbilateralrenalarterystenosis(RAS).
KidneyswithbilateralRASaredependentonangiotensinIImediatedconstrictionoftheefferent
arterioletomaintainglomerularfiltrationpressureandGFR.Thistonicangiotensinmediated
constrictionoftheefferentarterioleisdrivenbyhighplasmalevelsofangiotensinII.AnACE
inhibitor(oranARB)inthissettingwillcausetheefferentarteriolarvasoconstrictiontoabruptly
cease,andGFRinbothkidneysmaydropprecipitously,possiblyresultinginacuterenalfailure.

Totalbodywaterisabout60%ofbodyweightinmenand50%inwomen.Abouttwo
thirdsofthisisintracellularandonethirdisextracellular.Plasmaaccountsforaboutone
thirdtoonefourthofextracellularfluid(ECF)volume.
Undernormalconditions,ECFvolumeor,moreprecisely,effectivecirculationvolume
(ECV)isregulatedbyadjustingtherateofsodiumexcretion.Likewise,osmolarityis
regulatedbyadjustingtherateoffreewaterexcretion.Thebodywillalwaysattemptto
maintainsodiumbalancebeforewaterbalance.
ECVandbloodpressureareregulatedbythereninangiotensinaldosteronesystem,the
sympatheticnervoussystem,atrialnatriureticpeptide,andintrinsicrenalmechanisms.
WhenECVisdecreasedby5%to10%ofnormal,antidiuretichormone(ADH)releaseis
stimulated,helpingmaintainvolumewhilepotentiallysacrificingosmolarity.
Renalarterystenosis(RAS)leadstohypertension,inpart,viachronicactivationofthe
reninangiotensinaldosteronesystem.RASismostcommonlycausedbyatherosclerosis,
but,inyoungerwomen,maybecausedbyfibromusculardysplasia.
Angiotensinconvertingenzyme(ACE)inhibitorsandangiotensinreceptorblockers
(ARBs)shouldbeavoidedinpatientswithbilateralRASduetodependenceonhighlevels
ofangiotensinIIformaintenanceofglomerularfiltrationrate(GFR).
SummaryBox:RenalControlofExtracellularFluidBalance

RenalControlofExtracellularFluidOsmolarity
Case42

A48yearoldmanwasadmittedtothehospitalforanelectivecholecystectomy,anda
presurgicalworkuprevealsaserumsodiumlevelof125mEq/L(normal135145mEq/L).He
complainsofsomefatigue,anorexia,andverymildconfusion.Vitalsignsareallwithinnormal
limits,asishisphysicalexamination.Urinalysisrevealsaurineosmolarityof620mOsm/kg.

1 Whatisthemostlikelydiagnosisandwhatisitscause?
Unexplainedhyponatremiainthesettingofincreasedurineosmolaritystronglysuggeststhe
syndromeofinappropriatesecretionofantidiuretichormone(SIADH).
Inthisdisorder,excessreleaseoftheposteriorpituitaryhormoneADHcausesexcessfreewater
retentioninthecollectingductofthenephron.Thisadditionalwaterdilutestheplasma(resultingin
decreasedosmolarityandhyponatremia)andinappropriatelyconcentratestheurine(asopposedto
theappropriateconcentrationofurineinasettingofhypovolemia).

Case42continued:
Furtherquestioningrevealsa60packperyearhistoryofsmoking.Hedeniesanyrecent
vomitingordiarrhea,hasanormalalbuminlevel,andshowsnosignsofheartfailure.He
deniestakingdiureticsorothermedications.Achestxrayfilmrevealsa3cm,spiculated
pulmonarynodulenearthehilumoftherightlung.

2 Whatsyndromesareassociatedwithsyndromeofinappropriatesecretionof
antidiuretichormone?
SIADHisassociatedwithamyriadofdiseasesanddisorders.ForStep1,ectopicsecretionofADHby
smallcellcarcinomasofthelungistheclassiccauseofSIADH.Inadditiontothisparaneoplastic
syndrome,SIADHcanbeseeninavarietyofnonmalignantdiseasesofthelung,including
tuberculosis,pneumonia,pneumothorax,chronicobstructivepulmonarydisease(COPD),and
asthma.
SIADHcanalsooccurfromlesionsortumorsofthepituitaryorhypothalamusandinthesettingof
othertypesofintracranialpathologysuchasheadtrauma,strokeandintracranialbleeds,and
infection.Ithasalsobeenassociatedwithpain,nausea,andthepostoperativestate.
Importantly,SIADHalsooccursasamedicationsideeffect,particularlywithpsychotropic
medicationssuchasantiepileptics,antipsychotics,andantidepressants(includingselectiveserotonin
reuptakeinhibitors[SSRIs]).

3 Whatisthefunctionofantidiuretichormone?
ADH(previouslyknownasvasopressin)hastwoprimaryfunctionsthatcorrespondtoitstwonames:
themaintenanceofplasmaosmolarityandthemaintenanceofplasmavolumeandbloodpressure.

Tomaintainplasmaosmolarity,ADHissecretedinresponsetoincreasedplasmaosmolarityand
reducestheosmolaritybyincreasingfreewaterreabsorptioninthecollectingductsofthekidneys.It
doessobystimulatingtheinsertionofwaterchannels(aquaporins)intotheluminalmembranesof
thecollectingducts.Thisistheantidiureticfunctionofthehormone.
Asdescribedearlier,ADHhelpsmaintainplasmavolumeandbloodpressureinthesettingof
dramaticallydecreasedECV.InadditiontoincreasingwaterreabsorptiontoexpandECV,itworksto
maintainbloodpressurethrougharterialvasoconstrictionhencetheoldertermvasopressin.

4 WhyistheantidiuretichormonereleaseinSIADHinappropriate?
TheexcessADHreleaseseeninSIADHisconsideredinappropriatebecausetheADHreleaseisnot
occurringinresponsetoappropriatephysiologicstimuli,thatis,increasedosmolarityand
hypovolemia.Rather,theADHsecretionoccursinanunregulatedautonomousfashion.
Ingeneral,patientswithSIADHareeithereuvolemicoronlymildlyhypervolemic.Thisseemingly
unnaturalphenomenonoccursbecausedespitetheelevatedADHlevels,thebody'sother
mechanismsforregulatingvolumestatusarepreserved.Inparticular,theincreasedADHstimulated
waterreabsorptioniscounteredbydecreasedactivityofthereninangiotensinaldosteroneand
sympatheticnervoussystemsandincreasedlevelsofBNP,suchthatECVandECFvolumeare
maintainednearnormal.
EuvolemiainspiteofelevatedADHlevelsisimportantfortworeasons.First,itreinforcesthenotion
thatvolumeisregulatedthroughsodiumbalancewhereasosmolarityandsodiumconcentrationare
regulatedthroughwaterbalance.Second,ithelpsdistinguishSIADHfromothercausesof
hyponatremiathataregenerallyassociatedwithpronouncedhypovolemiaorhypervolemia(more
specifically,areducedeffectivecirculatingvolume[e.g.,profusediarrhea,CHF,nephrotic
syndrome]).

5 Whatistheresultofinadequateantidiuretichormonerelease?
AninadequatelevelofADHisthecharacteristicfeatureofcentraldiabetesinsipidus(DI),inwhich
theposteriorpituitarydoesnotsecretesufficientADH.DIismarkedbyexcessiverenallossoffree
water,resultingindiluteurine,increasedplasmaosmolarity,andhypernatremia.Notethatthe
productionofconcentratedurineinasettingofhypernatremiaisalsoinappropriate.
DIcanbegenerallydividedintotwomajorcategories:centralDIornephrogenicDI.Theinadequate
ADHreleaseofcentralDIisseenwithheadtrauma,surgery,orotherintracranialprocessessuchas
infection,tumor,orstroke.CentralDIistreatedwithasyntheticanalogofADHcalleddesmopressin
(orDDAVP).Thisdrugbindsspecificallytovasopressin2(V2)receptorsonthecollectingductthat
mediatetherenaleffectsofADH.
NephrogenicDIresultsfromtheinabilityofthekidneystorespondtoADHandiscausedbydrug
toxicity(e.g.,lithium),hypercalcemia,andvariousotherconditions.NephrogenicDI,bydefinition,
willnotrespondtoDDAVP.TheunderlyingcauseofnephrogenicDImustbecorrected,althoughsalt

restrictionandthiazidediuretics(toblockrenaldilutingability)maybehelpful.
Distinguishingcentraldiabetesinsipidus(DI)fromnephrogenicDIishighyieldforStep1.
RememberthatnephrogenicDIwillrespondpoorlytosyntheticanalogsofantidiuretic
hormone(ADH),whereascentralDIwillshowafullresponsetotreatment.
Step1Secret

6 Howdoeshyponatremiaresultinthecentralnervoussystemsymptoms
(fatigue,anorexia,andconfusion)seeninthispatient?
Sodiumistheprimarydeterminantofplasmaandinterstitialfluidosmolarity.Hyponatremiathus
resultsinanosmoticshiftofextracellularfluidintocells,includingbraincells.Thiscanresultin
cerebraledemaandawidevarietyofneurologiceffects.Ifhyponatremiaissevere,comaand
convulsionsmayoccur.

7 Whymustthehyponatremiabecorrectedslowlyinthispatient?
Overlyrapidcorrectionofhyponatremia(>12mEq/24hours)isthoughttoplacepatientsathighrisk
forthedevelopmentofcentralpontinemyelinolysis(CPM),adisorderthatcanresultinflaccid
quadriplegia,dysphagia,facialweakness,andinsomecases,coma.Ararebutclassicmanifestation
ofponsdestructioninCPMisthelockedinsyndrome,inwhichaconsciouspatientdemonstrates
paralysisofallmusclesexceptthoseinvolvedineyeopeningandverticalgaze.Thepathophysiology
ofCPMisbelievedtoinvolveoverlyrapidshiftoffluidbackoutofbraincellsinresponsetoarapid
increaseinplasmaosmolarity,resultinginthedeathofmyelinproducingoligodendrocytesandloss
ofmyelinintheponsandotherregionsofthebrain.Infact,magneticresonanceimaging(MRI)
studieshaveshownthatmultipleareasofthebrainaredamagedwiththeoverlyrapidcorrectionof
hyponatremia,leadingtosymptomssuchascognitiveandpsychiatricdysfunction.Thus,theterm
osmoticdemyelinationsyndromeisperhapsabetterdescriptorofthepathophysiologythanthe
classictermcentralpontinemyelinolysis.Thisdisorderisclassicallyseeninalcoholicswhoseemto
bepredisposedonthebasisofmalnutrition.
Overlyrapidcorrectionofhypernatremiaalsoleadstoseriouscentralnervoussystem(CNS)toxicity,
butintheformofcerebraledemaratherthanCPM.Again,itisprudenttoavoidcorrectingfaster
than0.5mEq/L/hour.

8 Whywasitimportanttoaskaboutvomiting,diarrhea,ordiureticuse?
Theseareallcausesofsodiumwastingthatcanresultinhyponatremia.Notethatinthesecasesthere
aresignificantfluid(i.e.,volume)lossesalso,resultinginhypovolemichyponatremia.Incontrast,
SIADHisassociatedwithaeuvolemicormildlyhypervolemichyponatremia(seequestion4earlier).
Othercausesofhypovolemichyponatremiaincludesaltwastingfrommineralocorticoiddeficiency
(i.e.,adrenalinsufficiency)orthirdspacingevents(e.g.,severeburns,pancreatitis,orbowel
obstruction).

Note:Inthissetting,theADHreleaseisappropriate(i.e.,stimulatedbythehypovolemiaand
decreasedECV).

9 Whywasitimportanttoexaminethealbuminlevelandlookforevidenceof
heartfailure?
Hypoalbuminemia(fromliverdiseaseornephroticsyndrome)andheartfailurebothcausefluid
retentionandhyponatremia,resultinginhypervolemichyponatremia.
Inthissetting,ADHreleaseisappropriatebecausedespitetotalbodyhypervolemia,theseconditions
arecharacterizedbydecreasedECV.

10 Howwouldlevelsofplasmaantidiuretichormone,plasmaosmolarity,and
serumosmolaritybeexpectedtodifferbetweensyndromeofinappropriate
secretionofantidiuretichormone,diabetesinsipidus,andpsychogenic
polydipsia?
ThesefeaturesarecomparedinTable41(t0010).

Antidiuretichormone(ADH,vasopressin)hastwoprimaryeffects:(1)increasedfree
waterreabsorptioninthecollectingducts(theantidiureticfunction)and(2)systemic
vasoconstriction(thepressorfunction).
Undernormalconditions,ADHisreleasedinresponsetoincreasedosmolarity.ADH
releaseisalsostimulatedbyalargedecrease(i.e.,dropof5%to10%ormore)ineffective
circulatingvolume(ECV).ReleaseofADHthatisnotcausedbyeitherstimulusis
consideredinappropriate.
Theclassiccauseofhyponatremiaduetosyndromeofinappropriatesecretionof
antidiuretichormone(SIADH)isectopicADHsecretionbyasmallcellcarcinomaofthe
lung.Othercausesincludepituitaryorhypothalamiclesions,variousotherlungand
centralnervoussystem(CNS)diseases,drugsideeffects,pain,nausea,andthe
postoperativestate.
HyponatremiaandhighADHlevelscanoccurincongestiveheartfailure(CHF),
cirrhosis,ornephroticsyndrome.Othercausesofhyponatremiaincludesodiumwasting
viathegastrointestinal(GI)tract(i.e.,fromvomitinganddiarrhea)orkidneys(i.e.,from
diureticuse).Inthesediseases,ADHreleaseisconsideredappropriateduetothe
reducedeffectivecirculatingvolume.
InadequateADHresultsindiabetesinsipidus(DI)withfreewaterwastingand
hypernatremia.DIiseithercentral(inadequateADHrelease)ornephrogenic(inabilityof
thekidneytorespondtoADH,thinklithiumexposure).
SummaryBox:RenalControlofExtracellularFluidOsmolarity

Table41
ADHLevelsinSiadh,DiabetesInsipidus,andPsychogenicPolydipsia

Syndrome

SIADH

Plasma Plasma

Urine

ADH

Osmolarity Osmolarity

High

Low

Pathophysiologic

ClassicStepI

Explanation

Association

Inappropriately ADHcausesexcesswater

Smallcell

high

reabsorptionbythekidneys,

carcinomaof

creatinganinappropriately
concentratedurineinthe

thelung

settingoflowplasma
osmolarity
Central

Low

High

Low

diabetes
insipidus

LackofhypothalamicADH

Headtrauma

secretioncauseswastingof
freewaterinexcessof

(disruptionof
thepituitary

sodium,resultingin

stalk)

hyperosmolarplasma
Nephrogenic High

Inabilityofthekidneysto

Lithium

diabetes

respondtoADHresultsin

toxicity

insipidus

wastingoffreewaterand
hyperosmolarplasma

Psychogenic Low

High

Low

polydipsia

Low

Maximally

Enormousfreewaterintake

Schizophrenia

dilute(about

overwhelmsabilityofnormal

orother

50mOsm)

kidneystoexcretefreewater

psychiatric

despitecreatingamaximally
diluteurine

disease

ADH,antidiuretichormoneSIADH,syndromeofinappropriateantidiuretichormone[secretion].

PharmacologyofDiuretics
1 Howdodiureticsworktolowerextracellularfluidvolume?
Alldiureticsacttoinhibitthereabsorptionofsodiumandtherebyincreasetherateofexcretionof
sodiumandlowerECFvolume.Inotherwords,alldiureticsarenatriuretics.
However,theeffectofalldiureticsisultimatelylimitedinthatthedecreaseinECFvolumeachieved
resultsinlesssodiumdeliverytothenephrons,suchthattherateofsodiumexcretioneventually
decreasesbacktobaseline(i.e.,equaltotherateofsodiumintake).Anewsteadystateisachievedat
alowerECFvolume.ContinuedadministrationofthediureticisrequiredtomaintainthelowerECF
volume.

2 Whatpercentageofthefilteredsodiumisreabsorbedundernormalconditions
(i.e.,intheabsenceofdiuretics)?
Themajorityofsodiumisreabsorbedintheproximaltubule,andeachsubsequentsegment
reabsorbsprogressivelyless.TheactualamountsarelistedinTable42(t0015).
Table42
SodiumReabsorptionAlongtheNephron

Segment

Percentage

Proximaltubule 60
LoopofHenle

25

Distaltubule

10

Collectingduct

Noticethatabout99%offilteredsodiumisnormallyreabsorbed.Ingeneral,theproximaltubuleand
ascendingloopofHenlearerelativelypermeableandreabsorblargeamountsofsodium(andwater
withit)viarelativelylowenergyrequiringmechanisms,whereasthesubsequentsegmentstendtobe
impermeableandusehigherenergymechanismstoactivelyextractincreasinglysmallamountsof
sodiumremainingagainstprogressivelyhigherconcentrationgradients.
Thesepercentagesareimportantinthatthepotencyofeachtypeofdiureticdependsnotonlyonthe
amountofsodiumreabsorptionthatcanbepotentiallyinhibitedineachregion,butalsoonthe
resorptivecapacityofthenephronsegmentsdistaltothesiteofdiureticaction.Thesedistal
segmentstendtocompensatefortheinhibitionofproximalNa+reabsorptionbyincreasing
reabsorption.Forexample,althoughonemightpredictthatinhibitionofproximaltubularsodium
reabsorption(by,forexample,carbonicanhydraseinhibitors)wouldresultinthegreatestdiuresis,it
tendstoproduceonlyasmalldiuresisbecausethedownstreamnephronsegmentsrespondto
increasedsodiumdeliverywithincreasedreabsorption.
Diureticsthatactinthedistaltubuleandcorticalcollectingduct(i.e.,theK+sparingdiuretics)are
alsoweakdiureticsbecausetheycaninfluence,atmost,only10%to15%ofthefilteredsodiumload.
Loopdiureticsarethemostpotentdiuretics,inpartbecausealargeamountofsodiumisreabsorbed
intheloopofHenleandthedistaltubuleandcollectingductdownstreamarelimitedintheirability
tocompensate.Note,however,thattheprimaryreasonwhyloopdiureticsarethemostpotent
diureticsisthattheyimpairthegenerationofthemedullaryinterstitialosmoticgradientthatallows
forurineconcentration(seequestion3,next).

3 Inwhichregionofthenephrondoeseachofthemajordiuretictypes(carbonic
anhydraseinhibitors,osmoticdiuretics,loopdiuretics,thiazides,andK+
sparingdiuretics)act?Whataretheirmechanismsofactionandtheirmajor

uses?
Thedetailsofrenalsodiumtransportandthemechanismsofeachdiuretictypearebestlearnedin
conjunctionwithtoregionsofthenephron.
Carbonicanhydraseinhibitors(CAIs),suchasacetazolamide,effectivelyacttoinhibitNaHCO3
(sodiumbicarbonate)reabsorptionintheproximaltubule(theirmechanismispreviouslyreviewed
indetailinRenalControlofAcidBaseBalance,question4).CAIsarenotcommonlyusedas
diuretics,inpartbecause,asmentionedearlier,theyhaveaweakdiureticeffectduetotheirproximal
siteofaction.Furthermore,becausethemetabolicacidosisthatdevelopsfromCAIadministration
decreasestheamountoffilteredbicarbonatethatispotentiallyreabsorbed,thissmalldiureticeffect
israpidlylost.Inadditiontothetreatment/preventionofaltitudesickness,CAIsareusedtotreat
openangleglaucoma,ascarbonicanhydraseisinvolvedinthesynthesisofaqueoushumorofthe
eye.
Osmoticdiureticsalsoactprimarilyintheproximaltubule.Osmoticdiureticsaresubstancesthatare
freelyfilteredattheglomerulusbutarepoorlyreabsorbed.Theyinhibitsodiumandwater
reabsorptionbyincreasingtheosmolarityofthetubularfluidandcounteractingthenormalsmall
osmoticgradientresponsibleforreabsorptionofwaterandsodiumintheproximaltubule.Osmotic
diureticsarenotoftenclinicallyusedfortheirdiureticeffects.Themostcommonlyused
pharmacologicosmoticdiureticismannitol,whichcanbeusedintheacutetreatmentofcerebral
edema/elevatedintracerebralpressureorelevatedintraocularpressure(asinacuteclosedangle
glaucoma).Certainendogenoussubstances(suchasglucose,urea,andcalcium)canactasosmotic
diureticswhenpresentatveryhighserumlevels(e.g.,glucoseindiabetesorureainrenalfailure).
Theloopdiuretics,suchasfurosemide(Lasix),blocktheNa+/K+/2Clcotransporterpresenton
theluminalsurfaceofthethickascendinglimboftheloopofHenle.Inadditiontotheirsiteofaction,
loopdiureticsarethemostpotentdiureticscurrentlyavailablebecausetheirinhibitionofsodium
transportintheloopofHenleabolishesthecountercurrentmechanismusedtogeneratethe
concentratedmedullaryinterstitiumrequiredtoexcretemaximallyconcentratedurine.Inaddition
toimpairingconcentratingability,theyalsoblockthenephron'sdilutingabilitybecausetheyactto
blockthereabsorptionofsodiuminthewaterimpermeablethickascendinglimb.Loopdiureticsare
firstlineagentsintreatmentoffluidoverloadcausedbycardiac,hepatic,orrenaldisease(i.e.,CHF,
cirrhosis,nephroticsyndrome).Theyareparticularlyusefulintreatmentofacutecardiogenic
pulmonaryedemabecauseloopdiureticsalsohaverapidvenodilatoreffectsthateffectivelydecrease
cardiacpreload.
Thiazidediuretics,suchashydrochlorothiazide,acttoblocktheNa+/Clcotransporteronthe
luminalsurfaceoftheearlydistaltubule(i.e.,thecorticaldilutingsegment).Again,becausethey
inhibitsodiumreabsorptionatawaterimpermeablesegmentofthenephron,thiazidesimpairthe
nephron'sdilutingabilityunlikeloopdiuretics,however,theydonotalterconcentratingability.
Theyarerelativelypotentdiuretics.Theirmajoruseisinthetreatmentofisolatedmildtomoderate
hypertensioninfact,theyareconsideredfirstlineagentsforthisusebecausetheyarevery

inexpensiveandwereshowninthelargestantihypertensivetrialtodate(ALLHATStudy)to
effectivelyreducecardiovascularmortalityrelativetocalciumchannelblockers,ACEinhibitors,and
alphablockers.
TherearetwosubtypesofK+sparingdiuretics.Theyallinhibit,eitherdirectlyorindirectly,the
activityofaspecifictypeofNa+channel(theamiloridesensitivechannel)ontheluminalsurfaceof
theprincipalcellsofthelatedistaltubuleandcorticalcollectingduct(Fig.45(f0030)).The
expressionofthesesurfacechannelsisnormallyincreasedbytheactivityofaldosterone.Amiloride
andtriamtereneactbydirectlyblockingtheseNa+channels(aldosteroneindependentaction),
whereasspironolactoneandeplerenoneactbyblockingthemineralocorticoidreceptoruponwhich
aldosteronenormallyacts(aldosteronedependentaction).Becausesodiumreabsorptioninthedistal
tubularcellsislinked(viathebasolateralNa+/K+ATPase)toK+excretion,inhibitionofNa+
reabsorptionresultsinretentionofK+aswell.K+sparingdiureticsaretheweakestdiureticsas
such,theyaretypicallyusedasadjunctstootherdiureticsinthetreatmentoffluidoverloadand
hypertension.Theyareparticularlyusefulinpreventinghypokalemiacausedbyotherdiuretics.The
twoaldosteroneantagonistsalsohaveanimportantroleinthetreatmentofCHF,forwhichtheyhave
beendocumentedtohaveamortalityriskbenefit(similarlytobetablockersandACEinhibitors).The
aldosteroneantagonistsarealsoparticularlyusefulinthetreatmentoffluidoverloadrelatedto
cirrhosis,which,likeCHF,ischaracterizedbylowECVandhighaldosteronelevels.Theycanalsobe
usedinthespecifictreatmentofothercausesofhyperaldosteronism.

Figure45
TransportpathwaysinprincipalcellsandH+secretingintercalatedcellsofthedistaltubuleandcollectingduct.

CA,carbonicanhydrase.
(FromKoeppenBM,StantonBA:RenalPhysiology,4thed.Philadelphia,Mosby,2007.)

4 Howdoeseachmajordiuretictypeaffecttheconcentrationsofserum
electrolytes?
AlldiureticsincreaseK+secretionexcept,ofcourse,forK+sparingdiuretics.Hypokalemiais
particularlycommonwiththestrongerloopandthiazidediuretics.Therearemultiplemechanisms
fordiureticinducedhypokalemia.First,proximallyactingdiureticsincreasetheflowrateoffluid
pasttheprincipalcellsofthelatedistaltubule,whichlowerstheK+concentrationofthetubular
fluidandtherebypromotesK+secretion.Similarly,proximallyactingdiureticsincreasedistal
deliveryofNa+whichalsopromotesK+secretionbyincreasingtherateofexchangeofNa+forK+
(seeFig.45(f0030)).DiureticsalsoindirectlypromotehypokalemiabydecreasingECV,whichleads
toincreasedaldosteronelevelsandaldosteronestimulatedK+secretion.CAIsalsopromote
hypokalemiaviainductionofametabolicacidosis.Inthelatedistaltubule,K+andH+aresecreted
inexchangeforreabsorbedNa+suchthatincreaseddistaldeliveryofH+resultsinadecreaseinH+
secretionandaconcomitantincreaseinK+secretion(seeChapter5,AcidBaseBalance,formoreon
theinteractionbetweenK+levelsandacidbasebalance).
Hyponatremiacanoccurasasideeffectofeitherlooporthiazidediuretics.Bothofthesediuretic
typesimpairthenephron'sdilutingability(i.e.,thenephron'sabilitytoseparatewaterandsodium).
Diureticinducedhyponatremiaoccursparticularlywiththiazidesinelderlypatients(whosekidneys
tendtohaveadecreaseddilutingabilityatbaseline).
TheeffectsofdiureticsonCa2+andMg2+balancearerelativelycomplicatedandinvolve,among
othermechanisms,changesintheluminalvoltagegradients.However,themostclinicallyrelevant
effectsarethedramaticincreaseinCa2+excretioncausedbyloopdiureticsandtheuniqueabilityof
thiazidestodecreaseCa2+excretion.Loopdiureticsarefrequentlyusedwithintravenousfluidsin
thetreatmentofsevereorsymptomatichypercalcemia(suchashypercalcemiaofmalignancy).
Thiazideshaveuniqueeffectsoncalciumbalancebecausethedistaltubuleistheonlysegmentofthe
nephroninwhichcalciumreabsorptiondoesnotoccurinparallelwithsodiumreabsorption.Because
thiazidestendtocausehypercalcemiabydecreasingurinarycalciumconcentration,theycanbeused
inthepreventionofcalciumkidneystoneformation.Recallthatelevatedserumcalciumpredisposes
tocalciumstoneformationonlybypromotingelevatedurinarycalcium.
Incontrastwiththedifferingeffectsoncalcium,loopandthiazidediureticscanbothcauseclinically
relevanthypomagnesemia.
Finally,eachofthemajordiuretictypeshasapredictableeffectonacidbasebalance.CAIs,by
inhibitingbicarbonateexcretion,tendtopromotemetabolicacidosis.Thestrongerthiazideandloop
diuretics,byproducingdecreasedECV,tendtopromoteacontractionalkalosis.Contractionalkalosis

referstoanincreaseinbicarbonatereabsorptionthatoccurswithfluidlossandsubsequentsodium
reabsorption.K+sparingdiureticstendtopromote(agenerallymild)acidosisbypromotingH+
retentioninamannersimilartotheirabilitytopromoteK+retention(i.e.,inhibitionofNa+
reabsorptioninthelatedistaltubuleinhibitsthesecretionofK+andH+inexchangeforNa+).
SeeChapter5,AcidBaseBalance,formoreontheeffectsofdiureticsonacidbasebalance.

5 Whataretheotherrelativelycommonorimportantsideeffectsofdiuretics?
Oneofthemostcommonsideeffectsofdiureticsisadirectextensionoftheirtherapeuticeffect:
overdiuresisandhypovolemia.Prerenalazotemiafromoverdiuresisisbyfarthemostcommontype
ofdiureticinducednephrotoxicity.
Multiplecommonlyuseddiureticscancausehypersensitivityreactionsassociatedwiththe
sulfonamideresiduestheycontain.Thesediureticsincludeacetazolamide,thiazides,andmultiple
loopdiuretics(includingfurosemide).Similarly,loopdiureticsandthiazidescancauseallergic
interstitialnephritis(albeitratheruncommonly).Ethacrynicacidisuniqueamongtheloopdiuretics
inthatitdoesnotcontainasulfonamideresidueandhencewouldhavenoallergiccrossreactivity
withotherloopdiureticsorsulfonamides.
Thepotentloopdiureticshavethemostpotentialforvolumedepletionandelectrolyteabnormalities.
Loopdiureticsalsotendtopromotehyperuricemia,which,alongwithdiureticinducedhypovolemia,
canaggravateortriggergout.Loopdiureticscanalsocauseototoxicity(particularlyringinginthe
earsorhearingloss),butgenerallyonlywithlarge,rapidlyadministeredintravenousdoses.
Ototoxicityisrareinthedosescommonlyusedinclinicalpracticetoday.
Thiazidesaresimilartoloopdiureticsintheirpotentialtoexacerbatehyperuricemiaandgout.
Thiazidescanalsosomewhatexacerbatehyperlipidemiaorhyperglycemia(thoughgenerallynottoa
clinicallyrelevantdegree).
Spironolactoneisarelativelynonspecificinhibitorofcorticosteroidreceptors,withsignificant
antiandrogeneffectsinadditiontoitsantimineralocorticoideffects.Theseeffectsonsexsteroidscan
manifestasgynecomastiaanderectiledysfunctioninmenandhirsutismorbreasttendernessin
women.Thesenonspecificeffectsofaldosteronismcanbeclinicallyusefulinthetreatmentofacne
vulgarisandhirsutism(i.e.,spironolactonemaybothcauseandtreathirsutism).Incontrast,
eplerenoneselectivelyblocksonlythemineralocorticoidreceptorsandthereforedoesnothavethese
antiandrogeneffects.
Mechanismsofaction,uses,andsideeffectsofdiureticsandangiotensinconvertingenzyme
(ACE)inhibitorsareextremelyimportanttofocusonwhenstudyingforStep1.This
importanceisprobablynosecrettoyouatthispoint.Thedetaileddiscussioninthischapter
speaksforitself!
Step1Secret

Alldiureticslowerextracellularfluidvolumebyinhibitingthetubularreabsorptionof
sodium.
Thepotencyofeachdiureticdependsbothonthecapacityforsodiumreabsorptionof
thesegmentuponwhichthediureticactsandupontheresorptivecapacityofthe
downstreamsegmentscapableofcompensatingfortheproximalinhibitionofsodium
reabsorption.
Carbonicanhydraseinhibitorsandosmoticdiureticsactprimarilyintheproximal
tubule.
Loopdiuretics,themostpotentdiuretics,actbyblockingNa+reabsorptioninthethick
ascendinglimbandabolishingthenephron'sconcentratingability.Theyarefirstline
agentsinthetreatmentoffluidoverloadduetocardiac,hepatic,orrenaldisease.
ThiazidediureticsblockNa+reabsorptionintheearlydistaltubuleandarefirstline
agentsinthetreatmentofisolatedmildtomoderatehypertension.
K+sparingdiureticseitherdirectlyblockNa+reabsorptioninthelatedistaltubuleor
corticalcollectingductordosoindirectlybyblockingtheactivityofaldosteroneupon
mineralocorticoidreceptors.
Alldiuretics,exceptforK+sparingdiuretics,tendtoincreaseK+excretionandcause
hypokalemia.
Loopdiureticscandramaticallyincreasecalciumexcretion,whereasthiazidesincrease
calciumreabsorption.
SummaryBox:PharmacologyofDiuretics

Copyright2015Elsevier,Inc.Allrightsreserved.

BOOKCHAPTER

Gastroenterology
ThomasA.BrownMDandSonaliJ.Shah
USMLEStep1Secrets,Chapter6,134163

Gastroenterologyisacomprehensivesubjectonboardsthattendstointegrateanumberof
fieldsintoitsquestions.Youwill,forexample,seegastrointestinal(GI)questionsrelatedto
physiology,pathology,andmicrobiology.AndalthoughGIpharmacologyisindeedfairgame
forthetest,itisnotashighyieldastheaforementionedtopics.Youmayhavealsonoticedthat
theGIsectioninFirstAidincludesanextensiveanatomysectionthereisareasonforthis!
Someofthehighestyieldanatomypointscomefromthissection,anditisthereforeimportant
thatyouknowthebranchesoftheceliactrunk,therelationshipofthepancreaticheadtothe
commonbileduct,thelocationsofvarioustypesofhernias,etc.Itisalsoadvisablethatyou
spendtimelearningtherolesofthevariousGIhormones.GIpathologyontheboardscoversan
extensivenumberoftopics,butdonotpanic.SeveralUSMLEfavoritesaredescribed
throughoutthecasesinthischapter,andyoushouldreadthemcarefullytodiscernthesubtle
differencesbetweenthevariousdiseases.GIpathologyisnotanintrinsicallydifficultsubject,
butdiseasepresentationscanblendtogetherifyoudonotclassifythemwellasyouare
attemptingtolearnthem.
Insider'sGuidetoGastroenterologyfortheUSMLEStep1

Basicconcepts
1 Whatisthemajorstimulusforgastrinsecretion?Whatarethephysiologic
actionsofgastrininthestomach?
ProteininthestomachistheprimarystimulusforthesecretionofgastrinbyGcellsintheantrumof
thestomach(otherstimuliincludestomachdistention,stomachalkalinization,andvagal
stimulation).Thesecretedgastrinstimulatesthesecretionofhydrochloricacidandintrinsicfactor
(IF)bythegastricparietalcellsandstimulatessecretionofpepsinogenbythechiefcells.Noticethat
allthesesecretionsareimportantinprotein/meatdigestion,becausetheacidenvironmentcreated
helpshydrolyzeproteinsandcreatesanoptimalpHinwhichpepsinworks(~pH2).Additionally,
thesecretedIFbindsandprotectsthevitaminB12thatispresentinmeat.Amajorinhibitorof
gastrinsecretionisdecreasedgastricpH,whichstimulatessomatostatinsecretionfromDcellsinthe
pancreasandgastrointestinalmucosa.SomatostatinanddecreasedpHarebothnegativefeedback
mechanismstokeepthestomachfrombecomingtooacidic.

2 Whatarethemainpancreaticenzymesandwhataretheirfunctions?
Pancreaticenzymesareallinvolvedindigestion(degradation)offoodmacromolecules.Pancreatic
amylasedegradesstarch/complexcarbohydrate,trypsinandchymotrypsindegradeproteins,and
lipasehydrolyzestriglyceridestherearealsoDNaseandRNaseenzymesinpancreaticsecretions.
Thepancreasalsosecretesbicarbonate,whichneutralizesacidicchymeenteringtheduodenumand
createsthepHnecessaryforpancreaticenzymestowork.
Pancreaticenzymesaresecretedaszymogens(inactiveprecursormolecules),whicharecleavedinto
theiractiveformwithintheintestinallumen.Thispreventsautodigestionofthepancreatictissueby
theactiveenzymeforms.Trypsinogeniscleavedintotrypsinbyenterokinasewithintheintestinal
mucosa.Trypsinthenactivatesmoretrypsinogenaswellasotherpancreaticenzymes.
Note:Thepancreaticacinarcellsarerichinsecretorygranulesfullofenzymes,andthepancreatic
ductalcellsareprincipallyresponsibleforbicarbonateandfluidsecretion.

3 Whataretheprimaryhormonalstimuliforthepancreaticexocrinesecretions
andhowdothesesecretionsdifferincontent?
Cholecystokinin(CKK)primarilystimulatesthesecretionofenzymes(e.g.,proteasessuchas
trypsinogen)fromthepancreaticacinarcells,whereassecretinprimarilystimulatesthesecretionofa
bicarbonaterichfluidfrompancreaticductalcells.

4 Whataretheprimarystimuliforthesecretionofcholecystokininandsecretin
andfromwherearethesehormonessecreted?
CCKissecretedprimarilyinresponsetofattyacidsenteringtheduodenum,whereassecretinis
releasedprimarilyinresponsetoacidificationoftheduodenum.Botharesecretedfromthe
duodenum.

5 Whatotherdigestiveprocessesdoescholecystokininstimulate?
CCKalsostimulatescontractionofthegallbladderandrelaxationofthesphincterofOddi(wherethe
commonbileductenterstheduodenum),therebypromotingbileentryintotheduodenum.By
stimulatingpancreaticsecretionoflipolyticenzymesaswellasthedeliveryofbiletothesmall
intestine,thishormonecreatestheappropriatemilieuforthedigestionoffats.
Note:CCKalsodelaysgastricemptying.Thisexplainswhythesensationoffullnesslastslongerafter
afattymeal.

6 Whatisthefunctionofthebilesalts?Howaretheyformed?
Thebilesaltssolubilizefatsinmeals,creatingabiggersurfaceareaonwhichpancreaticlipasecan
work.Theyadditionallyformmicellesthatfacilitatethedeliveryoffattyacidstotheintestinal
enterocytesforabsorption.Bilesaltsareformedfromthedegradationofcholesterolinthelivervia
theenzyme7hydroxylase.Asapointofinterest,thisistheprimarymethodthebodyhasfor
eliminatingcholesterol.

7 Whatistheenterohepaticcirculationandwhyisitimportantinthedigestion
offats?
Enterohepaticcirculationoccurswhensubstancesaresecretedbytheliver,reabsorbedbythe
intestine,andreturnedtotheliver.Themajorityofthebilesaltsthataredeliveredtotheintestine
duringdigestionarefromintestinalreabsorptionofsecretedbilesalts.Indiseasesthatdecreasebile
saltreabsorption(e.g.,Crohn'sdisease),theenterohepaticcirculationandsecretionofbilesaltsfrom
theliverareimpaired,resultinginimpairedfatdigestion.

8 Whatdefinestheforegut,midgut,andhindgutanatomically?Whichmain
arteriesprovidethebloodsupplytoeachsegment?Whatnervessupplythese
regions?
Theforegutcomprisestheuppergastrointestinal(GI)tractdowntoasitejustdistaltotheampullaof
Vater(wherethecommonbileductemptiesintothesecondarypartoftheduodenum).Itsmain
vascularsupplycomesfromtheceliacartery.Themidgutextendsfromthesecondpartofthe
duodenumdowntothesplenicflexureofthecolonandisservedbythesuperiormesentericartery.
Thehindgutextendsfromthesplenicflexureofthecolontotheanusandissuppliedbytheinferior
mesentericartery.Theforegutandmidgutareinnervatedbythevagusnerve,andthehindgutis
innervatedbythepelvicnerve.
Note:Thepancreasandliverareembryologicoutgrowthsoftheforegutandsharethevascular
supplyoftheforegut(i.e.,celiacartery).Althoughthespleenisnotanembryologicderivativeofthe
foregut(itismesodermalinorigin),itissuppliedbytheceliacarteryaswell.

9 Whataretheanatomiclayersofthegutwall?
Fromthelumenout,thelayersoftheGItractareasfollows:(1)mucosa,comprisingthemucosal
epithelium,laminapropria,andmuscularismucosae(2)submucosa,whichcontainsthesubmucosal
(Meissner's)nerveplexus(3)muscularispropria,comprisinganinnercircularsmoothmusclelayer,
myentericplexus,andouterlongitudinalsmoothmusclelayerand(4)serosa(adventitia),whichis
thefibrousoutercovering(Fig.61(f0010)).

Figure61
Layersofthegutwall.
(FromBrownTA:RapidReviewPhysiology.Philadelphia,Mosby,2007.)

Note:Thetaeniaecoliarebandlikemusclescomposingtheouterlongitudinalsmoothmusclelayer
ofthelargeintestine,exceptintheappendixandtherectum.

Case61
A45yearoldobesemanpresentswitha2yearhistoryofchestdiscomfortfollowingheavy
meals.Hedescribesthediscomfortasasubsternalburningsensationthatradiatestohisneck.
Thediscomfortworsenswhenheislyingdowninbedatnight.Hementionsthathealsohasa
chroniccoughandhoarsevoiceinthemorning.

1 Whatisthedifferentialdiagnosis?
Thedifferentialdiagnosisincludesgastroesophagealrefluxdisease(GERD),esophagealspasm,and
myocardialischemia.Notethatmyocardialinfarctionandanginashouldbepartofthedifferential
diagnosisinanycaseinwhichthereischestdiscomfort/pain,especiallyconsideringthatthispatient
isamiddleaged,obesemale,allofwhichareriskfactorsforheartdisease.GERDcharacteristically
presentswithheartburn,whichcanmimicmyocardialischemia,andregurgitationofsourmaterial
intothemouth.TheheartburnfromGERDistypicallyexacerbatedbybotheatinglargemealsand
bendingoverorbeingrecumbentinbed.Inadditiontoheartburn,patientswithGERDcanalso
presentwithchroniccoughfromgastricacidirritationofthetracheobronchialtreeandahoarse
voiceinthemorningfromthegastricacidirritationofthevocalcordsatnight.

Case61continued:
Physicalexaminationissignificantforobesity.Thereisnochestwalltenderness.Cardiac
examinationisunremarkable,exceptforauscultatingbowelsoundsinthepatient'schest.A
cardiacstresstestisnegativeforinducibleischemia.AbariumswallowisshowninFigure62
(f0015).

Figure62
Slidinghiatalhernia.Abulbouscollectionofcontrastrepresentingthestomachherniatedisevident
abovethediaphragm.Gastricfoldsarepresentinthehernia,identifyingitaspartofthestomach(
solidwhitearrow).Noticetheesophagusdoesnotnarrowasitnormallydoeswhenpassingthrough
theesophagealhiatus(dashedwhitearrow).Justabovetheherniaisathin,weblikefillingdefect
characteristicofaSchatzki'sring(dottedwhitearrow).TheSchatzki'sringmarksthelevelofthe
esophagogastricjunction.
(FromHerringW:LearningRadiology:RecognizingtheBasics,2nded.Philadelphia,Saunders,2012.)

2 BasedonthefindingsinFigure62(f0015),whatisthepatient'slikely
diagnosis?
Thepatienthasaslidinghiatalhernia.Inaslidinghiatalhernia,thegastroesophagealjunction
herniatesupwardthroughtheesophagealhiatusinthediaphragm.Theadditionalsphincteric
pressurethatisprovidedbythediaphragmisthenlost,allowingrefluxofgastriccontentstooccur
moreeasily.Thiscontrastswithaparaesophagealhiatalhernia,inwhichaportionofthegastric
fundusrollsintoandherniatesthroughthediaphragmbutthegastroesophagealjunctionremains
inplace.Althoughparaesophagealhiatalherniasusuallydonotcausereflux,theyaremoreserious
becausetheycanbecomeincarcerated(strangulated)andischemic.

3 Discussthepathophysiologyofgastroesophagealrefluxdisease
GERDiscausedbytherefluxofacidicorbiliousgastriccontentsintotheesophagus,whichirritates
theesophagealmucosaandcausespain.Manyfactorscanpredisposetothisreflux.Oneofthemost
importantfactorsisabnormaltransientrelaxationoftheloweresophagealsphincter(LES)unrelated
toswallowing.Acontinuallyrelaxed(atonic)LESwillalsoallowreflux.Increasedintraabdominal
pressure(duetoobesity,pregnancy,Valsalvamaneuver,etc.),increasedgastricvolume(dueto
eatingalargemealorgastroparesis),anddecreasingdistanceofgastriccontentsfromthe
gastroesophagealjunction(occurringwhenlyingdownorwithhiatalhernia)mayexacerbatereflux
ofgastriccontentsintotheesophagus.

Case61continued:

Thepatientisstartedona1weektherapeutictrialofomeprazole,whichprovidessubstantial
relief.Heisthereforestartedonalongtermcourseofomeprazole.

4 CoverthefarleftcolumnofTable61(t0010)andattempttonamethedrugs
usedinthetreatmentofGERDbasedontheclassofdrug,themechanismof
action,andtheprimarysideeffectslistedinthetable
Table61
DrugsUsedforTreatmentofGastroesophagealRefluxDisease

Drug

Class

MechanismofAction

Cimetidine
Ranitidine
Nizatidine

Histamine
Inhibitshistaminestimulated
H2receptor releaseofhydrochloricacidby
antagonists blockingH2receptorsonparietal

Famotidine

cells

Metoclopramide Antiemetic
Prokinetic

Prokineticdrug(increasesgastric
emptyingandLEStone)via

PrimarySideEffect
Cimetidineinhibitshepatic
cytochromeP450enzymes
andcausesgynecomastia

Parkinsoniansymptoms,
seizures

cholinergicsideeffects
Omeprazole

Proton

Irreversiblyinhibitstheparietalcell

Lansoprazole
Rabeprazole

pump
inhibitors

H+/K+ATPasepump

Hypergastrinemia

GI,gastrointestinalLES,loweresophagealsphincter.

5 WhichofthedrugsinTable61(t0010)arecontraindicatedinapatientin
whombowelobstructionissuspectedandwhy?
Prokineticdrugsshouldbeavoidedwheneverbowelobstructionissuspected,astheycanexacerbate
theobstructionandpotentiallycauseperforation.

6 Whymaytheomeprazolethispatientwasgivencausehimtodevelop
hypergastrinemia?
Protonpumpinhibitorssuchasomeprazoleinhibitgastrichydrochloricacid(HCl)secretionby
irreversiblyinhibitingtheH+/K+ATPasepumpongastricparietalcells.ThisraisesgastricpH,
whichdisinhibitsgastricsecretionbytheGcells,resultinginhypergastrinemia.

Case61continued:
Unfortunately,thepatientinitiallyislosttofollowup,butheshowsupathisphysician'soffice
3yearslatercomplainingof6monthsofprogressivelyworseningdysphagiatosolidsand
unintentionalweightloss.Hesayshetriedtobediligentabouttakinghisomeprazoleforthe

firstmonth,butsoonstoppedtakinghispillsandlearnedtotoleratethediscomfort.

7 Whichcomplicationsofgastroesophagealrefluxdiseasecouldberesponsible
forthepatient'sdifficultyswallowing?
Esophagealstrictureresultsfromfibrosingandscarringoftheesophagealmucosafromchronic
irritationbygastricacid.Thisscarringnarrowsthelumen,whichpreventsafoodbolusfrompassing
through,causingfoodtogetstuck.AnothercomplicationofGERDisBarrett'sesophagus,whichis
columnarmetaplasiaofstratifiedsquamousesophagealepitheliumthatcanleadtoesophageal
adenocarcinoma.Esophagealcarcinomausuallypresentswithrapidlyprogressivedysphagiaand
weightloss.

8 Differentiatebetweenthetwotypesofesophagealcancersadenocarcinoma
andsquamouscellcarcinomaintermsofriskfactorsandlocation
Adenocarcinomatypicallyoccursintheloweronethirdoftheesophagus.Barrett'sesophagusand
chronicirritationofesophagealepitheliumfromuntreatedGERDareriskfactorsforthistypeof
cancer.Squamouscellcarcinomausuallyoccursintheuppertwothirdsoftheesophagus.Smoking
andalcohol,especiallytogether(synergisticeffect),increasetheriskforthistypeofcancer.Bothof
theseesophagealcancerscancausedysphagiatosolids.

9 WhywouldapatientwithSjgren'ssyndromebemoresusceptibleto
esophagealpathologyingastroesophagealrefluxdisease?
Sjgren'ssyndromeisanautoimmunediseasecharacterizedbylymphocyticinfiltrationofthe
lacrimalandsalivaryglands,causingdryeyesanddrymouthduetodeficientsecretions.Because
salivaisrichinbicarbonate,itfunctionstoneutralizeacidintheesophagus.Theabsenceofthis
protectivefunctionpredisposespatientswiththisconditiontoesophagealdamagewithevenminimal
gastroesophagealreflux.

Gastroesophagealrefluxdisease(GERD)classicallypresentswithregurgitationofsour
contentsintotheesophagusororopharynxandsubsternalchestdiscomfortthatworsens
afterlargemealsandwheninarecumbentposition.
ThepathogenesisofGERDinvolvesinappropriaterelaxationoftheloweresophageal
sphincter(LES).
SlidinghiatalherniascanpredisposetoGERD.Paraesophagealrollinghiatalhernias
aresubjecttoincarceration.
GERDistypicallytreatedwithprotonpumpinhibitors,H2receptorantagonists,or
surgery(Nissenfundoplicationinrefractorycases).
LongtermcomplicationsofGERDincludeBarrett'sesophagus,adenocarcinoma,

esophagealstricture,andesophagealulceration.
SummaryBox:GastroesophagealRefluxDisease,HiatalHernias,andEsophageal
Carcinomas

Gastroesophagealrefluxdisease(GERD)isaboardfavorite.Beabletorecognizethesymptoms
ofchestpainfollowingmeals(maybedescribedasaburningsensationinthechest),sourtaste
inthemouth,andnocturnalcoughasaclassicpresentationforthiscondition.Youshouldalso
knowhowtorecognizecomplicationsofGERD(e.g.,Barrett'sesophagus,strictures,
adenocarcinoma)bybothmedicalhistoryandhistologicappearance(refertoChapter27,
Pathology,forimages).
Step1Secret

Case62
A50yearoldwomancomplainsofrecentdysphagiatosolidsandliquids.

1 Inpathophysiologicterms,howdoyouapproachdysphagia?
Dysphagiacanbeapproachedintermsoforopharyngealoresophagealetiology.
Oropharyngealdysphagiaiscausedbydifficultyinitiatingaswallowreflex,duetoeitherneurologic
ormuscularproblems.Typicalcausesincludestroke,amyotrophiclateralsclerosis(LouGehrig's
disease),andmyastheniagravis.Patientsusuallyhavecoughingorchokingwithdysphagiaof
oropharyngealetiology.
Esophagealdysphagiaiscausedbyfoodgettingstuckintheesophagusafterbeingswallowed.This
isduetoeithermechanicalobstructionoresophagealdysmotility.Historycanoftendistinguish
betweenanobstructiveetiologyoramotilitydisorder.Ifthepatienthasproblemswithonlysolid
foodsinitially,thenthissuggestsmechanicalobstruction.Thiscanprogresstoadvancedobstruction,
suchasfromesophagealadenocarcinoma,causingdysphagiatobothsolidsandliquids.Ifthepatient
hasdysphagiatobothsolidsandliquidsfromthebeginning,thissuggestsadysmotilitydisorder,
suchasachalasia,sclerodermalikeesophagus,ordiffuseesophagealspasm.

Case62continued:
Thepatientalsonotesoccasionalchestpainwitheating,nocturnalcough,andanunintentional
15lbweightlossinthelast2months.Shewasrecentlyadmittedtothehospitalfortreatment
ofpneumonia.EsophagealmanometryshowsincreasedLESpressurewithincompleteLES
relaxationandacompleteabsenceofperistalsisintheloweresophagus.Abariumswallowis
showninFigure63(f0020).

Figure63
Bariumswallowofpatientincase62.(st0125)
(FromCummingsCW,FlintPW,HaugheyBH,etal:Otolaryngology:HeadandNeckSurgery,4thed.
Philadelphia,Mosby,2005.)

2 Whatisthelikelydiagnosisinthiswoman?Describethepathophysiologyof
thisdisease
Theinitialdysphagiatosolidsandliquids,bariumswallowshowingtheclassicbird'sbeak
appearance,andesophagealmanometryfindingsofaperistalsisandincreasedLESpressureare
classicforachalasia.

TheLESisnormallytonicallyconstricted,generatingenoughintraluminalpressuretopreventthe
refluxofgastriccontentsintotheesophagus.Duringtheesophagealphaseofswallowing,theLES
relaxesinresponsetoafoodbolusdescendingthroughtheesophagus,aphenomenonreferredtoas
receptiverelaxation.Inachalasia,thereisdestructionofthemyentericplexus,whichmediates
receptiverelaxationandalsomediatesesophagealperistalsis(hencetheaperistalsis).Althoughthe
exactmechanismofincreasedLEStoneinpatientswithachalasiaisnotknown,researchsuggests
thatthesepatientshavelossofnitricoxidesecretingneurons,akeyfactorinLESrelaxation.The
failureoftheLEStorelaxtogetherwiththefailureofthedistalesophagustoundergoperistalsis
allowsfoodtoaccumulateanddilatetheloweresophagus,creatingthebird'sbeakappearanceon
bariumswallow(Fig.63(f0010)).
Note:Themyenteric(Auerbach's)plexusislocatedbetweentheinnercircularandtheouter
longitudinalsmoothmusclelayers(muscularispropria)oftheesophagus.

3 Chagasdiseaseisalsoknowntobeacauseofachalasia.Whatisthe
pathologicmechanismandwhatorganismistheprimaryculprit?
Chagasdisease(Americantrypanosomiasis)cancausedestructionofthemyentericplexusinthe
esophagus.ThisdiseaseiscausedbytheprotozoalparasiteTrypanosomacruziandisparticularly
commoninSouthAmerica.Inadditiontoachalasia,Chagasdiseasecanalsocausemegacolonby
destroyingthemyentericplexusofthecolon.

4 Whatwasthelikelycauseofthiswoman'spreviousepisodeofpneumonia?
Thepneumoniawasprobablycausedbyaspirationofesophagealcontents,especiallywhilesleeping,
duetothepresenceofundigestedmaterialintheesophagus.

5 Inadditiontopneumaticdilatationoftheloweresophagealsphincterand
surgicalmyotomy,injectionofbotulinumtoxinintotheloweresophageal
sphincterisatreatmentoptionforthiswoman.Whatisthisdrug'smechanismof
actioninthiscontext?
MuchofthetonicconstrictionoftheLESisduetovagalcholinergicinnervation.Becausebotulinum
toxinexertsitseffectsbyinhibitingthereleaseofacetylcholinefromnerveendings,itreducesthis
inputtoLEStone.

Becauseitisamotilitydisorder,patientswithachalasiatypicallypresentwithdysphagia
tobothsolidsandliquids.
Achalasiaiscausedbydestructionofthemyenteric(Auerbach's)plexus.InChagas'
disease,thisdestructionismediatedbytheprotozoanTrypanosomacruzi.
EsophagealmanometrywillclassicallyshowdistalaperistalsisandincreasedLEStone.
Bariumswallowwillshowabird'sbeakappearance.

TreatmentoptionsincludeLESdilatation,LESmyotomy,andinjectionofbotulinum
toxinintotheLES.
SummaryBox:Achalasia

Wheneveryouseeaclinicalvignettethatmentionsprogressivedysphagiatosolidsandliquids,
considerachalasia,scleroderma,andesophagealcancerinyourdifferentialdiagnosis.Consider
anoropharyngealetiology(e.g.,stroke)ifassociatedwithahistoryofchokingorcough.
Step1Secret

Case63
A33yearoldmanpresentstohisprimarycarephysiciancomplainingofa2monthhistoryof
gnawingepigastricpainthatdevelopsacoupleofhoursaftermeals.Thepainisparticularly
bothersomeatnightandoftenawakenshimfromsleep.

1 Whatisthedifferentialdiagnosis?
Thedifferentialdiagnosisofpostprandialepigastricpainincludesbiliarycolic,GERD,irritablebowel
syndrome(IBS),andpepticulcerdisease(PUD).Biliarycolicissevere,rightupperquadrantpain
causedbygallstonesandusuallyprecipitatedbyfattymeals.Ithasahigherfrequencyinwomenbut
isstillcommoninmen.GERDmaycauseepigastricorsubsternaldiscomfortthatworsenswithlarge
meals,bendingover,orlyingflat.IBScanproduceabdominalpainandbloatingthatisworsenedby
stressorfoodbutrelievedbydefecation.Patientstypicallyhavealteredbowelhabits(e.g.,diarrhea
orconstipation),whichthispatient'shistorydoesnotmention.PUDencompassesbothgastricand
duodenalulcers.Duodenalulcerstypicallycausemostpain1to3hoursafterameal,whentheacidic
chymeentersthesmallbowel.Painalsooccursatnight,duetocircadianrhythminducedacid
secretion.Foodorantacidscanrelievetheepigastricpain.Gastriculcershaveamorevariable
presentation,butepigastricpainistypicallyprecipitatedbyfoodandthereforepatientsmay
experienceweightlossduetofoodavoidance.Basedonthepatient'shistory,duodenalulceristhe
mostlikelydiagnosis.

Case63continued:
Afterabriefhistoryandphysicalexaminationthephysicianprescribessomeantacidsand
schedulesafollowupvisitin2weekstoseeifthistreatmentprovidesrelief.

2 Ifyouwerethisman'sphysician,whatwouldyouhavedonedifferentlyinthe
treatmentofthispatient?
Thepatientwithepigastricpainwarrantsathoroughhistoryandphysicalexamination,lookingfor
anyredflagsymptoms,suchasGIbleeding,anemia,dysphagia,persistentvomiting,or

unintentionalweightloss,astheseproblemscouldpointtowardmoreseriouspathologicprocess,
suchasmalignancy.ThetwousualculpritsforPUDarenonsteroidalantiinflammatorydrugs
(NSAIDs)andHelicobacterpylori.Amorethoroughhistorycouldhavediscoveredifthepatientwas
takingNSAIDs,whichyouwouldhaverecommendedthathestoptaking.TestingforH.pyloriwould
alsobenecessarytopreventrecurrence.
IntheabsenceofredflagsymptomsinayoungadultwithsuspectedPUD,visualizationoftheulcer
byupperendoscopyorbariumswallowisnotnecessary,andthepatientcanbeempiricallytreated
withH2receptorantagonists(e.g.,ranitidine)orprotonpumpinhibitors.

3 Howarenonsteroidalantiinflammatorydrugsthoughttopredisposetothe
formationofgastriculcers?Whatalternativesexisttolessengastrointestinal
sideeffects?
NSAIDsinhibittheproductionofprostaglandinsinthegastricmucosa.Theseprostaglandins
normallyfunctiontoprotectthegastricmucosabyincreasingmucusandbicarbonatesecretionand
bystimulatinglocalvasodilation,whichmaintainsmucosalperfusionandpreventsischemicinjury.
Twooptionsexisttocircumventthisproblem:takingmisoprostolwithNSAIDs(rarelydone)or
usingcyclooxygenase(COX)2inhibitors.MisoprostolisaprostaglandinE1analogthatdecreases
gastricacidsecretionandincreasesmucusandbicarbonatesecretion,therebydecreasingthe
deleteriouseffectsofNSAIDs.Asthenameindicates,COX2inhibitorsselectivelyinhibitCOX2and
donotaffectCOX1activity.Theconstitutivelyexpressedform,COX1,ispresentinmultipletissues
andisresponsiblefortheproductionofprotectiveprostaglandinsinthestomach.Theinducible
form,COX2,ispresentprimarilyininflammatorycellsandisresponsibleforproducing
proinflammatorysubstances.BypotentlyinhibitingCOX2andminimallyinhibitingCOX1,COX2
inhibitorscausefewerGIsideeffects.Unfortunately,COX2inhibitorscanbeprothromboticand
exacerbatehypertension,increasingtheriskofmyocardialinfarctionandstroke.Thismajorside
effectledtorofecoxib(Vioxx)beingpulledfromtheU.S.marketinrecentyears.

Case63continued:
Thepatientmisseshis2weekappointmentandreturns7monthslaterlookingpaleandtired.
Theantacidsinitiallyprovidedsomerelief,butnowthepainisbackandisworsethanbefore.
Inaddition,thepatienthasnoticedhehasdark,tarrystoolsandfeelstiredallthetime.

4 Basedontheappearanceofthestool,isthismorelikelyalower
gastrointestinalbleedoranuppergastrointestinalbleed?
Ourpatientpresentswithmelena,paleness,andfatigue,thelattertwoofwhicharesuggestiveof
anemia.ThemelenaindicatesanupperGIbleed(UGIB),likelycausedbyableedingduodenalulcer.
UGIBisanatomicallydistinguishedfromalowerGIbleed(LGIB)inthatitoccursproximaltothe
ligamentofTreitz.UGIBpresentswitheitherhematemesisormelena.Melenaoccursbecausethe
hemoglobinhastimetobebrokendownbybacteriaintheguttogivedark,tarrystools.

Hematochezia(brightredrectalbleeding)istypicallyindicativeofLGIB,inwhichbacteriadonot
havethetimetobreakdownhemoglobin.However,briskUGIBssuchasbleedingesophagealvarices
canpresentwithhematocheziabecausebloodstimulatesGImotilityanddecreasestransittime.

5 Whatarethemajorcomplicationsofpepticulcerdisease?
Hemorrhageisthemostcommoncomplicationandcanpresentashematemesisormelena.Potential
sourcesofhemorrhageincludeaposteriorpenetratingduodenalulcer(gastroduodenalartery)or
penetratinggastriculceronthelessercurvature(gastricartery).Perforationintotheabdominal
cavityusuallypresentsassuddenonsetofpainwithperitonitis.Achestorkidney,ureter,and
bladder(KUB)xraystudywillusuallydemonstratefreeairunderthediaphragm.Gastricoutlet
obstructioncanrarelyresultfromchroniculcersandtypicallypresentswithpersistentvomitingand
weightloss.GastriccarcinomacanulcerateandcausepainjustlikePUD.Therefore,gastriculcers
shouldtypicallybebiopsiedduringupperendoscopytoruleoutgastriccarcinoma.Chronicgastritis
andH.pyloripredisposetodevelopmentofgastriccarcinoma.Gastriccarcinomaisassociatedwith
metastasestotheleftsupraclavicularnode(Virchow'snode)andbilateralmetastasestotheovaries
(Krukenbergtumors),whichhavesignetringcellsasapathologicfinding.
Note:PancreaticadenocarcinomaalsometastasizestoVirchow'snode.

Case63continued:
Laboratoryevaluationrevealsahemoglobinlevelof10g/dL.Thepatientisreferredtoa
gastroenterologist,whoperformsanupperendoscopythatrevealsawelldemarcated,clean
basedulcerintheproximalduodenum,asshowninFigure64(f0025).

Figure64
Gastriculcer(whitebase)withbleedingvessel.
(FromGoldmanL,AusielloD:CecilTextbookofMedicine,22nded.Philadelphia,WBSaunders,2004.)

6 WhatenzymedoesHelicobacterpyloriproducethatcanbetestedfor?

H.pyloriproducestheenzymeurease,whichbreaksdownureatoliberateammoniaandcarbon
dioxide.Consequently,H.pyloricanbedetectedbyhavingapatientingest13Cor14Clabeledurea
andthendeterminingifthepatient'sbreathcontainsradiolabeledCO2.Interestingly,theliberation
ofammoniabyH.pylorineutralizesgastricacidandfacilitatesthesurvivalofthisorganisminthe
stomach.H.pyloriinfectioncanalsobedetectedbyserumantibodiestothisorganism,butthistest
doesnotdiscriminatebetweencurrentandpreviousinfectionunlessIgMisspecificallyrequested.

7 Duringendoscopy,ifmultipleduodenalulcershadbeenfoundinourpatient,
whatdiseasewouldwesuspectandhowwouldwetestforit?
Wheneverpepticulcersarerefractorytoaggressivetherapy,therearemultipleulcers,ortheulcers
arelocatedinabnormalpositionssuchasthejejunum,ZollingerEllisonsyndrome(ZES)(duetoa
gastrinsecretingtumororgastrinoma)shouldbesuspected.Theincreasedgastrinsecretionbythese
tumorscausesexcessivesecretionofacid.Amarkedlyelevatedgastrinlevel,usuallywithlevels
>1000pg/mL(fastinglevels<150pg/mL),isindicativeofZES.Thesecretintestwillalsoshowa
paradoxicalincreaseingastrin.
Note:RecallthatpatientsonPPIsmayalsohavesubstantiallyelevatedgastrinlevels,soitmaybe
difficulttoimmediatelydifferentiatebetweenapatientonPPIsandonewithZES.

Case63continued:
Thepatient'sureabreathtestispositiveandheisthereforeprescribedtripletherapy.

8 Whywasourpatientprescribedtripletherapy?
ThepatienthasanH.pyloriinfectionthatneedstobeeradicatedtoreducetherateofrecurrenceof
pepticulcerandtheriskofdevelopinggastricadenocarcinoma.
Note:TripletherapytypicallyincludestwoantibioticsandanH2blockeroraprotonpump
inhibitor.

Case63continued:
Athis4weekfollowup,thepatientdeniesanyabdominalsymptomsbutdoesnotethathehas
noticedfattyenlargementofhisbreasts.

9 Whatwasthemostlikelycauseofhisgynecomastia?
Oneofthedistinctivesideeffectsofcimetidineisgynecomastia.Anothernoteworthyfactabout
cimetidineisthatitistheonlyH2receptorantagonistthatinhibitsoneofthehepaticcytochromeP
450enzymes,whichmakesitparticularlydangerouswhengivenwithwarfarin,aswarfarinis
metabolizedthroughthispathway.

KnowwhichdrugsupregulateanddownregulatethehepaticcytochromeP450enzymes.These
drugshavebeenknowntoshowuponmanyUSMLEforms:
Inhibitors

Inducers

Cimetidine

Barbiturates

Macrolides

Quinidine

Azoleantifungals

Rifampin

Isoniazid

Phenytoin

Sulfonamides

Griseofulvin

Grapefruitjuice

Carbamazepine

Proteaseinhibitors St.John'swort
Ciprofloxacin

Chronicalcoholuse

Step1Secret

10 Whatalternativepharmacologictreatmentstrategiesexistforthispatient
withpepticulcerdisease?
1.Protonpumpinhibitors,suchasomeprazole,lansoprazole,andpantoprazole
2.OtherH2receptorantagonists,suchasranitidine
3.Anticholinergics,suchasatropine
4.Mucosalprotectiveagents,suchasmisoprostol,andsucralfate
5.Antacids,suchascalciumcarbonate,magnesiumhydroxide,andaluminumhydroxide
Note:Magnesiumcausesdiarrhea,whereasaluminumcausesconstipation,sothesetwocompounds
areoftenmixedtogetherinantacidformulationstobalancetheseeffects.

Classically,painwithgastriculcersoccurswithmeals,whilepainwithduodenalulcers
occurs1to3hoursaftermeals,butthereissubstantialoverlap.
Twomajorcausesofpepticulcerdisease(PUD)areHelicobacterpyloriand
nonsteroidalantiinflammatorydrugs(NSAIDs).
MisoprostolwithNSAIDSorcyclooxygenase(COX)2inhibitorsdecrease
gastrointestinal(GI)sideeffects.

MelenausuallyindicatesanupperGIbleed.HematocheziaindicatesalowerGIbleedor
abriskupperGIbleed.
HemorrhageandperforationaremajorcomplicationsofPUD.
Alwaysbiopsygastriculcerstoruleoutgastriccarcinoma.
ThinkZollingerEllisonsyndromewithrefractoryPUD,multipleulcersorulcersin
unusuallocations,andamarkedlyelevatedgastrinlevel.
Useureabreathtest(moreaccurate)andIgMserumantibodytesttodiagnosecurrent
H.pyloriinfection.
CimetidineinhibitsP450(contraindicatedwithwarfarin)andcausesgynecomastia.
SummaryBox:PepticUlcerDisease

Case64
A42yearoldobesewomanpresentswithahistoryofepigastricpainthathasworsenedinthe
pastweekandisexacerbatedbyfood.Shehaslost20lboverthepast2monthsandhasfelta
littlefeverishinthepastweek.Shehasalsobeenbotheredbynauseaandvomiting.

1 Whatisthedifferentialdiagnosisforpostprandialepigastricpain?
Thedifferentialdiagnosisforpostprandialepigastricpaininthepresenceofnauseaandvomiting
includesgallstonedisease,GERD,PUD,pancreatitis,andacutegastritis.Gallstonedisease,suchas
biliarycolicandacutecholecystitis,istheleadingdiagnosisbecausethepatienthasseveralrisk
factorsobesity,middleaged,female,andrecentweightloss(whichmayhavebeenlostdueto
fasting,whichisanotherriskfactor)thatpredisposehertogallstoneformation.Also,pain
associatedwithbiliarycolicisexacerbatedbyeating(classicallyassociatedwithfattyfoods).GERD,
PUD(particularlygastriculcers),andacutegastritisarealsoalltypicallyexacerbatedbyfood.

Case64continued:
Uponfurtherquestioning,thepatientattributestheweightlosstonoteatingbecauseit
exacerbatesthepain.Shealsoadmitstodrinkingsixalcoholicdrinksadayandtaking
ibuprofenregularlyforlowerbackpain.

2 Howdoesthisinformationalterthedifferentialdiagnosis?
Now,gastriculcers,acutegastritis,andacutepancreatitisarethemostlikelydiagnoses.Gastric
ulcersarerelatedtoNSAIDuseandareassociatedwithweightloss,butalcoholdoesnotusuallyplay
aroleintheironset.NSAIDsandalcoholaremajorriskfactorsforacutegastritisandthepainfrom

foodusuallyleadstoanorexiaandweightloss.Acutepancreatitisduetoalcoholoragallstoneis
anotherconcern.

Case64continued:
Physicalexaminationrevealsmildepigastrictenderness.Serumamylaseandlipasearewithin
normallimits.Aureasebreathtestisnegative.Endoscopyispositiveforpunctateerosionsin
theantrum(Fig65(f0030)),andatissuebiopsyrevealsdiffuseinflammationofthegastric
mucosawithnoevidenceofmalignancy.

Figure65
Acutegastritis.A,Grossviewshowingpunctateerosionsinanotherwiseunremarkablemucosa
adherentbloodisdarkbecauseofexposuretogastricacid.B,Lowpowermicroscopicviewoffocal
mucosaldisruptionwithhemorrhagetheadjacentmucosaisnormal.
(FromKumarV,AbbasAK,FaustoN:RobbinsandCotranPathologicBasisofDisease,7thed.
Philadelphia,WBSaunders,2005.)

3 Basedonthesefindings,thediagnosisofacutegastritisisconfirmed.Whatare
thetwoprimaryclassificationsofthisdisease?
Acutegastritiscanbedividedintoinfectiousandnoninfectiousgastritis.Noninfectiousgastritis(as
inthiscase)iscausedbyexposuretotoxinsanddrugs(ethanol,NSAIDs)andseverephysicalstress
(burns,headtrauma,surgery),andinfectiousgastritisistypicallycausedbyH.pylori.

Case64continued:
YoucounselthepatientaboutcuttingbackonalcoholandstoppingtheNSAIDs,whichthe
patientdoes,andhersymptomsresolvewithinweeks.Yearslater,thepatientbringsher68
yearoldmothertoyoubecauseshethinksyouarethegreatestdoctorintheworldafter

solvingherbellyproblem.Thepatient'smotherhasbeenexperiencingfatigue,memory
difficulties,andnumbnessandtinglinginherfeetforthepast6months.

4 Whatisthedifferentialdiagnosisforthemother'ssymptoms?
Thedifferentialdiagnosisisbroadbutincludesdepression,dementia,vitaminB12(cobalamin)
deficiency,diabeticperipheralneuropathy,andalcoholicperipheralneuropathy.Thedepression
couldexplainthememorydifficulties,especiallyintheelderly(pseudodementia),andthefatigue.
Dementiaisconsistentwithmemorydifficulties,whichshouldbeaconsiderationinanyelderly
patientwithmemorycomplaints.VitaminB12deficiencycancausememorydifficultiesand
paresthesias,asthispatientdescribes,andtheanemiaassociatedwithvitaminB12deficiencycan
explainthefatigue.Diabeticandalcoholicperipheralneuropathiescancauseparesthesiasinthe
extremities.

Case64continued:
Physicalexaminationissignificantforepigastrictendernessandimpairedvibratorysenseand
proprioceptioninthelowerextremities.Herhemoglobinis9.5g/dLandmeancorpuscular
volume(MCV)is105fL.AperipheralbloodsmearisshowninFigure66(f0035).

Figure66
PeripheralbloodsmearforpatientinCase64.(st0250)
(FromHoffmanR,BenzEJJr,ShattilSJ,etal:Hematology:BasicPrinciplesandPractice,4thed.
Philadelphia,ChurchillLivingstone,2005.)

5 Whatmostlikelyexplainsthemacrocyticanemiaandperipheralbloodsmear
asshowninFigure66(f0035)?
ThiswomanhasamegaloblasticanemiasecondarytovitaminB12orfolatedeficiency.The
peripheralbloodsmearshowshypersegmented(generally>5lobes)neutrophils,whichconfirmsthe
diagnosisofmegaloblasticanemia.VitaminB12isrequiredforDNAsynthesisinrapidlyproliferating
erythrocyteprogenitorcells.AdeficiencyofvitaminB12thereforemayresultinamacrocyticanemia.
Givenherothersymptoms,vitaminB12deficiencyisalmostcertainlythediagnosis.

Figure66(f0035)isahighyieldimageforStep1.Immediatelyassociatehypersegmented

neutrophilswithfolateandvitaminB12deficiency.Ifneurologicchangesarementionedinthe
clinicalvignette,vitaminB12deficiencyislikely.
Step1Secret

Case64continued:
AserumvitaminB12(cobalamin)levelis120pg/mL(normal300pg/mL).

6 WhatarethevariouscausesofvitaminB12deficiency?
AvitaminB12deficiencyiscausedbyeitheranutritionaldeficiencyorvariousmalabsorption
syndromes.DietarysourcesofvitaminB12includeredmeat,fortifiedcereals,anddairyproducts.
ReservesofvitaminB12arelonglasting(27years)evenwithseveremalabsorption.Atriskgroups
forthistypeofdeficiencyareelderlywithteaandtoastdietsandchronicalcoholics(secondaryto
nutritionaldeficiency).MalabsorptionsyndromesthatcausevitaminB12deficiencyinclude
perniciousanemia,celiacdisease,bacterialovergrowth,Diphyllobothriumlatuminfection,Crohn's
disease,andpancreaticinsufficiency.Parietalcellsproduceintrinsicfactor(IF),whichisneededfor
properabsorptionofvitaminB12intheterminalileum.PerniciousanemiaortypeAchronicgastritis
iscausedbyautoimmunedestructionofparietalcellsinthefundusandbodyofthestomach,which
willdecreasetheamountofIF(andhydrochloricacid)produced.Throughvariousmechanisms,
celiacdisease,bacterialovergrowth,andCrohn'sdiseasemayaffectabsorptionattheterminalileum.
PancreaticinsufficiencyresultsinadeficiencyinpancreaticenzymesthatusuallyseparatevitaminB
12fromotherfactorsthatallowIFtobindtoit.

7 WhattestcanhelpwithdiagnosingthecauseofvitaminB12deficiency?
TheSchillingtestcanhelpdeterminethecauseofmalabsorptioninpatientswithvitaminB12
deficiency.ThefirststageoftheSchillingtestistosaturateallthebloodandtissuevitaminB12
bindingsiteswithanintramuscularinjectionofvitaminB12.RadiolabeledvitaminB12isthengiven
orally.IntheabsenceofvitaminB12malabsorption,thevitaminB12/IFcomplexwouldnormallybe
absorbedintheterminalileumandexcretedintheurinebecausealltissueandbloodvitaminB12
bindingsitesaresaturated.Ifthelevelofurineradioactivityislow(suggestingthatmalabsorptionis
indeedpresent),thenthesecondstageofthetestisrepeatedwithoralvitaminB12plusintrinsic
factor(IF),whichspecificallydetermineswhetherthepatienthasperniciousanemia(recallthatsince
theparietalcellsproduceintrinsicfactor,supplementationwithIFshouldincreaseB12absorptionin

patientswithperniciousanemia).Iftheurineradioactivityisstilllowafterthesecondstage,thena
thirdstageusesoralvitaminB12pluseitherantibioticsorpancreaticenzymestotestforbacterial
overgrowthorpancreaticinsufficiency,respectively.
Note:EventhoughtheSchillingtestisnowrarelyused,youarestillexpectedtounderstandthe
principlesbehinditforboards.

Case64continued:
Thepatient'sSchillingtestshowsnormalvitaminB12absorptionwiththeadditionofIF,and
thediagnosisofperniciousanemiaisthereforemade.

8 Whatarethetwoprimaryclassificationsofchronicgastritis?
Chronicgastritisisalsosubdividedintotwotypes:typeA(noninfectious)andtypeB(infectious).
TypeAchronicgastritisiscausedbyautoimmunedestructionofparietalcellsinthefundusandbody
ofthestomach.TypeBchronicgastritisisassociatedwithH.pyloricolonizationofthegastric
antrum.TypeBchronicgastritisismorecommonthantypeA,accountingforapproximately80%of
thecasesofchronicgastritis.TypeAcausesperniciousanemia(asdescribedpreviously)and
achlorhydia(deficiencyofhydrochloricacid),whichcanresultinGcellhyperplasiafromelevated
gastrinlevelsandpredisposetoentericinfections,especiallysalmonella.Bothtypesofchronic
gastritisareriskfactorsforthedevelopmentofgastriccarcinoma.

Possiblecausesofacutegastritis:(1)Helicobacterpylori,(2)nonsteroidalanti
inflammatorydrugs(NSAIDs)oralcohol,and(3)stressinduced.
Twotypesofchronicgastritis:typeA(autoimmune)andtypeB(H.pylori).
Autoimmunegastritisaffectsparietalcells,inhibitingproductionofintrinsicfactor
(perniciousanemia)andhydrochloricacid(achlorhydria).
Polysegmentedneutrophils=megaloblasticanemia=vitaminB12orfolatedeficiency
untilprovenotherwise.
TheSchillingtestcanhelpdistinguishbetweenthevariousmalabsorptionsyndromes
causingvitaminB12deficiency.
SummaryBox:Acute/ChronicGastritis

Case65
A38yearoldalcoholicmanpresentstotheemergencyroomwithsuddenonset,severe
epigastricpainthatradiatestohisback.Healsocomplainsofnauseaandvomiting.

1 Whatisthedifferentialdiagnosisforabdominalpainradiatingtotheback?
Thedifferentialdiagnosisincludesacutepancreatitis,rupturedabdominalaorticaneurysm,
perforatedduodenalulcer,biliarycolic,andrenalcolic.Inamiddleagedmanwithheavyalcohol
use,acutepancreatitisseemslikely.

Case65continued:
Physicalexaminationissignificantforfever(101F),tachycardia,bloodpressure(BP)of98/60
mmHg,epigastrictenderness,andabsentbowelsounds.Laboratorytestsrevealelevated
serumamylaseandlipase,hypocalcemia,andaleukocytosis.

2 Whatisthediagnosis?Whatarethecausesofthisdisease?
Acutepancreatitischaracteristicallyhasepigastricpainradiatingtotheback,elevatedamylaseand
lipase,leukocytosis,andalowgradefever.Althoughuncommon,youshouldalsoknowthatacute
pancreatitismayalsobeassociatedwithhypocalcemia(duetofattyacidsaponificationofcalcium
salts).Themostcommoncausesarealcoholabuseandgallstones.Otherlesscommonbutwell
establishedcausesofacutepancreatitisincludeseverehypertriglyceridemia(typically>2000
mg/dL),markedhypercalcemia,trauma,medications,iatrogeniccauses(e.g.,followingendoscopic
retrogradecholangiopancreatography[ERCP]),annularpancreas,andscorpionbite.

3 Howcangallstonescausepancreatitis?
Gallstonescanpassintothelowercommonbileductandobstructtheegressofbileintotheintestine.
Thebilecanthenbackupintothepancreaticduct,irritatingandinflamingthepancreatictissue.The
pancreaticductalsocannotempty,sopancreaticsecretionsmaybuildupandcontributetothe
inflammatoryprocess.
Note:Patientswithcysticfibrosisoftendevelopchronicpancreatitisbecausethickpancreatic
secretionscanblockthepancreaticduct.

Case65continued:
Thepatientspends4daysinthehospitalandisdischargedhomefeelingwell.However,he
continuestodrinkalcohol,andseveralyearslater,heisevaluatedfora2monthhistoryof
abdominalpain,diarrhea,andweightloss.AnabdominalxraystudyisshowninFigure67
(f0040).

Figure67
Calcificationinpancreas.
(FromNobleJ:TextbookofPrimaryCareMedicine,3rded.St.Louis,Mosby,2001.)

4 Whatisthelikelydiagnosisinthiscaseandwhatisthemostcommoncauseof
thisdisease?
Thecombinationofchronicalcoholuse,abdominalpain,diarrhea,weightloss,andpancreatic
calcificationsonanabdominalxrayfilmisclassicforchronicpancreatitis.Alcoholisthemost
commoncauseofchronicpancreatitis(gallstonesarenotamajorcause).

5 Whataretheprimarycomplicationsofchronicpancreatitis?
Thesequelaeofchronicpancreatitisincludefatmalabsorption,fatsolublevitamin
deficiency(vitaminsA,D,E,K),persistentdiarrhea,andinsulindependentdiabetesmellitus.Alack
ofvariouspancreaticenzymesthathelpinthedigestionoffatinthesmallintestineresultsinfat
malabsorptionandfatsolublevitamin(A,D,E,andK)deficiency.Theseandotherpoorlyabsorbed
nutrientsarethenavailabletobecatabolized/fermentedbythebacterialflorainthelargeintestine.
Thefinalproductsofthiscatabolismaretypicallyosmoticallyactive,anddrawwaterintothelumen
oftheintestine,leadingtoanosmoticdiarrhea.Diabetesmellitusresultsfromchronicinflammation,
eventuallydestroyingthebetacellsoftheisletsofLangerhans.

Acutepancreatitisisclassicallycharacterizedbyepigastricpainradiatingtotheback,
elevatedamylaseandlipaselevels,leukocytosis,andlowgradefever.

Themostcommoncauseofchronicpancreatitisisheavyconsumptionofalcoholand
gallstones.
Chronicpancreatitiscanleadtopersistentdiarrhea,diabetesmellitus,fat
malabsorption,andvitaminA,D,E,andKdeficiencies.
SummaryBox:Acute/ChronicPancreatitis

Case66
A2weekoldboyisbroughttotheemergencydepartmentwithnonbilious,projectilevomiting
thatbeganearlierintheday.

1 Whatisthedifferentialdiagnosisforprojectilevomitinginanewborn?
Thedifferentialdiagnosisincludesinfantilehypertrophicpyloricstenosis,tracheoesophagealfistula,
esophagealatresia,duodenalatresia,annularpancreas,andgastroenteritis.Themostlikelydiagnosis
inthiscaseisinfantilehypertrophicpyloricstenosis,whichusuallypresentswithnonbilious,
projectilevomitingaround3to4weeksoflife.Tracheoesophagealfistulaisanabnormal
communicationbetweenthetracheaandesophagusthatusuallypresentswithcoughingandcyanosis
duringthefirstfeeding.Thefistulastypicallyoccuratthemidleveloftheesophagus,whichiswhere
thelungsbudofffromtheforegutduringembryologicdevelopment.Esophagealatresiaisanother
embryologicesophagealdisorderinwhichtheesophagusendsinablindpouch,resultinginfood
accumulationandrefluxintotheairway,alsocausingcoughingandcyanosisduringthefirst
feedings.Duodenalatresiatypicallypresentswithvomitingwithinthefirstdayoflife,andthe
vomitustendstobebiliousiftheatresiaoccursbelowwherethecommonbileductentersthesecond
partoftheduodenum.Finally,annularpancreasiscausedbytheventralanddorsalpancreaticbud's
beingabnormallyfusedaroundthesecondpartoftheduodenum,whichcanresultinduodenal
obstructionandprojectilevomitingwithinthefirstfewdaysoflife.

2 Whatacidbaseandelectrolytedisordercanbecausedbyprolongedvomiting
inthisbabyandhowdoesitdevelop?
Hypochloremicmetabolicalkalosis.Becausegastricparietalcellssecretehydrochloricacidintothe
lumenofthestomach,prolongedvomitingcandepletethebodyofbothhydrogenandchlorideions.
Thealkalosisthatdevelopsiscausedbythelossofhydrochloricacidandthesimultaneousretention
ofthebicarbonatethatisgeneratedwhentheparietalcellsmakehydrochloricacid.

Case66continued:
Physicalexaminationrevealsafirm,palpableolivelikemassintheepigastricregion,and
ultrasoundrevealsathickenedandelongatedpylorusmuscle(Fig68(f0045)).

Figure68
A,Transversesonogramdemonstratingapyloricmusclewallthicknessofgreaterthan4mm(
distancebetweencrosses).B,Horizontalimagedemonstratingapyloricchannellengthgreaterthan
14mm(wallthicknessoutlinedbetweencrosses).
(FromKliegmanRM,BehrmanRE,JensonHB,etal:NelsonTextbookofPediatrics,18thed.
Philadelphia,WBSaunders,2007.)

3 Whatisthemostlikelydiagnosis?
Projectilevomitingina2weekoldinfantandthefindingsonphysicalexaminationandultrasound
pointtothediagnosisofinfantilehypertrophicpyloricstenosis.
Palpableolivelikemassintheepigastricregionisapopularbuzzwordforpyloricstenosis.
Themassistheresultofmuscularhypertrophyofthepyloricsphincter.
Step1Secret

Case66continued:
Theinfantundergoessurgicalpyloromyotomy.Oneweeklater,themotherreturnstothe
physicianbecausetheinfantisexperiencingseverediarrheaafterbeingbreastfed.

4 Whatisthecauseofthediarrhea?
Dumpingsyndromeiscausedbythedeliveryofexcessiveamountsofhyperosmoticchymefromthe
stomachtothesmallintestine,whichmayoccurwithadysfunctionalpyloricsphincter.Theintestine
isunabletoprocesssuchalargequantityofchyme,resultinginanosmoticdiarrhea,whichmay
causedizziness,weakness,andtachycardiafollowingmeals.
Note:Dumpingsyndromemayalsoresultfollowinggastricbypasssurgery.Becausethemealis
deliveredtothesmallintestinemorequicklythanusual,theincreasedtonicityofthesmallintestine
causesalargefluidshiftintothegutlumen.Thisincreasesthemotilityofthesmallintestineand
resultsindiarrhea.Inseverecases,theluminalshiftoffluidstimulatesbloodflowtotheintestine,
whichdecreasestotalbloodvolume.Hypotensionandreflextachycardiacanresult.

Infantilehypertrophicpyloricstenosistypicallypresentsin2to3weekoldinfantswith
nonbilious,projectilevomiting.
Dumpingsyndromecanresultfromadysfunctionalpyloricsphinctercaused,for
example,bysurgicalpyloromyotomy.
Prolongedvomitingmayresultinahypochloremicmetabolicacidosis.
Tracheoesophagealfistulapresentswithcoughingandcyanosiswiththefirstfeeding.
Duodenalatresiapresentsinthefirstdayoflife,demonstratesadoublebubblesign
onradiographs,andcancausebiliousvomiting.ItisassociatedwithDownsyndrome.
SummaryBox:HypertrophicPyloricStenosis

Case67
A33yearoldwomancomplainsofalonghistoryofdiarrhea,flatus,andabdominalpain.More
recently,shehasexperiencedunintentionalweightlossdespiteapreservedappetite.Shehas
neverlefttheUnitedStates.

1 Whatisthedifferentialdiagnosis?
Thedifferentialdiagnosisincludesinflammatoryboweldisease(IBD),celiacdisease,giardiasis,IBS,
Whipple'sdisease,tropicalsprue,disaccharidase(lactase)deficiency,andabetalipoproteinemia.
Fromthehistory,IBD,giardiasis,andceliacdiseasearethemostlikelypossibilitiesbecauseofthe
chronicdiarrhea,weightloss,andabdominalpain.WeightlossdoesnotusuallyoccurinIBSand
lactoseintolerance.Whipple'sdiseaseisunlikelybecauseittypicallyoccursinmenovertheageof
40.TropicalsprueisunlikelybecausethepatienthasnottraveledoutsideoftheUnitedStates.
Abetalipoproteinemianormallypresentswithinthefirstfewmonthsoflifetherefore,thiscondition
isunlikelyinthispatient.

Case67continued:
Physicalexaminationisunremarkable.Stoolexaminationforovaandparasitesisnegative.A
completebloodcount(CBC)revealsmicrocyticanemia.Apathologyreportfromasmall
intestinalbiopsy(Fig69A(f0050))describesanintestinalmucosasignificantforvillous
atrophy,lymphocyticinfiltrationofthelaminapropria,andhyperplasticcrypts.Thewomanis
toldshehasamalabsorptionsyndromeandisputonaspecialdietdevoidofwheat,barley,and
rice.Weekslater,arepeatbiopsyshowscompleteresolutionofmucosaldamagetothesmall
intestines(Fig.69B(f0050)),andsheisencouragedtostayonherdiet.

Figure69
A,DuodenalbiopsyspecimenfrompatientinCase67.(st0400)B,Repeatduodenalbiopsyfrom
patientinCase67(st0400)afterimplementationofstrictglutenfreediet.
(FromFeldmanM,FriedmanLS,BrandtLJ:SleisengerandFordtran'sGastrointestinalandLiver
Disease,8thed.Philadelphia,WBSaunders,2006.)

2 Whatisthelikelydiagnosisinthiscase?
Thediagnosisofceliacdisease(celiacsprueornontropicalsprue)isestablishedbyresolutionof
mucosaldamagefollowingaglutenfreediet.Celiacdiseaseiscausedbyhypersensitivitytothe
gliadiningluten,whichispresentinwheat,barley,rice,andamultitudeofprocessedfoods.Through
anunknownmolecularmechanismtheexposuretogliadininahypersensitivepersoncausesdamage
totheintestinalmucosa.

3 Howisceliacdiseaseassociatedwithmicrocyticanemiainthispatient?
Celiacdiseasetypicallyaffectstheproximalsmallbowel(duodenumandjejunum),whereironand
folateareabsorbed.Thiscanresultinirondeficiency(microcytic)anemiaandafolatedeficiency
(macrocytic)anemia.Inthiscase,theironmalabsorptionisgreaterthanthefolate,asistypically
seen.VitaminB12malabsorption,whichcanalsocausemacrocyticanemiaandneurologic
symptoms,canoccurwithmoresevereceliacdiseasethataffectstheterminalileum(althoughthisis
rare).

4 Howdothesigns,symptoms,andintestinalbiopsyfindingsinWhipple's
diseasedifferfromceliacdisease?
Whipple'sdiseaseisduetoinfectionwithTorphyremawhipelli.Ittypicallyoccursinmenoverthe
ageof40.Thesignsandsymptomsarefairlysimilar,butintestinalbiopsyshowslipidvacuolation
withinfiltrationofPAS(periodicacidSchiff)positivemacrophageswithsmallbacilliinWhipple's
disease.

5 Howdoesceliacdiseasedifferfromtropicalsprue?
Boththesediseaseshavethesamesymptoms,buttropicalspruedoesnotrespondtoaglutenfree
diet.Intestinalbiopsyfindingsaresimilarinbothdiseases,althoughtropicalsprueaffectstheentire
smallintestine,andceliacdiseasemostlyconcentratesintheproximalsmallbowel.Inaddition,

tropicalsprueismostcommonlyfound,asitsnameimplies,inthetropics(e.g.,SoutheastAsia,
CentralandSouthAmerica,Caribbean).Althoughaninfectiousorganismissuspected,theprecise
etiologyoftropicalsprueremainsunknown.Broadspectrumantibioticsremainthetreatmentof
choicefortropicalsprue.

Celiacdiseasehasapseudoautoimmunecause,buttropicalsprueisinfectiousinorigin.
Celiacdiseaseimproveswithaglutenfreediet,whereastropicalspruedoesnot.
Celiacdiseaseoftencausesanirondeficiencyanemiaalthoughrarelycancausefolate
orB12deficientanemia.
Whipple'sdiseasehasasimilarpresentationtoceliacdisease,althoughittypically
occursinmenolderthan40,andintestinalbiopsyshowscharacteristicintestinalmucosa
infiltratedwithPAS(periodicacidSchiff)positivemacrophages.
SummaryBox:CeliacDisease/Diarrhea

Case68
A32yearoldwoman,ofAshkenaziJewishdescent,complainsofalonghistoryofabdominal
painanddiarrhea.Onatypicalday,sheusuallypasses15to20loosestools.Inthelastfew
weeks,shehaslost15lbandhasexperiencedseveralepisodesofbloodydiarrhea.Shedenies
anypainwithbowelmovements(tenesmus)butdoescomplainofabdominalpainfollowing
meals.

1 Whatisthedifferentialdiagnosis?
ThedifferentialdiagnosisincludesIBD(Crohn'sdiseaseandulcerativecolitis),infectiouscolitis(
Clostridiumdifficile,Shigella,Campylobacter,Escherichiacoli),andmesentericischemia.Crohn's
diseaseandulcerativecolitisarebothcharacterizedbyabdominalpain,frequentloosestools,bloody
diarrhea,andweightloss.ThediarrheatendstobenonbloodyinCrohn'sdiseaseifthecolonisnot
involved.Infectiouscolitisislikelyifthepatienthassignificanttravelhistory(Shigella,
Campylobacter,E.coli)ortakenantibioticsrecently(C.difficile).Mesentericischemiaismore
commoninadults>50yearsoldwithatheroscleroticdisease.Painisacuteandusuallyoccurs
followingameal.

Case68continued:
Physicalexaminationissignificantformildfeveraswellasabdominaltendernessintheright
lowerquadrant(RLQ).Laboratorystudiesshowanelevatederythrocytesedimentationrate
(ESR)aswellasdecreasedplasmalevelsofvitaminB12,vitaminD,andvitaminK.Lower

endoscopyrevealsacobblestoneappearanceandthepresenceoflesionsintheterminalileum
andproximalcolon.Biopsyoftheterminalileumrevealsgranulomasandtransmuralchronic
inflammation.

2 Whatisthediagnosis?
ThediagnosisofCrohn'sdiseasecanbemadebasedonthepatient'shistoryofbloodydiarrheaand
presenceofclassicpathologicfeaturesofthedisease,suchascobblestoneappearanceofthebowel,
absenceofcontinuouslesionsalongthebowel(socalledskiplesions),granulomas,andtransmural
inflammation.

3 CompareandcontrastthecharacteristicsofCrohn'sdiseaseandulcerative
colitis.Trytestingyourselfbycoveringtheentriesintheappropriatecolumns
ofTable62(t0015)foreachrespectivecharacteristic
LearntheinformationlistedinTable62(t0015)thoroughly.TheUSMLEcommonlyasks
studentstodifferentiatebetweenCrohn'sdiseaseandulcerativecolitis.
Step1Secret
Table62
CharacteristicsofCrohn'sDiseaseandUlcerativeColitis

Feature

Crohn'sDisease

UlcerativeColitis

Associations

PrevalenceinAshkenaziJews

Patientwhorecentlyquitsmoking

(thoughmostcasesoccurinnon
Jews)
Locationin

Anywhere(mostlyterminalileum),

Limitedtocolon,contiguousinflammation

gastrointestinal

sparesrectum,skiplesions

fromtherectum

Wallthickness

Transmuralinflammation,

Mucosalinflammationonly

involved

cobblestoneappearanceof

tract

mucosa
Complications

Dermatologic

Strictures,fistulas,abscess,

Toxicmegacolon,hemorrhage,primary

malabsorption,riskofcolon

sclerosingcholangitis,riskofcolon

cancer

cancer

Erythemanodosum

Pyodermagangrenosum

Arthritis,aphthousulcers,uveitis,

Uveitis

manifestations
Othernon

intestinal

perianalfistulas,cholelithiasis

manifestations

4 Inwhatspecialsituationdoestransmuralinflammationofthecolonoccurin
ulcerativecolitis?
Transmuralinflammationofthecolonoccursintoxicmegacolon,whichisamedicalemergency.
Surgery(usuallyacolectomywithileoanalpullthrough)isrequiredtopreventperitonitisandsepsis
andrestoresomesemblanceofnormalbowelactivity.

5 Whatcongenitaldisorderresultsinconstipationandaseverelydilatedcolon
(similartotoxicmegacolon)?Whatisthisdiseasecausedby?
Hirschsprung'sdisease,alsoknownascongenitalmegacolonoraganglionicmegacolon,iscausedby
neuralcrestcellsthatformthemyentericplexusfailingtomigratetothecolon.Therectumisalways
involvedbecauseneuralcrestcellsmigratecaudallyalongtheintestine.Rectalexaminationyieldsan
absenceofstoolonthefinger(canalsobeafindingwithcysticfibrosis).Hirschsprung'sdiseaseis
alsoassociatedwithDownsyndrome.Treatmentofthisdiseaseinvolvessurgicalresectionofthe
aganglionicsegment.

6 HowcanCrohn'sdiseasecausedeficienciesofthefatsolublevitamins?
Crohn'sdiseasecommonlyinvolvestheterminalileum,wherevitaminB12isabsorbed.Theterminal
ileumisalsowherebilesaltsarereabsorbed,andbecausemostofthebilesaltssecretedintothe
intestinearefromtheenterohepaticcirculation,adeficiencyofbilesaltsecretiondevelops.Thiscan
impairtheabsorptionoffatsandfatsolublevitamins(A,B12,D,E,andK).Surgicalresectionofthe
ileumcanalsocausethesevitamindeficiencies.

7 Whymightcholestyraminehelpwiththispatient'sdiarrhea?
Cholestyramineisabileacidsequestrantthatbindsbilesalts,whicharepoorlyabsorbedinCrohn's
diseaseandmightotherwisecausecolonicirritationsanddiarrheauponenteringthecolon.

Case68continued:
ThepatientisstartedonsulfasalazinefortreatmentofheractiveCrohn'sdisease.

8 Whatpharmacokineticpropertiesofsulfasalazinemakeitparticularlysuited
totreatinginflammatoryboweldisease?
Sulfasalazineisaprecursoroftheactivecompound5aminosalicylicacid,anonsteroidalanti
inflammatoryagentthatcanreduceinflammationinthebowel.However,if5aminosalicylicacid
itselfisgivenorallyinsufficientquantitiestoreduceinflammationinthelargebowel,significant

gastricirritationwilldevelop.Sulfasalazineavoidsthisproblembecauseitisnotbrokendowninto5
aminosalicylicaciduntilitreachesthedistalileumandcolon.Sulfasalazineisalsopoorlyabsorbed
fromtheGItract,thusincreasingtheconcentrationofactivedrugthatreachesthelargebowel.

9 Whatextraintestinalcomplicationofulcerativecolitisshouldbesuspectedina
patientwhopresentswithsignsofobstructivejaundice?
Primarysclerosingcholangitis,whichiscausedbyfibrosisofthelargebileducts,isarare
complicationassociatedwithbothulcerativecolitisandCrohn'sdisease.Forboards,associate
primarysclerosingcholangitiswithulcerativecolitis.

Case68continued:
Severalyearslater,thepatientisseenbyherphysicianforprogressivelyworseninglowerback
pain.XrayfilmsofthepelvisandlumbarspineareshowninFigure610(f0055).

Figure610
Ankylosingspondylitis.A,Alateralviewofthelumbarspinedemonstratescalcificbridgingacrossthe
diskspaces(arrows),causingthetypicalbamboospineappearance.B,Anteroposteriorviewofthe
pelvisshowsthattheregionofthesacroiliacjoints(arrows)isnoteasilyvisualizedowingtofusionof
bothsacroiliacjoints.
(FromMettlerFA:EssentialsofRadiology,2nded.Philadelphia,WBSaunders,2005.)

10 Whatextraintestinalcomplicationofinflammatoryboweldiseaseshouldbe
suspected?
Ankylosingspondylitis,withthecharacteristicbamboospineappearanceandbilateralsacroilitis,is
morecommonlyassociatedwithCrohn'sdiseasethanwithulcerativecolitis.

11 Willacompletecolectomyalleviatetheextraintestinalcomplicationsof
inflammatoryboweldisease?

No.Theextraintestinalmanifestations(i.e.,arthritis,sclerosingcholangitis)oftenpersist.

Abdominalpain,bloody(ormucous)diarrhea,andunintentionalweightlossare
characteristicofinflammatoryboweldisease(IBD)(Crohn'sdiseaseandulcerative
colitis).
Crohn'sdiseasehasahigherprevalenceamongAshkenaziJews.
Ulcerativecolitismightoccurafterapersonstopssmoking.
Crohn'sdiseasemostcommonlyinvolvestheterminalileumbutcanaffectanypartof
thegastrointestinaltract.Itisassociatedwithskiplesions,transmuralinflammation,
andcobblestoneappearance.
Ulcerativecolitiscausesacontinuouslesionthatoccursonlyinthecolonandaffectsthe
mucosaandsubmucosa(isnottransmural).Itisassociatedwithdiffuseascending
inflammationfromtherectumandpseudopolypformation.
Generally,sulfasalazineisusedforinitialtreatmentforIBD.
Bilesaltscancausediarrheaintwoways:(1)secondarytotheirsecretoryeffectonthe
colonor(2)malabsorptioncausedbytheirdepletionfromtheenterohepaticcirculation.
SummaryBox:InflammatoryBowelDisease

Case69
A13yearoldgirlwithahistoryofovariancystsisbroughttotheemergencydepartment
becauseofsevereabdominalpain.Shewasawakenedfromsleepseveralhoursearlierwithpain
thatshenowcomplainsisintherightlowerabdomen.Shealsocomplainsofnauseaandlossof
appetite.

1 Whatisthedifferentialdiagnosis?
Thedifferentialdiagnosisincludesacuteappendicitis,ovariantorsion,rupturedovariancyst,ectopic
pregnancy,pelvicinflammatorydisease(PID),Yersiniaenterocolitis,acuteonsetofIBD,Meckel's
diverticulitis,andrightsideddiverticulitis.Inthisgirl,withahistoryofovariancysts,ruptured
ovariancystsorovariantorsionareonthetopofthedifferentiallist.Acuteappendicitisshouldalso
beconsideredinanypersonwithRLQabdominalpain.Ectopicpregnancywouldalsowarrant
investigationwithapregnancytest.Rightsideddiverticulitiswouldbeunlikelyinagirlthisyoung.

Case69continued:

Examinationissignificantforatemperatureof100.2F,andthepatient'sabdomenistenderat
McBurney'spoint.LaboratoryworkuprevealsamildleukocytosisandanegativehCG
(humanchorionicgonadotropin).Apelvicultrasoundrevealsasmallovariancystontheleft
ovarybutadequatebloodflowtobothovaries.Acomputedtomography(CT)scanrevealsa
thickenedandinflamedappendix(Fig.611(f0060)).

Figure611
Computedtomographicscanshowingacuteappendicitis.Theappendixisedematous,andthereis
surroundingstrandingofthemesentericfat(arrow).
(FromGoldmanL,AusielloD:CecilTextbookofMedicine,22nded.Philadelphia,WBSaunders,2004.)

2 Whatisthediagnosis?
Althoughthepatient'sinitialpresentationmaybesuggestiveofovariantorsionorarupturedovarian
cyst,thephysicalexaminationfindingsandtheCTscanconfirmedthediagnosisofacute
appendicitis.

3 Whatisthemostcommoncauseofappendicitis?
Appendicitisismostoftencausedbyobstructionofthelumenoftheappendix,mostcommonlybya
fecalith,butobstructioncanalsobeduetolymphoidhyperplasia,tumors,oranintestinalstricture.
Obstructionoftheappendixlumenleadstobacterialovergrowthandacuteinflammation.
Polymorphonuclearcells(PMNs)areseeninthewalloftheappendix.

4 NotethatMeckel'sdiverticulitiscanpresentsimilarlytoappendicitis.Whatis
itspathophysiology?
MostMeckel'sdiverticulaareanasymptomatic,embryologicremnantthatconnectedthelumenof
thedevelopingguttotheyolksacinthedevelopingembryo.About50%ofthesediverticulaarelined
withheterotopicgastricorpancreatictissue.Thegastricmucosacansecreteacid,eventuallycreating

adjacentintestinalulcerationsthatcanbleedandcausepainthatmimicsacuteappendicitis.
Alternatively,diverticulacanleadtointussusception,incarceration,orperforation.
Note:Recallthelawofthethree2spertainingtoMeckel'sdiverticulum:itaffectsabout2%ofthe
population,isabout2incheslong,andislocatedabout2feetfromtheileocecalvalve.

Case69continued:
Thepatientistakentotheoperatingroomforalaparoscopicappendectomyandreturnshome
thesameday.

5 Whattypeofcanceroftheappendixisoccasionallyseenasanincidental
findingduringanappendectomy?Whatsubstancedothesetumorssecrete
andwhatsyndromecanitcause?
Carcinoidtumorssecretelargequantitiesofserotonin,resultinginelevatedlevelsof5
hydroxyindoleaceticacid(5HIAA),whichcanbeeasilydetected.Intestinalcarcinoidtumorsthat
metastasizetothelivercancauseacarcinoidsyndrome,whichisaconstellationofsymptoms
includingepisodicflushing,diarrhea,wheezing,andrightsidedheartvalvelesions(serotoninis
degradedbymonoamineoxidase[MAO]inthelungs,soitisunabletoreachtheleftsideofthe
heart).Hepaticmetastasesarenecessaryforthissyndrometooccurwithintestinalcarcinoidtumors
becausetheliverwouldotherwisemetabolizeserotoninandotherbiogenicaminesastheypass
throughtheliverfromtheintestines.
Note:Theappendixisthemostcommonsiteofgutcarcinoidtumor.
Carcinoidsyndromeisafavoriteonboards.Itreferstoatumorofneuroendocrinecellsandis
associatedwiththesymptomsofwheezing,diarrhea,flushing,andrightsidedheartmurmurs
thatincreaseoninspiration.
Step1Secret

6 Althoughnotdescribedintheprecedingcase,painfromappendicitis
classicallybeginsaroundtheumbilicusandmigratestotherightlower
quadrant.Whatistheneuroanatomicbasisforthispattern?
Theinitialpainfromappendicitisisduetoactivationofvisceralpainreceptorsintheinflamed
appendixandvisceralperitoneum.Thesensorynervesthatcarrythisinformationsynapseonspinal
neuronsthatalsoreceivesensorysignalsfromtheanteriorabdominalwallintheperiumbilicalarea.
Becausetheoriginofthesignalcannotbediscerned,thebrainmisinterpretsthevisceralpainasa
poorlylocalizedpainarisingfromtheperiumbilicalarea(T10dermatome).Later,whentheparietal
peritoneumadjacenttotheappendixbecomesinflamed,thepainbecomessharperandismore

accuratelylocalizedtotheRLQbysomaticpainfibers.Thisexactpositionisreferredtoas
McBurney'spoint,whichislocatedtwothirdsofthedistancebetweentheumbilicusandtheiliac
crest.Youshouldknowthisreferencepointforboardsandforyourclinicalyears.

7 Whatistheprincipaldangerifappendicitisremainsuntreated?
Perforationcanoccur,causingperitonitis(acuteabdomen)andpossiblyabdominalabscess
formation.Perforationmaybedetectedbythepresenceoffreeaironanabdominalxraystudyoras
airunderthehemidiaphragmonachestxrayfilm.

Painfromappendicitisclassicallybeginsintheperiumbilicalregionandmigratestothe
rightlowerquadrant(RLQ).Patientsaretypicallyanorexicandmayhavenausea,
vomiting,andalowgradefever.
Rememberthatmanydifferentdiseasescanpresentsimilarlytoacuteappendicitis(see
previoustextdiscussionofdifferentialdiagnosis).
GastricmucosacanbepresentinMeckel'sdiverticulumasheterotopicgastrictissueand
causesymptomssimilartothoseofacuteappendicitis.
Knowthethree2sforMeckel'sdiverticulum:2%ofpopulation,2incheslong,and2
feetfromileocecalvalve.
Carcinoidsyndromecausesflushing,diarrhea,bronchospasm,andrightsidedheart
lesions.
SummaryBox:Appendicitis

Case610
A50yearoldmanvisitsthedoctorwithahistoryofseverecolickyabdominalpain,vomiting,
andconstipation.Pastmedicalhistoryisunremarkablewiththeexceptionofappendectomy6
yearsprior.

1 Whatisthedifferentialdiagnosisforthispatient'scondition?
Thispresentationisclassicforsmallbowelobstruction.Generalcausesofbowelobstructioninclude
surgicaladhesions,hernia,tumor,volvulus,intussusception,Crohn'sdisease,gallstoneileus,
stricture,congenitalmalformation,andenteritis.

2 Whatisthecauseofthispatient'scondition?
Giventhispatient'spastsurgicalhistoryofappendectomyandotherwiseunremarkablemedical
history,asurgicaladhesionisthemostlikelydiagnosis.Adhesionsarefibrousbandsthatformafter
injuryduringsurgery.Theyconnectorgansandtissuesthatareotherwisenotnormallyconnected.

Abdominaladhesionscanresultinsmallbowelobstructioniftheytugonorkinktheboweland
preventthepassageofbowelcontents.

3 Whatwouldbeexpectedonabdominalauscultationinthispatient?
Highpitched,tinklingbowelsoundsaregenerallyheardwithasmallbowelobstruction.
Youmaybegivenamultimediaquestionwithafindingoftinklingbowelsoundsonabdominal
examination.Associatethiswithsmallbowelobstructionandusethemedicalhistoryto
prioritizeyourdifferentialdiagnosis.
Step1Secret

Surgicaladhesionsareacommoncauseofsmallbowelobstruction.Adhesionsare
fibrousbandsthatformaftersurgicalinjuryandjointwoorgansortissuesthatarenot
normallyconnected.
Symptomsofsmallbowelobstructionincludecolickyabdominalpain,abdominal
distention,vomiting,constipation,andaninabilitytopassgaswithsevereobstruction.
Physicalexaminationofapatientwhopresentswithsmallbowelobstructionmayreveal
ahighpitchedtinklingsoundonauscultationoftheabdomen.
SummaryBox:SurgicalAdhesionsandSmallBowelObstruction

Copyright2015Elsevier,Inc.Allrightsreserved.

BOOKCHAPTER

GeneticandMetabolicDisease
J.PedroTeixeira,ThomasA.BrownMDandSonaliJ.Shah
USMLEStep1Secrets,Chapter11,329370

Understandingbiochemistryandgeneticswillbecrucialtoachievingagoodscoreonthe
USMLEStep1.Thesesubjectslaythefoundationformanyofthediseasesthatyouareexpected
toknowforboards.Thisisanintimidatingthoughtformanystudentswhothinkthattheywill
beexpectedtomemorizeabunchofpathways,enzymes,andintermediates.Yetthisisnotthe
case.Althoughyouarenotexpectedtomemorizeeverystepofeverybiochemicalpathway,you
shouldunderstandtheimplicationsofabnormalitiesinthesepathwaysandhowtheyresultin
variousdiseasesymptoms.Focusontheratelimitingstepsandkeyenzymesofthepathways
thatyoulearnaswellaswherethereactionstakeplace(e.g.,cytosol,mitochondrialmembrane,
mitochondrialmatrix,oracombinationoftheaforementionedlocations).Moreimportantly,
youshouldknowhowthesepathwaysareregulated.Onefinaltipofadvice:Donotignore
vitaminsandothermicronutrientswhenstudyingforboards.Thisisoneofthehighestyield
Step1topics!
Insider'sGuidetoGeneticandMetabolicDiseasefortheUsmleStep1

BasicConcepts
1 Whatisanenzymopathyandhowdoesitresultinclinicalsymptoms?
Anenzymopathyisageneticdiseaseinwhichadeficiencyinactivityofanenzymeleadstoablockin
ametabolicpathway.Thealtered(usuallyreduced)enzymaticactivitycanbeduetoreducedcellular
expressionoftheenzymeortoexpressionofadysfunctionalenzyme.Thepathologicmanifestations
oftheenzymedeficiencyarearesultoftheaccumulationofsubstrate(oritsderivatives)priortothe
blockage,alackoftheproduct(s),oracombinationofboth(Fig.111(f0010)).

Figure111
Mechanismsbywhichanenzymopathyproducesclinicalsymptoms.=increased,=decreased.
(FromBrownTA,BrownD:USMLEStep1Secrets.Philadelphia,Hanley&Belfus,2004.)

2 Whatisthetypicalpatternofinheritanceobservedinenzymopathies?
Almostallenzymesareproducedinexcessofminimalrequirements.Soalthoughheterozygous
carriersofanenzymedeficiencytypicallyhaveonly50%ofnormalenzymeactivitylevels,usually
theyarephenotypicallynormal.Thus,almostallenzymopathieshaveanautosomalrecessivepattern
ofinheritance,inwhichaphenotypicabnormalitymanifestsonlywhenthereisnearlynoenzyme
activity.Thisgeneralizationisextremelyusefulfortheboards.

3 ExplainwhythepathologicconsequencesofXlinkedenzymopathiesare
manifestedalmostexclusivelyinmales
Again,enzymedeficienciesgenerallyrequireaneartotallossofenzymeactivitytoresultin
phenotypicabnormalities.AsmaleshaveonlyasingleXchromosome,inheritanceofasingle
defectivecopyofanXlinkedgenefromthemotherwillresultinthepathologicconsequencesofthe
enzymedeficiency/abnormality.BecausefemaleshavetwoXchromosomes,theywillgenerally
exhibitthediseaseonlyiftheyarehomozygousforthemutatedalleles,whichisfarlesslikely.(For
example,iftheoddsofamaleinheritingasingledefectivegeneis1/p,theoddsofafemale
inheritingtwodefectivecopieswillbeapproximately1/p2.)
ImportantexamplesofXlinkedrecessiveenzymopathiesincludehemophiliaA(factorVIII
deficiency),hemophiliaB(factorIXdeficiency),glucose6phosphatedehydrogenasedeficiency,and
LeschNyhansyndrome(LNS)(hypoxanthineguaninephosphoribosyltransferase[HGPRT]
deficiency).
TheUSMLElovestoaskstudentstocalculateinheritancerisk.Thiswillrequireyoutoknow
theinheritancepatternofthediseaseinquestionandapplyittotheHardyWeinbergprinciple.
RecallthatifapopulationisinHardyWeinbergequilibrium,andpisthefrequencyofthe
normalallelewhileqisthefrequencyoftheabnormalallele,diseaseprevalence=p2+2pq+q
2wherep2andq2representtheprevalenceofhomozygosityand2pqisheterozygosity

prevalence(seeCase112(st0085),q4).
Step1Secret

4 Whatistheprocessoflyonizationandhowmayitcausethemanifestationof
Xlinkeddiseasesinfemales?
BecausefemaleshavetwoXchromosomes,theywouldhavetwicethelevelofexpressionofgenes
locatedontheXchromosomewereitnotfortherandominactivationofoneXchromosomethat
occursineachsomaticcellearlyinembryogenesis.Thisprocessiscalledlyonization(namedafterthe
scientistLyon,whowasthefirsttoproposeit).OneofthemanifestationsoflyonizationistheBarr
body,acondensed,oftendrumstickshapedbodyofDNAseenattheperipheryofthenucleiofthe
cellsoffemales,whichcorrespondstotheinactivatedXchromosome.Anothernonpathologic
manifestationoflyonizationisthecolorationpatternofcalicocats.

OwingtothenormallyrandomnatureoftheXchromosomeinactivation,somefemalesmayhappen
tohaveamutatedXlinkedalleleontheactiveXchromosomeofalargenumberofcellsand,asa
result,mayexhibitsomepathologicfeatures.Theserareindividualsaretermedmosaicsor
manifestingheterozygotes.Forexample,somefemalecarriersofhemophiliaAwillhavesome
degreeofanemiaifalargeenoughproportionoftheirbonemarrowcellsinactivatetheX
chromosomecarryingthenormalfactorVIIIallele.

5 Whydosomediseasesshowanautosomaldominantpatternofinheritance?
Whydothegeneticdiseasesofconnectivetissueusuallyfallwithinthis
category?
Inautosomaldominantdiseases,thediseasemanifestseventhoughthereisanormalcopyofthe
generemainingthatproduces50%ofthenormalamountofgeneproduct.Dominanceofadefective
genecanbeattributedtooneofthefollowingreasons:morethan50%ofnormalgeneproductis
neededforanondiseasedphysiologicstatethedefectiveproteinadverselyaffectsthenormalgene
product(adominantnegativeeffect)orthedefectiveproteinhasacquiredanovel,detrimental
property.
Mostdiseasescausedbymutationsinnonenzymaticstructuralproteins(e.g.,collagen,fibrillin)orin
membranereceptors(e.g.,lowdensitylipoprotein[LDL]receptor)areinheritedinanautosomal
dominantmanner.Thisagainisausefulgeneralizationfortheboards.

6 Whatisthegeneralrelationshipbetweenthefunctionofaproteinandits
patternofinheritance?
Table111(t0010)showstheserelationships.
Table111
ProteinsinEnzymeDeficiency

Functional
Category*(tf0010)

Inheritance
Pattern

ExampleDisease(s)

DefectiveProtein

Enzymes

Autosomal
recessive

Phenylketonuria(PKU)

Phenylalanine
hydroxylase

Galactosemia

Galactose1phosphate
uridyltransferase

MediumchainacylCoA
dehydrogenase(MCAD)deficiency

MediumchainacylCoA
dehydrogenase

TaySachsdisease

HexosaminidaseA

Thalassemias

orHemoglobin

Cysticfibrosis

Chloridechannel

Transportprotein

Autosomal
recessive

Structuralproteins

Autosomal
dominant

Osteogenesisimperfecta

TypeIandtypeIIcollagen

Marfansyndrome

Fibrillin

Hereditaryspherocytosis

Spectrin(foundinthe
RBCmembrane)

Developmental
geneexpression

Autosomal
dominant

Achondroplasia

Fibroblastgrowthfactor
receptor3(FGFR3)

Metabolic
receptors

Autosomal
dominant

Familialhypercholesterolemia

LDLreceptor

CoA,coenzymeALDL,lowdensitylipoproteinRBC,redbloodcell.

*Theinformationpresentedconveysthegeneralpattern,butafewexceptionscanbefoundineach
category.

7 Whatarethefollowingmolecularbiologydiagnosticmethodsusedfor?
Explainbrieflyhowtheywork
A.Southernblotting
ThistechniqueinvolvesdetectingthepresenceofaspecificDNAsequencewithinamixtureofDNA
byusingasequencespecificstrandofcomplementaryDNAormessengerRNA(aprobe)thatis
abletohybridizetothetargetedDNA.ThespecificstepsincludeseparatingthemixtureofDNA
fragmentsbygelelectrophoresis,denaturingtheDNA(i.e.,alteringtheDNAsolutionsothatthe
doublestrandedDNAseparatesintosinglestrands),transferring(i.e.,blotting)theDNAontoa
membrane,andmixingtheblottedDNAmixturewithradioactivelylabeledprobestoallowfor
hybridization.Inthelaboratory,itisoftenusedtodetectthepresenceoflargeuniqueDNA
sequences(suchasagenemutation)withinapatient'sgenome.
B.Northernblotting
NorthernblottingisverysimilartoSouthernblotting,exceptthataspecificsequenceofRNA(rather
thanDNA)isdetectedusinganucleicacidprobe.Thistechniqueiscommonlyusedtomeasure
expressionofageneinapatient,asdeterminedbyitsproductionofmessengerRNA(mRNA).
C.Polymerasechainreaction(PCR)
PCRallowsfordetectionofaspecificDNAsequence(suchasamutantallele)bymakingbillionsof
copiesofthatallelefromaslittleasasingleDNAmolecule.Thistestisperformedbyusingtwo
primers,whicharecomplementarytotheDNAregionsattheendsofthesequenceofinterestthatis
tobeamplified.ThetargetDNAisamplifiedviamultipleroundsofDNAdenaturation,primer
hybridization(orannealing),andextensioncatalyzedbyatemperatureinsensitiveDNApolymerase.
D.Westernblotting

ThistestissimilartoSouthernorNorthernblotting,butratherthandetectinganucleicacid,it
measuresthelevelofaspecificprotein.First,theproteinmixtureiscoatedbyanegativelycharged
detergentmoleculethatdenaturestheproteins(i.e.,unfoldsitintolinearpeptides)suchthatthe
proteinscanbeseparatedaccordingtosizeusinggelelectrophoresis.Next,theproteinsareblotted
ontoamembranetowhichanantibodyagainsttheproteinofinterest(theprimaryantibody)is
added.Iftheproteinispresent,theprimaryspecificantibodywillbindtothemembraneandthis
binding,inturn,willbedetectedusingasecondaryantibodythatisbothdirectedagainstthefirst
antibodyandlabeledinanassayablefashion.(Forexample,theprimaryantibodymaybeaspecific
sheepantibody,butthesecondaryantibodyisanantisheepantibodylinkedtoanenzymethat
producesacoloredproductuponexposuretothereagents.)Westernblotsareusedclinicallyto
measurethedegreeofproteinexpressionofagene.Thisisimportantbecausediseasescanbecaused
bytranslationalproblems,inwhichtranscriptionofthegeneintomRNAoccursnormallybutthe
translationofthismRNAisdefective.

Enzymopathies(enzymedeficiencies)arecausedbyadeficiencyinactivityofan
enzyme,resultinginatoxicaccumulationofintermediates,alackofproducts,orboth.
EnzymopathiesareusuallyautosomalrecessiveorXlinkedrecessive.
Lyonization,therandominactivationofoneofthetwoXchromosomesinthesomatic
cellsinearlyfemaleembryonicdevelopment,canresultinmosaicfemalesthatexhibit
pathologicsymptomsofXlinkeddisorders.
Defectsinstructuralproteinstypicallyexhibitautosomaldominantinheritance.
BothSouthernblottingandpolymerasechainreactioncanbeusedtodetectspecific
sequenceswithinmixturesofpatientDNA.NorthernandWesternblottingsimilarly
measureRNAandproteinlevels,respectively.
SummaryBox:BasicConceptsinBiogenetics

Case111
A2dayoldinfantboytestspositiveforarelativelyrare,butsimplymanaged,medical
condition.Thediagnosisisbasedonthepresenceofmarkedlyelevatedserumlevelsofan
essentialaminoacid.Asecondpositivetestresultisobtainedduringhis2weekcheckupvisit.
Hisfamilyhistoryisremarkableformentalretardationina45yearoldaunt.

1 Whatisthemostlikelydiagnosisinthisbabyandhowisitinherited?
Themostlikelydiagnosisisphenylketonuria(PKU),which,aswithmostenzymopathies,isinherited
inanautosomalrecessivemanner.

2 Whatisthemajordefectandunderlyingpathophysiologyofthisdisorder?

PKUiscausedbythedefectiveconversionofphenylalaninetotyrosineresultingfrommutations
inthephenylalaninehydroxylase(PAH)gene(classicPKU).ThePAHenzymedeficiencyleadsto
bothanaccumulationofphenylalanine(substrate)anditsderivativesphenylpyruvicacidandother
phenylketones,aswellasadecreaseinthelevelsoftyrosine(product)anditsderivatives(suchas
dopaandmelanin).ArarerformofPKUinvolvesadefectinthesynthesisoftetrahydrobiopterin(BH
4),whichservesasacofactorforPAH.Thiscofactorisalsorequiredforthesynthesisofldopafrom
tyrosine.lDopaisthenconvertedtodopamine,whichcanbeusedtosynthesizethecatecholamines
norepinephrineandepinephrine.InordertodistinguishbetweenthegeneticcausesofPKU,onecan
examinelevelsofdopamineandprolactin.Recallthatdopamineisanegativeinhibitorofprolactin
release.BecauseclassicPKUdoesnotaffectdopaminesynthesis,prolactinlevelsshouldberelatively
normal.BH4deficiency,ontheotherhand,willreducedopaminesynthesisandthusprolactinlevels
willbeelevatedinthesepatients.
Thepathologyisprimarilyaresultofsubstrate(phenylalanine)accumulation,whichcausessevere
neuronaldamage,mentalretardation,growthretardation,andmotordysfunction.Thelackof
neurotransmittercompoundsderivedfromtyrosine(particularlythecatecholaminesdopamine,
norepinephrine,andepinephrine)mayalsocontributetodamageofthecentralnervoussystem
(CNS).Othermanifestationsincludeapredispositiontoeczema,amustyodor(causedby
phenylketoneexcretionintosweat),andfairskincoloring(duetotyrosinedeficiency,whichnormally
servesasaprecursortomelanin)(Fig.112(f0015)).
Thebiochemicalpathwayaffectedinpatientswithphenylketonuria(PKU)showsupquite
frequentlyonStep1.Befamiliarwiththefunctionsofphenylalaninehydroxylase(PAH)and
tetrahydrobiopterinandtheirrelevancetothisdisease.Youshouldalsoknoweverystepofthe
catecholaminesynthesispathway.
Step1Secret

Figure112
Pathologicmechanismsofphenylketonuria.
(FromBrownTA,BrownD:USMLEStep1Secrets.Philadelphia,Hanley&Belfus,2004.)

3 Howisphenylketonuriatreated?

PatientswithPKUneedtofollowastrictdietthatrestrictsphenylalanineintakeandissupplemented
withtyrosine.Ifstartedwithinthefirstmonthoflife,thisdietisveryeffectiveinpreventingmental
retardation.Becausephenylalanineisfoundinbreastmilk,mostbabiessufferingfromPKUmustbe
placedonspecialphenylalaninerestrictedformulas.Phenylalanineisalsofoundinhigh
concentrationsinartificialsweetenerssuchasaspartame.

4 Giventhefactthatphenylketonuriaisarelativelyrarecondition(prevalence
ratesrangefrom1in2600to1in200,000livebirths),whydoesitmakesense
toscreenallneonatesforthiscondition?
Thescreeningtest(Guthrietest)isinexpensive(asitsimplyinvolvesmeasuringplasma
phenylalaninelevels),andPKUiseasilypreventedbydietarymodifications.Furthermore,early
screeningdetectsthediseasebeforeirreparabledamage(particularlytotheCNS)hasoccurred(i.e.,
earlyinterventionaffectsoutcome).

5 Iftheparentshaveafemalechildwiththisdisease,whyisitcrucialtoadvise
thechildabouttheriskstoherbabyifshebecomespregnantwhensheis
older?
Aspatientsgenerallytoleratemoredietaryphenylalaninewithage,mostwomenwithPKUabandon
thediettherapybytheirearlyteens,wellbeforetheyreachchildbearingage.Thisterminationof
dietarytherapygenerallyhaslimitedilleffectsonthewomenthemselvesatthisagebutwillcause
irreparableharmtoadevelopingfetusshouldtheybecomepregnant.Specifically,highlevelsof
phenylalaninecandiffuseacrosstheplacenta,causingbraindamageinthedevelopingfetus.So,
althoughthesebabieswill(virtuallyalways)beheterozygousforthePAHmutationandarethusborn
withoutPKU,theycanexhibitseverementalretardation,aconditiontermedmaternalPKU.

Relatedquestion
6 Whyisscreeningforcongenitalhypothyroidism(cretinism),congenital
adrenalhyperplasia,andgalactosemiaalsoroutinelyperformedinnewborns?
Thesediseasesaresimilarlyscreenedforbecausetheyareadditionalpreventablecausesofmental
retardationordeath.Ingeneral,screeningisperformedondiseasesforwhichtreatmentisavailable,
forwhicharapidandlowcostlaboratorytestisavailable,andthatarefrequentandseriousenough
tojustifythescreeningcost.

Phenylketonuria(PKU)isduetoadefectinthephenylalaninehydroxylasegenethat
resultsintheaccumulationofthephenylalaninesubstrateanditsphenylketone
derivativesaswellasthelackofthetyrosineproductanditsderivatives(suchasmelanin
orcatecholamines).
PKUcanresultinmentalretardation,growthretardation,motordysfunction,eczema,a
mustyodor,andfairskincoloring.

ThemanifestationsofPKUcanbepreventedbyneonatalscreeningandearlyinstitution
ofadietwithlowphenylalanineandhightyrosinelevels.Womenwithahistoryof
phenylalaninewhoarepregnantormaybecomepregnantshouldstrictlyfollowsucha
dietregardlessoftheirpersonalsymptomsduetotheriskoffetalneurologicdamage
(maternalPKU)resultingfromembryonicexposuretohighphenylalaninelevels.
InadditiontoPKU,congenitalhypothyroidism,congenitaladrenalhyperplasia,and
galactosemiaareroutinelyscreenedforinnewbornsbecausetheirdiseasemanifestations
(suchasmentalretardationordeath)canbepreventedwithearlyintervention.
SummaryBox:PhenylketonuriaandNeonatalScreening

Case112
Awomanandherhusbandjustgavebirthtoachildwithcysticfibrosis(CF).Boththewoman
andherhusbandareintheir30sandarecompletelyasymptomatic.

1 Iftheparentsdecidetohaveanotherchild,whatistheprobabilityofthatchild
havingcysticfibrosis?
BecauseCFisanautosomalrecessivedisease,thechanceofthesecondchildhavingCFremainsat
25%.Eachparentisaheterozygouscarrierofthemutantallelesuchthateachparenthasa50%
chanceofpassingitontotheiroffspring,andthechanceofthechildreceivingbothmutantallelesis
0.50.5=0.25,or25%.
Note:Eachbirthisacompletelyindependentevent,suchthattheoutcomesofpriorpregnanciesdo
notaffecttheoddsofdiseasetransmissioninsubsequentpregnancies.
StudentsarefrequentlyaskedtocalculategeneticprobabilitiesontheUSMLE.
Step1Secret

2 Iftheparentswanttohaveanotherchild,whatkindofgeneticscreening
methodsareavailableforthemtoconsider?
Afewgeneticscreeningmethodsareavailable:
Thefirstmethodinvolvespreimplantationdiagnosisusinginvitrofertilization(IVF).Thezygotes
resultingfromIVFareallowedtodevelopintoan8cellor16cellblastomere,fromwhichasingle
cellisremoved.TheDNAisisolatedfromthissinglecellandPCRisthenusedtoscreenfora
mutationinthecysticfibrosistransmembraneregulator(CFTR)genelocus.Onlytheunaffected
embryos(wildtypeorcarrierstatus)areimplanted.

Othergeneticscreeningmethodsinvolveprenataldiagnosisusingeitheramniocentesis(the
withdrawalof2030mLofamnioticfluidat1517weeksofgestation)orchorionicvillussampling
(theaspirationofseveralmilligramsofvillustissuesat1011weeksofgestation).

3 Despitehavingmutationsinthesamegene,whydopatientswithcystic
fibrosisexhibitsignificantvariabilityindiseaseseverity?
First,differentpatientsmayhavedifferentmutationsofthesamegene,withcertainmutations
causinglessseverephenotypes.Forexample,mutationsofthechloridechannelgenethathavea
smallerdetrimentaleffectonitsfunctionresultinmilderclinicalmanifestations.Thereareovera
thousandmutationsofCFTRthathavebeenidentifiedinpatientswithCF.Thisphenomenonof
differentmutationsofthesamealleleresultingindifferingdiseasemanifestationsisknownasallelic
heterogeneity.Interestingly,manypatientswithCF(>33%)arecompoundheterozygotes,witha
differentlocusmutatedoneachcopyoftheirCFTRgenes.
Second,eveninpatientswithidenticalmutations,thereisoftensomedegreeofclinical
heterogeneity.Thismaybeduetoothergeneticdifferencesortoenvironmentalvariablesthat
influencediseaseexpression.
Note:Manygeneticdiseaseshaveallelicheterogeneity,leadingtosignificantheterogeneityin
clinicalmanifestations.

4 Assumingacysticfibrosisprevalencerateof1in2500,whatisthecarrier
frequencyforthisdisease?
Here,theHardyWeinberglawcanbeusedtodescribethegenotypicdistributionofanabnormal
allele(p+q=1)andthephenotypicdistributionofthedisorder:
p

+ 2pq + q

= 1

p=frequencyofnormalallele
q=frequencyofabnormalallele(1p)
p2=frequencyofunaffectedindividuals
2pq=frequencyofcarriers(usuallyasymptomaticinautosomalrecessivediseases)
q2=frequencyofdisease
Theseequationsareeasytorememberifonerealizesthatthephenotypeequationissimplythe
squareofthegenotypeequation:
(p + q)

= p

+ 2pq + q

= 1

= 1

AssumingaCFprevalenceof1in2500,q2=1/2500suchthatq=0.02.Becausep+q=1,pis0.98.
Therefore,thecarrierfrequencyforCFis2pq=2(0.98)(0.2)=0.039,orapproximately4%ofthe

population.Thus,inthisexample,1inevery25isacarrier.Thisisroughlythecarrierfrequencyin
whites,whereasthemutationandthediseasearelesscommoninnonwhites.
TheHardyWeinberglawcanbeappliedtoallelesandpopulationsthatareingeneticequilibrium
(i.e.,populationsinwhichtheallelefrequencyisnotundergoingrapidchange).Forthepurposesof
theUSMLE,usuallysuchequilibriumcanbeassumed.

Childrenborntotwocarriersofanautosomalrecessivemutationhavea25%chanceof
inheritingthemutation,regardlessoftheoutcomeofpriorpregnancies.
Geneticscreeningmethodsincludepreimplantationdiagnosisusinginvitrofertilization
aswellasamniocentesis(doneat1517weeksgestation)orchorionicvillussampling
(doneat1011weeksgestation).
Theprevalenceofvariousgenotypesandphenotypesrelatingtoanalleleingenetic
equilibriuminapopulationcanbepredictedusingtheHardyWeinbergequation.
SummaryBox:PopulationGeneticsandPrenatalGeneticScreening

Case113
A9montholdbabygirlisbroughttothehospitalbyherparents,bothofwhomareAshkenazi
Jews.Theparentsreportthatoverthepast3monthsthebabyhasbeenhavingtroublefeeding
andhasbecomelethargicandfloppyappearing.Theyhavealsonoticedthatthechildstartles
easily.Morerecently,thebabyhasdevelopedworseningmotordysfunction,nowwithrigidand
spasticmovements.Theparentsarealsoconcernedthebabygirlisgoingblind.On
examination,acherryredspotwasobservedatthecenterofthemacula.

1 Whattwodiagnosesaretopconsiderationsinthedifferentialdiagnosisatthis
point?
Progressiveneurodegenerationandmacularcherryredspotsinachildshouldautomaticallymake
youthinkofTaySachsdisease(TSD)andNiemannPickdisease(NPD).Bothareautosomal
recessivelysosomalstoragediseasesand,morespecifically,sphingolipidoses.TSDiscausedbya
deficiencyofhexosaminidaseA,andNPDiscausedbyadeficiencyinsphingomyelinase.The
prevalenceofboththesediseasesishigherinAshkenaziJews.
Note:Manyofthelysosomalstoragediseaseshavemultiplesubtypesbasedontheunderlying
biochemicalandmolecularcharacteristics.Thesyndromesdescribedherecorrespondtothemost
commonsubtype(i.e.,type1ortypeA)ofeachdisorder.

Case113continued:

Cellsfromthischildareisolatedandexaminedundertheelectronmicroscope,revealing
onionskinningoflysosomes.

2 Nowwhatisthemostlikelydiagnosis?
ThisfeatureisassociatedwithTSDbutnotNPD.Ontheotherhand,foamyhistiocytes
(macrophagesfilledwithsphingomyelin)arefoundthetissuesofpatientswithNPD.
Note:Theexaggeratedstartlereactionreportedintheinitialvignettewasparticularlysuggestiveof
TSD.Thestartlereactioniscausedbyhyperacusisand,asitdoesnotoccurinNPD,canbeamajor
cluetotheearlydiagnosisofTSD.

3 WhatisthepathogenesisofTaySachsdisease?
HexosaminidaseAisalysosomalenzymethatcleavesacerebralganglioside(GM2,asphingolipid).
Mutationsmakethisenzymelesseffective,leadingtomassiveaccumulationofGM2andits
byproductswithinthelysosomesofneurons.Theselysosomesbecomeenormouslyenlargedsuch
thattheybegintointerferewithnormalcellfunctionandultimatelycauseneuronaldeath.Neuronal
deaththatoverliesthefoveacentralisoftheretinaisresponsibleforthecherryredspotseenon
funduscopicexamination.Morespecifically,ganglioncellsintheretinafillwithlipid,whichimparts
anopaquegraycolortotheretina.Becausetheopticdiskdoesnotcontainganglioncells,itremains
redonagraybackground,givingthelookofacherryredspot.Thisisacommonbuzzwordusedby
USMLEtestmakers.
ThepathogenesisofothersphingolipidosessuchasNPDorGaucherdiseaseissimilarlydueto
accumulationofsubstratesoflysosomalenzymes.Thedifferingdiseasemanifestationsofthese
sphingolipidosesdependupontheorgansinwhichthesphingolipidsaccumulateandtheunderlying
organsensitivities.Forexample,visceralsphingolipidaccumulationandhepatosplenomegalyare
prominentinNPDbutnotinTSD.TSD,incontrast,ischaracterizedbyrelativelyisolatedcentral
nervoussystem(CNS)sensitivitytoGM2accumulation.

4 Whichisthemostcommonsphingolipidosis?
Gaucherdiseaseisnotonlythemostcommonsphingolipidosisbutalsothemostcommonlysosomal
storagedisease.Thediseaseiscausedbydeficiencyofglucocerebrosidaseandischaracterizedbythe
presenceofglucocerebrosideladenGauchercells,macrophageswithcharacteristiccrumpledtissue
paperappearanceandnucleidisplacedbylipid.ItalsomorecommonamongAshkenaziJews.
UnlikeTSDandNPD,Gaucherdiseaseusuallysparesneuronaltissue.Glucocerebrosideinstead
accumulatesinthereticuloendothelialsystem,namely,thespleenandliver(causing
hepatosplenomegaly),bone(causingbonepainandfractures),andbonemarrow(causing
pancytopenia,withparticularlyprominentthrombocytopenia).

Note:Despitethecharacteristichistologicorcytologicfindingsofmanystoragediseases,virtually
allofthesedisordersaretodaydiagnosedusingtestingfortheunderlyingspecificgeneticdefects.

5 Whatarethemucopolysaccharidoses?
Mucopolysaccharidosesareadifferenttypeoflysosomalstoragediseaseinwhichthesubstratesthat
accumulateinthelysosomesareextracellularmatrixmoleculescalledglycosaminoglycans(which
werepreviouslyknownasmucopolysaccharides).Likethesphingolipidoses,thesediseasesare
causedbyhereditarydeficiencyoflysosomalenzymes.Thetwomainexamplesofthistypeofdisease,
HurlersyndromeandthesimilarbutlesssevereHuntersyndrome,arebothcausedbyaccumulation
oftheglycosaminoglycansheparansulfateanddermatansulfate(Table112(t0015)).
Table112
Mucopolysaccharidoses:HurlerSyndromeandHunterSyndrome

Disorder

Hurlersyndrome

Deficient

Inheritance HighYieldAssociations

Enzyme

Pattern

Autosomal Coarsefacialfeatures(gargoylism),

(mucopolysaccharidosis Iduronidase recessive

hepatosplenomegaly,mentalretardation,joint

typeIH)

andskeletalabnormalities,cardiacdisease,
cornealclouding

Huntersyndrome
Iduronate
(mucopolysaccharidosis sulfatase
typeII)

Xlinked
recessive

Samefeaturesbutwithmildermental
retardationwithaggressivebehaviorandno
cornealclouding

Tokeepthesetwodisordersstraight,thinkofthehunterbeingmale(Xlinked)withaggressive
behaviorandgreatvision(nocornealclouding).Theonlyothercommonlytestedlysosomalstorage
diseasethatisXlinkedrecessiveisFabrydisease(galactosidaseAdeficiency).Lookforperipheral
neuropathyandcardiovascular/renalinvolvementinapatientpresentingwithFabrydiseaseonthe
USMLEStep1.

6 Quickreview:CoverthethreecolumnsontherightsideofTable113(t0020)
andattempttodescribetheenzymedeficiency,accumulatedsubstrate,
inheritancepattern,pathophysiology,andanyhighyieldassociationsforthe
listedlysosomalstoragedisorders
BothTaySachsdiseaseandNiemannPickdiseasearelysosomalstoragediseasesand
sphingolipidosesthatarecharacterizedbyprogressiveneurodegenerationalongwith
macularcherryredspotsandblindness.Bothdisordersaremorecommonamong
AshkenaziJews.

TaySachsischaracterizedbyeasystartling(duetohyperacusis),whileNiemannPick
patientsusuallyhavehepatosplenomegaly.Pathologically,TaySachsischaracterizedby
lysosomalonionskinning,whereasthetissuesofNiemannPickpatientscontain
foamyhistiocytes.
Gaucherdiseaseisthemostcommonsphingolipidosisandlysosomalstoragedisease.It
resultsinhepatosplenomegaly,bonepainandfractures,andmarrowfailurebutspares
neuronaltissue.PathologicallyitischaracterizedbylipidladenGauchercells
(macrophages)intheseorgansofthereticuloendothelialsystem.
ThemostcommonmucopolysaccharidosesareHurlersyndromeandtheXlinked
Huntersyndrome.Theybothresultinretardation,hepatosplenomegaly,andcoarsefacial
featuresbutdifferinseverity(Hurlersyndromeismoresevere)andinthepresenceof
cornealclouding(seenonlyinHurlersyndrome).
SummaryBox:LysosomalStorageDiseases

Case114
A2yearoldboywasbroughttotheclinicwithchoreoathetosis(constantandinvoluntary
writhingmovementsofthelegsandarms),spasticity(muscularhypertonicitywithincreased
tendonreflexes),impairedcognitivedevelopment,andselfmutilation(compulsivebitingofthe
fingers,lips,tongue,andinsideofthemouth).Theparentsalsoobservedthepresenceof
orangesandinthechild'sdiaperswhentheboywasafewmonthsold.

Table113
LysosomalStorageDiseases

Lysosomal
Storage

EnzymeDeficiency/Accumulated
Substrate

Inheritance PathophysiologyandHigh
Pattern
YieldAssociations

HexosaminidaseA/GM2ganglioside

Autosomal Accumulationofcerebral

Disease
TaySachs
disease

recessive

gangliosidecauses
progressivepsychomotor
deterioration,macular
cherryredspot,lysosomes
withonionskinning

Gaucher
disease

Glucocerebrosidase/glucocerebroside Autosomal Gauchercells(enlarged


recessive

lipidladenhistiocyteswith
wrinkledtissuepaper
cytoplasm)accumulatein

bone,marrow,liver,and
spleencausingbonepain
andfractures(bonecrises),
osteonecrosisoffemoral
head,massiveHSM,
pancytopenia
NiemannPick

Sphingomyelinase/sphingomyelin

disease

Autosomal Sphingomyelinaccumulation
recessive

inneuronsandliver/spleen
causesprogressive
psychomotordysfunction,
macularcherryredspots,
foamyhistiocytes,and
HSM

Fabrydisease GalactosidaseAceramide
(angiokeratoma trihexoside

Xlinked
recessive

Accumulationinthevascular
endotheliumresultsin

corporis

Maltesecrosses(fat

diffusum)

bodies)inurineandrenal
disease,angiokeratomas,
burningperipheral
neuropathy,stroke,and
cardiovasculardisease

Krabbedisease Galactosidase/ceramide

Autosomal Demyelinationand

(globoidcell

recessive

galactoside(i.e.,galactocerebroside)

leukodystrophy)

accumulationofgloboidcells
inCNSresultinoptic
atrophy,peripheral
neuropathy,and
psychomotorretardation

Metachromatic

ArylsulfataseA/cerebrosidesulfatides Autosomal Sulfatideaccumulationand

leukodystrophy

recessive

centralandperipheral
demyelinationresultin
ataxiaandpsychomotor
degenerationdementiain
adults

CNS,centralnervoussystemHSM,hepatosplenomegaly.

1 Whatisthemostlikelydiagnosis?

LeschNyhansyndrome(LNS),arareXlinkedrecessivedisease,iscausedbyadefectiveHPRT
(hypoxanthinephosphoribosyltransferase)enzyme.TheHPRTenzymeispresentinmostcelltypes
andisinvolvedinthesalvagepathwayofpurinemetabolism.Themoststrikingandcharacteristic
neurologicsymptomofthisdiseaseisselfmutilation.
Recallthatthepurinebasesareadenineandguanineandtherespectivenucleosidesareadenosine
andguanosine.Anitrogenousbaselinkedtoasugarriboseordeoxyriboseisreferredtoasa
nucleoside,whereasaphosphorylatednucleosideisreferredtoasanucleotide.
ThemnemonicsPUReAsGoldandCUTthePYcanbeusedtorememberthatAdenineandG
uaninearePurinebases,whereasCytosine,Uracil,andThyminearePYrimidines.Likewise,
althoughadditionofsugartoabaseproducesanucleoside,nucleotidesarenucleosidesthatare
tiedtoaphosphate.

2 Whatisthenormalfunctionofthepurinesalvagepathway?
Thepurinesalvagepathwayfunctionstosalvagepurinemetabolitessuchashypoxanthineand
guanine,preventingthemfrombeingunnecessarilydegradedandthenrenallyexcretedasuricacid.
(Hypoxanthineisanotherpurinethatisanintermediateinthesynthesisordegradationofadenosine
monophosphate[AMP]orguanosinemonophosphate[GMP]Fig.113(f0020)).AsshowninFigure
113(f0020),thesalvagepathwayrecyclesthesemetabolitestoreplenishthepurinebasesguanine

andadeninebytheactionoftheHGPRTenzyme.Normally,thedenovopathway(smallerdark
arrows)providesonlyabout10%ofthedailypurinerequirement,whereasthesalvagepathway(large
curvedarrows)providestheremaining90%.Theamountofnetdegradationtouricacid(open
arrows)isalwaysbalancedwiththeamountofpurinessynthesizedviathedenovopathway.It
followsthatthelossofthesalvagepathwaywouldresultinadramaticincreaseindenovopurine
synthesisandasimilarlydramaticincreaseinuricacidgeneration.

Figure113

Thepurinesalvagepathway.Seeadditionaldiscussionintext.AMP,adenosinemonophosphateGMP,
guanosinemonophosphateHPRT,hypoxanthinephosphoribosyltransferaseIMP,inosinemonophosphate
PRPP,5phosphoribosyl1pyrophosphateXO,xanthineoxidase.
(FromBrownTA,BrownD:USMLEStep1Secrets.Philadelphia,Hanley&Belfus,2004.)

3 Howdodefectsinthepurinesalvagepathwaycausehyperuricemia?
InLNS,HGPRTactivityislessthan1%ofnormal.OwingtotheabsenceofHGPRT,theabilityto
reutilizehypoxanthineandguaninetomakethepurinenucleotidesinosinemonophosphate(IMP)
andGMPislost,sotheseintermediatesaredegradedtouricacid.Asexplainedalreadyinthe
precedingquestion,thepurinerequirementsinthiscasemustbemetbyincreaseddenovosynthesis.
Additionally,becauseofthereducedlevelsofIMPandGMP,thefeedbackinhibitionnormally
exertedbyIMPandGMPuponthedenovopathwayislost(brokenarrowsinFig.113(f0020)),even
furtherpromotingtheactivityofthedenovosynthesispathway.Becausepurinesynthesisviathede
novopathwaymustbebalancedbypurinedegradationintotouricacid,thedramaticincreaseinde
novosynthesisresultsinseverehyperuricemia.
Inotherwords,bothexcessiveactivationofthedenovopathwayandinsufficientHGPRTsalvageof
purinemetabolitescontributetothehyperuricemiainLNS.

4 Whatwastheorangesandinhisdiapersobservedbyhisparents?
Thesandrepresentsuricacidcrystals.Uricacidhaslimitedsolubilitysuchthatinconditionsof
extremehyperuricemia,itwillprecipitatefromurine,formingvisibleorangesand.Itcanalso
precipitatefromtheplasmaandaccumulateinthejoints,causinggoutyarthritis.Interestingly,most
patientswithLNSdonotdevelopgout,presumablybecauseofthepatientsshortlifespan(ofabout
20years).However,patientswithonlyapartialdeficiencyofHPRT(with120%ofnormalactivity)
haveanormallifespanbutaresusceptibletodevelopingseveretophaceousgout.

5 WhydoboyswithLeschNyhansyndrometypicallypresentwithrenal
dysfunction?
PatientswithLNSdevelopkidneydiseaseprimarilyfromrepeateduricacidkidneystonesand
urinarytractobstruction.Inadditiontonephrolithiasis,chronichyperuricemia(fromanycause)can
resultinrenalinsufficiencycausedbyuratedepositionintherenalparenchyma,aprocessreferredto
asuratenephropathy.
Note:Incontrastwiththerenalandjointdisease,thecauseoftheneurologicsymptomsinLNSis
notwellestablished.

6 Howmightthispatientbemanagedpharmacologically?
Allopurinolisusefulinthetreatmentofhyperuricemiaofanycause.Itworksbypreventinguricacid
productionbyinhibitingtheenzymexanthineoxidase(XO)(seeFig.113(f0020)).Thexanthineand
hypoxanthinethataccumulateinsteadaremoresolubleandreadilyexcretedthanuricacid.

Other,morecommonusesofallopurinolincludethetreatmentor(morecommonly)theprevention
ofuratenephropathy,uricacidstones,goutyarthritis,andtumorlysissyndrome(whichiscausedby
treatmentofacuteleukemiasordisseminatedlymphomas).

LeschNyhansyndromeiscausedbyadefectinthepurinesalvagepathway,resultingin
bothoveractivityofthedenovopurinesynthesispathwayandexcessgenerationofthe
purinemetaboliteuricacid.
ThisXlinkeddisorderisclinicallycharacterizedbyretardation,motordysfunction
(choreoathetosisandspasticity),andverycharacteristicselfmutilatingbehavior.Patients
alsocanexhibitthemanifestationsofseverehyperuricemia,includingseveretophaceous
gout,frequenturicacidkidneystones,anduratenephropathy.
TheproductionofuricacidcanbecontrolledtosomedegreeinLeschNyhansyndrome
usingthexanthineoxidaseinhibitorallopurinol.Thisagentismorecommonlyusedin
patientswithoutLeschNyhansyndrometopreventcomplicationsofhyperuricemiaor
uricaciddeposition,includingattacksofgout,uricacidkidneystones,orurate
nephropathy,andtumorlysissyndromeinpatientsbeingtreatedforleukemiaor
lymphoma.
SummaryBox:LeschNyhanSyndromeandPurineMetabolism

Case115
A4yearoldboyisevaluatedforprofoundhypoglycemiaandseizures.Examinationis
remarkablefornontenderhepatomegaly.Hehasahistoryofmultiplehospitalizationsfor
seizuresandhypoglycemiasincehewas6monthsold.Hisparentshavenoticedthathehas
nevertoleratedevenshortperiodsoffastingwell.Inhisprevioushospitalstays,lowblood
sugarlevelswereconsistentlyobservedwithinafewhoursaftereachfeeding.Hehasalso
repeatedlyhadlacticacidosis,hyperlipidemia,andhyperuricemia.Aliverbiopsyindicates
excessiveaccumulationofglycogenandfat.

1 Whatisthediagnosis?
Type1glycogenstoragedisease(orvonGierkedisease),anautosomalrecessivedisorder,iscaused
bydeficiencyoftheenzymeG6Pase.

2 Whattypeofenzymaticdeficiencyispresentinalltypesofglycogenstorage
diseases?
Thesedisordersarecausedbyadefectineitheranenzymerequiredforglycogensynthesisoran
enzymerequiredforglycogencatabolism(i.e.,glycogenolysis)calledglucose6phosphatase
(G6Pase).Glycogenstoragediseasesprincipallyaffecteithertheliverorskeletalmuscle,whichare

themainsiteswhereglycogenisstored.Whentheyaffecttheliver,theycanleadtohepatomegaly
andcanpredisposetohypoglycemiaanditsattendantcomplications(e.g.,seizuresand,with
repeatedepisodesofhypoglycemia,neurologicimpairment).Whentheyaffecttheskeletalmuscles,
theycancausemusclepainandexerciseintolerance,buttheydonotresultinhypoglycemicepisodes,
asskeletalmuscleplaysnoroleinmaintainingplasmaglucose.Recallthatskeletalmusclelacks
G6Pase,soitcannotdeliverglucosetothebloodstream,asglucose6phosphate(G6P)isunableto
crosstheplasmamembrane(seequestion10).

3 Howisglycogennormallysynthesizedanddegradedintheliver?
Uponentryintoalivercell,glucoseispreventedfromdiffusingoutbyphosphorylationtoG6P,a
reactioncatalyzedbytheenzymeglucokinase(thisfunctionisservedbyhexokinaseinnonhepatic
tissues).G6Pisthenconvertedintoglucose1phosphatebyphosphoglucomutaseandthenuridine
diphosphoglucose(UDPglucose)priortoattachmenttoglycogenbytheenzymeglycogen
synthetase.
Glycogendegradationisprimarilydependentontheactivityoftheenzymeglycogenphosphorylase(
Fig.114(f0025)).

Figure114
Glycogensynthesisanddegradation.1,Glycogensynthetase2,brancherenzyme3,debrancherenzyme4,
phosphoglucomutase5,glucose6phosphatase.
(FromKliegmanRM,MarcdanteKJ,JensonHB,BehrmanRE:NelsonEssentialsofPediatrics,5thed.
Philadelphia,WBSaunders,2006.)

4 Whatisthefunctionofglucose6phosphatase?
Thishepaticenzymecleavesphosphateoffglucose,whichthenenablesglucosetodiffuseoutofthe
cell.Thisreactionisimportantinthemaintenanceofbloodglucosebecausethefinalproductofboth
glycogenolysisandgluconeogenesisisG6P.

5 Whyishepatomegalyseenonexamination?

ThedeficiencyofG6PasecausesG6Ptoaccumulate,whichstimulatesglycogensynthesis,which
inturnenlargestheliver.

6 Whatistheexplanationforhisseverefastinghypoglycemiaandlactic
acidosis?
Duringshorttermfasting,liverglycogenolysisisthemajorpathwaythatmaintainsbloodglucose.
WhenG6Paseisdeficient,glycogenolysisisnoteffectiveatreleasingglucoseintothebloodstream,
leadingtohypoglycemia.Inaddition,thereleaseofglucosemadebygluconeogenesisduringfasting
isalsoimpairedbecausethisprocessisalsodependentupontheenzymeG6Pase.Thisfurther
contributestohypoglycemia.ExcessiveaccumulationofG6Pgreatlypromotesglycolysis,resultingin
highlevelsofpyruvateproduction.Thepyruvateisthenconvertedintolactatewhenthe
mitochondrialuptakeofpyruvateissaturated,resultinginlacticacidosis(seeFig.114(f0025)).
Foryourownreview,recallthattheproductionoflacticacidfrompyruvatedoesnotcauseany
additionalincreaseinproductionofadenosinetriphosphate(ATP).Thefunctionofthispathwayisto
simplyregeneratetheelectroncarrierNAD+fromNADH.

7 Whyisthispatientsusceptibletohypertriglyceridemia?
ExcessiveaccumulationofG6Poverstimulateshepaticglycolysis,supplyingsubstratefor
downstreampathways.Inadditiontolactatesynthesis,thesepathwaysalsoincludedenovofatty
acidandtriacylglycerol(i.e.,triglyceride)synthesis.Becausethehypoglycemiastimulatesglucagon
productionoverinsulinrelease,lipolysisispromoted,providingabundantfattyacidstoothertissues
forenergyproduction(i.e.,oxidation).Asubstantialportionofthesefattyacidsentersthe
mitochondriaofvarioustissuestobeoxidized,buttheexcessisrepackagedinthelivertoform
triglyceriderich,verylowdensitylipoprotein(VLDL)particlestobereleasedintothecirculation.
HighinsulinlevelsnormallypreventformationandreleaseofVLDLparticlesfromtriglycerides,but
instatesofprofoundhypoglycemia,thisinhibitorysignalisnotpresent.
Youshouldknowhowglucagonpermitsoxidationtooccur(Fig.115(f0030)).NoteinFigure115
(f0030)thatglucagonstimulatesactivityofmalonylCoAdecarboxylase,whichcatalyzesthe
breakdownofmalonylCoAintoacetylCoA.Insulin,ontheotherhand,stimulatestheproductionof
malonylCoAalongthepathwayoftriglyceridesynthesis.MalonylCoAprovidesaninhibitorysignal
tocarnitinepalmitoyltransferaseI(CPTI),amitochondrialenzymethatshuttleslongchainfatty
acidsintothemitochondrialmatrixforoxidation.WithoutCPTIactivity,oxidationcannotoccur.
Itmakessensethatinsulinwouldpreventoxidationfromoccurring,becauseitwouldbeawasteof
energyforthebodytooxidizenewlysynthesizedtriglyceridesinthefedstate.Atthesametime,it
wouldbeadvantageoustopromoteoxidationwhenglucagonispresentinthefastingstate.See
howimportantitistoknowthebiochemistrybehindthediseasesthatyoustudy?
Noticethatwedidnotsuggestthatyoushouldmemorizeeverystepofthepathwaysthatwe
depictedthroughoutthecase.Bysimplyunderstandingthemajorsteps,keyenzymes,and
regulatorsofthepathwaysthatyoulearn,youcanbetterreasonthroughdiseasefindings.This

ishowtheUSMLEwillexpectyoutothinkontheexamination.
Step1Secret

Figure115
Regulationoffattyacidsynthesisandbreakdownbyglucagonandinsulin.

8 Whatcausesthehyperuricemiainthispatient?
Aspreviouslymentioned,alackofG6PaseactivityleadstoanaccumulationofG6Pinthecell.G6Pis
shuntedintothehexosemonophosphateshunt(alsoknownasthepentosephosphatepathway),
leadingtoaccumulationofbothribose5phosphateandPRPP(5phosphoribosyl1pyrophosphate).
PRPPisthemajorallostericactivatoroftheratelimitingenzyme(glutaminePRPP
amidotransferase)(seeFigure113(f0020)inCase114)ofthedenovopurinesyntheticpathway.As
discussedinCase114,anincreaseindenovopurinesynthesisleadstoanincreaseinpurine
degradation,leadingtoincreaseduricacidproduction.
Hypoglycemiaalsomakescellslessabletoresynthesizethehighenergymoleculesadenosine
triphosphate(ATP)andadenosinediphosphate(ADP)fromadenosinemonophosphate(AMP).The
resultingincreaseincytosolicAMPdrivesanincreaseinuricacidproduction.
Finally,theexcessplasmalactatecompeteswithuricacidforurinaryexcretion,furtherexacerbating
thehyperuricemia.

9 Quickreview:CoverthecolumnsontherightsideofTable114(t0025)and
explainhowthesediseasesaffecttheactivityoftheratelimitingenzymesin
thepathwayslistedinthetable
Table114
MetabolicPathwaysinGlycogenStorageDiseases

Pathway

RateLimiting

Effect

BasisofEffect

Enzyme
Glycolysis

Phosphofructokinase Increase Increasedsubstrate(G6P)


1

Glycogensynthesis

Glycogensynthetase

Increase Increasedsubstrate(G6P)

Fattyacidsynthesis

AcetylCoA

Increase Increasedsubstrate(G6Pisconverted

carboxylase

usingpyruvatetoacetylCoA)through
exaggeratedglycolysis

Hexose

G6Pdehydrogenase

Increase Increasedsubstrate(G6P)

monophosphate
shunt
Triacylglycerol
(triglyceride)

Increase Increasedsubstrate(glycerolandfatty
acidsfromdenovosynthesisorfrom

synthesis

lipolysis)

CoA,coenzymeAG6P,glucose6phosphate.

Relatedquestions
10 Whydoesadeficiencyofmuscleglycogenphosphorylase(seenintypeV
glycogenstoragedisease,orMcArdledisease)notresultinhypoglycemia?
Muscleglycogenphosphorylaseisrequiredforglycogenolysis(breakdownofglycogenintoG6P)in
muscle.Thereisasimilarenzyme,encodedbyadifferentgene,intheliver.However,unlikeinthe
liver,thereisnoG6Pasepresentinmuscletodephosphorylateglucose.Assuch,glucoseisunableto
diffuseoutofthemusclecell.Similarly,whenglucoseentersamusclecellandisphosphorylatedby
hexokinase,itremainspermanentlytrapped.Inotherwords,glucosethatentersamusclecellcannot
bereleasedand,instead,mustbeconsumedbythatcell.Forthisreason,musclenormallymakesno
contributiontothemaintenanceofbloodglucose.
InMcArdledisease,onlymuscleglycogenphosphorylaseislost.Thecorrespondingliverenzymeis
unaffected.TheabnormalaccumulationofG6Pinmuscleresultsinsymptomssuchascrampsand
musclefatigue,butthereisnoimpairmentinthemaintenanceofbloodsugar.Youmayalsosee
myoglobinuriaassociatedwiththisconditionasaresultofmuscledamage.

11 Reviewthehighyieldglycogenstoragediseases
SeeTable115(t0030)forasummaryofthesediseases.
Table115
SummaryofHighYieldGlycogenStorageDiseases

Glycogen
Storage

EnzymeDeficiency

ClinicalHallmarks

Hepaticandrenal

Massivehepatomegalyandliverdysfunction,renal

Disease
TypeI(von

Gierke

glucose6phosphatase

enlargement,severehypoglycemia,growthfailure

Lysosomalglucosidase

Cardiomegalyleadingtocardiacfailureskeletalmuscle
weaknessleadingtorespiratorymusclefailure

TypeIII(Cori

Amylo1,6glucosidase

Milderdiseaseleadingtostuntedgrowth,

disease)

(debranchingenzyme)

hepatomegaly,andhypoglycemia

TypeV

Muscleglycogen

Exerciseinducedmusclecramps,myoglobinuriawith

(McArdle

phosphorylase

strenuousexercise

disease)
TypeII
(Pompe
disease)

disease)
Note:Althoughtheliveraccountsforthemajority(about90%)ofgluconeogenesisandblood
glucosemaintenance,thekidneycontributesabout10%.Assuch,typeIdiseasecanalsoresultinless
severerenaldisease(withkidneyenlargement,proteinuria,andrenalinsufficiency).
AlsonotethattypeII(Pompe)diseaseisbothaglycogenstoragediseaseandalysosomal1,4
storagedisease.Normallyasmallpercentage(about2%)ofcellularglycogenbreakdowniscarried
outbylysosomalglucosidase.Ashepaticandrenalglycogenphosphorylasearestillfunctional,
deficiencyintheenzymedoesnotresultinhypoglycemiabut,instead,causesaccumulationof
glycogenwithinthelysosomes.Thisoccursmostsignificantlyinthecardiacandskeletalmuscles.

TypeIglycogenstoragedisease(vonGierkedisease)iscausedbyadefectinglucose6
phosphatase,theenzymerequiredforthereleaseofglucose(generatedeitherfrom
glycogenbreakdownorgluconeogenesis)fromtheliverorkidneys.Thelackofthis
enzymeresultsinincreasedsynthesisofglycogen,fattyacids,andtriglyceridesaswellas
increasedactivityofthehexosemonophosphateshunt.Thedisorderisclinically
characterizedbyprofoundhypoglycemia(resultinginseizures,poorgrowth,or
neurologicimpairment)lacticacidosis,hyperlipidemia,andhyperuricemiaandmassive
hepatomegaly,liverdysfunction,andrenalenlargementduetoglycogenandfat
accumulation.
TypeVglycogenstoragedisease(McArdledisease)isduetoabsentmuscleglycogen
phosphorylaseactivity.Thisresultsinmusclesymptoms(suchascramps,fatigue,or
myoglobinuriawithexercise)duetoabnormalaccumulationofglycogeninmuscle.The
maintenanceofbloodsugar,however,isnormal.
TypeIIglycogenstoragedisease(Pompedisease),duetodeficientlysosomal
glucosidase,isalysosomalstoragedisease.Itischaracterizedbyprominentcardiacand
skeletalmuscleinvolvement,withcardiomegaly,whichcanresultinheartfailure,aswell
asskeletalmuscleweakness,whichcanresultinrespiratorymusclefailure.

TypeIIIglycogenstoragedisease(Coridisease),duetoabsentdebranchingenzyme,is
similartotypeIdiseasebutismilder.Patientsexhibitstuntedgrowth,hepatomegaly,and
hypoglycemia.
SummaryBox:GlycogenStorageDiseases

Case116
An8montholdbabygirlisbroughttotheemergencyroombyherparents.Thebabyhasbeen
vomitingandirritableoverthepast2days,andinthepast8hoursshehasbecomevery
lethargic.Onexamination,herliverismildlyenlarged.Laboratoryfindingsindicate
hypoglycemia,moderatehyperammonemia,andabnormallylowurineketones(giventhe
degreeofhypoglycemiapresent).Analysisofthepatient'surinerevealsamixtureoforganic
acidsrangingbetween6and12carbonslong.

1 Whatisthemostlikelydiagnosis?
ThisbabylikelyhasmediumchainfattyacylCoAdehydrogenase(MCAD)deficiency,themost
commongeneticdisorderoffattyacidoxidation.

2 Whatarethethreelengthclassificationsoffattyacids?
Mostediblefatscontainamixtureofthreetypesoffattyacids:shortchain,mediumchain(with612
carbons),andlongchainfattyacids.Thesefattyacids(infattyacylCoAform)areoxidizedinthe
mitochondriaoftheperipheralcells,whichmetabolizefattyacidsbytheenzymesLCAD(longchain
acylCoAdehydrogenase),MCAD(mediumchainAD),andSCAD(shortchainAD),respectively.

3 Whatarethereasonsfortheclinicalandlaboratoryfindingsexhibitedbythis
patient?
Manytissues(especiallyheartandskeletalmuscle)relyheavilyonfattyacidoxidationastheprimary
fuelsourceforATPproductionduringfastingorduringtimesofmetabolicstress,suchasexerciseor
illness,especiallyillnessthatresultsindecreasedoralintake.Inaddition,asubstantialamountof
thesefattyacidsundergooxidationintheliver,whichusestheresultingacetylCoAtoproduce
ketonebodiesthatarereleasedtoprovideenergyforthebrain(whichisunabletodirectlyoxidize
fattyacids)(Fig.116(f0035)).

Figure116
Summaryofmetabolismoffattyacids.CoA,coenzymeACPT,carnitinepalmitoyltransferaseLCAD,longchain
acylCoAdehydrogenaseMCAD,mediumchainacylCoAdehydrogenaseSCAD,shortchainacylCoA
dehydrogenase.
(FromBrownTA,BrownD:USMLEStep1Secrets.Philadelphia,Hanley&Belfus,2004.)

WhenMCADisdeficient,mediumchainfattyacylCoAmoleculesareunabletoundergooxidation
inthesetissues.Inaddition,althoughlongchainfattyacylCoAcompoundscanbeoxidizedinto
mediumchainacylCoAmolecules,oxidationisarrestedatthe12carbonfattyacylCoAstage.Asa
result,mediumchainfattyacylCoAmoleculesaccumulateinthecytosolandmitochondrialmatrix.
Someofthesemediumchaincompoundsareconvertedtotheorganicacidderivativesthatcanbe
detectedintheurine.Thesederivativesmayalsobetoxictotissuesthatcarryoutoxidation.Inthe
liver,asubstantialportionofthesemediumchainfattyacylCoAmoleculesisusedincytosolic
resynthesisoftriglycerides,resultinginliverenlargementfromfattyinfiltration.
MCADdeficiencyalsocausesATPproductionandketogenesistobegreatlydecreased.Withoutan
adequatesupplyofenergy,therateoftheureacycleisdecreased,leadingtohyperammonemia.The
rateofgluconeogenesisissimilarlyreduced,andtheendogenousglucosesupply(liverglycogen)is
rapidlyexhausted,resultinginhypoglycemia.LossofMCADfunctionalsoresultsindecreased
productionofacetylCoA,which,inturn,leadstodecreasedketoneproductiondespitethe
hypoglycemia.ThisdistinctivehypoketotichypoglycemiapatternischaracteristicofMCAD
deficiency.
OtherclinicalfeaturesofMCADdeficiencyreflecttheinvolvementoforgansthatare(directlyor
indirectly)dependentonoxidation.Thesefindingsincludeliverdamageandliverfunctiontest
(LFT)elevationmuscleenzymeelevation,hypotonia,andrhabdomyolysiscongestiveheartfailure
andneurologicimpairmentandcerebraledema.Interestingly,thisconstellationoffeaturesissimilar
tothatseeninReyesyndrome(whichoccursrarelyinchildrenaftertreatmentofaviralillnesswith
aspirin).

4 Howshouldthischildbetreated?
Avoidanceoffastingandofmediumchainfattyacidsinthedietisessential.Bynotallowingthis
childtorelyonperipherallipolysisandoxidationforenergyneeds,hypoglycemiaand
accumulationofintermediatescausedbythemetabolicblockwillbeminimized.Frequentsmall
mealshighincarbohydrateandproteinandlowinfat(<20%ofcalories)arerecommended.Patients
withMCADdeficiencyshouldtakecarnitinesupplementstopromoteefficientlongchainfattyacyl
CoAtransportintothemitochondria.Becausethecarnitineshuttle(i.e.,mitochondrialuptakeof
longchainfattyacids)istheratelimitingstepforlongchainfattyacidoxidation,promotingthis
pathwaybysupplementingcarnitinemayreducetheenergydeficitcausedbyMCADdeficiency.

5 Howarethepathways/cycleslistedinTable116(t0035)affectedbymedium
chainfattyacylCoAdehydrogenasedeficiency?
MediumchainacylCoAdehydrogenase(MCAD)deficiency,themostcommon
inheriteddefectoffattyacidoxidation,resultsinarrestofoxidationatthe12carbon
stagewithresultanthypoglycemia,hyperammonemia,increasedtriglyceridesynthesis,
impairedketogenesis,andthebuildupoforganicacidsintheurine.
Possibleclinicalmanifestationsincludehypoketotichypoglycemiahepatomegalyfrom
fattyinfiltrationalongwithliverdamageandliverfunctiontest(LFT)elevationmuscle
enzymeelevation,hypotonia,andrhabdomyolysiscongestiveheartfailure(CHF)and
neurologicimpairmentandcerebraledema.
Treatmentrequiresdietarymodification.Specifically,patientsshouldavoidfastingand
theintakeofmediumchainfattyacidsandinsteadshouldeatfrequentsmallmealshigh
incarbohydrateandproteinandlowinfat.
SummaryBox:MediumchainAcylcoaDehydrogenaseDeficiencyandFattyAcid
Metabolism

Case117
Awomanwithacuteintermittentporphyria(AIP)isseekingadviceonhowtoavoid
exacerbationsofherdisease.

Table116
MediumChainAcylCoenzymea(CoA)DehydrogenaseDeficiency

PathwayorCycle

ActivityofPathway

Cause

Hepaticgluconeogenesis

Decreased,causinghypoglycemia LowATPlevels

Hepaticureacycle

Decreased,causing

LowATPlevels

hyperammonemia
Oxidationofmediumchainfatty

Decreased

Enzymedeficiency

Decreased

LowacetylCoA

acids
Hepaticketogenesis

production
ATP,adenosinetriphosphate.

1 Whataretheporphyrias?
Theporphyriasareaseriesofdisorderscausedbyenzymaticdefectsinhemebiosynthesis.The
enzymaticblockleadstotheaccumulationofintermediates,porphyrins,inthehemebiosynthetic
pathwaythathavetoxiceffectswhenpresentathighserumlevels.Thevariousformsofporphyria
canbediagnosedbydetectingthepresenceoftheseporphyrinintermediatesintheurine,serum,or
stool.Eachformofporphyriaisassociatedwithaparticularpatternofporphyrinaccumulation,with
thepatterndependingonthespecificlocationoftheblockinthehemesyntheticpathway.
Incontrastwithamajorityofenzymopathies,mostoftheporphyriashaveanautosomaldominant
modeofinheritancewithreduced,butnotaltogetherabsent,hemesynthesis.Completeabsenceof
hemesynthesisispresumablylethal.

2 Whatarethetypesofporphyria?
Thethreemajortypesofporphyriaareacute,chronic(hemolytic),andcutaneous.Inacute
porphyrias,patientsusuallyexperiencediseaseexacerbationsfollowingspecifictriggers,which
includecertaindrugsandchemicals,hormones(i.e.,estrogensandprogestins),smoking,andvarious
formsofstress(suchasillness,fasting,infection,orsurgery).Theacuteporphyriastendtocause
neurotoxicity,withprominentautonomicnervoussysteminvolvement,resultinginsocalled
neurovisceralsymptoms,suchasabdominalpain,vomiting,constipation,muscleweakness,and
confusion.AIP,causedbyadefectinporphobilinogen(PBG)deaminase,isthemostcommonacute
porphyria.
Defectsinthelaterstepsofhemebiosynthesiscauseaccumulationofphotoreactiveintermediates,
resultinginthephotosensitivitycharacteristicofcutaneousporphyrias.Porphyriacutaneatarda
(PCT)isboththemostcommoncutaneousporphyriaandthemostcommonporphyriaoverall.PCT
iscausedbyadefectintheenzymeuroporphyrinogendecarboxylaseandresultsinchronic,
blisteringlesionsofsunexposedskinand,overtime,scarring,pigmentchanges,andincreasedhair
growth(hypertrichosis).
Thelast(andleastcommon)groupofporphyriasarethehemolyticporphyrias.Significantheme
synthesisoccursnotonlyintheliverbutalsoinredbloodcellprecursors.Inthesedisorders,red
bloodcellsarelysedwhentheiraccumulatedphotoreactiveporphyrinintermediatesareexposedto
lightastheredbloodcellspassthroughcapillariesintheskin.

3 Whymaycigarettesmokingoranticonvulsantssuchasphenytoinand
phenobarbitaltriggeracuteporphyrias?
CigarettesmokeandmanyanticonvulsantsarepotentinducersofhepaticcytochromeP450
enzymes.TheseP450enzymescontainheme,andconsequentlytheirinductionincreasesthe
demandforhemeandincreasesitsbiosynthesis.Inpatientswithacuteporphyria,adrugorstress
inducedincreaseinactivityofthehemebiosynthesispathwayresultsinanincreasedaccumulationof
thetoxicporphyrinintermediates.InpatientswithAIP,acuteattacksareoftenheraldedbythe
developmentofredurine.

Interestingly,PCTisnottriggeredbyinducersofP450.However,itisstronglyassociatedwith
varioushepatotoxicagentssuchasalcohol,cigarettesmoke,hepatitisCinfection,and
hemochromatosisorironoverload.Inwomen,itisalsoassociatedwithestrogenexposure,both
exogenous(e.g.,oralcontraceptivesorhormonereplacement)andendogenous(i.e.,pregnancy).

4 Whyisheminorglucosegiventopatientswithacuteporphyrias?
Hemin,anoxidizedformofheme,actsvianegativefeedbacktodecreasethesynthesisof
aminolevulinicacid(ALA)synthase,theratelimitingenzymeofhemebiosynthesis.Thisnegative
feedbackmechanismisexploitedtherapeuticallyinthetreatmentofacuteporphyriasasintravenous
heministhemosteffectivewayofreducingporphyrinaccumulationandtheneurovisceralattacks.
Glucoseinfusion(theoppositeoffasting)canalsobeusefulbecauseglucoseexertsasimilarnegative
feedbackuponALAsynthase.
PCT,interestingly,iseffectivelytreatedbyphlebotomy.Itisthoughtthatbloodlossdecreases
porphyrinproductionintheliverbydecreasingtotalbodyand,mostimportantly,liverironstores.
Avoidanceoftriggersandtreatmentofexacerbatingfactorsisimportantinthetreatmentofallforms
ofporphyria.

Relatedquestions
5 Howdoesleadpoisoningaffecthemesynthesis?
LeadpoisoninginhibitsthehemesynthesisenzymesALAdehydraseandferrochelatase,resultingin
anemia.ForStep1,knowthatredbloodcells(RBCs)onaperipheralbloodsmearinpatientswith
leadpoisoningwilldemonstratebasophilicstippling.

6 Whatarethesideroblasticanemias?
Theseanemiasareduetodefectsinthehemebiosynthesispathway.Unliketheporphyrias,the
clinicallyapparentpathologyresultsfrominsufficienthemeproductionratherthantheaccumulation
oftoxichemeprecursors.Becauseasubstantialamountofthebody'sironisstoredinheme,defects
inhemesynthesiscanleadtoabnormalironaccumulation.Inthebonemarrowofpatientswith
sideroblasticanemia,ironwilldepositwithinmitochondriatoformsideroticgranulesinaring
patternaroundthenucleusofsomeoftheredbloodcellprecursors,socalledringedsideroblasts.
UsetheTAILSmnemonicformicrocyticanemias:BecauseThalassemia,Anemiaofchronic
disease,Irondeficiency,Leadpoisoning,andSideroblasticanemiasinvolveadefectinhemeor
hemoglobinsynthesis,theyarealloftenmicrocyticandhypochromic.

Theporphyriasarecausedbyenzymaticdefectsinhemebiosynthesis.Theyare
diagnosedbythepatternofbuildupofporphyrinintermediatesintheurine,serum,or
stool.Theytendtobeautosomaldominantandassociatedwithreduced(butnot
altogetherabsent)hemesynthesis.

Theacuteporphyrias,ofwhichacuteintermittentporphyria(AIP)ismostcommon,are
characterizedbyacuteepisodesofneurovisceralsymptomsfollowingexposureto
inducersoftheP450enzymesystem,suchasestrogen,smoking,certainanticonvulsants,
andphysiologicstress.
Thecutaneousporphyrias,ofwhichporphyriacutaneatarda(PCT)isthemostcommon,
resultinphotosensitivityduetotheaccumulationofphotoreactiveporphyrin
intermediatesintheskin.
PCTisthemostcommonporphyriaandresultsinblistering,scarring,dyspigmentation,
andhypertrichosis(hairiness)ofsunexposedskin.PCTisassociatedwithvarious
hepatotoxicagentsorprocessessuchasalcohol,smoking,hepatitisC,or
hemochromatosis.
Thehemolyticporphyriasareuncommonandarecharacterizedbythebuildupof
photoreactiveintermediateswithinredbloodcells,resultinginhemolysisasredblood
cellspassthroughskincapillaries.
Leadpoisoninginhibitshemesynthesis,resultinginamicrocyticanemiawithbasophilic
stipplingofperipheralredbloodcells.
Thesideroblasticanemiasaredefectsinhemebiosynthesisthatsimilarlyresultin
microcyticanemia(ratherthanthetoxicaccumulationofintermediates)theyare
characterizedbythepresenceinthebonemarrowofringedsideroblasts.
SummaryBox:thePorphyriasandDisordersofHemeSynthesis

Case118
A7weekoldbabygirlisbroughttotheofficeandpresentswithvomiting,diarrhea,poor
feeding,andfailuretothrive.Themotherhasrecentlynoticedthatthebabyvomitsorhas
diarrheaafternursing.Findingsonexaminationincludejaundice,dryskin,pallor,
hepatomegaly,andpoortone.Thelaboratoryfindingsincludehypoglycemia,elevationin
serumgalactoseandRBCgalactose1phosphate(Gal1P),galactosuria,albuminuria,and
indirecthyperbilirubinemia.

1 Whatisthediagnosis?
Thisbabylikelyhasgalactosemia,whichisadiseasethat,ifleftuntreated,canleadtoseveremental
retardation.Itisduetodefectivegalactosemetabolismandelevatedlevelsofserumgalactose(recall
thatlactose,thesugarpresentinmilk,ismetabolizedtoglucoseandgalactose).Typically,disorders
ofgalactosemetabolismresultfromdeficienciesingalactokinase,aconditioncalledgalactokinase

deficiency,orgalactose1phosphateuridyltransferase,termedclassicgalactosemia(Fig.117(f0040)
).BecausebothserumgalactoseandGal1Pwereelevatedthischild,shelikelysuffersfroma
deficiencyofGal1Puridyltransferase(GALT)orclassicalgalactosemia.

Figure117
Pathophysiologyofgalactosemia.UDP,uridinediphosphate.
(FromBrownTA,BrownD:USMLEStep1Secrets.Philadelphia,Hanley&Belfus,2004.)

AsdemonstratedinFigure116(f0035),galactoseismetabolizedbygalactokinaseintoGal1P.If
GALTisnotpresenttometabolizeGal1Pintoglucose1phosphate,Gal1Pcanaccumulatetotoxic
levels.

2 Whydoesgalactose1phosphateuridyltransferasedeficiencymanifestwith
jaundice?
Asinothertissuessuchasnervoustissue,thelens,andkidney,thepathologicmanifestationsof
galactosemiaintheliverandRBCsareduetotheaccumulationofGal1Pandgalactitol.This
accumulationresultsintheprematuredestructionofRBCs(i.e.,hemolysis),whichresults,viathe
breakdownofhemoglobin,inincreasedreleaseofunconjugatedbilirubinintothecirculation.
Prehepaticjaundicedevelopsbecause,ifthedegreeofhemolysisissignificantenough,theliverwill
beincapableofconjugatingtheentirebilirubinload,leadingtobuildupofunconjugatedbilirubin.
Thepathologiceffectuponthelivercanresultinliverfunctiontest(LFT)abnormalitiesandliver
dysfunction,furthercontributingtothehyperbilirubinemia.
Recallthatunconjugatedbilirubinisindirectwhereasconjugatedbilirubinisdirect.Theseterms
reflecthowconjugatedbilirubiniswatersoluble(i.e.,charged)andcanbemeasureddirectly,
whereasinordertomeasuretotalbilirubin,aserumsamplemustbetreatedsuchthatthewater
insoluble(i.e.,uncharged)unconjugatedbilirubinisbroughtintosolution.

3 Whydoinfantswithgalactosemiatendtodevelopcataractsifleftuntreated?
Whengalactose(thesubstrate)accumulatesduetothemetabolicblock,itentersanalternate
reactioncatalyzedbyaldosereductase(refertoFig.116(f0035)),leadingtoproductionofthesugar
alcoholgalactitol(thesubstratederivative).Accumulationofgalactitolinthelensleadstoincreased
osmolarity,resultingincataractformationaswaterisdrawnintothelenstissue.

Note:Theclassicalexplanationforcataractformationindiabetesinvokesthesamemechanismof
osmoticdamage,withprolongedorrepetitivehyperglycemiaresultingindepositioninthelensof
sorbitol(thesugaralcoholcreatedbytheactivityofaldosereductaseuponglucose).
Anynewbornwithcataracts(especiallyifbilateral)shouldbeevaluatedforgalactokinasedeficiency,
classicgalactosemia,andothermetabolicdisorders.Othercausesofneonatalcataractsinclude
trisomies(i.e.,Down,Edwards,andPatausyndromes)andothergeneticdisordersaswellas
intrauterineinfections(i.e.,toxoplasmosis,otheragents,rubella,cytomegalovirus,herpessimplex
[TORCH]infections),particularlyrubella.

4 Whatisthetreatmentforclassicalgalactosemia?
Thepatientneedstofollowastrictdietthateliminatesallgalactosecontainingcompounds,
includinglactose(milkanddairyproducts)andgalactosecontainingsupplements.Recallthat
lactoseisthedisaccharideformedfromgalactoseandglucose.Avoidanceofdietarygalactoseintake
quicklyeliminatestheriskofdevelopingthecomplicationsofmentalretardation,poorgrowth,
jaundice,anemia,liverdisease,andcataractsandmayhelpreverseanycomplicationsalready
present.
BeabletorecognizethesignsandsymptomsofclassicgalactosemiafortheUSMLE.These
symptomsincludeInfantilecataracts,Jaundice,Failuretothrive,Hepatomegaly,andM
entalretardation.Aneasywaytorecalltheseistothinkofthemotherinthiscasetellingthe
doctorthatherbaby'ssymptomsappearedafterIJustFedHerMilk.
Step1Secret

Galactosemiaisduetoadefectingalactosemetabolism,usuallydeficiencyofgalactose
kinaseorgalactose1phosphateuridyltransferase(GALT),whichresultsinelevated
serumandurinegalactoselevels.
Thedisorderresultsinmilkintolerance,hypoglycemia,andpoorgrowthaswellas
accumulationofgalactosederivatives(galactitolandgalactose1phosphate)inneurons
(ultimatelyleadingtomentalretardation),inthekidneys(resultinginproteinuriaand
renaldysfunction),inredbloodcells(causinghemolyticanemiaandjaundice),inliver
(withabnormalitiesonliverfunctiontests[LFTs],hepatomegaly,liverdysfunction,and
worseningjaundice),andinlenstissue(causingcataracts).
Thetreatmentisavoidanceofdietarygalactose(includinglactose).
Thedifferentialdiagnosisforneonatalcataractsincludesgalactosemiaandother
metabolicdisorders,geneticdisorderssuchasDownsyndromeandothertrisomies,and
rubellaandotherintrauterine(i.e.,TORCH)infections.

SummaryBox:Galactosemia

Case119
Atall,slim14yearoldboyisreferredtothegeneticdisordersclinicbyhisophthalmologist.He
hasectopialentis(detachedlens),flatcorneas,andhypoplasticirides.Hismother,whowasa
tall,thinwoman,diedsuddenlywhilejogging.Anautopsyrevealedaorticdissectiontobeher
causeofdeath.Onphysicalexamination,theboyhasanarmspanheightratioof1.15(normal
<1.05),pectusexcavatum(sunkenchest),scoliosis,jointhypermobility,arachnodactyly(spider
fingers),andstretchmarksonhisshouldersandthighs.Hehaspositivethumbandwristsigns.
Anearlydiastolicdecrescendomurmurheardbestattheleftsternalborderisappreciatedon
cardiacexamination.

1 Whatisthediagnosis?
ThepatientlikelyhasMarfansyndrome,anautosomaldominantdisordercausedbymutationsofthe
fibrillingene.Fibrillinisaglycoproteinthatservesasascaffoldforelasticfibersaroundconnective
tissue.

2 Whydoesthispatientexhibitmultiplepathologicpresentationsintheocular,
skeletal,andcardiovascularsystems?
Fibrillinisacomponentofmicrofibrils,whicharepartoftheextracellularmatrixofeyetissue,aorta,
skin,andperiosteum.Microfibrilscombinewithelastintoformelasticfibers,whichconvey
structuralsupportandelasticitytomanytissues.Detachedormalpositionedlenses(ectopialentis),
aorticdilation,andabnormallystretchyskinareallpathologicconsequencesofdefectivefibrillin.
Progressiveaorticdilationoftenresultsinseparationoftheleafletsoftheaorticvalve,resultingin
aorticregurgitationandthecorrespondingdiastolicmurmur.Theskeletaldefectssuchas
arachnodactyly(long,skinny,spiderlikefingersandtoes),tallstature,pectusexcavatum,scoliosis,
andjointhypermobilityarecausedbytheperiosteum'sinabilitytoprovidethenormaloppositional
forcetobonegrowth.Overgrowthofboneoccurswhentheperiosteumhasbecometooflexibleasa
resultofdefectivefibrillin.Thethumbandwristsignsareusefulindemonstratingarachnodactyly.
Thethumbsignispositivewhenthethumb,whencompletelyenclosedwithinaclenchedfist,
protrudesbeyondthefist'sulnarborderthewristsignispositivewhenthethumbandpinkyoverlap
whenwrappedaroundtheoppositewrist.

3 WhatpathologicconditionwouldyoususpectinapatientwithMarfan
syndromewhocomplainsabouttheworstheadacheofmylife?
Ruptureofaberryaneurysmislikely.PatientswithMarfansyndromearepronetoberry(saccular)
aneurysms,whicharearterialaneurysmsthatoccuratbifurcationsinthecircleofWillis(themost
commonsiteisthebifurcationoftheanteriorcommunicatingartery).Thesejunctionsarecommon

sitesforaneurysmformationbecausetheylackinternalelasticlaminaandsmoothmuscle,thus
weakeningthem.BecauseMarfansyndromealreadyresultsinweakenedvessels,thesepatientsare
especiallysusceptibletoberryaneurysms.
Themostcommoncomplicationofberryaneurysmsisrupture,whichresultsinsubarachnoid
hemorrhage.Spinaltapwillrevealbloodoryellowfluid,whichresultsfromthebreakdownof
oxyhemoglobinintobilirubin.Bloodbreakdownproductscanleadtovasospasm,sopatientsshould
beplacedoncalciumchannelblockers.

4 Whatarethemajorcause(s)ofdeathinthisdisorder?
ThemajorcausesofdeathinpatientswithMarfansyndromeareprogressiveheartfailurefromaortic
regurgitationandsuddendeathfromaorticdissection.Pregnancyorheavyexerciseincreasescardiac
outputgreatly,thuselevatingtheriskofthesemajorcardiovascularcomplications.

Relatedquestions
5 Whathereditaryskeletaldiseasepredisposestobonefracturesfromminor
stress?Whatstructuralproteindefectunderliesthedisorder?
Osteogenesisimperfecta(OI),causedbydefectsintypeIcollagen,isaninheriteddisorderofbone
fragility.Themostcommonformofthedisorder(typeI)isautosomaldominant,but,interestingly,
mostcasesaresporadic(i.e.,duetoanewmutation).TypeIcollagenisthepredominantcollagen
typeintheextracellularmatrixofbone,butitisalsofoundinotherstructuressuchasteeth,sclerae,
ligaments,skin,andbloodvessels.TypeIOIistheleastsevereform,associatedwithfewerfractures
andlimitedskeletaldeformity.DistinctfeaturesofOIincludebluesclerae,hearingloss(from
involvementofmiddleearossicles),andabnormalteeth(i.e.,dentinogenesisimperfecta,discolored
teeththatweareasily).Thereducedcollagencontentofthescleraeallowsthedarkchoroidlayerof
theeyetoshowthroughthesclerae.Other,lessspecificfeaturesofOIincludeligamentouslaxityand
jointhypermobilitykyphoscoliosissmooth,thin,andlaxskinandeasybruising(fromvessel
fragility).

6 WhatisthegeneticdefectinEhlersDanlossyndrome?
EhlersDanlossyndromecandevelopfrommultipledifferentgeneticabnormalitiesinoneofthe
collagentypes.TherearemanytypesofEhlersDanlossyndrome,withmostcommonbeing
autosomaldominant.Mostformsarecharacterizedbyextremeskinhyperextensibilityandjoint
hypermobilitytissuefragilitypoorwoundhealingleadingtothin,wide,atrophicscarsandeasy
bruising.Themostcommonform(typeI)isduetoadefectincollagenV.Thereisavascularor
ecchymoticformofthedisorder(typeIV),whichisduetoadefectintypeIIIcollagen,whichisthe
majorcomponentofreticularfibersfoundinvariousorgansandfoundalongtheliningofblood
vessels.Thisformofdiseaseisimportanttodiagnosebecause,inadditiontoecchymoses,these
patientsarepronetocatastrophicruptureofmediumsizedarteries,bowel,oruterus(during
pregnancy).

Marfansyndromeisaninheritedautosomaldominantdefectinfibrillin,acomponentof
themicrofibrilsfoundwithinelasticfibers.
Marfanpatientsoftenexhibitskeletaldeformity,withtallthinstature,arachnodactyly
(withpositivewristandthumbsigns),pectusexcavatum,scoliosis,andjoint
hypermobilitystretchyskinectopialentis(detachedlens)andaorticdilation,aortic
valveregurgitation,andriskofdevelopingberryaneurysms,heartfailure,orsudden
deathfromaorticdissection.
OsteogenesisimperfectaisageneticdefectintypeIcollagen,resultinginbonefragility
andfrequentfracturesofvaryingseverity,oftenwithabnormalteeth,bluesclerae,and
hearingloss.Lessspecificfeaturesincludeligamentouslaxity,jointhypermobility,
kyphoscoliosis,laxskin,andeasybruising.
ThevarioustypesofEhlersDanlossyndromearedefects,generallyautosomal
dominant,invariouscollagentypes.Patientstypicallyexhibitextremeskin
hyperextensibility,jointhypermobility,tissuefragility,poorwoundhealingwithatrophic
scarring,andeasybruising.
SummaryBox:MarfanSyndromeandGeneticConnectiveTissueDefects

Case1110
A6yearoldboyisbroughttothehospitalbyhismotherbecauseofsevereshortnessofbreath.
Hehasfeverandleukocytosis,andachestxraystudyshowsarightlowerlobeinfiltrate.His
motherisreallyworriedbecausehehashadpneumoniaalreadythreetimesasachild.A
sputumculturegrowsPseudomonasaeruginosa.Asweattestshowssignificantlyelevated
sodiumandchloride.

1 Whatisthemostlikelydiagnosis?
Cysticfibrosis(CF)ismostlikely.

2 Whatistheetiologyofthisdisease?
ThedefectunderlyingCFisfoundintheCFTRgene,whichcodesforachloridechannelfoundin
exocrineglandsthroughoutthebody.Morespecifically,themutationassociatedwithCFresultsin
defectiveproteinfoldingofthechloridechannelsothatitisdegradedbeforeiteverreachesthe
surfaceofthecellmembrane.Thischannelallowsforactivereabsorptionofchloridefromsweatand
activesecretionofchlorideinthelungs,liver,andpancreas.AsdiscussedinCase112(st0085),CFis
inheritedinanautosomalrecessivemanner.

3 Howdoesthecysticfibrosistransmembraneregulator(CFTR)mutationlead
todisease?
Thelackofactivechloridesecretionnormallycarriedoutbythischannelpreventspassivesecretion
ofsodiumandwaterbecausewaternormallyflowsdownitsosmoticgradient.Theconsequenceis
abnormallythicksecretionsintherespiratorytract,pancreas,gastrointestinal(GI)tract,andbiliary
tree.

4 Whydoesthischilddeveloprespiratoryinfectionssoeasily?
InCF,theabnormallyviscoussecretionsofthetracheobronchialmucousglandsimpairmucociliary
actionandpredisposetorecurrentpulmonaryinfections.Inadditiontoincreasedsusceptibilityto
therespiratorypathogensseeninnormalhosts,patientswithCFfrequentlybecomecolonizedand
infectedwithorganismssuchasStaphylococcusaureus,P.aeruginosaandothergramnegative
rods,andBurkholderiacepacia.Repeatedpulmonaryinfectionsfrequentlyleadtoprogressivelung
disease,whichisthemajorcauseofCFrelateddeath.Chronicinfectionandinflammationofairways
damagetheairways,ultimatelyresultinginairwayobstruction,airtrapping,andhyperinflation,
whicharephysiologicchangessimilartothoseseeninchronicobstructivepulmonarydisease
(COPD).

5 Whymightthischildbesusceptibletodevelopingpancreatitisashegrows
older?
Thereisalsoincreasedviscosityofpancreaticexocrinesecretions,whichimpairspancreaticsecretion
andpredisposestopluggingandobstructionofthepancreaticductules.Suchpluggingcancause
pancreatitis.Mostcommonispancreaticinsufficiency,oftenresultinginfailuretothriveinchildren,
weightlossorpoorweightgain,andfatmalabsorption(oftenwithfrequent,floating,greasy,
malodorousstoolsandflatulence).Fatmalabsorptioncanleadtodeficienciesinthefatsoluble
vitaminsA,D,E,andK.
Abnormalsecretionsoccurinmultipleorgansandexocrineglandsthroughoutthebody.Other
possiblemanifestationsofthisincludehighratesofmeconiumileusatbirthandpoorintestinal
motilityinadults,aswellasimpairedbilesecretion(whichinsomecasescanleadtocholestaticliver
diseaseorevenobstructivecirrhosis).Inthesweatglands,lossofCFTRfunctionresultsinimpaired
reabsorptionofchloride,allowingforthediagnosisofCFonthebasisofelevatedsweatchloride
levels.

6 Whatisbronchiectasisandwhydoesitcommonlydevelopincysticfibrosis?
Ectasisorectasiameansdilation.Bronchiectasisisanirreversibledilationofoneormorebronchi.
Theprimarycauseforthedilationisusuallyacombinationofinflammationandobstruction(often
subtotalobstruction)oftheairways.InCF,repeatedinfectionandpoormucusflowcontributeto
inflammationandairwayobstruction.

Otherclinicalsyndromesinwhichbronchiectasisoccursincludeobstructingtumors,mucus
impaction,foreignbodyaspiration,necrotizingpulmonaryinfections(suchasKlebsiella,S.aureus
pneumoniaortuberculosis),orcertainrheumaticandsystemicdiseases(suchasrheumatoid
arthritis).Thedilatedbronchiareeasilycollapsible,whichcausesoraggravatesexpiratoryairflow
obstructionandimpairedsecretionclearance,resultingintheclinicalsymptomsofdyspnea,cough
(whichisoftensevere),andoccasionallyhemoptysis.
ExpecttoseeatleastonequestiononcysticfibrosisanditscomplicationsontheUSMLEStep
1.
Step1Secret

7 WhyisNacetylcysteineusedasatreatmentforcysticfibrosis?
Nacetylcysteinecleavesdisulfidebondsintheglycoproteinsthatformmucus.Thisreducesthe
thicknessofthemucusseeninpatientswithCFandloosensmucousplugs.Nacetylcysteineisalso
usedasanantidoteforacetaminophentoxicityandtopreventradiocontrastinducednephropathy.

Relatedquestions
8 WhydoesKartagener'ssyndromeproduceclinicalmanifestationssimilarto
thoseofcysticfibrosis?
InKartagener'ssyndrome(whichisalsoknownasprimaryciliarydyskinesiaorimmotilecilia
syndrome),thereisaprimarydisturbanceinciliaryfunctionandmucociliaryclearancedueto
dysfunctionofthedyneinarmofmicrotubules.Recallthatmicrotubulesarefoundwithinciliaofthe
upperrespiratoryepithelialcellsandflagellaofsperm.Thisdefectivemucociliaryclearance
compromisessputumexpectoration,leadingtorepeatedsinusandpulmonaryinfections.Patients
oftenhavechroniccough,andbronchiectasiscandevelopinsome.Italsoleadstoinfertilitydueto
spermdysmotility.
Interestingly,inCFciliarydysfunctionissecondarytoincreasedsecretionviscosity,whereasin
Kartagener'ssyndromeitresultsfromanintrinsicdefectinthemicrotubules.Inbothdiseasesthere
arerecurrentsinopulmonaryinfectionsaswellasinfertilityinmen.Notethattheinfertilitythat
occursineachdisorderhasadifferentcause.InfertilityinCFisnotduetoabnormalsecretionsbut,
instead,isusuallyduetocongenitalabsenceofthevasdeferens(presumablybecausetheCFTRgene
isinvolvedintheembryonicdevelopmentofthevas).ManywomenwithCFhavedifficulties
conceivingduetothickenedcervicalmucus.Malnutritionsecondarytoimpairedfatabsorptionmay
alsodisruptovulationinthesewomen.
Note:Situsinversusisobservedin50%ofpatientswithprimaryciliadyskinesia.Theclassic
definitionofKartagener'ssyndromeisthetriadofsinusitis,bronchiectasis,andsitusinversus.Situs
inversusischaracterizedbycompletelefttorightreversalofallorgansandvesselsofthebody.For
example,theheartwouldontherightsideofthethorax(i.e.,dextrocardia)andtheliverwouldbein

theleftupperquadrantoftheabdomen.Thisisdistinctfromisolateddextrocardia,whichcanoccur
inthesettingofcongenitalheartdisease.Itisfeltthatsitusinversusoccursinciliarydyskinesia
becausemicrotubulesareinvolvedinthenormalpolarizationofthebodythatoccursearlyin
embryogenesis.Withoutfunctionalmicrotubules,thepolarizationofthebodyoccursrandomlysuch
that50%ofpatientsexhibitnormalpolarizationand50%exhibitcompletereversal.

Cysticfibrosis(CF)iscausedbyaninherited,autosomalrecessivedefectintheCFTR
(cysticfibrosistransmembraneregulator)chloridechannelthatresultsinabnormally
thicksecretionsofexocrineglands.
CFcanbediagnosedwithgenetictestingorbydocumentinganelevatedsweatchloride
level.
Thickrespiratorysecretionsandpoormucociliaryclearanceleadtorepeatedrespiratory
infectionsbycharacteristicorganismssuchasStaphylococcusaureus,Pseudomonas
aeruginosa,andBurkholderiacepacia.Theseinfectionsleadtoprogressive,obstructive
lungdisease,bronchiectasis,andsignificantratesofmorbidityandmortality.
PatientswithCFcandeveloppancreatitisandpancreaticinsufficiencyand,less
commonly,meconiumileusatbirthorpoorintestinalmotilityinadultsaswellas
cholestaticliverdisease.
InfertilityinCFresultsfromcongenitalabsenceofthevasdeferens.
NacetylcysteineisusedasatreatmentforCFtoloosenmucousplugs.
Kartagenerssyndrome(orprimaryciliarydyskinesia)isclassicallycharacterizedbythe
triadofsinusitis,bronchiectasis,andsitusinversus.Itisduetoanintrinsicdefectof
ciliaryfunction(dysfunctionofthedyneinarmofmicrotubules).Infertilityresults
secondarytoimmotilesperm.
SummaryBox:CysticFibrosisandDisordersofCiliaryDysfunction

Case1111
Whileplayingbasketballovertheweekend,a35yearoldcollegeprofessorexperiencessevere
chestpainthatradiatestohisleftjawandleftarm.Heistakenbyambulancetotheemergency
room,whereanelectrocardiogramconfirmsthathehassufferedamyocardialinfarction(MI).
Physicalexaminationissignificantforthepresenceofxanthelasmaandseveraltendinous
xanthomas.Hisfamilyhistoryisremarkableforcoronaryheartdiseaseintwofirstdegree
relativesdiagnosedwhentheywereintheirmid40s.Bloodworkrevealsplasmatotal
cholesterolof420mg/dL(Table117(t0040)),andacoronaryangiogramreveals90%

occlusionoftherightcoronaryarteryand50%occlusionoftheleftanteriordescendingartery.
Herespondswelltocoronaryangioplastyandstentingandisdischargedhomeafteronly3
days.

1 Whatisyourdiagnosisbasedonthispatient'sfamilyhistoryandfastinglipid
profile?
Hemostlikelyhasfamilialhypercholesterolemia(FH).HisMIatayoungage,hisfamilyhistoryof
prematurecardiovasculardisease,hiselevatedtotalcholesterolalmostentirelyintheLDLfraction,
hisnormaltriacylglycerol(triglyceride)levels,andthepresenceofxanthelasmaandtendinous
xanthomasonexaminationareallconsistentwithFH.Xanthomasarepathologicdepositionsof
lipidsinnonvasculartissues.InheterozygousFH,xanthomasareoftendetectableaspalpable
nodulesalongtendonssuchastheAchillesorextensortendonsofthehand.Xanthelasmasare
yelloworangeplaquesontheeyelidsandmedialcanthithatresultfromcholesteroldepositioninto
theseskinfoldsaroundtheeye.
FHiscausedbyadefectiveLDLreceptor.Thisreceptorisfoundonperipheraltissuesthatutilize
lipidsfoundinLDLparticles,andontheliver,whichplaysanimportantroleintheclearanceof
cholesterolfromtheblood.

2 Basedonhisclinicalpresentations,isthispatientlikelytobeheterozygousor
homozygousforthisdeficiency?
Thispatientislikelyaheterozygote.FHisusuallyinheritedinanautosomaldominantfashionin
whichasingleabnormalLDLreceptorgeneleadstodisease.However,thereisagenedosageeffect
inthatpatientswhoarehomozygousfortheabnormalLDLreceptorhavemoreseveredisease.
Homozygotestypicallysuffertheirfirstheartattackbeforeage20,andtheirtotalcholesterollevels
usuallyrangebetween500and1000mg/dL.Prominentpresenceofcutaneousplanarxanthomasis
anothertelltalesignofhomozygousFH.Heterozygotestypicallyhavecholesterollevelsbetween300
and500mg/dL,andtheydonotsufferheartattacksuntillaterinlife.Occasionallyafewxanthomas
canbefoundontheAchillestendoninheterozygousFH.

3 Whatistheprimarymechanismbywhichmutationsinthelowdensity
lipoproteinreceptorimpairlowdensitylipoproteinuptakefromtheplasma?
Physiologically,serumLDLiscontinuallymadefromIDL(intermediatedensitylipoprotein),which
inturnisderivedfromVLDLdegradation.About75%ofLDLisclearedviaLDLreceptormediated
endocytosis,whichoccursprimarilyintheliverand,toalesserdegree,incholesterolrequiring
tissuessuchastheadrenalcortex(wherecholesterolisusedasasubstrateforsteroidhormones).
TheremainderofLDLisclearedviapoorlyunderstoodLDLreceptorindependentmechanisms,
whichinclude,amongothers,endocytosisofoxidizedLDLbymacrophages.
Note:FiveclassesofmutationsoftheLDLreceptorresultinimpairmentofreceptormediated
uptakeofLDLfromthecirculation.Thesemutationsinclude(1)nullmutations(whichpreventLDL

receptorproteinsynthesis),(2)defectivetransportmutations(whichpreventnormalinsertionofthe
receptorintotheplasmamembrane),abnormalclathrinmediatedendocytosisdueto(3)defective
ligandreceptorbindingmutations,or(4)defectiveinternalizationmutations,and(5)defective
recyclingmutations(whichresultinaninabilityoftheLDLreceptortoberecycledbacktothe
membrane).

4 Whatisthepathologicconsequenceofelevatedplasmalowdensity
lipoprotein?
ThemechanismbywhichexcessplasmaLDLcanresultinatherosclerosisisbrieflydescribednext:
InthesettingofadefectiveLDLreceptororofexcessserumLDLofanycause,plasmaLDLclearance
occursviaothermechanisms,includingscavengerreceptorsonmacrophagesthatscavenge
(endocytose)oxidizedLDL.OxidizedLDLlevelsareproportionaltothelevelsofLDL,butLDLlevels
maybeincreasedbyothercardiacriskfactorssuchassmokingorsystemicinflammation.These
macrophagesthenformfoamcellsandreleasecytokines,causingproliferationofarterialsmooth
musclecellsatthesiteofcholesterolplaqueformation.Initially,thesmoothmusclecellsproduce
enoughextracellularmatrixproteinstoformafibrouscapoverthefoamcells,formingastable
fibrousplaque.However,becausethescavengerreceptorsarenotdownregulatedbyintracellular
cholesterolconcentration,thefoamcellscontinuetoaccumulate,toendocytoseoxidizedLDL,andto
releaseinflammatorymediators.Insomecases,theinflammatoryprocesserodesthefibrouscap,
resultingintheformationofathinlycoveredunstableplaquethatcaneventuallyrupture,resulting
inacutethrombusformation.Itistheprocessofunstableplaquerupturethatunderliesmanystrokes
andmostacutecoronarysyndromes(i.e.,unstableanginaandMI).Progressiveluminalocclusionby
growingstablefibrousplaquescanresultinchronicischemicsymptomssuchasstableanginaor
claudication.

5 Howmightthispatientbemanagedmedicallyandpharmacologicallyto
decreasehisriskoffuturecardiovascularcomplications?
Thispatientshouldbemanagedthesameasanyotherpatientwithcardiovasculardiseaseand
associatedriskfactors.Hewouldbeencouragedtoundertakedietaryandlifestylemodification,such
asincreasedexerciseandsmokingcessation,andwouldlikelybetreatedpharmacologicallywith
HMGCoA(3hydroxy3methylglutarylcoenzymeA)reductaseinhibitors(i.e.,statins),possiblyin
combinationwithotherantilipiddrugssuchasbilesequestrants(suchascholestyramine)or
ezetimibe(aninhibitorofdietarycholesteroluptake).
Alongwithstatins,aspirin,betablockers,andangiotensinconvertingenzyme(ACE)inhibitorshave
alsobeenprovedtobeeffectiveinthesecondarypreventionofMI(i.e.,inpatientswithahistoryof
MI,theydecreasetheriskofanotherMI).

Relatedquestion
6 HowdoHMGCoAreductaseinhibitorsspecificallyreduceserumlowdensity
lipoproteinlevels?

HMGCoAreductaseistheratelimitingenzymeincholesterolsynthesis.StatinslowerLDLlevelsby
decreasingintracellularlevelsofcholesterolwithinhepatocytes.Statinsdosobydecreasingdenovo
cholesterolsynthesis.Hepatocytesandothertissuesrequirefreecytosoliccholesterolforsynthesisof
theplasmamembrane,steroidhormones,andbileacids.Thehepatocytesthuscompensateforthe
decreasedcytosoliccholesterollevelsbyincreasingexpressionoftheirsurfaceLDLreceptors(the
samereceptorsthataremutatedinFH)inanattempttoreplenishthecytosoliccholesterolsupply.
Bileacidsequestrantssuchascholestyramineorcolestipolactinasimilarfashion.Thelossofbile
acidsinthegutresultsinacompensatoryincreaseinbileacidsynthesis,whichtendstoconsume
cytosolicfreecholesterolwithinhepatocytes.Again,thisresultsinacompensatoryincreasein
hepatocyteexpressionofsurfaceLDLreceptors.

Familialhypercholesterolemia(FH)isduetoadefectinthelowdensitylipoprotein
(LDL)receptor.Patientsexhibitelevationoftotalcholesteroland,specifically,ofLDL
withnormaltriglyceridelevelssignsofpathologiclipiddeposition,suchasxanthelasma
andtendinousxanthomasandapersonalorfamilyhistoryofprematureatherosclerosis
orcoronaryarterydisease.
HighlevelsofLDLofanycauseresultinincreasedclearanceofLDLviascavenger
receptorsonmacrophagesthatrecognizeoxidizedLDL.Thesemacrophagesaccumulate
asfoamcellswithinatheroscleroticplaques,andtheyreleaseinflammatorymediators
thatdrivesmoothmusclecellproliferationandfibrouscapformation.Ultimately,
inflammationwithinplaquescanleadtofibrouscaperosionandtheformationofthe
unstableplaquesthatunderlieacutecoronarysyndromes.
Statins(i.e.,HMGCoAreductaseinhibitors)inhibittheratelimitingstepofcholesterol
synthesisintheliver.Lowcytosoliccholesterollevelswithinhepatocytesleadto
compensatoryupregulationofsurfaceLDLreceptorsandenhancedclearanceofLDL
fromplasma.
SummaryBox:FamilialHypercholesterolemiaandLipoproteinMetabolism

Case1112
A15yearoldboyissenttoadevelopmentalpediatricclinicforevaluationofintellectualdelay
andhyperactivity.Hisexaminationisnotableforanunusuallylongfacewithaprominentjaw
andlargeearsandlargetestes(macroorchidism).His17yearoldsisterwasdiagnosedwith
mildmentalretardationandalearningdisabilitylastyear.Hisfamilyhistoryisalsonotablefor
learningdisabilitiesinamaternalauntdiagnosedwhenshewasyoungandamaternalgreat
uncle.Hismotherandmaternalgrandfather,however,bothhadnormalintellect.Neitherhis
fathernoranyofhispaternalrelativeshadlearningdisabilitiesorretardation.

1 Whatisthemostlikelydiagnosis?
ThischildlikelyhasfragileXsyndrome,anXlinkeddominantdisorder.Itisassociatedwithmental
retardationandautisticspectrumbehaviorssuchasattentiondeficit/hyperactivitydisorder,
perseveration,andsocialavoidance.Thecharacteristicphysicalfeaturessuchaslongface,
protrudingjaw(prognathism),andlargetestesusuallybecomeapparentaroundpuberty.
FragileXsyndromeisthemostcommonformofinheritedmentalretardation.Amonggenetic
disorders,itisthesecondmostcommoncauseofmentalretardation(afterDownsyndrome,which
canbeinheritedbutusuallyisnot).

2 Whatisthepathogenesisofthisdisorder?
FragileXiscausedbyexpansionofatrinucleotiderepeatintheFMR1(familialmentalretardation1)
gene.TheFMR1genenormallycontainsalimitednumberofrepeats(lessthan40).However,
patientswithfragileXsyndromehaveanexpandednumberofrepeats(usuallyover200).The
expansionresultsinaproportionaldecreaseinexpressionofthefragileXmentalretardation
(FXMR)protein.
Thedecreasedexpressionoccursbecausetherepeatregionisasiteofgenemethylation.Recallthat
genemethylationisanormalregulatorymechanismforsilencinggeneexpression.Inthiscase,
expansionoftherepeatregionresultsinhypermethylationofFMR1andabnormallylowlevelsof
expression.
Thediseasepathogenesisispoorlyunderstood.TheFXMRproteinisthoughttoplayanimportant
roleintransportingnuclearmRNAfromthenucleusintothecytoplasmpriortotranslation,butitis
notclearhowdeficientlevelsofFXMRproteinresultinclinicaldisease.However,diseaseseverity
clearlycorrelatesinverselywithproteinexpressionlevels.

3 Southernblotanalysisindicatesthathismotherhas90CGGtrinucleotide
repeats,whereashehas350CGGrepeats.Whataccountsforthisfindingand
whatisitsclinicalsignificance?
FragileXsyndromeandothertrinucleotiderepeatdisordersexhibitgeneticanticipationdueto
germlineexpansionofthetrinucleotiderepeat.
ThenormalunexpandedFMR1geneistransmittedinastablefashion.However,anexpandedFMR1
gene(withover40repeats)isunstableandissubjecttofurtherexpansion.Furthermore,thelonger
thegene,themoreunstableitis.However,inthecaseoftheFMR1gene,thisexpansionoccurs
duringoogenesisbutnotspermatogenesis.Inotherwords,instabilityoccursonlyinmeiosisin
women.TheresultisthatafemalecarrierofanexpandedfragileXgenewilloftentransmitanFMR1
genethatisfurtherexpanded.Asmentionedearlier,longerrepeatregionsresultinmoresevere
proteindeficiencyandmoreseveredisease.Asaresult,subsequentgenerationsexperiencemore
severesymptomsandearlieronsetdisease,aphenomenonreferredtoasgeneticanticipation.
Geneticanticipationisuncommoningeneticdisordersandusuallyresultsfromexpansionof
trinucleotiderepeats(Fig.118(f0045)).

Figure118
Anticipationduetoexpansionoftrinucleotiderepeats(amplification).
(FromBrownTA,BrownD:USMLEStep1Secrets.Philadelphia,Hanley&Belfus,2004.)

Note:BecausethedisorderisXlinkeddominant,femaleswithonenormalFMR1geneandone
mutated(expanded)genewilloftenhavesomediseasefeatures.However,thenormalfunctioning
FMR1geneusuallyattenuatesthediseasetosomeextent,suchthatfemalesusuallyhavemilder
intellectualimpairmentratherthansevereretardation,eveninthesettingofafullmutation(i.e.,
>200repeats).
Thesetwofeatures,geneticanticipationandXlinkeddominance,resultinthesignificantbut
predictablevariabilityofdiseaseseverityseeninfamilieswithfragileXsyndrome.

Relatedquestion
4 Whatotherrelativelycommoninheriteddisorderexhibitsgenetic
anticipation?
Huntington'sdiseaseisalsoduetoatrinucleotiderepeatexpansionandexhibitsgeneticanticipation
becauseofprogressiveexpansionofthetrinucleotiderepeatinsubsequentgenerations.Thedisorder
ischaracterizedbythetriadofmotordysfunctionwithchoreoathetosis,behaviorandpersonality
changes(includingaggressionanddepression),anddementia.Thedisorderisoftensuspectedonthe
basisoffamilyhistoryandfindingsfromheadcomputedtomography(CT)ormagneticresonance
imaging(MRI),specifically,prominentatrophyofthecaudatenucleus.Thediagnosiscanbe
confirmedwithgenetictesting.
Huntington'sdiseaseisthemostcommonofagroupofinheritedneurodegenerativedisorders(so
calledCAGrepeatdisorders)which,likefragileX,allexhibitgeneticanticipationduetoexpansionof
atrinucleotiderepeat.However,unlikefragileX,theyshareautosomaldominantinheritance.Also
unlikefragileX,thegermlineexpansionoccursinbothmalesandfemales(i.e.,inbothoogenesis
andspermatogenesis),asonemightexpectforautosomaldisorders.InthecaseofHuntington's
disease,expansionoftheCAGrepeatresultsanabnormallylongHuntingtonproteinwithinneurons
andothertissues.Thiselongatedproteinexhibitsatoxicgainoffunction,suchthatanormalgene
copyisunabletomaskamutatedgene.

FragileXsyndromeisanXlinkeddominantdisordercausedbyexpansionofaregionof
trinucleotiderepeatswithinthefamilialmentalretardation(FMR1)gene.Theexpansion
leadstogenehypermethylationandabnormalsilencingofgeneexpression.

FragileXisthemostcommoncauseofinheritedmentalretardation.
FragileXisclinicallycharacterizedbymentalretardationandautisticlikebehaviors
(inattentiveness,hyperactivity,socialavoidance,etc.)aswellascharacteristicphysical
features,includingprotrudingjaw(prognathism),longface,andmacroorchidism(large
testes),thatappeararoundpuberty.
FragileXexhibitsgeneticanticipation,whichistheprocessbywhichsubsequent
generationsexperiencemoreseveresymptomsorearlieronsetdiseaseduetogermline
expansionofthetrinucleotiderepeat.
Huntington'sdiseaseischaracterizedbythetriadofchoreoathetosis,behaviorand
personalitychanges(includingaggressionanddepression),anddementia.Pathologically,
itisassociatedwithprominentatrophyofthecaudatenucleus.Itbelongstoagroupof
neurodegenerativeCAGrepeatdisordersthat,likefragileX,exhibitgeneticanticipation
duetotrinucleotiderepeatexpansion.BecausetheCAGrepeatsresultintoxicgainof
proteinfunction,thesedisordersexhibitautosomaldominantinheritance.
SummaryBox:FragileXSyndromeandTrinucleotideRepeatDisorders

Case1113
Anewbornboy,borntoa42yearoldwoman,presentswithupslantingpalpebralfissures,
excessskinoftheinnereyelid(epicanthalfolds)andatthebackoftheneck,aflattened
maxillaryandmalarregion,andasinglepalmarcrease(simiancrease).Thechildalsoexhibits
poormuscletone.Cytologictestingrevealsanabnormalkaryotype.

1 Whatisthemostlikelydiagnosis?
ThechildhasDownsyndrome,ortrisomy21,ananeuploidconditionthatleadstothemostcommon
geneticcauseofmentalretardation.Aneuploidyreferstoanabnormalchromosomenumber(thatis
notamultipleof23),whichistypicallyduetoeitheramissingchromosomeoranextrachromosome
(i.e.,monosomyortrisomy,respectively).Generallyspeaking,maternalageolderthan35(socalled
advancedmaternalage)greatlyincreasestheincidencerateofDownsyndromeandother
chromosomalabnormalities.
NotethatalthoughDownsyndromeisthemostcommoncauseofcongenitalmentalretardation,itis
notthemostcommonhereditarycauseofmentalretardation,becausemostcasesofDownsyndrome
areduetomeioticnondisjunctionduringoocytedevelopment(duringmeiosisIofoogenesis).A
smallerpercentageofcasescanbeattributedtorobertsoniantranslocationandmosaicism
AlsonotethatalthoughadvancedmaternalageisanimportantriskfactorforDownsyndrome,most
childrenwithDownsyndromeareborntomothersyoungerthan35yearsofage.Ononehand,this
maysimplyreflectthefactthatwomenunder35yearsoldbecomepregnantmoreoftenthanwomen

overtheageof35.Ontheotherhand,itillustratesthaterrorsofmeiosisarefairlycommonin
youngerandolderwomenalike.NondisjunctioneventscanresultinviablestatessuchasDown
syndrome,buttheyoftenresultinnonviablestates.Infact,chromosomalabnormalitiesaccountfor
abouthalfofallfirsttrimesterspontaneousabortions.

2 Whatisarobertsoniantranslocationandhowdoesarobertsonian
translocationinaparentresultinDownsyndromeintheoffspring?
Arobertsoniantranslocationcausestrisomy21viaadifferentmechanismthanthemeiotic
nondisjunctioneventsthatgiverisetoamajorityofDownsyndromecases.Itresultsinafamilial
formofDownsyndromethatisunrelatedtomaternalageandaccountsfor2%to4%ofallcases.
Arobertsoniantranslocationiscausedbyatranslocationeventinthegermlineofeitherparent.
Duringmeiosis,thelongarmofchromosome21mayattachtothelongarmofanotherchromosome,
usuallychromosome14(withlossofthenegligibleshortarmsofbothchromosomes).Thiscanresult
intheoffspringhavingtheequivalentoftrisomy21,withtwonormalcopiesofchromosome21and
anadditionalcopyofchromosome21fusedtochromosome14.
Asecondpossibleoutcomeisabalancedtranslocation,whichresultsinaphenotypicallynormal
individualwhohasasinglenormalcopyofeachofchromosomes14and21aswellthetranslocation
productcontainingthefusedlongarmsofchromosomes14and21.Anypatientwitharobertsonian
translocation,regardlessofwhetherthatpatientisaffectedbyDownsyndromeduetofunctional
trisomy21orwhetherthepatientappearsnormalduetothepresenceofabalancedtranslocation,
hasachanceoftransmittingDownsyndrometooffspring.

3 WhatisthemechanismthatgivesrisetomosaicDownsyndrome,whereby
onlyselecttissuesexpressthetrisomy21?
MosaicDownsyndrome(whichaccountsfor13%ofcases)iscausedbyamitoticnondisjunction
eventthatoccursearlyinembryonicdevelopment.Thus,mosaicDownsyndromeisentirely
independentofmaternalevents.Thiscontrastswiththetypicalmeioticnondisjunctionevent
responsibleforamajorityofDownsyndromecases(andwiththerobertsoniantranslocation,which
isalsoduetoanerrorinmeiosis).Asonewouldexpect,theearlierduringfetallifethatthe
nondisjunctionoccurs,themoretissuesthatareaffectedandthegreatertheseverityofdisease.

4 Ifthechildweretofailtoinitiateproperfeeding,whatabnormalityshouldbe
suspected?
ChildrenwithDownsyndromeareatriskforGItractabnormalitiessuchascongenitalbowel
obstructioncausedbyduodenalatresia.Duodenalatresiaiscausedbyfailureofnormal
recanalizationoftheGItractduringfetaldevelopment.Itcanbesuspectedonthebasisofthe
doublebubblesignonabdominalradiographscausedbygaseousdistentionofboththestomach
andduodenumproximaltotheatreticsegment.PatientswithDownsyndromearealsoatriskfor
otherGIabnormalitiessuchasimperforateanusorHirschsprungdisease(aganglionicmegacolon).
AnnularpancreatitisisalsomorecommoninpatientswithDownsyndrome.

5 WhatcausesmostofthedeathsininfancyandinchildhoodinDown
syndrome?
PatientswithDownsyndromeareatincreasedriskforvariouscongenitalheartdefects,whicharethe
majorcauseofearlydeath.
Themostcommoncardiacabnormalitiesareendocardialcushiondefects,whichincludedefectsof
theatrioventricularcanal(i.e.,atrioventricularvalveabnormalitiesandatrialandventricularseptal
defects).PatentductusarteriosusandtetralogyofFallotcanalsooccur.

6 WhichcancertypesareassociatedwithDownsyndrome?
PatientswithDownsyndromeareatsubstantiallyincreasedriskfordevelopingacuteleukemia(both
acutelymphoblasticand,toalesserextent,acutemyelogenousleukemias).

7 Whatcharacteristicneuropathologicchangesareseeninbrainsofolder
people(i.e.,>yearsofage)withDownsyndrome?
AlmostallpatientswithDownsyndromeinthisagegroupdevelopthecharacteristicmanifestations
ofAlzheimer'sdisease,includingneurofibrillarytanglesandamyloiddepositsinbraintissue.This
predispositiontoearlyonsetAlzheimer'sdiseasemayrelatetothefactthatthegenefortheprotein
foundintheamyloiddeposits(amyloidprecursorprotein[APP])isfoundonchromosome21.
ThecomplicationsofDownsyndromearefrequentlytestedontheUSMLEStep1.
Step1Secret

Relatedquestions
8 Whatothertwoautosomaltrisomiescansometimesproduceliveborn
infants?
Trisomy18(Edwardssyndrome)andtrisomy13(Patausyndrome)arebothviableconditions.
However,bothtrisomiesproducemoreseverediseasethanintrisomy21,causingdeathwithinthe
firstfewmonthsoryearsoflife.AswithDownsyndrome,mostcasesresultfrommaternal
nondisjunctionduringmeiosisIoftherespectivechromosomepair.
BothEdwardssyndromeandPatausyndromeoftenresultinrockerbottomfeet,cardiacdefects,and
renaldefects.AnimportantdistinctionisthatEdwardssyndromeoftencausesmicrognathia(small
lowerjaw),whereasPatausyndromeresultsinmicrophthalmia(smalleyes)andcleftlipandpalate.

9 Whattwodiseasesresultfrommicrodeletionofthesamesectionof
chromosome15?
AngelmansyndromeandPraderWillisyndrome.

10 Whyistheparentalsourceofthechromosomesignificantinthe
aforementionedmicrodeletionsyndromes?

Ithadoncebeenacentraldogmaofgeneticsthatphenotypeisthesamewhetheragivenalleleis
fromapaternaloramaternalsource.Likemostdogmas,however,thisideahasnotstoodthetestof
timeanditappearsthattheexpressionofaselectnumberofgenesdependsonwhethertheyare
maternallyderivedorpaternallyderived.Thisphenomenonofgeneexpressionbasedonparental
originisknownasgenomicimprinting.Genomicimprintingisfelttobeduetodifferential
methylationofchromosomeregionsinfemaleandinmalegonads,resultingindifferentpatternsof
genesilencingdependingontheparentalsourceofthechromosomes.
Angelmansyndromeresultsfromamicrodeletioninthematernallyderivedchromosome15,leading
toalossoffunctionofmanygeneslocatedinthatregion(Fig.119(f0050)).Patientswiththisdisease
oftenexhibituncontrollableandinappropriatelaughter(happypuppetsyndrome),aswellas
mentalretardation,ataxia,andseizures.

Figure119
MaternalmicrodeletioninAngelmansyndrome.
(FromBrownTA,BrownD:USMLEStep1Secrets.Philadelphia,Hanley&Belfus,2004.)

PraderWillisyndromeisusuallyduetoamicrodeletionofthepaternallyderivedchromosome15.
Patientsoftenexhibituncontrolledappetite,frequentlyresultinginobesity,aswellasmental
retardation,hypogonadism,hypotonia,andbehavioralproblems.

Downsyndrome(trisomy21)isusuallyduetonondisjunctionofchromosome21
occurringduringmeiosisIofoogenesis.Suchmeioticnondisjunctioneventsoccurmore
commonlyaswomenage.
Downsyndromeislesscommonlyduetoarobertsoniantranslocation,usuallyinvolving
chromosomes14and21.Itresultsinafamilial(i.e.,inherited)formoftrisomy21that
accountsfor2%to4%ofDownsyndromecases.
MosaicDownsyndrome,accountingfor1%to3%ofcases,istheleastcommonformof
trisomy21.Itresultsfromamitoticnondisjunctioneventearlyinembryonic
development.
ThecharacteristicphysicalfindingsinDownsyndromeincludeupslantingpalpebral

fissures,exaggeratedepicanthalfolds,singlepalmar(simian)crease,andhypotonia.
ComplicationsofDownsyndromeincludegastrointestinal(GI)tractanomalies(suchas
duodenalatresia,imperforateanus,orHirschsprungdisease),congenitalheartdisease
(usuallyendocardialcushiondefectssuchasdefectsoftheatrioventricularvalvesor
septa),earlyonsetAlzheimer'sdisease,andincreasedriskofacuteleukemia.
Edwardssyndrome(trisomy18)andPatausyndrome(trisomy13)arethetwoother
viabletrisomies.Theyarebothassociatedwithrockerbottomfeet,cardiacdefects,and
renaldefects.Edwardssyndrome,however,isassociatedwithmicrognathia(smalljaw),
whereasPatausyndromeisassociatedwithmicrophthalmia(smalleyes)andcleftlipand
palate.
AngelmansyndromeandPraderWillisyndromeresultfromamicrodeletionof
chromosome15Angelmansyndromeresultsfrommicrodeletionofthematernally
derivedchromosome,whereasPraderWillisyndromeresultswhenthemicrodeletionis
paternallyinherited.
Thedifferentialexpressionofparentallyderivedgenes,aphenomenonreferredtoas
genomicimprinting,isthoughttobeduetodifferentialmethylation.
SummaryBox:DownSyndrome,ChromosomalDisorders,andGenomicImprinting

Case1114
A54yearoldmalesmokerissenttotheemergencyroomwithcomplaintsofseverechestpain.
SerialcardiacenzymelevelsshownoevidenceofMIafter24hours,butanangiogramis
performedandrevealssevereatheroscleroticcoronaryarterydisease.Atriplecoronarybypass
isthenperformed.Postoperatively,heremainshospitalizedfor2weeksbecausethehealing
ofhissternalwoundisprogressingslowly.Hehasbeeneatinganormaldiet,thoughhis
appetitehasbeensomewhatpoor.Onexamination,hehasobviousskinpeelingontheback
andbuttocksandpittingedema.Hishairislightcoloredatthebaseandcanbeplucked
painlessly.Laboratoryfindingsareremarkableforserumalbuminof1.9g/dL(normalrangeis
3.45.0g/dL).Hisdailyurinaryureaexcretionisabout15g/day(normalislessthan5g/day).

1 Whatisthe(noncardiac)diagnosisinthispatient?
Thispatienthaskwashiorkor,whichresultsfrominadequateproteinintakedespitefairoroverall
adequateoverallcaloricintake.Itisamaladaptivestatecausedbyproteincaloriemalnourishmentor
severelyincreasedcatabolicrate.Indevelopednations,itistypicallycausedbyacute,lifethreatening
illnessessuchastraumaandsepsis.Itisalsoseenaftermajorsurgicaloperations.
Indevelopingnations,kwashiorkorusuallyoccursinyoungchildrenwhoarebeingweanedoffbreast
milk(agoodproteinsource)andstartedonahighcarbohydrate,verylowproteindiet.

2 Whatisthereasonforhiselevatedurinaryureanitrogenexcretion?
Criticalillnessdramaticallyelevatesone'scalorieand,evenmoreso,one'sproteinrequirements.
Thus,despitefairlynormalintake,hisbodyisinahypercatabolicstateandisbreakingdown
endogenousproteininanattempttosupplyhisliverwithprecursorsforgluconeogenesis.Because
ureaisformedexclusivelyfromaminoacidmetabolismandisexclusivelyexcretedbythekidneys,
elevatedproteincatabolismresultsinaproportionalincreaseinbothureasynthesisandurea
excretion.
Patientswithcriticalillnessshouldhavetheseincreasedproteinrequirementsprovided.Asmany
intensivecareunit(ICU)patientsareintubatedandunabletoeat,theproteinoftenmustbegivenin
theformofaminoacidorproteinenrichedtotalparenteralnutritionorentericfeeds(i.e.,tube
feeds).

3 Whatisthereasonforthelowserumalbuminlevel?
Thispatient'sdramaticserumalbumindecreaseisaresultofcatabolismofalbumin(amida
generalizedincreaseinproteincatabolism)causedbytheseverestress(bypasssurgery)and,toa
lesserextent,bypoorfeeding.Additionally,inflammatorymediatorsmayalsoreducehepatic
albuminproduction.Hepaticalbuminproductionisunabletocompensateforthedegreeof
catabolism.Thelowalbuminlevelcontributessignificantlytohisedema.
Thisedemaistheprimaryclinicaldistinctionbetweenkwashiorkorandmarasmus(inwhichedema
doesnotoccur).Patientswithkwashiorkoroftenhaveanasarca,severediffuseedemaoftheentire
body,duetolowoncoticpressure.Theedemaaffectsthefaceandperiorbitalarea,producinga
roundedfacewithpuffyeyes.Patients,particularlychildren,alsooftenhaveacharacteristic
abdominaldistention,orpotbelly,duetoacombinationofasciticfluidaccumulationand
hepatomegaly(fromfattyinfiltration).Othercharacteristicfeaturesincludedryatrophic,peeling,
hyperkeratoticskinanddry,hypopigmentedhairthateitherspontaneouslyfallsoutoriseasily
pluckedout.Thedecreasedhairpigmentationresultsfromalackofmelanin,whichisderivedfrom
aminoacids(specificallytyrosine,aderivativeoftheessentialaminoacidphenylalanine).Thehair
hypopigmentation,afterrestorationofadequateproteinintake,willresultinthesocalledflagsign
withinterspersedbandsofnormalpigmentationandhypopigmentation.

4 Wouldyouexpectthispatienttohaveanobviouslyemaciatedappearanceand
musclewasting?
No.Infact,itistypicallynotedthatpeoplewithkwashiorkorusuallyhaverelativelypreserved
peripheralmusclemasswithincreasedfatreserves.Themostdramaticdeficitisthelossofvisceral
(i.e.,organassociated)protein,whichismoredifficulttoappreciate.Theanasarcaalsoprevents
patientsfromappearingemaciated.Thislackofemaciationcanbeverydeceivingbecauseitwill
oftenmasktheseverityofapatient'smalnutrition.
Thesefindingsareincontrastwiththoseinmarasmus,inwhichthereisseveremusclewastingand
broomstickextremitiesasaresultoftotalcaloriemalnutrition.

5 Whataresomepotentialdangersifthisconditionislefttreated?
Kwashiorkor,inadditiontotheforegoingdescription,resultsinpoorwoundhealingandimpaired
immunefunction.This,inturn,resultsinincreasedsusceptibilitytoinfections,particularlysepsis,
pneumonia,andgastroenteritis.

6 Compareandcontrastmarasmusandkwashiorkor
Table118(t0045)comparesthefeaturesofmarasmusandkwashiorkor.

Kwashiorkorresultsfrominadequateproteinintakedespitefairoradequateoverall
calorieintake.Itresultsfromlackofproteinintakeorseverelyincreasedcatabolicrate(as
incriticalillnessoraftermajorsurgery).
Excessproteincatabolismcanbedetectedbymeasuringurinaryureanitrogen
excretion.
Kwashiorkorischaracterizedbyseverehypoalbuminemiaedema,oftenwithascitesor
anasarcahepatomegalyfromfattyinfiltrationabdominaldistentionorpotbelly
appearanceatrophicscalyskinandhypopigmented,easilypluckedhair.Incontrastwith
marasmus,thereisrelativepreservationofperipheralmusclemassandfatreserveswith
severedepletionofvisceralprotein.Mostrelevanttohospitalizedpatientsarepoor
woundhealingandimpairedimmunefunction.Thelatterresultsinanincreasedriskof
infectionssuchassepsisorpneumonia.
Marasmusresultsfromtotalcaloriemalnutritionandischaracterizedbysevere
generalizedwastingwithseveredepletionofmusclemass(andlowcreatinine).
SummaryBox:KwashiorkorandMarasmus

Case1115
A20yearoldwomancomestotheofficebecauseofnewonsetheadachesandjointpain.She
alsonotesthatherhairhasbeenfallingout.Physicalexaminationrevealsdry,peelingskinand
brittlenails.Youbelievethathersignsandsymptomsmayberelatedtoanewmedicationthat
shehasbeentakingforsevereacne.

Table117
BloodWorkupforCase1111(st0435)

Lipid

MeasuredValues
Patient'sLevel ReferenceRange

Triacylglycerol

(mg/dL)

(mg/dL)

150

60160

Totalcholesterol 420

<200

HDLcholesterol

35

31

VLDLcholesterol 30

2040

LDLcholesterol

<100

359

HDL,highdensitylipoproteinLDL,lowdensitylipoproteinVLDL,verylowdensitylipoprotein.
Table118
KwashiorkorandMarasmus

Feature

Kwashiorkor

Marasmus

Cause

Stressinducedproteincatabolismand/orlow

Chroniclowtotalcalorieintake

proteinintake(developsoveronlyweeks)

(developsovermonthsto
years)

Lossofvisceralproteinwithincreasedvisceralfat

Severelossofperipheral

Basic

characteristic andrelativelypreservedperipheralprotein

protein

Clinical

Poorlypigmented,easilypluckedhairanasarca

Emaciatedappearance,often

presentation

(severediffuseedema),increasedinfectionrisk,

withweightlowerthan80%of

poorwoundhealing

idealweight

Verylowserumalbumin(<2.8mg/dL)

Lowcreatinine(from

Laboratory
testresults

decreasedmusclemass)

1 Whatisthelikelydiagnosisinthispatient?
HypervitaminosisA(vitaminAtoxicity)islikely.BecausevitaminAisfatsoluble(alongwith
vitaminsD,E,andK),itcanaccumulateinadiposetissueandhasthepotentialtoreachtoxiclevelsif
consumedinexcess.Isotretinoin,adrugusedtotreatsevereacne,isavitaminAderivativethatcan
causetoxicityinpatientswhodonotadheretotheirdoctor'sinstructionswhentakingthis
medication.
OtherpotentialsourcesofvitaminAtoxicityincludeoverconsumptionofvitaminAcontaining
foods(eggyolk,butter,milk,bearliver)orvitaminA/multivitaminmegadoses.

2 WhatarethesymptomsofvitaminAtoxicity?

VitaminAtoxicityresultsinHeadache,Arthralgias,Sorethroat,Skinchanges,Alopecia,Fatigue,
andTeratogenicity(cardiacproblems,cleftpalate,andspontaneousabortions).Owingtothehigh
potentialforteratogenicitythatresultsfromhypervitaminosisA,womentreatedwithisotretinoin
mustnotbecomepregnantwhentakingthismedication.
Remember:VitaminAHASSAFT(safety)issues.

3 WhatarethesymptomsassociatedwithvitaminAdeficiency?
BecausevitaminAisanessentialcomponentofvisualpigments(e.g.,rhodopsin),deficiencyresults
innightblindness.Itisalsorequiredfordifferentiationofepithelialcellsintospecializedtissuesand
preventssquamousmetaplasia.Deficiencycanresultindryskinandsquamousmetaplasiaofthe
conjunctiva(Bitot'sspots).
Asmentionedintheintroductiontothischapter,nutritionisanextremelyimportantsubjectto
understandforboardsandwhendiscussinghealthanddietrelatedissueswithpatients.Most
studentswillhavemultiplequestionsonthistopicontheirUSMLEexamination.Spendagreat
dealoftimelearningthesymptomsassociatedwithdeficiency(andtoxicity,whenrelevant)of
thedifferentwatersolubleandfatsolublevitamins.Thebestresourceforthisinformationis
yourcopyofFirstAid.
Step1Secret

FatsolublevitaminsincludevitaminA,D,E,andK.Watersolublevitaminsincludethe
BvitaminsandvitaminC.
VitaminAtoxicitycanresultfromoverconsumptionofvitaminAcontainingfoods,
multivitaminabuse,ortreatmentofsevereacnewithisotretinoin.Toxicitysignsand
symptomsincludeheadache,arthralgias,sorethroat,skinchanges,alopecia,fatigue,and
teratogenicity.
VitaminAdeficiencyresultsinnightblindess,dryskin,andsquamousmetaplasia.
summarybox:hypervitaminosisa

Copyright2015Elsevier,Inc.Allrightsreserved.

BOOKCHAPTER

HematologicMalignancies
DanaM.Carne,ThomasA.BrownMDandSonaliJ.Shah
USMLEStep1Secrets,Chapter14,418438

HematologicmalignanciesareeasypointsonStep1ifyoustudyforthesetopicscorrectly.This
chapterisincludedtofocusyouonthemosthighyieldpointstotakeawayforboards.Before
youreadthroughthischapter,youshouldskimthefollowinglistoftipstohelpyougainthe
maximumnumberofpointsonyourexamination:
KnowallofthechromosomaltranslocationsmentionedinthischapterandFirstAid.
Boardexamslovetoteststudentsonthesepoints.
Paycarefulattentiontopatientageswhenevertheyaregivenintheclinicalhistory.Age
canbeveryhelpfulindistinguishingvarioustypesofhematologicmalignancies.You
shouldespeciallymemorizetheagerangesforthemostcommontypesofleukemias.
Acutelymphoblasticleukemia(ALL)occursbetweentheagesof0to15,acute
myelogenousleukemia(AML)occursbetweentheagesof15to59,chronicmyelogenous
leukemia(CML)occursbetweentheagesof40to59,andchroniclymphocyticleukemia
(CLL)occursaftertheageof60years.Keepinmindthatboardswillprovideyouwith
classicpresentationsfordiseases,somemorizingparticularruleslikethisoneforthe
USMLEcanbeextremelyhelpful.
Manyhighyieldimagesderivefromthissection.Youshouldknowhowtodiagnose
differentleukemiasandlymphomasfromimagesofperipheralbloodsmearsorbone
marrowaspirates.YoushouldalsoknowhowtorecognizeAuerrodsandReedSternberg
cells,twoespeciallyhighyieldimagescoveredinthissection.
Whenlearningthevarioushematologicmalignancies,focusontheinformationthat
helpsyoudifferentiatethesecancersfromoneanother.Sometimes,thisisallyouneedto
knowforboards.Agreatexampleishairycellleukemia,forwhichyoushouldknowthat
thesecellshavehairlikeprojectionsandstainpositivelywithtartrateresistantacid
phosphatase(TRAP).Thesearethetwomostuniquefactsabouthairycellleukemia,and
atleastoneofthemisguaranteedtobeprovidedinanyquestionstemyoumayreceiveon
thistopic.It'sassimpleasthat.Usetheinformationinthischapter(especiallythetables)
toguidewhatyoushouldtakeawayfromeachindividualdisease.

Insider'sGuidetoHematologicMalignanciesfortheUsmleStep1

Basicconcepts
1 Whatarethetwoprincipallineagesalongwhichleukocytesdifferentiate?
ThelymphoidlineagegivesrisetoBandTlymphocytesaswellasnaturalkiller(NK)cells,andthe
myeloidlineagegivesrisetothegranulocytes(eosinophils,basophils,neutrophils),monocytes,
platelets,anderythrocytes(i.e.,thenonlymphoidcells).Cellsoftheinnateimmunesystemderive
fromthemyeloidlineage,butcellsoftheadaptiveimmunesystemderivefromthelymphoidlineage.
TheexceptiontothisruleisNKcells,whichareconsideredtobeplayersintheinnateimmune
system.

2 Whatcategoriesofhematologicmalignancyarisefromthelymphoidlineage?
Thelymphomas,includingHodgkin'slymphomaandthevarioustypesofnonHodgkin's
lymphoma(NHL)
Thelymphocyticleukemias,includingacutelymphoblasticleukemia(ALL)andchronic
lymphocyticleukemia(CLL)
Tumorsofplasmacells(antibodysecretingBcells),whichincludemultiplemyelomaand
lymphoplasmacyticlymphoma
Note:Alllymphoidneoplasmsarisefromasingletransformedcellandareconsequently
phenotypicallymonoclonal.

3 Whatisthegeneraldistinctionbetweenlymphomaandleukemia?
Leukemiagenerallyindicatessignificantbonemarrowinvolvementwithneoplasticcells(andoften
significantperipheralbloodinvolvement).Lymphomaindicatesamassoriginatingintheperipheral
tissues(lymphnodes).However,thislineisoftenblurredbecauselymphomascanevolvetoinfiltrate
themarrow(aleukemicpicture),andmalignanciesotherwiseidenticaltoleukemiasmaystartoutas
peripheraltissuemassessimilartolymphomas.Currently,leukemiaorlymphomareferstotheusual
tissuedistributionofneoplasmsofaparticularcelltype.

4 Whatisthedistinctionbetweensmallcelllymphocyticlymphoma(SLL)and
(Bcell)chroniclymphocyticleukemia(BCLL)?
BCLLandSLLareessentiallythesameentitywiththeexactsamecelltype,immunophenotype,and
geneticalteration.TheyarethereforetypicallyreferredtoassimplySLL/BCLL,withoutdesignating
aspecificdiagnosis.Thedistinctionisthatwhentheperipheralbloodlymphocytesreachacertain
level,thenthediseaseisconsideredleukemic.Thisisafantasticexampleoftheblurbetween
leukemiasandlymphoma.
Note:SmudgecellsareacharacteristicfeatureofCLLonaperipheralsmear.

5 Whatcategoriesofhematologicneoplasmsarisefromthemyeloidlineage?
Acutemyelogenousleukemia(AML),whichhasmultiplesubclassifications
Myelodysplasticsyndromes,allpotentialprecursorsofAML
Myeloproliferativedisorders,includingpolycythemiavera(PV),chronicmyelogenous
leukemia(CML),andessentialthrombocythemia
Histiocytoses(ofmonocytemacrophageorigin)
Note:Allmyeloidneoplasmsarisefromatransformedhematopoieticprogenitorcell.Featuresof
theseneoplasmsoftenoverlap,confusingmedicalstudentsandphysiciansalike.Theymayalso
infiltratethebonemarrow,resultinginanemia,thrombocytopenia,andleukopenia.Additionally,
bothALLandCLL,aswellasmanytypesoflymphoma,canhavesignificantbonemarrow
involvement.

6 Whatisthedistinctivefeatureofacutemyelogenousleukemiaonbone
marrowbiopsy?
Asubstantialamountofthebonemarrowisreplacedbyrelativelyundifferentiatedblastcellsthat
resembleone(ormore)oftheearlystepsofmyeloiddifferentiation.Consequently,therearemultiple
subclassificationsofAML,dependingonwhichearlycelltypepredominates(e.g.,acute
promyelocyticleukemia,acutemyelomonocyticleukemia,acutemegakaryocyticleukemia).
Themostimportantformofacutemyelogenousleukemia(AML)toknowforboardsistheM3
subtype,whichischaracterizedbycirculatinggranulocytesandmyeloblastscontainingAuer
rods.Auerrodsareperoxidasepositiveinclusionbodiesinthecytoplasmoftheneoplasticcells
thatarepathognomonicforthiscondition.ReleaseoftheseAuerrodscanresultin
disseminatedintravascularcoagulation(DIC),whichisassociatedwiththeM3subtypeofAML.
TheM3formofAMLismarkedbyacharacteristict(15,17)translocationthatinvolvesthe
retinoicacidreceptor.Fortunately,thisdiseaserespondstotreatmentwithalltransretinoic
acid.
Step1Secret

7 Whatisthedistinctivefeatureofmyelodysplasticsyndromes?
Inthesediseases,amutantstemcellthatcangiverisetoallcelltypesofthemyeloidlineagereplaces
thebonemarrow(partlyorwholly).However,thismutantstemcellproducescellsofthemyeloid
lineageinanineffectivemanner.Consequently,anemia,thrombocytopenia,andleukopeniaareseen,
frombothineffectivehematopoiesisandreplacementofbonemarrow.Moreover,the
myelodysplasticsyndromesmayevolveintoAML.

DysplasticchangesofmyelodysplasticsyndromeincludePelgerHuetcellswithaviatornuclei
(bilobedpolymorphonuclearcells)andringsideroblasts.ThissyndromeisassociatedwithBCL2
mutationaswellasmethylation(andinactivation)oftumorsuppressorgenes.

8 Whatisthedistinctfeatureofthemyeloproliferativedisorders?
Inthesediseases,atransformedhematopoieticprogenitorcellcausesapathologicallyincreased
productionofoneofthefinalproductsofthemyeloidlineage(granulocytes,erythrocytes,platelets).
Inotherwords,myeloproliferativedisordersdifferfromotherneoplasticleukocyticdisordersinthat
theycauseanoverproductionofmaturecells.Examplesarepolycythemiavera(increasedredblood
cell[RBC]production),essentialthrombocytosis(increasedplateletproduction),andCML,which
resultsinanincreasednumberofgranulocytes.
Note:CMLisuniqueamongthemyeloproliferativedisordersinthatithasacharacteristicgenetic
defect:achromosomaltranslocation(t9:22),knownasthePhiladelphiachromosome.

9 Whatarethehistiocytoses?
Ahistiocyteisatissuemacrophagefoundwithintheinterstitium.Thehistiocytosesaredisorders
involvinguncontrolledproliferationofhistiocytes.ForStep1,studentsneedbefamiliaronlywith
Langerhanscellhistiocytosis(alsoknownashistiocytosisX),which,dependingonageat
presentationandclinicalcourse,isclassifiedasLettererSiwedisease,HandSchllerChristian
disease,oreosinophilicgranuloma.AcharacteristicfeatureofhistiocytosisXisBirbeckgranulesin
thecytoplasm,whichyoucaneasilyrecognizeonStep1becausetheylooklikeminiatureintracellular
tennisrackets.
Note:Thetermeosinophilicreferstothecharacteristicpinkproteinstainingandnotthecelltype.

10 Whatistherelationshipbetweenmyelofibrosisandthemyeloproliferative
diseases?
Myelofibrosisissimplyfibrosisofthebonemarrowwithcollagen.Ithasmultiplecauses,suchas
radiation,drugs,andchemicals,butismostcommonlyidiopathic.Itcanalsooccurasaresultofthe
myeloproliferativedisorders.Myelofibrosisismostlikelyduetosignalingbyabnormal
megakaryocytes.Theresultingfibrosiscanimpairhematopoiesis,resultinginhypocellularbone
marrowandpancytopenia,whichcanunfortunatelyberapidlyfatal.Thebodytriestocompensate
forthisdeficiencywithextramedullaryhematopoiesis.
MyelofibrosisismarkedbyteardropRBCs,whichresultfromdamagetotheRBCstructureasthese
cellstrytoescapethefibroticbonemarrow.Ifyouseethesecellsonaperipheralbloodsmear,you
shouldimmediatelyassociatethisfindingwithmyelofibrosis.

11 Howdoestheleukocytealkalinephosphataselevelhelpdifferentiatereactive
leukocytosisfromatrueleukemia?
Reactiveleukocytosis(leukemoidreactionleftshift)isapronouncedincreaseinwhitebloodcell
(WBC)countwithinfectionandisassociatedwithhighlevelsofleukocytealkalinephosphatase

(LAP).Incontrast,theabnormallylargeincreaseinWBCsseeninleukemiaistypicallyassociated
withlowlevelsofLAPbecausetheneoplasticcellscannotproducethisenzyme.Thisisanimportant
distinctiontolookoutforonboards.

12 Quickreview!CovertherightcolumninTable141(t0010)andattempttolist
thecelltypeandpertinenthighyieldfactsregardingthehematologic
malignanciesintheleftcolumn
Table141
LeukemiaLineages

Disorder

CellType/Comments

LymphoidLineage
Acutelymphoblastic
leukemia

PrecursorBorTlymphocytes(lymphoblasts)

Chroniclymphocytic
leukemia/small
lymphocyticlymphoma

Bcells,smudgecellsonperipheralsmear

Hodgkin'slymphoma

ReedSternbergcells

Lymphoplasmacytoma

Plasmacells(matureBcellsIgMsecreting)acommoncauseof
Waldenstrm'smacroglobulinemia

Multiplemyeloma

Plasmacells(matureBcellsIgGandIgAsecreting)

NonHodgkin'slymphoma

Multipletypes,whichgenerallyareclassifiedaccordingtositein
lymphnodetheyresemble(e.g.,follicular,mantlezone,marginal
zone)

MyeloidLineage
Acutemyelogenous
leukemia

Cellsofearlymyeloidlineage

Histiocytosis

Langerhanscell(ofmonocytemacrophageorigin)

Myelodysplastic
syndromes

Hematopoieticstemcellwithineffectivehematopoiesisthatreplaces
bonemarrow

Myeloproliferative
syndromes

Clonalexpansionofamultipotenthematopoieticstemcell,ultimately
givingrisetoexcessnumbersofoneormorefinalproductsofthe
myeloidlineage

IgA,IgG,IgM,immunoglobulinsA,G,andM.

13 Quickreview!CovertherightcolumninTable142(t0015)andattemptto
definethehematologictermsintheleftcolumn
Table142
HematologicTerminology

Term

Definition

Leukemia

Malignantproliferationofbloodcellswithinthebonemarrowandcirculatorysystem

Leukemoid
reaction

AbnormallyhighWBCcount,similartothatoccurringinvariousformsofleukemia,
butnotastheresultofleukemia(typicallyinresponsetoinfection)

Lymphoma

Proliferatingmassoflymphocyticcells:Bcells(mostcommon),Tcells,NKcells

Myelofibrosis Fibrosisofbonemarrow,oftencausingextramedullaryhematopoiesisthatresultsin
hepatosplenomegaly
Myelophthisic Anemiathatarisesfromaspaceoccupyinglesionthatdisplacesnormal
anemia
hematopoieticelements
NK,naturalkillerWBC,whitebloodcell.

14 WhatarethegeneticalterationsinthefollowingnonHodgkin'slymphomas?
SeeTable143(t0020).
Table143
GeneticAlterationsinNonHodgkin'sLymphoma

Non

GeneticAlterations

Hodgkin's
Lymphoma
Burkitt's

Translocationbetweenchromosomes8and14,involvescmycactivationon

lymphoma

chromosome8oftenassociatedwithHIVinfectionEBVinfectioncommoninAfrican
Burkitt's(patientspresentwithjawmass)
Sporadicformisassociatedwithpelvicandabdominallesions
LookforcharacteristicstarryskyappearanceofBurkitt'stype,duetosheetsof
neoplasticlymphocytesinfiltratedwithoccasionalmacrophages

Follicular

Translocationbetweenchromosomes14and18involvingBCL2activation

lymphoma

(antiapoptosisgene)
Associatedwithpainless,generalizedlymphadenopathy,primarilyinadults

Mantlecell Translocationbetweenchromosomes11and14,whichincreasescyclinDexpression
lymphoma

andpromotescellcycleprogression

AssociatedwithCD5expression
Usuallyfatal,occursinadults
Cellsresemblemantlezonecellsthatsurroundfollicularcenters
EBV,EpsteinBarrvirusHIV,humanimmunodeficiencyvirus.

15 WhataresomecharacteristicsoftheothernonHodgkin'slymphomas?
SeeTable144(t0025).
Table144
CharacteristicsofNonHodgkin'sLymphoma(NHL)

NHLType

Description

CutaneousTcelllymphoma CausedbyaTcelldyscrasia,unlikemostNHLs,whichareBcellin
origin
ExamplesaremycosisfungoidesandSzarysyndrome
Tcelllymphomasareassociatedwithcutaneouslesions
DiffuselargeBcell

Veryaggressivelymphomaoccurringmostlyinolderadultsbut

lymphoma

sometimesinchildren
AssociatedwithBCL6overexpression,whichsilencesp53
Earlystageisresponsivetotreatment

Marginalzonelymphoma

BeginsasreactivepolyclonalBcellproliferation,culminatingin

(MALToma)

monoclonalBcellneoplasm

16 Quickreviewwithhighyieldwordassociations:Covertherightcolumnof
Table145(t0030)andattempttolisttheassociateddiseases
Case141
A65yearoldmanisevaluatedforrecentonsetoflowbackpain,despiteanegativehistoryof
traumaorinjurytotheback.Thepainismoderatelysevereandhasbeenwakinghimatnight.
Healsoreportsa10lbweightlossinrecentmonthsaswellasfatigueandrecurrentsinus
infections.Thephysicalexaminationissignificantonlyforfocaltendernesstopalpationatthe
leveloftheT12vertebra.

Table145
HighYieldWordAssociations

Description

Disease

>30%myeloidblastsinbonemarrow

Acutemyelogenous
leukemia(AML)

8:14translocation

Burkitt'slymphoma

14:18translocationinvolvingBCL2

Follicularlymphoma

Auerbodies

Acutemyelogenous
leukemia

BenceJonesproteinuria

Multiplemyeloma

Massivesplenomegaly,Bcellproliferation,andpresenceoftartrate

Hairycellleukemia

resistantacidphosphataseinBcells
Philadelphiachromosome

Chronicmyelogenous
leukemia(CML)

ReedSternberg(lacunar)cells

Hodgkin'slymphoma

Smudgecellsonperipheralbloodsmear

Chroniclymphocytic
leukemia(CLL)

1 Whatisthedifferentialdiagnosisforbackpaininanolderpatient?
Thedifferentialdiagnosisisquitebroadandincludesmusculoskeletalbackpain(musclestrain,
herniateddisk),vertebralcompressionfracture,malignancies(primaryormetastatic),abdominal
aorticaneurysm,andaninfectiouscausesuchasosteomyelitis,abscess,ortuberculosis(Pott's
disease).
Forboards,youneedtohaveahighindexofsuspicionformalignancy,particularlyinanolder
individualwithconstitutionalcomplaintssuchasunintentionalweightlossandfatigue,backpain
thatiswakingthepatientatnight(oneofthealarmsymptomsofbackpain),andrecurrent
infections.Malignanciestoconsiderinanoldermanincludemetastaticprostatecancerandmultiple
myeloma.

Case141continued:
AnxrayfilmoftheskullforevaluationofhissinusinfectionsisshowninFigure141(f0010).
SerumproteinelectrophoresisrevealsanMspike.Aurinedipsticktestisnegativefor
proteinuria,buta24hoururineproteincollectionrevealsmarkedproteinuria.

Figure141
XrayfilmofskullfrompatientinCase141.
(FromAbeloffMD,ArmitageJO,NiederhuberJE,etal:ClinicalOncology,3rded.Philadelphia,
ChurchillLivingstone,2004.)

2 Whatistheexpecteddiagnosis?
Thisisaclassicpresentationforapatientwithmultiplemyeloma.Thepunchedoutlyticlesionsare
causedbycytokinesreleasedbythemyelomacells,whichpromoteosteolysis.TheMspikeonserum
proteinelectrophoresisindicatestheoverproductionofamonoclonalantibody.The24hoururine
collectionshowingincreasedproteinrepresentstherenalexcretionoflightchains(BenceJones
proteins).Notethattheurinedipsticktest,whichdetectsnegativelychargedproteinssuchas
albumin,doesnotshowproteinuria,whereasa24hoururineproteincollectionrevealsmarked
proteinuriabecauseofthedetectionofthepositivelychargedlightchains.

3 Whatbloodabnormalitiesoftenpresentwithmultiplemyelomaandwhy?
Anemia,increasedcreatinine(renalinsufficiency),andhypercalcemiaoftenaccompanymultiple
myeloma.
Theanemiaoccursasaresultofsuppressederythropoiesis.Thereasonforthisistwofold:(1)
cytokinesecretion,particularlyIL6byplasmacells,inhibitsnormalRBCproduction,and(2)
widespreadmarrowinfiltrationbytheplasmacellsinhibitsRBCformation.
Therenalinsufficiencycanoccurforavarietyofreasons,buttheprimarycausesaremyelomaofthe
kidneyandhypercalcemia.Lightchainsaredirectlytoxictothetubularepithelialcellsofthe
nephron,candepositwithinthetubularlumencausingobstruction,andcanalsodepositwithinthe
renalinterstitium.Inaddition,myelomapatientsareatincreasedriskofdevelopingamyloidosis,
whichcanfurtherdamagethekidneys.
Thehypercalcemiacanoccurasaresultofincreasedbonebreakdownbothfrombonymetastasisand
fromtheelaborationofosteoclaststimulatingcytokinesbytumorcells(primarilyIL1).Thisbone
breakdownexplainswhymyelomapatientsarepredisposedtopathologicfractures.Recallthat
symptomsofhypercalcemiaincludeconfusion,muscleweakness,polyuria,andconstipation.

Case141continued:
Bloodworkrevealsthefollowing:

Hematocrit:29%
Hemoglobin:9.2g/dL
Albumin:2.2g/dL(normal3.14.3g/dL)
Totalplasmaprotein:8.6g/dL(normal6.38.2g/dL)
Bloodureanitrogen(BUN):22mg/dL(normal720mg/dL)
Creatinine:3.2mg/dL(normal0.71.4mg/dL)
Plasmacalcium:12.4mg/dL(normal8.510.5mg/dL)
AbonemarrowaspirationsampleisshowninFigure142(f0015).

Figure142
BonemarrowaspirationfrompatientinCase141.
(FromHoffmanR,BenzEJ,ShattilSJ,etal:Hematology:BasicPrinciplesandPractice,4thed.
Philadelphia,ChurchillLivingstone,2005.)

4 Whatcelltypeabnormallyproliferatesinmultiplemyeloma?
Plasmacells,whichareterminallydifferentiatedantibodysecretingBcells.Thesemalignantcells
continuouslysecreteexcessiveamountsofasinglemonoclonalimmunoglobulin,explainingthe
presenceoftheM(formonoclonal)spikeseenonserumproteinelectrophoresis.Thissecretionof
excessiveamountsofasingleimmunoglobulinisreferredtoasamonoclonalgammopathy.The
resultinghypergammaglobulinemiaexplainstheelevatedtotalplasmaproteinlevelscommonlyseen
inmultiplemyeloma.Hypoalbuminemiaisalsotypicalofmultiplemyeloma,andisthoughttobe
secondarytoelevatedcytokinelevels(e.g.,IL6)thatreducethehepaticsynthesisofalbumin.

5 WhatistheassociationbetweenBenceJonesproteinuriaandthepreviously
mentionedmonoclonalgammopathy?
BenceJonesproteinsareimmunoglobulinlightchainsubunits(mostcommonlyoftheisotype)that
arefilteredbythekidneyandexcretedintheurine.Themalignantplasmacellsalsosecreteheavy
chainimmunoglobulinsubunits,buttheseareofhighenoughmolecularweightthattheyarenot
typicallyexcretedintheurine.

6 Whatisamyloidosisandwhyisthismanatriskfordevelopingit?
Amyloidosisisaclinicalsyndromecausedbythedepositionofinsolublefibrillarproteininvarious
tissues.Inamyloidosiscausedbymultiplemyeloma,themonoclonalgammopathyresultsinelevated
levelsoffreelightchainsintheblood,whichcanundergoprocessingandbepathologicallydeposited
intissuesthroughoutthebodyasamyloid.HistologicstainingofinvolvedtissueswithCongoredwill
revealtheclassicapplegreenbirefringencewhenvisualizedunderpolarizedlight.
Note:Amongthenumerouscomplicationsofamyloidosisiscarpaltunnelsyndrome,whichcan
occurasaresultofamyloiddepositioninthecarpaltunnel,causingcompressionofthemedian
nerve.

7 Explainwhythismanisatanincreasedriskforinfectioneventhoughplasma
levelsofimmunoglobulinsareabnormallyelevated
Althoughtotalimmunoglobulinsarehigh,thisispredominantlyduetotheMprotein.Theremaining
immunoglobulinsarelow,predisposingthepatienttoinfection.

8 Thispatient'sanemiacanbecharacterizedasmyelophthisis.Why?
Thesuffixphthisisreferstowastingawayoratrophyofabodypart.Inthiscasethemyelomahas
infiltratedintothebonemarrowandcausedittowasteaway,thusreducingproductionofRBCs.

9 Whatisthecharacteristicfindingonaperipheralbloodsmear,asshownin
Figure143(f0020),andwhydoesthisoccur?
Rouleauxformation,inwhichRBCsarestackedoneachotherlikearowofcoins,occursduetothe
agglutinationofRBCsthatismediatedbyMprotein.

Figure143
PeripheralbloodsmearofpatientinCase141(100).
(CourtesyofJeanShafer.)

10 Howmightthepresenceofelevatedserumlightchainsandamyloidosis
affectkidneyfunctioninthispatient?
Theycanseriouslyimpairrenalfunction,makingrenalfailureamajorcauseofdeathinmultiple
myeloma.Elevatedserumlightchainsgetfilteredandsecretedbythekidney.Theseproteinsare
toxictorenaltubularepithelialcellsandmayalsocombinewithaurinaryglycoprotein,theTamm

Horsfallprotein,toformcaststhatobstructthetubules.Renaldepositionofamyloid,whichmay
occurinprimaryamyloidosis,canalsoseriouslycompromiserenalfunction.

11 Whatisthepharmacologicbasisforgivingthispatientallopurinolpriorto
andduringchemotherapy?
Allopurinolinhibitstheenzymexanthineoxidase,whichcatalyzestheconversionofthepurine
metabolitexanthinetotherelativelyinsolubleuricacid.Inpatientsreceivingtoxicchemotherapy,
DNAcatabolismsecondarytoincreasedcelldeathresultsintheexcessiveproductionofpurine
metabolitesand,ultimately,uricacid.Thesepatientsarethereforeatincreasedriskfordeveloping
painfulgouturicacidkidneystones,butallopurinolreducesthisriskbydecreasingtheproductionof
uricacid.

Differentialdiagnosis
12 IfworkuprevealsanIgMmonoclonalgammopathyratherthananIgGorIgA
gammopathy,whatdiseasemightyoususpect?
Waldenstrm'smacroglobulinemia(macroforthelargepentamericIgMmolecule)ismost
commonlyduetolymphoplasmacyticlymphoma,aplasmacelltumorthatsecretesIgM.The
increasedconcentrationofIgMincreasesplasmaviscosity,whichimpairsbloodflowandcauses
sludgingintheretinalvesselsandthecerebralvasculature(andelsewhere),predisposingtovisual
disturbancesandneurologicproblems.
Note:AlthoughWaldenstrm'smacroglobulinemiaisassociatedwithanMspikeandneoplastic
cells,itdiffersfrommultiplemyelomainthattherearenolyticbonelesionsorhypercalcemia.

Multiplemyelomaoftenpresentswithanemia,renalinsufficiency,hypercalcemia,and
rouleauxformationonabloodsmear.
Suspectmultiplemyelomainanolderpatientwithunexplainedbackpainora
pathologicfracture.
Multiplemyelomaiscausedbyanabnormalclonalproliferationofantibodysecreting
plasmacells,resultinginamonoclonalgammopathy.
Renalfailureinmultiplemyelomacanbecausedbylightchaintoxicityanddeposition
withinthetubules,hypercalcemialeadingtonephrocalcinosis,andrenalamyloidosis.
Primaryamyloidosiscanoccurasaresultofprocessingofthelightchainswith
depositioninvarioustissues,includingthekidneysandcarpaltunnel.
Allopurinolisgivenwithchemotherapyformultiplemyelomatopreventthereleaseof
largeamountsofuricacidthatcanbeseenintumorlysissyndrome.
Waldenstrm'smacroglobulinemiaissimilartomultiplemyelomabutisassociatedwith

secretionofIgM.ThisshouldnotbeconfusedwiththeMspikeofmultiplemyeloma.
Waldenstrm'smacroglobulinemiaisnotassociatedwithlyticbonelesionsor
hypercalcemia.
SummaryBox:MultipleMyeloma

Case142
A4yearoldgirlisevaluatedforfatigue,nightsweats,anorexia,anda10lbweightlossover
thepastmonth.Shealsocomplainsofpaininherlowerbackthatsheisunabletolocalize
precisely.Becauseofthispain,shehasstartedwalkingwithalimpand,wheneverpossible,
triestoavoidwalkingatall.

1 Herparentsareworriedshemighthavecancer.Whatisthemostcommon
cancerinchildren?
ALL,causedbyamalignantproliferationofimmatureprecursorBorTlymphocytes(i.e.,preBor
preTcells).

Case142continued:
Physicalexaminationissignificantforconjunctivalpallor,hepatosplenomegaly,painless
lymphadenopathy,anddiffusepetechiae.

2 Whyarefatigue,petechiae,andfevercommonlyseeninpatientswithacute
lymphoblasticleukemiasandmyeloproliferativediseases?
Thepancytopeniathatoccursinthesediseasesasaconsequenceofmarrowreplacementis
responsible,withtheanemiacausingfatigue,thrombocytopeniacausingpetechiae,andleukopenia
predisposingtoinfectionandfever.

3 Howdoeslymphadenopathyfrominfectiondifferfromlymphadenopathydue
tomalignancy?
Tenderlymphnodesoftenindicateareactive(infective)process,whereaspainlesslymphadenopathy
mayindicatecancer,particularlywhenassociatedwithconstitutionalsymptomssuchasfatigue,
anorexia,weightloss,nightsweats,andfever.

Case142continued:
Acompletebloodcount(CBC)revealsamarkedlymphocytosis,thrombocytopenia,and
anemia.Abonemarrowbiopsyisperformedandreveals>50%lymphoblasts,karyotype
analysisofwhichrevealsmultiplechromosomaltranslocations.Immunostainingofthe
abnormalmarrowcellsispositiveforterminaldeoxytransferase(TdT).

4 Howdoestheprecedinginformationaffectthedifferentialdiagnosis?
AdiagnosisofALLismuchmorelikely.TdTisanenzymeinvolvedinthenormalgene
rearrangementofimmaturelymphocytes(lymphoblasts)thatcreatesuniqueantigenspecificity.Itis
presentinthevastmajorityofpatientswithALL.NotethatALLismostcommoninchildrenyounger
than15yearsofage.

5 Howdoesthepresentationclassicallydifferiftheacutelymphoblastic
leukemiaiscausedbyamalignancyofpreTcellsratherthanpreBcells?
ALLcausedbypreBcellshasapeakageofincidenceof3to4years.ThistypeofALLisprimarilya
leukemia,withpredominantbonemarrowandperipheralbloodinvolvement.However,theless
frequentALLcausedbypreTcellstypicallyoccursinadolescentsandpresentsprimarilyasa
lymphoma,withpredominantinvolvementofthelymphaticsystem.ThislymphomatoustypeofALL
maypresentwithmediastinalmasses,markedlymphadenopathyandsplenomegaly,andthymic
involvement.
Note:BecausepreBcellsandpreTcellscanbedifficulttodistinguishmorphologically,
immunophenotypingbasedoncellsurfacemarkersisnecessarytomakeadefinitivediagnosis.

6 Howisdiagnosisofacutelymphoblasticleukemiadifferentiatedfromacute
myelogenousleukemia?
AlthoughALLandAMLarebothacuteleukemiasinwhichblastcellspredominate,ALLoccursin
childrenundertheageof15years,andAMLoccursintheagerangeof15to59years.Bloodsmears
willshowlargelymphocyteprecursorsinALLandmyeloblasts(perhapswithAuerrods)inAML.
ALLmayspreadtothecentralnervoussystemandtestessocalledsanctuarysites.Thesesitesare
generallysparedinAML.

7 Ifsheisstartedonhighdosechemotherapyandsuddenlydevelopsmarked
hyperuricemia,hyperkalemia,hyperphosphatemia,andhypocalcemia,what
hashappened?
Shehasdevelopedthetumorlysissyndrome,ametabolicemergencythatiscausedbymassive
destructionoftumorcellsafterinitiationofhighdosechemotherapy.Thedeathoflargenumbersof
tumorcellsresultsinthemetabolismoflargeamountsofDNA.Thisproducesexcessiveamountsof
uricacid,whichcanprecipitateintherenaltubulesandcauserenaldamage.Intracellularionssuch
aspotassiumandphosphatearealsoreleasedinlargeamountsbydyingtumorcells.Thereleased
potassiumcancausemarkedhyperkalemia,resultingpotentiallyinfatalarrhythmias.The
hyperphosphatemiacanprecipitatewithcalciumwithintubules,furthercontributingtorenal
damage.
Tumorlysissyndromecanbetreatedwithhydrationandallopurinol.Alkalizationofurinetoprevent
uricacidprecipitationiscontroversialbecauseitmaycausecalciumprecipitation.

Acutelymphoblasticleukemia(ALL)classicallypresentsinchildren.Symptomsinclude
fatigue,nightsweats,anorexia,andweightloss.
LaboratoryfindingsinALLarenotableforpancytopeniaduetoreplacementofbone
marrowbytumor.Notethatthepatientisdeficientinnormalwhitebloodcells,whilethe
actualwhitebloodcell(WBC)countmaybehighowingtomalfunctioningtumorcells.
Tenderlymphadenopathytypicallyindicatesaninfection.Incontrast,painless
lymphadenopathymayindicateamalignancy.
Tumorlysissyndromecandevelopafterinitiationofchemotherapyandisduetothe
releaseoflargeamountsofintracellularuricacid,potassium,andphosphatefromdying
tumorcells.
SummaryBox:AcuteLymphoblasticLeukemia/Lymphoma

Case143
A35yearoldwomancomplainsoffatigueforthepast3weeksandrecurrentspontaneous
nosebleeds.Physicalexaminationissignificantfornontenderlymphadenopathy,
hepatosplenomegaly,truncalpetechiae,andmultipleecchymoses.Aperipheralbloodsmear
showsthepresenceofnumerousmyeloblasts,cytogeneticanalysisofwhichshowsa
translocationbetweenchromosomes9and22.ACBCrevealsahematocritof25%,hemoglobin
of7.2g/dL,WBCcountof22,000/L,andplateletcountof30,000/L.

1 Whatisthelikelydiagnosis?
Thepresenceofatranslocationbetweenchromosomes9and22thesocalledPhiladelphia
chromosomeishighlysuggestiveofCML.

2 Whatisthepathogenesisofthisdisorder?
CMLisamyeloproliferativedisorderinwhichatransformedhematopoieticprogenitorcellcausesan
increasedproductionofgranulocyticcells(ofthemyeloidlineage)inthebonemarrow,causinga
myeloidleukocytosis(predominantlyneutrophils).Bonemarrowbiopsyinthesepatientswillreveal
anabnormallyincreasedmyeloiderythroidratioofapproximately15:1to20:1.ThePhiladelphia
chromosome,presentinamajorityofCMLpatients,involvesatranslocationbetweentheBCRgene
onchromosome9andtheABLgeneonchromosome22.
Note:Othermyeloproliferativedisordersincludepolycythemiaveraandessentialthrombocythemia.

3 Whatistheblastcrisisthatmayoccurinchronicmyelogenousleukemia?

Blastcrisisisanacuteworseningofthedisease,inwhichthereissubstantialmedullaryor
extramedullaryproliferationofblasts.Marrowblastswilloftenexceed30%ofmarrowcells.Thisis
anominousprognosticsign.

4 Compareandcontrastchroniclymphocyticleukemiaandchronic
myelogenousleukemia
SeeTable146(t0035)forthiscomparison.
Table146
ChronicLymphocyticLeukemia(CLL)andChronicMyelogenousLeukemia(CML)

Feature

BCLL/SLL

CML

Evolution

ToprolymphocyticlymphomaorlargeBcell

Blastcrisis(ominous)

lymphoma(bothominousevents)
Immunophenotype BcelltumorthatexpressesBcellantibodies

9:22translocation

andgenetics

(Philadelphiachromosome)
BCRABLfusiongene

andCD5(anantibodyuniquetoTcells)
variedchromosomalabnormalities

product
Peripheralblood

Smudgecells

Blasts,myelocytes

Olderadults

Youngtomiddleagedadults

smear
Typicalage

SLL,smalllymphocyticlymphoma.

ThePhiladelphiachromosomeisatranslocationbetweenchromosomes9and22andis
associatedwithchronicmyelogenousleukemia(CML).
CMLisamyeloproliferativedisorder.Othermyeloproliferativedisordersinclude
polycythemiaveraandessentialthrombocythemia.
CMLclassicallyaffectsmiddleagedadults,withsymptomsoffatigue,spontaneous
bleeding,lymphadenopathy,hepatosplenomegaly,andpetechiae.
CLLclassicallyaffectsolderadultsandoftenisassociatedwithsmudgecellsona
bloodsmear.
SummaryBox:ChronicMyelogenousLeukemia

Case144

A22yearoldmaletrackathleteatthelocaluniversitypresentswitha3monthhistoryof
fatigue.Uponquestioninghealsoadmitstodrenchingnightsweatsandanunintentional15lb
weightlossduringthistime.Hispastmedicalhistoryisunremarkable.

1 Isthishistoryconcerningforaseriousdisease?
Thesesymptomsareveryworrisome,especiallyforlymphoma.Lymphomapatientsoftenpresent
withsocalledBsymptoms,whichincludefevergreaterthan38C,unintentionallossof>10%of
bodyweight,andnightsweats.Thisstagingisusedtodistinguishsymptomaticlymphomapatients
fromthosewithoutsystemicsymptoms.

2 Otherthanmalignancy,whatelsedoesthedifferentialdiagnosisinclude?
Ina22yearold,youwanttoconsiderandsearchthequestionstemforthefollowingclues:
Depression:Doesthepatienthaveadepressedmood,sleepdisturbances,orlossofinterestin
normalactivities?
Infection:Mononucleosispatients,forexample,mayalsocomplainofasorethroatandhave
posteriorcervicallymphadenopathyonexamination.Alsoruleouthumanimmunodeficiency
virus(HIV).
Note:Inanolderpatient,youshouldalsoconsidermalignanciessuchasAMLorCML,aswellas
diabetes(lookforpolydipsia/polyuria.)

Case144continued:
Onexamination,nontendersupraclavicularlymphadenopathyisnotedaswellasafeelingof
fullnessintheupperleftabdominalquadrant.Acomputedtomography(CT)scanofthechest
revealsamediastinalmass,andheisreferredtooncologyforfurtherevaluation.

3 Howdoesthelocationoftheenlargedlymphnodeshelpinthedifferential
diagnosis?
Asmentionedearlier,mononucleosisorupperrespiratoryinfectionsoftenareassociatedwith
enlargedcervicalnodes,butcancermustalwaysberuledoutinpatientswithenlarged
supraclavicularnodes.

Case144continued:
Biopsyoftheenlargedsupraclavicularnoderevealsthepresenceoflarge,binucleatedcells
amidnormalappearinglymphocytes,histiocytes,andgranulocytes,asshowninFigure144
(f0025).

Figure144
ASupraclavicularlymphnodebiopsyfrompatientinCase144.
(FromHoffmanR,BenzEJ,ShattilSJ,etal:Hematology:BasicPrinciplesandPractice,4thed.
Philadelphia,ChurchillLivingstone,2005.)

4 Namethesecellsanddescribetheirlineageoforiginandrelationtothis
patient'sdiagnosis
ThiscellulardescriptionisclassicforReedSternbergcells,clusterdesignation(CD)30+andCD15+
monoclonalBcells,thepresenceofwhichindicatesHodgkin'slymphoma.Lymphomacanbe
categorizedintoHodgkin'sandNHL,withthebasicdistinctionthatHodgkin'slymphomahasReed
Sternbergcells.
Note:ThispresentationwasaclassicdescriptionofHodgkin'slymphoma(Table147(t0040)).
Table147
Hodgkin'sLymphoma

Feature

ClinicalManifestations

Symptoms/signs Patientstypicallypresentwithintermittentfevers,nightsweats,unintentional
weightloss,andfatigue.
Physical

Patientsoftenhavesupraclavicularorcervical,nontenderlymphadenopathy.

findings
Imaging

Themediastinalmassislikelytorepresentlymphadenopathy,oftenseenin

Hodgkin'slymphoma.
Epidemiology

Patientsarepredominantlymale.
Hodgkin'slymphomashowsabimodaldistribution,withonepeakbetween15
and35yearsofageandanotheraftertheageof50.
PatientswithnonHodgkin'slymphomaalsoaretypicallymale,butthemedian
ageatdiagnosisisaround65years.

ReedSternbergcellsarecommonlypresentedinimageformatonStep1.Youshouldbeableto
recognizethesecellsanddistinguishthemfromtheowl'seyeinclusionbodiesthatare
associatedwithcytomegalovirus(CMV).
Step1Secret

5 WhatarethemainvariantsofHodgkin'slymphomaandhowdotheyrelateto
theprognosis?
Nodularsclerosis(6080%ofpatients):LacunarvariantofReedSternbergcellwithfibrotic
collagenbandsthatdividetumorintocircumscribednodules(excellentprognosis).Patientsare
predominantlyyoungandfemale.
Mixedcellularitytype(1530%):Heterogeneous(mixed)cellularinfiltrateoflymphnodes
(goodprognosis).Thesepatientsareusuallyolder.
Lymphocyticpredominance:Predominantlylymphocyticinfiltrateoflymphnodes(excellent
prognosis).
Note:Thelymphocyticdepletionvariantiscontroversial,soitisnotaddressedhere.

6 WhatisthevalueofdistinguishingHodgkin'slymphomafromnonHodgkin's
lymphoma?
Unlikeotherlymphomas,Hodgkin'slymphomabeginsasalocalizedprocessandspreadsina
consistentfashiontoadjacentnodes,commonlyonlyaffectingasinglesetofaxialnodes,andrarely
hasextranodalinvolvement.Consequently,thistumorisoftensusceptibletolocalcure.NHLs
frequentlyinvolvemultipleperipheralnodesandextranodalsites,sotheyarelesslikelytorespondto
purelylocalizedtherapy.Patientsareclassically20to40yearsold,areoftenHIVpositiveor
otherwiseimmunosuppressed,anddonothavethehypergammaglobulinemiaseeninmultiple
myeloma.

Lymphomapatientsusuallypresentwithsystemic(B)symptomsoffever,weightloss,
andnightsweats.
ReedSternbergcellsarelargebinucleatedcells(owl'seyes)seenonnodebiopsyandare

nearlypathognomonicforHodgkin'sdisease.
Hodgkin'slymphomahasmanysubcategoriesaslistedhere,eachoftenwithaunique
geneticalterationandcourse.
SummaryBox:Hodgkin'sAndNonHodgkin'sLymphomas

Case145
A62yearoldwomanwithanunremarkablemedicalhistoryisevaluatedfora4weekhistory
ofheadaches,dizziness,andgeneralizeditching,particularlyaftertakinghotshowers.Shealso
complainsofboutsofsuddenonsetintenseburninginherhandsandfeetwithaccompanied
bluishdiscolorationofthesurroundingskin,thoughshehasfoundthroughtrialanderrorthat
aspirinbringsrapidrelief.

1 Whatisthedifferentialdiagnosis?
Theparesthesiasandchangesinskincolorinherhandsandfeetmightbeseenwithperipheral
vasculardisease,Raynaud'sphenomenon,ordiabeticneuropathy.Thepruritus,particularlyafter
warmshowers,cansometimesbeseeninmyeloproliferativedisorderssuchaspolycythemiavera
(PV)oressentialthrombocytosis.

Case145continued:
Physicalexaminationissignificantforelevatedbloodpressureandsplenomegalybutis
otherwisenormal.Laboratoryevaluationrevealsthefollowing:
Hemoglobin:19.6g/dL
Hematocrit:60%
WBCcount:15,800
Platelets:500,000

2 Whatisthedifferentialdiagnosisoftheerythrocytosis(elevatedhematocrit)?
Differentialdiagnosisincludesdehydration,secondarypolycythemia(duetohypoxemia,renal
tumors,exogenouserythropoietin,Cushingsyndrome,carboxyhemoglobinemia),andprimary
polycythemia(i.e.,polycythemiavera[PV]),amyeloproliferativedisorder.

Case145continued:

Furtherbloodworkshowsalowleveloferythropoietin,normaloxygensaturation,andan
elevatedleukocytealkalinephosphataseanduricacid.Aperipheralsmearisunremarkable,and
abonemarrowaspirateshowshypercellularitywithmegakaryocytichyperplasiaandreduced
ironstores

3 Whatisthelikelydiagnosis?
PVislikely.Patientswiththisdisorderoftencomplainofpruritus,whichisduetoincreased
basophilsandconsequentlyincreasedhistamineproductionbymastcells.Thisisresponsibleforthe
generalizeditchingthatoccursfollowinghotshowers,aclueoftenprovidedintheclinicalhistoryfor
PVonboards.Theincreaseduricacidlevelisduetoincreasedcellturnoverassociatedwiththis
condition.Patientsalsocommonlypresentwitharuddyface,blurredvision,andheadachesecondary
tovascularcongestion.

4 Whatisthepathophysiologyofhersymptomsofsuddenonsetburning?
Thisburningisaconditioncallederythromelalgia,andiscausedbymicrovascularthrombi.Itisseen
inPVbecausethisdisorderalsocausesdysfunctioningplatelets,resultinginincreasingstickiness
amongthecells.Erythromelalgiaisalsoseeninthemyeloproliferativedisorderessential
thrombocytosisbecausetheincreasedplateletcountresultsinincreasingclotting.Aspirinisthemost
effectivetreatmenttoreducepain.Becauseofplateletdysfunctionandmarkedlyelevatedblood
viscosityandvolume,patientsareatanincreasedriskforotherthromboticeventsincluding
myocardialinfarction,deepvenousthrombosis(DVT),stroke,andBuddChiarisyndrome.

5 Whatisthesignificanceoflowerythropoietin(epo)levelsindiagnosing
polycythemiavera?
Erythropoietin(epo)isproducedbythekidneyswhenthebodysenseslowRBCvolume.During
statesofRBCoverproductionsuchasPV,epolevelsaresuppressed.Thiswillbeanimportantclinical
clueindiagnosingPVversusothercausesofpolycythemia,whichisatermusedtodescribean
abnormallyincreasedRBCcount.Appropriatepolycythemiaoccursinhypoxemicstatesthat
reactivelytriggerRBCproductionsuchashighaltitude,pulmonarydisease,andcertainformsof
heartdisease.Inthesecases,oxygensaturationisdecreasedandeposecretionisupregulated.A
changeinoxygensaturationdoesnotoccurinPV.Ininappropriateabsolutepolycythemia,ectopic
eposecretionisresponsibleforincreasedRBCmass.Thismostcommonlyoccursinrenalcell
carcinoma,pheochromocytoma,andhepatocellularcarcinoma.Oxygensaturationremains
unaltered.Thefinalformofpolycythemiathatyoushouldknowforboardsisrelativepolycythemia,
whichismarkedbydecreasedplasmavolumesecondarytodehydration.Becauseplasmavolumeis
reduced,RBCcount(numberofcells/plasmavolume)appearstobeelevated,butRBCmass
(absolutenumberofRBCs)remainsunaltered.Bloodoxygenlevelsarenormal.Youshouldknow
howtodifferentiatebetweenthesecausesofpolycythemiaforStep1,asthisisanotherhighyield
topic.

6 Whatistheclassicpresentationforessentialthrombocytosis?

PatientswithessentialthrombocytosispresentwithmanyofthesamesymptomsseeninPV,such
aserythromelalgia,bleeding,pruritus,andsplenomegaly.Theyhaveplateletcountsgreaterthan
600,000/LandmayhaveassociatedincreasesinRBCsandgranulocytes.Aperipheralsmear
classicallyshowslarge,hypogranularplatelets,andthebonemarrowaspirationalsoshows
megakaryocytichyperplasiabutnormalironstores.
Note:AmajorityofPVcasesandhalfofessentialthrombocytosiscasesoccursecondaryto
mutationsinJAK2,whichisinvolvedinhematopoieticgrowthfactorsignaling.

7 Whatconcernswouldyouhaveifherbonemarrowcellsrevealedthe
Philadelphiachromosome?
Asdiscussedinapriorcase,thepresenceofthePhiladelphiachromosomewouldbeconcerningfor
CML.Thispatientwouldlikelyhaveincreasedneutrophilsandmetamyelocytesonperipheralsmear,
reducedserumLAP,andabonemarrowaspiratewithahighmyeloiderythroidratio.

Myeloproliferativedisorderssuchaspolycythemiaveraandessentialthrombocytosis
oftenpresentwithsymptomssuchasitchingandburningaswellasamarked
proliferationinasinglecellline.
Secondarypolycythemiaiscausedbyhypoxemia,renaltumors,exogenous
erythropoietin,Cushingsyndrome,orcarboxyhemoglobinemia.
SummaryBox:MyeloproliferativeDisorders

Case146
A75yearoldmanisevaluatedforworseningfatigueandpersistentfever.Onphysical
examination,younoticemanybruisesonhisarmsandlegs,whichheadmitsisarelativelynew
phenomenonthathehasattributedtogettingold.Nolymphadenopathyor
hepatosplenomegalyisnoted.

1 Whatisthedifferentialdiagnosisforacombinationoffatigue,easybruising,
andrecurringfeverorinfectioninanolderman?
Thedifferentialdiagnosisisbroadbutforthemostpartcomprisesinfection,malignancy,and
vasculitis.

Case146continued:
BloodworkshowsHb9.5mg/dL,Hct35%,plateletcount62,000,andWBCcount50,000with
30%blastcellsaperipheralbloodsmearisshowninFigure145(f0030).

2 Whatcharacteristicsofthesmearaidthediagnosis?
ThesmearinFigure145(f0030)showscirculatinggranularleukocytesknownasmyeloblasts,with
eosinophilicneedlelikeinclusionscalledAuerrods.Auerrodsandperipheralblastcellsare
indicativeofAML.

Figure145
PeripheralbloodsmearfrompatientinCase145.
(FromGoldmanL,AusielloD:CecilTextbookofMedicine,22nded.Philadelphia,WBSaunders,2004.)

YoushouldbeabletorecognizeAuerrodsonaperipheralbloodsmear.Thisisahighyield
imagefortheUSMLE.
Step1Secret

3 Howdohislaboratoryvalueshelptoexplainhispresentingsymptoms?
Anemiaisthemostlikelycauseofhiscomplaintoffatigue,andthrombocytopeniaexplainshiseasy
bruising.AlthoughhehasanexcessofWBCsduetoAML,thesecellsarenonfunctional,causinga
clinicalneutropeniawithconsequentfevers/infections.

4 Whatcytochemistrytestsareusedtohelpdiagnoseacutemyelogenous
leukemiaandtodistinguishamongacutemyelogenousleukemiasubtypes
accordingtotheFrenchAmericanBritishclassificationsystem?
Clusterdesignations(CDs),myeloperoxidase(MPO)andnonspecificesterase(NSE),periodicacid
Schiff(PAS),andSudanblack(SBB),asdepictedinTable148(t0045),shouldbeperformed.

Case146continued:
BonemarrowbiopsyshowedapredominanceofpromyelocyteswithstronglypositiveMPOand
negativeNSE.Fluorescenceinsituhybridization(FISH)revealsatranslocationbetween
chromosomes15and17intheaffectedcells.Thetranslocationinvolvestheretinoicacid
receptorgeneonchromosome15.Thepatientisdiagnosedwithacutepromyelocytic
leukemia(PML),FrenchAmericanBritish(FAB)classificationofM3.

Table148
CytochemistryTestsUsedtoDiagnoseAcuteMyelogenousLeukemiaandDifferentiateAcuteMyelogenous

LeukemiaSubtypes

FAB
Incidence Morphology
Subtype
M0

2%

Classical
Characteristics

Undifferentiated Diploid

DiagnosticFeatures

Cytochemistry

20%blastsoftotal

<3%reactive

nucleatedcell,

toMPOor

immunophenotyping

SBB

CD33andCD13
M1

1018%

Acute

Auerrods

20%blasts

myeloblastic

>3%reactive
toMPOor
SBB

M2

2729%

Acute

Auerrods

myeloblastic
M3

510%

20%blasts,t(8:21)

MPO+,SBB+,

chromosomeabnormality NSE+

Acute

>30%

20%blastsand

MPO+,SBB+,

promyelocytic

promyelocytes,

abnormalpromyelocytes PAS,NSE

Auerrods

intenseMPOandSBB
reactivityt(15:17)
cytogeneticabnormality

M4

1625%

Myelomonocytic Myelomonocytic Associatedwithinv(16)


blasts

M5

1322%

Monocytic

M6

13%

Erythroleukemia Dysplastic

PAS+,>20%

chromosomeabnormality NSE+

Monoblasts

NSE+
50%erythroid

PAS+,MPO+

precursors
M7

13%

Megakaryocytic Megakaryocytes Infants<1yearoldwith


t(122),myelofibrosis,
Downsyndrome

CD,clusterdesignationFAB,FrenchAmericanBritish[classification]MPO,myeloperoxidaseNSE,nonspecific
esterasePAS,periodicacidSchiffSBB,SudanblackB.
ModifiedfromHoffmanR,BenzEJJr,ShattilSJ,etal:Hematology:BasicPrinciplesandPractice,4thed.,
Philadelphia,ChurchillLivingstone,2005.

5 Whatisthestandardtreatmentforpromyelocyticleukemia?
Asmentionedearlierinthebasicconceptssectionofthischapter,PMLisuniqueinthatinduction
therapy(usuallyanthracycline)iscombinedwithalltransretinoicacid(ATRA),whichbindstothe
mutatedRARgeneandpromotesdifferentiationandthuseventualcelldeath.Arsenicisusedfor
refractorycases.

Auerrodsonaperipheralbloodsmeararepathognomonicforacutemyelogenous
leukemia(AML).
CytochemistrytestsandcellmorphologicappearanceareusedtosubclassifyAML
patients.Subclassificationsoftenhavedifferentclinicalcoursesandtreatments.
At(15,17)involvingtheretinoicacidreceptorgeneonchromosome15isuniqueto
acutepromyelocyticleukemia(APML),FrenchAmericanBritish(FAB)classificationof
M3.Itstreatmentusingalltransretinoicacid(ATRA)isequallyunique.
SummaryBox:AcuteMyelogenousLeukemia

Copyright2015Elsevier,Inc.Allrightsreserved.

BOOKCHAPTER

Hepatology
EricB.Roth,ThomasA.BrownMDandSonaliJ.Shah
USMLEStep1Secrets,Chapter7,164195

ManymedicalstudentsstrugglewithhepatologyontheUSMLEStep1,andunderstandablyso.
Althoughafairnumberofquestionswillbestraightforward,certainhepatologyrelatedtopics
onboardswilldemandthatyoumakefinedistinctionsbetweenvariousclinicalfindingsand
laboratoryresultstoarriveatthecorrectdiagnosis.Someofthesehighyielddetailsarecovered
butnotexplainedwellinFirstAid.Notsurprisingly,thestudentswhodobestonhepatology
aretheoneswhoknowhowtoanalyze(andnotjustmemorize)thesedetails.Therefore,itis
ourgoaltoteachyouhowtoapproachconfusinghepatologyrelatedtopicswithinthischapter.
Forthoseofyoulookingtoturnhepatologyintoastrengthontestday,wesuggestthefollowing
approach:ReadthroughtheFirstAidsectionandthenstudythischaptercarefully.Youshould
thengobackandthoroughlystudyFirstAidagain,butthistime,youshouldannotateyour
copyofFirstAidwithnotesfromthischapter.Besuretoincludedetailedinformationfromthe
subjectswehavelaidoutinboxeswithinthischapter.Thiswillprepareyoutotackle
hepatologyonStep1.
Insider'sGuidetoHepatologyfortheUSMLEStep1

Basicconcepts
1 Reviewtheanatomyofthehepaticlobuleandportaltriad.Inwhatmannerdo
bloodandbileflowthroughalobule?
Acentralhepaticveinislocatedatthecenterofeachhepaticlobule.Multipleportaltriads(hepatic
artery,portalvenule,bileduct)surroundthiscentralvein.Hepatocytesarearrangedinsheetsof
singlecellthicknessandaresurroundedbybloodfilledsinusoids.Bloodflowsfromthehepatic
arteryandportalveintowardthecentralveinthroughthesinusoids.Bileisformedbythe
hepatocytesandemptiedintobilecanuliculiinthelateralwallofthehepatocyte.Thebileflowsfrom
heretowardthebileducts(Fig.71(f0010)).

Figure71
Diagrammaticrepresentationofahepaticlobule.Acentralveinislocatedinthecenterofthelobule,withplates
ofhepatocytesdisposedradially.Branchesoftheportalveinandhepaticarteryarelocatedontheperipheryof
thelobule,andbloodfrombothperfusesthesinusoids.Peripherallylocatedbileductsdrainthebilecanaliculi
thatrunbetweenthehepatocytes.
(FromBloomW,FawcettDW:ATextbookofHistology,10thed.Philadelphia,1975,WBSaunders.)

Bloodflowsfromtheportaltriadtowardthecentralvein,andbileflowsfromthehepatocytestoward
theportaltriads.
Knowingthestructureofthehepaticlobuleisclinicallyrelevant.Thehepatocyteslocatednearthe
portaltriad(zone1)areclosesttotheoxygenatedbloodsupplyandarethusthefirstcellsaffectedby
toxinsthatreachtheliverviathebloodstream.Ontheotherhand,hepatocytesclosesttothecentral
vein(zone3)arethefurthestfromtheoxygenatedbloodsupplyofthelobuleandarethusthefirst
cellsaffectedbyischemia.

2 Whatisthechemicaldifferencebetweenconjugatedandunconjugated
bilirubinandhowarethesesubstancesformed?
Bilirubinisabreakdownproductofthehememoietyfoundinredbloodcells(RBCs),bonemarrow,
liver,andmitochondrialcytochromeenzymes.Unconjugatedbilirubinisthebreakdownproduct
formedintheperipheraltissues.Conjugatedbilirubinthenisformedintheliverbyconjugating
glucuronicacidtobilirubintomakeitmoresoluble.
Note:Jaundiceisayellowishdiscolorationoftheskin,mucusmembranes,andscleradueto
elevatedlevelsofeitherconjugated(direct)orunconjugated(indirect)bilirubin.

3 Whyisunconjugatedbilirubinnotnormallyexcretedintheurine?

Unconjugatedbilirubinishydrophobicandcirculatesboundtoalbumin.Albumin,anegatively
chargedprotein,cannotcrossahealthyglomerularbasementmembranebecausethe
glycosaminoglycansthatformthismembranearenegativelychargedandrepelthealbumin.

4 Whatarethemaincausesofjaundiceandhowdoeseachaffectthetypeof
hyperbilirubinemiaobserved?
SeeTable71(t0010)forthecausesofjaundiceandthecharacteristicsofeach.
Youshouldbeabletodifferentiatebetweenthecausesandpresentationsofconjugatedand
unconjugatedhyperbilirubinemiasshowninTable71(t0010).Thisisacommonlytested
principleonboards.ForthoseofyouwhodonotfullyunderstandthedetailsofTable71(t0010)
,wehaveprovidedourhandyapproachtoreasoningthroughthecausesofjaundice.

Secrettodiagnosingcommoncausesofjaundice
Forthoseofyouwhostillstrugglewiththisconcept,wewillattempttoexplainoureasymethod
forapproachingevaluationofjaundice.Inordertounderstandjaundice,youmustfirst
understandthepathwayofbilirubinformationandexcretion.Redbloodcell(RBC)breakdown
leadstotheformationofunconjugatedbilirubin,whichisboundtoalbumininthe
bloodstream.Thisunconjugatedbilirubinisreferredtoasindirectbilirubinandiswater
insoluble(therefore,itcannotbeexcretedintourine).Theindirectbilirubinisthentakenupby
theliverandconjugatedtoglucuronicacidbytheenzymeuridinediphosphate(UDP)
glucuronyltransferase.Thisformsawatersolubleproductcalleddirectbilirubin.Direct
bilirubinisthenexcretedintothebilethatisformedintheliver.Bileitselfiscomposedofbile
salts,bilirubin,phospholipids,cholesterol,electrolytes,andwater.Itisstoredinthe
gallbladder,whereitisconcentrated.Whenbileissecretedintothegutlumen,theconjugated
bilirubinisdeconjugatedbybacterialfloraintourobilinogen.Urobilinogenhasthreepossible
fates:(1)itisexcretedintofeces,whereitgivesthecharacteristicbrowncolorofstool(2)it
returnstotheliverviatheenterohepaticcirculationor(3)itisreabsorbedviathesystemic
circulationandexcretedbythekidney,givingthecharacteristiccolorofurine.
Nowthatyouhavethebackgroundonbilirubinformationandexcretion,wecanbeginto
explorethecausesofjaundice.Jaundicemaybetheresultofanyabnormalityalongthe
aforementionedpathway.Itcanthereforeoccurasaresultof(1)excessivebilirubinproduction,
(2)decreasedhepaticuptakeorconjugationofindirectbilirubin,(3)decreasedhepatocellular
secretionofbilirubinintobile,or(4)impairedorobstuctedbileflow.Ifthismakessenseto
you,itbecomesveryformulaictoteaseapartthevariouscausesofjaundice.Allyouhavetodo
ismatchthevariouscausesofjaundicelistedinTable71(t0010)tothesebasicmechanisms.
Let'sstartwithincreasedbilirubinproduction(item1inourprecedinglist).Themostnotable
causeofincreasedbilirubinproductionishemolyticjaundice,whichleadstoanunconjugated
hyperbilirubinemia.Whydoesthisoccur?HemolysisreferstoacceleratedbreakdownofRBCs,
whichrapidlyincreasesindirectbilirubinlevelsinthebloodstream.Theliver,whichmust

uptakeandconjugateallofthisbilirubin,hastroublekeepingupwiththerapidrateofbilirubin
production.Asaresult,indirectbilirubinlevelsincreaseinthebloodstream.Donotconfuse
thiswithhepatocellularjaundice.Theliver,inthiscase,isperfectlyfunctional!Itcanuptake
andconjugatebilirubin,butitcannotdosoattherequiredpace.Infact,absoluteamountsof
conjugatedbilirubinandurobilinogenincreaseabovenormalbecauseoftheincreased
productionandconjugationofindirectbilirubin.Thebilirubinthatisconjugatedisresponsible
formaintainingurobilinogenconcentrationsinurineandfeces.Thus,botharenormally
colored.
Decreasedhepaticuptakeandconjugationofbilirubin(items2and3inourlist)areadditional
causesofjaundice.Considerascenarioinwhichtheliverdoesnotfunctionnormally,suchas
viralhepatitis.Thisleadstohepatocellularjaundicebecausethesickliverisunableto
performitsnormaltaskofconjugatingbilirubin.Somebilirubinwillbeconjugated(thus
maintainingthenormalcolorofurineandfeces),butindirectbilirubinlevelswillalsoincrease
abovenormal.Notethatconjugatedbilirubinintheliveralsoleaksoutintothebloodstream
throughthedamagedhepatictissue.Itisnevernormaltoseebilirubin(whetherindirector
direct)inthebloodstream.Ifthisfindingappearsonlaboratorytests,itisaredflagfordisease.
ThefinalcauseofjaundicementionedinTable71(t0010)isobstructivejaundice.Obstructive
jaundiceresultswhenconjugatedbilirubinisunabletobeexcretedintotheguteitherdueto
impairedliversecretionofbile(seeitem3)orimpairedbileflow(seeitem4).Themost
commoncauseofobstructivejaundiceisbileductobstruction(e.g.,gallstones,pancreatic
tumor).Althoughconjugatedbilirubinisproducedandsecretedintobile,thebileisunableto
besecretedintothegutlumenduetoanobstructioninthebileductsystem.Bilebacksupinto
theliver,causingengorgementandruptureofintrahepaticducts.Thisleadstospillageof
conjugatedbilirubinintosinusoidalbloodand,ultimately,thesystemiccirculation.Thus,
conjugatedbilirubinlevelsbecomeelevatedwhileurobilinogenlevelsdecreasedueto
inadequateconcentrationsofconjugatedbilirubininthegutlumen.Urinecolorremains
normal(conjugatedbilirubininthesystemiccirculationiswatersolubleandcanbeexcretedby
thekidneys),butstoolbecomesclaycoloredduetolackofurobilinogenexcretionintofeces.
Nowthatyouhaveabetterunderstandingofthecausesofjaundice,revisitTable71(t0010)and
attempttofillitinonyourown.
Step1Secret
Table71
CausesofJaundice

Jaundice
Type

Hyperbilirubinemia

Urine
Bilirubin
Levels

Urobilinogen
Formation

UrineandStool
Color

Hemolytic

Unconjugated(<20%
conjugatedbilirubin)

Normal

Hepatocellular Conjugated/unconjugated(20
50%conjugatedbilirubin)

Normal

Obstructive

Normalurine,clay
coloredfeces

Conjugated

5 Whichveinsfeedintotheportalvein?
Venousreturnfromtheforegut,midgut,andhindgutfeedsintotheportalveinfromthegastric
veins,splenicvein,andsuperiorandinferiormesentericveins.Consequently,portalhypertension
cancausevenouscongestioninanyandallofthesevascularbeds(e.g.,congestivesplenomegaly
fromsplenicvein,esophagealvaricesfromgastricveins).

6 Whatarethesymptomsofportalhypertension?
Portalhypertensionleadstoincreasedresistancetoflowinthesystemicvenoussystem.Asaresult,
bloodcannotpassfreelyfromtheportalsystemtothesystemicsystemandbacksupintothe
portacavalanastomoses,whichcausesthemtobecomeengorged,dilated,orvaricose.Thelocationof
theseanastomosesdeterminesthespecificsymptomsthatresultfromportalhypertension.These
symptomsandsignsarelistedinTable72(t0015).Generalsymptomsofportalhypertensioninclude
ascites(secondarytoincreasedhydrostaticpressure),spontaneousbacterialperitonitis(notethat
asciticfluidisawonderfulculturemediumforbacteria),hepatorenalsyndrome,andsplenomegaly
duetodecreaseddrainageofvenousbloodfromthespleen.Splenomegalycanresultinanemia,
thrombocytopenia,orpancytopeniaduetocellularsequestrationwithintheengorgedspleen.
Table72
PortalHypertension:AnastomosesandRelatedSigns

PortacavalAnastomosis

ClinicalSign

Leftgastricveinwith
esophagealvein(branch
ofazygosvein)

Esophagealvarices(leadingtoheavybleeding/hematemesis)

Paraumbilicalveinwith
epigastricvessels

Caputmedusae

Superiorrectalveinwith
middleandinferiorrectal
veins

Internalhemorrhoids(unlikeexternalhemorrhoids,thesearenot
painfulbecausethevisceralnervesthatareabovethedentateline
sensepressureandnotpain)

Note:Portalhypertensionresultsfromprehepatic,intrahepatic,andposthepaticcauses.Cirrhosisis
acommoncauseofintrahepaticportalhypertension,whileportalveinthrombosisisaprehepatic
cause.RightsidedheartfailureandBuddChiarisyndrome(seenextquestion)arecommon
precursorstoposthepaticportalhypertension.
Youshouldmemorizeeachportacavalanastomosisandthesymptomsthatresultfromportal
hypertension.
Step1Secret

7 HowcanBuddChiarisyndromeariseandwhatcanitleadto?
BuddChiarisyndromeoccurswhenthehepaticvenousoutflowbecomesobstructed,usuallybecause
ofthrombosis.Polycythemiavera,pregnancy,andclottingdisorderspredisposetothrombus
formationandcanleadtohepaticvenousoutflowobstruction.Theobstructionmayresultinportal
hypertensionwithaclassicpresentationofabdominalpain,ascites,andhepatomegaly.Althoughthis
maybeconfusedwithrightsidedheartfailure,BuddChiarisyndromedoesnotcausejugularvenous
distentionbecauseitaffectsonlytheinferiorvenacava.

8 Whatarethecommonliverbiochemicaltestsandwhatdotheyindicate?
Themostcommonlaboratorytestsorderedaretheaminotransferases(alaninetransaminase[ALT],
aspartatetransaminase[AST]),alkalinephosphatase(ALP),glutamyltranspeptidase(GGTP),
bilirubin,prothrombintime(PT),andalbumin.SerumproteinconcentrationandPTtestsevaluate
syntheticfunctionoftheliverbecausealbumin(thepredominantserumprotein)andclottingfactors
aremanufacturedbyhepatocytes.PT,inparticular,isnotgenerallyelevateduntilsevereliverdisease
hasoccurred.Serumbilirubinconcentrationisusedtoevaluatefunctionalclearancebytheliver.
ALT,AST,ALP,andGGTPmoreaccuratelyreflectliverinjury.SerumelevationsinALTandAST
resultfromleakageoftheseenzymesfromdamagetohepatocytes.ALTisgenerallyelevatedmore
thanASTwithviralinfection,butASTiselevatedmorethanALTwithalcoholabuse.Serum
elevationsinALPandGGTPareseenwhenincreasedproductionoftheseenzymesareinducedby
bileductdamage.ALPislessspecificforbileduct/liverdamagethanGGTP,asitisoftenelevated
duringboneremodeling.BothALPandGGTPlevelsshouldbeobtainedifbileductdamageis
suspected.GGTPlevelsarealsoelevatedwithalcoholabusebecauseitisamarkerofmitochondrial
damage.Asageneralruleofthumb,ALTandASTshowgreaterelevationinhepatitis,andALPand
GGTPshowgreaterelevationincholestaticdisease.
Itisexpectedthatyouknowthesignificanceofabnormalliverfunctiontest(LFT)resultsfor
boardsandforyourclinicalyears.Asillytrickforrememberingthatalaninetransaminase
(ALT)iselevatedwithviralhepatitisistopicturetheALTkeyonthekeyboardofaVIRALLY
infectedcomputer.
Step1Secret

Case71
Youareworkingintheemergencydepartmentwhenamanpresentsat5amvomitingblood
(hematemesis).Atfirstglance,youcanseethathismentalstatusisimpaired,hisskinis
jaundiced,andhehasscleralicterus.Becauseyourecognizehimasthemanyoufrequentlysee
drinkingfromabrownpaperbaginthepark,youthinkyouknowwhyheishere.

1 Whatisthemostcommoncauseofuppergastrointestinalbleeding?
Themostcommoncausesofuppergastrointestinal(GI)bleedingareulcersorerosions.Thesecan
occurintheesophagus,stomach,orduodenumanddevelopwhenacidsecretionoverrunsprotective
factors(mucusandbicarbonatesecretion).InfectionwithHelicobacterpylori,nonsteroidalanti
inflammatorydrug(NSAID)use,andcigarettesmokingalldisrupttheprotectivefactors.Inother
cases,thereishypersecretionofgastricacid,suchasseeninZollingerEllisonsyndrome
(gastrinoma).OthercausesofupperGIbleedingincludeMalloryWeisstearandruptured
esophagealorgastricvarices.

2 WhyshouldaMalloryWeisstearbeincludedinthedifferentialdiagnosis?
MalloryWeisstearsareseenmostcommonlyinalcoholics,andareduetoexcessivevomitingthat
causesmucosallacerationsthatextendthroughthegastroesophagealjunction.Itisanothercommon
causeofhematemesisinalcoholicsorpeoplewithconditionscausingexcessivevomiting(e.g.,
bulimia).However,MalloryWeisstearsdonottypicallycausemassivehematemesislikethatseen
withrupturedesophagealvarices.

3 Whatisscleralicterus?
Scleralicterusisyellowdiscoloration(icterus)seeninthewhitesoftheeye(sclerae).Itisan
indicationofincreasedunconjugatedbilirubinintheserum,andisoftenthefirstplaceyouwillsee
jaundiceonphysicalexamination.Jaundiceusuallyindicateshepaticorcholestaticpathologybut
mayindicatebleedingorhemolysis.
Note:VitaminAtoxicitycancauseyellowdiscolorationoftheskin,whichisdistinctfromthescleral
icterusobservedwithjaundice.

4 Toconfirmyoursuspicionaboutsevereliverdiseaseinthispatient,what
physicalexaminationfindingsmightyouexpectandwhy?
Gynecomastiaandtesticularatrophycanbepresent,resultingfromimpairedabilityofthedamaged
livertometabolizeestrogen.Estrogenalsoweakensvascularwalls,leadingtospiderangiomatawith
increasedestrogenlevels.Hemorrhoidsmayresultfromaportacavalanastomosisbetweenthe
superiorrectalveinandtheinferiorrectalvein,andacaputmedusae(engorgedveinsradiatingfrom
theumbilicus)canresultfrombloodbeingdivertedfromtheportalveinintotheperiumbilicalveins
thatrunalongtheroundligamentofthelivertotheanteriorabdominalwall(seeTable72(t0015)).

Anenlargedlivermaybepalpatedwithalcoholichepatitis,butoncecirrhosisdevelops,theliverwill
becomefirmandshrunken.Aspleentipmaybepalpable,ascongestivesplenomegalymayoccurwith
portalhypertensionbecausethesplenicveindrainsintotheportalvein.

Case71continued:
Onexaminationyoudoindeedfindspiderangiomataonhisfaceandthorax,gynecomastia,
andaperiumbilicalcaputmedusae.Hisabdomenisdistended,hisspleenisenlarged,hehas
pedalandperiorbitaledema,andhisbreathhasasweet,ammoniacalodor(fetorhepaticus).

5 Whatisthepathophysiologyofhisascites,pedaledema,andperiorbital
edema?
Insevereliverdiseasethereisinadequateproductionofserumalbumin,themajordeterminantof
plasmaoncoticpressure.Consequently,fluidreabsorptionfromtheinterstitiumbackintothe
capillarybedsisreduced.Thisexplainshispedalandperiorbitaledema.Inadditiontothereduced
capillaryoncoticpressure,theincreasedvenouspressureintheportalsystemfromportal
hypertensioncausesgreaterintracapillaryhydrostaticpressure,whichopposesmovementoffluid
fromtheinterstitiumintothecapillariesandresultsinascites.

6 Whatisthepathogenesisofthesuspectedcauseofhematemesisinthis
patient?
Hemostlikelyhasrupturedesophagealvarices(seeTable72(t0015)).Thispatienthasportal
hypertensionsecondarytoalcoholinducedcirrhosis.Thiscreatesportacavalanastomoses,inwhich
thepressureintheportalvenoussystemdivertsbloodfromtheportalsystemintothesystemic
circulationatsiteswherethereareanastomoses.Inthispatient'scase,bloodfromthegastricveins
backedupintohisesophagealtributaries,whichbecamedistendedandeventuallyruptured(Fig.72
(f0015)).

Figure72
Diagramoftheportalcirculation.Themostimportantsitesforthepotentialdevelopmentofportosystemic
collateralsareshown.A,Esophagealsubmucosalveins,whicharesuppliedbytheleftgastricveinanddraininto
thesuperiorvenacavaviatheazygousvein.B,Paraumbilicalveins,whicharesuppliedbytheumbilicalportion
oftheleftportalveinanddrainintoabdominalwallveinsneartheumbilicus.Theseveinsmayformacaput
medusaeattheumbilicus.C,Rectalsubmucosalveins,whicharesuppliedbytheinferiormesentericvein
throughthesuperiorrectalveinanddrainintotheinternaliliacveinsthroughthemiddle(mid.)andinferior(inf.
)rectalveins.D,Splenorenalshunts,whicharecreatedspontaneouslyorsurgically.E,Shortgastricveins,which
aresuppliedbytheesophagealsubmucosalveinsanddrainintothesplenicvein.
(FromFeldmanM,FriedmanLS,BrandtLJ:Sleisenger&Fordtran'sGastrointestinalandLiverDisease,8thed.
Philadelphia,WBSaunders,2006.)

Note:Theroundligamentoftheliver(ligamentumtereshepatica)isanembryologicremnantofthe
umbilicalvein.Themajormorphologiccharacteristicsofcirrhosisareextensivefibrosiswithnodules
ofregeneratinghepatocytes.

Case71continued:
Whenanintravenous(IV)lineisstartedtoadministerfluids,afairlylargehematomadevelops
attheIVsite.Laboratorytestsrevealanelevateddirectbilirubin,indirectbilirubin,andPT
lowbloodureanitrogen(BUN)andanormalcreatininelevel.SerologictestsforhepatitisB
andC,aswellasantimitochondrialantibodies,arenegative.Serumiron,transferrin,iron
saturation(%),ferritin,andceruloplasminareallwithinnormallimits.

7 Whatisthevalueofthefollowingtests:hepatitisserology,serumiron,
ceruloplasmin,andantimitochondrialantibodies?
Thesetestsallidentifydifferentcausesoflivercirrhosis.HepatitisBandCbothcancauseliver
cirrhosis,ascanhemochromatosis(toomuchiron),Wilson'sdisease(toomuchcopper),andprimary
biliarycirrhosis(antimitochondrialantibodies).
Note:Alloftheabovelistedcausesoflivercirrhosisincreasetheriskforhepatocellularcarcinoma
(HCC),asdo1antitrypsindeficiencyandaflatoxinexposure.

8 Assumingheisnottakinganyanticoagulants,whatisthemostlikelyreason
thispatientdevelopedalargehematomaattheintravenoussite?

Theliveriswheremostoftheclottingfactorsareproduced,andsome(factorsI,II,VII,IX,X)are
modifiedposttranslationally.Severeliverdiseaseimpairstheirproductionandprocessing,
producingacoagulopathy.NoticethathisPTwaselevatedbecauseofthis.

9 Listallthelaboratoryfindingsyouwouldexpectinapatientwithliverfailure
SeeTable73(t0020)forthelaboratoryfindingsinliverfailureandtheirunderlyingmechanisms.
Table73
CommonFindingsinPatientsWithLiverFailure

LaboratoryValue

Mechanism

Elevatedor
normalLFT

Liverenzymesmaybeelevatedduringinitialdamage,butifcirrhosisispresent
orthelivershrinksovertime,liverenzymelevelsmayappearnormalinthe

values

contextofdecreasedhepatictissue.

ElevatedPT

Theliveristhesiteofcoagulationfactorproduction.Withsevereliverdamage,
PTbecomeselevated.

Elevatedserum
bilirubin

Theliverisresponsibleforbilirubinuptake.Liverfailurecausesaspikein
serumbilirubinconcentrationduetodecreasedhepaticuptake.

concentration
Hypoalbuminemia Theliveristhepredominantsiteofalbuminproduction.Decreasedserum
proteinconcentrationmayclinicallymanifestasascites.
Fasting
hypoglycemia

Impairedgluconeogenesisandglycogenolysisduringfasting.

Elevatedestrogen Theliveristhesiteofestrogenbreakdown.Liverdamageelevatesestrogen
levels
levels,whichcanleadtotesticularatrophyandformationofspiderangiomata.
Elevated

Theliverproducestheenzymesinvolvedintheureacycle,whichconverts

ammonialevels
withdecreased

ammoniatourea.Elevatedammonialevelscanresultinhepatic
encephalopathy,markedbyconfusion,lossofconsciousness,asterixis,

BUN

irritability,tremor,andcoma.Increasedammoniaalsocanresultinfetor
hepaticus(breathofthedead),whichischaracteristicofliverdisease.

ADH,antidiuretichormoneBUN,bloodureanitrogenLFT,liverfunctiontestPT,prothrombintime.

10 Wouldyouexpecttheasciticfluidtobeatransudateoranexudate?
Atransudatedevelopsasaresultoffluidmovingacrossamembraneasaresultofhemodynamic
forces.Becausethereisnoalterationinthepermeabilityofthemembranesthatthefluidismoving
across,thefluidthataccumulateshaslowproteincontent.Incontrast,exudatesarefluidcollections
thatdevelopbecauseofalterationsinmembrane/vesselpermeability,soproteinsandcells

accumulateinthesefluidcollections.Becausethisascitesiscausedbyloweredoncoticpressure
(secondarytohypoalbuminemia)andportalhypertension,bothofwhichalterhemodynamicforces
butnotvesselpermeability,theresultisatransudate.Table74(t0025)liststhedifferencesbetween
transudatesandexudates.
Transudatesandexudatescanbothresultinedema.Youshouldknowthedifferencebetween
transudatesandexudatesandthecausesofeach.ThesecomparisonsarelistedforyouinTable
74(t0025).
Step1Secret
Table74
TransudatesVersusExudates

FluidType

Mechanism

Transudate Disturbancesofhydrostatic
oroncoticpressures

Common
Causes

Characteristics

Nephrotic

Clearfluid,proteinpoor,specificgravity

syndrome
<1.012,fluid[LDH]/plasma[LDH]ratio<
Liverfailure 0.6,oftenresultsinpittingedema
CHF

Exudate

Increasedvessel

Inflammation Cloudyfluid,proteinrich,specificgravity

permeabilityoftenmediated Septicshock >1.020,fluid[LDH]/plasma[LDH]ratio>


byacutephasecytokines
0.6
CHF,congestiveheartfailureLDH,lactatedehydrogenase.

11 Howdoeslivercirrhosiscausethefollowingabnormalities?
A.Unconjugatedandconjugatedhyperbilirubinemia
Becausetherearefewerfunctionalhepatocytesthereisareducedabilitytotakeupandconjugate
bilirubin,whichresultsinanunconjugatedhyperbilirubinemia.Additionally,bilirubinthatis
conjugatedmayleakbackintothebloodstreamduetohepatocytedamage.Thisconceptisexplained
infurtherdetaillaterinthechapter.
B.Reducedbloodureanitrogen,fetorhepaticus,andmentalstatusabnormalities
Theliverisamajorsiteofaminoacidmetabolismandisthesiteoftheureacycle.Reducedoutputof
theureacyclebecauseofhepatocytedestructionresultsinalowerBUN.Becausetheureacycleis
alsothemajorsiteofammoniadetoxification,elevatedbloodammonialevelsbecomedetectableasa

sweetodorinthebreath(fetorhepaticus).Theelevatedbloodammonia,whichcanenterthebrain,
canaltercerebralmetabolismandcontributetoconfusion(hepaticencephalopathy)(seeTable73
(t0020)).

12 Howcananacutealcoholbingecauseafattylivertodevelop?
Thisisalsocalledhepaticsteatosisandresultsfromtheshuntingofsubstratestolipidbiosynthesis,
impairedsecretionoflipoproteinsfromtheliver,andincreasedperipheralcatabolismoffat.Itis
reversible.

13 Howdoeschronicalcoholconsumptionleadtoamorerapidcatabolismof
ingestedalcohol?
Alcoholcanbedegradedbytwoseparatemetabolicpathways,asshowninFigure73(f0020).

Figure73
Thetwopathwaysforalcoholmetabolism.MEOS,microsomalethanoloxidizingsystem.
(CourtesyofGloriaYueh,PhD,MidwesternUniversity.)

Thepathwaythatbeginswithalcoholdehydrogenase(ADH),inthecytosolofmosttissues,is
constitutivelyactiveandcanonlymetabolizeafixedamountofalcohol.Individualswhodonotabuse
alcoholrelyprincipallyonthispathwayformetabolism.Incontrast,themicrosomalethanol
oxidizingsystem(MEOS)pathwaybeginsinlivermicrosomeswithenzymesthatareinducedby
alcoholandotherdrugs.Suchinductionenhancesthecapacitytometabolizeethanol.Inalcoholics,
theMEOSpathwayissubstantiallyupregulated.

14 Whyareethanolandfomepizoleusedtotreatmethanolpoisoningand
ethyleneglycolpoisoning?
Methanolandethyleneglycolaremetabolizedthroughthesamepathwayasethanol,andthe
intermediatesubstancesthatareformedinthisprocess(formaldehydefrommethanolandoxalic
acidfromethyleneglycol)areverytoxic.Alcoholcompeteswithbothmethanolandethyleneglycol
formetabolismbyADH,therebyreducingtherateofformationofthetoxicmetabolites.Fomepizole
furtherinhibitsconversionofmethanolorethyleneglycoltotheirtoxicintermediatesbydirectly
inhibitingADH.

Thepathogenesisofhepaticdamageinchronicalcoholabuseispoorlyunderstood,but
theresultisoftenhepaticcirrhosisandportalhypertension.
Alcoholicsteatohepatitis(fattyliver)isinitiallyreversiblebutcanprogresstocirrhosis
withcontinuedalcoholabuse.Cirrhosisisirreversible.
RupturedesophagealvaricesandMalloryWeisstearsareuncommoninthegeneral
populationbutareoftenassociatedwithalcoholicliverdisease.
Asciticfluidresultingfromportalhypertensionisatransudate.Ithasalowprotein
concentrationandresultsfromalterationsinhemodynamicforces.Exudateshavehigher
proteincontentandresultfromalterationsinvesselpermeability,andmostcommonly
resultfrominfection.
Hepaticencephalopathyisinpartcausedbyincreasedlevelsofplasmaammonia.
Becausedietaryproteinincreasesproductionofammonia,patientswithcirrhosismaybe
placedonalowproteindiettominimizetheriskofhepaticencephalopathy.
SummaryBox:AlcoholicHepatitisandRupturedEsophagealVarices

Case72
Thethirdyearmedicalstudentthatyouaresupervisinginyourprimarycareclinicdoesthe
initialhistoryandannualphysicalexaminationonan18yearoldman.Thestudenttellsyou
thatthepatientcomplainsonlyofmalaiseandanorexia.Onphysicalexamination,thepatient
isjaundicedandhasgoldenbrownringsatthelimbusofthecornea(Fig.74(f0025)).This
remindsthestudentofadiseaseforwhichshecannotrecallthename.

Figure74
OcularexaminationofpatientinCase72.(st0145)
(FromGoldmanL,AusielloD:CecilTextbookofMedicine,22nded.Philadelphia,WBSaunders,2004.)

1 Towhatdiseaseisthestudentreferring?

InWilson'sdisease(hepatolenticulardegeneration)thegoldenbrowncornealdeposits(typically
seenthroughslitlampexaminationoftheeyes)aretermedKayserFleischerringsandresultfrom
copperdepositioninthecorneallimbus.

2 Whatlaboratorytestsandfurtherphysicalexaminationcomponentswould
youliketodotostrengthenyoursuspicionforthisdisease?
Inadditiontohepaticenzymes,youwillwanttoorderaserumceruloplasmin,totalserumcopper,
freecopper,andurinecopper.Aslitlampexaminationwouldhelpconfirmthesuspectedeye
abnormality.

Case72continued:
Laboratoryworkuprevealsreducedtotalserumcopperbutanincreasedleveloffreecopper.
Increasedurinarycopperexcretionisalsodemonstrated.Slitlampexaminationconfirmsthe
presenceofKayserFleischerrings.YoursuspicionforWilson'sdiseaseisnowquitehigh.

3 HowisthediagnosisofWilson'sdiseasemadedefinitively?
Aliverbiopsyshowinganelevatedfreecopperconcentrationisrequired.Histologicstainingfor
coppercanalsobedone,butthistestisnotassensitive,andanegativeresultdoesnotexcludethe
diagnosisbecausecoppercanbedepositedheterogeneously.Thebiopsywillalsoshowpiecemeal
necrosisandlymphocytosis,whichcanevolvetocirrhosis.
Note:Totalserumcopperlevelsaredecreasedsecondarytodecreasedceruloplasminlevels.
ElevatedfreecopperlevelsaretoxicandthusresponsibleforthesymptomsobservedinWilson's
disease.

4 WhatisthepathogenesisofWilson'sdisease?
Asmentionedpreviously,deficiencyinceruloplasmin,whichnormallyfunctionstobindplasma
copper,resultsinlowtotalplasmacopperbutelevatedfreeplasmacopper.Itisthiselevatedfree
plasmacopperthatcausesthedisease,resultingindepositionofcopperinthelenticularnuclei
(neurologicsymptoms),cornea(KayserFleischerrings),liver(cirrhosis,HCC),andotherorgans
throughoutthebody.
Note:Thereareover200mutationsoftheATP7BgeneknowntocauseWilson'sdisease,whichis
inheritedinanrecessivemannerandusuallymanifestsatayoungage(between6and20years)
prenatalgeneticscreeningisavailablewithinanaffectedfamilyiftheresponsiblemutationhasbeen
identified.

5 Ifthispatientremainsuntreated,whatneurologicmanifestationsmay
develop?

Becauseofthedegenerationofthelenticularnuclei(putamenandglobuspallidus)inthebasal
ganglia,aParkinsonlikesyndrome,characterizedbytremors,dysarthria,bradykinesia,and
spasticity,candevelop.

6 WhatisthetreatmentforWilson'sdiseaseandhowdoesitwork?
Copperchelationtherapywithlifelonguseofdpenicillamineortrientinehydrochloride,whichare
drugsthathelpremovecopperfromtissue.Takingextrazinc,whichcompeteswithcopperfor
intestinalabsorption,iscommonlyusedincombinationwithcopperchelationtherapy.

Wilson'sdiseaseiscausedbydeficientproductionofthecoppercarryingprotein
ceruloplasmin.Itisinheritedinanautosomalrecessivemanner.
Goldenbrownringsaroundthelimbusofthecornea(KayserFleischerringsresulting
fromcopperdeposition)areanimportantdiagnosticclue.
Laboratoryteststypicallyshowlowserumceruloplasminandtotalcopperbutincreased
freecopper.Urinarycopperexcretionisalsotypicallyincreased.
Aliverbiopsyshowingincreasedcopperconcentrationisthediagnosticgoldstandard.
NeurologicmanifestationsinlaterstagesofthediseaseincludeaParkinsonlike
syndrome.
Treatmentiscopperchelationtherapy.Ifleftuntreated,Wilson'sdiseaseisfatal.
SummaryBox:Wilson'sDisease

Case73
A32yearoldmandevelopsfever,nausea,vomiting,malaise,anorexia,andabdominalpain
withinafewweeksofreturningtotheUnitedStatesfromvacationinginaThirdWorld
country.Healsomentionsthathisurineisdark.

1 Intermsofinfections,whatdoyouincludeinthedifferentialdiagnosisfora
patientwhohasrecentlytraveledoutofthecountry?
Youmustbroadenyourdifferentialdiagnosistoincludediseasesendemictothatarea,suchas
schistosomiasisinsubSaharanAfricainfectionspotentiallyobtainedfromlocalfood,suchasVibrio
choleraeinfectionfromdrinkingcontaminatedwaterinSouthAmericadiseasesassociatedwith
wildlife,suchasplagueincountrieswithinfectedrodentsandinfectionfrominsects,suchas
malariatransmittedbyAnophelesmosquitoesintropicalandsubtropicalcountries.Sexually
transmitteddiseases,suchashumanimmunodeficiencyvirus/acquiredimmunodeficiencysyndrome

(HIV/AIDS),canbemoreabundantinothercountries.Itisestimatedthatupto80%ofsexworkers
insomepartsofAfricaareinfectedwithHIV/AIDS.Thesearejustafewexamples,butitis
importanttoremembertoconsidersuchpossibilitiesintravelers.

Case73continued:
Thepatientstatesthatheatesomeshellfishthatwereharvestedfromabayinwhichsewage
enters.Hewasbittenbyseveralmosquitoesandthinkshemayhavebeeninfectedwith
malaria.Hehadnotreceivedanyimmunizationsbeforeleavingforhisvacation.Onphysical
examination,heisjaundicedandhastenderhepatomegaly.Laboratorystudiesreveala
peripheralsmearnegativeformalaria,markedelevationsofASTandALT,mildlyelevatedALP,
andelevateddirectandindirectbilirubin.Ahepatitisprofilereveals+antiHAVIgM(hepatitis
AvirusimmunoglobulinM),antiHAVIgG(hepatitisAvirusimmunoglobulinG),and
HBsAg(hepatitisBsurfaceantigen).

2 Whatisthediagnosis?
AcutehepatitisAinfectionismostlikely.HepatitisAisanentericallytransmittedvirusthatishighly
endemicinpartsofthedevelopingworldwheresanitationispoor.Itisusuallytransmittedfroman
infectedfoodhandlerwhodoesnotthoroughlywashhandsbeforehandlingfoodotherswill
consume.

3 Whyisthereanelevationofbothdirectandindirectbilirubin?
Inviralhepatitis,theindirecthyperbilirubinemiaiscausedbyreducedabilityoftheinfected
hepatocytestotakeupunconjugatedbilirubin.Thedirecthyperbilirubinemiaiscausedbyleakageof
conjugatedbilirubinfrominfectedhepatocytesintothesystemiccirculation.

4 Explainhowtheresultsofthehepatitisprofilefacilitatethediagnosisofan
acuteinfectionratherthanachronicone
IgMisthefirstantibodyisotypeproducedinresponsetoanewinfectiousagentandremainsinthe
circulationforabout12weeksinhepatitisAinfection.Apreviousinfectionwouldhavebeennegative
forantiHAVIgMandpositiveforantiHAVIgG,becausetheIgGisotypeisproducedlaterinthe
infection,andmemoryBcellsdonotmakeIgMbutgenerallymakeIgG.

5 Shouldhebeconcernedaboutdevelopingachronicinfectionorhepatic
cirrhosis?
No,becauseinthevastmajorityofcases,hepatitisAdoesnotdevelopintoachronicinfectionorlead
tocirrhosis.However,hepatitisBandCvirusescancausebothchronicinfectionandcirrhosis.

6 Howwouldyouexpectliverfunctiontestpatternstodifferbetween
parenchymalliverdiseaseandcholestatic(biliary)disease?

Generally,indiseasesthatprimarilyaffecttheliverparenchyma,bothASTandALTareelevatedtoa
greaterextentthanALPandglutamyltransferase(GGT).Inbiliary(cholestatic)diseases,the
converseisgenerallythecase,withALPandGGTbeingelevatedtoagreaterextentthanASTand
ALT.

7 Nowlet'sreviewsomecharacteristicfeaturesofthedifferenthepatitisviruses
SeeTable75(t0030).
Hepatitisisverycommonlytestedonboards.Youmustknowallthecharacteristicsofthe
differenthepatitisviruses.TheonethatconfusesstudentsmostishepatitisBbecauseitis
associatedwithavarietyofantigensandantibodiesthatareeitherpositiveornegative
dependingonthestageofthedisease.Donotworry!WewilldiscusshepatitisBinextensive
detaillaterinthischapter.
Step1Secret
Table75
CharacteristicsofHepatitisViruses

Hepatitis Typeof
Virus

Transmission Chronicity Cirrhosis Hepatocellular Comments

Virus

Carcinoma
Risk

ssRNA

Fecaloral

No

No

No

dsDNA

Parenteral

Yes

Yes

Yes

ssRNA

Parenteral

Yes

Yes

Yes

Mostcommoncauseof
posttransfusion
hepatitis

ssRNA

RequireshepatitisB
virustoreplicate

ssRNA

Fecaloral

No

No

No

20%mortalityratein
pregnantwomen

dsDNA,doublestrandedDNAssRNA,singlestrandedRNA.

8 CanhepatitisAbeprevented?
AliveinactivatedvaccineforhepatitisAisnowavailable.Ithasbeenaddedtothevaccine
recommendationsoftheCentersforDiseaseControlandPrevention(CDC)forallchildrenover1
yearofage.

HepatitisAiscontractedfromthefecaloralrouteandisoftenassociatedwithfoodsthat
areprocessedinanunsanitaryway,includinginadequatehandwashingbyinfectedfood
handlers.
HepatitisAandEgenerallydonotresultinchronichepatitis.HepatitisBandCboth
havepotentialforchronicdisease.
Thepatternofliverenzymeabnormalitiesvariesdependingonthecauseofliverdisease.
Viralhepatitistypicallycausesamarkedtransaminitis.Alcoholicliverdiseaseclassically
causesanASTelevationthatis2ALT.Cholestaticliverdiseasetypicallycausesgreater
elevationsinALPandGGTthanASTandALT.
AntiHAVIgMispresentforroughly12weeksafteracquiringhepatitisAinfection.Anti
HAVIgGbecomespresentshortlyafterIgMandpersistsforseveralyears,indicatingpast
infectionandprovidingprotectiveimmunity.
AvaccinetopreventhepatitisAisnowavailable.ThereisalsoavaccineforhepatitisB.
NovaccinesarecurrentlyavailableforhepatitisC,D,orE.
SummaryBox:AcuteHepatitisA

Case74
A3dayold,fulltermbabypresentswithjaundicethatstartedonhisfaceandspreadtohis
body.Oncloseinspection,therearenohematomaspresent.Laboratorytestsshowelevated
indirectbilirubin,anegativedirectCoombs'test,normalreticulocytecount(forhisage),and
normalcompletebloodcount(CBC).Enzymeassaysforglucuronyltransferaseactivityare
withinnormallimits.

1 Whatisthemostlikelydiagnosis?
Physiologicjaundiceofthenewbornismostlikely.Theunconjugatedhyperbilirubinemiasuggests
hemolysisoraliverthatcannotadequatelyprocessthebilirubinload.ThenegativedirectCoombs'
testhelpsexcludemoreserioushemolyticdiseasesofnewbornsinwhichthebodyproduces
autoantibodiesagainstthenewborn'snativeRBCs.HemolyticdiseasesofnewbornsalsocauseCBC
abnormalities,includinganemiafromextensivehemolysisandneutropeniaandthrombocytopenia
fromsuppressionofmyelopoiesisandplateletproductioninfavoroferythropoiesis.Normal
glucuronyltransferaseactivityexcludescongenitalenzymedeficiencies.

2 Whydoesphysiologicjaundicedevelop?
IntheprocessofconvertingfromRBCswithfetalhemoglobintoRBCswithadulthemoglobin,there
isanapproximatesixfoldincreaseintheamountofunconjugatedbilirubinpresentedtotheliver.
Theneonatalliveroftendoesnothavethecapacitytocompletelytakeupandconjugatethisamount

ofbilirubinduetoimmatureuridinediphosphate(UDP)glucuronyltransferase,resultingina
transientphysiologicjaundicewithunconjugatedhyperbilirubinemiaandpossiblekernicterus.
Treatmentinvolvesphototherapy,whichconvertstheunconjugatedbilirubinintoawatersoluble
formthatcanbeexcretedintheurine.
Note:Thisconditionismorecommonamongbreastfedbabiesbecausebreastmilkcontains
deconjugatingenzymes.Notreatmentisrequired.

3 Whydidthephysiciancheckforhematomasonphysicalexamination?
BreakdownofRBCsinhematomasandtheattendantbilirubinformationcanbeacauseofjaundice.

4 Whyareanormalreticulocytecountandanormalcompletebloodcount
importantinthediagnosticworkupforthisneonate?
Ahemolyticanemiacancausejaundiceandwillgenerallyshowanelevatedreticulocytecount.

5 Whatisthemostseriouscomplicationofneonataljaundiceandhowdoesit
develop?
Kernicterus,whichisdepositionofinsolubleunconjugatedbilirubininthebrain,canresultwhenthe
bilirubinconcentrationisespeciallyhigh.Kernicterusisdangerousbecauseitcanleadtobrain
damage.

Case74continued:
Themotherandbabyreturntoyourofficewhenthechildis1weekoldforascheduledfollow
upvisitwithalactationconsultant.Thejaundicehasresolved,buttheanxiousfirsttime
motherisconcernedaboutlongtermconsequencesforthebaby.

6 Aretherelongtermrisksassociatedwithphysiologicjaundiceofthe
newborn?
No.Mostnewbornswillappearsomewhatjaundicedinthefirstfewdaysoflifeandthisdoesnot
predisposethemtoanyfuturedisorders,hepaticorotherwise.Themothershouldbereassured.
However,hadthejaundicebeensevereandthebilirubinconcentrationgreaterthan20mg/dL,there
wouldbeanincreasedriskforkernicterus.

7 Wouldyouexpectphysiologicjaundicetobeexacerbatedorattenuatedby
Gilbert'ssyndrome?
Gilbert'ssyndromeischaracterizedbymildlydecreasedUDPglucuronyltransferaseactivity,
resultinginincreasedunconjugatedbilirubinlevels,sothisdiseasewouldexacerbatephysiologic
jaundice.

8 Whatisthehereditarysyndromewithamoreseriousdeficiencyofuridine
diphosphateglucuronyltransferasethaninGilbert'ssyndrome?

CriglerNajjarsyndrome,whichalsocausesanunconjugatedhyperbilirubinemia.Twoformsof
CriglerNajjarsyndromehavebeendescribed.Type1ischaracterizedbyhighlevelsofunconjugated
hyperbilirubinemiaandkernicterus.Type2demonstratesalowerdegreeofhyperbilirubinemiaand
carriesamuchsmallerriskofkernicterus.
Note:Phenobarbitalcanbeusedtotreattype2CriglerNajjarsyndrome,asitupregulatesUDP
glucuronyltransferaseactivity,therebyincreasingthecapacityofthelivertoconjugatebilirubin.

9 Whatarethehereditaryformsofconjugatedhyperbilirubinemiaandwhatis
themajorhistologicdifferencebetweenthem?
DubinJohnsonandRotor'ssyndromes.DubinJohnsonsyndromeischaracterizedbyblack,coarse
pigmentationofcentrilobularhepatocytes.Table76(t0035)outlinesthedifferencesbetweenthe
hereditaryformsofbilirubinmetabolismandtransport.

Manynewbornsdevelopjaundiceshortlyafterbirththevastmajorityofthesecasesare
physiologic.
Pathologiccausesofneonataljaundiceincludehemolysis,hematomaformation,
polycythemia,verticaltransmissionofhepatitisB,andhereditaryhyperbilirubinemias.
Ahighbilirubinconcentration(>20mg/dL)inanewborncanresultinkernicterus.For
unclearreasons,extremehyperbilirubinemiadoesnotcausekernicterusinadults.
Ahereditarydeficiencyofuridinediphosphate(UDP)glucuronyltransferaseactivity,as
occursinGilbert'sandCriglerNajjarsyndromes,mayresultinanunconjugated
hyperbilirubinemia.
DubinJohnsonsyndromeandRotor'ssyndromearethetwohereditaryformsof
conjugatedhyperbilirubinemia,resultingfromaninabilitytotransferconjugated
bilirubinfromthelivertothegastrointestinal(GI)system.
SummaryBox:PhysiologicJaundiceoftheNewborn

Case75
A60yearoldmalesmokerwhowasinfectedwithhepatitisBinhislate20spresentstoyour
clinic.Hehadbeenfeelingwelluntilrecentlyandhasavoidedseeingadoctorforthepast10
years.

Table76
HereditaryDisordersofHepaticBilirubinMetabolismandTransport

Feature

Gilbert's

CriglerNajjar

CriglerNajjar DubinJohnson

Rotor's

Syndrome

TypeI

TypeII

Syndrome

Syndrome

Syndrome

Syndrome

Incidence

612%

Veryrare

Uncommon

Uncommon

Rare

Gene

UGT1A1

UGT1A1

UGT1A1

MRP2

Unknown

Metabolic

Bilirubin

Nobilirubin

Bilirubin

Impairedcanalicular Impaired

defect

conjugation

conjugation

conjugation

exportofconjugated canalicular
bilirubin
exportof

affected

conjugated
bilirubin
Plasma

3in

Usually>20

Usually<20

Usually<7,about

Usually<7,

bilirubin

absenceof

(range,17

(range,6

onehalfconjugated

aboutonehalf

(mg/dL)

fastingor
hemolysis,

50),all
unconjugated

45),nearlyall
unconjugated

Normal

Normal

conjugated

nearlyall
unconjugated
Liver

Usually

histologic
normal,
appearance occasional

Coarsepigmentin

Normal

centrilobular
hepatocytes,leading

lipofuscin

toagrosslyblack
liver

Other

Bilirubin

features

concentration to

concentration concentrationwith

coproporphyrin

with
phenobarbital
phenobarbital

with
estrogens,
phenobarbital urinary

I/IIIratio,very
slowBSP

Prognosis

Noresponse

Bilirubin

Normal

Deathin

Usually

Jaundice

infancyif

normal

maybe

untreated

Bilirubin

Mildurinary

coproporphyrinI/III

elimination

ratio,slowBSP

kineticswithout

eliminationkinetics
withsecondaryrise

secondaryrise

Normal

Normal

evidentonly
withfasting
andstress
Treatment

None

Phototherapy

Phenobarbital Avoidestrogens

asabridgeto

forbilirubin

Noneavailable

liver

concentration

transplantation
BSP,sulfobromophthaleinMRP2,multidrugresistanceassociatedprotein2geneUGT1A1,bilirubinUDP
glucuronyltransferasegene.
FromFeldmanM,FriedmanLS,BrandtLJ:SleisengerandFordtran'sGastrointestinalandLiverDisease,8thed,
Philadelphia,WBSaunders,2006.

1 TowhatailmentsarepersonsinfectedwithhepatitisBsusceptible?
TheonesthatshouldcomeimmediatelytomindareliverdiseaseandHCC.Othersinclude
glomerulonephritis(fromantibodyantigen[AbAg]depositionintheglomeruluswhichelicitsan
inflammatorycascade)andpolyarteritisnodosa(PAN)(fromimmunecomplexdepositioninthe
bloodvessels).
Note:HepatitisDvirusisalsoparenterallyandsexuallytransmitted,andinfectioncanoccuronly
withconcominanthepatitisBinfection.

Case75continued:
Hedecidestoseehisgastroenterologistbecauseoverthelast2monthshehasnoticedadull
epigastricpainthatisnownearlyconstantandheisfeelingincreasinglyfatigued.Henotesan
unintentional30lbweightlossinrecentmonthsandayellowdiscolorationofhisskin.Heasks
thephysicianifmaybethatoldvirusisuptosomething.

2 Howdoestheprecedingadditionalinformationchangethedifferential
diagnosis?Whatspecificlaboratorytestsmightyouwanttoordertofurther
investigate?
Fatigueandunintentionalweightlossshouldalwaysmakeyouconsidermalignancyinyour
differentialdiagnosis,especiallyinolderpatients.InapatientwithahistoryofhepatitisBinfection
weneedtoconsiderHCC,whichcanpresentwithjaundice,rightupperquadrant(RUQ)pain,
ascites,andnausea.fetoprotein(AFP)isanonspecificserummarkerforHCC.

Case75continued:
Inadditiontotheusualliverenzymebloodtests,atestforAFPandultrasoundareordered.
TheAFPismarkedlyelevatedat1200ng/mL(normal<10ng/mL),andahyperechoicpattern
isappreciatedonultrasound(Fig.75(f0030)).

Figure75
LiverultrasoundofpatientinCase75.(st0295)
(FromGraingerRG,AllisonD:Grainger&Allison'sDiagnosticRadiology:ATextbookofMedical
Imaging,4thed.London,ChurchillLivingstone,2001.)

3 Whatisthelikelydiagnosisandhowcanitbeconfirmed?
HCCismostlikely.HCCisthemostcommonprimaryhepaticmalignancyitisthefifthmost
commoncancerinmenandtheeighthmostcommoninwomen.Histologicdiagnosisisdefinitive,
andsamplescanbeobtainedbyfineneedleaspiration(FNA)orpercutaneousbiopsy.

4 Whatothertypeofmalignancywillproduceamarkedlyelevated
fetoprotein?
Nonseminomatousgermcelltumors(thinkyolksactumor)andHCCaretheonlyprimary
malignanciesthatwillyieldavaluegreaterthan500ng/mLlivermetastasescanalsoyieldavalue
thishigh.AFPisalsousedinprenatalscreeningforDownsyndrome(decreasedlevels)andneural
tubedefects(increasedlevels).

5 Whataresomeriskfactorsforhepatocellularcarcinoma?
ThefourmajorriskfactorsthathavebeenidentifiedarechronichepatitisBinfection,chronic
hepatitisCinfection,cirrhosis,anddietaryexposuretoaflatoxinB1.AflatoxinB1isderivedfrom
AspergillusspeciesthatcancontaminatefoodstuffsintropicalandsubtropicalregionsofAfricaand
Asia.Thereareseveralminorriskfactors,includingcigarettesmoking,oralcontraceptivesteroids,
Wilson'sdisease,1antitrypsindeficiency,andhereditaryhemochromatosis.

6 Wherearelikelysitesformetastatichepatocellularcarcinoma?
Themostcommonsitesofmetastasesarethelung,regionallymphnodes,andadrenalglands.The
liveristhemostcommonsiteformetastasesofothermalignanciesbecauseofthehighdegreeof
bloodsupplytotheliverfromtheportalvenoussystem.Inadditiontomalignanciesinorganswhose
bloodsupplyfeedsintotheportalsystem,lungandbreastcancersoftenmetastasizetotheliver.

Youcangenerallydifferentiateprimarycancerfrommetastaticcancerwithimagingtechniquesby
thepresenceofsingleversusmultipletumorswithintheorganofinterest,respectively.

7 Whatparaneoplasticsyndromesareassociatedwithhepatocellular
carcinoma?
HCCisassociatedwiththeproductionofinsulinlikefactor,erythropoietin,andparathyroid
hormonerelatedpeptide(PTHrP).Clinicalfindingsmayincludepolycythemiaandconstitutive
hypoglycemia.Notethatthelattercontrastswithliverfailuresecondarytononcancerouscauses,
whichleadstofastinghypoglycemiaonly.

8 Inawomanwhotakesoralcontraceptivesandhasasinglehepaticnodule
detectedonultrasoundandanormalfetoprotein,whatkindofneoplasm
mightyoususpect?
Hepatocellularadenomasarebenignneoplasmsthatwereveryrarebeforeoralcontraceptives
becamewidelyused.Steroiduseisalsoassociatedwithhepatocellularadenomarisk.Hepatic
angiographycanbeusefulinmakingthediagnosisbecausemanyhepatocellularadenomasare
avascular.Surgicalresectionisrecommendedbecauseoftheriskofruptureand,inaverysmall
percentageofcases,transformationtoHCC.

9 Whatarethetreatmentoptionsforhepatocellularcarcinomaandwhatfactors
guidedecisionmaking?
Nooneoftheseoptionshasbeenprovedtobebetterthananyotherintermsofsurvival(Table77
(t0040)).
TheinformationinTable77(t0040)isbeyondwhatisexpectedforStep1,butmaybeusefulto
youinyourclinicalyears.
Step1Secret

Hepatocellularcarcinoma(HCC)isthemostcommonprimaryhepaticmalignancyitis
thefifthmostcommoncancerinmenandeighthmostcommoninwomenworldwide.
Fetoprotein(AFP)isaserummarkerforHCCaswellasnonseminomatousgermcell
tumors.
TheprognosisforHCCisgrim.Thosewhosecanceristooadvancedfortreatmentcan
expectamediansurvivalof3to6months.
HCCcommonlymetastasizestothelungs,regionallymphnodes,andadrenalglands.
HepatitisBhasbeenassociatedwithglomerulonephritis,polyarteritisnodosa,
fulminanthepaticfailure,andHCC.

InfectionwithhepatitisDoccursonlyinapatientalreadyinfectedwithhepatitisB
becauseitrequireshepatitisBviralparticlestoreplicateandinfectotherhepatocytes.
SummaryBox:HepatocellularCarcinoma

Case76
Yourfirstclinicpatientofthedayisa15yearoldboywhomovedtotheUnitedStatesfrom
Indonesiaattheageof5.Heishomeschooledbyhismother,whoreportsthathehasbeenless
activethannormalandhaslosthisappetite.HepointstotheRUQofhisabdomenandtellsyou
thathehasbeenhavingastomachacheinthatarea.Laboratoryworkuprevealsanelevated
ALTandAST.Otherlaboratorytestsareunrevealing.

Table77
TreatmentOptionsforHepatocellularCarcinoma

TreatmentModality

Indications/Comments

Arterialembolizationor

Multiplesmalltumorsorthosethatareinaccessible

chemoembolization

Embolizationalsocanbeusedtodecreasethesizeoflargertumors,to
allowforresection

Chemotherapy

Responseratesarelessthan20%,multipleagentsarealmostalways
required,andmultidrugresistanceisamajorproblem

Livertransplantation

Unresectabletumorsorhighlycirrhoticordysfunctionalliver
Recurrenceratesarehigh,andlifelongimmunosuppressionisrequired
aftertransplantation

Surgicalresection

Tumorconfinedtoonelobeoftheliverandaccessible,ideallywithout
significantcirrhosis
Only15%ofsymptomaticpatientsaresurgicalcandidates

1 Whatstructuresarelocatedintherightupperquadrant?
TheRUQcontainstheliver,thegallbladderandbiliarytree,thefirst,second,andthirdpartsofthe
duodenum,theheadofthepancreasandpancreaticduct,thehepaticflexureofthecolon,andthe
righthemidiaphragm.

2 Whatisthedifferentialdiagnosisconsideringtherelevantanatomy?
Hepatitis(viralbeingmostcommon),biliarycolic,cholelithiasis,cholecystitis,cholangitis,peptic
ulcer,pancreatitis,mesentericischemia,perforatedbowel,peritonealabscess,andmalignancy
shouldbeconsidered.

Note:Usinganatomiccuesisimportantforbuildingadifferentialdiagnosis,butabdominalpain
doesnotalwaysfollowanatomicdivision.PathologyinotheranatomiclocationscanpresentasRUQ
pain,suchasadissectingabdominalaorticaneurysm,arightlowerlobepneumonia,oranatypical
appendicitis,tonameafew.

Case76continued:
Ahepatitisprofilerevealsthefollowing:
AntiHAVIgM
AntiHAVIgG
+HBsAg
AntiHBsAg
+AntiHBcAg(IgG)
AntiHBcAg(IgM)
AntiHCV

3 Whatisthediagnosis?
ChronicactivehepatitisBvirus(HBV)infectionisindicated.Thehepatitisprofileindicatesachronic
hepatitisBinfectionreviewTable75(t0030).Thesymptomsoffatigue,poorappetite,andlowgrade
RUQpainaswellaselevatedaminotransferases(ALT>AST)suggestactiveinfection.Patientswith
chronicinactivehepatitisBareoftenasymptomaticwithnormalaminotransferases.
DiagnosinghepatitisBinfectionbasedonlaboratoryvaluesisimportantforStep1.Forthoseofyou
whostrugglewiththisconceptorsimplywishforadditionalclarification,youwillfindithelpfulto
studyFigure76(f0035)andthedescriptionintheSecretsbox(SecretsforScrutinizingHepatitisB
Infection).
TheeasiestwaytothinkabouthepatitisBistofirstgroupthethreetypesofantigensassociated
withthisinfection.Thesearesurfaceantigen(HBsAg),coreantigen(HBcAg),andanantigen
thatcirculatesinthebloodduringviralreplicationcalledhepatitisBeantigen(HBeAg).HBcAg
isnotclinicallyusefulbecauseitisnotdetectableinserum,sodon'tevenworryaboutthatone.
HBsAgisthemostimportantofthethree.Itisthefirstantigentoappearandlastantigento
disappear,andifyouhaveit,youareinfectedwithhepatitisBvirus(althoughyoucannottell
fromthisquiteyetwhetheritisanacute,chronic,orcarrierstateinfection).HBeAgisyour
infectivitymarker.Thisisespeciallyimportantinpregnantwomen.ThepresenceofHBeAg
indicatesahighinfectivityrate,period.Approximately90%ofneonateswillacquirehepatitisB
infectionfrommomifsheisHBeAgpositive.

Let'ssaysomeoneisacutelyinfectedwithhepatitisBvirus(HBV).Rightoffthebat,serum
HBsAgwillbecomepositive(coreantigenwillalsobepositive,butrememberthatwedonot
seethis).HBeAgbecomespositivesoonafter,andantiHBcAgisthenproduced.Thisantibody
ishugelyimportantfortworeasonsonewewillmentionnowandtheotherwewillmention
later.First,thinkofantiHBcAgasyourchronicitymarker(rememberCforchronicity).Ifa
patientispositiveforantiHBcAgoftheIgMisotype,itindicatesacute/recentinfection.Ifthe
isotypeisIgG,thepatienthaschronicdisease(>6monthsofinfection)orhasrecoveredfrom
disease.
Asmanyofyouprobablyknow,mostpeoplewithHBVdonotdevelopchronichepatitis
becausetheirimmunesystemwilleventuallyfixtheproblemthatitcausedinthefirstplace.
TheimmunesystemwillstarttomakeantibodiestoHBsAgandHBeAg.Inthiscase,the
antibodyisotypereallyisn'timportant.Whatisimportantisthatwhenthisstartstohappen,
thepatientwillenterthewindowperiod.Thebestwayforunderstandingthewindowperiodis
topicturethefollowingscenario:Let'ssayyouhaveconcentrationXofHBsAginyourserum.
Asthebodyproducesantibodiestothisantigen,theantigenprecipitatesoutofserumthrough
immunecomplexformation.WhentheconcentrationofantibodymatchesconcentrationXof
antigen,alloftheantigenprecipitatesout,andHBsAgbecomesundetectableinserum.This
alsoappliestoHBeAganditsantibody.
Tosummarize,thewindowperiodmarksthetimewhenthepatientproducessurfaceande
antibodiesinequalconcentrationstotheirantigenssuchthatneitherantibodynorantigenis
detectableinserum.
ThisiswhereantiHBcAgagainbecomesimportant.Theonlymarkerthatispositiveduringthe
windowperiodisantiHBcAg,anditcanthusbeusedtotestforinfectionduringthewindow
periodifyoususpectit.RememberthatsomeonewhohasbeenvaccinatedagainstHBVwill
bepositiveonlyforantiHBsAgandnotantiHBcAg.
Oncethewindowperiodhasended,levelsofantiHBsAgandantiHBeAgriseoverthelevelsof
antigen,andtheseantibodieswillbecomedetectableintheserum.Ifapatienthasdetectable
levelsofantiHBsAg,HEISCURED,nomatterwhat.Howdoyoudistinguishbetweenacured
patientversusanimmunizedpatient?Easy!LookforantiHBcAg(IgG).
Theonlyotherthingthatyouneedtoknowiswhatacarrierwilllooklike.Thesearepeople
whoarepositiveforsurfaceantigenbutdonothaveantibodiestosurfaceantigen,although
theyareotherwiseasymptomatic.Inthiscaseitishelpfultothinkofimmunocompromised
patients,becausethesearepeoplewhoseCD8+Tcellswillnotattacktheviralantigenson
hepatocytesandmountsymptomsofthedisease.Carrierscan,however,passthediseaseonto
othersbutwillnotbesymptomaticforhepatitisthemselves.
Wehopethismakesmoresensetoyounowandrecommendrereadingthissectionwhile
studyingFigure76(f0035)totestyourself.

SecretsforDiagnosingStagesofHepatitisBInfection

Figure76
SchematicdiagramofhepatitisBvirus.
(FromLongSS,PickeringLK,ProberCG:PrinciplesandPracticeofPediatricInfectiousDiseases,2nded.
Philadelphia,ChurchillLivingstone,2003.)

4 Whatisthemostlikelymodeoftransmissioninthiscase?
VerticaltransmissiontoneonatesfromHBVcarriermothersisthemostcommonmodeof
transmissioninendemicregionsoftheworld,suchasSoutheastAsia,China,andAfrica.Inthe
UnitedStates,sexualcontactisthemostcommonmodeoftransmission.Mostneonatalinfections
becomechronic,whereasonlyasmallpercentageofinfectionsacquiredinadulthooddo.HBVis
thoughttobe10timesmoreinfectiousthanhepatitisCvirus(HCV)and100timesmoreinfectious
thanHIV.

5 WhatisthesignificanceofhepatitisBeantigen?
HepatitisBeantigen(HBeAg)ispresentintheserumearlyinacuteinfection.Persistenceinthe
serumforlongerthan3monthsindicateshigherinfectivityandagreaterlikelihoodoftransitionto
chronicHBV.MotherswhoareHBeAgpositivehavethehighestriskofperinataltransmissionto
theirchildren.

6 IfapatientwithhepatitisBinfectionalsopresentedwitharthralgias,
mononeuritis,fever,abdominalpain,renaldisease,andhypertension,what
diseasemightyoususpect?
Polyarteritisnodosa(PAN)isoftenassociatedwithHBV.AlthoughonlyasmallpercentageofHBV
patientswilldevelopPAN,almostonethirdofpatientswithPANhaveacuteor,morecommonly,
chronicHBV.Membranousglomerulonephritisandmembranoproliferativeglomerulonephritisare
alsosometimesassociatedwithHBV.

Ifsymptomsofpolyarteritisnodosaaresuggestedinaclinicalvignetteonboards,thequestion
stemwillmostlikelymentionhepatitisBassociation.
Step1Secret

7 WhenisthehepatitisBvirusvaccinetypicallygiven?
UniversalvaccinationofallchildrenintheUnitedStatesisrecommended.Thefirstofthreedosesis
usuallygivenatbirth,thesecondat1to2months,andthethirdat6to18months.Childrenbornto
HBsAg+mothersshouldalsobegivenhepatitisBimmunoglobulinwithin12hoursofbirthto
achievepassiveimmunity.Membersofotherhighriskgroups,includinghealthcareworkers
(YOU!!),shouldalsobevaccinatediftheypreviouslywerenot.

8 WhatarethetwomostcommontreatmentoptionsforhepatitisBvirus?
Theprimarygoaloftreatmentislongtermsuppressionofthevirus.INFandnucleosideanalogs
arebothusedtoachievethis.Interferonislessexpensiveandonlyneedstobetakenforalimited
time(412months),butithasmanysideeffects.Nucleosideanalogshavefewersideeffectsbutneed
tobetakenlongterm.Insomecases,combinationtreatmentisoptimal.

9 Whataresomeotherinfectiouscausesofhepatitis?
LesscommoncausesofhepatitisaregiveninTable78(t0045).

HepatitisBisahepadnavirusthatcanbetransmittedbybodilyfluidsorperinatally.
HepatitisBandCvirusesarecapableofproducingchronicinfection.
HepatitisDinfectioncanoccuronlyalongwithhepatitisBinfection.
PresenceofhepatitisBeantigen(HBeAg)indicateshigherviraltitersandhigher
infectivity.
Thegoaloftreatmentislongtermsuppressionofthevirusinterferonandnucleoside
analogsarecurrentlythemainapprovedtherapies.
SummaryBox:HepatitisBVirus

Case77
A19yearoldcollegestudentpresentswithnausea,vomiting,andabdominalpain.Initially,
sheisslightlyconfusedandwithdrawn,makingitdifficulttocollectagoodhistory,butshe
doestellyouthatshewasatafraternityparty2nightsagoandgotprettydrunk.Shedenies
usinganyotherdrugsattheparty.Youdoapelvicexaminationandarectalexaminationto

assessforoccultbloodandorderstatlaboratorytestsincludingaCBC,basicmetabolicpanel,
liverenzymes,urinalysis,urinepregnancytest,andlipase.Youalsopreparetodoanabdominal
ultrasound.

Table78
LessCommonCausesofHepatitis

Pathogenic

PotentialEtiologicDisorder(s)

Category
Amebic

Entamoebahistolyticaabscess

Bacterial

Pyogenichepaticabscessmaybecausedbygrampositiveaerobiccocciin
neonatesandgramnegativerodsinadults

Parasitic

Leptospirosis,schistosomiasis,liverflukes(trematodes),toxoplasmosis

Viral

Cytomegalovirus(CMV),EpsteinBarrvirus(EBV),herpessimplexvirus(HSV),
varicellazostervirus(VZV)infections

1 Whatarethesomeofthecommoncausesofacuteabdominalpainwithnausea
andvomiting?
SeeTable79(t0050).

Case77continued:
Pelvicandrectalexaminationsareunrevealingwithstoolnegativeforblood.Laboratorytests
revealamildanemiaandthrombocytopeniaaswellassignificantlyelevatedtransaminases.
Theurinepregnancytestisnegative,andotherlaboratorytestsarenormal.Abdominal
ultrasoundisunremarkable.Youpresentthesefindingstothepatient,tellingherthatit
appearsthatherliverseemstohavebeendamaged.Somewherealongtheway,yougarnered
hertrustandshenowtellsyoumoreabouttheparty.Shesawherboyfriendkissingoneofher
sororitysisters,andafterchuggingthreemorebeers,shewentbacktoherdormandtooka
bunchofTylenolbeforepassingouttoavoidahangoverinthemorning.Afterlearningthis,you
orderlaboratorytestsfortotalbilirubinandPT,whichareelevatedandprolonged,
respectively.

Table79
CommonCausesofAcuteAbdominalPain

Causative
Condition

NatureofPain/AssociatedFindings

Acute
appendicitis

PainmaybelocatedintheperiumbilicalareaorintheRLQanorexiaiscommon.

Acute

PainislocatedintheRUQ,andultrasoundimagingmayshowgallstones.

cholecystitis
Acute

Diarrheaoftenisaprominentcomponent,anditscharacteristics,alongwith

gastroenteritis characterizationofitsonsetwithregardtomeals,canhelptodeterminethe
underlyingdisorder.
Acute

Epigastricpainradiatestotheback,associatedwithanorexia,nausea,and

pancreatitis

vomitingplasmaamylaseandlipase(amorespecificmarker)levelsmaybe
elevated(althoughoftennotinchronicpancreatitis).

Acute

Bilateraladnexalpainiscommon,withcervicalmotiontendernessonbimanual

salpingitis

examination.

Biliarycolic

RUQpainisintermittentultrasoundimagingmayshowgallstones.

Ectopic

Nauseaandvomitingoftenareabsent,andaurinepregnancytestispositive

pregnancy

pelvicultrasoundimagingisusedtoruleoutanintrauterinepregnancyandwill
sometimesrevealanadnexalmassorblood.

Intestinal

Painoftenisdiffuseandcrampyinnature.

obstruction
Perforated

Painusuallyisepigastricdark,tarrybloodmaybefoundinthestool.

duodenal
ulcer
Renalcolic

Flankandcostovertebralanglepainareseverehematuriaiscommon.

RLQ,rightlowerquadrantRUQ,rightupperquadrant.

2 Whatisthediagnosisandsuspectedetiology?
Shehasfulminanthepaticfailure(FHF)resultingfromacetaminophentoxicity.FHFisdefinedasthe
rapiddevelopmentofhepatocellulardysfunctionandmentalstatuschangesinapatientwithout
previouslyknownliverdisease.
Note:FHFoftenmanifestsasacoagulopathyorencephalopathy.Coagulopathyoccursbecausethe
liverisnotabletoadequatelyproduceclottingfactors,andtherecanbeplateletdestruction.Cerebral
edemamayleadtoencephalopathyofvaryingseverity.Indeed,thedurationoftimebefore
encephalopathybeginsissometimesusedtocharacterizetheseverityofFHF.Hypoglycemia,
infections,andrenalfailureareothercomplicationsthatcanarisefromFHF.

3 Whatisthemechanismofhepaticdamageinacetaminophentoxicity?

AcetaminophenisoxidizedbythecytochromeP450systemintoNacetylpbenzoquinoneimine
(NAPQI).NAPQIistoxictolivercells,butnormallyitisdetoxifiedinaphaseIIreactionby
glutathione.Ifatoxicdoseofacetaminophenisingested,theglutathionesupplyisdepleted,leaving
NAPQItocauseliverdamage.

4 Whatistheantidoteforacetaminophentoxicity?
Inadditiontosupportivetreatment,acetaminophentoxicityshouldbetreatedwithNacetylcysteine
(Mucomyst)thesoonerthebetter.Treatmentwithin8hoursofingestionisnearly100%
hepatoprotective.AcetylcysteinesubstitutesforglutathioneanddetoxifiesNAPQI.Activatedcharcoal
shouldbegivenifthepatientpresentsshortlyafteringestion.Thecharcoalabsorbstoxinssuchas
acetominopheninthestomach.Theeffectivenessofactivatedcharcoaldropssharplyifmorethan1
hourhaspassedfromtimeoftoxiningestion.

5 Whatisthemaximumdailydosageofacetaminophenforadults?
Themaximumdoseforadultsis4g/day.Themaximumdoseforchildrenisvariablebyage.
Acetaminophenisaningredientinmanymedications,suchasVicodin,Percocet,andcoldandflu
formulations.Patientsoftendonotrealizethisandcanaccidentallyoverdosewhenusingmultiple
medications.
YoudoNOTneedtoknowbrandnamesormedicationdosagesfortheUSMLE.
Step1Secret

6 Whyshouldalcoholicsavoidacetaminophen?
Alcoholconsumptionincreasestheactivityoftheenzymethatmetabolizesacetominopheninto
NAPQI,which,asyouknow,ishepatotoxic.Chronicalcoholusecanalsodepleteglutathionestores
intheliverthusreducingitsprotectionagainstdamagecausedbyreactiveoxygenspecies.

7 Whatotherpotentiallyhepatotoxicdrugsshouldyouknowfortheboards?
Therearefartoomanypotentiallyhepatotoxicdrugstomentionhere,butsomeofthemore
commonlyusedonesincludethefollowing:amiodarone,amoxicillin,chlorpromazine,ciprofloxacin,
erythromycin,fluconazole,isoniazid,methotrexate,methyldopa,statins,niacin,rifampin,salicylates,
andvalproicacid,aswellasseveralantiretroviralsandanticancerdrugs.Itisalsoworthreminding
youthatmanydrugsundergohepaticmetabolismandtheirdosagesshouldbeadjustedinpatients
withliverdisease.

Fulminanthepaticfailure(FHF)istherapidonsetofhepatocellulardysfunctionand
mentalstatuschangesinapatientwithoutpreviousliverdisease.
EncephalopathyandcoagulopathyarecommonmanifestationsofFHFother
complicationsincludehypoglycemia,infection,andrenalfailure.

Hyperbilirubinemia,elevatedtransaminases,andprolongedprothrombintimecanbe
expectedinFHF.
Manydrugsarehepatotoxic,butacetaminophentoxicityisthemostcommoncauseof
FHFintheUnitedStates.
Nacetylpbenzoquinoneimine(NAPQI),ametaboliteofacetaminophen,is
responsibleforhepaticdamageinacetaminophentoxicity.
Activatedcharcoalandacetylcysteine(Mucomyst)areusedtotreatacetaminophen
toxicity.
Themaximumdailydoseofacetaminophenis4gforadults.Themaximumdosein
patientswithliverdiseaseislower,typically2g/day.
SummaryBox:AcetaminophenInducedFulminantLiverFailure

Case78
A41yearoldobesemotheroffourcomplainsofnausea,vomiting,fever,andrightsidedupper
abdominalpainaftereatingfattymeals.Onexamination,sheisnotjaundiced,hasa
temperatureof100.5F,andexperiencessharppainoninspirationwhenpressureisprovided
totheloweredgeofherrightcostalcartilage(Murphy'ssign).Laboratorytestsshowa
leukocytosiswithaleftshift.

1 Whatdiagnosisdoyoususpect?
Cholecystitis(inflammationofthegallbladder),whichisusuallyduetoobstructionofthegallbladder
neckorcysticductbyagallstone,islikely.However,otherdiagnosessuchasascendingcholangitis
shouldbeconsidered.

2 Whatriskfactorsforgallstonesdoesthepatientexhibit?
ThisclassicpresentationincludestheriskfactorsthatcanberememberedasthefourF's:female,f
at(althoughnotpoliticallycorrect,thisisausefulmnemonic),fertile,forties(age).

3 Whydopatientswithgallstonesexperiencepain,particularlyaftereatinga
highfatmeal?
Entryoffattyacidsintotheduodenumstimulatesthereleaseofcholecystokinin(CCK),whichcauses
gallbladdercontraction.Thiscreatespainbyincreasingbiliarypressure.

Case78continued:

UltrasoundoftheRUQrevealsadistendedgallbladdercontaininggallstonesanddemonstrates
asonographicMurphy'ssign.Sheisadmittedtothehospitalandplacedonantibiotics,anda
surgeryconsultisobtained.

4 WhatisasonographicMurphy'ssign?
Tendernesswithpressurefromtheultrasoundprobedirectlyoverwherethegallbladderis
visualized.Thisresponsecanbenegativeingreaterthan50%ofcasesofacutecholecystitis.In
contrast,Murphy'ssignonphysicalexaminationreferstoamaneuverinwhichthephysicianplaces
thehandsbelowthecostalmarginattherightmidclavicularline(immediatelybelowthelevelofthe
gallbladder)afterinstructingthepatienttoexhale.Thepatientisaskedtobreathein,andthe
diaphragmandabdominalcontentsareshifteddownwardasthelungsexpand.Thegallbladdernow
makescontactwiththeexaminer'shands,whichcausesthepatienttowinceinpainifgallbladder
disease(e.g.,inflammation,gallstones)ispresent.ThisisconsideredaMurphy'ssign.Murphy'ssign
isnotgenerallypositivewithcholangitis.
Note:Ultrasoundistheimagingofchoicetoevaluateforgallstonesandcholecystitis.Itisquickand
noninvasiveandcanbedoneatthebedside.

Case78continued:
Alaparoscopiccholecystectomyisscheduledandcarriedoutwithoutcomplicationduringthe
samehospitalization.

5 Whatarethemostcommontypesofgallstones?
Cholesterolmonohydrate(80%)orcalciumbilirubinate(20%)aremostcommon.

6 Whydoesanobstructingstoneinthecommonbileductpredisposeto
jaundice,whereasastoneinthecysticductgenerallydoesnot?
Astoneinthecommonbileduct(choledocholithiasis)cancompletelypreventtheflowofbiletothe
intestines(cholestasis),causingbiliarybackpressurethatdamagestheliverandresultsin
hyperbilirubinemiaandjaundice.However,astoneinthecysticductwillonlypreventbilefrom
flowingintooroutofthegallbladder,leavingbileflowfromthelivertotheintestinesunimpeded.

7 Whatischolangitis?
Cholangitisisaninfectionofthebiliarytree,usuallyoccurringasaresultofastoneinthecommon
bileduct.Itrequiresaggressivetreatment.

8 WhatisCharcot'striadforcholangitis?

Charcot'striadconsistsof(1)fever,(2)RUQpain,and(3)jaundiceandispresentinapproximately
50%ofpatientswithcholangitis.Thefeverisduetotheresponsetoinfection,thejaundiceisdueto
obstructionofthecommonbileduct(orotherbileducts),andthecauseofRUQpainisobvious.
RUQpainsecondarytoascendingcholangitiscanradiatetotheshoulderortipofthescapula.

9 Whereinthepancreaswouldaneoplasmcausingobstructivejaundicemost
likelybelocatedandwhy?
Itwouldbeintheheadofthepancreas.Thecommonbileductrunsthroughtheheadofthepancreas
onitswaytothesecondpartoftheduodenumandcangetobstructedalongtheway.TheUSMLE
lovesthisanatomicrelationshipbetweenthepancreasandthecommonbileduct.Notethatthe
classicpresentationisoneofpainlessjaundiceinapatientwithmalaiseandunintentionalweight
loss.

10 Howcancholestasiscausepalestools?
Conjugatedbilirubinisnormallymetabolizedtourobilinogen(clearcolor)bycolonicbacteriaand
ultimatelytostercobilin(browncolor)viaautooxidation,whichcausesthenormalstoolcolor.
Neitheroftheseprocessesoccursifbiledoesnotreachtheintestines.

11 Definesteatorrheaandexplainwhyitcandevelopfromcompleteobstruction
ofthecommonbileduct
Steatorrhea,typicallycharacterizedbyfoulsmellingstools,referstothepresenceofsignificant
amountsoffatinstool.Bileacidsemulsifyfatssothattheycanbedigestedbypancreaticlipases,
thenformmicellesofthedigestedfattyacidsanddeliverthemtotheintestinalmucosafor
absorption.Consequently,impaireddeliveryofbiletotheintestinesinterfereswithallthese
processesandcausessteatorrhea.

12 Whatpreventstheformationofcholesterolstonesinthenormalphysiologic
setting?
Bilesaltsandphospholipidssolubilizecholesterolandpreventitfromprecipitatingoutofsolution.
Infact,forpatientswithsmallstoneswhoarepoorsurgicalcandidates,oralbileacidsaregivento
facilitatedissolutionofthestone.Decreasedbilesaltandphospholipidconcentrationsorincreased
cholesterolconcentrationscanallleadtostoneformation.

13 WhyarepeoplewithCrohn'sdiseasepredisposedtothedevelopmentof
cholesterolstones?
Crohn'sdiseaseofteninvolvestheterminalileum,wherebilesaltsarereabsorbed.Becausethese
saltsareimportantinthesolubilizationofcholesterol,reducedreabsorptionfacilitatesstone
formation.

14 Whichcholesterolloweringdrugsbindbileacidsintheintestine?Explain
howthesedrugslowerserumcholesterol

Cholestyramineandcolestipol,whicharenonabsorbableionicresins,bindbileacidsintheintestine
andareeliminatedinthefeces,promotingtheexcretionofbilesalts.Asaresult,morebileacidsneed
tobeproduceddenovo.Becauseserumcholesterolisusedasasubstrateforbileacids,bileacid
synthesisresultsinreducedplasmacholesterol.Recallthattheformationofbilesaltsistheonly
methodavailabletothebodytoeliminatecholesterol.

15 HowcaninfectionwithClonorchissinensisalsoleadtoobstructivejaundice?
Thistrematodeinfectsthehepatobiliarytree.Chronicinflammationfromthisinfectioncancause
fibroticstrictureswithinthebileductsthatimpedetheegressofbile.

Cholelithiasisisthepresenceofstonesinthegallbladderorcysticduct.Cholecystitisis
inflammationofthegallbladder.Choledocholithiasisisthepresenceofstonesinthe
commonbileduct.Cholangitisisinflammationofthebiliarytree.
Mostgallstonesarecholesterolstones.Calciumbilirubinatestonesarethenextmost
commontype.
Ultrasoundistheinitialimagingofchoiceforsuspectedbiliarydisease.
Releaseofcholecystokinin(CCK)resultsingallbladdercontraction,whichoccurs
followingfattymealsandtherebyexacerbatesRUQpain.
Charcot'striadisfever,rightupperquadrantpain,andjaundiceitisassociatedwith
acutecholangitisbutispresentinonly~50%ofpatients.
SummaryBox:BiliaryDisease

Case79
Amotherbringsher4yearoldchildintoyourofficeandcomplainsthathehasbeenlethargic,
sleepy,irritable,andquietforthepastfewdays.Hehasalsodisplayedheavyvomitingthathas
notbeenrelievedbymeals.Youaskifthechildhasbeenfeverishatall.Notanymore,his
motheranswers.Hejustgotoverthechickenpoxafewdaysagoandhadseveralhighfevers
duringthattime,butwegavehimaspirinaroundtheclockandtheyeventuallyresolved.

1 Whatisthemostlikelydiagnosis?
ThisisaclassicpresentationofReyesyndrome,whichisararebutseriouschildhood
hepatoencephalopathy.Itisassociatedwithsalicylateadministrationinchildren,especiallyfollowing
viralinfection.Forthisreason,aspirinuseisalmostneverrecommendedforchildren.However,a
notableexceptiontothisguidelineistodecreaseriskofcoronaryarteryaneurysminchildrenwith
Kawasakidisease.

2 WhatisthepathophysiologyofReyesyndrome?
Aspirinmetabolitescanreversiblyinhibitamitochondrialenzymeinvolvedinoxidationoffatty
acids,leadingtobuildupoffattyacidsandmicrovesicularfattychangeintheliver.Thismayinduce
hepaticdamageanddisruptotherprocessesthatoccurintheliversuchasgluconeogenesis,
glycogenolysis,andtheureacycle.Clinicalmanifestationsofthesedisruptionsincludehypoglycemia
andencephalopathyorcomawithincreasedammonialevelsintheblood.

3 ListthecomponentsofthepostinfectioustriadassociatedwithReyesyndrome
Encephalopathy,fattyliverdegeneration,andtransaminaseelevationoccur.

Reyesyndromeoccursinchildrensecondarytosalicylateadministration.Ingeneral,
childrenshouldnotbegivenaspirinexcepttopreventcoronaryarteryaneurysmwith
Kawasakidisease.
Reyesyndromeleadstomicrovesicularfattychangeintheliver.Damageisreversible.
ClinicalsymptomsofReyesyndromeincludelethargy,irritability,somnolence,heavy
vomiting,andcoma.
SummaryBox:ReyeSyndrome

Copyright2015Elsevier,Inc.Allrightsreserved.

BOOKCHAPTER

Immunology
ThomasA.BrownMDandSonaliJ.Shah
USMLEStep1Secrets,Chapter15,439485

Immunologycanbeanintimidatingsubjectformanymedicalstudents.Fortunately,
immunologyrelatedquestionsontheUSMLEfocusprimarilyonbasicconceptsthatcanbe
masteredbyachievingathoroughunderstandingofthetopicslistedinFirstAidandworking
throughthequestionsandcasesinthischapter.Paycloseattentiontothetablesinthischapter.
ThesetopicsareparticularlyhighyieldfortheUSMLEStep1examination.
Insider'sGuidetoImmunologyfortheUsmleStep1

Basicconcepts
1 Outlinehematopoiesis,beginningwithapluripotentstemcell
SeeFigure151(f0010).

Figure151
Stemcellbasedmodelofhematopoiesis.
(FromNobleJ:TextbookofPrimaryCareMedicine,3rded.St.Louis,Mosby,2001.)

2 Whatarethemajorprimaryandsecondaryorgansthatmakeupthehuman
lymphoidsystem?
Primarylymphoidorgansincludebonemarrow,thymus,andfetalliverandarethesitesatwhich
lymphocytesmature.PrecursorBandTlymphocytesareproducedinthebonemarrow.Inadults,B
cellscontinuetomatureinthebonemarrow,butTcellsleavethemarrowtomatureinthethymus.
Maturationintheseorgansoccursintheabsenceofstimulatingantigenandinvolvesastringent
selectionprocessthateliminatesbothpoorlyfunctioningandautoreactivelymphocytes.Secondary
lymphoidorgansincludelymphnodes,spleen,andmucosaassociatedlymphoidtissue(MALT).
MALTrangesfrompoorlyorganizedclustersoflymphoidcellstohighlyorganizedstructuressuchas
appendix,tonsils,andPeyer'spatches.Thesesecondarylymphoidorgansprovideasiteformature
lymphocytestorespondtoantigenpresentedonthesurfaceofdendriticcellsandotherprofessional
antigenpresentingcells(APCs).

3 Whatisthefunctionofinnateimmunesystem?
Theinnateimmunesystemformsthefirstlineofdefenseagainstpathogensandtoxiccompounds.
Cellsoftheinnateimmunesystemareactivatednonspecificallyandcontainvariouspattern
recognitionreceptors(PRRs)thatrecognizeadiversegroupofpathogenassociatedmolecular
patterns(PAMPs)sharedbydifferentclassesofmicrobes(e.g.,LPSandflagellin).Theintensityof
theinnateimmuneresponsedoesnotincreasewithrepetitiveexposuretoidenticalantigens.
Thefirstcomponentofinnateimmunityincludesfactorsthatpreventmicrobesfromenteringthe
body.Examplesofthisareintactepithelium,respiratorycilia,alveolarmacrophages,lysozyme,
cationicpeptides(defensins),RNase(ribonuclease),andnormalflora.Thesecondcomponent
includesfactorsthatdestroyorlimitthegrowthofmicrobesthathavealreadyenteredthebody.
Examplesofthisarephagocytes,naturalkiller(NK)cells,complement,interferon(IFN),iron
sequesteringproteins(e.g.,lactoferrin),feverandtheinflammatoryresponse(i.e.,releaseof
interleukin1[IL1],IL6,andtumornecrosisfactor[TNF]).Neutrophilsformthemostabundant
typeofphagocyteandarefirsttoreachthesiteofinfection.Here,theyengulfanddestroymicrobes.
Pusisaviscousexudatethatiscomposedofdeadneutrophilsattheinfectionsite.

4 Whatisadaptive/acquiredimmunity?
AdaptiveimmunityinvolvestheresponseofBandTcellsfollowingexposuretoanantigentowhich
theyarespecific.Itischaracterizedbymemory,resultinginrapid,stronger,andmoreefficient
eliminationofthesourceoftheoffendingantigenuponrepeatexposuretosaidantigen(Table151
(t0010)).Itisveryimportanttorecognizethatstimulationoftheadaptiveimmuneresponsedepends
onprioractivationoftheinnateimmunesystem.Dendriticcellsareconsideredtoformthebridge
betweentheinnateandadaptiveimmunesystems,becausetheypresentantigentonaiveTcells.
StimulationofdendriticcellPRRsresultsinantigenprocessingandpresentationtonaiveTcellsvia
majorhistocompatibilitycomplex(MHC)Tcellreceptor(TCR)interactions.
Table151
CharacteristicsofInnateandAcquiredImmunity

Effectorcells

Chemical
mediators

Innate(Natural)

Acquired(Adaptive)

Neutrophils(polymorphonuclear
neutrophils),macrophages,
eosinophils,basophils,mastcells,
naturalkiller(NK)cells

Bcellsandplasmacells,Thelpercells
(e.g.,TH1,TH2cells),cytotoxicT

Complement,lysosomalenzymes,
cytokines,interferons,acutephase
proteins

Antibodies(immunoglobulins),cytokines,
granzyme,perforin

Response
Rapid,nonspecificsameintensity
characteristics againstallantigens,nomemory

lymphocytes(CTLs)

Slow,antigenspecificlongtermmemory
andenhancedresponsegeneratedafter
firstexposure(e.g.,morerapidand
intense)

Note:Theprimaryandsecondaryimmuneresponsesrefertotheactivityoftheadaptiveimmune
system.Theprimaryresponseistheactivityofthissystemafterfirstexposuretothepathogen,
whereasthesecondaryresponseoccursafterimmunologicmemoryhasbeengeneratedfroma
previousexposure,sothesecondaryresponseismorerapidandpowerful.

5 Whatarethebasiccharacteristicsofcellmediatedandhumoralimmunity?
CellmediatedimmunityinvolveshelperT(TH)cellsandischaracterizedbyTHcellmediated
defenseagainstviruses,fungi,andmycobacteria.ItisalsoresponsiblefortypeIVhypersensitivity
reactions,tumordestruction,andgraftrejection.
NonspecificactivationofdendriticcellsandmacrophagesleadstosecretionofIL12,which
stimulatenaiveThelper(TH0)cellstodifferentiateintoTH1cells.ThisTcellsubsetsecretesIL2,
leadingtopropagationoftheTHcellresponseandactivation/conversionofTCcellstocytotoxicT
lymphocytes(CTLs),whichfunctiontodestroytumorsandvirusinfectedcells.NotethatIL2
secretionfromTH1cellsconstitutesonlypartofthesignalsequencenecessaryforCTLactivation
CTLsalsorequireTCRandCDBinteractionwithMHCclassIoninfectedcells(seequestion6,next).
TH1cellsalsosecreteinterferon(IFN),whichactivatesmacrophagesandstimulatesintracellular
microorganismdestruction.
HumoralimmunityisprimarilymediatedbyBcellproductionofantibodies.Antibodieshavea
diversesetofroles,includingneutralizationofpathogensandtoxins,opsonizationofpathogensto
facilitatephagocytosis,complementactivation,stimulationofmastcellandbasophildegranulation,
andBcellactivation.Antibodiesaregroupedintofivedifferentisotypes(IgM,IgD,IgA,IgE,IgG)
accordingtotheirconstantdomains.ProductionofIgA,IgE,andIgGreliesonBcellinteractionwith
Tcellsinaprocessknownasclassswitching(seequestion9).

6 DescribethedifferencebetweenclassIandclassIImajorhistocompatibility
complexmolecules
TcellspossessTcellreceptors(TCRs)thatinteractwithMHCmolecules.MHCclassImoleculesare
foundonthesurfaceofallnucleatedcells,thusexcludingonlymatureerythrocytes,andinteractwith
theTCRofCD8+Tcells(i.e.,cytotoxicTlymphocytes).TheCD8moleculeisnecessarytocomplete
theinteractionoftheTCRandMHCclassI.MHCclassIIliesontheplasmamembraneofantigen
presentingcells(dendriticcells,macrophages,andmemoryBcells).ItinteractswiththeTCRofCD4
+Tcells(i.e.,T cells),withtheCD4moleculeservinganecessaryroleforthisinteractiontooccur.
H

AneasywaytorememberwhichMHCmoleculeinteractswithwhichTcelltypeistousetheruleof
8:
Class1 CD8 = 8

Class2 CD4 = 8

Forthepurposeofboards,MHCclassIdisplaysnonselfpeptidesthathavebeenprocessed
intracellularly(asmightbeseeninacellinfectedwithavirus),andMHCclassIIdisplaysexogenous
nonselfpeptides(obtainedviaphagocytosisorendocytosis).Thisisanimportantdistinctionto
make.MHCclassI,whendisplayingnonselfpeptides,marksthecellfordestructionbytheCTLwith
theTCRspecificforthatpeptide.ThisdestructionismediatedbyFasFasligandbinding(leadingto
apoptosis),granzymes(proteases),orperforins(createsachannelintheplasmamembraneand
mediatescytolysis),thuseliminatingtheinfectedcell.Ontheotherhand,whenMHCclassII
displaysnonselfpeptides,itleadstoactivationoftheTHcellwiththeTCRspecificforthatpeptide.
ThisactivationprocessupregulatesTHcytokineproduction,resultinginsubsequentmacrophage
activationandproliferationofplasmacellswiththeappropriatespecificity,thuscatalyzingthe
eliminationoftheoffendingextracellularagent.

7 Howdoantibodieseliminateextracellularpathogens?
Extracellularpathogens(e.g.,bacteria,freevirions)commonlyinduceproductionofhumoral
antibodies.Extracellularpathogensthatarecoatedwithopsonizingantibodies(IgG)areefficiently
phagocytizedbymacrophagesandneutrophils.Followingphagocytosisandprocessingof
extracellularantigens,expressionofantigenicpeptidesinassociationwithMHCclassIIonthe
surfaceofantigenpresentingcells(APCs)stimulatesTH2cells,whichfurtherenhancesthehumoral
response.KeepinmindthatIgGandIgMcanactivatecomplementthatmaylyse,neutralize,or
opsonizeextracellularpathogens.

8 Bywhatprocesscanantibodiescatalyzetheeliminationofintracellular
pathogens?

Antibodydependentcellularcytotoxicity(ADCC)involvesthebindingofIgG(attachedtoantigens
onthesurfaceofthetargetcell)toFcreceptors.Thisleadstodestructionofthecellbyphagocytic
cellsorNKcellspossessingthesereceptors.

9 Whatarethefiveclasses(isotypes)ofimmunoglobulins?Describetheir
respectivedistributionsinthebody
SeeTable152(t0020).
Table152
ImmunoglobulinIsotypes

Immunoglobulin Location
Isotype

Functions

IgA

DimericIgA,joinedbyaJchain,
isfoundinsecretions
MonomericIgAisfoundinthe
blood

Foundinmucosalsecretions(e.g.,tears,
saliva,colostrum,gastrointestinalsecretions)
mediatesmucosalimmunity
Pooractivatorofcomplement

IgD

SurfaceofmatureBcellssome
foundintheblood

Unclear

IgE

BoundtoFcR1receptorsonthe
surfaceoftissuemastcellsand
bloodbasophils

Mediatestype1hypersensitivity
MediatesparasitickillingviaADCC
(eosinophilspossessingFcreceptorsare
theeffectorcellsandinducedamagewith
majorbasicprotein)

IgG

Foundinthebloodcrosses
placenta
Hasthelongesthalflifeofall
isotopesandisthususedfor
passiveimmunization

LigandforFcreceptors
Activatesclassicalcomplementpathway
Mostabundantimmunoglobulininsecondary
immuneresponse
Highestaffinityforantigen(greateststrength
ofinteractionbetweenantigenandantibody)

IgM

Foundintheblood,oftenin
pentamers(joinedbyJchains),
onthesurfaceofimmatureand
matureBcells

Activatesclassicalcomplementpathway
Earliestantibodyproducedinanyhumoral
response
Mostabundantimmunoglobulininprimary
immuneresponse
Highestavidityforantigen(greatestnumber
ofantigenbindingsitesasaresultof
pentamericform)

ADCC,antibodydependentcellularcytotoxicity.

Note:WhenBcellsdirectlyinteractwithCD4+helperTcells,allclassesofimmunoglobulincan
potentiallybeproduced.TheTCRinteractswithMHCIIontheBcell,andCD40Lexpressedonthe
activatedTcellsimultaneouslybindstotheconstitutivelyexpressedCD40ontheBcell.This
interactionfacilitatestheproductionofvariouscytokinesfromtheTcellthatinfluenceBcellclass
switchingviaVDJrecombinationandproductionofIgG,IgE,andIgA.Bcellscapableofproducing
theseantibodiesarereferredtoasplasmacells.
Incontrast,theTcellindependentresponseconsistsalmostexclusivelyofIgM.TheTcell
independentresponsegenerallyoccurswhentheprimaryantigenisapolysaccharide(e.g.,bacterial
capsule,lipopolysaccharideonthegramnegativeoutermembrane),becausethesemoleculesarenot
processedbyAPCsinthesamemannerasforpeptideantigen.Therefore,THcellsarenotactivated
andisotypeswitchingcannotoccur.ThismeansthatonlyIgMwillbeproducedandnosecondary
(memory)immuneresponsewilloccur.Vaccinescontainingpolysaccharidecapsules(e.g.,
meningococcalvaccine,pneumovax,HaemophilusinfluenzaetypeBvaccine)arethuscommonly
conjugatedtoproteinsinordertopromoteTHcellactivation,classswitchingandtheformationof
immunologicmemory.

10 Mosthumanscanproduce106to109uniqueimmunoglobulin(Ig)molecules.
However,thenumberofimmunoglobulingenesisordersofmagnitudeless
thanthis.Howisthispossible?
Thisisprimarilyduetogenerearrangement.NotethateachIgmoleculeconsistsoftwoidentical
lightchains(classifiedasorbytheconstantregion)andtwoidenticalheavychains(classifiedas
,,,,andaccordingtotheisotype,asdeterminedbytheconstantregion).Diversityisgained
frommixingandmatchingthesechains.However,eachofthesefourchainsalsocontainsa
variableregion,whichiscreatedbygenerearrangement.Inthisprocess,variousuniquegene
segmentsingroupsnamedV,D(heavychainsonly),andJareexcisedfromthestrand,reorderedto
createauniquecodeforthevariableregion,andreconnectedbyRAG1andRAG2(recombination
activatinggenes)toeachotherandtotheconstantregiongenesegments.Thistypeofrearrangement
isalsoseenintheandchainsoftheTCRtoachievediversityinthesemoleculesaswell.
ForthepurposeoftheUSMLE,itisnotimportanttomemorizethedetailedmechanismby
whichV(D)Jrecombinationoccurs.Rather,focusonunderstandingthesignificanceof
recombinationthatis,thediversityofantigenrecognitionsitesresultsfromthe
recombinationprocess.Itis,however,particularlyhighyieldtounderstanddetailsregarding
generalantibodystructure(Fig.152)(f0015).

Figure152
Thestructureofimmunoglobulin(Ig)molecules.Note:The12complementdeterminingregions
(CDRs)aretheareasthatmostdeterminetowhichantigentheantibodywillbind.
(FromMasonRJ,MurrayJF,BroaddusVC,etal:Murray&Nadel'sTextbookofRespiratoryMedicine,
4thed.Philadelphia,WBSaunders,2005.)

Step1Secret

11 Whatarecomplementproteinsandhowdotheyfunctioninanimmune
response?
Complementproteinscompriseanetworkofsolubleplasmaproteinsthatbecomeactivatedasa
cascadebyIgMandIgG(classicalpathway)orbysurfacemoleculesofmicroorganisms(alternative
andlectinpathways).Thecomplementproteinshavemanyimportantbiologicactivities.The
membraneattackcomplexmediatescelllysis,whereasothercomponentsparticipateinopsonization,
chemotaxis,neutralizationofpathogens,andclearanceofimmunecomplexes(Fig.153(f0020)and
Table153(t0025)).

Figure153
Thecomplementcascade.

(FromMandellGL,BennettJE,DolinR:PrinciplesandPracticeofInfectiousDisease,6thed.Philadelphia,
ChurchillLivingstone,2005.)
Table153
ComplementPathwayComponentsandActivity

BiologicActivity

ComplementComponent(s)

Celllysis

C5bC9(membraneattackcomplex
[MAC])

Degranulationofmastcellsandbasophils

C3a,C4a,C5a(anaphylatoxins)

Opsonizationofparticulateantigens

C3b,C4b,iC3b(opsonins)

Chemotaxisofleukocytes(mainlypolymorphonuclear
neutrophils)

C5a,C3a,C5b67(chemotactic
factors)

Viralneutralization

C3b,C5b9

Solubilizationandclearanceofimmunecomplexes

C3b

ThereisnoneedtofocusonlearningeverystepofthecomplementcascadefortheUSMLE.
Instead,focusonunderstandingtheprinciplesbehindactivationofdifferentcomplement
pathways,formationoftheC3andC5convertases,andconvergenceonafinalcommon
pathway.TheUSMLElovestoteststudentsontherolesofdifferentcomplementproteinsand
consequencesofcomplementfactordeficiencies(seequestion12,next).Forexample,you
shouldknowthatpatientswithC5C9(membraneattackcomplex)complementdeficienciesare
highlysusceptibletoinfectionbyNeisseriabacteria.
Step1Secret

12 Describetheramificationsofthemostcommoncomplementprotein
deficiencies
SeeTable154(t0030).
Table154
ComplementProteinDeficiencies

Complement Deficiencies
Protein(s)
C1esterase DeficiencyofC1esteraseinhibitorresultsinC1esteraseoveractivityand
inhibitor
overproductionofanaphylatoxins,leadingtorecurrentepisodesofangioedemathis
isalsoknownashereditaryangioneuroticedemaandisinheritedinanautosomal

dominantfashion.C1esteraseinhibitoralsoisinhibiteddirectlybybradykinin,which
explainstherarebutlifethreateningangioedemathatmayresultasasideeffectof
ACEinhibitors(ACEisresponsibleforthedegradationofbradykinin).
C2orC4

Thesedeficienciesoftenresembleautoimmunediseases(e.g.,systemiclupus
erythematosus,vasculitis)butfrequentlyareasymptomatic.
C2deficiencyisthemostcommoncomplementdeficiencyandalsomaybe
associatedwithsepticemia(typicallyduetoS.pneumoniae).

C3

ReducedlevelsofC3bpredisposepatienttorecurrentpyogenicinfectionsasa
consequenceofdecreasedopsonization.
RedbloodcellsalsorecognizeantigenantibodyC3bcomplexesincirculationand
transportthemtotheliverorspleenforphagocyticdegradation.DecreasedC3b
levelsinserumthuspredisposepatientstotypeIIIhypersensitivityreactionsdueto
decreasedimmunecomplexclearancefromcirculation.

C5C8or
mannan

ThesegreatlyincreaseriskofNeisseriainfectionsofnote,C5bC9formstheMAC,
whichcankillmostunencapsulatedgramnegativeorganisms.

binding
lectin(MBL),
ormannose
binding
protein
Decay

ThisproteinislocatedonthesurfaceofallhumancellsanddestabilizesC3

accelerating convertaseandC5convertase,preventingMACformation,therebyprotecting
factor(DAF) humancellsfromlysisdeficiencyismanifestedasanincreaseincomplement
orCD55and mediatedhemolysis,clinicallyapparentasparoxysmalnocturnalhemoglobinuria.
CD59)
DAFdeficiencyisdiagnosedwiththeHamtest,whichcheckstoseewhetherthe
fragilityofredbloodcellsincreaseswhenplacedinamildlyacidicsolution.
ACE,angiotensinconvertingenzymeMAC,membraneattackcomplex.

Note:Sincecomplementrepresentsasetofproteinsproducedbytheliver,generalizedcomplement
deficienciescanbeseeninliverfailureorindietarydeficienciesofcertainessentialaminoacids.

13 Whichcomplementcomponentsandcytokinesarerequiredforneutrophil
chemotaxis?
NeutrophilmigrationisdependentonIL8C5a(complementcomponent),andLTB4(aleukotriene).
Leukotrienesynthesisisdependentupontheenzymelipoxygenase,whichisinhibitedbythedrug
zileuton.

14 Asareview,listtheeffectorfunctionsofthemajorleukocyteclasses
SeeTable155(t0035).

Table155
LeukocytesandtheirEffectorFunctions

Cell

GeneralDescription

EffectorMechanism

Monocyte

Aphagocyticcellthatconstitutes410%ofthe
WBCsinperipheralblood.Severalhoursafter

Phagocytosis

releasefromthebonemarrow,monocyteswill
dieormigrateintothetissueanddifferentiatein
macrophagesordendriticcells.
Macrophage Thisisahighlyphagocytictissuedwellingcell.
Majorfunctionsincludephagocytosisof
particulatematerial,antigenpresentationtoT

Antimicrobicidalactivityincludes
generationofbothoxygen
dependentmediators(e.g.,

cells,andsecretionofIL1,TNF,IL8,andIL12. superoxideanion,hydrogen
peroxide,hypoclorousacid)and
oxygenindependentmediators
(e.g.,TNF,lysozyme,
defensins,hydrolyticenzymes).
Dendritic

Thispotentantigenpresentingcellformsan

cell

extensivewebintissuesfortrappingantigen
(e.g.,Langerhancellsintheepidermis).

Neutrophil

Thistypeofgranulocytemakesup~70%of

Phagocytosis

Likemacrophages,neutrophils

WBCsinperipheralblood.Neutrophilsareactive employbothoxygendependent
phagocytesthatarethefirstcelltoarriveatsites andoxygenindependent

Eosinophil

ofinflammation.

pathwaystogenerate
antimicrobialsubstances.

Thistypeofgranulocytemakesup25%ofthe
WBCsinperipheralblood.Itisaphagocyticcell

Exocytosisofgranulescontains
extremelybasicproteins.

thatmigratesintothetissuespaces,whereit
playsaroleindefenseagainstparasitic
organisms.
Basophil

Thisnonphagocyticgranulocytemakesup0.5
1%ofperipheralWBCs.Basophilsplayamajor

Basophilsrelease
pharmacologicallyactive

roleinallergicresponses.

substancesfromcytoplasmic
granules(histamineandother
vasoactiveamines)oncross
linkingofsurfaceboundIgEby
allergen.

Mastcell

Thistissuedwellingcellplaysaroleinallergic

Thiscellreleases

responsessimilartothatofbasophils.Mastcells pharmacologicallyactive
haveFcreceptors(forIgE)andhistamine
substancesfromcytoplasmic
containinggranules.

granules(histamineandother
vasoactiveamines)oncross
linkingofsurfaceboundIgEby
allergen.

HelperT

ThisCD4+lymphocytematuresinthethymus

TH1cells:IL12inducestheir

cell

andfunctionsincytokineproduction.HelperT
cellsplayacentralroleintheimmuneresponse

differentiation(fromTH0cells)

byregulatingthefunctionofcellssuchasCTLs,
Bcells,NKcells,andmacrophages.HelperT

macrophages),andIL2activates
CTLsandpropagatesthe

cellsareactivatedbyforeignantigeninthe
contextofMHCclassIImolecules.

response)
TH2cells:IL4inducestheir

secretesIFN(activates

differentiationactivateBcellsto
plasmacellswithIL4andIL5.
CytotoxicT

ThisCD8+lymphocytematuresinthethymus

Perforins,granzymes,IFN,

cell

andfunctionsindirectcellkillinguponactivation
byforeignantigenpresentedbyMHCclassI

TNF,FasL

molecules.
Bcell

ThisCD19+CD20+lymphocytehasmembrane
boundimmunoglobulin.Uponactivationby

Antibody

helperTcells,Bcellsmaydifferentiateinto
plasmacells,whichproducelargevolumesof
antibodies.Bypresentingendocytosedantigen
inthecleftofMHCclassII,Bcellsalsofunction
asantigenpresentingcellsintheactivationof
helperTcells.
NKcell

Thisisalargegranularlymphocytethathasno

Perforins,granzymes,IFN,

markersincommonwithBorTcellsandisnot
MHCrestricted.NKcellsacttolysevirally

TNF

infectedcells(viaADCC)andtumorcellswith
decreasedlevelsofMHCclassI.
ADCC,antibodydependentcellularcytotoxicityCTLs,cytotoxicTlymphocytesFasL,FasligandIFN,
interferonIgE,immunoglobulinEIL,interleukinMHC,majorhistocompatibilitycomplexNK,naturalkiller
TNF,tumornecrosisfactorWBCs,whitebloodcells.

15 Listthefunctionsofthemajorcytokinessecretedbyvariousclassesof
immunecells

SecretedbyTcells:
IL3:Hematopoieticstemcelldifferentiationintomyeloidprogenitors
SecretedbyTH1cells:
IL2:Tcellproliferation
IL12:TH1celldifferentiation
INF:ActivationofmacrophagesandTH1cells,suppressionofTH2cells,killingof
intracellularpathogens
SecretedbyTH2cells:
IL4:TH2celldifferentiation,promotesIgEandIgGclassswitch
IL5:Promoteseosinophilproliferation,aidsinIgAclassswitch
IL10:AntiinflammatoryactivatesTH2cellsandinhibitsTH1cells
Secretedbymacrophages:
TNF:Acutephasecytokinethatactivatesadhesionmoleculeexpressionon
endotheliumpromotesvascularleakresponsibleforsepticshockandcachexia
IL1:Acutephasecytokineactivatesadhesionmoleculeexpressiononendotheliumand
causesfever
IL6:Acutephasecytokinepromotesfever
IL8:Neutrophilchemotaxis
IL12:TH1celldifferentiation,NKcellactivation

16 Listthemajorcellsurfacemarkersusedtoidentifyvariousclassesofimmune
cells
SeeTable156(t0040).

Case151
A2yearoldboybecomesacutelyshortofbreath,hasaudiblewheezing,anddevelopspruritic
hives.Healsohasaboutofnauseaanddiarrhea.Hismothertakeshimtotheemergencyroom,
whereheisfoundtobedangerouslyhypotensive,withmarkedtachycardiaandtachypnea.

Table156
CellSurfaceMarkersUsedtoIdentifyClassesOfImmuneCells

TCells

BCells

Macrophages

NK

RBCs/WBCs/Platelets

Cells
CD2

Ig

CD14

CD16

(endotoxin
receptor)
CD3

CD19

CD4+(helper CD20

CD55andCD59(DAFprevents
complementmediateddamage)

CD16(Fc
receptor)

CD56

CD40

MHC
classI

Tcells)
CD8+(CTLs)

CD21

B7

CD28(binds

CD40(binds

MHCclassI

toB7)

toCD40L)

andII

CD40L(binds B7(bindsto

CR1(C3b

toCD40)

CD28)

receptor)

TCR

MHCclassI
andII

CD40L,CD40ligandCTLs,cytotoxicTlymphocytesDAF,decayacceleratingfactorMHC,major
histocompatibilitycomplexNK,naturalkillerRBCs,redbloodcellsTCR,TcellreceptorWBCs,whiteblood
cells.

1 Whatisthedifferentialdiagnosisforthispresentation?
Asthma,anaphylacticshock,bronchiolitis(inflammationofthesmallairwaysusuallyfollowinga
viralinfection),foreignbodyaspiration,andtoxinorallergeningestionmustbeconsidered.

Case151continued:
Themotherisquicklyquestioned,anditisdiscoveredthatthesesymptomsdevelopedshortly
afterthechildtookthefirstdoseofacourseofamoxicillin.Thechildisimmediatelygivena
subcutaneousinjectionofepinephrine,aswellasintravenous(IV)diphenhydramineand
methylprednisolone.Itisexplainedtothemotherthatthisacuteepisodewasmostlikelydueto
theamoxicillin,butsheseemsconfusedbecausehehadtakenamoxicillinpreviouslytotreatan
earinfectionandhadnoproblemswithit.

2 Whatisthemostlikelydiagnosis?

Acutesystemicanaphylaxis(anaphylacticshock)thecombinationofbronchospasm,urticaria,
andhypotensionmakesthisdiagnosismuchmorelikelythantheothers.Itshouldbenotedthat
anaphylaxisisthemostseriousresultofdrughypersensitivity.However,themostcommontypeof
drughypersensitivityinvolvescutaneoussymptoms,suchasgeneralizedflushing,urticaria(hives),
ormildangioedema.Penicillinsarethemostcommoncauseofmedicationinducedanaphylaxis,but
thisisstillaveryrarephenomenonwiththisclassofantibiotics.

3 Canfreepenicillincauseanaphylaxis?
No.Penicillinisonlylargeenoughtobindtoonearmofanantibody(i.e.,thevariableregionsofonly
oneheavychainandonelightchain)andthusisreferredtoasunivalent.Penicillinmustbeboundto
acarrierproteintoinducearesponse,andthereforeitisahapten.Whenpenicillinisboundtoits
carrierprotein,itcancrosslinkIgE(bindtoonearmoftwoseparatebutidenticalantibodiesand
increasethepropinquityofthecorrespondingFcreceptorsonthemastcellsurface),thustriggering
thereleaseofmediators.

4 Whattypeofhypersensitivityisanaphylaxis?Whatisits
immunopathogenesis?
Anaphylaxisisthesystemicformoftheimmediatehypersensitivityresponse(typeI).Uponfirst
exposuretoantigen,theantigenbindstoaBcell,whichpresentstheantigenonMHCclassIItoaT
H2cell.TheTcellprovidescytokinestoinduceIgEclassswitching.IgEbindstoFcreceptorsonthe

surfaceofmastcellsandbasophils,whereitpersistsuntilthesecondexposuretoantigen.This
antigenthencrosslinkstheIgEboundtomastcellsandbasophils,triggeringthereleaseofpre
formedhistamine(viacGMP,cAMP,andCa2+)andthedenovosynthesisandreleaseoflipid
mediators(e.g.,leukotrienes,prostaglandins,plateletactivatingfactor[PAF]).Intheimmediate
phase(occurswithinminutescausedbydegranulationofpreformedmediators),histamineinduces
awidespreadincreaseinvascularpermeabilityandsmoothmusclecontraction.Inthelatephase(>6
hourslater,asthesemediatorsweresynthesizedatthetimeofexposure),leukotrieneselicit
prolongedbronchoconstriction,mucussecretion,andanincreaseinvascularpermeability,while
prostaglandinD2andPAFcauseleukocytemigrationandactivation.Eosinophiliaoccursalongwith
thelatephasereactionduetoreleaseofeosinophilchemotacticfactor.Eosinophilsreleasecationic
granuleproteinsintendedtodestroyparasites,butintheabsenceofparasites,tissuedamageand
remodelingoccur.

5 Whatisthepathophysiologicexplanationforthewheezinganddiarrheathat
developed?
Releaseofmastcellmediators,suchashistamineandleukotrienes,causesanincreaseinvascular
permeabilityandcontractionofcertainsmoothmuscletypesinmultipleorgansystems.Inthe
airways,thisleadstolaryngealedema,bronchoconstriction,andmucushypersecretion,resultingin
theclinicalsymptomofwheezing.Inthegastrointestinaltract,theconsequenceofsmoothmuscle
contractionandedemaisnausea,vomiting,anddiarrhea.

6 Howdoesanaphylaxisresultintheurticariaobservedinthispatient?
Thewidespreadvasodilationandincreasedvascularpermeabilitythatdevelopinanaphylaxisallow
fluidtoaccumulateinthesuperficialdermis,producingsmallwhealswithapalecenterencircledby
aredflare.Thissamemechanismisresponsibleforcausingedemainthedeepdermis(angioedema)
andthelaryngealedemathatcandevelopinanaphylaxis.

7 WhydidthechildNOThaveareactiontoamoxicillinwhenitwasfirst
administeredforhispreviousearinfection?
Hewasnotsensitizedtotheallergenatthattime.Initialexposuretoanallergengeneratesallergen
specificIgEantibodiesafterseveraldays,whichbecomeboundtoFcreceptorsonthemastcell
surface(sensitization).Reexposuretothesameallergen(challengephase)crosslinkstheIgEbound
mastcellsandstimulatesdegranulation(seequestion4).

8 Whatclinicaltestingcanbeperformedtoconfirmthatthisimmediate
hypersensitivityreactionwascausedbyamoxicillin?
Afewdaysfollowingtheattack,askintestforamoxicillinandothercommondrugallergenscanbe
performed.Ifthechildisallergic,intradermalinjectionsofamoxicillinwillcausedegranulationof
localmastcellsatthesiteofinjection.Theresultantreleaseofhistamineandothermediatorswill
causealargewhealandflaretodevelopwithin30minutes.Thewhealisacentralraisedarea
reflectingleakageofplasmafromvenules(edema),andtheflareisasurroundingredareacausedby
vasodilation(erythema).

9 Whywasthechildimmediatelygivenepinephrine?
Epinephrinealleviatesthesymptomsofanaphylaxisthroughitspositiveadrenergiceffects.By
stimulating2receptors,itrelaxesbronchialsmoothmusclestoopentheairwaysbystimulating
peripheral1receptors,itconstrictssmallbloodvessels,therebyreducingvascularleakageand
raisingbloodpressure.

10 Whywasthechildgivendiphenhydramineandmethylprednisolone?
Diphenhydramine(Benadryl)isanH1receptorantagonistandwillamelioratethehistamine
mediatedcomponentsofanaphylaxis.
Methylprednisoloneisacorticosteroidthatactssynergisticallywithepinephrineby
upregulatingadrenergicreceptors.Italsoblocksaportionofthelatephaseoftheallergic
responsebydemarginalizingneutrophils(preventingsubsequentchemotaxis),reducing
eosinophilcounts,andblockingphospholipaseA2(theenzymethatproducesarachidonicacid,
whichiseventuallyconvertedintoprostaglandinsandleukotrienes).Notethatalthough
corticosteroidadministrationdecreasesplasmacountsofmanyleukocytesubtypes,itincreases
neutrophilcountsecondarytodecreasedadhesionmoleculesynthesis.Thisreducesadhesionof
polymorphonuclearcells(PMNs)totheendothelialwallandthusallowsmoretoenterthe

circulation.

Relatedquestions
11 WhatwasthemotivationfordevelopingthesecondgenerationH1receptor
antagonistssuchasfexofenadine(Allegra)andloratadine(Claritin)?
TheseagentsdonotcrossthebloodbrainbarrierlikethefirstgenerationH1receptorantagonists
anddonotcauseasmuchdrowsiness.Theyaregenerallyusedforallergicrhinitis,another(milder)
typeIhypersensitivityresponse.

12 ListthecommonclassesofdrugsthatareusedtotreattypeIhypersensitivity
disordersandtheirgeneralmodeofaction
SeeTable157(t0045).
Table157
DrugsUsedtoTreatTypeIHypersensitivityDisordersandtheirGeneralModeofAction

TypeofDrug

ModeofAction

Clinical

Comments

Indication(s)
Antihistamines

BlockhistamineH1andH Allergic
2receptorsontargetcells

Secondgenerationhistamine

rhinitis,atopic

receptorantagonistshavefewer

dermatitis,
allergic

adversesideeffects(e.g.,they
arenonsedating)

conjunctivitis
Mastcell

Prophylacticinhibitorsof

stabilizers

mastcellmediatorrelease andasthma

nedocromil

Methylxanthines

Inhibitionof

Asthma,

Includetheophyllineand

phosphodiesterase,in

bronchospasm aminophylline

additiontomanyother

resistantto

(debated)effects

othermodes
oftreatment

Antiinflammatoryblock

Allergic

Corticosteroids

Allergicrhinitis Includecromolynsodiumand

Includeprednisone,

productionofinflammatory asthma,atopic beclamethasone,triamcinolone,


cytokinesmultipleeffects

dermatitis

flunisolide

onseveraltypesof
leukocytes
Sympathomimetics Adrenergiceffects

Asthma
Epinephrinehasbothand
epinephrinein adrenergiceffectsalbuterol,
anaphylaxis

salmeterol,andmetaproterenol

areselectiveadrenergic
bronchodilators
Monoclonalanti

BindstoFcreceptors,

Severe

IgEantibody

preventingIgEbinding

asthma,

Includesomalizumab

uncontrolled
with
corticosteroids
Leukotriene
pathwayinhibitors

Prophylacticinhibitorsof
leukotrienesynthesisor

Asthma

receptorantagonists

Includecysteinylleukotriene
receptorantagonists(e.g.,
zafirlukast,montelukast)and5
lipoxygenaseinhibitors(e.g.,
zileuton)

IgE,immunoglobulinE.

Anaphylacticshockhasmanypotentialmanifestations,includingbronchospasm,
urticaria,hypotension,andgastrointestinal(GI)problems.
AnaphylacticshockisatypeI,IgEmediatedhypersensitivityreaction,withhistamine
andleukotrienesplayingthemajormediatorroles.
Epinephrine,diphenhydramine(anantihistamine),andcorticosteroidsarethemost
effectivetreatmentsforanaphylacticshock.
SeeTable158(t0050)formoreinformation.
SummaryBox:AnaphylacticShock

Case152
A43yearoldmanwithuncomplicatedpneumococcalpneumoniaisprescribeda10daycourse
ofpenicillinV.Ontheninthday,heappearsmildlyjaundiced,andhishematocrithasdropped
significantlyfromwhenhewasfirstseen.Hedeniesanyhemoptysis,hematemesis,melena,or
hematochezia.

1 Whatisthedifferentialdiagnosisforthispresentation?
Hemolyticanemia,chronicliverdisease(jaundiceiscommon,asisamacrocyticanemia),occult
hemorrhage,drugtoxicity,tumorobstructingbiliarytract(anemiaiscommonincancer,andbiliary
obstructionwouldleadtojaundice),andGilbertsyndrome(jaundicecanoccurifstressoranemiain

thiscaseissevereenoughtoinducemarkedlyreduceduridinediphosphate(UDP)glucuronyl
transferaseactivity)areallconsidered.

2 Whatadditionaltestsshouldbeorderedtofurtheranalyzetheanemiaand
jaundice?
Allpatientswithanemiashouldreceiveacompletebloodcount(CBC)witherythrocyteindices
(meancorpuscularvolume[MCV],meancorpuscularhemoglobin[MCH],meancorpuscular
hemoglobinconcentration[MCHC],redbloodcell(RBC)distributionwidth,andareticulocyte
count).Thepatientisalsopresentingwithjaundice,necessitatingliverfunctiontestsaswellasdirect
(conjugated)andindirect(unconjugated)bilirubinlevels.

Case152continued:
Theadditionaltestsrevealareticulocytecountof7%(normalis<2.5%),anMCVof90fL
(normalis80100fL),andanelevatedindirectbilirubin.Withtheseresults,youdecideto
immediatelyorderonemoretest.TheresultsofthedirectCoombstestarepositive.

3 Whatisthemostlikelydiagnosis?
Druginducedwarmautoimmunehemolyticanemia(AIHA).

4 WhatisthesignificanceofapositivedirectCoombstestandhowdoesit
supportthediagnosisconsideredinthispatient?
ACoombstestisanantiglobulinassaythatdetectsbothimmunoglobulinthatisattachedtothe
surfaceofapatient'sRBCs(directtest)andthepresenceofcirculatingimmunoglobulinagainstRBCs
(indirecttest).ApositivedirectCoombstestsupports,butdoesnotprove,thepresenceofan
immunemediatedhemolyticprocess.
Note:DistinguishingbetweendirectandindirectCoombsassayscanbetricky,butitisimportantto
understandthedifferencesbetweenthemethodologiesandusesforthetwotests.InadirectCoombs
test,antiIgantibodyisaddedtoapatient'sRBCs.IftheRBCsarealreadyboundtoimmunoglobulin
presentinthepatient'sownblood,theadditionofantiIgantibodywillcauseagglutination.Thistest
isusedtodetectthepresenceofimmunoglobulinsagainstapatient'sownRBCs(e.g.,hemolytic
diseaseofthenewborn,druginducedAIHA,transfusionreactions).Incontrast,anindirectCoombs
testrequiresadditionofnormalRBCstoapatient'sserum.IfimmunoglobulinsagainsttheRBCsare
presentintheserum,agglutinationwilloccur.Thus,anindirecttestmeasuresforimmunoglobulins
againstforeignbloodproductsthatarenotboundtothepatient'sownRBCs(e.g.,screeningfor
antibodiespriortobloodtransfusion,detectingRhantibodies).

5 Howdomedicationssuchaspenicillincauseautoimmunehemolyticanemia?
Certainantibiotics(e.g.,penicillin,cephalosporins,streptomycin,tetracycline)andothersmall
moleculesmaynonspecificallyadsorbtoproteinsonRBCsurfaces,formingacomplexsimilartoa
haptencarriercomplex.Thesedrugsaregenerallytoosmalltoelicitanimmuneresponseby

themselveshowever,theycanbecomeimmunogenicwhencombinedwithlargermoleculessuchas
membraneassociatedproteins.Insuchindividuals,thiscomplexcaninducetheformationof
antibodies,whichthenbindtotheadsorbeddrugonRBCs.Itisthepresenceoftheseautoantibodies
thatisdetectedinthedirectCoombstest.

6 Whattypeofhypersensitivitydoesautoimmunehemolyticanemiarepresent?
ThisisanexampleofatypeIIhypersensitivityreaction,inwhichIgMorIgGantibodiesbindtocell
surfaceantigens.OtherexamplesoftypeIIhypersensitivityincludebloodtransfusionreactions,
hyperacuterejectionoforgantransplants,andhemolyticdiseaseofthenewborn.

7 Whatisthemechanismbywhichhemolysisoccursindruginduced
autoimmunehemolyticanemia?
IgGdamagesRBCsbybindingandactivatingeffectorcellscarryingFcreceptors(e.g.,neutrophils,
macrophages,andNKcells).NotethatbothIgGandIgMcaninducecomplementmediatedlysisof
RBCs,butthismodeofdestructionisnotcommoninthisformofAIHA.
Note:Insomeindividuals,theextendeduseofcertaindrugs,includingmethyldopa,levodopa,
quinidine,andprocainamide,canstimulateproductionofantiRBCantibodies.Themechanismby
whichtheautoantibodiesareinducedisunknown,andtheantibodiesdonotcrossreactwiththe
drugthatappearstoelicittheirproduction.

Relatedquestion
8 IfthispatienthadhadaMycoplasmapneumoniaeinfectionratherthana
pneumococcalinfection,whattypeofautoimmunehemolyticanemiawould
havebeenconsidered?
ColdAIHA,inwhichantiM.pneumoniaeIgMcrossreactswitherythrocytesurfaceantigens.
Optimalbindingoccursat0to5C(comparedwith37CinwarmAIHA),andIgMinitiates
complementmediatedhemolysis.Thisoccursintravascularly,whereasinwarmAIHA,thesiteof
sequestrationandhemolysisisthespleen(extravascularhemolysis).

Warmautoimmunehemolyticanemia(AIHA)involvesantibodiesagainstredblood
cells(RBCs).Normocyticanemiawithahighreticulocytecountisseen.
TypeIIhypersensitivityischaracterizedbyantibodiesbindingtocellsurfacesand
catalyzingcellulardamage.
Haptensaresmallmoleculescapableofinducinganimmuneresponseonlywhen
coupledtolargercarrierproteins.Penicillinisahapten.
SeeTable158(t0050)formoreinformation.
SummaryBox:AutoimmuneHemolyticAnemia

Case153
A12yearoldboypresentstotheemergencydepartmentwithpuffyeyes,perioralswelling,a
tightfeelinginhisthroat,andwidespreadurticaria.Afewhoursafterarrival,hedevelopsa
fever,swollenlymphnodesandspleen,andswollenandpainfulankles.Hismotherreportsthat
hesteppedonanailduringvacationinZimbabwe1weekagoandwasgivenhorseantitetanus
immuneserum.Hismotheralsonotesthathisurineappearsfoamyanddiscolored.

1 Whatisthedifferentialdiagnosisforthisboy'spresentation?
Nephroticsyndrome,serumsickness,infectiousmononucleosis,glomerulonephritis,anaphylaxis,
juvenilerheumatoidarthritis(acutefebriletype),andupperrespiratorytractinfectionwouldbe
considered.

Case153continued:
Laboratorytestsofabloodsamplefromtheboyrevealanelevatedwhitebloodcell(WBC)
countinwhichamajorityofthecellswerelymphocytes.Plasmacellsweredetectedina
peripheralbloodsmear.HistotalserumcomplementlevelandhisC1qandC3levelswere
decreased.Urinalysisrevealedproteinuriaandhematuria.Thepatientwasstartedon
prednisone,andallofhissymptomsprogressivelyimproved.

2 Whatisthemostlikelydiagnosis?
Serumsickness.

3 Whatisthepathogenesisofserumsickness?
SerumsicknessisatypeIIIhypersensitivityreactioncausedbytheformationofantibodyantigen
(immune)complexesandtheirdepositionintissues,eventuatingintheactivationofthecomplement
cascade.Immunecomplexesformwhenantigenandantibodybindtogether.Inthiscase,the
antibodyismostlikelythepatient'sIgGwithspecificityforthehighlyimmunogenichorseserum
proteins,mostlikelytheFcportionofthehorseimmunoglobulins.Thesecomplexesmaydepositin
tissuesiflargeamountsofantigenandantibodyarepresent,creatingmoreimmunecomplexesthan
themononuclearphagocytesystemcanclearfromthecirculation.Additionally,someofthesmaller
immunecomplexesthatarenotclearedeffectivelybyphagocytosisaretakenupbyendothelialcells
invariouspartsofthebodyanddepositedintissues.Localactivationofthecomplementsystemby
thesecomplexesprovokesaninflammatoryresponse.

4 Whatisthereasonforthedelayfromtheserumadministrationtotheonsetof
symptoms?

Thepresenceofantigen(thoseinthehorseantitetanusimmuneserum)aloneisinsufficientto
producethereactionseeninserumsickness.Atimeintervalof2daysto2weeksisgenerally
necessaryfortheproductionoftherespondingantibodytoreachacriticallevelatwhichthenumber
ofantibodyantigencomplexesexceedsthethresholdatwhichtheycouldbeeffectivelycleared.The
resultingincreasingserumconcentrationofantibodyantigencomplexesleadstodepositioninthe
tissuesandthesymptomsseeninthispatient.
Note:Thenumberofantigensinthebodywillnotincreaseinaonetimeinjectionsuchasthisone,
sothesymptomswillsubsideoncethemononuclearphagocytesystemcatchesupandisableto
cleartheantigens.
BesuretopaycloseattentiontotimeintervalsontheUSMLEexamination,astheycanoften
providesignificantcluestothetypeofhypersensitivityreactiontakingplace.Althoughserum
sicknessmaycauseurticariasimilartothatassociatedwithtypeIhypersensitivity,thisreaction
willoccurdaysafterantigenexposureincontrastwithminutesafterantigenexposureseen
withtypeIreactions.
Step1Secret

5 Whatisthesignificanceofdecreasedserumlevelsofcomplement?
Thissignifiesconsumptionofcomplementsecondarytotheactivationoftheclassicalpathwayby
immunecomplexes.NotethatonlyIgGandIgMcontainingcomplexescanfixcomplement,which,
intheclassicalpathway,isaccomplishedbyC1bindingtoasiteintheFcregion.Althoughonlyone
IgMmoleculeisnecessarytoaccomplishthis,twocrosslinkedIgGmoleculesmustbepresentifthe
classicalpathwayistobeinitiated.

6 Whatcausedthehivesandfacialswellinginthepatient?
InitiationofthecomplementcascadeleadstotheproductionofcomplementcomponentsC3a,C4a,
andC5a(anaphylatoxins),whichinturnelicitmediatorreleasefrommastcells.Histamineand
leukotrienescausevasodilationandincreasedvascularpermeabilitythatleadtolocalizedand
systemicedema.

7 Whatisthesignificanceofredbloodcellsandproteinintheurine?
OnepotentialsiteofimmunecomplexdepositionintypeIIIhypersensitivityistherenalglomeruli.
Inflammationatthissite(glomerulonephritis)resultsinhematuriaandmildproteinuria.Notethat
mostcasesofglomerulonephritisarealsocausedbyatypeIIIhypersensitivityreaction:
poststreptococcalglomerulonephritisbycomplexesofIgGandStreptococcuspyogenesantigensand
systemiclupuserythematosus(SLE)nephritisbycomplexesofIgGandnuclearcomponentsthe
antigeninvolvedinIgAnephropathyisunclear(butnotethatthemesangialdepositionofIgA
complexesandC3suggeststhatthisdiseaserepresentsarareinstanceofIgAfixingcomplement).

8 Whatisthecauseofthisboy'sjointpain?

Immunecomplexdepositioninthesynovialtissue(oftheanklesinthiscase)resultsininflammation
andcausespain.Thismechanismisidenticaltothatforjointpainseeninrheumatoidarthritis,an
autoimmunediseasethatisalsoclassifiedasatypeIIIhypersensitivity.

9 DoesatypeIIIhypersensitivityreactionrequirepreviousexposure
(sensitization)toantigentooccur?
No.Ifanexogenousantigenisgiveninlargeexcessonfirstexposure,itcancausetheformationof
largenumbersofimmunecomplexesovertime[seequestion4](duringwhichsensitizationis
occurring),theirsubsequentdepositionintissue,andtheactivationofthecomplementcascade.

10 WhyhastheincreasinguseofantibodytherapyNOTledtoasharpincrease
intheincidenceofimmunecomplexmediatedillness?
Previously,therapeuticantibodiesweretypicallysynthesizedinotherspeciesandthuswerehighly
immunogenicinhumans.However,recombinanttechnologyhasnowallowedfortheproductionof
chimericantibodies(Fabisnonhuman,Fc[theusualsiteofantigenicity]ishuman)thataremuch
lessimmunogenic.Nowthatpurelynonhumanserum/antibodiesarerarelyused,drugsarethemost
commoncauseofserumsickness.

TypeIIIhypersensitivityinvolvesimmunecomplex(antigenboundtoantibody)
depositionatvarioussites.
Immunecomplexdepositioncausesdamagebyactivationofthecomplementcascade
andresultantneutrophilattraction.
Administrationofnonhumanantibodiescanleadtoserumsickness,butbymakingthe
Fcantibodyregionnonantigenic(suchasachimericantibodyFabisnonhuman,Fcis
human),thisreactioncanbeavoided.
SeeTable158(t0050)formoreinformation.
SummaryBox:SerumSickness/TypeIiiHypersensitivity

Case154
A23yearoldwomancomestoyourofficewithapruriticrashandnoothersymptoms.Sheis
alreadywearingagownwhenyouentertheroom,andyouimmediatelynoticelargeweepy,
erythematous,crustedpatchesandplaquesonherchest,face,andarms.Asyoumovecloser,
youseewithintheseareasclearanderythematousvesicles.Shealsohasswolleneyelids.

1 Whatisthedifferentialdiagnosisforthiswoman'srash?

Contactdermatitis(allergictype),contactdermatitis(irritanttype),atopicdermatitis,varicella
zostervirusinfection,andseconddegreeburnareconsidered.

2 Howcanallergictypeandirritanttypecontactdermatitisbedifferentiated?
Allergictypecontactdermatitisisarashthatdevelopsfromskinsubstancecontact,afterthepatient
hasbeenimmunologicallysensitized(asseeninpoisonivy).Irritanttypecontactdermatitisisarash
thatdevelopsfromrepetitiveskinirritation(asseeninrepetitivebodywashing,oftenamanifestation
ofobsessivecompulsivedisorder).
Additionally,atopicdermatitisandallergictypecontactdermatitiscanbedifferentiatedbyrash
characteristicsandbythetimeofonset.Atopicdermatitistendstoshowlichenificationfromchronic
scratching,anditappearsontheflexuralbodyregions,whereascontactdermatitistendstobe
erythematousandweepyanditissharplydemarcated,asitonlyappearsintheareaswheredirect
contactoccurred.Also,atopicdermatitisisatypeI,IgEmediatedhypersensitivityreactionthat
tendstobechronic(andassociatedwithasthmaandallergicrhinitis),butwhenanacuteexposure
occurs,symptomsshowupwithinminutes.Conversely,contactdermatitisisatypeIV,delayedtype
hypersensitivityreactionthus,whenanacuteexposureoccurs,symptomsshowup12to72hours
later(assumingthepatienthasalreadybeenexposedandsensitized).
BothatopicandcontactdermatitisarehighyieldtopicsfortheUSMLEStep1examination.
Step1Secret

Case154continued:
Youelicitthehistoryfromthepatientandfindthatsherecentlybegangoingtoatanningsalon.
About1weekago,shestayedinthelighttoolongandacquiredasunburncoveringherarms
andupperchest.SheusedSolarcaineaerosol(benzocaine20%,triclosam0.13%)torelievethe
painofthesunburn.After2days,shehadarashcoveringbothofherarms,herchest,andher
face.Shetookdiphenhydraminetocontroltheitching,buttherashdidnotimproveoverthe
lastweek.Shedeniesfever,fatigue,oranyotherassociatedsymptoms.

3 Whatisthediagnosis?
Shehascontactdermatitis(allergictype).

4 Whatisthecausativeagentandthemechanismbywhichitinducedan
immuneresponseinthepatient?
Benzocaineandotheringredientsintopicaldrugsconstituteamajorcauseofcontactdermatitis,a
typeIVordelayedtypehypersensitivity(DTH)response.Inthisresponse,antigenpresentingcellsin
theskin(dendriticLangerhanscells)internalizeselfproteinsthathaveexogenousmoleculesbound
tothem(haptenatedselfproteins),whichrenderstheseselfproteinsimmunogenic.These

complexesaredisplayedonclassIIMHCmolecules,whichbindtoTHcellreceptors(TCRs).This
resultsinactivationofsensitizedTH1(TDTH)cells,releaseofcytokines,andsubsequentactivation
oftissuemacrophages(viaIFN)andCD8+Tcells(viaIL2).Thereleaseoflyticenzymesbythe
macrophages,aswellasepidermaldamagebyCTLs,resultsintheerythema,weepiness,andvesicles
thatcharacterizedthispatient'sreactiontobenzocaine.

5 Whydidthepatienthavelesionsinareasotherthanonherarmsandupper
chest(wheresheappliedthespray)?
Thetopicaldrugcanbetransferredfromtheinitialpointofcontacttootherareasoftheskinbythe
fingernailsafterscratchingtheitchylesionsattheprimarysite.Therefore,unexpectedsitescanbe
affected(e.g.,theeyelidsandgenitals).Itisbeneficialtocutfingernailsshortandthoroughlywash
offtheskintoremovethedrugandpreventfurtherspread.

6 Whatisthetreatmentforallergictypecontactdermatitis?
Thepatientshouldbegivenacorticosteroidcontainingcream,andshecantakeoral
diphenhydraminetocontrolfurtheritching.Withthistreatment,therashshouldresolvein1to2
weeks.

7 Whyisitimportantforthepatienttoavoidtheuseofbenzocaineinthe
future?
Onceanindividualissensitized,eachsubsequentexposurenotonlyproducesthehypersensitivity
reactionbutalsogeneratesmoreeffectorandmemoryTcells.Thus,thereactionbecomesmore
severewitheachsubsequentexposure.MemoryTcellscanpersistforthelifetimeofanindividual.
Note:ThefirstcontactwithanallergendoesnottypicallyresultinatypeIVallergicresponse(asis
alsothecaseintypeIreactions).Accordingly,acontacthypersensitivityhasasensitizationstagein
whichaclonalpopulationofmemoryCD4+Tcellsisproducedandanelicitationstagewherebythe
memoryTcellsbecomeactivateduponsubsequentexposuretoantigen.Additionally,anindividual
maybeabletotouchthematerial(allergen)formanyyearswithoutsufferinganadversereaction(
Table158(t0050)).
Table158
SummaryofHypersensitivityReactions

Classification Definition

Mediator Mechanismof
Destruction

Clinical
Presentations

Detection

TypeI

IgE

Mastcell

Systemic

Skintesting,

immediate

degranulation

anaphylaxis,

RIST,RAST

hypersensitivity

inducedbyallergen

allergicrhinitis,

crosslinkedIgE

bronchialasthma,

IgEmediated

atopicdermatitis,

atopicdermatitis,
foodallergies
TypeII

Antibody

IgM,IgG Antibodiesdirected

Transfusion

Directand

mediated

againstcellbound

reactions,

indirect

cytotoxic

antigensinduce

hemolyticdisease

Coombs

hypersensitivity

destructionofcellsor ofthenewborn,
tissues

test

autoimmune
hemolyticanemia

TypeIII

Immune

Usually

Antibodiesdirected

Serumsickness,

complex

IgG

againstsoluble

glomerulonephritis, counts,total

mediated

serumantigenform

rheumatoid

hypersensitivity

circulatingcomplexes arthritis,SLE,

complement

thatdepositintissue

hypersensitivity

levels,

nonspecifically

pneumonitis

serumC3

damageis
complement

WBC
serum

andC1q
levels

mediated
TypeIV

Cellmediated

TH1

AntigenspecificTH1 Contactdermatitis, Patchtest

delayed

cells

cellsactivatetissue

tuberculintype

macrophagesand

hypersensitivity,

stimulatealocal

granulomatous
hypersensitivity,

hypersensitivity

inflammatory
responseover1272
hours

acutetissuegraft
rejection

IgE,IgG,IgM,immunoglobulinsE,G,MRAST,radioallergosorbenttestRIST,radioimmunosorbenttestSLE,
systemiclupuserythematosusWBC,whitebloodcell.

TypeIVhypersensitivityismediatedbyTH1cells,makingittheonlytypeof
hypersensitivitythatisnotantibodymediated.
Thedirectcausesofthedamageinthistypeofhypersensitivityareactivated
macrophagesandcytotoxicTlymphocytes(CTLs).
Contactdermatitisappearsonlyatsitesofdirectcontactwiththeoffendingsubstance
andcanbetreatedmosteffectivelywithcorticosteroids.
SeeTable158(t0050).
SummaryBox:ContactDermatitis

Case155
A4montholdbabyboyisbroughttoyourofficebyhisparentsforevaluationofarunnynose
andacoughthathavepersistedforoveramonth.Examinationrevealsoralthrush,absenceof
tonsilsandlymphnodes,andafallfromthe50thpercentileinweighttothe10thpercentile.A
chestxraystudyisperformedanddemonstratesdiffuse,symmetricalinterstitialopacities.
LaboratorytestsrevealmarkedlymphopeniawithnormalnumbersofCD20+cells(B
lymphocytes)andanabsenceofCD3+cells(Tlymphocytes).

1 Whatisthedifferentialdiagnosis?
Severecombinedimmunodeficiency(SCID),humanimmunodeficiencyvirus(HIV)infection,bare
lymphocytesyndrome,ataxiatelangiectasia,andatypicalDiGeorgesyndrome(thymicaplasia)are
considered.

Case155continued:
FurtherspecializedtestingrevealedthatbloodlymphocyteswereunresponsivetotheBandT
cellmitogen,pokeweed(PWM).AdiagnosisofPneumocystisjirovecii(formerlyPneumocystis
carinii)pneumoniawasmade,andthebabyrespondedwelltointravenoustrimethoprim
sulfamethoxazole(TMPSMX).Althoughhewasconsideredlowrisk,testingtoruleoutHIV
infectionwasperformedandwasnegative.

2 Whatisthemostlikelydiagnosisinthisinfant,giventhefactthatspecialized
testingrevealeddefectsinbothcellularandhumoralfunction?
SCIDismostlikely.Thisconditionisinvariablyfatalininfantsunlessrecognizedandtreatedbybone
marrowtransplantation.Priortothis,SCIDpatientsmustavoidexposuretoanymicroorganismsby
residinginaplasticbubble.Typically,infantswithSCIDbecomeillwithinthefirst3monthsoflife,
sufferingfromrecurrentrespiratoryinfections,pneumonia,thrush,diarrhea,andfailuretothrive.
OpportunisticinfectionswithintracellularpathogenssuchasCandidaalbicans,Pneumocystis
jirovecii,Cryptococcusneoformans,cytomegalovirus,andmycobacteriaarecommonlyobservedin
theseinfants.

3 Whymightabonemarrowtransplantfromanappropriatedonorcurethis
boy?
ThepathogenesisofSCIDinvolvesabnormalproductionandfunctionofmatureBandTcells.By
transplantingnormallymphocyteprecursors(nogeneticdefect)theinfant'simmunesystemwill
reconstitutewithmature,functionalBandTlymphocytes.

4 WhatisthesignificanceofthemarkedlymphopeniaandcompletelackofCD3
+
cells?

SCIDisafamilyofprimaryimmunedisordersthatischaracterizedbylownumbersofcirculating
lymphocyteswhilemyeloidanderythroidcellsappearnormalinnumberandfunction.Withinthe
lymphocytepopulation,Tcellnumbersaretypicallylowtoabsent,Bcellnumberscanrangefrom
fairlynormaltoabsent(althoughtheseBcellsaretypicallynonfunctional),andNKcellnumberscan
belowtonormal.Thus,thereisfailuretomountbothhumoralandcellmediatedimmuneresponses,
butthelackofcellmediatedimmunitymakespatientswithSCIDhighlysusceptibletoopportunistic
infectionswithintracellularorganisms,thehallmarkofthisdisease.
Note:SCIDmaylookverysimilartoDiGeorgesyndrome,inwhichfailuretodevelopthethirdand
fourthpharyngealpouchesresultsinanabsentthymusandcompletelackofTcells.BothSCIDand
DiGeorgesyndromescanproduceanabsentthymicshadowonchestxrayfilms.However,DiGeorge
syndromedoesnotaffectBcells,isgenerallyassociatedwithcongenitalheartdefects,andoften
producessymptomsofhypocalcemiaduetoabsentparathyroidglands.

5 WhyareBcelldefectsnotevidentinmanybabieswhentheyarefirst
diagnosedwithseverecombinedimmunodeficiency?
Theinfantshaveantibodiesintheircirculationthathavebeenpassivelyobtainedfromtransplacental
circulation(IgG)andfrombreastmilk(IgA).Astheseantibodiesareclearedovertimeandtheinfant
becomesresponsibleforitsownimmunoglobulinsynthesis,alllevelswillbelowtoabsent.

6 MutationsinthesubunitoftheIL2receptorarefoundinthemost
common,Xlinkedformofseverecombinedimmunodeficiency(SCID).How
canthisexplainthemanifestationsofthedisease?
ThecommonchainisencodedbytheIL2Rgene(locatedontheXchromosome)andisaprotein
sharedbyalargenumberofinterleukinreceptorsinvolvedinlymphocytedevelopment.Thus,in
additiontotheIL2receptor,thesubunitisassociatedwithreceptorsforIL4,IL7,IL9,andIL
15.MutationsintheIL2Rgeneresultinanabsenceofthesefunctionalreceptorsandcanleadto
decreasedgrowthanddevelopment(aswellasdecreasedactivationandproliferation)ofbothBand
Tcells.Interestingly,thelackofTcellsresultsprimarilyfromanonfunctionalIL7receptor.As
discussedpreviously,theIL2receptorisalsoessentialforTcelldevelopment.
Note:OthergeneticdefectsknowntocauseSCIDincludeabsenceofclassIIMHC,dysfunctional
ZAP70(Tcellsignaltransductionprotein),defectiveRAG1orRAG2genes(TCRandIg
recombination),andabsentadenosinedeaminaseorpurinenucleosidephosphorylase(aidsinpurine
breakdown).AdenosinedeaminasedeficiencyisthesecondmostcommonformofSCIDandresults
inthebuildupofdATP(deoxyadenosinetriphosphate),whichinhibitsactivityofribonucleotide
reductase.Decreasedproductionofdeoxyribonucleotidesinhibitslymphocyteproliferationandthus
resultsinimmunosuppression.

RelatedQuestions:BCellDisorders

7 Whichprimaryimmunodeficiencyshouldbesuspectedinachildwithnormal
cellmediatedimmunitybutalmostcompleteabsenceofplasma
immunoglobulins?
Bruton'sagammaglobulinemiaisanXlinkedimmunodeficiencysyndrome.AffectedboyshavefewB
cellsintheirbloodorlymphoidtissue,andtheirserumusuallycontainsverylowlevelsofall
immunoglobulins.ThedefectinvolvesBruton'styrosinekinase,whichiscrucialtothematurationof
preBcells.Maternalimmunoglobulinremainstoprotectthechilduntilabout6monthsofage.
XlinkedimmunodeficienciescanberememberedusingthemnemonicMissingWBCs,which
standsforhyperIgMsyndrome,WiscottAldrichsyndrome,Bruton'sagammaglobulinemia,
Chronicgranulomatousdisease,andseverecombinedimmunodeficiency(SCID)(most
commontype).
Step1Secret

8 WhichBcelldisorderisassociatedwithanaphylactictransfusionreactions?
IgAdeficiency(themostcommonimmunodeficiency,~1:700)isduetoadefectinheavychain
isotypeswitching,soaffectedpatientscannotproduceIgA(andsometimesIgG2andIgG4aswell).
Althoughmanyofthesepatientsareasymptomatic,theymayalsopresentwithrecurrentsinusand
pulmonaryinfections,persistentgiardiasis,diarrhea,allergies,andasthma.Themostserious
complicationofthisconditionistheproductionofspecificIgEagainsttheFcportionoftheIgA
heavychain,whichcanresultinalifethreateninganaphylacticreactiontobloodproductsthathave
notbeenpurgedofthisimmunoglobulin.
ThescenarioofananaphylacticreactionfollowingbloodtransfusionisafavoriteontheStep1
examination.Considerthiswheneverapatienthasananaphylacticreactionafterblood
transfusion.
Step1Secret

Severecombinedimmunodeficiency(SCID)involvesdeficitsinbothTandB
lymphocytes,leadingtoopportunisticinfections.
Ininfantsbefore6monthsofage,mostinfectionsinSCIDarebyintracellular
pathogens,evidenceofdeficientcellmediatedimmunity.Duringthistime,humoral
immunityiskeptintactbyIgAcontainingbreastmilkandIgGfromtransplacental
circulationthathasnotyetbeendegraded.
Bruton'sagammaglobulinemiaisaBcell(andthusimmunoglobulin)deficit.Severe
(usuallybacterial)infectionsappearafter6monthsofage,whenmaternal
immunoglobulinlevelsfall.

SeeTable1510(t0060)formoreinformation.
SummaryBox:SevereCombinedImmunodeficiency

Case156
A5yearoldboypresentstotheemergencydepartmentwithsevereshortnessofbreath,a
persistentcough,andchestpain.Achestxrayfilmrevealsthepresenceofmultiplelarge,fuzzy
opacitiesinbothlungfields.LaboratorytestsrevealanormalWBCwithnormalproportionsof
neutrophils,lymphocytes,andmonocytes.Abriefreviewoftheboy'smedicalhistoryshows
multipleinfectionswithorganismsinthegeneraStaphylococcusandBurkholderia.
Laboratorytestsfrom1monthagoshowedasimilarWBCprofile,aswellasserumantibody
levelsinthehighnormalrangeandnormallevelsofcomplementproteins.

1 Whatisthedifferentialdiagnosisforthisboy'sapparentimmunodeficiency?
ComplementdeficienciesandBcelldisorderscancauserecurrentbacterialinfections(although
thesegenerallyinvolveencapsulatedorganisms[Streptococcuspneumoniae,Haemophilus
influenzae,Neisseriameningitidis,Salmonellaspp.,etc.]ratherthanthoseseeninthispatient),but
thesehavebeenessentiallyruledoutwiththeresultsfrom1monthago.Thepresenceofrecurrent
StaphylococcusaureusandBurkholderia(agenusofgramnegativerodsquitesimilarto
Pseudomonas)infectionsisstronglyofsuggestiveofaphagocytedeficiency.ThetwoWBCprofiles
makecyclicneutropeniaandsecondaryneutropeniamuchlesslikely,thusleavingchronic
granulomatousdisease(CGD),ChdiakHigashisyndrome(CHS),andleukocyteadhesiondeficiency
atthetopofthedifferentialdiagnosislist.

Case156continued:
InaWBCfunctiontest,theboy'smonocytesandneutrophilsfailedtoreducenitroblue
tetrazolium(NBT),atestoftheadequacyoftheoxidativerespiratoryburst.Sputumcultures
returnandarefoundtohavegrownAspergillusfumigatus.HeispromptlystartedonIV
liposomalamphotericinB.Heslowlyimprovesovera2monthperiod,butduringthistimein
thehospitalhecontractstwobacterialrespiratoryinfections.Whenheisdischarged,heis
startedontreatmentwithinjectionsofIFNanddailyprophylacticTMPSMX.

2 Whatisthediagnosis?
HehasCGD.
Note:ItisimportanttounderstandthebasisoftheNBTtest.ThecolorofNBTischangedfrom
yellowtodeepblueinthepresenceofreactiveoxygenspecies.Thus,absenceofcolorchangewhen
NBTisaddedtoapatient'sWBCsindicateslackoffunctionalnicotinamideadeninedinucleotide
phosphate(NADPH)oxidase.

3 Whatarethetwomechanismsthatamacrophagecanemploytokillbacteria
followingphagocytosis?
Destructionofinternalizedpathogensbymacrophages(andneutrophils)involvesbothoxygen
dependentandoxygenindependentmechanisms.Duringphagocytosis,ametabolicprocessknown
astherespiratoryburstoccurs.ThisprocessisdependentupontheactivityofNADPHoxidase,which
catalyzestheproductionofsuperoxide(O2)fromoxygen.Superoxidedismutasethenconverts
superoxidetohydrogenperoxide(H2O2),whichisconvertedtohypochlorite(HOCl,averypotent
bactericidaloxidant,commonlyknownasbleach)bymyeloperoxidase.Themacrophagemayalsokill
ingestedpathogensusinglysozyme,varioushydrolyticenzymes,andcytotoxicpeptides.The
degradativeactivitiesofthesefactorsdonotrequireoxygen.Hydrogenperoxidethatleaksoutofthe
phagolysosomeispotentiallyharmfultotissuesandisthusneutralizedtowaterbytheenzyme
catalase,whichrequiresreducedglutathioneforitsaction.

4 Whatisthegeneticbasisofchronicgranulomatousdisease?
AllformsofCGDinvolvemutationsinthegeneencodingNADPHoxidase(65%oftheseareXlinked
recessiveand35%areautosomalrecessive).Adeficiencyinthisenzymeresultsinaninabilityof
phagocytestokillmanytypesofingestedpathogens.Therefore,thesemicroorganismspersist,
leadingtotheformationofgranulomasinanattempttowallofftheinvaders.Notethatwidespread
granulomasarefoundinpatientswithCGDevenintheabsenceofinfectionthisispresumablydue
tounopposedproinflammatorycytokineproduction.

5 Whatarethecontentsofagranulomaandwhyisitformed?
Agranulomaisacollectionofepithelioidandgiantcellssurroundedbyafibrouscapsule.Generally,
agranulomaformstocontainanintracellularpathogenthathasbeenresistanttoelimination(but
sterilegranulomasarecommoninCGD,asdiscussedpreviously).Notethatepithelioidcellsare
modifiedmacrophages,andgiantcellsaretheresultofthefusionofmultiplemacrophages(hence
thesupernumerarynuclei).ThesecellsaremaintainedbycontinuedsecretionofIFNbylocalTH1
cells(Fig.154(f0025)).

Figure154
Schematicillustrationofgranulomaformation.
(FromKumarV,AbbasAK,FaustoN:RobbinsandCotranPathologicBasisofDisease,7thed.Philadelphia,WB
Saunders,2005.)

Itisimportanttobeabletorecognizeimagesofgranulomasandtheirmajorconstituentsfor
thepurposeoftheUSMLE.
Step1Secret

6 Patientswithchronicgranulomatousdiseaseoftenexperiencerecurrent
staphylococcalinfections,butstreptococcalinfectionsarerare.Why?
OrganismsinthegenusStreptococcusarecatalasenegative.Asmentionedearlier,catalasecatalyzes
thedegradationofhydrogenperoxide(2H2O22H2O+O2).Intheabsenceofcatalase,
hydrogenperoxideisnotdegradedandcanbestolenbythephagocyteandconvertedto
hypochloritebymyeloperoxidase.Thisallowsforkillingoftheorganismintheabsenceoffunctional
NADPHoxidase.
Conversely,catalasepositiveorganismsquicklyeliminateallhydrogenperoxidethattheyproduce
andthuscannotbekilledviarespiratoryburstinhostslackingfunctionalNADPHoxidase.
Therefore,commoninfectingorganismsinCGDincludeS.aureus,Serratia,Burkholderiacepacia,
Aspergillus,Nocardia,andCandida.NotethatNeisseria,althoughcatalasepositive,rarelycauses
infectionsinCGDbecausephagocyteshaveapropensitytokillthisorganismbyoxygenindependent
mechanisms(seequestion3).

7 WhatisthemechanismofactionofamphotericinB?
Thepolyenefamily,includingamphotericinBandnystatin,actsbyselectivelybindingtoergosterols
(intheplasmamembraneoffungalcells)andcreatingporesinthemembrane.Thisresultsinleakage
ofintracellularcomponents,precipitatingcelldeath.AmphotericinBpossessesanextremelybroad
antifungalspectrum,butitsundesirablesideeffectprofile(chills,fever,nephrotoxicity,arrhythmias,
hypotension)canbeprohibitive.Nystatinisreservedfortopical(diaperrash,vaginalcandidiasis)
andoral(oralcandidiasis)use.

8 HowdoesliposomalamphotericinBdifferfromamphotericinB?

Themechanismofactionandtheantifungalspectrumareidentical,butliposomalamphotericinB
causessignificantlyfewersideeffects,mostnotablyintherealmofrenaltoxicity.Notethat
amphotericinB'srenaltoxicityresultsfromafferentarteriolarconstrictionanddirectdamageto
distaltubularepithelium.

RelatedQuestions:PhagocyteDisorders
9 HowdoesChdiakHigashisyndromedifferfromchronicgranulomatous
disease?
CHSisalsoaprimaryimmunodeficiencyofthemyeloidlineage.Itisanautosomalrecessivedisorder
thatrenderslysosomesofneutrophilsincapableoffusingwithphagosomes(membranebound
pocketscontainingingestedmicroorganisms).CHSisthuscharacterizedbyintracytoplasmic
granulesinPMNsandmonocytes.RecurrentinfectionswithS.aureus,aswellasorganismsofthe
StreptococcusandPseudomonasgenera,areseen.Inadditiontoafailureofphagosomelysosome
fusion,defectsaffectmelanocytes,nervecells,andplatelets.CommonsymptomsofCHSthusinclude
neuropathy,partialalbinism,andbleedingdiathesis.IncontrastwithCGD,CHSyieldsanormal
NBTordichlorofluorosceintestbecausephagocyticNADPHoxidaseisfunctional.

10 Compareandcontrastmonocytesandmacrophageswithrespecttoorigin,
location,lifespan,andfunction
Bothmonocytesandmacrophagesarephagocytesderivedfrommyeloidstemcellsinthebone
marrow.Monocytescirculateinthebloodstreamforseveralhours,duringwhichtimemanyenlarge,
migrateintovarioustissues,anddifferentiateintomacrophages.Monocytesthatremainintheblood
willsurvivefor1to2days,whereasthosethatbecometissuedwellingmacrophageswillhavelonger
lifespansof4to15days.Macrophagesservedifferentfunctionsindifferenttissuesandarenamed
accordingtotheirlocation(Table159(t0055)).
Table159
MacrophageTypeswithLocationandFunction

MacrophageType

TissueLocation

Function(s)

Kupffercells

Liver

RemovalofsenescentRBCsanddebris

Splenicmacrophages

Spleen

RemovalofsenescentRBCsanddebris

Mesangialcells

Kidney

Phagocytosis,contraction,support

Microglialcells

CNS

Ingestdegeneratedmyelin

Histiocytes

Connectivetissue Phagocytosis

Alveolarmacrophages Lung

Phagocytosisofinhaledparticles

CNS,centralnervoussystemRBCs,redbloodcells.

Phagocyticcellscankillviaoxygendependent(productishypochlorite)andoxygen
independent(e.g.,lysozyme,cytotoxicpeptides)mechanisms.
Chronicgranulomatousdisease(CGD)stemsfromdefectivenicotinamideadenine
dinucleotidephosphate(NADPH)oxidaseandresultsinrecurrentinfectionswith
catalasepositiveorganisms.
ChdiakHigashisyndromeisadisorderoflysosomalfusion.UnlikeinCGD,
streptococcal(inadditiontostaphylococcal)infectionsareseen.
SeeTable1510(t0060)formoreinformation.
SummaryBox:ChronicGranulomatousDisease

Case157
A5dayoldgirlisbroughttoyoubyhermotherforevaluationofrepeatedmuscleseizures
thatbegan3daysago.Sheisafebrile.Physicalexaminationrevealsseveralfacialabnormalities,
includinglowsetears,wideseteyes(hypertelorism),andmicrognathia.Laboratorytestsare
remarkableforhypocalcemiaandlymphopenia.Achestxrayfilmrevealsabootshapedheart,
decreasedpulmonaryvasculaturemarkings,andtheabsenceofathymicshadow.

1 Whatisthelikelydiagnosisinthisinfantandwhatisthepathophysiologyof
herdisorder?
DiGeorgesyndrome(thymicparathyroidaplasia).Thisgeneticdisorderiscausedbymicrodeletions
ontheqarmofchromosome22,resultingindysmorphogenesisofthethirdandfourthpharyngeal
pouchesduringembryologicdevelopment.Becausethethymus(thirdpouch)andtheparathyroids
(inferiortwofromthirdpouch,superiortwofromfourthpouch)normallyarisefromthistissue,
thesestructuresmaybeabsentinDiGeorgesyndrome(Fig.155(f0030)).Forsimilarembryologic
reasons,theheartandaortaareoftenaffectedaswell.DiGeorgesyndromeistypicallydetectedinthe
firstweekoflifefollowingevaluationofinfantssufferingfromhypocalcemia(from
hypoparathyroidism)inducedtetany,which,asseeninthisinfant,mayresembleseizures.

Figure155
Maturationofthepharyngealpouches.
(FromCummingsCW,FlintPW,HaugheyBH:Otolaryngology:Head&NeckSurgery,4thed.Philadelphia,
Mosby,2005.)

Note:Themechanismforhypocalcemiainducedtetanyinvolvesincreasedsodiuminfluxintonerve
terminalswithdecreasedcalciumconcentrations.

2 JosephHellermightask,Whattriedandtruemedicalschoolmnemoniccan
beusedtoremembertheclassicmanifestationsofDiGeorgesyndrome?
Congenitalheartabnormalities(commonlytruncusarteriosusortetralogyofFallot)
Abnormalfacialfeatures
Thymicaplasia
Cleftpalate
Hypocalcemiafromhypoparathyroidism
22qdeletion

3 Towhattypeofinfectionsmightthischildbevulnerable,giventhatthymic
developmentisabnormal?
ThethymusisaprimarylymphoidorganthatisresponsibleforthematurationofprogenitorTcells.
Whenthethymusisunderdevelopedorlacking,thereisadramaticdecreaseinallpopulationsofT
cellsandacorrespondinglackofcellmediatedimmunity.PatientswithDiGeorgesyndromewho
survivetheimmediateneonatalperiodaresusceptibletorecurrentorchronicfungalandviral
infectionsthatwouldnormallybeeradicatedbyCTLs.Additionally,becauseTHcellsandtheir
cytokines(e.g.,IL2,IL4,IL5)arecriticaltotheproliferationanddifferentiationofBcells,
deteriorationofhumoralimmunitywilloccurovertimeasmaternalantibodiesaredegraded(by
about6monthsofage).Thus,thepatientwillalsobecomesusceptibletoinfectionwithextracellular
pathogenssuchasbacteria,whicharenormallywardedoffbythehumoralsystem.

4 Whatmightalymphnodebiopsyinthisinfantreveal?
ThymicabsenceorhypoplasiainDiGeorgesyndromeresultsininadequateproductionof
functionallymatureTcells.Histologicanalysisoflymphnodeswouldthereforerevealnearcomplete
absenceofTcellsinthenormalTcelldependentzonesoflymphnodes,suchastheparacortex(this
liesbetweenthenodularcortexandthemedulla).

Relatedquestion
5 Whatdoestheprocessofthymiceducationinvolve?

Theeducationofthymocytesinvolvesbothpositiveandnegativeselectionandensuresthat
matureTcellswillbeMHCrestrictedandselftolerant,respectively.Duringpositiveselection,T
cellsabletorecognize(bind)MHCmoleculesareselectedtosurvive(othersundergoapoptosis).
NegativeselectioninvolvesapoptosisofTcellsthatavidlybindselfpeptideinassociationwithMHC
molecules(selfreactiveTcells).Lessthan5%ofthethymocyteswillsurvivetheseselection
processes.
AquickreviewofprimarydeficienciesisgiveninTable1510(t0060)
Table1510
SummaryofPrimaryImmunodeficiencies

Classification Example(s)of

ImmuneDefect

Susceptibility

Immunodeficiency
Syndromes
Bcell

Bruton's

FewtonoBcellsonly

Recurrent

deficiencies

agammaglobulinemia
(Xlinked

immunoglobulinfoundinserumis
IgG(atverylowlevels)

pyogenic
infectionsappear

hypogammaglobulinemia)

afterageof6
months

IgAdeficiency

BcellsfailtoproduceIgAandoften

Recurrentsinus,

IgG2andIgG4

respiratory,and
gastrointestinal
infections
anaphylactic
transfusion
reactions

XlinkedhyperIgM

NoBcellclassswitchingfromIgMto Recurrent

syndrome

otherisotypes(duetoadefectin
CD40ligand)

pyogenic
infections,poor
responseto
immunizations

Commonvariable

Lateonset(ages2035years)

Recurrent

immunodeficiency

agammaglobulinemiathathas
acquiredandinherited

pyogenic
infections

characteristicscommonlyfollows

increasedriskof

viralinfection

various
autoimmune
diseasesand
lymphoma

Tcell

DiGeorgesyndrome

deficiencies

Tcelldeficitresultingfromthymic

Opportunistic

aplasia(pharyngealpouch

infections:viral

maldevelopment)

andfungal
infections

Chronicmucocutaneous
candidiasis

Tcelldysfunctionspecificto
Candidaalbicans

Recurrentskin
andmucous
membraneC.
albicans
infections

Combined

Severecombined

DeficitofbothBandTcells

immunodeficiency(SCID)
Ataxiatelangiectasia

Opportunistic
infections

11q2223(codesforDNArepair

Recurrent

enzyme)deletionleadstoTcell

infections,

deficit,IgAdeficiency,cerebellar

increasedriskof

degeneration,andskinand

lymphoma

conjunctivatelangiectasia
WiskottAldrichsyndrome XlinkedFactinassemblydefectin

Infectionswith

TcellsandplateletsnormalIgG,

encapsulated

decreasedIgM,andincreasedIgA

bacteriaatopic

andIgE

dermatitis
thrombocytopenia

Barelymphocyte

DefectiveorabsentclassIand/orII

Recurrent

syndrome

MHCmolecules

infections,
commonlyviral

Complement SeeBasicConcepts,question12
deficiencies
Phagocyte

Chronicgranulomatous

Defectivephagocyticrespiratory

deficiencies

disease(CGD)

burst,resultingfrommutationsinthe susceptibilityto
NADPHoxidasegene

Increased
certainbacteria
andfungi
widespread
granuloma
formation

ChdiakHigashi

Defectiveneutrophillysosomal

Recurrent

syndrome

fusion

staphylococcal
andstreptococcal
infections

Leukocyteadhesion

Defectiveneutrophilintegrin(LFA1: Recurrentsevere

deficiency

mediatesadhesion),resultingin
defectivemigration

pyogenic
infections
delayedumbilical
cordseparation

IL12receptordeficiency

DefectiveIL12receptorprevents

Disseminated,

theinitiationofaTH1response

severe
mycobacterial
infections

IFNreceptordeficiency

DefectiveIFNreceptorprevents

Disseminated,

macrophageactivation(andthe

severe

killingofintracellularpathogens)

mycobacterial
infections

Job'ssyndrome

FailureofhelperTcellstoproduce

Recurrentcold

IFN,preventingmacrophage

(i.e.,theabsence

activationandtiltingthebalance

ofinflammation)

towardtheTH2pathwaythisleads

staphylococcal

toincreasedIgEandalevelof

abscesses,atopic

histaminethatinhibitsneutrophil

dermatitis

chemotaxis
Cyclicneutropenia

Autosomaldominantirregular

Severebacterial

productionofGCSFleadsto

infectionsduring

neutropeniafor36daysofa21day neutropenic
cycle

phaseonly

GCSF,granulocytecolonystimulatingfactorIFN,interferonIgA,IgE,IgG,IgM,immunoglobulinsA,E,G,
MIL,interleukinLFA1,lymphocytefunctionassociatedantigen1MHC,majorhistocompatibilitycomplex
NADPH,nicotinamideadeninedinucleotidephosphate[reduced].

A22qdeletionresultsinDiGeorgesyndrome(thymicparathyroidaplasia),inwhichthe
thirdandfourthpharyngealpouchesfailtodevelopproperly.
Thethirdpouchformsthethymusandtheinferiortwoparathyroidglands.
Thefourthpouchformsthesuperiortwoparathyroidglands.
RememberthemnemonicCATCH22:Congenitalheartabnormalities(commonly
truncusarteriosusortetralogyofFallot),Abnormalfacialfeatures,Thymicaplasia,C
leftpalate,Hypocalcemiafromhypoparathyroidism,22qdeletion.

SeeTable1510(t0060).
SummaryBox:DigeorgeSyndrome

Case158
A25yearoldmancomestoyouroffice,forthefirsttimeinmanyyears,withcomplaintsof
generalfatigue,aheadache,muscleaches,andamildfever.Hestatesthatthishaspersistedfor
4or5daysandthathejustwantstogetmyenergyback.Hetakesnomedications.He
appearssomewhattiredandshowsmildpsychomotorretardation.Physicalexaminationis
notableforscatteredlymphadenopathyandasmallpatchynonspecificrashontheneck.

1 Whatisthedifferentialdiagnosis?
Infectiousmononucleosis,hypothyroidism,influenza,occultinfection,anemia,depression,acute
HIVinfection,malnutrition,adrenalinsufficiency,nonHodgkin'slymphoma,anddermatomyositis
shouldbeconsidered.

Case158continued:
Youwriteordersforabatteryofstandardtestsandgivethepatientdirectionstothelaboratory.
Asyouareabouttheleavetheroom,thepatientstopsyouandasks,Afewmonthsago,I
startedusingintravenousheroin.Doyouthinkthatmightberelatedtotheseproblems?

2 Whatmustbesuspected?Whattestsareusedtodiagnosethisdiseaseafter
theacutestage?
HIVinfectionisnowmorelikely.DetectionofHIVantibodiesisaccomplishedbyenzymelinked
immunosorbentassay(ELISA),thepreferredscreeningtest(highsensitivitytest).IftheELISAis
positive,thetestmustbeconfirmedbyWesternblotanalysis(highspecificitytest),whichispositive
ifantibodiesfromthepatient'sserumaredemonstratedtointeractwithHIV1proteinsdisplayedon
theacrylamidegelusedinthetest.AWesternblotmustbepositiveforantibodiestoatleasttwo
importantHIVantigens(e.g.,gp120,gp41,p24).Ifonlyoneantibodyispositive,theresultis
indeterminateandthetestmustberepeatedafterafewmonths,oranHIVPCRassaymustbedone.
HIVtestresultsareoftenfalselypositiveinnewbornsborntoHIVinfectedmothersbecausethese
antibodiescancrosstheplacenta,butareoftenfalselynegativewithinthefirstfewmonthsof
infection.

3 Describetheenzymelinkedimmunosorbentassaytest
First,knownantigens(HIV1proteins,inthisinstance)arefixedtothebottomofawell.Next,the
patient'sserumisadded,andifheorshehappenstopossesstheantibodiesofinterest,theywillbind
totheantigensinthewell.Then,antibodiesagainsthumanIgGwithenzymeslinkedtotheirFc
regionareadded.Iftheoriginallymentionedantibodiesarepresent(andtheyareIgG),thesecond

antibodywillbind,andwhenthesubstrateisadded,theenzymewillinduceachangeincolor.This
activitycanbedetectedusingaspectrophotometer.Thehighestdilutionoftheoriginalserumin
whichcolorchangeisdetectedrepresentstheantibodytiter.

Case158continued:
TheresultsofhisHIVELISAandWesternblottestarepositive.Youcontinuetofollowupwith
himregularly,and5yearslaterhecomesforanotherregularlyscheduledappointmentwitha
fewquestions.Younoteinhischartthathehasbeenonhighlyactiveantiretroviraltherapy
(HAART)forafewyearsnow.HestatesthathehasbeenreadingabouttheHIVstages,and
asks,WhichstageamIin?Additionally,hesaidheheardaboutsomethingcalled
Pneumocystiscariniipneumonia(PCP)thatcanhappenwhenyourCD4+Tcellcountgets
below200andrequestsmoreinformationonthistopic.

4 Whataretheclassesofdrugsusedinhighlyactiveantiretroviraltherapy?
Thefirstclassofantiretroviraldrugsincludestwotypesofreversetranscriptaseinhibitors.
Nucleosideanalogreversetranscriptaseinhibitors(nRTIs)areincorporatedbyreversetranscriptase
intothetranscribedDNAstrand,wheretheyblockfurtherextensionofthestrandandthereby
inhibitviralreplication.Theyrequirephosphorylationbythymidinekinasetobecomeactive.Non
nucleosidereversetranscriptaseinhibitors(nnRTIs)inhibittheactionofthereversetranscriptase
enzymebutbindatasiteotherthanthecatalyticsiteanddonotrequireactivationbythymidine
kinase.Asecondclassofdrugscalledproteaseinhibitors(PIs)mimicpeptidesthatHIVprotease
cleaves,butbindmoretightlyandspecifically,thusinhibitingtheprotease.HAARTrefersonlyto
combinationsoftwonRTIspluseitheronennRTIoronePI.Table1511(t0065)listssomeantiHIV
drugs(commonnames)thatareinclinicaluse.
Table1511
DrugsinClinicalUseforTreatmentofHumanImmunodeficiencyVirus(HIV)Infection

ReverseTranscriptaseInhibitors
NucleosideAnalogs

ProteaseInhibitors*(tf0010)

Nonnucleoside
Analogs

Bonemarrowsuppression

Bonemarrow

Hyperglycemia,lipodystrophy(buffalohump),

andlacticacidosis

suppressionand

andgastrointestinalintolerance

rash
Zidovudine

Delavirdine

(azidothymidine[AZT])

Indinavir
Thrombocytopenia

Megaloblasticanemia
Lamivudine(3TC)

Nevirapine

Ritonavir

Fewersideeffects
Stavudine(d4T)

Pancreatitis
Efavirenz

Saquinavir

Pancreatitis,peripheral
neuropathy
Didanosine(ddI)

Nelfinavir

Pancreatitis,peripheral
neuropathy
Zalcitabine(ddC)

Amprenavir

Pancreatitis,peripheral
neuropathy
Abacavir

Lopinavir

Hypersensitivityreactions
Combivir(3TCandAZT)

Atazanavir

Italicprintindicatescommonsideeffects.
Otheragents:tenofovir(nucleotideanalog)andenfuvirtide(preventHIVTcellfusionusedonlyifmultipleother
drugshavefailed).

*Notethatalloftheseagentsendinnavir.
Learningthevarioushumanimmunodeficiencyvirus(HIV)drugscanbeadifficulttask,as
theirnamesaresomewhatcomplicated,buttheyarecommonlytestedontheUSMLE.Itis
importanttorecognizeboththemechanismofactionandpotentialsideeffectsforallHIV
drugsmentionedinTable1511(t0065).
Step1Secret

5 Suboptimalcompliancewithhighlyactiveantiretroviraltherapyrapidlyleads
toresistance.Why?
Anyallowanceofcontinuedviralreplication(e.g.,frommisseddoses)isdangerous,becausetheHIV
reversetranscriptaseenzymeisextraordinarilyerrorprone.Therefore,mutationsarisequickly,and
ifoneleadstodrugresistance,itcanproliferaterapidly.Ingeneral,nRTIresistanceoccursthrough
mutationsthatpreventnucleosideanalogincorporationorcreateamechanismofATPmediated
nucleosideanalogremoval.ResistancetonnRTIsandPIsusuallyoccursthroughmutationsthatalter
thebindingsitesforthesedrugs.

6 Brieflydescribehumanimmunodeficiencyvirusanditslifecycle

HIVisanencapsulatedretrovirus.Twoviralenvelopeglycoproteins,gp120andgp41,allowthevirus
toinfectCD4+Tcells(aswellassomemacrophages,dendriticcells,andmicroglialcells)that
expressanappropriatecoreceptor(thechemokinereceptorsCXCR4orCCR5).Notethat
macrophagesanddendriticcells,onceinfected,displayDCSIGN(aCtypelectinreceptor),which
allowsthesecellstobindtoanddirectlyinfectTcells.Uponentryintothecell,thevirusefficiently
copiesitsRNAgenomeintodoublestrandedDNAusingtheviralenzymereversetranscriptase.The
viralDNAcopyisintegratedintothehostcellgenome,aidedbytheviralenzymeintegrase.This
proviralformofthevirusmayremainlatentinthecelluntilitsexpressionissignaled.Expressionof
functionalproteinsbythevirusinvolvesavirallyencodedproteasethatcleavespolyproteinsinto
smallerfunctionalproteins.Whentheprovirusisexpressedtoformnewvirions,thehostcellswill
oftenlyse(Fig.156(f0035)).

Figure156
Thelifecycleofhumanimmunodeficiencyvirus(HIV).
(FromKumarV,AbbasAK,FaustoN:RobbinsandCotranPathologicBasisofDisease,7thed.Philadelphia,WB
Saunders,2005.)

7 WhatarethemajormechanismsofCD4+Tcelldepletioninhuman
immunodeficiencyvirusinfection?
SeeFigure157(f0040).

Figure157
MechanismsofCD4+Tcelllossinhumanimmunodeficiencyvirus(HIV)infection.
CTLs,cytotoxicTlymphocytes.(FromKumarV,AbbasAK,FaustoN:RobbinsandCotranPathologicBasisof
Disease,7thed.Philadelphia,WBSaunders,2005.)

8 Definethefourmajorstagesofhumanimmunodeficiencyvirusinfection
WithacuteHIVinfection,theindividualmayremainasymptomaticordevelopanacuteillnessthat
resemblesinfluenzaorinfectiousmononucleosissymptomsusuallydevelopwithin2to6weeks
afterinfection(asseeninthispatient).Duringthisstage,antibodiestoHIVaregenerally
undetectable.Seroconversionusuallyoccursduringclinicallatency,anasymptomaticperiodthat
wouldlastapproximately7to10yearsinanuntreatedpatient.Lowlevel(butpersistent)replication
ofHIVcausesagradualdecreaseinCD4+Tcells,andminoropportunisticinfectionsmayoccur.
Duringthecrisisphase,escalationofviralreplicationleadstoamorerapidTcelldecline(Fig.158
(f0045)).Thisisclinicallyapparentasweightloss,fever,fatigue,andlymphadenopathy.Acquired

immunodeficiencysyndrome(AIDS)isthediagnosisforapersonwhoisHIVpositiveandhasaT
cellcountbelow200L1orpresentswithoneoftheAIDSdefiningopportunistic
infections/malignancies.

Figure158
Typicalcourseofhumanimmunodeficiencyvirus(HIV)infection.
CTLs,cytotoxicTlymphocytes.(FromKumarV,AbbasAK,FaustoN:RobbinsandCotranPathologicBasisof
Disease,7thed.Philadelphia,WBSaunders,2005.)

9 Whatarethemostimportantdeterminantsoftheprogressionofhuman
immunodeficiencyvirusinfection?
CD4+Tcellcountindicatesthedamagethathasoccurredtotheimmunesystem,andhowclosethe
patientistoprogressingtoAIDS(acquiredimmunodeficiencysyndrome).Ahighcountisideal.
Normalcountrangesfrom500to1500L1.
Viralloadisanindicationofthepaceatwhichthedamageisoccurring.Alowviralloadisideal.
Viralloadservesasamarkerfordiseaseprogressionanddrugtherapyeffectivenessbymeasuring
theamountofactivelydividingHIVvirus.

10 WhatisPCPpneumoniaandwhatotheropportunisticinfectionsand
malignanciesmightbefallanAIDSpatient?
PCPreferstoPneumocystiscariniipneumonia(thepreferrednameisnowPneumocystisjirovecii
pneumonia).P.jirovecii(onceconsideredaprotozoanandnowclassifiedasafungus)pneumoniais
themostcommonsignificantopportunisticinfectioninHIVpatientsandtypicallyoccursinpatients
withaCD4+countofbelow200.PatientswithCD4+countsbelowthislevelshouldbestartedon
TMPSMXprophylaxis.Otheropportunisticpathogensandmalignanciesthatareamajorcauseof
deathinAIDSarelistedinTable1512(t0070).
Table1512
OpportunisticPathogensandMalignanciesWithMajorMortalityRiskinAcquiredImmunodeficiencySyndrome

Parasites

Bacteria

Fungi

Viruses

Malignancies

Toxoplasma

Mycobacterium

Pneumocystis

Herpessimplex

Kaposi's

spp.

tuberculosis

jirovecii

virus

sarcoma

Cryptosporidium Mycobacteriumavium

Cryptococcus

Cytomegalovirus Burkitt's

spp.

neoformans

Varicellazoster
virus

complex

Leishmaniaspp.
Microsporidium
spp.

lymphoma

Candidaspp.
Salmonellaspp.

Histoplasma

NonHodgkin's

capsulatum

lymphoma

Coccidioides

immitis
CommonopportunisticinfectionsatnotableCD4+counts:Toxoplasmaencephalitisat<100,
Cryptococcalmeningitisat<100,Mycobacteriumaviumcomplexat<50,andcytomegalovirus
retinitisat<50.CMVretinitisistreatedwithganciclovir,acompetitiveguanosineanalog.Inthe
eventthatganciclovirfails,foscarnet(viralDNApolymeraseinhibitor)isused.

11 Explainwhycertainindividualsremaininfectedwithhuman
immunodeficiencyvirusforlongerthan10yearswithnosymptoms
Theclinicallatencystageisvariableandtendstorunlongerinpatientswhoseviralloadislowor
whopossessamutantHIVstrainwithpartiallydefectivereplicationmachinery.Intheseindividuals,
CTLsareabletocontrolthevirus(i.e.,keeptheviralloadatlowlevels)withoutdestroyingverylarge
numbersofCD4+Tcells,thusdelayingtheonsetofopportunisticinfections.Additionally,certain
geneticpolymorphismsintheCXCR4andCCR5geneshavebeenshowntodelaydisease
progression.HomozygotesforCCR5mutationhavebeenshowntoberesistanttoHIVinfection.

Acutehumanimmunodeficiencyvirus(HIV)infectionischaracterizedbymany
nonspecificsymptomsandcanresembleinfectiousmononucleosis.
TheHIVscreeningtestisenzymelinkedimmunosorbentassay(ELISA).Westernblotis
usedastheconfirmatorytest.
HIV,withgp120andgp41,infectsTcellsthatexpressCD4andeitherCXCR4orCCR5.
HIVcanalsoinfectphagocyticcells(e.g.,macrophages,dendriticcells,microglialcells).
MicrogliainfectedwithHIVformsmultinucleatedgiantcellsinthecentralnervous
system(CNS).
HIVreversetranscriptaseisveryerrorprone,andthus,suboptimalhighlyactive
antiretroviraltherapy(HAART)complianceleadstorapidproliferationofresistant
strains.
HIVinfectionissaidtohaveprogressedtoacquiredimmunodeficiencysyndrome
(AIDS)whentheCD4+Tcellcountfallsbelow200L1orthepatientpresentswithone
oftheAIDSdefiningopportunisticinfections/malignancies.
Pneumocystisjiroveciipneumoniaisthemostcommonsignificantopportunistic
infectioninHIVpatientsandcausesadiffusesymmetricalinterstitial[lung]infiltrate.
Trimethoprimsulfamethoxazole(TMPSMX)isthetreatment(andprophylaxis)of
choice.
SummaryBox:HumanImmunodeficiencyVirus

Case159
A55yearoldwomandevelopedendstagerenaldiseaseduetopoorlycontrolledtype1
diabetesmellitus.Sheunderwenthemodialysistwiceaweekfor6monthswhilewaitingfora
kidneytransplant.Becauseshehadnolivingbloodrelatives,shewasinneedofacadaveric
donor.Acadaverickidneywasfoundfroma28yearoldwomanwhowasfatallyinjuredina
caraccident.ThedonorwasbloodtypeB,Rhpositive(matchingthepatient'sblood)withone
matchedHLAAallele.Priortothetransplantation,afinalcrossmatchwasperformedinwhich
therecipientwasshowntobenonreactive.Immunosuppressivetherapyfollowingthe
transplantprocedureconsistedofazathioprine,cyclosporine,methylprednisolone,and
antithymocyteglobulin(ATG).Twoweeksfollowingtheprocedure,thepatientwasdischarged
fromthehospitalonazathioprine,cyclosporine,andprednisone.Herbloodpressureand
serumcreatininewerenormal.
Shenowpresentstoyourofficeforher2weekfollowupvisit.Shecomplainsofdecreased
urineoutput,andherbloodpressureisfoundtobe155/95mmHg.Onphysicalexamination,
theregionofthegraftisenlargedandtendertothetouch.Youorderstandardlaboratorytests,
andtheChem7isnotableforaserumcreatinineof4mg/dL.

1 Whattwodiagnosesimmediatelyrisetothetopofyourdifferentialdiagnosis
list?
Inviewofherrecentsurgicalhistory,transplantrejection(acute,giventhetimingoftheprogression
ofthevariablesinthecase)mustbeconsidered.Additionally,notethatthetwomostcommon
adverseeffectsofcyclosporinearenephrotoxicityandhypertension,thusmakingcyclosporine
toxicityquitelikelyaswell.

2 Whatisthemechanismofactionofcyclosporine?
Cyclosporinebindstocyclophilin(oneoftheimmunophilins),andthiscomplexbindstoandinhibits
calcineurin.ThispreventscalcineurinfromdephosphorylatingthenuclearfactorofactivatedTcells
(NFAT).NFATisatranscriptionfactor,anddephosphorylationwouldallowittoenterthenucleus
andinducetranscriptionofthegenesforIL2andtheIL2receptor(essentialfortheclonal
expansionandactivationofTcells),aswellasanumberofothercytokines(Fig.159(f0050)).

Case159continued:
Arenalbiopsyisundertakenandshowsthepresenceofmanylymphocytesinthetransplant,
indicatingthatthekidneyisundergoingacuterejection.Sheisgivenhighdose
methylprednisoloneandOKT3.Thisimmunosuppressiveregimenprovedsuccessful,and
withinayearofthetransplant,sheisdoingwell.Shecontinuestotakelowdosesof
azathioprine,prednisone,andcyclosporine.

Figure159
Mechanismofactionofcyclosporineandtacrolimus.C,cyclosporineT,tacrolimusI,immunophilins.
(FromAdkinsonNFJr,YungingerJW,BusseWW:Middleton'sAllergy:PrinciplesandPractice,6thed.
Philadelphia,Mosby,2003.)

3 Discussthegeneticsofthemajorhistocompatibilitycomplexmoleculesand
theroleofclassesIandIIinhistocompatibility
EveryhumanhasasetofMHCglycoproteinsthatarereferredtoashumanleukocyteantigens
(HLAs).ClassIglycoproteinsareknownasHLAA,B,andCantigensandappearonthesurfaceof
nearlyallnucleatedcells.ClassIIglycoproteinsareknownasHLADR,DP,DQantigensand
appearonlyonantigenpresentingcells(e.g.,dendriticcells,Bcells,macrophages,Langerhanscells).
Therearealargenumberofgenes(multiplealleles)thatcodeforeachoftheclassIandclassII
glycoproteins.Eachpersonreceivesonesetofgenes(ahaplotype)encodingallsixoftheseantigens
fromeachparent.TheHLAsarecodominantlyexpressedonallcellsandareantigenicamong
differentindividuals.Becauseofthesemoleculardifferences,transplantedtissuesbetween
geneticallydifferentmembersofthesamespecies(allogeneic)arelikelytobeantigenicallydifferent
andthereforestimulateanimmuneresponse.
Note:Evenwhenatissueistransplantedbetweengeneticallydifferentindividualswithaperfect
ABObloodgroupmatchandaperfectMHCmatch,rejectioncanoccurbecauseofdifferencesat
variousminorhistocompatibilityloci.Althoughtherejectionisusuallylessvigorous,successful
transplantationbetweenMHCmatchedindividualsstillrequiressomeimmunosuppression.

4 Whattypesofrejectionmayoccurintransplantrecipients?
Hyperacuterejectionismediatedbypreformedserumantibodiesintherecipient,usuallyagainst
ABOantigensongraftendothelialsurfaces.Itgenerallyoccurswithinminutesoftransplantation,
andoftenfirstappearsasgraftwhitening(duetovascularthrombi)seenbeforetheconclusionofthe
surgery.Hyperacuterejectionisuntreatable,sotheorganshouldberemoved.

AcuterejectionismediatedprimarilybyCTLs,whichdirectlyattacktheparenchymalcellsofthe
graft.HLAmismatchesarethemostcommoncause,andclassIIMHCmismatchesarethemost
importantdeterminants.Asseeninthispatient,ittypicallyoccurs10to14daysaftertransplantation
andistreatablewithimmunosuppressivedrugs.
Chronicrejectiongenerallyoccursseveralmonthstoseveralyearsaftertransplantation.Themajor
pathologicfindingisgraftvasculardamage(atherosclerosisandfibrinoidnecrosis),anditisthought
thatthisisduetoeitherantibodymediateddamage(resultingfromminorhistocompatibilityantigen
mismatches)orsideeffectsofimmunosuppressivedrugs.Chronicrejectionisuntreatable.

5 Whatisthesignificanceofthepatient'shavingnolivingbloodrelatives?
ItismucheasiertoidentifygoodHLAmatchesinbloodrelatives.Identicaltwinshavethesame
histocompatibilitytype(HLAidentical).Theprobabilityisapproximately0.25thattwosiblingswith
thesameparentsareHLAidenticalatagivenlocusandapproximately0.50thattheyareonehalf
HLAmatched(haploidentical)atagivenlocus.Furthermore,parentsandchildrenarealmostalways
haploidenticalacrossallloci.Becausethepatienthadnolivingrelatives,acadavericdonorwasher
onlyoption.Acadavericdonorthatisbloodgroupcompatibleisoftenconsideredevenwithapoor
MHCmatch.

6 Whatdoesanonreactivefinalcrossmatchindicate?
ItindicatesthattherecipienthasnoantibodiesagainsttheWBCsofthepotentialdonor.Sensitizing
eventsincludingbloodtransfusions,pregnancies,andpreviouslyfailedtransplantsmayelicitthe
productionofantiHLAantibodiesintherecipient.Thepresenceofsuchpreformedantibodies(as
indicatedbyapositivecrossmatch)ishighlylikelytocausehyperacuterejection(inthisinstance,
withantiHLA,ratherthanantiABOantibodies,asnormallyseeninthistypeofrejection)ofthe
transplantedtissue.
Note:HyperacuterejectionoftransplantedtissueisatypeIIhypersensitivityresponse.

7 Howdoesazathioprineworkasanimmunosuppressiveagent?Antithymocyte
globulin?OKT3?Methylprednisolone?
Azathioprine,metabolizedto6mercaptopurine,isapurineanalogthatinterfereswithpurine
synthesis/metabolismandcanbeincorporatedintoDNAtoinhibitreplication.Itismosteffectivein
interferingwithrapidlydividingcells,suchasactivatedlymphocytes.Note:Mycophenolatemofetil
isanIMP(inosinemonophosphate)dehydrogenaseinhibitor.Thisleadstoadecreaseinthe
synthesisofpurines.Duetoitsincreasedspecificityforlymphocytes(andthus,decreasedside
effects),ithasreplacedazathioprineinmanyprotocols.
ATGsareantibodiesagainsthumanlymphocytesthatareproducedinlaboratoryanimals
(heterologousantibodies).Theantithymocyte(Tcell)fractionisisolatedandusedintransplant
recipientstodecreaseTcellnumbers.

OKT3/muromonabisamurinemonoclonalantibodythatisspecificforCD3onthesurfaceofTcells.
ItnotonlyhastheabilitytoinhibitTcellrecognitionofalloantigenbutalsoreducesTcellcounts.It
isquiteeffectiveinmanagementofacuterejection.
Methylprednisoloneisacorticosteroidantiinflammatoryagentthatfunctionstoreducethenumber
ofcirculatinglymphocytesaswellastheabilityoflymphocytestoactivateandproliferate.Italso
causesdecreasedchemotaxisofcellssothatfewerinflammatorycellsareattractedtothesiteof
activation(thegraft)(Fig.1510(f0055)andTable1513(t0075)).

Figure1510
Mechanismsandsitesofactionofimmunosuppressivedrugs.CsA,cyclosporineFKBP12,animmunophilinIL,
interleukinMHC,majorhistocompatibilitycomplexMMF,mycophenolatemofetilMPA,mycophenolicacid(NF
AT)CN,NFATtranslocatesfromcytosoltonucleusNFAT,nuclearfactorofactivatedTcellsTOR,targetof
rapamycin.
(FromMcPhersonRA,PincusMR:Henry'sClinicalDiagnosisandManagementbyLaboratoryMethods,21sted.
Philadelphia,WBSaunders,2006.)
Table1513

TransplantationRelatedImmunosuppressiveDrugs

Agent(s)

ModeofAction

Cyclosporineand

Calcineurininhibitorsblock CyclosporineisnephrotoxicFK506canbe

tacrolimus(FK506)

IL2synthesis,thereby

administeredatlowerdosesandhasfewer

preventingTcellactivation

sideeffectsthanwithcyclosporine

Rapamycinand

TORinhibitors,blockIL2

Rapamycincanbeadministeredatlower

sirolimus

dependentactivationofT

dosesandhasfewersideeffectsthanwith

cells

cyclosporine

Cyclophosphamide

Comments

Onemetaboliteisalkylating Acrolein(anothermetabolite)causes
agent(crosslinksDNA)

hemorrhagiccystitis(treatedwithmesna)

suppressesBcellsmore
thanTcells
Azathioprineand

Mitoticinhibitors(interfere

MMFhasgreaterlymphocyticselectivityand

mycophenolate

withpurinesynthesisand

thereforefewersideeffectsthanthose

mofetil(MMF)

metabolism),target

associatedwithitspredecessors,azathioprine

proliferatinglymphocytes

andcyclophosphamide

Blockcytokineproduction

Sideeffectsincludehypertension,

bymacrophages(TNF,IL

osteoporosis,musclewasting,acne,

1)decreasenumberof

hyperglycemia,cataracts

Corticosteroids

circulatinglymphocytes
Antilymphocyte

Decreaselymphocyte

Areheterologoussera(fromanotherspecies),

globulin

numbersandTcell

soadversereactionsincludeserumsickness,

(ALG)/antithymocyte numbers,respectively

immunecomplexinducedglomerulonephritis,

globulin(ATG)

andanaphylacticreactions

OKT3/muromonab

Murinemonoclonal

Isanonhumanantibody,sosideeffectsin

(antiCD3)

antibodyagainstCD3that

humansincludeserumsicknessandimmune

functionstoeliminateT

complexinducedglomerulonephritis

cells
Basiliximaband

Monoclonalantibody

Chimericmurinehumanmonoclonal

daclizumab(anti

againstIL2receptor

antibodiesthus,sideeffectsless

CD25)

(CD25),blocksIL2
binding,preventingTcell

severe/frequentthanwithpuremurineform

activation
CTLS,cytotoxicTlymphocytesIL1,2,interleukin1,2TNF,tumornecrosisfactor.
Note:Themajorgoaloftransplantrelatedimmunosuppressionistodecreasethegenerationandactivationof

helperTcellsandCTLs,whichmediateacuterejection.

Cyclosporineinhibitscalcineurin,preventingtranscriptionofseveralimportant
cytokines,suchasinterleukin2(andtheIL2receptor).Itsmajorsideeffectsare
nephrotoxicityandhypertension.
Hyperacuterejection:PreformedantiABOantibodiesoccludevesselsofgraftwithin
minutesoftransplantation.
Acuterejection:Tcellmediateddestructionofgraftparenchyma,usuallyduetohuman
leukocyteantigen(HLA)mismatches,occurswithindaystoweeksandcanbetreated
withimmunosuppressivedrugs.
Chronicrejection:Graftvasculardamageoccursmonthstoyearsaftertransplantation
(antibodymediatedvs.immunosuppressivedrugsadverseeffects).
SummaryBox:TransplantRejection

Case1510
A52yearoldmanreceivedabonemarrowtransplant(fromanHLAmatchedbrother)inan
attempttocuremultiplemyeloma.Thepatientwasadmittedtothehospitalandgivenacourse
ofbusulfanandcyclophosphamidetoeradicatehisownlymphocytes.Hewasthen
intravenouslyadministeredbonemarrowremovedfromthedonor'siliaccrest.Hishospital
recoverywentsmoothlyandhewassenthome,onlytoreturn4weeksafterthetransplant.He
nowpresentstoyouwithcomplaintsofanitchyrashonhischestandupperextremities,aswell
asseverediarrhea.Physicalexaminationrevealsamaculopapularrashwithfineexcoriationsin
theaforementioneddistribution,mildjaundice,andmildhepatomegaly.Thepatientisthen
startedontacrolimus(FK506),methotrexate,andtopicalcorticosteroids.Closefollowupover
thenextfewweeksisrecommended.Therashsoonfades,buttheintestinalsymptomspersist.
AfterinitiationofweeklyinjectionsofamonoclonalantibodytoCD2,thediarrheafinally
resolves.

1 Whatcausedtheadversereactioninthispatient4weeksafter
transplantation?
Graftversushost(GVH)diseasewasacute,becausethesignsandsymptomsbeganwithin100days
oftransplantation.

2 Whatisthepathogenesisofgraftversushostdisease?

GVHdiseaseisareactioninwhichmaturedonor(graft)Tcellsintransplantedbonemarrowactto
attackcellsintheimmunosuppressedrecipient(host).Thereactionisinitiatedwhen
immunocompetentTHcellsfromthegraft(allowedtosurvivebecausethehost'sleukocyteshave
alreadybeeneradicatedinpretreatment)areactivatedbyhostproteinsrecognizedasforeign.These
cellsproducehighlevelsofcytokinesthatrecruitandactivateotherTcells,macrophages,andNK
cellstocreatetheinflammationthatisseeninGVHdisease.However,notethatthebulkofthe
damageiscausedbygraftCTLsandtheiruncheckeddestructionofhostcells.Theseeventsleadto
theclassicGVHdiseasetriadofskinrash,diarrhea,andhepaticdysfunction.

3 Whydidthedonor'scellsreactagainsttherecipient'swhentheywereHLA
matched?
IndividualswhoareHLAmatchedsharethesameMHChaplotypes.Inotherwords,theyhaveMHC
classIandclassIImoleculesthataregeneticallyandantigenicallyidentical.However,eveninHLA
matchedindividuals,disparitiesinminorhistocompatibilityantigensexist.Theseallogeneic
molecules,whicharelikelytovaryinalldonorrecipientpairsotherthanidenticaltwins,havethe
potentialtoactivatematureTcellsofthedonor.
Note:UnlikeMHCantigens,whicharerecognizeddirectlybyTcells,minorhistocompatibility
antigensarerecognizedonlywhenpresentedbyselfMHCmolecules.Thus,tissuerejectiondueto
minorhistocompatibilitydifferencestakeslongertodevelop(severalweeks)andisoftenless
vigorous.

4 Whyaretheskinandtheintestinaltractthemajorsitesofgraftversushost
disease?
Recallthattheepidermalbarriersoftheskinandthegastrointestinaltractaretwoofthemajor
portalsofattemptedentryformicroorganisms.Therefore,theimmunologicprotectionatthesetwo
sitesisquiterobustthisincludesahighlevelofexpressionofhistocompatibilityantigens.InGVH
disease,suchexpressionwouldincreasetheprobability(andseverity)ofattackatthesetwosites.

5 HowdomonoclonalantibodiesagainstCD2helptreatgraftversushost
disease?
CD2isanantigenfoundonthymocytesandmatureTcells.AntibodyagainstCD2iseffectiveat
decreasingTcellnumbersbyelicitingtheirclearancebythereticuloendothelialsystem(e.g.,splenic
macrophages).ATGwouldhaveasimilareffect.

6 Whatisthemechanismofactionofbusulfan?
BusulfanisanalkylatingagentthatcausesDNADNAcrosslinkingandDNAproteincrosslinking.
Thisleadstocytotoxicity,whichisspecifictomyeloidcellsandcirculatinglymphocytesatlowdoses
andtohematopoieticstemcellsathighdoses.Whengivenincombinationwithcyclophosphamide,
highdosebusulfancanbeusedtoeradicateapatient'sleukocytesinpreparationforbonemarrow
transplantation,aswasseeninthispatient.CommonadverseeffectsincludeGIupset,pancytopenia,
andhyperpigmentation.Pulmonaryfibrosisisrare,butveryserious.

7 Whatisthemechanismofactionofmethotrexate?
Methotrexateisafolicacidanalogthatinhibitsdihydrofolatereductase.Thisleadstoalackof
tetrahydrofolate(THF),causingalackofmultiplenucleotideprecursors.ThispreventsDNA
synthesisand,importantlyforGVHdisease,preventslymphocyteproliferation.Notableadverse
effectsincludemyelosuppression,hepatotoxicity,interstitiallungdisease,andatransientneurologic
syndromewithproteanmanifestations.

8 Performanceofwhatlaboratorytestcouldhavepredictedthatgraftversus
hostdiseasewouldresultinthispatient?
Onewaymixedlymphocytereaction(MLR)testmeasuresproliferation(activation)ofdonor
lymphocytesinresponsetoirradiated(killed)recipientlymphocytes.Proliferationofthesecellsin
vitroisagoodpredictorofGVHdiseaseinvivo.

9 Howmightthedonor'smarrowhavebeentreatedtopreventgraftversushost
disease?
PartialTcelldepletionofdonormarrowbeforeinfusioncanreducetheincidenceofGVHdisease.
MonoclonalantibodiesagainstTcellantigens(e.g.,OKT3,ATG,antiCD2)areusefulintheremoval
ofTcells.

Graftversushost(GVH)diseaseischaracterizedbygraftTcellsattackingthecellsof
the[immunocompromised]host.
Theskinandgastrointestinal(GI)tractaretwoofthemajorsitesofpathogenentryand
thereforeconsistofcellswithaveryhighmajorhistocompatibilitycomplex(MHC)
expressiondensity.GVHdiseaseaffectsthesetwositesprominentlybecauseofthis.
Mismatchesbetweendonorandrecipientoccurnotonlyfromhumanleukocyteantigen
(HLA)MHCmismatchesbutalsofromdifferencesinminorhistocompatibilityantigens.
SummaryBox:GraftVersusHostDisease

Case1511
A26yearoldmancomestoyourofficewithachiefcomplaintofnewonsetpaininhisright
kneeandleftankleafterreturningfromatriptonorthernNewEngland2weeksago.Healso
mentionedtheonsetoffatigue,amildfever,andaneyeproblemtothemedicalassistant.
Beforeseeinghim,youbrieflyreviewhismedicalhistoryandseeonlyahistoryofpsoriasisand
alaboratorytestresultshowingthatheisHLAB27positive.

1 Hispresentation,coupledwithhisHLAB27status,bringswhatthree
diagnosesintoconsideration?

Ankylosingspondylitis,reactivearthritis(previouslyknownasReiter'ssyndrome),andpsoriatic
arthritisarepossible.

2 Completetheinitialdifferentialdiagnosis
Traumamustbeconsideredinapotentiallyactivepatient.Also,untilthesexualhistoryisknown,
gonococcalarthritiscannotberuledout.Lymediseaseshouldbeconsideredaswell,givenhistravel
history,andthetimecourseandsymptomswouldbesuggestiveofstage2.Otherpossibilities,
dependentonthependingdetailedhistory,includerheumatoidarthritis,SLE,substance/drug
inducedarthritis,septicarthritis,andviralarthritis.

Case1511continued:
Youinterviewthepatientandherecountsthefollowingstoryabouthisjointpain:Firstitwas
myrightknee,thenitwasmyleftankle,andnowIthinkmyrightanklemightbehurtinga
littletoo.However,healsonotesthathehadmultipleepisodesofdysuria2weeksago,and
thesehavenowreturned.Uponfurtherquestioning,headmitstohavinghadsexualintercourse
withapartnerhemetforthefirsttimeonhistrip.Physicalexaminationrevealsthattheright
kneeandleftanklearetenderandimmobile,withmoderateeffusions.Ophthalmologic
examinationrevealsanterioruveitisofthelefteye.Examinationofthegenitalsis
unremarkable.

3 Whatisthemostlikelycauseofthepatient'sclinicalpicture?
Reactivearthritisismostlikely.

4 Whatorganismsaremostcommonlyassociatedwithreactivearthritis?
Chlamydiatrachomatis,Campylobacterjejuni,YersiniaenterocoliticaandY.pseudotuberculosis,
SalmonellaenteritidisandS.typhimurium,andShigellaflexneri.

5 Whatshouldbedonenext?
Arthrocentesisshouldbeperformedtoruleoutinfection.Owingtothehighlikelihoodthathis
reactivearthritisisrelatedtoChlamydia,heshouldhaveaurinetestforthisorganism,aswellasfor
Neisseriagonorrhoeae.Becausehemightnotreturntofollowuponhisresults,itwouldbe
advisabletoinitiatepresumptivetreatmentforChlamydia(azithromycinordoxycycline)atthis
time.Additionaltestingforsexuallytransmittedinfections,suchasHIVandsyphilis,shouldbe
considered.Intermsofhisreactivearthritis,thesymptomsarebesttreatedwithnonsteroidalanti
inflammatorydrugs(NSAIDs).

6 Describethepathogenesisofreactivearthritis
Thoughstilloftendebated,reactivearthritislikelyresultsfrombacterialantigenmolecularmimicry.
Inshort,thismeansthatcertainantigens(fromChlamydia,inthiscase)haveamolecular
compositionthatissimilartothatofproteinsinthejoints,genitourinarytract,andtheeye.Thus,

whenTandBcellsinitiateattackontheoffendingorganism,theseselfproteinscanbeboundbyTCR
(ifpresentedonMHC)orantibody,anddamageensues.

7 Thecardiacmanifestationsofrheumaticfeverrepresentoneofthebest
examplesofmolecularmimicry.Discussthecrossreactingproteins
TheMproteinofStreptococcuspyogenesistheorganism'sprimaryantiphagocyticfactor.Certain
typesoftheMproteinverycloselyresemblemyocardialmyosin.IfaS.pyogenesinfectionisnot
treatedinlessthan9days,somepatientswillproducecrossreactiveantibodies,leadingto
myocardialdamage.NotethatMproteinmolecularmimicryisthoughttoplayaroleinother
symptomsofrheumaticfever(polyarthritis,cardiacvalvulardisease,skinnodules,skinrash,and
Sydenham'schorea)aswell.

8 Inadditiontomolecularmimicry,whatarethetwootherproposedmajor
mechanismsofautoimmunity?
Alterationofselfproteins:Asituationinwhichasubstancebindstoanormalselfproteinand
causesittoappearforeigntotheimmunesystem.ExamplesarepenicillininducedAIHA(see
Case152(s0170))anddruginducedlupus.Asimilarscenariooccurswhencertainviruses

infectacellandalterselectsurfaceproteins.
Compromiseofimmunologicallyprivilegedsites:Certaintissuesareneverexposedtothe
immunesystem,andthustolerancetowardthemisneverdeveloped.Examplesarethesperm
(viatheSertolicellbarrier),thebrain(viathebloodbrainbarrier),andcertainpartsoftheeye.
Ifdamagetothetissueandthebarrieroccurs,proteins(nowantigens)arereleasedinto
circulation,andimmunologicattackonthesourcetissueensues.Similarly,theimmunesystem
usuallydoesnotdeveloptolerancetointracellularproteinssuchasthoseassociatedwiththe
nucleus(e.g.,DNA,histones).Therefore,cellulardamageandreleaseoftheseantigens(as
mightbeseenwithalyticvirus)arethoughttoplayaroleinthedevelopmentofautoimmune
diseasessuchasSLE.

RelatedQuestions:MechanismsofTolerance
9 Tcelltoleranceinthethymus(centraltolerance)wasdiscussedinCase7,
question5.DiscusstheimportantprinciplesofperipheralTcelltolerance
AllTHcellsrequiretwothingstobecomeactivated:theinteractionoftheTCRandCD4withMHCII
plusantigen,andcostimulatorysignals.Ifthefirstinteractionispresent,butoneofthevital
costimulatorysignalsisabsent,theTcellbecomesanergic(nonreactive).Themostcommonlycited
costimulatorysignalisthebindingofB7fromtheAPCtoTcellCD28.Themeetingofthesetwo
moleculescanbepreventedbydownregulationofAPCB7expressionorcompetitionforB7binding
sitesbyCTLA4ontheTcell.CytokinesarealsoimportantinTcellactivation.Akeyplayerin
peripheraltoleranceisthesuppressorTcell,atypeofTHcell.Thiscellstrivestocontrolselfreactive
cellsandtoputthebrakesontheimmuneresponsebysecretingamyriadofinhibitorycytokines.

10 DescribeBcelltolerance
BcelltoleranceisnotaswellcharacterizedasTcelltolerance.Centraltoleranceoccursinthebone
marrow,butincontrastwiththymiceducation,theapproachissinglepronged:onlyselfreactiveB
cellsaredeleted(negativeselection).Peripheraltoleranceisalsoimportantandlikelyisdirectedtoa
largedegreebysuppressorTcells.

Molecularmimicry,oneoftheprobablemechanismsofautoimmunity,describesanon
selfmolecule(oftenpartofaninfectingvirionorbacterium)thatresemblesaself
molecule.Theresultantimmuneattackcancrossreactandleadtodamageofselftissues
containingthemimickedmolecule.
Autoimmunitymayalsobeduetovirus/druginducedalterationofselfproteinsand
compromiseofimmunologicallyprivilegedsites.
HelperT(TH)cellsrequiretheinteractionoftheTcellreceptor(TCR)andCD4with
classIImajorhistocompatibilitycomplex(MHC)plusantigenANDacostimulatory
signaltobecomeactivated.AbsenceofthecostimulatorysignalleadstoTHcellanergy.
DisruptionoftheCD28B7interaction(byCTLA4,forexample)isacommon
mechanismofcostimulatorysignalblockageandanergyinduction.
Reactivearthritisisareactiontoabacterialinfection,isassociatedwithHLAB27,and
isseenclinicallyasasymmetricalarthritis,urethritis,anterioruveitisand/or
conjunctivitis,rash,andfatigue.
SummaryBox:ReactiveArthritis

Copyright2015Elsevier,Inc.Allrightsreserved.

BOOKCHAPTER

MaleandFemaleReproductiveSystems
DavidAustinSchirmerMD,ThomasA.BrownMDandSonaliJ.Shah
USMLEStep1Secrets,Chapter9,252290

Studentstendtobrushoffreproductivephysiologyandpathologyduringtheirboardsstudying,
butinouropinion,thisisahugemistake.Thissubjectisincrediblystraightforward(andbound
togiveyoualotoffreepoints!)ifyouworkhardatit.Reproductivephysiologyistremendously
important.Youshouldhaveathoroughunderstandingofthemenstrualcycle,hormonal
regulation,andthefactorscontrollingmaleandfemalesexualdifferentiation.Thiswillbeof
enormoushelpwhenitcomestounderstandingpathologyandpathophysiology.Wehavedone
ourbesttoofferthemostboardsrelevantcasepresentationsinthischapter.Whenever
relevant,youshouldpaycloseattentiontothelaboratoryfindingsassociatedwiththese
diseasesastheyrelatetodisordersofthehypothalamicpituitarygonadalaxis.
Insider'sGuidetoMaleandFemaleReproductiveSystemsfortheUSMLEStep1

Basicconcepts
1 Whatisthenormaldurationofthemenstrualcycle?Whatarethetwoovarian
phases,andwhichoccursfirst?
Normalcycletimeisapproximately28days,andtheovarianphasesconsistofthefollicularand
lutealphases,withthefollicularphaseoccurringfirst.Thefirstdayofthemenstrualcycle(and
follicularphase)isdefinedasthedayonwhichmenstruationbegins(Fig.91(f0010)).

Figure91
Menstrualcycle.FSH,folliclestimulatinghormoneLH,luteinizinghormone.
(FromBrownTA:RapidReviewPhysiology.Philadelphia,Mosby,2007.)

2 Whichphaseoftheovariancycleisgenerallyresponsibleforthecyclebeing
longerorshorter?
Thelutealphaseisfixedat14daysinamajorityofwomen,sothedifferencesinthelengthofthe
follicularphaseaccountforcyclelengthdifferences.Forboards,youshouldsimplyassumethatthe
lutealphaseaccountsfor14daysofthemenstrualcycle.

3 Duringthefollicularphase,whathormonalchangesoccurinthepituitaryand
theovary?:
A.Pituitary
Becausethecorpusluteumhasinvolutedpriortothebeginningofthefollicularphase,andbecause
thisstructureistheprincipalsourceofestrogenduringtheprecedinglutealphase,plasmaestrogen
levelsatthebeginningofthefollicularphasearelow.Thisreducesestrogen'snegativefeedbackeffect
onpituitaryproductionoffolliclestimulatinghormone(FSH),soFSHsecretionbeginstorise.Note
thatthecorpusluteumisalsothemajorsourceofprogesteroneduringthelutealphase,so
progesteronelevelsarelowduringthefollicularphase.
B.Ovary
Duringthefollicularphase,therisingFSHstimulatesthedevelopmentofseveralovarianfollicles,
eventuallycausingtheemergenceofadominantfollicle,whichbecomesasiteofestrogensynthesis.
Thegrowingfolliclesecretesincreasingamountsofestrogen,whichhastheeffectofinhibiting
pituitaryFSHproduction,therebygraduallyreducingserumFSHlevelsinthelaterpartofthe
follicularphase.However,thedominantfolliclebecomesincreasinglysensitivetocirculatingFSH,so
plasmaestrogenlevelsstillcontinuetorisethroughoutthefollicularphase.
Note:InhibinisalsosecretedbytheovariesandselectivelyinhibitsFSHsecretionwithnoeffecton
luteinizinghormone(LH)secretion.

4 Whatoccursintheuterusduringthefollicularphase?
Theestrogensecretedbytheovariesstimulatesproliferationoftheendometrialliningoftheuterus
throughoutthefollicularphase.

5 Whathappenstocauseovulationattheendofthefollicularphase?
Whenplasmaestrogenreachesacriticallevel,itswitchesfromcausingnegativefeedbackonthe
pituitarytocausingpositivefeedbackbysensitizinggonadotropestogonadotropinreleasing
hormone(GnRH).ThisstimulatesthepituitarytoreleaseasurgeofLHandFSH.Thesurgeofthese

hormonescausesruptureofthefollicleandreleaseoftheovum.Thisisanelegantdesignfeature,
becausehighestrogenlevelsindicatetothepituitarythattheovarianfollicleissufficientlymature
tobereleased.

6 Whathappensintheovaryandendometriumduringthelutealphase?
Afterovulation,thecellsthatlinedthefollicle(granulosacellsandthecainternacells)formthe
corpusluteum(yellowbody)undertheinfluenceofLH.Thecorpusluteumsynthesizesboth
estrogenandalargeamountofprogesterone.Theprogesteronestimulatestheendometriumto
becomemoresecretoryandglandularinpreparationforimplantation.Italsostimulatesthespiral
arteriestodevelop.Thesearteriesemptyintotheintervillousspacesothatchorionicvillifromthe
cytotrophoblastcanextractoxygeniffertilizationoccurs.Iffertilizationdoesnotoccur,thecorpus
luteumdegenerates,thelevelsofestrogenandprogesteronefall,andtheendometriumsloughsoffas
themenstrualflow.
Youareexpectedtobeabletodifferentiatebetweenthehistologicappearancesoftheendometrial
liningduringthefollicularandlutealphases(Fig.92(f0015)).Noticetheglandularhypertrophy,
irregularshapeofglands,andwelldevelopedspiralarteriesintheluteal(secretory)phase.In
contrast,theglandsinthefollicularphasearesmallerandstraighter,andthereislessvascularityin
thestroma.

Figure92
Lowmagnificationviewoftheuterineendometriumduringtheproliferative(estrogenic)phaseofthe

reproductivecycle.Growingendometrialglandshavestraightprofiles.
(FromTelserA,YoungJ,BaldwinK:Elsevier'sIntegratedHistology.Philadelphia,Mosby,2008.)

7 Inwomenwithamenorrhea(absenceofmenses),whydoesbleedingafterthe
cessationofabriefcourseofprogesteroneindicatethatthecauseof
amenorrheaisduetothelackofovulation?
Inordertoanswerthisquestion,onehastohaveathoroughcomprehensionofthenormal
physiologyofthemenstrualcyclehence,thisisagoodboardstylequestion.
Iftheadministrationandwithdrawalofprogesteronecausemenses,thisindicatesthatthe
endometriumhasbeensufficientlyprimedbyestrogenandrequiresonlyacourseofprogesteroneto
havemenses.However,theonlysourceofendogenousprogesteroneisthecorpusluteum,which
formsonlyafterovulation.Ifthereisnoovulation,thereisnoprogesteronesecretion,andtherewill
notbemenses.
Themostcommoncausesofanovulationareobesity,polycysticovarysyndrome,eatingdisorders
(canresultindecreasedGnRHlevels),prematureovarianfailure,hyperprolactinemia(prolactin
inhibitsGnRHrelease),thyroiddisorders(increasedthyrotropinreleasinghormone[TRH]levels
canstimulateprolactinrelease),adrenalinsufficiency,andAsherman'ssyndrome.

8 Howdoesfertilizationpreventdegenerationofthecorpusluteum?
Iftheovumisfertilized,thedevelopingembryowillsynthesizehumanchorionicgonadotropin
(hCG),whichactssimilarlytoLHandmaintainsthecorpusluteum.

9 Howdoesthecorpusluteumfunctioninthemaintenanceofpregnancy?
Duringthefirst6weeksofpregnancythecorpusluteumistheprimaryproducerofestrogenand
progesterone,hormonesrequiredforthecontinuationofpregnancy.Afterthesixthweekof
pregnancy,theplacentabeginstotakeoverastheprincipalsiteofsteroidogenesis.Afterdeliveryand
removaloftheplacenta,bothestrogenandprogesteronelevelsfallmarkedly.

10 Describethehormonesassociatedwithnormaltesticulardescent
Normaltesticulardescentinvolvesatransabdominalphase,whichismediatedbymllerian
inhibitingfactor(MIF)(aproductofSertolicells),andascrotalphase,whichismediatedby
androgensandhCG.Cryptorchidismreferstoincompletedescentofthetestesintothescrotalsac.
Themostcommonlocationofundescendedtestesisintheinguinalcanal.Generally,theconditionis
unilateralandresolvesspontaneouslyby3months.Ifuncorrected,cryptorchidismcanresultin
infertilitysecondarytoarrestedgermcellmaturation(spermpreferthecoolertemperatureofthe
scrotalsac),increasedriskofseminoma,andtesticularinfarctionsecondarytotorsionofthe
undescendedtestes.

11 WhatisthefunctionoftheSRYgene?

TheSRYgeneislocatedontheYchromosomeandisresponsiblefortesticulardevelopment
(mediatedbytestisdeterminingfactor).TestescontainLeydigandSertolicells.Leydigcellssecrete
testosterone,whichstimulatesthedevelopmentofthemesonephricducts.Themesonephricducts
developintoallofthemaleinternalgenitaliaexceptfortheprostate(epididymis,seminalvesicles,
ejaculatoryduct,andductusdeferens).Testosteronecanalsobeconvertedintodihydrotestosterone
(DHT),whichstimulatesthedevelopmentofthemaleexternalgenitaliaandprostate.Sertolicells
produceMIF,whichpermitsthedegenerationoftheparamesonephricduct.Ifdegenerationdoesnot
occur,theparamesonephricductdevelopsbydefaultintothefemaleinternalgenitalia(uterus,
fallopiantubes,andupperthirdofthevagina).

12 Whatishermaphroditism?
Atruehermaphroditeisapersonwhohasbothmaleandfemaleinternalgenitalia(ovotestes).This
conditionisveryrare.Pseudohermaphroditismismorecommonthetermdescribesanypersonin
whomthereisdiscordancebetweenthesexoftheinternalandexternalgenitalia.Becausefemale
pseudohermaphroditeslackaYchromosome,theydonothaveanSRYgeneandthusdonotdevelop
testes.LackofMIFproductionresultsindevelopmentoffemaleinternalgenitaliafromthe
paramesonephricduct.Exposuretoexcessiveandrogenconcentrationsduringtheearlygestational
period(eitherbyexogenousintakeduringpregnancyorthroughcongenitaladrenalhyperplasia)
stimulatesproductionofvirilizedexternalgenitalia.Malepseudohermaphrodites,ontheotherhand,
docontainaYchromosomeandthusdeveloptestes.BecausetheirtestesproduceMIF,theydonot
developfemaleinternalgenitalia.However,theirexternalgenitaliaarefeminized.Thisismostoften
duetoandrogeninsensitivitysyndrome,inwhichthereisadefectintheandrogenreceptor.Asa
result,genotypicmalesareunabletodevelopmaleexternalgenitaliabuthavehighconcentrationsof
testosteroneandestrogenbecausethereisnonegativefeedbackfromthepituitaryduetolackof
receptorstimulation.

13 Whatis5reductasedeficiency?
Theenzyme5reductaseisresponsibleforconvertingtestosteroneintoDHT.BecauseDHT
stimulatestheproductionofmaleexternalgenitaliaandtheprostate,lackofDHTatbirthleadsto
ambiguousexternalgenitaliainthesemales.Internalgenitaliaareunaffected,becausetesticular
developmentisgovernedexclusivelybythepresenceoftheYchromosome.Atpuberty,increased
concentrationsoftestosteroneareadequatetostimulatedevelopmentofthemaleexternalgenitalia(
Table91(t0010)).Patientswiththisautosomalrecessiveconditionarethussaidtodevelopapenis
at12.
DisordersthatinfluencethedevelopmentofmaleandfemalegenitaliaarehighyieldforStep1.
Donotbesurprisedifyoureceiveaquestiononyourtestthatasksyoutoidentifywhether
varioushormonelevelsareincreased/decreased/normalinapatientwithagenital
developmentdisorder.UseTable91(t0010)asaguide.
Step1Secret

Case91
A29yearoldG2P2(gravida2para2)womanwhojustdeliveredhersecondbabywantsto
startbirthcontrolpillsforcontraception.Sheplansonbreastfeedinghernewborn.

Table91
ConditionsAffectingDevelopmentoftheGenitalia

Condition

Testosterone LH Explanation

Exogenous
steroiduse

Thebodyperceivesthatthereistoomuchtestosterone,so
itreducesproductionofLHandintratesticulartestosterone,
leadingtotesticularatrophy.

Androgen
insensitivity
syndrome

Althoughtestosteroneconcentrationismuchhigherthan
normal,LHproductionisnotinhibited,becausethe
testosteronereceptorsonthepituitarythatmediate
feedbackalsoaredysfunctional.

Primary

hypogonadism
Hypogonadotropic
hypogonadism

Problemwithtestosteroneproductionoriginatesatthe
levelofthetestes.

ProblemwithproductionofGnRHorLHleadstodecreased
productionoftestosterone.

GnRH,gonadotropinreleasinghormoneLH,luteinizinghormone.

1 Whatimportantinformationshouldyoufindoutbeforeprescribinghormonal
contraceptives?
Itwouldbeimportanttoaskabouttobaccouse,breastfeeding,riskforsexuallytransmitteddiseases
(STDs),pastmedicalhistory,andcurrentmedicationsbecausethesefactorsmayinfluencedecisions
aboutwhichtypeofcontraceptivetoprescribe.

Case91continued:
Youfindoutshehassmokedonepackperdayforthepast10years.Herpastmedicalhistoryis
significantforhypertension,forwhichshetakesabetablocker.Shehasnopersonalhistoryof
breastorendometrialcancer,buthermotherdidhaveendometrialcancer.

2 Whatareyouconcernedaboutinthiswoman'shistoryinregardtohormonal
contraception?

Estrogencontaininghormonalcontraceptivesaloneincreasetheriskforvariousthromboticand
thromboembolicphenomena,includingstroke,myocardialinfarction,deepvenousthrombosis,and
pulmonaryembolism(PE).Inwomenwhosmoke,suchcontraceptivesincreasethisrisk
substantially(asdoesthiswoman'shypertension)andshouldthereforebeusedwithcaution.Ifthis
womanwereover35aswellasasmoker,estrogencontainingcontraceptiveswouldbe
contraindicated.

3 Whataretheabsolutecontraindicationstousingestrogencontaining
contraceptives?
Smokerolderthan35,historyofthromboembolicphenomena(PE,stroke),coronaryarterydisease,
hepatictumors(estrogenscanmakehepatictumorsgrowandrupture),andpersonalhistoryof
estrogendependentcancerssuchasendometrialcarcinomaorbreastcarcinoma,unexplained
vaginalbleeding,andimpairedliverfunction.

4 Whatdooralcontraceptivestypicallyconsistofandwhatistheirmechanism
ofaction?
Thetwogeneraltypesoforalcontraceptivearethecombinationpills(estrogenandprogesterone)
andtheprogesteroneonlypills.Inthecombinationpills,theconstantlevelofestrogensupplied
continuouslysuppressespituitarygonadotropinsecretion,therebyremovingthestimulusfor
ovulation.Theprogesteroneinthecombinationpillsservestwofunctions:first,itthickensthe
cervicalmucussecretions,essentiallymakingthevaginal/uterineenvironmentlessreceptiveto
sperm,andsecond,itopposestheproliferativeeffectsofestrogen,causingthinningoftheuterine
lining(whichisimportantinreducingtheriskofendometrialcancerfromunopposedestrogen).The
progesteroneonlypillsareonlyabout50%effectiveatinhibitingovulation,butasmentioned,they
alsoworkbythickeningthecervicalmucusandalteringthemotilityandsecretionsofthefallopian
tubes,aswellasthinningtheendometrium.
Importantinthiswoman'shistoryisthatshewishestobreastfeedherbaby.Combinationhormonal
contraceptivespostpartumcaninterferewithmilkproduction,soprescribingaprogesteroneonly
contraceptivewouldberecommended.
Othermethodsofhormonalcontraception:
Transdermalpatchandthevaginalringareotherformsoftheestrogenandprogesterone
therapy.
Injectableprogesteroneisanintramuscularformofprogesteroneinjectedevery3months.
Othertypesofcontraceptionincludetheintrauterinedevice(IUD)withorwithouthormonesbarrier
methods(condoms,diaphragm,cervicalcap)ortuballigation/vasectomy.

5 Howcanmenstrualcyclesbemaderegularbyhormonalcontraceptives?

Inorderformenstruationtooccur,theremustfirstbeestrogenicstimulationofendometrial
proliferationthenprogesteronemustinducematurationandstimulatesecretionbytheendometrial
glands.Mensesbeginfollowingthedeclineinprogesteroneandestrogenlevelsneartheendofthe
menstrualcycle.Inmanywomen,thesehormonaleventsdonotoccurinsuchawellorchestrated
manner.Consequently,theestrogenandprogesteronestimulationoftheuteruscanbeprovided
artificially,whichcanmimicthenaturalmenstrualperiod.Typically,toachievethistypeofcontrol,
estrogensaregivenwithprogesteronesfor21days,andthenplacebopillsaregivenfor7daysto
allowformenstruation.

6 Yourpatientasksiftakinghormonalcontraceptiveswillincreaseherriskfor
endometrialcancer.Howdoestakingoralcontraceptivesaffecttheriskof
endometrial,ovarian,andbreastcancer?Whatissupposedtoexplainthiseffect?
Hormonalcontraceptives(combined)actuallydecreasetheriskofovarianaswellasendometrial
cancer.Theproposedexplanationfordecreasingriskofovariancanceristhatbyinhibiting
ovulation,oralcontraceptivesreducetheinflammatoryresponseandcellproliferationthatusually
occursontheovariansurfaceaftereachfolliclerupturesthroughtheovariansurface,whichiswhat
predisposesovariansurfaceepitheliumtomalignancy.Theproposedexplanationfordecreasingthe
riskofendometrialcanceristheeffectprogesteronehasonthinningtheuterineliningand
preventingunopposedgrowthbyestrogen.
Theriskofbreastcanceriscontroversial:Studiesshoweithernoeffectoraslightincreaseinrisk.

7 Howdoovariancystsformandhowdooralcontraceptivesreducetheir
occurrence?
Themostcommonovariancystsarefunctionalcyststhatformwhenthenormalfollicular
maturationandthecorpusluteumformationprocessbecomessomewhataberrant.Thesearethe
follicularcystsandcorpusluteincysts,respectively.Follicularcystsdevelopafterfailureofamature
ovarianfollicletoruptureandbeovulated.Corpusluteincystsareformedduringthelutealphase
andoccurwhenthecorpusluteumbecomesabnormallylargeorhemorrhagic(corpus
hemorrhagicum).Clearly,theinhibitionoffollicularmaturationandovulationbyoralcontraceptives
shouldreducethechancethatthesecystswilldevelop.

8 Whymaythedrugsphenytoin,phenobarbital,andrifampinmakeoral
contraceptiveslesseffectiveatpreventingpregnancy?
ThesedrugsallinducehepaticcytochromeP450enzymes,whichcanacceleratetherateofhepatic
catabolismofestrogenandprogesteronecompounds.

Hormonalcontraceptivesincreasetheriskforthromboembolicevents.Theyare
contraindicatedforsmokersoverage35.
Hormonalcontraceptivescanbeprogesteroneonlyoracombinationofestrogenand
progesterone.

Itisprincipallythewithdrawalofprogesterone(notestrogen)thatcausesnormal
menses.
Hormonalcontraceptivesdecreaseriskofovarianandendometrialcancer.Howthey
affecttheriskofbreastcancerisunclear.
SummaryBox:OralContraceptives

Case92
A26yearoldwomanhasnothadaperiodforapproximately2months,whereasshewas
previouslyregular.Shehasalsohadseveralepisodesofnauseaandvomitingbuthasnot
otherwisefeltill.

1 Whatisthedifferentialdiagnosisforsecondaryamenorrhea?
Thedifferentialdiagnosisforsecondaryamenorrheaincludespregnancy(whichismostcommon),
tumorofovaryoradrenals,anatomicabnormalities(suchasAsherman'ssyndrome,inwhich
intrauterineadhesionsmayoccurfollowinguterinesurgery),ovarianfailureorpolycysticovary
syndrome(PCOS),hypothyroidism,hyperprolactinemia,andcentralnervoussystem(CNS)or
hypothalamicdisorder.

Case92continued:
Sheisabsolutelyandpositivelyemphaticthatshecannotbepregnant.Physicalexamination
doesnotrevealanypalpableabdominalorpelvicmassesortenderness.

2 Whatlaboratorytestsshouldyouorder?
Eventhoughthewomanisemphaticthatshecannotbepregnant,aserumhCGshouldalwaysbe
donefirsttoruleoutpregnancy.IftheHCGtestisnegative,onethencanthenproceedwithfurther
testing:thyroidstimulatinghormone(TSH),prolactinlevel,aprogesteroneonlychallengetest,
estrogenandprogesteronechallenge,FSHlevel,andLHlevel.
Ifwithdrawalbleedingispresentwiththeprogesteroneonlychallengetest,thentheamenorrheais
secondarytoanovulation(seequestion7underBasicConceptssectionforfurtherdiscussion).If
withdrawalbleedingisabsentwithprogesteronealone,butpresentwithestrogenandprogesterone,
thensuspectinadequateendogenousestrogenandevaluationofFSHandLHlevelsmighthelp
distinguishbetweenahypothalamic/pituitaryprocessandovarianfailure.Ifwithdrawalbleedingis
completelyabsent,ananatomicdisordersuchasAsherman'ssyndromemightbepresent.
Asherman'ssyndromereferstotheremovalofthestratumbasalisowingtorepeatedcurettage.
Becausethestratumbasalisservesasthestemcelllayeroftheendometrium,destructionofthislayer
preventsregenerationofthefunctionalendometrialtissue.Instead,endometrialfibrosispersists.

Case92continued:
AserumhCGisdoneandispositive.

3 Doesanelevatedhumanchorionicgonadotropinlevelalwaysindicatea
developingembryo/fetus?
Itdoesnotnecessarilymeansheispregnant,becausegestationaltrophoblastictumors(hydatiform
moles/invasivemoles/choriocarcinoma)aswellasseveralgermcelltumorsoftheovaryalso
elaboratehCG.Note,however,thattheseareallrareentities.Youshouldconfirmthepresenceofa
developingembryowithintheuterusbyultrasound.

4 Whatishumanchorionicgonadotropinandwhatisitsnormalfunction,
asidefromservingasamarkerforpregnancy?
ThehormonehCGissimilarinstructureandactivitytoLH.Itissecretedearlyinpregnancybythe
placenta(specifically,thesyncytiotrophoblastcells)andfunctionstomaintainthecorpusluteum,
whichistheprincipalsiteofovariansteroidogenesisduringthelutealphase.Productionbegins
approximately1weekafterconceptionanddoublesinquantityevery2days.Thehormoneis
detectableinthebloodby8daysafterconceptionandintheurineapproximately14daysafter
conception(aroundthetimeawomanwouldexpecthernextperiod).PlasmalevelsofhCGpeakin
thefirsttrimester(by10weeksofgestation).Then,astheplacentabeginstotakeoverasthemain
siteofmaternalestrogenandprogesteronesecretion,hCGlevelstaperoff(Fig.93(f0020)).

Figure93
Humanchorionicgonadotropinstimulatesproductionofestrogenandprogesteronebythecorpusluteum.Asthe
levelsofthishormonedrop,theplacentatakesoverasthemajorsiteofsynthesisofovariansteroids.
(FromGuytonAC,HallJ:TextbookofMedicalPhysiology,11thed.Philadelphia,WBSaunders,2007.)

5 Relativetoanormalpregnancy,howwouldthehumanchorionic
gonadotropinleveldifferforanectopicpregnancy?

Inectopicpregnancythereispoorplacentation(therearefewersyncytiotrophoblastcellstoproduce
hCG),andtherefore,theserumhCGissignificantlylowerthanwouldbeexpectedfornormal
pregnancyofthesamegestationalage.Asageneralruleofthumb,ifhCGlevelsdonotdouble
appropriatelyandreachexpectedlevels,youshouldsuspectanabnormalitywiththepregnancy.

6 Assumingthepositivehumanchorionicgonadotropintestconfirmsa
pregnancyinthiswoman,whatwastheapproximatedateofconception?
Becauseovulationoccursapproximately2weeksaftertheonsetofmenses,andherlastmenses
began9weeksago,theapproximatedateofconceptionwas7weeksago.

7 Whatisthedifferencebetweenthegestationalageandthetimesince
conception(developmentalage)?
Thegestationalageistheperiodthathaselapsedsincethefirstdayofherlastmenstrualperiod.Itis
notthetimesinceconception,amistakestudentscommonlymake.Becauseconceptiontypically
occurs2weekslater,thetimesinceconceptionis2weeksshorterthanthegestationalage.
Gestationalageismostcommonlyused:Sheis9weeksbygestation.

8 Atwhattimeduringdevelopmentistheembryo/fetusmostsusceptibleto
teratogens?
Thefetusismostsusceptibleduringthethirdtoeighthweeks(days1556,theembryonicperiod),
whenorganogenesisoccurs.Commonteratogensincludealcohol,cocaine,nicotine,excessivevitamin
A,lithium,warfarin,angiotensinconvertingenzyme(ACE)inhibitors,alkylatingagents,certain
antibiotics,andvalproicacid,tonameafew.

9 Whatisfetalalcoholsyndrome?
FetalalcoholsyndromeistheleadingcauseofcongenitalmalformationsintheUnitedStatesand
resultsfromexcessivealcoholintakeduringpregnancy.Itisassociatedwithawiderangeof
congenitaldefectsincludingmentalretardation,microcephaly,seizures,andmotordisorders,allof
whichresultfromadisruptionofneuroblastmigration.Otherdefectsincludelimbdislocation,heart
abnormalities,andfacialabnormalities(flatnasalbridge,upturnednose,railroadtrackears,
epicanthalfolds,smoothphiltrum,andsmallpalpebralfissures)(Fig.94(f0025)).

Figure94
Infantwithfetalalcoholsyndrome.Noteshortpalpebralfissures,mildptosis,appearanceofthenostrils,smooth
philtralarea,andnarrowvermilionoftheupperlip.

(FromGilbertBarnessE:Potter'sPathologyoftheFetus,InfantandChild,2nded.Philadelphia,Mosby,2007.)

10 Theriskofwhichfetaldevelopmentalabnormalitiescanbereducedbytaking
supplementalfolicacidearlyduringpregnancy?
TheriskofdevelopmentalabnormalitiesoftheCNSandthespinalcord,whichmaycauseneural
tubedefectssuchasspinabifidaandanencephaly,maybereduced.Ideally,prenatalsupplementsare
takenpriortopregnancy,becausesignificantneuraldevelopmentmayalreadyhaveoccurredbythe
timeawomanrealizessheispregnant.

11 IsthewomaninCase92(st0130)athighorlowriskforhavingababywith
Downsyndrome(trisomy21)?
TheincidenceofDownsyndromeincreasessignificantlywithmaternalage(asovaage,theyacquire
mutations),andbecausethiswomanisonly26,herriskisquitelow.Theriskincreaseswithagefrom
approximately1in1500forbabiesof16yearoldmotherstoapproximately1in25babiesfor45
yearoldmothers.
Note:Maternalserumfetoprotein(AFP)levelsareoftencheckedaroundthe16thweekof
pregnancy.Highlevelsmayindicateaneuraltubedefectsuchasspinabifidawhereaslowlevelsmay
indicateDownsyndrome.OtherlaboratoryfindingsassociatedwithDownsyndromeinclude
decreasedestriolandelevatedhCGandinhibinA.

12 Whyshouldergotalkaloids(e.g.,ergonovine),triptans(e.g.,sumatriptan),
andsyntheticprostaglandins(e.g.,misoprostol)allbestringentlyavoided
duringpregnancy?
Theseagentsallcausepowerfuluterinecontractionsthatcanresultinabortion.Theeffectsofergot
alkaloidsandtriptansareprincipallymediatedthroughserotoninreceptors.Misoprostolisa
syntheticprostaglandinthatcausescontractionsinananalogousfashiontoendogenous
prostaglandins.

Mostcommoncause:Pregnancy.Alwayscheckserumhumanchorionicgonadotropin
(hCG)levels.
hCGlevelspeakat10weeksgestation(approximately100,000mIU/mL).
EctopicpregnanciesareassociatedwithlowhCGlevels,andhCGlevelsdonot
doubleevery48hoursastheydoinnormalpregnancy.
Gestationalageisdeterminedfromthefirstdayofthelastmenstrualperiodandis
therefore2weeksearlierthanthedateofconception.
SummaryBox:SecondaryAmenorrhea

Case93
A19yearoldG1P0(gravida1,para0)womanwhois2monthspregnantbylastmenstrual
periodcomesinforanurgentvisitbecauseofheavyvaginalbleeding.

1 Whatisthedifferentialdiagnosisforfirsttrimesterbleeding(<1214weeks)?
Thedifferentialdiagnosisforfirsttrimesterbleedingincludesspontaneousabortion,ectopic
pregnancy,molarpregnancy,andpostcoitalbleeding,aswellasothernonpregnancyrelatedcauses
suchasvaginallacerationortrauma.

Case93continued:
Shehasnotbeenseeninclinic,butsheknowssheispregnantbyapositiveurinepregnancy
test.Shehashadseverenauseaandvomitingbutthinksthatisnormalmorningsickness.You
areconcernedaboutaspontaneousabortionandectopicpregnancy.Apelvicultrasound(Fig.
95(f0030))revealsasnowstormpatternandnodiscerniblefetus.SerumhCGlevelsare
elevatedfarandabovewhatwouldbeexpectedduringpregnancy.

Figure95
PelvicultrasoundfrompatientinCase93.(st0205)
(FromGabbeSG,NiebylJR,SimpsonJL:ObstetricsNormalandProblemPregnancies,4thed.
Philadelphia,ChurchillLivingstone,2002.)

2 Whatisthediagnosis?
Molarpregnancy(hydatidiformmole)islikely.Molarpregnancyisonesubtypeofgestational
trophoblasticdisease(GTD).

3 Howcanhumanchorionicgonadotropinlevelsdifferentiatebetweena
normalpregnancyandamolarpregnancy(completeorincompletemole)?
Becausethesemolesaremadeexclusivelyoftrophoblastic(placental)tissue,thesiteofhCG
synthesis,serumhCGlevelsaresubstantiallyelevatedincomparisontoanormalpregnancyof
similargestationalage.Otherfeaturesthatsuggestamolarpregnancyincludeseverevaginal
bleedingearlyinthepregnancyandvaginalpassageofmolarvesicles.

4 Whatisthedifferencebetweenaninvasivemoleandchoriocarcinomaand
fromwhatdoeseachgenerallyarise?
Aninvasivemoleinvadesthemyometriumbutdoesnotnormallymetastasize.Amajorityofthem
arisefrombenignmolarpregnancies.Choriocarcinoma,ontheotherhand,isoftenmetastaticand
spreadshematogenously.Halfofthesedevelopfrombenignmolarpregnancies,onefourthafter
normaltermpregnancy,andonefourthaftermiscarriage,abortion,orectopicpregnancy.Akey
histologicdistinctionbetweeninvasivemolesandchoriocarcinomasisthatchoriocarcinomasareless
differentiatedandlackavillouspattern.

5 CoverthecolumnsontherightsideofTable92(t0015)andtrytoidentifythe
characteristicsofthedifferentgestationaltrophoblasticdiseases
SeeTable92(t0015)andFigure96(f0035).

Molarpregnancyischaracterizedbyheavyvaginalbleedinginthefirsttrimester,
passageofgrapelikevesicles,snowstormappearanceonultrasound,andincreasein
normalhumanchorionicgonadotropin(hCG)level.
Gestationaltrophoblasticdisease(GTD)referstoabnormalproliferationofplacental
tissue:mostlybenignmolarpregnanciesandthemalignantGTD:invasivemoles,
choriocarcinoma,andplacentalsitetrophoblastictumor(PSTT).
Completemolesarethemostcommonmolarpregnancyandhaveahigherpercentageof
persistentmalignantdisease.
SummaryBox:FirstTrimesterBleedingandGestationalTrophoblasticDisease

Case94
A32yearoldG2P1(gravida2,para1)womanat12weeksgestationbylastknownmenstrual
periodcomplainsofoccasionalpalpitations,irritability,andheatintolerance.

Table92
GestationalTrophoblasticDiseases

Type

Subclassification Persistent Pathogenesis

Molar

Complete

Benign(in

pregnancy/
hyatidiform

mole(90%of general)
molar

moles

pregnancies)

Karyotype Fetal

Malignant

Parts

Disease

Present

1525%

Spermfertilizes

Diploid

emptyegg,then
duplicates

(46,XX)

Spermfertilizes

Triploid

normalegg

(69,XXY

No

(80%)
Incomplete/

Benign(almost

partialmole

always)

Verylow

Yes

or
69,XXX)
Invasive
moles(10

Malignant

Monthstoyears
aftermolar

15%)

Diploid

No

pregnancy(50%),
normalpregnancy
(25%),after
abortion,ectopic
(25%)

Choriocarcinoma(25%)

Malignant

Diploid

No

Placentalsitetrophoblastic Malignant

Diploid

No

tumor(rare)

Figure96
Patternsoffertilizationtoaccountforchromosomaloriginofcomplete(46,XX)andtriploidpartialmoles(XXY).In

acompletemole,oneortwospermfertilizeaneggthathaslostitschromosomes.Partialmolesaredueto
fertilizationofaneggbyonediploidortwohaploidsperm,depictedinthisexampleasone23,Xandone23,Y.
(FromKumarV,AbbasAK,FaustoN:RobbinsandCotranPathologicBasisofDisease,7thed.Philadelphia,WB
Saunders,2005.)

1 Whatisthedifferentialdiagnosis?
Thedifferentialdiagnosisforpalpitationsandheatintoleranceincludeshyperthyroidism,normal
pregnancy,anxiety,cardiacarrhythmia,andpheochromocytoma.

Case94continued:
Exceptforagravidabdomen,physicalexaminationisunremarkable.Cardiacauscultation
revealsaregularrateandrhythmwithoutmurmursorgallops,andanelectrocardiogram
(ECG)isnormal.

2 Whatwouldyouliketodoforfurtherworkup?
Athyroidpaneltoruleouthyperthyroidismwouldbehelpful.

Case94continued:
Theresultsrevealelevatedlevelsoftotalandfreethyroidhormones(triiodothyronine[T3]and
thyroxine[T4])aswellasreducedlevelsofTSH.

3 Whatisthediagnosis?
Theseresultssuggestgestationalhyperthyroidism.Inpregnancy,thetotalT4isusuallyelevated
becauseestrogenincreasesthesynthesisofthyroxinebindingglobulinbytheliver.Notice,however,
thatthiswomanhadelevatedfreeT4,levelsaswellanditisthiselevatedfreeT4(orfreeT3)that
causeshyperthyroidism.

4 Howdoesthepregnantstatepredisposetohyperthyroidism?
ThiseffectisduetothepresenceofhCGinthematernalcirculation.hCGisaglycoprotein
synthesizedandsecretedbytheplacentainlargeamounts.OwingtoitssimilarityinstructuretoTSH
(TSH,FSH,LH,andhCGallsharethesamesubunit),itoftenhyperstimulatesthethyroidgland
duringpregnancy,resultingingestationalhyperthyroidism.Thisconditionoftenspontaneously
resolvesfollowingdeliveryofthefetusandplacenta.

5 Howarematernallevelsoffolliclestimulatinghormoneandluteinizing
hormonelikelytobeaffectedinthiswoman,giventhatsheispregnant?
Theyshouldbelowandvirtuallyundetectablebecauseofthesignificantnegativefeedbackthatthe
highlevelsofestrogenandprogesteroneseenwithpregnancyproduceonthepituitary.

6 Whyisthedeclineinestrogenandprogesteroneafterdeliverybeneficialfor
thebeginningoflactation?
Bothestrogenandprogesteroneinhibitlactation,andtheirwithdrawalallowstheelevatedprolactin
presentatthetimeofdeliverytofacilitatemilkletdownandlactation.Recallthattheplacentaisthe
majorsiteofprogesteroneandestrogensecretion,soitsexpulsionafterdeliverywillreducethelevels
ofthesehormones.

7 Howdoesbreastfeedingactasanaturalcontraceptive?
Nipplesucklingbythebabystimulatesthereleaseofprolactinbytheanteriorpituitarygland.
BecauseoneofthefunctionsofprolactinistoinhibitthehypothalamicsecretionofGnRH,this
resultsinreducedFSHandLHsecretionbythepituitary.Reducedlevelsofthesegonadotropins
inhibitovulationinthenursingmotherandactsasanaturalbirthcontrolpill.However,
breastfeedingmustbepracticedcontinuouslyinordertobeefficaciousasacontraceptivemethod.

Inpregnancythetotalthyroxine(T4)isusuallyelevatedbecauseestrogenincreasesthe
synthesisofthyroxinebindingglobulinbytheliver.
IncreasesinfreeT4andtriiodothyronine(T3)levelsresultsinhyperthyroidism.
Highlevelsofprolactininanursingmotherinhibithypothalamicsecretionof
gonadotropinreleasinghormone(GnRH),resultinginreducedgonadotropinsecretion
andsuppressionofovulation.
SummaryBox:EndocrinologyofPregnancy

Case95
A28yearoldwomaninher33rdweekofpregnancyiscomplainingofincreasedfatigueand
swellingofherhandsandface.Urinedipsticktestingshows2+proteinuria,andherblood
pressureiselevatedat150/110mmHg(upfrom130/90mmHg).Briskdeeptendonreflexes
arealsonotedonexamination.

1 Whatisthediagnosis?
Preeclampsiaisdefinedbynewonsethypertension(>140/90mmHg)andproteinuria(>300
mg/day)occurringafter20weeksgestation.Althoughthedefinitivecauseofpreeclampsiaisstill
unknown,theunderlyingcauseisthoughttobegeneralizedarterialvasospasm.
Note:Theseverityofpreeclampsiaisgenerallydeterminedbythedegreeofproteinuriaandblood
pressureelevation.

2 Howshouldthispatientbemanaged?

Thiswomanisclassifiedashavingmildpreeclampsia.Becausesheisnotyetatterm(37weeks),bed
restandexpectantmanagementwouldbeappropriate,asonewouldnotwanttoinducepremature
labor.Shecouldalsobegivensteroidssuchasbetamethasonetoenhancefetallungmaturityinthe
eventthatprematuredeliveryisnecessary.

Case95continued:
Thenextweekshedevelopsrightupperquadrant(RUQ)pain,herproteinuriaworsens,andher
bloodpressureincreasesto170/115mmHg.

3 Whatisyourprimaryconcernatthispoint?
Thepatientnowhasseverepreeclampsia.TheRUQpainisconcerningforHELLPsyndrome,which
canbeacomplicationofseverepreeclampsia.TheHELLPsyndromeconsistsofHemolysis,E
levatedLiverenzymes,andLowPlatelets.About10%ofpatientswithseverepreeclampsiadevelop
theHELLPsyndrome.

4 Whatchangesoccurinthespiralarteriesinpatientswithpreeclampsia?
Preeclampsiaisassociatedwithmechanicalorfunctionalobstructionofthespiralarteries.The
reasonforthisismultifold:Abnormaltrophoblastictissueinvadesthespiralarteries,andpatients
demonstrateincreasedlevelsofvasoconstrictorsanddecreasedlevelsofvasodilatorsalongwith
increasedconcentrationsofgrowthfactors.Theendresultisplacentalhypoperfusionwithspiral
arteryatherosclerosis.

Case95continued:
Laboratorytestsrevealanemia,thrombocytopenia,andelevatedliverenzymes,anda
peripheralbloodsmearshowsthepresenceofschistocytes.

5 Ifthiswomandevelopsseizuresalso,howdoesthatchangethediagnosis?
Then,assumingshedoesnothaveapreexistingseizuredisorderormetabolicabnormality,shehas
eclampsia,whichisdefinedbythepresenceofseizuresinapatientwithpreeclampsiaandwithout
otherknowncausesofseizures.

6 Whatisthedefinitivetreatmentforpreeclampsia,eclampsia,andtheHELLP
syndrome?
Deliveryofthefetusistheonlydefinitivetreatment.Supportivemanagementincludesmagnesium
sulfate(MgSO4)forseizureprophylaxisineclampticpatients.Inpreeclampticpatients,MgSO4is
alsooftengivenasseizureprophylaxisduringlaboranddelivery.AlthoughMgSO4isconsideredto

bethefirstlinetreatmentforseizuresinthesepatients,diazepamcanalsobegiven.Patientsshould
beplacedonsaltrestricteddiets.Ifthebabymustbedeliveredprematurelyforthehealthofthe
mother,steroidsshouldbegiventoimprovefetallungmaturity.

7 Whyareangiotensinconvertingenzymeinhibitorsorangiotensinreceptor
blockersnotusedtotreathypertensioninpreeclampsia?
ACEinhibitorsandangiotensinreceptorblockers(ARBs)shouldnotbeusedtotreatanypregnant
womanbecausetheycarrytheriskofcausingfetalrenalfailureandevenfetaldeath.
Preeclampsiaandeclampsiaarehighyieldsubjectsforboards.Youshouldbeabletorecognize
thefindingsassociatedwiththesediseasesinpregnantpatientsandunderstandbasicconcepts
regardingtreatment.
Step1Secret

Preeclampsiaisdefinedbyhypertension(>140/90mmHg),proteinuria(>300mg/24
hours),andnondependentedemaafter20weeksgestation.
Eclampsiaisdefinedbythepresenceofpreeclampsiawithseizures.
HELLPisHemolysis,ElevatedLiverenzymes,andLowPlatelets.
Definitivetreatmentisdeliveryofthefetus.
SummaryBox:Preeclampsia,Eclampsia,andtheHellpSyndrome

Case96
A26yearoldwomanat32weeksgestationhascometothehospitalbecauseshehasbeen
havingcontractionsforthepast3hours.Contractionsarenowoccurringevery10minutes.The
diameterofhercervicalcanalis2cm.Sheistoldshemightbegoingintoprematurelabor.

1 Whatpharmacologicagentscanbeusedtosuppresslaborinthiswoman?
Agentsthatinhibituterinecontractionsareknownastocolytics.MgSO4ismostwidelyused.Other
classesofdrugsinclude2receptoragonists(usuallyterbutalineandritodrine)andcalcium
channelblockers(nifedipineismostwidelyused).Theselattertwodrugclassesaresmoothmuscle
relaxants.Additionally,indomethacinandothernonsteroidalantiinflammatorydrugs(NSAIDs)can
decreaseuterinecontractionsbyinhibitingprostaglandinsynthesis.

Note:Afterapproximately32weeksgestation,thereisconcernaboutusingNSAIDsbecauseof
theirpotentialtocauseprematureconstrictionoftheductusarteriosus.Rememberthat
prostaglandinsarevasodilatoryandthatNSAIDsinhibitprostaglandinsynthesis.

2 Whatisthemainsourceofrisktothiswoman'sbabyassociatedwith
prematuredelivery?
Theprincipalconcernwithprematuredeliveryisimmaturefetallungs,whichcancauseneonatal
respiratorydistresssyndrome.Fetallungmaturityisdeterminedbytheamountofsurfactant
present,whichcanbeassessedwithamniocentesisandevaluationofthelecithinsphingomyelin
ratio,whichshouldbegreaterthan2formaturelungs.Glucocorticoidscanbegiventoawomanin
prematurelabortoincreasetheproductionofsurfactant.Typically,surfactantproductionbeginsby
28weeksandiscompleteby36weeks.

RelatedQuestionsonLaborandDelivery
3 Ifplacentapreviawerepresentatterm(orwhendeliveryisnecessary),why
wouldacesareansectionbemandatory?
Placentapreviaoccurswhentheplacentacoverstheinternalcervicalos.Withavaginaldelivery,the
placentawouldhavetoruptureforthebabytopassthroughthecervix(ahorrifyingbloodymess).
Note:Placentapreviacanalsopresentaspainlessbleedingduringanytrimester.Placental
abruption,whichoccurswhentheplacentalosesitsattachmenttotheuterus,isverypainfuland
presentsonlyinthethirdtrimester.

4 Howcanplacentaaccretacomplicatethelaboranddeliveryprocess?
Placentaaccretaoccurswhentheplacentahasinvadedintoandattachedfirmlytothemyometrium.
Inthissituation,theplacentadoesnotseparateofftheendometrialliningafterdeliveryoftheinfant.

5 Whatisoxytocinandhowisitusedtoaugmentorinducelabor?
Oxytocinisapeptidehormoneproducednaturallybytheposteriorpituitaryandisastimulantfor
uterinecontractions.Exogenousoxytocin(pitocin)enhancesuterinecontractionsandaccelerates
thefirststageoflabor.
Note:Duringpregnancy,thenumberofoxytocinreceptorsontheuterusincreases,whichmakesthe
uterusparticularlysensitivetoendogenousorexogenousoxytocinattheendofterm.

6 Whatpharmacologicagentscouldbeusedifdeliveryiscomplicatedby
postpartumhemorrhage?
Severaldifferentpharmacologicagents,includingtheergotalkaloids,oxytocin,andcertain
prostaglandins,allcauseuterinecontractions,whichreducepostpartumbleedingbyclampingdown
onbleedingvessels.

Note:Ischemicnecrosisoftheanteriorpituitarygland,knownasSheehan'ssyndrome,isa
potentialcomplicationofseverepostpartumhemorrhage.Itresultsfromtheenlargementofthe
pituitaryduringpregnancythatoccurswithoutaproportionalincreaseinvascularsupply.This
substantiallyincreasestheriskofinfarction,especiallyafterdeliverywhenseverebleedingand
hypoperfusionarelikelytotakeplace.

Tocolytics(usedtostopcontractionsofprematurelabor):magnesiumsulfate,2
agonists,calciumchannelblockers,andnonsteroidalantiinflammatorydrugs(NSAIDs).
Pitocin(exogenousoxytocin):usedtostimulateoraugmentcontractions.
NSAIDsshouldnotbeusedafter32weeksbecauseoftheriskofprematureclosureof
ductusarteriosus.
Surfactantproductionbeginsby28weeksandiscompleteby36weeks.
Placentaprevia:Placentacoverstheinternalcervicalos.Canpresentaspainless
bleedinginanytrimester.
Placentalabruption:Placentaseparatesfromwallofuterus.Presentsaspainfuluterine
bleedingduringthethirdtrimester(>2428weeks).
Placentaaccreta:Placentainvadesintomyometriummaycausehemorrhageafter
delivery.
Sheehan'ssyndromenecrosisoftheanteriorpituitaryglandisacomplicationof
postpartumhemorrhage.
SummaryBox:LaborandDelivery

Case97
A26yearoldwomancomplainsoffeverandpelvicpain,neitherofwhichisrelatedtoher
menstrualcycle.

1 Whatisthedifferentialdiagnosisforpelvicpain?
Thereisaverybroaddifferentialdiagnosisforpelvicpain:
Gynecologiccauses:
Uterinedisease
Endometriosis
Adenomyosis

Leiomyomata(fibroids)
IUD,polyps
Extrauterinediseases
Adhesions
Pelvicinflammatorydisease(PID)
Ovariancysts
Abscess
Urologiccauses:
Chronicurinarytractinfections(UTIs)
Detrusoroveractivity
Interstitialcystitis
Stone
Gastrointestinal(GI)causes:
Chronicappendicitis
Constipation
Diverticulardisease
Irritableboweldisease
Irritablebowelsyndrome
Malignancy
Musculoskeletalcauses:
Coccydynia
Diskproblems
Degenerativejointdisease(DJD)
Lowbackpain
Levatoranisyndrome(spasmofpelvicfloor)
Nerveentrapment
Osteoporosis

Psychiatriccauses:
Trauma/abuse
Sexuallytransmitteddisease
PID

Case97continued:
Shehasmultiplesexualpartnersandrarelyusesanyformofbarriercontraception.Cervical
examinationissignificantforbilateraladnexaltendernessandapurulentcervicaldischarge.

2 Basedontheprecedingadditionalinformation,whatisthemostlikely
diagnosis?
PIDsecondarytoinfectionwithChlamydiatrachomatis.Neisseriagonorrhoeaeisanothercommon
causeofPID.PIDtypicallypresentswithfever,lowerabdominalpain,abnormaluterinebleeding,
vaginaldischarge,andcervicalmotiontenderness.

Case97continued:
Laboratorytestsrevealamildleukocytosisandaslightlyelevatederythrocytesedimentation
rate(ESR).AquantitativehCGisnegative,butacervicalsmearispositiveforChlamydia,
confirmingthediagnosis.

3 Whatlongtermcomplicationsmaypossiblybepreventedbytreatingthis
woman?
Tubalstricturescandevelopbecauseoftheinflammatoryprocess,whichcancauseinfertilityor
ectopicpregnancy.Tubescanalsofillwithpus,leadingtohydrosalpinx.Adhesionsbetweensmall
bowelandpelvicstructurescanalsodevelop,causingsymptomsofbowelobstruction.Anabscesscan
formaroundthetubesandovaries(tuboovarianabscess).Ruptureofatuboovarianabscesscanbe
alifethreateningevent.AnotherpotentialcomplicationofPIDisFitzHughCurtissyndrome,in
whichtheinfectionspreadstotheperitoneumandcausesscartissueformationonthesurfaceofthe
liver.ThismanifestsinthesymptomofRUQpain.
AlthoughtreatmentofPIDcannoteliminatethesecomplications,itcanpotentiallyreducetheir
frequency.

4 Howshouldshebetreated?
Sheshouldbegivenantibiotics.C.trachomatis,anobligateintracellularparasite,istypicallytreated
withtheantibioticdoxycycline.However,endocervicalculturewilloftenrevealapolymicrobial
infection,necessitatingtheadditionaluseofabroadspectrumantibioticsuchasceftriaxone(athird
generationcephalosporin).Additionally,N.gonorrhoeaeisgenerallytreatedempiricallyinsomeone
withchlamydialinfection(itissusceptibletocephalosporins).

5 Ifsomeonepresentedwithsimilarsignsandsymptomsbutalsohadacute
onsetofrightkneepainandswellingwithoutanyrecenttraumatothejoint,
whatinfectingorganismshouldyoususpect?
N.gonorrhoeae,agramnegativeintracellulardiplococcus,shouldbesuspected.Inadditionto
beingacommoncauseofPID,N.gonorrhoeaecanalsocauseasepticarthritisifitdisseminates.

6 Isitsensibletorecommendtheuseofanintrauterinedevicetothiswoman?
No.IUDsarespecificallycontraindicatedinwomenwhohavehadpreviousepisodesofPIDor
multiplesexualpartnersbecausetheymayincreasetheriskofthesubsequentdevelopmentofPID.
Aswithmostimplanteddevices,thesedevicesmakeiteasierforbacteriatocolonizeandcausean
infection.
Note:YoushouldsuspectinfectionwithActinomycesisraeliiinawomanusinganIUDwho
presentswithsymptomsofPID.Thisbacteriumcanbetreatedwithpenicillin.

Pelvicinflammatorydisease(PID)canbesecondarytoinfectionwithChlamydia
trachomatis.NeisseriagonorrhoeaeisanothercommoncauseofPID.
PIDischaracterizedbypurulentdischargefromcervicalos,cervicalmotiontenderness,
andcervicalsmearpositiveforchlamydialinfectionorgonorrhea.
Treatmentiswithantibioticsdoxycyclineorceftriaxone.
Treatmentreducescomplicationsoftuboovarianabscess,infertility,andectopic
pregnancy.
SummaryBox:PelvicInflammatoryDisease

RelatedQuestiononGynecologicInfections
7 WhatbacteriumisresponsibleformaintainingthenormalacidicpHofthe
vagina?
Lactobacillusacidophilus(acidloving)maintainsthevaginalpH<4.5(Table93(t0020)).

Case98
A28yearoldwomanwhohasneverbeenpregnantwithnohistoryofpriorsurgeries
complainsofchronicpelvicpain.

Table93
InfectiousCausesofVaginalDischarge

Feature

BacterialVaginosis

Trichomoniasis

Candidiasis

Chief

Malodorousdischarge

Thinyellowishgreenish

Whitecheesy

complaint

frothydischargestrawberry exudate
cervix

Pathogenesis Overgrowthofnormalvaginalflora

itching

Sexuallytransmitteddisease Yeast
infection

Tests

Salinepreparationshowscluecells

Motileprotozoaonsaline

*(tf0010)

preparationsmears(tf0015) hyphae
pH67
pH45

KOHtestproducesafishyodor

KOHshows

pH56

Cluecellsarevaginalepithelialcells
thatarestuddedwithadherent
bacteria
Treatment

Metronidazole

Metronidazole

Nystatinor
fluconazole

*FromHolmesKK:Lowergenitaltractinfectionsinwomen:cystitis/urethritis,vulvovaginitis,and
cervicitis.InHolmesKK,MardhPA,SparlingPF,etal(eds):SexuallyTransmittedDiseases.New
York,McGrawHill,1984.CopyrightMcGrawHill,Inc.UsedbypermissionofMcGrawHillBook
Company.
FromKaufmanRH,FaroS,BrownD:BenignDiseasesoftheVulvaandVagina,5thed,St.Louis,
Mosby,2004.

1 Whatisthedifferentialdiagnosisforpelvicpain?
Thebroaddifferentialdiagnosisforpelvicpainwasdiscussedinthepreviouscase:
Gynecologiccauses:
Endometriosis
Adenomyosisleiomyomata(fibroids)
IUD
Polyps

Adhesions
PID
Ovariancysts
Abscess
Urologiccauses
ChronicUTIs
Detrusoroveractivity
Interstitialcystitis
stone
GIcauses:
Chronicappendicitis
Constipation
Diverticulardisease
IBD
IBS
Malignancy
Musculoskeletalcauses:
Coccydynia
Diskproblems
DJD
Lowbackpain
Levatoranisyndrome(spasmofpelvicfloor)
Nerveentrapment
Osteoporosis
Othercauses:
Psychiatricdisorders
Abuse

Thepainwouldneedtobecharacterizedtonarrowthedifferentialdiagnosis.

Case98continued:
Thepainisparticularlysevereduringhermenstrualperiod(dysmenorrhea).Shealso
complainsofsignificantpainduringsexualintercourse(dyspareunia).Pelvicexaminationis
significantforslightadnexaltenderness.StainsforN.gonorrhoeaeandC.trachomatisare
negative,andahCGisalsonegative.Apelvicultrasounddoesnotrevealanycysts,fibroids,
orstructuressuggestiveofovarianneoplasm.

2 Nowwhatisthelikelydiagnosis,whatisitsetiology,andhowdoweconfirm?
Themostlikelydiagnosisisendometriosis,giventhehistoryofseverepainduringthemenstrual
periodandnegativeworkupforinfectiousandanatomiccauses.Endometriosisiscausedbythe
presenceofendometrialtissueinectopic(extrauterine)locations,suchastheovariesoruterine
ligaments.Perhapsthemostwidelyacceptedtheorytoexplainthepresenceofendometrialtissuein
extrauterinesitesisthephenomenonofretrogrademenstruationthroughthefallopiantubes,a
processthatisthoughttooccurinmostwomen.Unfortunately,suchretrogradeflowdoesnot
completelyexplainthepresenceofectopicendometrialtissuesindistantanatomicsitessuchasthe
pleuralcavity.Anexploratorylaparoscopyfordirectvisualizationmighthelpconfirmthediagnosis.

Case98continued:
Laparoscopyisperformedandrevealsthepresenceofchocolatecystsonbothovaries.

3 Whatdoesthepresenceoftheseovarianchocolatecystsindicateandwhydo
theyappearblack?
Chocolatecystsarenonfunctionalovariancysts(endometriomas)thatdevelopfromectopic
endometrialtissuepresentinadvancedendometriosis.Theyappearblackbecausetheycontaina
bloodfilledcavity.

4 Whyispainworseduringthemenstrualperiodinthispatient?
Ectopicendometrialtissueundergoesthesamecycleofproliferationandbreakdownasthenormal
endometriallininginresponsetoestrogenandprogesterone.Theresultingbleedingcauses
inflammationandpain.Overthelongterm,thisinflammationcanleadtotissuedamage,fibrosis,
adhesions,andcompressionofadjacentstructures,resultinginsignsandsymptomssuchaschronic
pelvicpainandinfertility.Becauseoftheseeffects,itisimportanttodiagnoseandtreat
endometriosisearlyinitsdevelopment.
Note:Althoughectopicendometrialtissueismostfrequentlyfoundinthepelvis,itcanalsobe
foundinvariousotheranatomicsites,suchastheupperabdomenorthoraxandthecolon,leadingto
rectalbleeding.Thesesitescanalsobecomepainfulduringmenstrualcycling.Ifectopictissue

depositsinthefallopiantubes,itcanresultininfertility.
Notethatendometriosisresultsincyclicbleedingbecausetheectopictissueisalsogovernedby
hormonalregulation.Ingeneral,itwillbehelpfulforyoutoclassifygynecologicdiseases
accordingtothesetypesofpatterns(e.g.,doesthediseaseresultincyclicor
noncyclic/anovulatorybleeding,isthediseaseassociatedwithmenstrualpain).
Step1Secret

5 Whatisfirstlinetreatmentina28yearoldwoman?
NSAIDsorhormonalcontraceptivesaregoodfirstlineagentsformildendometriosis.Thechoicewill
dependonthesymptomsandwhethersheistryingtoconceive.

6 Whatisthemechanismofactionofleuprolide,agonadotropinreleasing
hormoneanalog,intreatingendometriosis?
Becausethepituitaryglandnormallyreleasesgonadotropinsinresponsetothepulsatilesecretionof
GnRHfromthehypothalamus,thecontinualpresenceofleuprolideinhibitsthepituitaryreleaseof
LHandFSH.SuppressionofFSHandLHsecretioneliminatestheirstimulationofestrogenand
progesteroneproduction,essentiallyputtingthewomaninanartificialstateofmenopause.Without
theestrogenicstimulationoftheendometrialtissueforproliferationandtheprogesterone
stimulationformaturationandeventualmenses,thereisnocyclingwithassociatedbleedingof
ectopicendometrialtissue.
Note1:Becauseleuprolidecausesanartificialstateofmenopause,itisusedforonlybriefperiodsof
timebecauseoftherisksofosteoporosis,hotflashes,andotherpostmenopausalproblems.
Note2:Danazol,aderivativeoftestosterone,whichnonethelesshassomeprogestationalactions,is
alsousedtotreatendometriosis.ItdecreasespituitaryFSHandLHsecretionbuthassome
unpleasantsideeffects(hirsutism,deepeningofthevoice)duetoitsandrogenicactions.

7 Anotheroptionfortreatingthiswomanistotalabdominalhysterectomywith
bilateralsalpingooophorectomy.Whatisthevalueofexcisingtheovaries,in
termsoftreatingendometriosis,iftherearenoendometrialimplantsonthe
ovaries?
Theestrogensproducedbytheovariesareresponsibleforstimulatingthecyclingofanyectopic
endometrialtissuethatisnotremovedwithhysterectomy.

8 Anothercauseofdysmenorrheaisadenomyosis.Whatisthis?
Adenomyosisisingrowthoftheendometrialglandsintothemyometrium.Inadditionto
dysmenorrhea,itcanalsocauseheavymenstrualbleeding(menorrhagia).

Endometriosisiscausedbythepresenceofendometrialtissueinectopic(extrauterine)
locations,suchastheovariesoruterineligaments.
Theconditionischaracterizedbyseverepainwithmenstrualcycleandchocolatecysts
onovaries.
Treatmentcanbewithnonsteroidalantiinflammatorydrugs(NSAIDs),hormonal
contraceptives,agonadotropinreleasinghormone(GnRH)agonist(leuprolide),an
androgenderivative(danazol),orsurgery.
Adenomyosisisgrowthofendometrialglandsintomyometriumandcancauseheavy
menstrualbleeding.
SummaryBox:Endometriosis

Case99
A37yearoldAfricanAmericanwomanisbeingevaluatedforabnormaluterinebleeding.She
hasregularperiods,buttheyareheavy,andshealsohassomebleedingbetweenherperiods.
Intercoursehasbecomesomewhatpainful.

1 Whatisthedifferentialdiagnosisforabnormaluterinebleedingina
premenopausalwoman?
Thedifferentialdiagnosisforabnormaluterinebleedinginapremenopausalwomanincludes
adenomyosis(invasionoftheendometriumintothemyometrium),endometriosis,endometrial
polyps,uterinefibroids(leiomyomata),endometrialhyperplasia,andendometrialcancer.

Case99continued:
Physicalexaminationrevealstheuterustobeenlargedandhardened,possiblywithnodules.A
pelvicultrasoundrevealsanenlargeduteruswithseveraltumorousgrowthswithinthe
myometrium.

2 Whatistheprobablediagnosisnow?
Leiomyomata(singular,leiomyoma),commonlyreferredtoasuterinefibroids,ismostlikely.These
tumorsarebenign,localproliferationsofsmoothmusclecellsoftheuterusthatoccurinwhorled
patterns,andarethemostcommonofalltumortypesinfemales.Uterinefibroidsarethemost
commongynecologictumor,andAfricanAmericanwomenhaveasignificantlygreaterriskof
developingthem.AsshowninFigure97(f0040),uterinefibroidsmaybesubserous,intramural,or
submucosaldependingontheirlocation.Submucosalfibroidsareacommoncauseofuterine
bleeding.

Figure97
Histopathologyofleiomyoma.Uterinefibroidsaredesignatedsubserous,intramural,orsubmucosaldepending
ontheirlocation.Submucosalfibroidsoftencauseabnormaluterinebleeding.
(FromSabistonD:TextbookofSurgery,16thed.Philadelphia,WBSaunders,2000.)

3 Whatarethecommonsymptomsassociatedwithleiomyoma?
Althoughtheconditionisasymptomaticinasizableproportionofpatients,othersmayexperience
menorrhagia(ifthelocationissubmucosal)becausethesmoothmusclecellscannotproperlyclamp
downonthespiralarteriesduringmenses.Othersymptomsincludeobstructivedelivery,cramping
withmensessecondarytocontractionofthesmoothmusclecells,constipationsecondarytopressure
onthecolon,andfrequencyandurgencysecondarytopressureonthebladder.

4 Howdooralcontraceptivesaffectuterinefibroids?
Becausetheseareestrogensensitivetumors,exogenousestrogensinoralcontraceptivesmaymake
themgrow.Additionally,thesubstantialincreaseinestrogenthatoccursduringpregnancycanmake
thesetumorsgrowtosignificantproportions.Aftermenopause,whenestrogenlevelsfalloff,these
tumorsoftenshrinkinsizeandbecomeasymptomatic.
Note:Pharmacologicagentsthatdecreaseplasmaestrogensuchasleuprolide,danazol,and
progesteronecanallbeusedtoshrinkuterinefibroids.

5 Mostfibroidsareasymptomatic,butwhenistreatmentwithhysterectomyor
myomectomyindicated?
Ifthereisseverepain,rapidgrowth,verylargeormanyleiomyomata,orurinarysymptoms,
hysterectomyisindicated.Recurrentmiscarriageisalsoararecomplicationoffibroids,soiffuture
fertilityisdesired,myomectomycanbeperformedtoresectjustthetumor,leavingaviableuterus.

6 Whatisaleiomyosarcoma?

Leiomyosarcomaisahighlyaggressive,malignanttumorderivedfromsmoothmusclecells(ofthe
uterus).Itisnotbelievedtoarisefromuterinefibroids.Leiomyosarcomasareveryrare,butwouldbe
consideredifthereisveryrapidgrowthofamassintheuterus.Incidenceishigherinblackwomen
thaninwhitewomen.

Uterinefibroids(leiomyomata)arebenignlocalproliferationsofsmoothmusclecellsof
theuterus,responsivetoestrogen.
TheyarethreetimesmorecommoninAfricanAmericanwomen.
Fibroidsmaybeasymptomatic,butpatientsoftenexperiencemenorrhagia,obstructive
delivery,cramping,constipation,andfrequency/urgency.
Mostfibroidsdonotrequiretreatment.Drugssuchasleuprolideanddanazoldecrease
theamountofcirculatingestrogenandcanshrinkfibroids.
Hysterectomyisdefinitivetreatment.Myomectomyisindicatediffuturefertilityis
desired.
Leiomyosarcomaisararemalignanttumorofsmoothmusclecellsthatdoesnotarise
frombenignleiomyomata.
SummaryBox:AbnormalUterineBleeding,Fibroids

Case910
A30yearoldwomanpresentstothefreehealthcarecliniccomplainingofpostcoitalbleeding
forseveralmonths.Sheisotherwisehealthyanddeniesanypainwithintercourse
(dyspareunia)orothergynecologicsymptoms.

1 Postcoitalbleedinginawomanwhodoesnotseeadoctorregularlyisared
flagformalignancy.Whatriskfactorsforcervicalcancerdoyouwanttoask
herabout?
Multiplesexualpartners
Ageatonsetofsexualactivity
Tobaccouse
Note:Inuterodiethylstilbestrol(DES)exposureofafetusisariskfactorforthedevelopmentofa
rarecancertypecalledcervicalorvaginalclearcellcarcinoma.

Case910continued:

Sheadmitstobecomingsexuallyactiveatanearlyageandhavingmultiplemalesexual
partners.Shehasa15yearhistoryofcigarettesmoking.Apelvicexaminationisperformedand
revealsanexophyticgrowthonthecervix.Colposcopy(lightpoweredmagnificationofthe
cervix)withcervicalbiopsyisperformed,revealingstageIcervicalcancer.

2 Cervicalcancerismostcommonlywhattypeofcancer?
Squamouscellcarcinomamakesup90%ofcervicalcancers.Adenocarcinomaaccountsfor10%.

3 Withwhatvirushasshelikelybeeninfected?
Sheismostlikelyinfectedwithhumanpapillomavirus(HPV),whichisbelievedtocausethevast
majorityofcervicalcancers.Subtypes16,18,31,and45andmanyothersareassociatedwithcervical
cancer.
Note:HPVtypes6and11areassociatedwithcondylomaaccuminatum(genitalwart)butcuriously
don'tincreasetheriskofcervicalcancer.Theothergenitalwart,condylomalatum,isdueto
secondarysyphilis.

4 WhatisthemechanismbywhichthehumanpapillomavirusviralproteinsE6
andE7predisposetothedevelopmentofcervicalcancer?
Theyinterferewithfunctioningofthetumorsuppressorproteinsp53andretinoblastoma(Rb).
Specifically,theE6proteinbindsp53andincreasesitsrateofproteolysis,ineffectreducinglevelsof
p53.TheE7proteinpreventstranscriptionoftheRbgenebybindinganddisplacingbound
transcriptionfactorsthatarenecessaryforRbtranscription.

5 Whatdoescervicalintraepithelialneoplasia(CIN)referto?Differentiate
amongCINI,CINII,andCINIII
Thesediagnosesaremadeonlybycervicalbiopsy(colposcopywithbiopsy).CINIcorrespondsto
dysplasiaofonethirdorlessofthedepthoftheepithelium.CINIIcorrespondstodysplasiaoftwo
thirdsoftheepithelium,andCINIIIreferstodysplasiaoftheentireepitheliallayer,whichisalso
knownascarcinomainsitu.CINI,II,andIIIarealsoreferredtoasmild,moderate,andsevere
dysplasia,respectively.Ittypicallytakesabout7yearsforCINItoevolveintocervicalcancerand4
yearsforCINIItoevolveintocervicalcancer.

6 Howcancervicalcancercauserenalfailureinthiswoman?
Thecancercangrowandobstructtheureters(obstructivenephropathy).Thisisoneofthemost
commoncausesofdeathfromcervicalcancer.

7 Whymightthiswomanhavebenefitedfromthehumanpapillomavirus
vaccineorannualPap(Papanicolaou)smears?

HPVvaccineagainstserotypes6,11,16,and18wasapprovedin2006foruseamongfemalesaged9
to26yearsforpreventionofHPVrelatedcervicalcancercervical,vaginalandvulvarcancer
precursorsandanogenitalwarts.Approximately70%ofcervicalcancersworldwidearecausedby
types16and18,butbecausethevaccinedoesnotcoveralltheserotypesthatcanleadtocancer,
regularPapsmearsarestillrecommended.
ThePapsmearhasbeenshowntoeffectivelydetectpreinvasiveandearlycancerouscervicallesions
afterHPVinfection.Recallthatcervicalcancerscanhaveaprolongedlatentperiod,soearly
detectionviaPapsmearsandexcisionofpremalignantlesionshavebeenabletodrasticallyreduce
mortalityriskassociatedwithcervicalcancer.
Note:PerhapsthelargestriskfactorfordevelopingcervicalcancerissimplythefailuretohavePap
smearsonaregularbasis.

8 WherearecellssampledfromduringaPapsmear?
Cellsaretakenfromthesquamocolumnarjunction,thesocalledtransformationzone,where
columnarcervicalcellsmeetstratifiedsquamousvaginalepithelium.Papsmearscannotdiagnose
canceratissuebiopsyisrequiredforthis.

Cervicalcancermostcommonlyissquamouscellcarcinoma.
Cervicalcancerriskfactorsincludeearlyintercourse,multiplepartners,highrisk
partners,lowsocioeconomicstatus,sexuallytransmitteddiseases,tobaccouse,
diethylstilbestrol(DES)exposure.
Humanpapillomavirus(HPV)vaccine,approvedforwomenages11to26,iseffective
againstserotypes6,11,16,and18.
Papanicolaou(Pap)smearshavebeenshowntoreducemortalityriskassociatedwith
cervicalcancer.
SummaryBox:CervicalCancer

Case911
A49yearoldasymptomaticwomanseenforaroutineexaminationisnotedtohaveenlarged
ovariesbilaterallyonpelvicexamination.

1 Whatisthedifferentialdiagnosisforanovarianmass?
PCOS,multiplecystsbilaterally
Ovariancyst(functionalfollicularandcorpusluteumcystchocolatecysts)

Primaryovariantumor
Epithelialmass(benignormalignant)
Germcelltumor(benignteratoma=dermoidcystyolksacchoriocarcinoma)
Sexcordstroma
MetastasiscommonlyfrombreastorGItract(Krukenbergtumor)

2 Whatispolycysticovarysyndrome?
PCOSoccursasaresultofincreasedpituitaryproductionofLHoutofproportiontoFSH(LH/FSH
ratio>2)andhyperandrogenismsecondarytounregulatedsynthesisbythecacells.
Hyperandrogenismcanresultinhirsutismandvirilization(malesecondarysexcharacteristicsand
clitoromegaly),butandrogensarealsoaromatizedtoestrogeninadiposetissue,thusincreasingthe
riskofendometrialcarcinoma.PCOSisassociatedwithanovulation,obesity,andinsulinresistance.
Theconditionisoftendiagnosedatayoungerage,assymptomsgenerallyappearatmenarche.
Polycysticovarieshavemultiple2to8mmsubcapsularcyststhatcanbevisualizedonultrasound(
Fig.98(f0045)).Thesecystsresultinamenorrheaandinfertility.

Expecttoseeaquestiononpolycysticovarysyndrome(PCOS)ontheUSMLEStep1.
Step1Secret

Case911continued:
Pelvicultrasoundshowsa10cmcysticandsolidmasswithseptationsontheleftanda6cm
massontheright.Giventheseultrasoundcharacteristics(>8cmsolidseptations),itis
thoughttobehighlysuspiciousformalignancy.

Figure98

Sagittalsectionofapolycysticovaryillustratingalargenumberoffollicularcystsandathickenedstroma.
(FromStencheverMA:ComprehensiveGynecology,4thed.St.Louis,Mosby,2001.)

3 Whatisthemostlikelydiagnosisinthispatient?
Although80%ofovariantumorsarebenign,ageisariskfactorformalignancy.Germcelltumorsare
commoninwomen<20yearsofage,whereasepithelialtumorsaremorecommoninthose>
20years.

4 Whatismostcommontypeofovariancancer?
Serouscystadenocarcinoma,anepithelialcelltumor,ismostcommon.

5 Isovariancancerhereditary?
About90%ofovariancancersaresporadic10%haveafamilialsyndrome.Hereditarynonpolyposis
colorectalcancer(HNPCC)LynchIIsyndromeisHNPCCsyndromeinwhichthereisahighrateof
familialbreast,ovarian,colon,andendometrialcancers.Breastcancers(BRCA1andBRCA2
mutations)arealsoimplicatedinfamilialovariancancer.

6 Whatarethemajorriskfactorsforovariancancer?
Familyhistoryofovariancancer
Longperiodsofuninterruptedovulation(lowparity)
Historyofcolonorbreastcancer
Remember:Hormonalcontraceptivesdecreaseriskofovariancancerbecausetheysuppress
ovulation.

7 Arethereanyeffectivescreeningtestsforovariancancer?
Notreally.Cancerantigen125(CA125)isatumormarkerinsomeovariancancersandisusedto
followresponsetotreatment,butitisnotanappropriatescreeningtoolforthegeneralpopulation
becauseCA125isnotspecificforovariancancer.Routinepelvicexaminationsarethebestscreening
toolwehaveforovariancancer.
Germcelltumorsoftenareassociatedwithelevatedtumormarkers(AFP,hCG,llactate
dehydrogenase),whichcanbehelpfulindiagnosisandresponsetotreatment.AFPiselevatedin
embryonicsinusandyolksactumorshCGiselevatedinchoriocarcinoma.
Dermoidcystssometimesproducethyroidhormoneiftheycontainthyroidtissue,calledstruma
ovarii.
Granulosacelltumorssecreteestrogenandcancauseprecociouspuberty,endometrialhyperplasia,
orcarcinoma.TheyarecharacterizedbyCallExnerbodies,whichareeosinophilicsecretions.

8 Whatistheprognosisforovariancancer?

Whatistheprognosisforovariancancer?
Notgreat.The5yearsurvivalrateforovariancarcinoma25%to30%,typicallybecausethediseaseis
asymptomaticuntiladvancedstages.

Polycysticovarysyndrome(PCOS)ischaracterizedbyanovulation,hyperandrogenism,
multiplecystsontheovaries,andelevatedluteinizinghormone(LH).
Krukenbergtumorisametastasistotheovaryfromanothersource:commonly
gastrointestinaltractorbreast.
Hormonalcontraceptivesdecreasetheriskofovariancancer.
Cancerantigen125(CA125)iselevatedinmanyovariancancersbutisNOTascreening
markerforovariancancer.
SummaryBox:OvarianCysts,OvarianCancer

Case912
A58yearoldpostmenopausalwomanpresentswithabnormaluterine/vaginalbleeding.She
hasbeenpostmenopausalfor6yearsandhasneverexperiencedthisproblempreviously.

1 Whatisgenerallyassumedtobethecauseofpostmenopausalbleedinguntil
provenotherwise?
Untilprovedotherwise,postmenopausalbleedingindicatesendometrialcancer,whichisthemost
commongynecologiccancerintheUnitedStates.

2 Whatelseshouldbeconsideredinthedifferentialdiagnosisfor
postmenopausalbleeding?
Otheruterinesourcesofbleedingsuchasendometrialhyperplasiaaswellasvaginalsourcesof
bleedingsuchasvaginalatrophy/lacerationorcervicalpolypswouldbeconsidered.
Note:Endometrialhyperplasia,whichalsocommonlypresentsaspostmenopausalbleeding,isa
precursortoendometrialcancer.

Case912continued:
Aphysicalexaminationrevealsthesourceofthebleedingtobethecervicalos(i.e.,uterine
bleeding,notavaginalsource).Therearenoabnormalmassespalpatedonbimanual
examination,andtheuterusisofnormalsizeandshape.

3 Whatstudiescanbedonetohelpwiththediagnosis?

Pelvicultrasoundwillshowtheanatomyfibroids,polyps,orendometrialhyperplasia.Normally,a
postmenopausalwomanshouldhaveaverythinendometrialstripebecauseoflackofestrogen.

Case912continued:
Apelvicultrasoundrevealsathickenedendometrium,soanendometrialbiopsyisdone.The
pathologyreportindicatesmalignancy.

4 Whatisthemostcommontypeofendometrialcancer?
Adenocarcinoma(80%)isthemostcommontype.

5 Whydoesendometrialcancer,oncedetected,generallyhaveamuchbetter
prognosisthannewlydiagnosedovariancancer?
Endometrialcancerisusuallydetectedatamuchearlierstagethanovariancancerthankstothefact
thatabnormaluterinebleedingisanearlywarningsign.Incontrast,therearefewwarningsignsof
earlyovariancancer.

6 Whataretheriskfactorsforendometrialcarcinoma?
Abnormallyincreasedestrogenlevels(e.g.,PCOS,granulosacelltumor)oruseofestrogenwithout
progesterone(e.g.,hormonereplacementtherapy),obesity(adiposetissuecanconvertandrogens
intoestrogens),hypertension,diabetes,earlymenarche,latemenopause,andLynchsyndromeareall
riskfactors.

7 Inpathophysiologicterms,whydoesitmakesensethattheriskfactorsfor
endometrialcanceraresimilartothoseforbreastcancer?
Becauseestrogenplaysanimportantroleintheetiologyofbothtypesofcancer.Thisiswhyobesity
(increasedperipheralproductionofestrogen),unopposedestrogens,nulliparity(increasedexposure
tocyclingestrogens),andlatemenopause(increasedestrogenexposure)arecommonriskfactorsfor
thedevelopmentofbreastcancerandendometrialcancer.
Remember:Hormonalcontraceptivesactuallydecreaseriskofendometrialcancerbecause
progesteronethinstheuterineliningandpreventsunopposedgrowthbyestrogen.

8 Ifthiswomanalsohasatrophicvaginitis,whyshouldvaginalestrogencreams
notbeusedtotreatit?
Asignificantpercentageofthecreamgetsabsorbedsystemically,andbecauseendometrialcanceris
anestrogendependentcancer,theresultingelevatedplasmaestrogencancausethecancerto
progress.
Remember:Estrogendependenttumors/cancersareanabsolutecontraindicationtotheuseof
exogenousestrogens.

Endometrialcarcinomaisthemostcommon,andmostcurable,gynecologiccancerin
theUnitedStates.
Postmenopausalbleedingshouldbeconsideredendometrialcanceruntilproven
otherwise.
Riskfactorsforendometrialcancerincludeobesity,unopposedestrogens,nulliparity,
latemenopause,diabetes,andhypertension.
Hormonalcontraceptivesdecreasetheriskofendometrialcancer.
SummaryBox:PostmenopausalBleedingandEndometrialCancer

Case913
A22yearoldwomanisbeingevaluatedforabreastmassshedetectedwhileshowering.She
hasneverhadamammogramandiswonderingifsheshouldgetone.

1 Whatisthedifferentialdiagnosisforabreastmass?
Fibrocysticdisease
Benign:fibroadenoma,cystosarcomaphyllodes,intraductalpapilloma
Malignantcarcinoma

2 Whatcharacteristicsofabreastmassaremoreconsistentwithbenignversus
malignantdisease?
SeeTable94(t0025)forthesecharacteristics.

Case913continued:
Themassisround,mobile,rubbery,andnontender.Thereisnofamilyhistoryofbreastor
ovariancancer.

Table94
CharacteristicsofBreastMasses

Benign:Fibroadenoma

Malignant:Carcinoma

Soft

Firm

Tender

Nontender

Round,distinctborders

Irregularindistinctborders

Mobile

Fixed

Changesseenduringcycle

Nochangesseenduringcycle

Affectswomen2035yearsofage Affectswomenolderthan35yearsofage

3 Shouldamammogrambeorderedtoevaluatethisbreastmass?Whatother
studycanbedone?
Mammogramcanbeusedtofurtherevaluateasuspiciousmass,butultrasoundisusefulinaless
suspiciouspalpablemasstodetermineifitiscysticorsolid.Afluidfilledcystcanbedrainedwitha
needleintheoffice.

Case913continued:
Anultrasoundrevealsthemasstobesolid,notcystic,soanexcisionalbiopsyisperformed.

4 Whatisthemostlikelydiagnosis?
Fibroadenomaisthemostcommonbreasttumorinpremenopausalwomenitisbenign.

5 Whatisthedifferencebetweenfibroadenomaandfibrocysticbreastdisease?
Botharebenignprocesses,butfibroadenomaisanencapsulatedtumor,whereasfibrocysticbreast
diseaseencompassesawidespectrumofabnormalities,allduetoanexcessivestromalresponseto
hormonesandgrowthfactors.Thesechangescanincludecystformation,noduleformation,and
epithelialhyperplasia.
FibroadenomaisNOTaprecursortobreastcarcinoma.Fibrocysticdiseaseusuallydoesnotincrease
riskofcarcinomaexceptincasesoffibrocysticdiseasewithatypicalhyperplasia.

6 Whatiscystosarcomaphyllodes?
Phyllodestumorsareavariantoffibroadenomaandpresentasalargebulkymasseswithrapid
growth.Mostarebenign,butafewtumorsdohavemalignantcells.

7 Ifawomanpresentswithbloodynippledischarge,whatshouldbedone,and
whatarethetwodiseasesthatcancausethis?
Thedischargeshouldbesentforcytologicevaluation.Abenignprocess,intraductalpapilloma,
whichindicateslocalproliferationoftheepithelialliningofthelactiferousducts,isonepossibility.
Theotherpossibilityisinvasivepapillarycarcinoma,amalignantprocess.

Soft,round,mobilemasssuggestsbenigndisease.Firm,irregular,andimmobilemassis
concerningformalignancy.
Ultrasoundisusedtodetermineifamassiscysticorsolid.

Fibroadenomaisthemostcommonbenigntumorofbreast.Itoccursinwomenaged20
to35.
Cystosarcomaphyllodesisavariantoffibroadenomawithmalignantpotential.
Intraductalpapillomaisthemostcommoncauseofbloodynippledischarge,but
invasivepapillarycarcinomashouldberuledoutbysendingdischargeforcytologic
evaluation.
SummaryBox:WorkupofPalpableBreastMass

Case914
A59yearoldwomanisbeingevaluatedforabreastmassshedetectedwhileshowering.Sheis
concernedaboutbreastcancerbecausehermotherwasdiagnosedwithbreastcancerattheage
of72.

1 Basedsolelyonthefamilyhistoryandtheknowngeneticsofbreastcancer,is
thiswomanathighriskfordevelopingafamilialbreastcancer?
No.Thevastmajorityofbreastcancers(~90%)aresporadic.Becausehermotherwaselderlywhen
shedevelopedbreastcancer,itishighlyunlikelythatthispatienthadafamilialpredisposition(e.g.,
BRCA1orBRCA2mutations)todevelopbreastcancer.Afirstdegreerelativefamilyhistoryisarisk
factorforsporadicbreastcancer,butpostmenopausalbreastcancerinafirstdegreerelativeonly
slightlyincreasesrisk.

Case914continued:
Historyissignificantforanontendermassthatdoesnotchangeinsizeorshapewithher
menstrualcycleandasingleepisodeofbloodynippledischarge.Hermenstrualcyclesbeganat
age13andsheenteredmenopauseat55.Shehasnochildrenandhasneverbeenpregnant.

2 Whatriskfactorsforbreastcancerdoesshehave?
Female
Age(postmenopausal)
Latemenopause
Nulliparity

3 Whatisthesignificanceoftheageatmenarcheandageatmenopauseforthe
riskofdevelopingbreastcancer?

Theyoungertheageatmenarcheandtheoldertheageatmenopausearebothcorrelatedwitha
higherriskofbreastcancer.Theserelationshipsareexplainedbyincreasedcumulativeexposureto
estrogen.

4 Whydoesawomanwhohasbeenpregnantmultipletimeshavealowerrisk
fordevelopingbreastcancerthananulliparouswoman,giventhatshehas
actuallybeenexposedtolargeramountsofcirculatingestrogen?
Noonereallyknows,butthereareseveraltheoriesthatareworthmentioning.Estrogenlevelsduring
pregnancyarehigherthaninthenonpregnantwoman,butthisestrogenisnotcycling(asitdoesin
thenonpregnantwoman)anditsactionsareopposedbythehighlevelsofprogesteroneassociated
withpregnancy.

Case914continued:
Onexamination,herbreastsareasymmetricthereisafirm,nontender,irregularmassinher
rightbreast,withrednessanddimplingoftheskin,andherrightnippleappearsslightly
retracted.Allpreviousmammogramsinthiswomanhavebeennormal.

5 Basedonthephysicalexaminationfindings,doesthiswomanlikelyhave
breastcancer?
Yes,shehasmanyoftheclassicsigns:anirregular,firmbreastmasscausingretractionordimpling
oftheskinornipple(peaud'orange)andabloodydischarge.
Remember:Intraductalpapilloma,abenignprocess,isactuallythemostcommoncauseofa
bloodynippledischarge.Invasivepapillarycarcinomaisnext.

Case914continued:
Adiagnosticmammogramisordered,andsheisreferredtoasurgeonforbiopsy.

6 Whichtypeofbreastcanceristhiswomanmostlikelytohave?
Infiltrating(invasive)ductalcarcinomaisthemostcommontypeofbreastcancerandshouldbethe
presumeddiagnosispendingadefinitivepathologyreport.

7 Ifbiopsyrevealsestrogenreceptorpositive(ER+)andprogesterone
receptorpositive(PR+)cells,whymighttreatmentwithtamoxifenor
anastrazolebeuseful?
Tamoxifenisaselectiveestrogenreceptormodulator(SERM).Itactsasanestrogenreceptoragonist
incertaintissues(e.g.,bone,uterus)butasanestrogenantagonistatothertissues(e.g.,breast).
AnastrazoleisanantiestrogenaromataseinhibitorthatisalsousefulinER+andPR+cancers.

BecausegrowthofER+andPR+tumorcellsissomewhathormonedependent,thesecancerscanbe
treatedwithagentssuchastamoxifenoranastrazole.

RelatedQuestions
8 Ifanelderlywomanpresentswitheczematousnipplechanges,whatshould
yoususpectthisis,whatshouldyoudo,andwhy?
Pagetdiseaseofthenippleproducesthisfinding,andabreastbiopsyshouldbedonebecausean
underlyingmalignancyisfoundinthevastmajorityofpatientswiththisdisease.

9 Ifawomanhasaunilateralinflamedbreastandorangepeel(dimpled)
appearancetotheskinofthatbreast,whatdiseaseprocessshouldyoususpect
andwhatisthepathophysiology?
Sheprobablyhasinflammatorybreastcarcinoma,whichcanbeattributedtothetumorembolizing
intothedermallymphatics.Thisinturncausestheredness,swelling,andwarmth.Becausetherehas
beentumorembolization,itisnotsurprisingthatthereisaxillarylymphnodeinvolvement.Distant
metastasesarefrequentwhenthisisfound.

Mostbreastcancersaresporadic,withaminorityduetogeneticfactors.
Riskfactors:sex,age,previousbreastcancer,nulliparous,earlymenarche,late
menopause,obesity,highdietaryfirstdegreerelativewithbreastcancer.
Infiltratingductalcarcinomaisthemostcommonbreastmalignancy.
Tamoxifen,aselectiveestrogenreceptormodulator,antagonizesestrogeninthebreast
tissue.Anastrazoleisanaromataseinhibitor,whichdecreasesestrogenproduction.
SummaryBox:BreastCancer

Case915
A16yearoldboyvisitsthedoctorbecauseheworriesthathehasnotyetgonethroughpuberty.
Hehasbeenunabletogrowfacialhair,andhisvoicehasnotbecomedeep.Embarrassed,he
admitsthathisgenitalsaresmallerandlessdevelopedthanwhathethinksisnormalforhis
age.

1 Whatisthedifferentialdiagnosisfordelayedpuberty?
Becausethisvignettedoesnotmentionthatthepatienthaseverhadambiguousorfemaleexternal
genitalia,forthepurposeofboardswecantentativelyruleouthermaphroditism,androgen
insensitivitysyndrome,and5reductasedeficiency(thoughasageneralrule,youshouldneverrule

outadiseasebecausethepatientdoesnotspecificallymentionsomethingtoyou!).Consider
constitutionaldelay,familyhistoryofdelayedpuberty,malnutrition,hypopituitarism,andKallmann
syndrome.

Case915continued:
Atbirth,thepatientpresentedwithmicropenisandcryptorchidism.Hehasalsohadapoor
senseofsmellformostofhislife.

2 Whatisthemostlikelydiagnosisinthispatient?
Kallmannsyndrome,whichisageneticconditionthatleadstoanabsenceofGnRHproducing
neuronsinthehypothalamus.Thisleadstolackoftestosteroneproductionandsecondarysexual
characteristics.

3 Whydothesepatientsoftenpresentwithanosmia?
Kallmannsyndromecanbeassociatedwithalackofolfactoryneuronsinthebrainaswell.This
resultsinadecreasedortotallossofsmell.Patientsmayalsopresentwithcolorblindness.

Kallmannsyndromeresultsfromalackofgonadotropinreleasinghormone(GnRH)
neuronsinthebrain.
ThetriadforKallmannsyndromeisdelayedpuberty,anosmia,andcolorblindness.
SummaryBox:KallmannSyndrome

Case916
A15yearoldgirlisexaminedbyanendocrinologistinthehospital.Shehasshortstature,a
webbedneck,andabroadshieldlikechestwithwidelyspacednipples.Secondarysexual
characteristics,suchasbreastdevelopment,areabsent,andherexternalgenitaliaareinfantile
appearing.Shehasneverhadaperiod.Shehasnormalintellectandseemstobeahappy,
healthyperson.Apelvicultrasoundrevealsstreakovaries.Thelaboratoryfindingsincludea
normalgrowthhormonelevelandanelevatedFSH.Herkaryotypeis45,XO.Thereisan
absenceofBarrbodiesobservedincellsfromabuccalsmear.

1 Whatdiseasedoyoususpectisresponsibleforthispatient'samenorrhea?
Turnersyndrome,whichoccursin1inevery2500females,ismostlikely.

2 WhatcausesthedevelopmentofTurnersyndrome?

Thepresenceofanincompletesexgenotype(45,XO)duetochromosomalnondisjunction(60%)or
mosaicism(40%)isthecause.Asaresult,thesewomenhavedecreasedestrogenlevels.About80%
ofthecasesarecausedbymeioticerrorinthefatherthatis,thepatientdidnotreceiveanX
chromosomefromthefather.
Thisdiseaseischaracterizedbyanumberofphysicalabnormalities,includingshortstature,webbed
neck,andbroadchest.Girls(women)withTurnersyndromealsohaveovariandysgenesis,inwhich
insteadofnormalovariestheyhavestreaksofconnectivetissuethatdonotproducenormal
quantitiesofestrogenorprogesterone,hormonesthatarerequiredforthedevelopmentofsecondary
sexualcharacteristics.Becausetheovariesarereplacedbyfibrousstromaandaredevoidofoocytes
bytheageof2,manyofthesewomenareinfertile.However,asmallpercentage(510%)have
sufficientovariandevelopmenttosupportfertility.
Note:Turnersyndromeisthemostcommoncauseofprimaryamenorrhea.

3 Whatphenomenonisresponsibleforthispatient'swebbedneck?
Thisconditioniscalledcystichygromaandisaformoflymphangiomacommonlyfoundintheneck.
Althoughtheconditionisbenign,itcanbequitedisfiguringinsomepatients.

4 Ifthispatientweretodevelophypertension,whatdiagnostictest(s)shouldbe
performed?
PatientswithTurnersyndromeareatincreasedriskforpreductalcoarctationoftheaorta,which
oftenresultsinhypertensionlimitedtotheupperextremitiesandcerebralvessels.Inamanner
analogoustorenalarterystenosis,aorticcoarctationcauseshypertensionbecausechronic
underperfusionofthekidneysresultsinactivationofthereninangiotensinaldosteronesystemand
volumeretention.
Theeasiestinitialdiagnostictestisacomparisonofupperandlowerextremitybloodpressuresto
assessforasignificantdiscrepancy.Coarctationresultsinalowerfemoralarterypressureanda
delayedfemoralpulserelativetothepressureandpulseofthebrachialartery.Achestxraystudy
willoftenshowtheclassicribnotchingduetoincreasedcollateralbloodflowthroughthe
intercostalarteries,whichbypassthecoarctation.Imagingoroftenechocardiogramisusually
performedtoconfirmthediagnosis.

5 WhataresomeothercomplicationsofTurnersyndrome?
Othercomplicationsincludebicuspidaorticvalve,horseshoekidney,andhypothyroidism.

6 Howmightthispatientbemanagedpharmacologicallytocorrectthelackof
secondarysexualcharacteristics?
EstrogentherapyforteenagegirlswithTurnersyndromecanhelppromotedevelopmentof
secondarysexualcharacteristics.

7 WhatisKlinefelter'ssyndrome?

ThegenotypeofKlinefelter'ssyndromeis47,XXY(sometimes48,XXXY).Becausetheseindividuals
haveaYchromosome,theyaremaleswhopossessaBarrbody.Theyundergofibrosisofthe
seminferoustubules,leadingtoazospermia,infertility,andlossofSertolicells.Decreasedinhibin
productionsecondarytoSertolicelllossleadstoincreasedproductionofFSH.Klinefelterpatients
alsoexhibitabnormalLeydigcellfunction,whichresultsindecreasedproductionoftestosteroneand
testicularatrophy.Becausetestosteroneisunavailabletoexertnegativefeedbackuponthepituitary,
LHconcentrationsbecomehigh.Thisstimulatestheproductionofestrogens,leadingto
gynecomastia,eunuchoidbodyshape,andfemalehairdistribution.Thesepatientstendtobetall
withlongextremitiesduetodelayedclosureoftheepiphysealplatesthatresultsfromdecreased
androgenconcentrations.
TurnerandKlinefeltersyndromesappearveryfrequentlyonboardssobesurethatyoucan
recognizetheirpresentations.
Step1Secret

8 HowdoesthepresentationoftheXXYKlinefelterphenotypedifferfromthe
XYYphenotype?
XYYordoubleYmalesarephenotypicallynormalwithnormalfertility.Theytendtobetallerthan
normalmalesandoftenpresentwithsevereacne.Itisalsotypicalforthesemalestopresentwith
aggressive,antisocial,orevencriminalbehavior.

9 WhatistrisomyX?
TrisomyXreferstothegenotype47,XXX.Thesefemalesoftenhavenoapparentabnormalitiesbut
mayexperiencemildmenstrualirregularitiesandexhibitmildmentalretardation.Theyare
frequentlytallerthannormalfemales.

Turnersyndrome(45,XO)resultsindecreasedestrogenlevelsandstreakedovaries.
Patientstypicallypresentwithshortstature,webbedneck,andbroadenedchest.Turner
syndromeisthemostcommoncauseofprimaryamenorrhea.Itisassociatedwithan
increasedriskofcoarctationoftheaorta.
Coarctationoftheaortaresultsinadiscrepancybetweenupperandlowerextremity
pressurescharacterizedbyupperextremityhypertension.Achestxrayfilmwill
classicallyshowribnotching.OthercomplicationsofTurnersyndromeincludebicuspid
aorticvalve,horseshoekidney,andhypothyroidism.
Klinefeltersyndrome(47,XXYor48,XXXY)resultsindecreasedlevelsoftestosterone
andincreasedlevelsofestrogen.Patientstypicallypresentwithinfertility,gynecomastia,
femalehairdistribution,eunuchoidbodyshape,tallstature,andlongextremities.
TrisomyX(47,XXX)femalesaretypicallynormalbutmayexperiencemildmenstrual

irregularitiesorexhibitmentalretardation.Theyaregenerallytall.
XYYindividualsarephenotypicallynormalbutmaybetallerandmoreaggressivethan
normalmales.
SummaryBox:SexChromosomeDisorders

Case917
A16yearoldgirlisconcernedbecauseshehasn'tstartedhavingherperiodyet,whereasallof
herfriendshavehadperiodsforatleast2yearsnow.Sheadditionallyhasnobreast
development.Onphysicalexaminationshehasscantaxillaryandpubichair,andtheuterusis
notpalpable.Onspeculumexaminationnocervixisvisible(i.e.,thevaginaendsinablind
pouch).Apelvicultrasoundisperformed,andsheisfoundtohavenouterusorovariesbut
insteadundescendedtestes.Consequently,akaryotypeisperformed,whichcomesback46,XY.

1 Whatisthispatient'ssyndrome?
Thischildlikelyhasandrogeninsensitivitysyndrome,whichisalsoknownastesticularfeminization
syndrome.Thesepatientsaregeneticallyandgonadallymalebutphenotypicallyfemale.

2 Whatisthecauseofthissyndrome?
Asmentionedinthebasicconceptssectionofthischapter,geneticalterationsintheandrogen
receptormakethetissuesunresponsivetotheandrogeniceffectsoftestosteroneandother
androgens.Testiclesarepresentandfunctional(theyproducetestosterone)butthetissuesdonot
respond.

3 Whydoesthispatienthaveavaginalpouch?
Inanormalmale,thepotentandrogenDHT,whichisproducedfromtestosteronebytheactionof
theenzyme5reductase,actsupontheandrogenreceptorinthetissuesoftheurogenitalfoldto
stimulatetheformationoftheexternalgenitalia(i.e.,penis,prostate,andscrotum)during
development.
Intheabsenceofandrogenactivity,eitherinanormalfemaleorinamalewithandrogen
insensitivity,theurogenitalfolddevelopsintoavaginalpouch.
TheeffectofDHTongenitaltissuehasclinicalrelevanceinthetreatmentofbenignprostatic
hyperplasia(BPH).Inhibitorsoftheenzyme5reductase,(e.g.finasteride)havebeenshownto
reduceprostatesize,oftenresultinginsymptomaticimprovementinmenwithBPH.

4 Whydoesthispatienthavenopubicoraxillaryhair?

Inbothmalesandfemales,theinitialdevelopmentofsecondarysexualcharacteristics,particularly
pubicandaxillaryhair,adultbodyodor,andsebaceousglandactivity,dependsonadrenalandrogen
production.Thesechangesusuallyoccurbeforethehormonalchangesofcentralpubertythatis,
increasedproductionofestrogen(specificallyestradiol)infemalesandtestosteroneinmales.In
androgeninsensitivity,lackofactivityofadrenalortesticularandrogensresultsinthelackofaxillary
andpubichair.
Recallthatthemajoradrenalandrogensincludedehydroepiandrosterone(DHEA)and
dehydroepiandrosteronesulfate(DHEAS).Infact,DHEASisspecifictotheadrenalgland,andits
levelswillbeincreasedindisordersofadrenalandrogenoverproductionsuchasrarehormone
producingadrenalcarcinomasandthevirilizingformsofcongenitaladrenalhyperplasia.

5 Howcanweexplainthelackofuterusinthischild?
Recallthattheuterusisformedfromthemllerianduct.Inmalesthesestructuresaredissolvedby
mllerianinhibitingsubstance(MIF),whichisproducedbythetestes.Asthereisnoabnormalityin
thetestesthemselvesinthisdisease,thissubstancewillbesecretedandtheductwilldissolve,
resultingintheabsenceofaninternalfemalereproductivetract(i.e.,fallopiantubes,uterus,and
cervix).
Recallthattheinternalmalegenitaltract(i.e.,theseminiferoustubules,theepididymis,andvas
deferens)developsfromthewolffianduct,butthisdifferentiationrequirestheactivityof
testosterone.
Thus,becausethetestesarenormalandproducemllerianinhibitingsubstanceinandrogen
insensitivitysyndromebuttheactivityoftestosteroneisabsent,neitheramalenorafemaleinternal
genitaltractdevelops.

6 Wouldtestosteronelevelsbeloworhighinthispatient?
Testosteronelevelswouldbehighbecausethenonfunctionalandrogenreceptorspreventnegative
feedbackuponpituitaryLHproduction.HighLHlevelsinturnincreasetestosteroneproductionby
thetestes.Inotherwords,thelackoftestosteroneactivityinthepituitarydisinhibitsLHrelease,
whichstimulatesthetestestoproducehighlevelsoftestosterone.

7 Whyshouldthispatient'stesticlesberemoved?
Undescended(orcryptorchid)testesfromanycause,whetherspontaneouslyundescendedor
undescendedaspartofasyndrome,areatincreasedriskforgivingrisetotesticularcancer.Ifan
undescendedtestisfailstospontaneouslydescendwithinaboutayearorsoofbirth,itshouldbe
surgicallyreplacedwithinthescrotum(i.e.,orchiopexy)orsurgicallyremoved(i.e.,orchiectomy).

Normallyinfemales,intheabsenceoftestes,internalandexternalfemalereproductive
organsdevelop.

Normallyinmales,thedevelopingtestesproducebothtestosteroneandmllerian
inhibitingsubstance(MIF).Themaleinternalreproductivetractdevelopsunderthe
influenceoftestosterone,andthefemaleinternalreproductiveorgansinvoluteunderthe
influenceofMIF.
Themaleexternalgenitalia,includingtheprostate,normallydevelopunderthe
influenceofdihydrotestosterone(DHT).Theenzyme5reductaseconvertstestosterone
intothemorepotentandrogenDHT.Inhibitorsofthisenzyme(suchasfinasteride)are
usedtotreatbenignprostatichyperplasia.
Adrenalandrogens,suchastheadrenalspecifichormonedehydroepiandrosterone
sulfate(DHEAS),arenormallyresponsiblefortheinitialdevelopmentofsecondary
sexualcharacteristics(suchasadultbodyhair,adultbodyodor,andsebaceousgland
activity)duringpubertyofbothmalesandfemales.
Androgeninsensitivitysyndrome(ortesticularfeminizationsyndrome)resultsfroma
defectintheandrogenreceptor.Patientshaveamalekaryotype(46,XY)and
undescendedtestes,buttheyfailtodevelopmaleexternalgenitaliaandphenotypically
appearfemale.BecausetheirtestesareabletoproduceMIFtheylackafemaleinternal
reproductivetracthowever,becausetheylacktestosteroneactivity,theyalsolackamale
reproductivetract.Patientsareleftwithfemaleexternalgenitalia,includingavaginathat
endsinablindpouch.Lackofandrogenactivityalsoresultsinabsenceofsecondarysex
characteristicssuchaspubicandaxillaryhairoradultbodyodor.
Undescended(orcryptorchid)testesofanycauseshouldeventuallybereplacedwithin
thescrotumorsurgicallyremovedbecauseofincreasedriskofmalignancy.
SummaryBox:AndrogenInsensitivitySyndromeandSexualDevelopment

Copyright2015Elsevier,Inc.Allrightsreserved.

BOOKCHAPTER

Nephrology
ThomasA.BrownMDandSonaliJ.Shah
USMLEStep1Secrets,Chapter3,6796

Nephrologycanbeatrickysubjectformanymedicalstudents,especiallybecausemastering
renalphysiologyrequirescomfortwithandevenmanipulationofseveralformulas.Luckily,the
scopeofnephrologyrelatedquestionsontheUSMLEtendstoberatherlimited.Focusheavily
onthesubjectslistedinFirstAidandthenusethecasesinthischaptertothentestyour
understandingofthematerial.Besuretounderstanddetailedprinciplesregardingnephron
functionandactivationofthereninangiotensinaldosteronesystem.Withregardtorenal
pathology,themostcommonlytestedsubjectsincluderenalfailure,glomerulonephritis,
urinarytractinfections(UTIs),andrenalstones.
Insider'sGuidetoNephrologyforTheUSMLEStep1

Clinicalrenaldisease
Case31
A78yearoldmancomestotheemergencyroombecauseofinabilitytourinateforthepast2
daysandisexperiencinglowerabdominalpain.Headmitstourinarydifficultiesoverthelast
fewyears,includingdifficultyinitiatinghisstream,aweakstream,nocturia,anddribblingafter
voiding.Hedeniestakinganytricyclicantidepressants,antipsychotics,antihistamines,or
sympathomimeticagents.Onphysicalexaminationhisbladderseemspalpablyenlarged,anda
digitalrectalexaminationrevealsanenlargedprostate.Microscopicurineexaminationshows
nohematuriaorcrystalluria.Routinelaboratorytestsshowsignificantlyelevatedbloodurea
nitrogen(BUN)andcreatinineandahighnormalprostatespecificantigen(PSA)of3.8.An
abdominalultrasoundrevealsamarkedlydistendedbladderandenlargementoftherenal
pelves.

1 Whatistheprobablecauseofthispatient'sproblems?
Thispatienthasacuteurinaryretention,mostlikelysecondarytoocclusionofthebladderneckby
benignprostatichyperplasia(BPH).BPHwillgenerallyresultinamildelevationofPSA,but
typicallynothighenoughtopromptevaluationforprostatecancer(whichistraditionallyundertaken

forPSAlevels>4).AlthoughtotalPSAlevelsareelevatedinbothBPHandprostatecancer,freePSA
levelswillonlybeelevatedwithBPH.TheelevatedBUNandcreatininestronglysuggestthatthis
patientalsohasacuterenalfailure.

2 Whatarethethreeetiologicclassificationsofacuterenalfailure?
Renalfailureisclassifiedasprerenal,renal(intrinsic),orpostrenal(obstructive).Thisman'srenal
failureclearlyhasanobstructiveetiology.AcutebladderdistentioncausedbylongstandingBPHhas
resultedinbilateralhydronephrosis(dilatedrenalpelves)andacuterenalfailure.Figure31(f0010)
showsseverehydronephrosisfromlongstandingobstruction,withmarkeddilatationoftherenal
pelvisandcalycesandlossofcorticalparenchyma.

Figure31
Hydronephrosisofthekidney,withmarkeddilationofthepelvisandcalycesandthinningoftherenal
parenchyma.
(FromKumarV,AbbasAK,FaustoN:RobbinsandCotranPathologicBasisofDisease,7thed.Philadelphia,WB
Saunders,2005.)

3 Howisglomerularfiltrationrate(GFR)calculated?
TheGFRcanbecalculatedusingthefollowingformula:
GFR = Kf (PG PB G + B )

where
Kf=filtrationcoefficient,aconstant
PG=hydrostaticpressureintheglomerularcapillaries
PB=hydrostaticpressureinBowman'scapsule
G=oncoticpressureintheglomerularcapillaries

B=oncoticpressureinBowman'scapsule,typically0

4 Whyistheabsenceofhematuriaonurinalysisimportantinestablishingthe
diagnosis?
Urinarycalculi(stones)cancauseurinarytractobstructionandareoftenassociatedwithhematuria
ormicroscopiccrystalsintheurine.Stonescanuncommonlycauseobstructiverenalfailure,but
usuallyonlyiftheyarepresentintheuretersbilaterally.Tumorsofthebladderorureters(either
primaryurothelialtumorsortumorsthatinvadeorcompresstheurinarytract)canalsocauselower
tractobstructionwithrenalfailureandhematuria.
Broadlyspeaking,obstructiverenalfailureisusuallycausedbyobstructionatthelevelofthebladder
neckortheureters.Othercausesofobstructionatthelevelofthebladderincludeneurogenicbladder
andmedicationeffects.Anotherclassic(albeituncommon)causeofureteralobstructionis
encasementoftheuretersbyretroperitonealfibrosis.

5 Whywasitimportanttoaskthispatientabouttricyclicantidepressants,
antipsychotics,antihistamines,andsympathomimetics?
Theseclassesofdrugscanallcauseurinaryretentionsecondarytocholinergicblockade,which
inhibitscontractionofthedetrusormuscle.
Tricyclicantidepressants(e.g.,amitriptyline,imipramine)thephenothiazineantipsychotics,which
includelowpotencytypicalantipsychoticssuchaschlorpromazineandthioridazineandfirst
generationantihistaminessuchasdiphenhydramine(Benadryl)areallolderdirtyagentsthatact
onamultitudeofreceptortypes,includingmuscarinicacetylcholinereceptors.Hence,commonside
effectsofthesedrugsincludevariousanticholinergicactionssuchasdrymouth,constipation,and
urinaryretention.Recallthatthedetrusormuscleofthebladderisstimulatedtocontractby
parasympathetic(cholinergic)innervation.Theanticholinergicagentatropinehasasimilareffect.
Theanticholinergiceffectsofthewidelyusedagentsoxybutynin(Ditropan)andtolterodine(Detrol)
areusedtherapeuticallytocontrolurgeincontinence.
Thesympathomimetics,ontheotherhand,cancauseurinaryretentionbyincreasingthetoneofthe
internalurethralsphincter.
Opiates(i.e.,narcotics)havemanyanticholinergiclikesideeffectsand(inadditiontoconstipation,
drymouth,pupillaryconstriction,etc.)cancauseacuteurinaryretentionwhengiveninhighdoses.

Thecausesofacuterenalfailure(ARF)aredividedintothreegroups:prerenal,renal
(intrinsic),andpostrenal(obstructive).
Commoncausesofpostrenalfailureincludebenignprostatichyperplasia(BPH)or
prostatecancer,bladdertumors,andurinaryretention(duetoneurogenicbladderor
anticholinergic,opiate,orsympathomimeticdrugs).Unlesspresentbilaterally,renal

stonesrarelyleadtoobstructivekidneyfailure.
Thediagnosisofobstructiverenalfailureismade,inpart,bydetectionof
hydronephrosis(pelvicalicectasis)onrenalultrasound.
SummaryBox:ObstructiveAcuteRenalFailure

Case32
A68yearoldmanexperiencedarupturedabdominalaorticaneurysmandwasrushedtothe
hospitalandintotheoperatingroom(OR).Duringsurgery,thesurgeonshadtoclampthe
abdominalaortaatalevelsuperiortotherenalarteriesforalittleoveranhour.Althoughthe
patientsurvivedtheoperation,thenextmorninghewasfoundtohaveasevereacutedeclinein
hisrenalfunctionwithaBUNof75mg/dLandcreatinineof3.2mg/dL.HiscalculatedGFR
wasdepressedto~10mL/min,hisurineoutputdiminishedto~200mLin24hours(despite
hisbeingnormotensive),andaurinemicroscopyrevealed"muddybrown"granularcasts.

1 Whatistheprobablediagnosis?
Thelikelydiagnosisisacutetubularnecrosis(ATN),secondarytoprolongedischemiaduringthe
operation.

2 Whatisthepathophysiologyofacutetubularnecrosis?
InATN,therenaltubularepithelialcellsaredamagedandsloughedofffromthetubularbasement
membrane.Thesesloughedcellscanbeformedbythetubulesintoepithelialcellcasts.Suchcasts
cansometimesbeseenintheurineofpatientswithATN.Morecommonly,theepithelialcellcasts
havebeendegradedsomewhatintopigmentedmuddybrowngranularcaststhat,liketheepithelial
cellcasts,arehighlysuggestiveofATN.Insomecases,theurinesedimentwillrevealnonpigmented
transparenthyalinegranularcastssuchcastscanbeformedfromfurtherdegradationofmuddy
browncasts,buttheycanalsobeformedfromotherprocessesaswellandsoarenotspecificfor
ATN.
ATNcanbeischemicortoxic.Thatis,thetubularepitheliumcanbedamagedbyeitherischemia(as
inthispatient)orbynephrotoxinssuchasthechemotherapeuticagentcisplatin,aminoglycoside
antibiotics,theantifungalamphotericinB,intravenousradiographiccontrastmedia,heavymetals
(Fanconisyndrome),andthehemepigmentsmyoglobinandhemoglobin.

3 Whatisthecauseofthedecreasedglomerularfiltrationrateandoliguriaseen
inacutetubularnecrosis?
Thesloughedtubularepithelialcellsblockthelumenoftherenaltubules,impedingurineflow.
Additionally,thedenudedareasofbasementmembraneallowbackleakageoffilteredfluid.
Althoughthisbackleakagedoesnotchangetheamountoffluidfilteredacrosstheglomerular

membrane,itdoeschangetheamountofwasteproductsthatareexcreted,whichconsequently
changesthecalculatedGFR.
Boththeobstructiontoflowandthebackleakageexplainwhyoverallurineoutputisfrequently
reducedinATN.

4 Whatarethethreephasesofacutetubularnecrosis?
Intheinitiationphase,theinjuriousagentorconditionispresent,butthedeteriorationofrenal
functionhasnotyetbegunorisjustbeginning.Inthemaintenancephase,GFRisreducedand
oliguriapersistsitisatthistimethaturemiccomplicationsarelikelytomanifest.Finally,inthe
recoveryphase,renaltubularepithelialcellsproliferateandrepopulatethedenudedareas,andurine
outputnormalizes.

5 Otherthanthepresenceofmuddybrowncasts,howcanprerenalazotemia
duetoischemiabedifferentiatedfromischemicacutetubularnecrosis?
Inclinicalpractice,thisisaveryimportantdistinctiontomake,becauseprerenalazotemia(by
definition)willrespondtofluidresuscitation,whereasischemicATNwillnot.
Inprerenalazotemia,inadequaterenalperfusionreducesGFR,butthereducedperfusionisnotso
severeastocausecellulardamage.Thekidneysthereforefunctionnormallyinresponsetothe
hypoperfusionbyretainingNa+.Thistypicallyresultsinadecreasedfractionalexcretion(FE)ofNa
+

(FENa+)tobelow1%.FENa+issimplytheratioofurinesodiumtoplasmasodium,butwitheach
valuenormalizedaccordingto(i.e.,dividedby)thecorrespondingcreatinineconcentration:
FENa

= CNa /GFR = (UNa V/PNa )/(UCr V/PCr ) = (UNa PCr )/ (PNa /UCr )

where
CX=clearanceofsubstanceXvolumeofplasmafromwhichsubstanceXisclearedperunit
time
UX=excretionrateofsubstanceX
V=urinevolume
PX=plasmaconcentrationofsubstanceX
Cr=creatinine
Ifthehypoperfusionissevereenough,prerenalazotemiawillprogress,resultingintheischemic
damagetothetubularepitheliumthatischaracteristicofATN.InATN,becausemanyofthetubular
cellsarenolongerfunctional,thekidneyisunabletoretainNa+asitshouldinthesettingof
decreasedGFR.Asaresult,theFENa+inATNistypically>2%.

InadditiontotheFENa+,theBUN/creatinineratiocansometimesbeusefulindistinguishing
betweenprerenalazotemiaandATN.Ureaisprimarilyreabsorbedintheproximaltubule.Inthe
settingoflowextracellularvolume(ECV),theincreasedNa+reabsorptionintheproximaltubule(as
stimulatedbyangiotensinII,thesympatheticnervoussystem,andintrinsicglomerularprocesses)
willtendtopulladditionalureaoutofthefiltratethroughbulkflow.Thus,BUNcanbeusedasa
markerforproximalNa+reabsorption.Creatinine,incontrast,islessaffectedbyNa+reabsorption
(recallthatcreatinineisausefulmarkerforGFRbecauseitisnotsignificantlyreabsorbed).
Specificallyinprerenalazotemia,oneexpectsaBUN/Crratioelevatedto>20.InATN,inwhichthe
reabsorptionofNa+isimpaired,thisratioisoften<10(damagetotherenaltubulesimpairsurea
reabsorption,sotheBUN/Crratioisdecreased).AlthoughtheBUN/Crratioismorereadilyavailable
(asitrequiresonlystandardbloodtestsinsteadofbothurineandblood),itissignificantlyless
accuratethantheFENa+andcanchangeindependentlyofrenalfunction.Aclassicexampleisa
gastrointestinal(GI)bleedinwhichlargeamountsoftheproteinhemoglobinarebrokendowninthe
GItractintoureathatisextensivelyreabsorbedintothecirculation,elevatingtheBUNlevelandthe
BUN/Crratioinamannerthatiscompletelyindependentofrenalfunction.

RelatedQuestion
6 Whatisrhabdomyolysisandhowcanitcauseacutetubularnecrosis?
Rhabdomyolysisisacuteextensivedestructionofskeletalmusclecellsitcanoccurwithtrauma
(especiallyprolongedcrushinjuries),drugs(suchasstatins),andahostofotherscenarios.With
muscleinjury,largeamountsoftheO2storagemoleculemyoglobinarereleasedintothecirculation
and,uponarrivaltothekidneys,leadtorenalfailureviamultiplemechanisms.First,myoglobinis
directlytoxictorenaltubularepithelialcells.Inaddition,myoglobincancausesevererenal
vasoconstriction(forpoorlyunderstoodreasons),resultinginanischemiccomponenttotheATN.
Finally,thereleasedmyoglobincanprecipitateintherenaltubulesandcauseobstruction.
ContrastinducedATNissimilartomyoglobininducedATNinthatitinvolvesbothdirecttoxicand
vasoconstrictive/ischemiccomponents,thoughtheprimarymechanismoccursthrough
vasoconstriction.ContrastinducedATNcanbepreventedwithNacetylcysteine,whichisalsoused
forcysticfibrosistreatmentandacetaminophentoxicity.

ATNisthemostcommoncauseofinstrinsicrenalfailure.
ATNcanbeischemicortoxic.ToxinscapableofcausingATNincludecisplatin,
aminoglycosides,vancomycin,amphotericin,intravenouscontrastagents,and
myoglobin.
IschemicATNisdistinguishedfromprerenalazotemiabythepresenceofmuddy
browncastsintheurinarysedimentandanFENa+>2%.Prerenalazotemia,incontrast,

ischaracterizedbyablandsediment,byaFENa+<1%,and(lessreliably)byabloodurea
nitrogen/creatinine(BUN/Cr)ratio>20.
SummaryBox:AcuteTubularNecrosis

Case33
A53yearoldwhitefemalepatientdevelopsmediastinitisfollowingcoronarybypasssurgery.
Shebecomessepticwithmultiorganfailure,requiringintensivecareunit(ICU)admission,
intubation,pressorsupport,andhemodialysis.Hersternalwoundissurgicallydbrided,and
intraoperativeculturesgrowmethicillinsensitiveStaphylococcusaureus.After1week,sheis
dischargedhomeonlongtermintravenousnafcillin.Tendaysafterdischarge,shereturnsto
theemergencydepartmentcomplainingofseveraldaysoffever,nausea,malaise,andrash.On
examination,sheisfebrileto38.7C,andshehasafullbody,intenselyerythematous
maculopapularrash.Initiallaboratoryfindingsaremostnotableforanelevatedcreatinineof
3.9mg/dLhercreatininehadcompletelynormalizedpriortodischarge.Herwhitebloodcell
(WBC)countisalsomoderatelyelevatedat12units,withadifferentialof62%neutrophils,22%
lymphocytes,12%eosinophils,and4%monocytes.

1 Whatisthemostlikelycauseforheracuterenalfailure?
Thispresentationismostconsistentwithacuteinterstitialnephritis(AIN)causedbynafcillin.AINis
mostcommonlyanallergicreactiontoadrug.Commonculpritsincludeantibiotics,particularly
lactams(especiallypenicillinsandcephalosporins)andsulfonamidesnonsteroidalanti
inflammatorydrugs(NSAIDs)cimetidineandprotonpumpinhibitors(PPIs),suchasomeprazole
andpantoprazole.AINislesscommonlycausedbyinfectionsandisrarelyamanifestationofan
autoimmunedisease(suchassarcoidosis).
InasettingofARF,thetriadoffever,rash,andperipheraleosinophiliafollowinginitiationofnew
medicationishighlysuggestiveofAIN,butallthreeofthesenonrenalmanifestationsarepresentin
onlyaminorityofpatients.UrinemicroscopymayshowWBCs,WBCcasts,andredbloodcells.
Urineeosinophilsarehighlysuggestive,butmaynotbepresentinallcases.Whenthediagnosisof
AINisunclear,biopsycanbeperformed,butoftenempirictherapywithcorticosteroidsisattempted
first.

2 WhatarethethreemajortypesofNSAIDinducedrenaltoxicity?
NSAIDscancauseabewilderingarrayofrenalsideeffects.
Inadditiontointerstitialnephritis,NSAIDscanalsocausenephroticsyndrome.Thistypically
manifestsasminimalchangediseaseinanadulttakingNSAIDs,butothertypesofglomerular
processes(suchasmembranousnephropathy)arepossibleaswell.

ThemostcommonrenaltoxicityofNSAIDsishemodynamicallymediatedacuterenalfailure.Inthe
normalkidney,vasodilatoryprostaglandins,suchasPGI2(prostacyclin)andPGE2,areproducedto
helpmaintainadequaterenalperfusion.Theenzymecyclooxygenase(COX)isrequiredfor
prostaglandinproduction.NSAIDadministrationandtheresultantinhibitionofcyclooxygenase1or
2byNSAIDscanresultinvasoconstrictionoftherenalarterioles,renalhypoperfusion,anda
dramaticdecreaseinGFR.RiskfactorsforNSAIDinducedrenalfailureincludeage>65years,
baselinerenaldysfunction,andintravascularvolumedepletion(e.g.,diureticuseandcirrhosis).This
isinpartbecausesuchpatientswithrenaldysfunctionorvolumedepletiondependmoreheavily
thannormalonprostaglandinproductiontomaintainadequaterenalperfusion.
Nonsteroidalantiinflammatorydrug(NSAID)induceddecreaseinglomerularfiltrationrate
(GFR)isanimportantconcepttoknowforStep1.Ingeneral,youshouldbeawareofallofthe
mechanismsbywhichGFRandfiltrationfractioncanbealtered.Youarelikelytobeaskeda
questiononthistopic.
Step1Secret

Rememberingthemostcommonconditionsthatcauseeosinophiliacanbehelpful.These
conditionsincludehelminthicinfections,asthmaandallergicdisorders,druginducedacute
interstitialnephritis(AIN),andcertainformsofmalignancy(e.g.,Hodgkin'slymphoma).
Step1Secret

Acuteinterstitialnephriyis(AIN)isclassicallycharacterizedbythetriadoffever,rash,
andperipheraleosinophiliainasettingofARFafterintroductionofanewdrug.Urine
eosinophils,althoughoftennotpresent,arealsohighlysuggestiveofAIN.
AgentsassociatedwithAINincludepenicillins,cephalosporins,sulfonamides,
nonsteroidalantiinflammatorydrugs(NSAIDs),andPPIs.
NSAIDscancauseAIN,nephroticsyndrome,andmostcommonly,hemodynamicrenal
failurefrominhibitionofvasodilatoryprostaglandins.Patientswithadvancedage,renal
dysfunction,orloweffectivecirculatingvolume(e.g.,congestiveheartfailure[CHF])are
athighestriskforNSAIDinducedacuterenalfailure(ARF).
SummaryBox:AcuteInterstitialNephritis

Case34

A65yearoldAfricanAmericanwomanwithalonghistoryofpoorlycontrolledhypertensionis
evaluatedforanelevatedplasmacreatininethathasbeennotedonroutinelaboratorytests
overthepast6months.Additionallaboratoryfindingsincludehyperkalemia,hypocalcemia,
hyperphosphatemia,andametabolicacidosis.HemoglobinA1candfastingglucosevaluesare
bothnormal.Urinemicroscopydoesnotrevealanyhematuriaorcasts.

1 Whatismostlikelycausingherbloodureanitrogenandcreatinineelevation?
Shehasrenalfailureresultingfromalonghistoryofpoorlycontrolledhypertension.Hypertensionis
thesecondmostcommoncauseofchronickidneydisease.Renalfailurefromhypertensionis
particularlycommoninAfricanAmericans.
Thepathologicchangesassociatedwithlongstandinghypertensionaretermedhypertensive
nephrosclerosis(alsoknownasbenignnephrosclerosisorhyalinearteriolarnephrosclerosis).As
showninFigure32(f0015),hyalinearteriosclerosisisassociatedwithhyalinedeposition,marked
thickeningofthewalls,andanarrowedlumen.

Figure32
Hyalinearteriolosclerosis.Highpowerviewoftwoarterioleswithhyalinedeposition,markedthickeningofthe
walls,andanarrowedlumen.
(CourtesyofDr.M.A.Venkatachalam,DepartmentofPathology,UniversityofTexasHealthSciencesCenter,
SanAntonio,TX.)

2 WhatisthevalueofthenormalfastingglucoseandhemoglobinA1clevelsin
thedifferentialdiagnosis?
Thisfindingessentiallyrulesoutacomponentofdiabeticnephropathy,themostcommoncauseof
chronicrenalfailure(CRF).

3 Whatisthedifficultyinestablishingthatthispatient'srenalfailurewas
definitelyduetohypertension,evenifnootherspecificdiseaseprocessescan
beidentifiedonrenalbiopsy?
Thedifficultystemsfromthefactthathypertensionisbothapotentialcauseandapotentialresultof
renaldisease.Furthermore,ifrenalfailureischronic,renalbiopsyfindingscanbenonspecificand
mayfailtodistinguishbetweenotherprecipitatinginsults.Forexample,herrenalfailuremayhave

beenduetoaboutofglomerulonephritisthatpermanentlydamagedthekidneys,andher
hypertensioncouldhaveresultedfromthekidneydamage.Nevertheless,hypertension,regardlessof
itscause,contributestoprogressivelossofrenalfunction.

4 Howdoesthiswoman'srenalfailureexplainherhypocalcemia?
Ingeneral,renaldysfunctionleadstotheaccumulationofthevariouselectrolytesthatarenormally
excretedbythekidneys,resultinginhyperkalemia,hyperphosphatemia,andacidosistypicalofrenal
failure.Calciumisratherunusualinthatitsserumlevelsmaybedecreasedinrenalfailure.Keepin
mindthatinclinicalpractice,truehypocalcemiawithchronicrenalfailureisrareduetotherapid
compensatoryincreaseinserumPTHlevelsthatismediatedbytheparathyroidglands.
Whenitdoesoccur,hypocalcemiaresultsfromseveralprocesses.First,thekidneyisthesiteof1,25
dihydroxyvitaminD(i.e.,calcitriol)synthesisfrom25hydroxyvitaminD,viatheactivityofrenal1
hydroxylaseintheproximaltubule.Sincethe1,25dihydroxyformofvitaminDistheactiveform
thatstimulatesintestinalcalciumabsorption,lossofrenalparenchymareducessynthesisofthis
compoundandreducesintestinalcalciumabsorption.Second,asGFRdeclines,renalphosphate
excretiondeclinesandleadstohyperphosphatemia.Thiselevatedserumphosphatecancomplex
withserumcalciumandreducefreeionizedcalciumlevels.Inaddition,theincreasedphosphate,
throughnegativefeedback,alsoinhibitsthesynthesisof1,25dihydroxyvitaminD.(Recallthat1,25
dihydroxyvitaminDtendstoincreaseserumlevelsofbothCa2+andphosphateasitpromotesthe
intestinalabsorptionofbothsubstances.Parathyroidhormone[PTH],ontheotherhand,increases
serumcalciumlevelswhilepromotingphosphateexcretionatthelevelofthekidney.)
Note:TherearetwoformsofvitaminD:plantderivedvitaminD2(ergocalciferol)isacquiredinour
dietsvitaminD3ismadeendogenouslyinourskininareactionthatiscatalyzedbyultraviolet(UV)
rays(i.e.,sunlight).Inordertobecomebiologicallyactive,bothvitaminD2andD3mustbe
hydroxylatedtwice.Thefirsthydroxylationisunregulatedandoccursinthelivertoproducethe25
hydroxyvitaminDdiolform.Thesecondstep,whichisimpairedinrenalfailure,ishighlyregulated
andproducesthe1,25dihydroxyvitaminDtriolform(hencethenamecalcitriol).

5 Wouldparathyroidhormonelevelsbeincreasedordecreasedinthispatient?
BecausePTHisreleasedinresponsetohypocalcemia(orhyperphosphatemia),PTHlevelsare
increasedinrenalfailure.
BecausethisincreaseinPTHisreactiveinnature(i.e.,anappropriateresponsetothehypocalcemia
andhyperphosphatemiaofrenalfailure),itistermedsecondaryhyperparathyroidism.Patientsin
earlyrenalfailurewilloftenhaveahighPTHlevelwithrelativelynormalserumcalcium.However,in
severerenalfailure,asboth1,25hydroxyvitaminDlevelsdecreaseandphosphatelevelsincreasetoa
greaterextent,theprogressivelyelevatedPTHlevelsareunabletocompensate,andprogressively
severehypocalcemiadevelops.

Note:Primaryhyperparathyroidism,inwhichanabnormalityoftheparathyroidgland(typically
glandularhyperplasiaoranadenoma)resultsinincreasedPTHlevelsasaprimarydisturbance,
causeshypercalcemiaratherthanhypocalcemia.

6 HowdoesparathyroidhormonenormallyacttoregulateserumCa2+?How
areparathyroidhormonelevelsnormallyregulated?
ThevariousactionsofPTHactoveralltoincreaseserumCa2+whilemaintainingserumphosphate
levels.
Firstofall,PTHstimulatesbonereabsorption,whichreleasesCa2+andphosphateintocirculation.It
alsostimulatesactivationofvitaminDbystimulatingtheactivityof1hydroxylaseintheproximal
tubule.Theactive1,25dihydroxyvitaminDthenincreasesintestinalabsorptionofCa2+and
phosphateand(athighlevels)promotesboneresorption,againactingtoincreaseserumlevelsof
bothCa2+andphosphate.Finally,PTHstimulatesCa2+reabsorptionatthelevelofthekidneywhile
stronglypromotingphosphateexcretion.
Thus,PTHactsdirectlyandindirectlyatbone,theGItract,andthekidneystoincreaseplasmaCa2+
.However,becausethephosphatereleasedfromboneandabsorbedfromtheGItractisexcretedin
thekidneys,thecombinationofPTHandvitaminDactivationtendstohavenoneteffectonserum
phosphate.
Ingeneral,PTHreleaseisstimulatedbylowCa2+levels,whereasitsreleaseisinhibited(ina
negativefeedbackmechanism)bytheactive1,25dihydroxyformofvitaminD(Fig.33(f0020)).

Figure33
Parathyroidhormone(PTH)overview.

(FromBrownTA:RapidReviewPhysiology.Philadelphia,Mosby,2007,p128.)

7 Whatarethepotentialpathologicmanifestationsofthehyperparathyroidism
thatdevelopsinrenalfailure?
AlthoughtheelevatedPTHseeninrenalfailureisanappropriateresponsetohypocalcemia,ithasa
negativeimpactonbonemetabolism.ThechronicallyhighPTHlevelsstimulatechronicbone
resorption,whichcanresultinosteoporosisaswellasabnormalcystsinareasofdemineralizedbone
(osteitisfibrosacystica).
Inthechronicmanagementofrenalfailure,inadditiontothemaintenanceofrelativelynormal
calciumandphosphateconcentrations,arelativelynormalPTHlevelisalsoanimportant
therapeuticgoalthatoftenrequiresspecificdrugtherapy.TheagentsusedincludeactivevitaminD
analogs(thathavenegativefeedbackonPTHrelease)aswellasneweragentsthatsuppressPTH
releasebymimickingtheactionofcalciumontheparathyroidgland.
Parathyroidhormone(PTH)andvitaminDareroutinelytestedtopicsontheUSMLE.Itisvery
importanttounderstandthedifferencebetweenprimaryandsecondaryhyperparathyroidism
andtoknowwhichconditionselevatelevelsofPTHandvitaminD.
Step1Secret

8 Whyisthispatientalsopredisposedtoosteomalacia?
OsteomalaciaisadiseaseofadultsresultingfromactivatedvitaminDdeficiencyinwhichthereis
impairedmineralizationofnewlydepositedosteoidmatrixinbone,makingthebonesmore
malleable(malacia).IfthisvitaminDdeficiencyandimpairedmineralizationoccurinprepubertal
children,priortoclosureoftheepiphysealplates,itisreferredtoasrickets.
Renalosteodystrophyincludesthespectrumofbonychangesthatresultfromrenalfailure,and
includestheosteitisfibrosacysticacausedbysecondaryhyperparathyroidism,osteomalaciafrom
impairedvitaminDsynthesisanddecreasedmineralization,andbonelossresultingfromtheneedto
bufferthemetabolicacidosisthataccompaniesrenalfailure.

9 Howcouldthiswoman'srenalfailureexplainthefollowingfindings?
A.Metabolicacidosis
Thekidneysnormallyexcretealargequantityofnonvolatileacids,includingbothinorganicacids
(suchasammoniumandhydrogenions)andorganicacids(suchassulfateandphosphate).Thus,a
mixedaniongapandnonaniongapmetabolicacidosisoccursinrenalfailureastheseacids
accumulateinthebody.
B.Hyperkalemia

Normally,aldosteronedrivesthesecretionofexcesspotassium(inexchangeforsodium)inthedistal
tubule.Chronicrenalfailurepredisposestohyperkalemia,bothbecausetherearefewernephrons
capableofengaginginpotassiumsecretionandbecauseasGFRdecreases,lesspotassiumisfiltered.
C.Anemia.Wouldyouexpectamicrocytic,normocytic,ormacrocyticanemia?
Renalerythropoietinsynthesisislikewisecompromisedinsevererenalfailure,resultinginanemia.
Wewouldexpectanormocyticanemia,becauselackoferythropoietinwillsimplyreducetherateof
erythropoiesis.Insufficienciesofironorfolate/vitaminB12wouldcausemicrocyticormacrocytic
anemias,respectively.

10 Isthiswomansufferingfromazotemiaoruremia?
AzotemiareferstoincreasedBUNandcreatinineinanasymptomaticperson.Ifthiswomanhad
symptomsfromherrenalfailure,shewouldbedescribedassufferingfromuremia(i.e.,shewouldbe
uremic).
Inotherwords,uremiaisnotdefinedbylaboratoryvalues.Instead,itisaclinicalsyndrome
consistingofaconstellationofsymptomsorcomplicationsattributabletorenalfailure.Possible
manifestationsincludenausea,pruritus,malaise,seizures,confusion(uremicencephalopathy),
bleeding(fromuremiainducedplateletdysfunction),pericarditis,andfluidoverload.

11 Ifshewereuremic,andafrictionrubwasdetectedonphysicalexamination,
whatmightyoususpect?
Uremicpericarditis,characterizedbyafibrinousexudatewithinthepericardialspace,wouldbe
likely.Youcanspotpericarditisbythepresenceofafrictionrubonexaminationandchestpainthat
isrelievedbysittingforward(asleaningforwardelevatestheheartfromthediaphragmaticportion
ofthepericardium).

Hypertensionisbothamajorcauseandamajorcomplicationofchronicrenalfailure
(CRF).
CRFischaracterizedbynormocyticanemia(duetolossofrenalerythropoietin
production),hyperkalemia,mixedaniongap/nonaniongapmetabolicacidosis,
hypocalcemia,andhyperphosphatemia.
Azotemiareferstotheaccumulationofrenallyclearednitrogenoustoxinsinan
asymptomaticindividual.
Uremiaisaclinicalsyndromeofspecificsymptomsorcomplicationsattributableto
renalfailuresuchasnausea,pruritus,malaise,seizures,confusion,bleedingpericarditis,
andfluidoverload.
HypocalcemiainCRFisdueto(1)decreasedsynthesisoftheactiveformofvitaminD

(1,25dihydroxyvitaminD)intheproximaltubule,(2)complexingoffreecalciumby
elevatedphosphatelevels,and(3)furtherinhibitionofactivevitaminDsynthesisbythe
elevatedphosphate.
CRFresultsinsecondaryhyperparathyroidism.Elevatedparathyroidhormone(PTH),
inturn,cancauseabnormalboneformation(osteitisfibrosacystica),whereasdeficiency
ofactivatedvitaminDcan(inadults)leadtoosteomalacia.Thefullspectrumofbony
changesresultingfromhighPTH,lowCa2+,andchronicacidosisiscalledrenal
osteodystrophy.
SummaryBox:ChronicRenalFailure

Case35
A32yearoldwomanwithahistoryofhypertensionpresentsforevaluationofrightsidedflank
pain.Shedeniesurinarysymptomssuchasdysuria,urgency,andfrequency.Examinationis
unremarkable,butlaboratoryworkuprevealsanelevatedcreatininelevelandRBCsand
proteininherurine.Arenalultrasoundshowsenlargedkidneyswithnumerouscysts
bilaterally.Thepatientthenrecallsthathermotherhadsomekindofkidneydisease.

1 Whatisthediagnosis?
Shehasautosomaldominant(adult)polycystickidneydisease(ADPKD).Notethediffuse,bilateral
distributionofcystsinADPKDshowninFigure34(f0025).Befamiliarwiththishighyieldimagefor
Step1.

Figure34
Autosomaldominantpolycystickidneydisease(ADPKD)oncutsection.Notediffuse,bilateraldistributionof
cysts.
(CourtesyofF.E.Cuppage,KansasCity,KS.)

2 Whatistheprimarycomplicationofthisdisease?

Endstagerenalfailureistheprimarycomplication.

3 Whymusturinarytractinfectionsbetreatedaggressivelyinpatientswiththis
disease?
PatientswithADPKDaretreatedaggressivelytopreventpyelonephritis,whichcanberemarkably
difficulttotreatinthesepatients.Thisisbecausethecystsareessentiallyurinecesspoolsthatdonot
drain,providinganexcellentbreedinggroundforbacteria.

4 Ifthispatientsuddenlydevelopsasevereheadache,whatvascular
abnormalitymustbesuspected?
Intracranial(berry)aneurysmsareassociatedwithADPKD.Ruptureofaberryaneurysmclassically
resultsinacuteonsetofasevereworstofmylifeheadache.Presumably,themutationsinthe
polycystingenethatcausestissuetoseparateinthekidneysandformcystsalsomakeiteasierfor
vascularconnectivetissuetoseparateandformaneurysms.
Therelationshipbetweenberryaneurysmformationandautosomaldominant(adult)
polycystickidneydisease(ADPKD)isacommonlytestedStep1principle.
Step1Secret

5 Howdoesautosomaldominantpolycystickidneydiseasedifferfrom
autosomalrecessivepolycystickidneydisease(otherthanthepatternof
inheritance)?
Autosomalrecessive(infantile)polycystickidneydiseasetypicallypresentsininfancy,although
therearelessseverechildhoodandadolescentforms.Itisalwaysassociatedwithliverabnormalities,
includinghepaticcystsandcongenitalhepaticfibrosis.Inmanypatients,thecongenitalfibrosisleads
toportalhypertensionandliverdysfunction.
Hepaticcysts(andlesscommonly,cystsinotherorganssuchasthepancreasandlungs)dooccurin
ADPKD,buttheyarenotassociatedwithliverfibrosisororgandysfunction.

6 WhatistuberoussclerosisandhowcanitbedifferentiatedfromADPKD?
Tuberoussclerosisisageneticdiseasethatisalsoinheritedinanautosomaldominantmanner.In
thisdisease,multiplecysts(andtumors)forminthekidneys,butthedisorderisadditionally
characterizedbyavarietyofcentralnervoussystem(CNS)abnormalities,includingmental
retardationandseizures(resultingfromcerebraltuberformation),aswellasavarietyof
characteristicdermatologiclesionssuchasashleafmacules,adenomasebaceum(i.e.,
angiofibromas),shagreenpatches(connectivetissuenevi),andsubungualandperiungualfibromas.
Tuberoussclerosisisassociatedwithavarietyoftumorsincludingastrocytomas,renal
angiomyolipomas,retinalhamartomas,andcardiacrhabdomyomas.

7 WhatisvonHippelLindausyndromeandhowcanitbedifferentiatedfrom
ADPKD?
LikeADPKD,autosomalrecessive(infantile)polycystickidneydisease(ARPKD),andtuberous
sclerosis,vonHippelLindau(VHL)syndromeischaracterizedbymultiplecystsinbothkidneys.Itis
anautosomaldominantdisordercharacterizedbyatendencytoformmultipletypesofneoplasms
andhamartomas.Inadditiontocystsofthekidneysandotherorgans,affectedpatientsdevelop
hemangioblastomasoftheCNS(Lindautumors)andoftheretina,pheochromocytomas,and
pancreatictumors.Therenalcystsareoftencomplicatedbydevelopmentofrenalcellcarcinoma,
frequentlybilateral.
InadditiontotypeIandIIneurofibromatosis,tuberoussclerosisandvonHippelLindau
diseasearehighyieldtopicsforStep1.Beabletoidentifythevarioustumorsassociatedwith
theseconditions.
Step1Secret

8 Whatismedullarycysticdiseaseandhowcanitbedifferentiatedfrom
ADPKD?
Inmedullarycysticdisease,thecystsareconfinedtothemedullathecystsarenotpresent
throughoutthekidneyasinADPKD.Thisrarecysticdiseaseisalsocharacterizedbysevererenal
dysfunction.

9 Quickreview:CoverthefarrightcolumninTable31(t0010)andgivethe
characteristicfeaturesofeachofthecystickidneydiseases
Table31
CysticKidneyDiseases

Disease

Siteof
Cystsin

Modeof
Ageat
Inheritance Onset

KeyAssociatedFeatures

Kidney
ADPKD

Throughout AD

Adulthood Intracranialberryaneurysmandasymptomatic
hepaticcysts

ARPKD

Throughout AR

Infancy

Hepaticcystsandcongenitalhepaticfibrosis
withpossibleportalhypertensionandliver
dysfunction

Tuberous
sclerosis

Throughout AD

Childhood Mentalretardation,seizuredisorder,renal
angiomyolipomas,cardiacrhabdomyomas,
dermatologiclesions

vonHippel Cortex

AD

Teensto

CNSandretinalhemangioblastomas,bilateral

Lindau
disease
Medullary
cystic

young
adulthood
Medulla

renalcellcarcinoma

Childhood

disease
AD,autosomaldominantADPKD,autosomaldominantpolycystickidneydiseaseAR,autosomalrecessive
ARPKD,autosomalrecessivepolycystickidneydiseaseCNS,centralnervoussystem.

10 WhyismedullaryspongekidneynotincludedinTable31?
Truecystsdonotforminmedullaryspongekidney.Rather,segmentsofthecollectingtubules
becomeabnormallydilatedinthemedullaatthetipsoftherenalpapillae.Theprimarycomplication
ofthesedilationsisapredispositiontonephrolithiasisandpyelonephritis.Isolatedhematuriaor
urinarytractinfections(UTIs)canalsooccur.Thisdisorderisseenprimarilyinadultsand,
comparedwithmedullarycysticdisease,isrelativelycommon.

11 Whatisthemostcommoncauseofrenalcysts?
Mostrenalcystsareincidentalnonneoplasticsimplecyststhatarenotassociatedwithaparticular
disease.Suchsimplecystsaremorecommonwithincreasingage,occurringinupto33%ofpeople
olderthan50years.
Cystsareparticularlycommoninpatientsonhemodialysis,increasinginincidence,size,andnumber
withdurationofdialysis.Dialysisassociatedcystsarealsogenerallyasymptomatic,butcanbe
complicatedbyhematuria.

Autosomaldominant(adult)polycystickidneydisease(ADPKD)isthemostcommon
cysticrenaldiseaseandischaracterizedbycolickyabdominalorflankpain,hematuria,
earlyonsethypertension,and,ultimately,endstagerenaldisease.
ADPKDisassociatedwithincreasedriskofpyelonephritis,intracranial(berry)
aneurysms,andasymptomatichepaticcysts.
Thelesscommonautosomalrecessive(infantile)polycystickidneydisease(ARPKD)is
associatedwithcystsintheliver,pancreas,andlungsaswellasliverdysfunctiondueto
congenitalhepaticfibrosis.
Othercystickidneydiseasesincludetuberoussclerosis,vonHippelLindausyndrome,
andmedullarycysticdisease.Truecystsdonotforminmedullaryspongekidney.
Mostrenalcystsareasymptomaticsimplecystsnotassociatedwithanydiseaseor
neoplasm.Suchcystsarecommoninpersonsolderthan50years.

Hemodialysisisassociatedwithasymptomaticcyststhattendtoincreaseinincidence,
size,andnumberwithdurationofdialysis.
SummaryBox:CysticKidneyDisease

Case36
A5yearoldboyisbroughttotheclinicbyhisparents,whoareconcernedbecausehehasbeen
lethargicrecentlyandappearsswollentothem.Markedwholebodyedema(anasarca)is
notedonphysicalexamination.Laboratorytestsrevealhyperlipidemiaandhypoalbuminemia.
Urinalysisrevealsthepresenceofproteinsandlipidsintheurine,butnoRBCs.Urine
microscopyrevealsthestructureshowninFigure35(f0030).

1 Whatisthelikelydiagnosisinthischildandwhy?
Thischildlikelyhasnephroticsyndrome,characterizedbymassiveproteinuria(>3g/24hours),
hypoalbuminemiaresultinginsevereedema,andhyperlipidemia.Duetolipiduriaandsubsequent
cholesterolprecipitation,urinemicroscopymayshowthepresenceoffattycasts(Maltesecrosses),as
showninFigure35(f0030).Themassiveproteinuriacanmakeurineappearfoamyorfrothy.
Althoughslighthematuriaissometimesseeninthenephroticsyndrome,itistypicallytransientand
muchlessseverethanthatassociatedwiththenephriticsyndrome.

Figure35
UrinemicroscopyforpatientinCase36.
(FromHenryJB:ClinicalDiagnosisandManagementbyLaboratoryMethods,20thed.Philadelphia,WB
Saunders,2001,Plate1812.)

Thecauseofthehyperlipidemiaisunclear,butisbelievedtorelatetoincreasedproteinsynthesisby
theliver.Inresponsetothelossofserumproteins,thesynthesisofmanytypesofserumproteinby
theliverisincreased.Infact,elevatedlipoproteinsynthesisisoftenresponsiblefortheonsetof
hyperlipidemia.Althoughthesynthesisofprocoagulant(e.g.,tissuefactor,FactorVIII)and
anticoagulant(e.g.,antithrombinIII)proteinsisalsoincreased,ithappensthatthebalanceisshifted
towardsprocoagulantssuchthatnephroticsyndromeisassociatedwithanincreasedriskofclotting
(particularlyvenousthromboembolism).Ingeneral,however,thromboticcomplicationsofnephrotic
syndromearerareinchildrenandtendtooccurinadultswithotherriskfactorsforclotting.

2 Whatisthelikelycauseofnephroticsyndromeinthispatient?

Minimalchangedisease(alsoknownasnildiseaseorlipoidnephrosis)isthemostcommoncause
ofnephroticsyndromeinchildrenandcommonlyoccursfollowinginfections.

3 Assumingminimalchangediseaseastheunderlyingpathologiccondition,
whatwouldyouexpectgrosshistologicexaminationtorevealifarenalbiopsy
wereperformed?
Thekidneywouldbenormalornearlynormalonlightmicroscopy.Thisisthereasonfortheterm
minimalchangedisease.Thepathologicdiagnosisisgenerallymadeusingelectronmicroscopy,
whichdemonstratesdiffuseflattening(effacement)oftheglomerularfootprocesses,resultingin
increasedpermeabilityoftheglomerularmembraneandsubstantialproteinuria.
Note:Focalsegmentalglomerularsclerosis(FSGS),anotherimportantcauseofnephrotic
syndrome,wouldalsoshowdiffuseeffacementoftheglomerularfootprocessesonelectron
microscopy.However,lightmicroscopywouldrevealareasofsclerosisinsome(butnotall)glomeruli
(i.e.,focalsclerosis).FSGSisunlikelyinayoungchildbutisthemostcommoncauseofprimary
nephroticsyndromeinAfricanAmericanadults.Itisalsoassociatedwithhumanimmunodeficiency
virus(HIV)infection,obesity,andheroinuse(Fig.36(f0035)andTable32(t0015)).

Figure36
Fusionofthepodocytes.Arrowheadsshowfusionofthepodocytes,whichshouldbeseparatedbyslitpores.
Thisfindingoccursinallglomerulardiseasesthatpresentwiththenephroticsyndrome.
(FromGoljanEF:RapidReviewPathology,2nded.Philadelphia,Mosby,2007,p406.)
Table32
SummaryofPrimaryRenalDiseasesthatManifestasIdiopathicNephroticSyndrome

Minimal

Focal

Membranous

Membranoproliferative

Change
Nephrotic

Segmental
Sclerosis

Nephropathy

Glomerulonephritis

Syndrome

TypeI

TypeII

Frequency(tf0015)
Children

75%

10%

<5%

10%

10%

Adults

15%

15%

50%

10%

10%

26,some

210,some

4050

515

515

adults

adults

Nephroticsyndrome

100%

90%

80%

60%

60%

Asymptomatic

10%

20%

40%

40%

Hematuria

1020%

6080%

60%

80%

80%

Hypertension

10%

20%early

Infrequent

35%

35%

Timetoprogression

Doesnot

10years

50%in1020

1020years

515years

torenalfailure

progress

Laboratoryfindings

Manifestations Manifestations Renalvein

None

Partial

ClinicalManifestations
Age(years)

proteinuria

Lightmicroscopy

years

ofnephrotic

ofnephrotic

thrombosis,

syndrome

syndrome

cancer,SLE,
hepatitisB

(tf0010)BUN

(tf0010)BUN

in1530%

in2040%

Manifestations LowC1,C4,
ofnephrotic
C3C9

NormalC1,
C4,lowC3

syndrome

C9

Normal

lipodystrophy

Focalsclerotic Thickened

Thickened

lesions

GBM,

GBM,spikes

Lobulation

proliferation
Immunofluorescence Negative

Electronmicroscopy Footprocess
fusion

IgM,C3in

Finegranular

GranularIgG,

C3only

lesions

IgG,C3

C3

Footprocess

Subepithelial

Mesangial

fusion

deposits

and
deposits
subendothelial

Dense

deposits
Responseto
steroids

90%

1520%

Mayslow

Not

Not

progression

established

established

ModifiedfromGoldmanL,AusielloD:CecilTextbookofMedicine,22nded.Philadelphia,WBSaunders,2004.

,elevatedBUN,bloodureanitrogenC,complementGBM,glomerularbasementmembraneHLA,
humanleukocyteantigenIg,immunoglobulinSLE,systemiclupuserythematosus.
*Approximatefrequencyasacauseofidiopathicnephroticsyndrome.About10%ofcasesofadult
nephroticsyndromeareduetovariousdiseasesthatusuallymanifestwithacuteglomerulonephritis.

4 Whymightthisboybesusceptibletoinfectionswhilesufferingfromthis
illness?
Hypogammaglobulinemiaoccursasaresultoflossofimmunoglobulins(alongwithotherproteins,
suchasalbumin)intotheurine,thuspredisposingthispatienttoinfection.

5 Howshouldthisboybemanaged?Shouldarenalbiopsybeperformed?
Nephroticsyndromecausedbyminimalchangedisease(MCD)typicallyrespondsextremelywellto
steroids(i.e.,prednisone).Becauseamajorityofnephroticsyndromecasesinchildrenarecausedby
MCD(roughly80%),arenalbiopsyisgenerallynotneeded.However,ifthisboydoesnotrespond
welltosteroids,arenalbiopsywouldbenecessarytodeterminetheprecisecauseofthenephrotic
syndrome.
Incontrast,FSGScharacteristicallyrespondspoorlytosteroids.Thus,FSGSshouldbesuspectedin
anypatient(particularlyanyolderpatient)withpresumedMCDthatdoesnotrespondrapidlyto
steroids.

6 Whatareothermajorcausesofnephroticsyndrome?
Therearefivemajorcausesofnephroticsyndrome.Threeareprimaryrenaldiseasestwoare
systemicdiseasescapableofproducingnephroticsyndrome.Althoughtheyallcausenephrotic
syndrome,somehavenephriticcharacteristics.Inotherwords,thereisaspectrumbetween
nephroticandnephriticsyndrome.
TheprimaryrenaldiseasesareMCD(themostcommoncauseofnephroticsyndromeinchildren),
FSGS(themostcommoncauseinAfricanAmericanadults),andmembranousnephropathy(the
mostcommoncauseofnephroticsyndromeinwhiteadults).
FSGS,ofthethree,isthemostnephriticinotherwords,itismostlikelytobeassociatedwithsome
degreeofhypertension,hematuria,orrenaldysfunction(seeCase37(s0260)formoreonnephritic
syndrome).MCD,incontrast,iscompletelynephrotic.
MembranousnephropathyisusuallyidiopathicbutcanbeassociatedwithhepatitisBinfection,
autoimmunedisease(particularlylupus),andmalignancy(particularlycarcinomas),ordrugs.Light
microscopyrevealsbasementmembranethickeningandelectronicmicroscopyshowscharacteristic
densedepositsontheepithelialsideofthebasementmembrane(Fig.37(f0040)).

Figure37
Membranousglomerulopathy.Onultrastructuralexamination,therearenumerous,closelyapposed
epimembranouselectrondensedepositsseparatedbybasementmembranespikes(uranylacetate,leadcitrate
stainoriginalmagnification2500).
(FromGoldmanL,AusielloD:CecilTextbookofMedicine,22nded.Philadelphia,WBSaunders,2004.)

Thesystemicdiseasesarediabetesandamyloidosis.Althoughdiabetesisamajorcauseofnephrotic
syndrome(becausediabetesissocommon),nephroticsyndromeisratheruncommonamong
diabetics.Amyloidosisistheleastcommonofthefive.
AsmentionedintheInsider'sGuidetoNephrologyfortheUSMLEStep1,glomerulonephritic
syndromesareafavoritetopicforthetestmakers.Inadditiontodifferentiatingbetween
nephriticandnephroticsyndromeandknowingwhichsyndromesareassociatedwithwhich
diseases(e.g.,hepatitisBandCassociatedwithmembranoproliferativeglomerulonephritis
typeI),youshouldbeabletoidentifyeachtypeofglomerulonephritisbylightmicroscopy,
electronmicroscopy,andimmunofluorescence(ifapplicable).
Step1Secret

SomeDifferentialDiagnosisConcepts
7 Whatconditionmightyoususpectina6yearoldgirlwhopresentswith
abdominalpain,jointpain,hematuria(ormelena)andproteinuria,anda
palpablepurpuraonherbuttocksandlowerextremities?
HenochSchnleinpurpura(HSP)isasmallvesselvasculitisofchildrenthatcommonlyfollowsan
upperrespiratorytractinfection.Presumably,themucosalimmunestimulationcausedbytheupper
respiratorytractinfectionstimulatestheproductionofIgAthus,thisdiseaseischaracterizedbyIgA
depositioninsmallvesselsofthegastrointestinaltract,glomeruli,joints,andskinInadditionto
hematuriaandproteinuria,therenalinvolvementcanprogresstohypertensionand,uncommonly,
acuterenalfailure.Likewise,thecharacteristicallycolickyabdominalpaincanbeaccompaniedby
significantGIbleedingand,rarely,evenintussusception.

Note:Palpablepurpuraalwayssuggestsavasculiticprocessbecauseinflammationofthesmall
vesselsoftheskinallowsRBCstoextravasate(i.e.,leak)intothedermisandformthepalpable
lesion.InHSP,therashtendstobepresentonthebuttocksandlowerextremitiesinawaistdown
distribution.

Nephroticsyndromeischaracterizedbymassiveproteinuria(>3g/24hours),
hypoalbuminemia,edema(oftenanasarca),hyperlipidemia,andlipiduria,and(inadults)
increasedriskofvenousthromboembolism.
Thenephriticsyndromeischaracterizedbymodestproteinuria(<3g/24hours),
hematuria,renalinsufficiency,hypertension,andurinarydysmorphicredbloodcells
(RBCs)and/orRBCcastsonurinemicroscopy.
Themostcommoncauseofnephroticsyndromeinchildrenisminimalchangedisease,
whichisnamedforitsnormalappearanceonlightmicroscopy(LM).Itusuallyresponds
rapidlytosteroids.
Focalsegmentalglomerularsclerosis(FSGS)isassociatedwithhuman
immunodeficiencyvirus(HIV)andheroinuse,isfrequentlyaccompaniedbynephritic
complications,andtendstorespondpoorlytotherapy.
Membranousnephropathyisthemostcommoncauseofnephroticsyndromeinwhite
adultsitisassociatedwithhepatitisB,lupus,andvariouscarcinomas.
HenochSchnleinpurpuraisasmallvesselvasculitisofchildrencausedbyIgAimmune
complexesandaffectstheglomeruli,theskin(withwaistdownpalpablepurpura),
joints(resultinginarthritis),andgastrointestinal(GI)tract(withabdominalpainorGI
bleeding).
SummaryBox:GlomerularDisease

Case37
A42yearoldmancomesinforapreemploymentphysicalexamination.Hehasnocurrent
complaintsbutmentionedthathehasrecoveredonly1to2weeksagofromabadsorethroat.
Physicalexaminationissignificantformildhypertension.BloodworkrevealselevatedBUN
andcreatinine,andurinalysisissignificantforhematuriaandmicroscopicproteinuria.
AntistreptolysinO(ASO)andantiDNaseBtitersaremarkedlyelevated.Laboratorytestsfrom
apreviousvisit3monthsagorevealednormalBUNandcreatininelevels.

1 Whatisthelikelydiagnosis?

Thismanhasacutenephriticsyndrome(i.e.,acuteglomerulonephritis)characterizedbyarelatively
suddenonsetofmildtomoderateproteinuria(i.e.,<3g/24hours),hematuria,renalinsufficiency
(withelevatedcreatinine),andhypertension.
UrinemicroscopywillalsooftenshowdysmorphicRBCsandRBCcastsinglomerulonephritis(Fig.
38(f0045)).

Figure38
Erythrocytecast(200).
(FromMcPhersonRA,PincusMR:Henry'sClinicalDiagnosisandManagementbyLaboratoryMethods,21sted.
Philadelphia,WBSaunders,2006.)

2 Whatsegmentofthenephronisinvolvedinacutenephritis?Howdoesthis
comparewiththenephroticsyndrome?
Theglomerulusisdamagedinglomerulonephritis.ThepresenceofdysmorphicRBCs(whichare
deformedbypassagethroughthedamagedglomeruli)orRBCcasts(whichformwithintherenal
tubules)indicatethathematuriaisofaglomerularorigin.
Theglomerulusisalsotheprimarysiteofinjuryinnephroticsyndrome,butingeneral,different
sidesoftheglomeruliaredamaged.Innephroticsyndromethefootprocessesandglomerular
basementmembranearemostdamaged(exemplifiedbythefootprocesseffacementseeninMCDor
FSGS),whereastheendotheliumandbasementmembranearetypicallymoreinvolvedin
glomerulonephritis.ItistheendothelialdamagethatallowsfortheleakageofRBCsthroughthe
glomerulusthatistypicalofglomerulonephritis.
Again,realizethatnephroticandnephriticsyndromesaretwoendsonthespectrumofglomerular
disease.Certaindiseasesareverycharacteristicallynephrotic(i.e.,MCD),othersareverynephritic
(i.e.,rapidlyprogressiveorcrescenticglomerulonephritis),andmostothersfallsomewherein
between.

3 Whatwasthelikelycauseofthisman'ssorethroatandwhatisitsrelationship
totherenaldysfunction?
Thepatientlikelyhadstreptococcalpharyngitis,causedbygroupAhemolyticstreptococcus(i.e.,
Streptococcuspyogenes),andisnowsufferingfrompoststreptococcalglomerulonephritis(PSGN).
Thisconditioniscausedbyantibodyantigencomplexdepositionintheglomerulus,resultingin

inflammatorydestructionoftheglomerulus(primarilymediatedviacomplementactivation).For
boards,lookforsubepithelialhumplikedepositsalongthecapillarybasementmembranes,as
depictedbythestraightarrowinFigure39(f0050).

Figure39
Electronmicrographofaportionofaglomerularcapillaryfromapatientwithacutepoststreptococcal
glomerulonephritis(PSGN)showingsubepithelialdensedeposits(straightarrow),condensationofcytoskeleton
inadjacentepithelialcytoplasm(smallcurvedarrow),andaneutrophil(N)marginatedagainstthebasement
membranewithnointerveningendothelialcytoplasm(magnification5000).
(FromBrennerBM:BrennerandRector'sTheKidney,7thed.Philadelphia,WBSaunders,2004.)

Insomepatients,inwhomthehistoryofapriorinfectionmaybedifficulttoelicit,antibodyevidence
ofrecentstreptococcalinfectioncanhelpmakethediagnosis.Thetwomostcommonantibodies
testedforarethosedirectedagainstthestreptococcalantigensstreptolysinO(diagnosticbutnot
prognostic)andDNaseB.Theimmunecomplexformationalsoresultsinconsumptionof
complement,whichisreflectedinadecreaseinserumcomplementlevels.

4 Ifthepatienthadrecentlysufferedfromcellulitisratherthanpharyngitis,
woulditalterthediagnosisofhisrenaldisorder?
No.StreptococcalinfectionsoftheskinandsofttissuecanalsocauseaPSGN.Note:Thiscontrasts
withacuterheumaticfever,whichisapostinfectiouscomplicationthatexclusivelyfollows
streptococcalpharyngitis.

5 Whatisthetreatmentforpoststreptococcalglomerulonephritis?
Thetreatmentissupportive.Hypertensionmustbecontrolledwithmedicationsandsaltrestriction.
Ingeneral,steroidsorimmunosuppressiveagents(usedinmanyotherglomerulardiseases)arenot
used.Prognosisisgenerallymuchbetterinchildrenthaninadults.
Note:Againincontrastwithrheumaticfever,theincidenceofPSGNisnotdecreasedbyantibiotic
administration.

6 Howwouldthediagnosischangeifarenalbiopsyrevealedglomerular
crescents?
Glomerularcrescentsarethehallmarkofcrescenticorrapidlyprogressiveglomerulonephritis
(RPGN),which,asthenamesuggests,isaformofrapidlyevolvingglomerulardiseasethatfrequently
respondspoorlytotreatmentandprogressestorenalfailure.
Crescentsareformedbyclustersofrapidlyproliferatingparietalepithelialcellsandinfiltrating
leukocytes(Figs.310(f0055)and311(f0060)).

Figure310
Normalglomerulus.
(FromDamjanovI:PathologyfortheHealthRelatedProfessions,2nded.Philadelphia,WBSaunders,2000.)

Figure311
Crescenticglomerulonephritis.ArrowspointtoaproliferationofparietalepithelialcellsinBowman'scapsule,
occupyingapproximately50%oftheentireurinaryspace.Thecellsencaseandcompresstheglomerulartuft.
(FromKumarV,AbbasAK,Fausto,N:RobbinsandCotranPathologicBasisofDisease,7thed.Philadelphia,
WBSaunders2005,p977,Fig.2017.)

ThemajortypesofRPGNaredistinguishedusingimmunofluorescence.Pauciimmune
glomerulonephritisisthemostcommoncauseofRPGN.Itsnamederivesfromthefactthatonrenal
biopsy,immunecomplexesarenotseenonimmunofluorescence.Pauciimmuneglomerulonephritis
iscommonlyassociatedwithvasculiticconditionssuchasWegener'sgranulomatosisandthesimilar
disordermicroscopicpolyangiitis.Theseconditionsareinturncommonlyassociatedwithpositive
antineutrophilcytoplasmicantibodies(ANCA)Wegener'sgranulomatosisisusuallycANCApositive
(withadiffusecytoplasmicstainingpattern),whereasmicroscopicpolyangiitistendstobepANCA
positive(withaperinuclearcytoplasmicpattern).Wegener'sgranulomatosisusuallypresentswitha
triadofglomerulonephritisandinvolvementofboththelowerandupperrespiratorytracts.The
respiratorydiseaseisusuallyintheformoforalulcersorpurulentorbloodynasaldischargealong
withpulmonarynodules,infiltrates,orcavities.

GoodpasturesyndromeisanotherformofRPGNcausedbyantibodiesdirectedagainstthe
glomerularbasementmembrane(GBM).TheseantiGBMantibodiesformimmunecomplexesthat
canbeseenonimmunofluorescenceaslineardepositsofIgGandC3.TheseantiGBMantibodies
canalsoreactwiththealveolarbasementmembraneofthelungstocausepulmonaryhemorrhagein
additiontoglomerulonephritis.
RPGNcanalsorepresenttheendstageofmanyotherformsofimmunecomplexmediated
glomerulonephritis.Insuchacase,immunofluorescencetypicallyrevealslumpybumpygranular
(i.e.,nonlinear)depositionalongtheglomeruli.
Wegener'sgranulomatosispresentswithupperrespiratorysymptomsbutGoodpasture
syndromedoesnot.Goodpasturesyndromeinvolveslineardepositsalongtheglomerular
basementmembrane(GBM)butWegener'sgranulomatosisdoesnot.Youwillbegivenatleast
oneofthesetwopiecesofinformationonyourexaminationifthequestionrequiresyouto
distinguishthetwodiagnosesfromoneanother.
Step1Secret

7 Whatisthemostcommoncauseofnephritisworldwide?
IgAnephropathy(i.e.,Berger'sdisease)isoverallthemostcommonprimaryglomerulardisease
(nephriticornephrotic).
ThisdisorderisinseveralwayssimilartoHSPbutwithoutanyextrarenalmanifestations.Itis
commonlyseenafewdaysafteraviralupperrespiratoryorGIillness.Renalbiopsytypicallyshows
immunecomplexdeposition(composedofprimarilyIgA)withinthemesangium(thesame
mesangioproliferativepictureseenonbiopsyinHSP).
Thepatientsoftenpresentwithchronicasymptomaticmicroscopichematuriaandtraceproteinuria,
withexacerbationscharacterizedbygrosshematuriaoccurringafterviralinfections.IgA
nephropathyisagoodexampleofdisorderthatcanpresentwithfeaturesofbothnephroticand
nephriticsyndromes.

8 Whatsyndromedoyoususpectina13yearoldboywithmicroscopic
hematuriaandhearingloss?
Alportsyndrome(i.e.,hereditarynephritis),althoughmuchlesscommon,isclinicallysimilartoIgA
nephropathyinthatitoftenpresentswithasymptomaticmicroscopichematuria.Itiscausedby
variousmutationsintypeIVcollagennormallyfoundintheglomerularbasementmembraneand
elsewhere.ItisusuallyinheritedinanXlinkedmanner(canbebothdominantandrecessive).The
majorcluetoitsdiagnosisistheassociationwithabnormalhearingandvision(i.e.,sensorineural
hearinglossandlensabnormalities).ThepresenceofcollagenIVintheskinallowsforthediagnosis
tobeusuallymadebyskinbiopsy.Renalbiopsy,ifperformed,typicallyshowsfragmentationofthe
basementmembraneinabasketweavepattern.

Asanaside,thereareonlyafewmedicationsthatcancausebothototoxicityandnephrotoxicity.
Thesedrugsincludeloopdiuretics(suchasfurosemide),vancomycin,cisplatin,and
aminoglycosides.

DifferentialDiagnosis
9 Quickreview:CovertherightcolumninTable33(t0020)andattemptto
diagnosethecauseoftheglomerulonephritisbasedonthelaboratoryfindings
andhistoryprovidedintheleftcolumn
Poststreptococcalglomerulonephritisisanacutenephriticsyndromeoccurring1to2
weeksafterstreptococcalsorethroatorskin/softtissueinfectionantistreptolysinO
(ASO)orantiDNaseBtiterscanbeusedtodocumentrecentinfection.Ittypically
resolveswithoutspecifictherapy.
Crescenticorrapidlyprogressiveglomerulonephritisisaformofglomerulonephritis
withrapidonsetandpoorprognosis.Thethreetypesaredistinguishedby
immunofluorescence:(1)pauciimmune(i.e.,antineutrophilcytoplasmicantibody
associated)disease(Wegener'sgranulomatosisormicroscopicpolyangiitis),(2)anti
glomerularbasementmembranesyndromes(e.g.,Goodpasturesyndrome),and(3)end
stageimmunecomplexmediateddiseases.
BothIgAnephropathyandAlportsyndromefrequentlypresentwithasymptomatic
microscopichematuria.IgAnephropathyisthemostcommonprimaryglomerular
diseaseworldwide.AlportsyndromeiscausedbyinheriteddefectsincollagenIVandis
oftenaccompaniedbyhearingloss.
SummaryBox:PoststreptococcalGlomerulonephritis

Case38
A45yearoldmanisevaluatedintheemergencyroom(ER)foraseveralhourhistoryofsevere
leftflankpainthatradiatesintohisgroin.Hereportssomemildnauseawiththepain,but
deniesvomiting,diarrhea,orabdominalpain.Heiswrithinginpainbutabdominalexamis
largelyunrevealing.UrinemicroscopyrevealsgrosshematuriawithoutRBCcasts,dysmorphic
RBCs,orWBCcasts.RoutinelaboratorytestsdoneintheERalsoshowhypercalcemia.

Table33
DifferentialDiagnosisforGlomerulonephritis

FindingsonLaboratoryTestsandHistory

LikelyCauseofGlomerulonephritis

AntiGBMantibodies,hematuria,andhemoptysis

Goodpasturesyndrome(i.e.,anti

GBMdisease)
cANCApositivewithahistoryofbloodynasaldischargeand Wegener'sgranulomatosis
hemoptysis
DefectintypeIVcollagenwithcongenitalhearingandocular

Alportsyndrome

impairment
MassiveproteinuriaandedemainanadultwithhepatitisB

Membranousnephropathy

Hematuria,episodicabdominalpain,jointpain,andalower

HenochSchnleinpurpura

extremitypurpuricrash
cANCA,cytoplasmicantineutrophilcytoplasmicantibodiesGBM,glomerularbasementmembrane.

1 Whatisthediagnosis?
Thesevereflankpain(i.e.,renalcolic)andurologic(i.e.,nonglomerular)hematuriasuggestthe
diagnosisnephrolithiasis,whichisconfirmedbycomputedtomography(CT)scan.Thisstoneismost
likelysecondarytochronichypercalcemia.

2 Whatarethemostcommoncausesofnephrolithiasis?
Thevastmajorityofkidneystonesarecalciumstones(calciumoxalateorcalciumphosphate)
struvitestones(magnesiumammoniumphosphate)arenextmostcommon,followedbyuricacid
stonesandcystinestones.Themostcommoncausesincludedehydration,hypercalcemia(calcium
oxalatestones),infection(struvitestones),hyperuricosuria(uricacidstones),andimpaired
absorptionofbasicaminoacids(cystinestones)(Figs.312to314(f0065)).

Figure312
Coffinlidcrystalsofmagnesiumammoniumphosphate(struvite).
(FromJohnsonRJ,FeehallyJ:ComprehensiveClinicalNephrology.London,Mosby,2000.)

Figure313
Uricacidcrystals(160).
(FromMcPhersonRA,PincusMR:Henry'sClinicalDiagnosisandManagementbyLaboratoryMethods,21sted.
Philadelphia,WBSaunders,2006.)

Figure314
Hexagonalcystinecrystals,whichindicatecystinuria(400).
(FromPiccoliG,VareseD,RotunnoM:AtlasofUrinarySediments:DiagnosisandClinicalCorrelationsin
Nephrology.NewYork,Raven,1984.)

3 Howcannephrolithiasiscauserenalfailure?
Ifaureteroruretericpelvisiscompletelyobstructed,thisincreasesthehydrostaticpressureinthe
uretersandultimatelyintherenaltubules.Theincreasedhydrostaticpressuresubstantially
diminishesthenetfiltrationpressureattheglomerulus,causinganacutedeclineintheGFR.
However,if,asisusuallythecase,theotherkidneyisintactandhealthy,itcancompensateforthe
decreasedGFRintheobstructedkidney.
Recallthatobstructionofurineflowoutofakidneycauseshydronephrosis(pelvicalicectasis),in
whichtherenalpelvisandcalycesdilatesignificantly.Hydronephrosisisusuallydetectedonrenal
ultrasoundorCTscan.

4 Whatarethemostcommoncausesofhypercalcemiaamongbothoutpatients
andinpatients?
Primaryhyperparathyroidismisthepredominantcauseinthecommunity,whereasmalignancy
inducedhypercalcemiapredominatesinhospitalizedpatients.Together,thesetwoconditions
accountforover90%ofhypercalcemiacases.
Recallthatprimaryhyperparathyroidismisusuallycausedbyeitheraparathyroidadenoma(inthe
majorityofcases),hyperplasia(inabout10%ofcases),or(rarely)bycarcinoma.Apopular
mnemonicforthecomplicationsofhyperparathyroidismisstones,bones,moans,andgroans,
referringtotheincreasedriskforrenalcalculi,pepticulcers(presumablyfromincreasedgastricacid
secretion),andpathologicfracturesaswellasthepossiblesymptomsofvaguebonepains,muscle
achesorweakness,malaise,fatigue,anddepression.Anyformofhypercalcemia(analogousto
hyperglycemia)cancausepolydipsiaandpolyuriafromosmoticdiuresis,whileconstipationand
hypertensioncanoccurfromincreasedsmoothmuscletone.
Hypercalcemiaisthemostcommonmetabolicderangementthatcomplicatesmalignancy,occurring
in10%to20%ofcancerpatients.Malignanciessuchasbreastcancerandmultiplemyelomacan
producehypercalcemiabymetastasizingtoboneandcausinglyticdestructionothers,suchas
squamouscellcancerofthelung(andotherorgans)andrenalcellcarcinoma,secreteanectopic
hormoneknownasparathyroidhormonerelatedpeptide(PTHrP).
Note:Granulomatousdiseases,suchassarcoid,andasubsetofmalignancies,particularly
lymphomas,canproducehypercalcemiaviaexcessformationandactivationofendogenousvitamin
D(i.e.,calcitriol)byactivatedmacrophages,presumablythesamemacrophagesfoundwithinthe

granulomasorlymphomatouslesionsthathistologicallycharacterizethesediseases.Othercausesof
hypercalcemiaincludehypervitaminosisD(vitaminDintoxication)andthemilkalkalisyndrome,in
whichexcesscalciumandabsorbablealkali(suchasthecalciumcarbonateofTums)areingested,
resultinginalkalosis,hypercalcemia,andrenalinsufficiency.
AcommonlyusedmnemonicforcausesofhypercalcemiaisCHIMPANZEES:excessCalcium
ingestion,HyperparathyroidismorHyperthyroidism,Iatrogenic(i.e.,druginduced,asfrom
thiazideadministration)orImmobilization,Myeloma,Paget'sdiseaseofbone,Addison'sdisease,
Neoplasm,ZollingerEllisonsyndrome(typicallyinassociationwithmultipleendocrineneoplasia
typeIsyndrome),ExcessvitaminD,ExcessvitaminA,Sarcoidandothergranulomatousdiseases.
Notethatmanyoftheseconditions,althoughpotentialcausesofhypercalcemia,aremorecommonly
associatedwithnormal,ratherthanelevated,serumcalcium.

5 Whyishypercalcemiasecondarytohyperparathyroidismlesslikelytocause
renalcalculiformationthanothercausesofhypercalcemia?
Generally,theriskofcalciumoxalatestoneformationintheurineisproportionaltotheurine
calciumconcentration(ratherthanserumcalciumconcentration).Althoughprimary
hyperparathyroidismincreasesurinecalciumconcentrationasinmostothercausesof
hypercalcemia,itdoessotoalesserextent,becausePTH,inadditiontopromotingGIuptakeof
calciumandcalciumreleasefrombone,promoteshypercalcemiabystimulatingrenaltubular
calciumreabsorption.

RelatedQuestions
6 HowdoesurinarytractinfectionwithbacteriasuchasProteusmirabilis
predisposetostruvitestoneformation?
Struvitestones,whichconsistofmagnesiumammoniumphosphate,precipitateathigherurinary
pH.Proteusspeciesandotherurinarypathogenselaboratetheenzymeureaseintotheurinarytract,
whichcleavesureatoformammoniaandcarbondioxide.Theammoniareleasedbothincreasesthe
urinarypHand,withinbasicurine,formstheammoniumthatcanprecipitateaspartofthestruvite
stones.Struvitestones,likecalciumoxalatestones,areradiopaque.
Note:Struvitestonesareoftenlarge,attimesoccupyingalltherenalcalycestoformcharacteristic
staghorncalculi.

7 HowdoesurinarypHinfluencetheprecipitationofuricacidstones?
Uricacid,asaweakacid,islesssolubleinits(neutral,protonated)uricacidformthaninthe
(unprotonated,negativelycharged)urateform.Thus,decreasingtheurinarypHincreasesthe
concentrationofthelesssolubleuricacidform,therebyfacilitatingcrystallization.
BecausethediureticacetazolamideincreasesurinarypH(asinhibitionofcarbonicanhydrase
preventsbicarbonatereabsorptionandresultsinincreasedurinarybicarbonateconcentration),itcan
alsobeusedtohelpdissolveuricacidstones.

Uricacidstonesareassociated,likegout,withhyperuricemiaand,mostspecifically,with
hyperuricosuria.Thus,inadditiontoincreasedfluidintake(asusedwithallstonetypes)andurine
alkalinization,allopurinolcanbeusedtopreventrecurrenceofuricacidstonesbydecreasinguric
acidsynthesis(viainhibitionoftheenzymexanthineoxidase).

8 Whatiscystinuriaandhowdoesitleadtocystinestones?
Thisisageneticdiseaseinwhichrenaltubularreabsorptionofcystine(andseveralotheramino
acids)isimpaired.Specifically,thereisadefectinthetransporterresponsiblefortheabsorptionof
thebasicaminoacidscystine,ornithine,lysine,andarginine.Becausecystineisnotassolubleas
theseotheraminoacids,itprecipitatesselectivelyinthisdisease.
Cystinuriaisdistinctfromthediseasehomocystinuria.Inthelattercaseageneticdefectin
homocysteinecatabolismcausesextremeelevationsinplasmaandurinehomocysteine
concentrations.
Alsonotethatcystineanduricacidstonesarebothorganicstones,whicharetypicallyradiolucent,
andsotheycannotbevisualizedusingxraysbutcanbeseenwithCTscan.Thiscontrastswithmost
calciumcontainingstones,whichareradiopaqueandcanbevisualizedusingxrays.

9 HowdoesCrohn'sdiseaseleadtoanincreasedriskofkidneystones?
Crohn'sdiseaseveryfrequentlyinvolvestheterminalileum,whichistheprimarysiteofbilesalt
reabsorption.Thus,Crohn'sileitiscancausebilesaltmalabsorption.Asbilesaltsnormally
extensivelybindcalcium,theincreasedlevelsofbilesaltintheintestinallumendecreasethelevelsof
freecalciuminthegut.Lessfreecalciumisthereforeavailabletobindoxalate,whichinturn,isthen
leftunboundandfreetobeabsorbedfromthegut.Asaresult,patientswithCrohn'sdiseasetendto
hyperabsorboxalate.Thisoxalatemusteventuallybeexcretedinurine,andthehighurinaryoxalate
concentrationpredisposestocalciumoxalatestoneformation.
Interestingly,bydisruptingthesolubilityofbile,thebilesaltmalabsorptionthatoccursinCrohn's
diseasealsoincreasestheriskofgallstoneformation.

Nephrolithiasispresentswithrenalcolicandurologic(i.e.,nonglomerular)hematuria,
isusuallydiagnosedbycomputedtomography(CT),andcancauseobstructionand(if
bilateral)renalfailure.
Kidneystonesarecomposedof(inorderofdecreasingincidence)calcium,struvite,uric
acid,orcystine.
Themostcommoncauseofhypercalcemiainhospitalizedpatientsismalignancy,
whereasprimaryhyperparathyroidism(causedbyparathyroidadenomaorhyperplasia)
ismostcommoninoutpatients.
Mechanismsofhypercalcemiaincludedirectlysisofbone(asinmyelomaorbreast

cancer),ectopicproductionofparathyroidhormone(PTH)relatedpeptide(asin
squamouscelllungcancer),andexcessproductionofactivatedvitaminDby
macrophages(bylymphomasorgranulomatousdiseases).
Other,lesscommoncausesofhypercalcemiaincludeexogenouscalciumingestion,milk
alkalisyndrome,hypervitaminosisDorA,prolongedimmobilization,drugeffect(e.g.,
thiazides),Pagetdisease,and(uncommonly)endocrinedisorders(i.e.,Addison'sdisease
orhyperthyroidism).
Primaryhyperparathyroidismiscausedbyadenomas(mostcommonly),byhyperplasia
(in~10%),and(rarely)bycarcinoma.Possiblesymptomsandcomplicationsinclude
kidneystones,pathologicfractures,pepticulcers,vaguemuscleandbonepains,
polydipsia,constipation,andhypertension.
Struvitestonesconsistofmagnesiumammoniumphosphateandprecipitateatahigh
urinarypH.Asaresult,theyoftencomplicateinfectionscausedbyureaseproducing(i.e.,
ureacleavingandammoniareleasing)organismslikeProteus.Staghorncalculiare
usuallycomposedofstruvite.
UricacidstonesprecipitateatlowerurinepHand,asaresult,thecarbonicanhydrase
inhibitoracetazolamidecanbeusedtodissolvethem.
Cystinuriaisadefectintubularbasicaminoacidreabsorptionthatresultsinrecurrent
cystinestones.
Crohn'sdiseaseleadstobileacidmalabsorption,whichinturnpredisposestoboth
gallstonesandkidney(calciumoxalate)stones.
SummaryBox:Nephrolithiasis

Case39
A38yearoldwomancomplainsofsuddenonseturinarysymptoms,includingburning
urination,frequenturination,urgency,andafeelingofincompletebladderemptying,aswellas
nausea,vomiting,andrightsidedbackpain.Onexaminationshehasafeverof102.5and
rightsidedcostovertebralangletenderness.UrinalysisrevealsnumerousWBCs,bacteria,and
WBCcasts.Sheisadmittedtothehospitalandprescribedciprofloxacin.Sheisalsotoldto
drinkplentyoffluids.

1 Whatisthemostlikelydiagnosis?
Shemostlikelyhasacutepyelonephritis.

Althoughtheycanbeseenwithothercausesofrenalparenchymalinflammation(suchasacute
interstitialnephritis),WBCcastsinthesettingofsuchurinarysymptomsareessentially
pathognomonicforacutepyelonephritis.

2 Whatisthemostcommonsourceofinfectioninpyelonephritis?
Inthevastmajorityofcases,theseinfectionsaretheresultofascendinginfectionfromthebladder
andurinarytract,whereasinaminority,theinfectionisfromhematogenousdissemination.The
mostcommoninfectiousagentsarefecalflorathathavecolonizedthevaginalintroitus.Escherichia
coli,inparticular,isthepredominantpathogen.Insexuallyactiveyoungwomen,infectionwith
Staphylococcussaprophyticus(atypeofcoagulasenegativestaphylococcusdistinctfrom
Staphylococcusepidermidis)issecondmostcommon.Amongtherelativelyuncommoncasesof
hematogenousseedingoftherenalparenchyma,coagulasepositiveS.aureusisthemostcommon
pathogen.
Note:UnlikethegramnegativeorganismsthatarecommoncausativeagentsofUTI,S.
saprophyticustestsesterasepositiveandnitritenegative(gramnegativeorganismsarepositivefor
bothmarkers).

3 Whyarepregnantwomenwithasymptomaticbacteriuriatreatedmore
aggressivelythannonpregnantwomenwiththesamecondition?
Althoughasymptomaticbacteriuria(bacteriaintheurinewithoutsymptomsorotherevidenceof
infection)rarelycausesproblemsinnonpregnantwomen,asignificantpercentageofpregnant
womenwiththisconditionwillgoontodeveloppyelonephritis.Asidefromtheobviousdangersof
developingbacteremiaandsepsis,pyelonephritisalsoincreasestheriskofprematuredelivery.

4 Whyarepregnantwomenwithbacteriuriamoresusceptibletopyelonephritis?
Pregnancyresultsinrelaxationofthebasaltoneoftheureteralsmoothmuscle.Thisureteraldilation
orphysiologichydronephrosisincreasesurinepoolingand,inturn,theriskofascendinginfection.
Thisureteralrelaxationisbelievedtobearesultofincreasedprogesteronelevelsofpregnancy,
similartotherelaxationofGIsmoothmusclethatresultsintheconstipationthatisalso
characteristicofpregnancy.

5 Whatisacuteinterstitialnephritisandhowdoesitdifferfromacute
pyelonephritis?
AIN,likepyelonephritis,ischaracterizedbyinflammationoftherenalinterstitium.However,rather
thaninfection,itiscausedbyanallergicreaction,mostcommonlyadrughypersensitivity.Although
infectiousagents(lesscommonlythandrugs)canalsoprecipitateAIN,theinfectiousprocessitselfis
minimallyinvolvedinthepathogenesis,whereastheimmunologichypersensitivitytotheinfectious
agentisparamount.AINcanalsobeamanifestationofautoimmunedisorderssuchassarcoid(albeit
rarely).
SeeCase33(s0080),question1,forareviewofAINasacauseofintrinsicacuterenalfailure.

6 Whyareurinarytractinfectionsinmenyoungerthan50oftenevaluated
aggressively?
Theseinfectionsareusuallyduetourologicabnormalities,becauseordinarilyyoungermenarequite
resistanttourinarytractinfections(withtheexceptionofsexuallytransmitteddiseases[STDs]).
WomenarepresumablymorepredisposedtoUTIsbecauseoftheshorterlengthofthefemale
urethraandtheproximityofthevaginaandperineumtotheurethralmeatus.

7 Whyaremenolderthan50predisposedtourinarytractinfection?
Thebenignprostatichyperplasia(BPH)thatcommonlydevelopsinthisgroupcausesgreaterurinary
retentionandstasisofbladderurine,whichfacilitatesbacterialovergrowth.

Acutepyelonephritiscanresultinurinarytractinfection(UTI)symptoms,fever,and
nauseaandvomiting.Costovertebralangletendernessonexaminationandwhiteblood
cell(WBC),bacteria,andWBCcastsintheurinearehighlysuggestive.
Pyelonephritisusuallyresultsfromascendingurinaryinfectioncausedbyfecalflora,
mostcommonlyEscherichiacoli.Inyoungwomen,Staphylococcussaprophyticusis
secondonlytoE.coli.Hematogenousinfectionofthekidneysismostcommonlycaused
byStaphylococcusaureus.
Pregnancypredisposestoascendingurinaryinfectionthus,asymptomaticbacteriuriain
pregnancyistreatedtopreventpyelonephritis(andthesubsequentriskofsepsisand
pretermdelivery).
Menolderthan50yearsarepredisposedtoUTIfrombenignprostatichyperplasia
relatedurineretentionandstasis.Likewise,adultmenunder50withaUTIshouldbe
evaluatedforsexuallytransmitteddiseases(STDs)orurinaryanomalies.
SummaryBox:AcutePyelonephritis

Copyright2015Elsevier,Inc.Allrightsreserved.

You might also like